Sunteți pe pagina 1din 1128

CALCULUS

Trigonometric Identities

Fundamental Identities Even-Odd Identities Addition Formulas

= •^n^^os a
CSC
1
^ sin(-0) = -sin 6
fsin(a + P) sin a cos /3(

(?)
sin
^ cos(-e) = cos e cos(a + P) = cos a cos /3 — sin a sin /3

= -tan a +
@ sec e
1

cos 6
11 tan(-0) 6 tan
-
tan /3

12 cot(-0) = -cot 1

cot = J 13 sec(-0) = sece


(3)
tan Subtraction Formulas
14 csc(-0) = -esc e
sin e - = a - a
^ tan e
cos
22

23
sin(a

-
j8)

=
sin

a
cos /3

+
sin /3

a
cos

Double- Angle Formulas'


cos(a /3) cos cos )8 sin sin ^
cos
®cot 20= tan a — tan 6
sin e
15 sin 2 sin 6 cos
24 tan(a - /3)
=
16 cos 26=2 cos- - 1

^ cos- e + sin- e = 1 = 1-2 sin^


= cos- 6 - sin^
7 1 + tan- = sec- Product Formulas
17 cos- = ^(1 + cos 20)
8 1 + cot- Q = csc^ e 25 sin a cos ^ = 5[sin (a + /3) + sin (a - /3)1

18 sin^ = 5(1 - cos 20)


26 cos a cos /3 = |[cos (a + ^) + cos (a - /3))

27 sin a sin /3 = 5[cos (a - /3) - cos (a + ^)]

Geometry (see Appendix A, page 1015)

Calculus

Differentiation

1 D,u" = nu"~^ D^ 11 Dv sinh M = cosh u Dji


Vl -M^
2 D^u + v) = D^ + D^v 12 D;^ cosh u = sinh m D^m
22 D, cos"' M
3 D,{uv) = mO,v + vD^M 13 D( tanh « = sech- u D^m

uD^v 14 Dt coth M = -csch- u D^ 23 D, tan"' m


= — sech h tanh u D^u
1 + M^
15 D, sech M
5 D^ sin u = cos u D,u 16 Dj csch M = —csch w coth « D^w 24 D, cot"' M :

1 + u^
6 D, cos M = — sin M D,u 17 D^" = e" D^
7 D, tan u = sec^ m D^m 25 D, sec"' M
18 D,a" = a" In a D,m |m|Vm^ - 1

8 Dj cot M = -csc^ u D,M


19 D, log,, « = 26 Dj CSC"' «
9 D^ sec M = sec u tan u D^ u In a u\Vu^ - 1

10 D^ CSC u = —CSC M cot « D^M = = f(u)D,u


20 D. in w 27 D f(t) dt
J
nsw BOOK $51. 45 ijcrn
Tables of .Integrals (constants o^n^^gf'"'^ l^^^^ '*^'' omitted to save space)

Elementary forms

1 du = (n^-l) 10 cot u du = \n |sin u| 19 cosh u du = sinh m


n + 1

J
—u
= In |m|
11 sec u du = \n |sec m + tan u\ 20 sech" M ^M = tanh u

12 CSC M Jm = In |csc M — cot m| 21 csch" u du = -coth m


sin M du = —cos «
J

13 sin^ u du = 2U — 1 sin 2m 22 sech M tanh u du = —sech u


cos M <iM = sin u
I

14 COS" u du = 2U + 4 sin 2m 23 csch M coth u du = —csch u


sec" M c/u = tan u
rfw M
IS
,

24 u dv = uv — \ V du
csc^ M ^« = —cot u

du 25 e" du = e"
7 sec M tan u du = sec m 16
a^ + M- a"
26 a" du =
8 j
CSC M cot u du = —CSC m du In a
17 1—^ 1 ~ ^'
mVm" — a~ a
9 I tan M t/w = —In |cos m|
18 sinh u du = cosh u

IWgonometric Forms

27 tan" u du = ta.n u — u
I

28 cot^ u du = —cot M — M
"-2 r „_,
J H sec " M Jm
/I - 1 J
r sin" ' u cos « n — \ { _,
29 sin" u du = 1
sin" - u du r i

^ " « ' 34 CSC" udu= esc" ^ M cot M


-I -^ "-1
r cos M sin M •
w — ,
1
r
_-,
du = -
I
30 cos" M 1
cos" ~ u du n 2
J n n J -t ^ . + CSC - M du
n - 1

31
J
tan" M du = ——— tan""' m - I tan"~^ u du 35 I u" sin u du = -u" cos u + n \ u" cos M c/«

m" cos u du = u" sin m — n «" ' sin m du


32 cot" u du = cot""' u — cot""- u du
J M- 1 J

;3a^
Inverse Trigonometric Forms
f _, lu^ - 1 mVi -
37 sin"' u du = u sin"' u + Vl — m" 40 M sin u du = sin m +
J 4

38 cos"' // rfw = M cos"' M — Vl — M^ _, 2u- - 1 _,


f =
41 M cos u du cos M -
J 4 4
39 I tan"' u du = u tan"' u In (1 + u^)

42
r
\ u tan
- u du = —
1
(m"
-
+ 1) tan
-.
m —— «
Digitized by the Internet Archive
in 2010

http://www.archive.org/details/calculuswithanalOOmune
SECOND EDITION

CALCULUS
with Analytic Geometry
SECOND EDITION

CALCULUS
with Analytic Geometry

M. A. Muneni Macomb Community College

D. J. Foulis University of Massachusetts

Worth Publishers, Inc.


Calculus with Analytic Geometry, second edition

Copyright © 1984 by Worth Publishers. Inc.


All rights reserved.

Printed in the United States of America

Library of Congress Catalog No. 83-50583

ISBN: 0-87901-236-6

First printing, January 1984

editor: Anne Vinnicombe

production: George Touloumes and Pat Lawson

designer: Malcolm Grear Designers

illustrator: Mel Erikson Art Services

typographer: York Graphic Services. Inc.

printer and binder: Kingsport Press

cover: Sinusoidal surface. Computer graphics by Thomas Banchoff. David Laidlaw. and
David Margolis.

Text figure and picture credits appear on page 1033. which constitutes a continuation of the
copyright page.

Wortli Publishers, Inc.


444 Park Avenue South
New York, New York 10016
CONTENTS

Preface
Introduction

CHAPTER 1 FUNCTIONS AND LIMITS 1

1.1 Real Numbers, Inequalities, and Absolute Value 1

1.2 The Cartesian Coordinate System 10

1.3 Straight Lines and Their Slopes 16

1 .4 Functions 22
1.5 Types of Functions 31

1.6 Trigonometric Functions 41

1.7 Limits of Functions 52


1.8 Properties of Limits of Functions 60
1.9 Continuous Functions 69

CHAPTER 2 THE DERIVATIVE 85


2.1 Rates of Change and Slopes of Tangent Lines 85

2.2 The Derivative of a Function 92


2.3 Basic Algebraic Rules for Differentiation 98
2.4 Tangent and Normal Lines 109

2.5 Rules for Differentiating Trigonometric Functions 114

2.6 Function Composition 120

2.7 The Chain Rule 124

2.8 Implicit Functions and Implicit Differentiation 132

2.9 The Rational-Power Rule 138

2.10 Higher-Order Derivatives 143

2.1 Algebraic Signs and Zeros of Functions 150

CHAPTER 3 APPLICATIONS OF THE DERIVATIVE 165


3 . The Mean- Value Theorem 165

3.2 Monotonicity and the First-Derivative Test 172


3.3 Concavity and the Second-Deiivative Test 180
3.4 Absolute Extrema 188
3.5 Asymptotes and Limits Involving Infinity 194
3.6 Graph Sketching 204
3.7 Applied Maximum and Minimum Problems 211
3.8 Related Rates 221
3.9 Applications to Economics and Business 228

( HAPTiiR 4 ANTIDIFFERENTIATION AND DIFFERENTIAL EQUATIONS 243


4.1 Differentials and Linear Approximation 243
4.2 Antiderivatives 251
4.3 The Method of Substitution, or Change of Variable 258
4.4 Differential Equations 264
4.5 Applications of Differential Equations 274
4.6 The Harmonic-Oscillator Equation 281
4.7 Areas by the Method of Slicing 290

CHAPTER 5 THE DEFINITE INTEGRAL 299


5.1 The Sigma Notation for Sums 299
5.2 The Definite Integral 307
5.3 Basic Properties of the Definite Integral 316
5.4 The Fundamental Theorem of Calculus 327
5.5 Approximation of Definite Integrals — The Trapezoidal Rule and
Simpson's Rule 338
5.6 Areas of Regions in the Plane 346

CHAPTER 6 APPLICATIONS OF THE DEFINITE INTEGRAL 357


6. Volumes of Solids of Revolution 357
6.2 The Method of Cylindrical Shells 365
6.3 Volumes by the Method of Slicing 369
6.4 Arc Length and Surface Area 375
6.5 Work, Force, and Energy 383
6.6 Applications to Economics and the Life Sciences 393

CHAPTER 7 TRANSCENDENTAL FUNCTIONS 401


7.1 Inverse Functions 401
7.2 Inverse Trigonometric Functions 409
7.3 Derivatives of and Integrals Yielding Inverse Trigonometric Functions 418
7.4 The Natural Logarithm Function 426
7.5 Properties of the Natural Logarithms 431
7.6 The Exponential Function 438
7.7 Exponential and Logarithmic Functions with Base Other Than e 445
7.8 Hyperbolic Functions 452
7.9 Exponential Growth and Decay 458
7.10 Mathematical Models for Biological Growth 468
7.11 First-Order Linear Differential Equations 474
CHAPTER 8 TECHNIQUES OF INTEGRATION 485
8.1 Integrals of Powers of Sines and Cosines 485
8.2 Integrals of Powers of Other Trigonometric Functions 490
8.3 Integration by Trigonometric Substitution 494
8.4 Integration by Parts 499
8.5 Integration by Partial Fractions — Linear Case 507
8.6 Integration by Partial Fractions — Quadratic Case 516
Integration by Special Substitutions 524
Tables of Integrals and Reduction Formulas 528

CHAPTER 9 POLAR COORDINATES AND ANALYTIC GEOMETRY 534


9. Polar Coordinates 534
9.2 Sketching Polar Graphs 541
9.3 Area and Arc Length in Polar Coordinates 549
9.4 The Ellipse 555
9.5 The Parabola 564
9.6 The Hyperbola 570
9.7 Rotation of Axes 577
9.8 Eccentricity and Conies in Polar Form 585

CHAPTER 10 INDETERMINATE FORMS, IMPROPER INTEGRALS, AND


TAYLOR'S FORMULA 596
10. The Indeterminate Form 0/0 596
10.2 Other Indeterminate Forms 603
10.3 Improper Integrals with Infinite Limits 608
10.4 Improper Integrals with Unbounded Integrands 613
10.5 Taylor Polynomials 617

11 INFINITE SERIES 630


11.1 Sequences 630
1 1.2 Infinite Series 641
11.3 Properties of Infinite Series 650
1 1.4 Series of Nonnegative Terms 658
11.5 Series Whose Terms Change Sign 669
1 1.6 Power Series 680
1 1.7 Differentiation and Integration of Power Series 686
11.8 Taylor and Maclaurin Series 692
11.9 Binomial Series 701

12 VECTORS IN THE PLANE 709


12.1 Vectors 709
12.2 The Dot Product 717
12.3 Vector and Parametric Equations 724
12.4 Vector- Valued Functions of a Scalar 733
12.5 Velocity, Acceleration, and Arc Length 740
12.6 Normal Vectors and Curvature 745
12.7 Applications to Mechanics 750

13 COORDINATE SYSTEMS IN THREE-DIMENSIONAL SPACE 760


13.1 Cartesian Coordinates in Space 760
13.2 Vectors in Three-Dimensional Space 764
13.3 Cross and Box Products of Vectors in Space 771
13.4 Equations of Lines and Planes in Space 781
13.5 The Geometry of Lines and Planes in Space 789
13.6 Curves in Space 797
13.7 Spheres, Cylinders, and Surfaces of Revolution 806
13.8 Quadric Surfaces 812
13.9 Cylindrical and Spherical Coordinates 820

14 PARTIAL DIFFERENTIATION 830


14.1 Functions of Several Variables 830
14.2 Limits and Continuity 837
14.3 Partial Derivatives 845
14.4 Linear Approximation and Differentiable Functions 853
14.5 The Chain Rules 863
14.6 Directional Derivatives. Gradients. Normal Lines, and Tangent Planes 873
14.7 Higher-Order Partial Derivatives 886
14.8 Extrema for Functions of More Than One Variable 894
14.9 Lagrange Multipliers 904

CHAPTER 15 MULTIPLE INTEGRATION AND VECTOR CALCULUS 915


15.1 Iterated Integrals 915
15.2 The Double Integral 920
15.3 Evaluation of Double Integrals by Iteration 928
15.4 Double Integrals in Polar Coordinates 936
15.5 Applications of Double Integrals 944
15.6 Triple Integrals 954
15.7 Triple Integrals in Cylindrical and Spherical Coordinates 961
15.8 Applications of Triple Integrals 968
15.9 Line Integrals and Green's Theorem 973
15.10 Conservative Vector Fields 985
15.11 Surface Area and Surface Integrals 993
15.12 The Divergence Theorem and Stokes' Theorem 1003

APPENDIX A FORMULAS FROM GEOMETRY 1015


APPENDIX B PROOFS OF BASIC PROPERTIES OF LIMITS AND
CONTINUOUS FUNCTIONS 1016
APPENDIX C MATHEMATICAL INDUCTION 1021

APPENDIX D TABLES 1025

ANSWERS TO SELECTED PROBLEMS A-1


ILLUSTRATION CREDITS 1033
INDEX 1035
PREFACE

PURPOSE The second edition of Calculus has been extensively rewritten and improved. Its
basic purpose, however, remains the same — to provide the necessary background in
calculus and analytic geometry for students of mathematics, engineering, physics,
chemistry, economics, or the life sciences.

PREREQUISITES Students who use this textbook should know the basic principles of algebra, geome-
try, and trigonometry covered in the usual precalculus mathematics courses. No
previous study of analytic geometry is required.

OBJECTIVES This book was written and revised with two main objectives in mind:
First, to provide the reader with a competent working knowledge of calculus. We
have taken pains to insure adequate coverage of the traditional topics and tech-
niques, especially those that will be needed in courses such as engineering, physics,
chemistry, and other sciences. Furthermore, we have tried to attune the textbook to
the —
two outstanding mathematical trends of our time the computer revolution and
the burgeoning use of mathematical models in the life sciences, the social sciences,
economics, and business.
Second, to complement and enhance effective classroom teaching and to allay
students'"math anxiety." Our own classroom experience has shown that success-
ful teaching is fostered by a textbook that provides clear and complete explanations,

cogent illustrations, step-by-step procedures, pertinent examples and problems, and


a multitude of topical, real-world applications. Such a textbook should build student
confidence by beginning with familiar material, explaining new concepts in terms
of ideas already well understood, offering worked-out examples that show in detail
how problems are to be solved, and providing opportunities for students to reinforce
and test their understanding via problem sets in which the level of difficulty in-
creases gradually.

SPECIAL FEATURES Aspects of the book that have worked well have been carried over from the first
edition and a number of new features, designed to enhance readability and stimulate
student interest, have been introduced.
1 Examples and Figures numerous examples, we offer guidance to
In the
students in organizing work and in deciding when a shortcut may h>e
liieir

reasonable. The examples are worked in detail with all substitutions shown.
Figures and graphs are used to enhance verbal explanations whenever possible.
2 Problem Sets Problems at the end of each section progress from simple
drill-type exercises, amply by worked-out examples, to more de-
illustrated

manding conceptual questions. To solve the odd-numbered problems requires a


level of understanding sufficient for most purposes, whereas the even-num-

bered problems toward the end of each problem set are more challenging and
probe for deeper understanding. To generate student interest, the problems
include applications to a wide variety of fields — not only to engineering, ge-
ometry, and the physical sciences but also to biology, business, the earth
sciences, ecology, economics, medicine, navigation, and the social sciences.
Answers to odd-numbered problems are given in the back of the book and
solutions to all problems are provided in a Solutions Manual for instructors.
In each problem set, a group of problems that provide a good representa-
tion of the main ideas of the section has been identified with numerals printed
in color. This feature is helpful for students preparing for quizzes and exams,
and it can be used by instructors in selecting problems for assignment.
3 Review Problem Sets The review problems at the end of each chapter can
be used in a variety of ways: Instructors may wish to use them for supplemen-
tary or extra-credit assignments or for quizzes and exams; students may wish to
scan them to pinpoint areas where further study is needed. In some places,
these problems purposely are not arranged by section so that students can gain
experience in recognizing types of problems as well as in solving them.

4 Use of Calculators In keeping with the recommendations of the National


Council of Teachers of Mathematics (NCTM) and the Mathematical Associa-
tion of America (MA A), we have de-emphasized the use of tables in favor of
the use of scientific calculators. We feel obliged to prepare students to function

outside the classroom, where virtually everyone using mathematics in a practi-


cal way — from the actuary to the zoologist — routinely employs a calculator.

In this book, we have tried to assign the calculator its rightful place — a tool,
useful at times, unnecessary at other times. Problems and examples that call
for the use of a calculator are marked with the special symbol E
5 Computer Graphics The use of computers to generate graphs is fascinat-
ing and illuminating to students and instructors alike. We have included a
number of computer-generated graphs in Chapters 3, 13, and. 14.

6 SI {Metric) Units In the majority of physics and engineering problems we


have employed the SI (Systeme International) system of units, which is rapidly
replacing the customary British and U.S. systems in all applied work.

7 Historical Notes and Portraits Brief historical notes and portraits of


prominent mathematicians serve to remind the reader that mathematics is a
human endeavor.
8 Design In this edition, a more open design is used for ease of reading and
bettercomprehension. Newly defined terms are indicated in boldface type:
important results and formulas are highlighted in color and .set off by ruled
boxes: and the symbol is used to signal the end of a proof or the end of the
solution to an example.

9 Formulas Pertinent formulas from geometry, trigonometry . and calculus


are listed in the appendix and inside the front and back covers of the book.
ARRANGEMENT OF TOPICS Although the table of contents clearly identifies the topics covered and their order of
presentation, the following supplementary comments may be helpful.

1 Trigonometry Because deficiencies plague many calculus


in trigonometn,'
students, a substantial review of trigonometry precedes coverage
of limits and
continuity of the trigonometric functions in Chapter 1 At the
request of many
.

of our first-edition users, we have introduced early differentiation


the trigo- of
nometric functions in Chapter 2. This provides for more meaningful applica-
tions of the chain rule and enlarges the scope of the applications of the deriva-
tive in Chapter 3.

2 Algebraic Signs and Zeros of Functions An expanded section emphasiz-


ing the use of test values to determine algebraic
signs of functions and present-
ing both the bisection method and Newton's method for approximating zeros is
placed in Chapter 2. This material is particularly well-adapted to the use of
computers and calculators.

3 Concepts Used in Physics and Engineering The tools and


techniques that
are needed by students of engineering and science are developed
as early as
possible in the text. The important idea of setting up and solving simple differ-
ential equations is given suitable emphasis.

4 Geometry Throughout the book, the reader is continually encouraged to


visualize analytic relationships in geometric form. This
is particularly tirie for
the treatment of vectors— all concepts involving
vectors are first introduced
geometrically.

5 New Material To reflect the current needs and interests of students


takins
calculus, new material has been added on the following topics:

(a) Harmonic oscillators (Chapter 4)


(b) Linear differential equations (Chapters 4 and 7)
(c) Logistic growth model (Chapter 7)
(d) Method of least squares (Chapter 14)
(e) Jacobians (Chapter 15)
(f )
Conservative vector fields and potentials (Chapter 15)

6 Optional Material Because a topic that one person regards as optional


could be deemed essential by others, we have refrained
from marking any of
the material in this book as 'optional." However, material
that could'be con-
sidered optional has been placed at the ends of sections
or chapters, where it
can be omitted without loss of continuity.

PACE With adequately prepared students, the entire book can be covered in three semes-
ters or in five quarters. In general, chapters 1 through 5 include enough material for
a fu-st-semester course, chapters 6 through 1 1 are suitable for the second semester,
and the remaining chapters can be covered in the third semester. The book is written
for maximum flexibility; there are many ways to arrange the material coherently to
conform to a wide variety of teaching situations.

SUPPLEMENTARY MATERIALS Study Guide An accompanying Study Guide is available for students who require
more or more assistance in any topic. The Study Guide conforms with the
drill

arrangement of topics in the book and contains many carefully graded fill-in
state-
ments and problems broken down into simple units. Study objectives and tests are
also included for each chapter. Answers are given for all problems in the Study
Guide in order to encourage the building of skills and confidence.

COMPUCALC — Computer Calculus Supplement A microcomputer diskette and


workbook by Robert J. Weaver of Mt. Holyoke College, designed to be used in

conjunction with the textbook, are available for students with access to a microcom-
puter. COMPUCALC consists of 19 "user friendly" computer programs, many of
them employing high-resolution graphics, and related workbook projects that guide
the student to make full use of the computer's ability to vividly demonstrate the
conceptual and computational power of calculus.

Instructor' s Resource Manual This manual provides a comprehensive testing pro-


gram, closely coordinated with the textbook. It includes an examination for each
chapter and three comprehensive examinations. Suggestions are offered on how
each topic might be presented and alternative sequences of topics are suggested.

Solutions Manual The step-by-step solution to each problem in the textbook is

available in this manual for instructors. By glancing through the worked-out solu-
tions, an instructor can select those problems that will provide the kind of practice
students need for each section of the book.

ACKNOWLEDGMENTS By the time a book reaches its second edition, a great many people have contributed
to its development. We wish to thank the many individuals who have provided
remarks and helpful suggestions on the first edition. In particular, we would like to
express our gratitude to the following people: Nancy Angle, University of Colo-
rado, Denver. Frank Anger, University of Puerto Rico; Olga Beaver, Williams
College: Murray Eisenberg, University of Massachusetts. Amherst; Robert C. Geb-
hardt, Coimty College of Morris: David Hayes, College of Notre Dame: Herbert
Kamowitz, University of Massachusetts, Boston: Eleanor Killam, University of
Massachusetts, Amherst: William McKinley, Eastern Montana College: Bruce
McQuarrie, Worcester Polytechnic Institute: Robert Piziak, Baylor Universir\': Karl
Rehmer, Blackburn College: Franklin Schroeck, Jr., Florida Atlantic University:
Ronald Smit, Universits' of Portland: Charles Stone, DeKulb Community College:
Thomas Tredon, Lord Faiifax Community College: James Wahab, University of
South Carolina, Columbia: Harry Whitcomb, The Philadelphia College of Phar-
macy and Science: Robert White, North Carolina State University.
In preparing this second edition, we were fortunate to have the advice and guid-
ance of many reviewers; the knowledge and skills they shared with us have greatly
enhanced this book. For their contribution, we thank:

Robert B. Burckel, Bruce Edwards,


Kansas State University University of Florida

Ray E. Collings, August J. Carver,


Tri-County Technical University of Missouri, Rotla
College Louis M. Herman,
Daniel G. Dewey, Kansas State University
College of the Holy Cross Frank E. Higginbotham,
Roger T. Douglass, University' of Puerto Rico,

Alfred University Rio Piedras


Jerry Johnson, Laurence Small,
Oklahoma State University Los Angeles Pierce College
Roy Kelly, Howard E. Taylor,
Jackson Community College West Georgia College
Gary Lippman, John von Zellen,
California State University, Macomb Community College
Hayward Elaine Kirley Whittlesy,
J. J. Malone, Sierra College
Worcester Polytechnic Institute
James L. Moseley,
West Virginia University

Special thanks are due to Hyla Gold Foulis for reviewing each successive stage of
manuscript, proofreading, and solving problems; to Steve Fasbinder, Oakland Uni-
versity, for reviewing the manuscript, reading page proof, and solving problems;
and to Jarema Chypchar. Barbara Bloom, and Lela Grant for reading page proofs.
We also extend our appreciation to the problem checkers:Roger Douglass, Alfred
University: Kathy Franklin, Larry Small, and Ann Watkins, Los Angeles Pierce
College. John Spellman and Ricardo Torrejon, Southwest Texas State University,
and Elaine Kirley Whittlesy, Sierra College. Finally, we wish to express our grati-
tude to the staff at Worth Publishers, especially Rory Baruth, for their constant help
and encouragement.

M. A. Munem
January, 1984 D. J. Foulis
INTRODUCTION

Mathematics has been developed, cultivated, and refined


not only for its practical
applications to science, commerce, and industry,
but also because of its aesthetic
attractions. The ancient Egyptians developed some of the fundamental ideas of
tngonometry for the very practical purpose of relocating
property lines after the
periodic Nile floods; the early Greeks, however,
studied the conic sections largely
because they were intrigued with the beautiful geometry
of these graceful curves
Nearly two thousand years later, the Renaissance astronomer Johannes Kepler
(1571-1630) discovered that the paths of the planets as they orbit the
sun are conic
sections (ellipses). Thus, mathematical ideas,
originally developed because their
elegance appealed to the human mind, later found
practical application in astron-
omy and celestial mechanics.
The Italian mathematician, astronomer, and physicist Galileo
Galilei (1564-
1642), a contemporary of Kepler, showed that objects thrown into the air also

Johannes Kepler Galilfo Galilei


INTRODliCTION

follow conic sections (parabolas). Galileo's experiments and meditation on the prin-
ciples of mechanics moved him to declare that the ""Book of Nature is written in
mathematical characters." Within fifty years of this astonishing proclamation. Sir
Isaac Newton (1642-1727) in England and Gottfried Leibniz 1646-1716) in Ger- (


many independently created calculus the mathematical language with which
many chapters of Galileo's "Book of Nature" have been and continue to be written.
Fired with enthusiasm by this major intellectual triumph, generations of mathemati-
cians and scientists over the next two centuries developed and perfected the subject
of classical mechanics — the foundation of most branches of engineering.
Two more quantum mechanics, were added to the "Book
chapters, relativity and
of Nature" in the twentieth century. As Galileo had foreseen, these chapters are
also "written in mathematical characters," among which the derivatives and inie-
grals of Newton and Leibniz' calculus are especially prominent.
What accounts for the conspicuous role of calculus in Galileo's "Book of Na-
ture"? The fundamental reason is that the world is full of change and calculus is the
mathematics of change. For the same reason, calculus is profoundly involved in the

burgeoning use of maiheinatical models in the life and social sciences, economics,
and business. These mathematical models usually take the form of differential equa-
tions that express relationships among the rates of change of variable quantities.
In this book, we stress the idea of calculus as the mathematics of change and we
emphasize the use of differential equations as mathematical models for real-world
phenomena. We hope that our readers will be able to recapture some of the excite-
ment felt by the architects of calculus when they first began to realize the immense
power and beauty of their creation. Some of these readers, we like to think, may
themselves help to write the next chapter in Galileo's "Book of Nature."

Sir Isao, \ GollJncJ l-cihtiiz


i fTTV
FUNCTIONS AND LIMITS

We begin this chapter with a brief review of some precalculus mathematics, after
which we introduce the two critical ideas upon which calculus is based function
and limit. Using the idea of a limit of a function, we formulate and study the
important concept of continuity.

1.1 Real Numbers, Inequalities, and


Absolute Value
The familiar idea of a number scale, number line, or coordinate axis makes it

possible to visualize real numbers as points along an infinite straight line (Figure 1).

Figure 1 o"&"

The real number corresponding to a point on the line is called the coordinate of the
point. An arrowhead may be used to indicate the direction (to the right in Figure 1)

in which the coordinates are increasing. The point with coordinate is called the
origin, and the point with coordinate 1 is called the unit point. The distance be-
tween the origin and the unit point is called the unit distance on the number scale.
On a horizontal number scale, points to the right of the origin have positive coordi-
nates, and points to the left of the origin have negative coordinates. The set of all

coordinates of points on the line is called the set of real numbers and is denoted by
the special symbol U.
Two real numbers .x and v in IR can be combined by the usual arithmetic opera-
tions to yield new real y, x — y, .vy. and (provided y ^ 0) .v/y. If y is
numbers x +
positive, can also raise y to the power x and obtain a definite real number y\
we
however, if y is negative, y' is not always defined as a real number. For instance,
(—1)1/2 _ Y'^Tj jg fiQj defined as a real number.
CHAPTER I FINCTIONS AND LIMITS

The real numbers 1 , 2. 3, 4. 5. and so on are called the natural numbers, or the
positive integers (Figure 2). The integers consist of all the natural numbers, the
negatives of the natural numbers, and zero (Figure 3). Real numbers that can be

Figure 2 the natural numbers Figure 3

written in the form a/h. where a and b are integers and fc 5^ 0, are called rational
numbers. The irrational numbers are those real numbers that are not rational. A
real number is irrational if and only if its decimal representation is nonterminating
and nonrepeating. Examples of irrational numbers are

V2= 1.4142135 •• •
and tt = 3.1415926 • •

Inequalities

Two different real numbers .v and y can always be compared to determine which is

greater. If the point with coordinate .v lies to the left of the point with coordinate y
Figure 4
on the number line (Figure 4). we say that y is greater than .v (or, equivalently that .

X is less than v). and we write

y>x x<y
Note that

.V > means that .v is positive.


.V < means that .v is negative.

The integer is neither positive nor negative.


A statement of the form x <y (or y > .v) is called an inequality. You may
already be familiar with the following rules for handling inequalities.

Let a, h. and c denote real numbers.

1 Trichotomy One and only one of the following is true:

a < b or b < a or a = b

2 Transitivity U a < b and b < c. then a < c.


3 Addition U a < b, then a + c < b + c.
4 Subtraction If a < b. then a - c < b — c.

5 Multiplication U a < b and c > 0, then ac < be.

then — < —
a b
6 Division If a < b and c > 0.

According to the addition and subtraction rules, you can add or subtract the same
quantity on both sides of an inequality. The multiplication and division rules permit
you to multiply or divide both sides of an inequality by a positive number. Accord-
ing to the following rule, if you multiply or divide both sides of an inequality by a
negative number, you must reverse the inequality.
SECTION 1.1 REAL NUMBERS, INEQUALITIES, AND ABSOLUTE VALUE

7 Inequality reversing If a < b and c < 0, then ac > be and —>—


For instance, if you multiply both sides of the inequality 2 < 5 by -3, you must
reverse the inequality and write -6 > - 15; that is, - 15 < -6. Indeed, on a num-
ber line, —15 lies to the left of —6.
Sometimes we know only that a certain inequality does not hold. If .v < v does
not hold, then by the trichotomy rule either .v > y or.v = y. In this case, we say that
X is greater than or equal to y, and we write

X >y (or, equivalently, y ^ x)

Statements of the form y < x (or .v > y) are called strict inequalities; those of the
form y x > y) are called nonstrict.
—x (or
If we < y < z, we mean that < y and y < z. Likewise, x^y>z means
write x .v

that.r S y andy > z. This notation for combined inequalities is used only when the

inequalities run in the same direction.


Statements analogous to Rules 2 through 7 can be made for nonstrict inequalities
and for combined inequalities. For instance, if you know that

-1 <3 - — <5
2

you can multiply all members by -2, provided that you reverse the inequalities to
obtain

2>.v- 6> -10


or -10<a:-6<2
If you are asked to "solve" an equation, it's usually clear what is wanted:

namely, the value (or values) of the variable (or variables) that makes the equation
true. Similarly, you solve an inequality by finding all values of the variable (or
variables) that make the inequality true. The set of all such values is called the
solution set of the inequality.

EXAMPLE 1 Solve the inequality x + 3 < 5a: - 1

SOLUTION jr + 3 < 5.V - 1 (Given)

3 < 4x — 1 (We subtracted x from both sides.)

4 < 4jc (We added 1 to both sides.)

l<x (We divided both sides by 4.)

Thus, the solution set consists of all real numbers that are greater than 1 (Figure 5).

Figure 5 1 does not belong


to the solution set

\ solulion set
-i

In Figure 5, the solution set consists of one connected piece. Such sets, called
intervals, often arise as the solution sets of inequalities.
CHAPTER 1 FUNCTIONS AND LIMITS

DEFINITION 1 Bounded Intervals

Let (/ and /; be fixed real numbers with a < b.

(i)The open interval (a, b) with endpoints a and b is the set of all real
numbers .v such that a <x< b.

a does not b does not


belong to the interval belong to the interval
f ) -
a h

(ii) The closed interval [a, b] with endpoints a and b is the set of all real

numbers x such that a < .v < fe.

a belongs b belongs
to the interval to the interval

E 3

(iii) The half-open interval [a, b) with endpoints a and b is the set of all

real numbers .r such that a ^x< b.

a belongs b does not


to the interval belong to the interval

E )

(iv) The half-open interval (a, b] with endpoints a and b is the set of all

real numbers .v such that a < x^ b.

a does not b belongs


belong to the interval to the interval
(
\

Notice that a closed interval contains its endpoints, but an open interval does not. A
half-open interval (also called a half-closed interval) contains one of its endpoints,
but not the other.
Unbounded intervals, which extend indefinitely to the right or left, are written
with the aid of the special symbols +^ and — ^c, called positive infinity and
negative infinity, respectively.

DEFINITION 2 Unbounded Intervals

Let a be a fixed real number.

(i) (a, +0C) is the set of all real numbers x such that a<x.
a does not
belong to the interval

(ii) (-00, a) is the set of all real numbers x such that x < a.
a does not
belong to t!ie interval

(iii) [a, +0O) is the set of all real numbers x such that a < j:.

a belongs
to the intenal
E -

(iv) (-00, a] is the set of all real numbers x such that x^ a.


a belongs
to the interval
^
SECTION 1.1 REAL NUMBERS, INEQUALITIES, AND ABSOLUTE VALUE

It must be emphasized that +=<= and -^c are just convenient symbols they are not
real numbers and should not be treated as if they were. In the notation for un-
bounded intervals, we usually write ^ rather than +^. For instance, (5, oc) denotes
the set of real numbers greater than 5. The notation (-^, oo) is often used to denote
the set R of all real numbers.

In Examples 2 and 3. solve the inequality and show the solution set on a number
line.

EXAMPLE 2 - 11 S Iv - 3 < 7

SOLUTION We begin by adding 3 to all members to help isolate x in the middle:

-8 < 1y < 10

Then we divide all members by 2, to obtain

Figure 6 _4 < ^.
< 5
1-4,51
r ^ Therefore, the solution set is the half-open interval [—4, 5) (Figure 6).
)
-4 5

EXAMPLE 3 X^ -I- 3a + 2 >


SOLUTION We begin by factoring x^ + 3x + 2 = (x + 1) (x + 2) and rewriting
the given inequality as
(x+ l){x + 2)>0
Here, equality holds exactly when .v = —1 or x = —2. Also, the strict inequality

{x+ l)(x + 2)>0


holds if and only if .v -I- 1 and x + 2 have the same algebraic sign. First, let's
consider the case in which both x + 1 and .r -I- 2 are positive. Then

a: -I- 1 > and .v -I- 2 >


that is, .V > -I and x > -2
Note that if .v > - 1 , then .v > -2 automatically holds; hence, both x + 1 and x + 2
will be positive precisely when

x> -1

Next, we consider the case in which both x + 1 and j: -I- 2 are negative. Then

.V -1-
1 < and .x: + 2 <
that is, .v< -1 and x < -2
Note that if.v < —2, then.v < —1 automatically holds; hence, both.v -i- 1 and.x + 2
will be negative precisely when
x< -2
It follows that

.r -I- 3.V -t- 2 >


holds if and only if

Figure? .v<-2 or v > -


(-«,-21 1-1. ~i
^ [ I
^ In other words, the solution set consists of the two intervals (-^, -2] and [-1, 0°)
-2 -1 (Figure 7). I
CHAPTER 1 FUNCTIONS AND LIMITS

Absolute Value
The idea of absolute value plays an important role in analytic geometry and calcu-
lus, especially in expressions that involve the distance between two points on a
number line.

DEFINITION 3 Absolute Value

if A is a real number, then the absolute value of .v, denoted l.v|

Figure 8
SECTION 1.1 REAL MMBERS. INEQl ALITIES. AND ABSOLLTE \ALVt.

Many of the properties of absolute value can be established by considering all


possible cases in which the quantities involved are positive, negative, or
zero.

EXAMPLE 4 Show that -|.r| < .t < |.r| holds for every real number .v.

SOLUTION > 0, then = x, so < holds. Also, if > 0, then -\x\ =


If .r |jc| a: \x\ .v

-j: < < so -\x\ < X holds. Thus, -|.r| <


jc, < holds when x>0. On .v |.r|

the other hand, if x < 0, then = -.r, so -|.t| = < < -.r = hence, |.t| .t |.r|:

-|.v| < X < also holds when x < 0.


l.vl

One of the most imponant properties of absolute value is the triangle inequality
liven in the following theorem.

THEOREM 1 Triangle Inequality

If a and b are real numbers, then |a + fo| < \a\ + \b\.

PROOF By the preceding example, -|a| < a s |a| and -\b\^ bs \b\. Adding these in-
equalities, we obtain

-(|a| + \b\) ^a + b^\a\ + \b\

U a + b>Q. then \a + b\ = a + b ^\a\ + \b\. On the other hand, if a + b<0,


then |a + fc| = -(a + b) < \a\ + \b\. In either case, we have |a + fc| < \a\ + \b\.

In solving equations and inequalities involving absolute values, you will tlnd the
followina rules useful.

Let ;/ and v be real numbers, and suppose a is a positive real number. Then
(i) |m| = a if and only if w = a or it = -a.
(ii) \u\ = |v| if and only if u = v or u = -v.
(iii) \u\ <a if and only if -a < it < a.
(iv) |h| >a if and only if u < —a or u > a.

Rules (i) and (ii) are obvious from the definition of absolute value. Rules (iii) and
(iv) are easy to understand and remember if you keep in mind that \ii\ is the distance
between u and 0.

In Examples 5 to 7, solve each equation or inequality

EXAMPLE 5 1 1 - lx| = 7

SOLUTION By Rule (i). |l - 2r| = 7 is true if and only if

1
- Iv = 7 or I - 2v = -7
that is,

-Iv = 6 or -2r = -8
Therefore, the equation has two solutions, x = -3 and x = 4. In other words, the
solution set consists of the numbers -3 and 4. i
CHAPTER 1 FUNCTIONS AND LIMITS

EXAMPLE 6 |3.r - 2| <4


SOLUTION By Rule (iii), the inequality is true if and only if

-4 < 3jc - 2 < 4

that is, -2 < 3a: < 6

or -§ < ;c < 2

Therefore, the solution set is the interval (-§, 2).

EXAMPLE 7 |3y + 2| > 5

SOLUTION By Rule (iv), the inequality is true if and only if

3.V + 2 < -5 or 3.V + 2 > 5

that is, 3y < -7 or 3^ > 3

Hence, the inequality is true if and only if

y < -i or y> I

In other words, the solution set consists of the two intervals (-oc, -J) and (1, ^).

Problem Set 1.1

V 1 Answer true or false. Use suitable rules for inequalities to justify 8 If .V > 0. is it necessarily true that l/.v > 0? Why?
the true statements, and give examples to show that the false
9 The following are solution sets for certain inequalities. Illustrate
statements really are false.
each solution set on a number line.
(a) If j: is a positive number, then 5;i: is a positive number.
(a) [-2, 3)
(b) If jt <3 and V > 3, then x<y.
(b) All numbers x such that -3 < x < 4 and also —6 < jt < 2
(c) If jc < _v, then -5jc < -5y.
(c) All numbers belonging to [-2, 0] or to [-^, 1] or to both of
(d) If jr2<9, then a: < 3.
these intervals
(e) If j: > 2 and v > -t, then y > 0.
(d) All numbers belonging to (0, ^) or to (-^. 0)

2 Show that if j: t^ 0, then .r > 0. (Hint: By the trichotomy rule, (e) All numbers .v that belong to both intervals (0, ») and (3, «)
if X 7^ 0, then either a: > or x < 0. Consider the two cases
(f) All numbers x that belong to both intervals (— =o, -2] and
x> and .v < separately.) (-°o, -5)

3 Use the rules for inequalities to show that A < ^- [Hint: Begin
10 Use interval notation to represent each shaded set below.
by showing that 3(233) < 28(25). Then use the division rule for
inequalities.]
(a) -) ir ^^
4 Show that ifO<x<y, then \/x > \/y. {Hint: Use the division
rule for inequalities twice.) (b)

5 (a) Under what conditions is -x > 0? Explain, (b) Under what


conditions is -x < 0? Explain, (c) Under what conditions is
(c)

—X = 0? Explain.

6 (a) If < Jr < y. show that .r^ < y^. (b) If < a^ < v, show that
(d)
-E h
3 3 is excluded
Vjt< Vy.
(e)
7 If -ir s 9, is it necessarily true that x > 3? Explain.
SECTION 1.1 REAL NUMBERS, INEQUALITIES, AND ABSOLUTE VALUE

In Problems 1 1 to 24, solve each inequality. Express the solution in

interval notation, and illustrate it on a number line.

11
10 CHAPTER 1 FUNCTIONS AND LIMITS

1.2 The Cartesian Coordinate System


In Section 1.1, we saw that a point P on a number line can be specified by a real

number A called its coordinate. Similarly, by using a Cartesian coordinate system,


named in honor of the French philosopher and mathematician Rene Descartes
( 1596-1650), we can specify a point P in the plane with two real numbers, also
called coordinates.
A Cartesian coordinate system consists of two perpendicular number lines,

called coordinate axes, which meet at a common origin O (Figure 1). Ordinarily,
one of the number lines, called the x axis, is horizontal, and the other, called the y
axis, is vertical. Numerical coordinates increase to the right along the .v axis and
upward along the v axis. We usually use the same scale (that is, the same unit
distance) on the two axes, although in some figures, space considerations make it

convenient to use different scales.

Figure 1 y axis

Rene Descartes

If P is a point in the plane, the coordinates of P are the coordinates x and y of


the points where perpendiculars from P meet the two axes (Figure 2). The .v coordi-
nate is called the abscissa of P. and the y coordinate is called the ordinate of P. The
coordinates of P are traditionally written as an ordered pair (.v, y) enclosed in
Figure 2 parentheses, with the abscissa first and the ordinate second. (Unfortunately, this is

the same symbolism used for an open interval; however, it is always clear from the
P= ix.y)
context what is intended.)
To plot the point P with coordinates {x, y) means to draw Cartesian coordinate
axes and to place a dot representing P at the point with abscissa x and ordinate y.
You can think of the ordered pair (.v, y) as the numerical "address" of P. The
correspondence between P and (x, y) seems so natural that in practice we identify
the point P with its address (x, y) by writing P = (x, y). With this identification in

\ mind, we call an ordered pair of real numbers (.v, y) a point, and we refer to the set
abscissa of all such ordered pairs as the Cartesian plane, or the xy plane.
The .V and y axes divide the plane into four regions called quadrants, denoted Qi,
Qii, Gni. and 2iv and called the first, second, third, and fourth quadrants (Fig-
ure 3). Notice that a point on a coordinate axis belongs to no quadrant.

Figure 3

Oil al
all (.V, y) with
v < 0, >• >

ll(.v. ,1) witli


.•<0,y<
SECTION 1.2 THE CARTESIAN COORDINATE SYSTEM 11

Figure 4
12 CHAPTER 1 FUNCTIONS AND LIMITS

Graphs in the Cartesian Plane


Recall (from prccalculus mathematics) the following definition of the graph of an
equation.

DEFiMTiON I Graph of an Equation

The graph of an equation involving the variables .v and v is the set of all points
(.V, v) in the Cartesian plane, and only those points, whose coordinates x and y
satisfy the equation.

For instance, consider the equation

X- 4- A

We can rewrite this equation as


V.r + v^ = V9
or as VCv - 0)- + (y - 0)- = 3

By the distance formula (Theorem 1 ), the last equation holds if and only if the point
P = {x, y) is 3 units from the origin O = (0, 0). Therefore, the graph of .\r + y^ = 9
Figure 6 is a circle of radius 3 units with its center at the origin (Figure 6).
If we are given a curve in the Cartesian plane, we can ask whether there is an
graph of equation for which it is the graph. Such an equation is called an equation for the
curve, or an equation of the curve. For instance, .v~ + y- = 9 is an equation for the
circle in Figure 6. Two
equations in x and y are said to be e quivale nt if they have
the same graph. For example, .v" + v" = 9 is equivalent to V.x"^ + y" = 3. We often
use an equation for a curve to designate the curve; for instance, if we speak of "the
circle x^ + y^ = 9," we mean "the circle for which .x~ + y" = 9 is an equation."
If r > 0, then the circle of radius r with center (h, k) consists of all points (.v, y)
such that the distance between (.v. y) and (h. k) is /• units (Figure 7). Using the
distance formula (Theorem 1), we can write an equation for this circle as

VU -/?)- + (.V - kf = r

or, equivalently.

'"
(.V -/!)- + O'-^-)" =

This last equation is called the standard form for an equation of a circle of radius r
with center (/i, k).

Figure 7
SECTION 1.2 THE CARTESIAN COORDINATE SYSTEM 13

EXAMPLE 3 Find an equation for a circle of radius 5 with center (-2, 3).

SOLUTION Here r = 5 and {h, k) = (-2, 3), so, in standard form, the equation
of the circle is

[x - (-2)]- + Cv - 3)2 = 52

(x + If + 0' - 3)- = 25

If desired, we can expand the squares,

x^ + Ax + A + y~ -(,y + 9 = 25

and combine the constant terms to obtain an equivalent equation in the form

r- + y^ + Ax- 6v -12 =

The last equation can be restored to standard form by "completing the squares."
The work is arranged as follows;

x^ + v- + 4.V - 6v - 12 =
x^ + Ax + ^'^ - 6^ = 12

x^ + 4;c + 4 + 3;- - 6}' + 9 = 12 + 4+ 9


{x + lf ^- (y- 3)- = 25

Here, we added 4 to both sides of the equation to change x^ + Ax to the perfect


square x" + 4x + 4, and we added 9 to both sides to change y^ — 6}' to the perfect
square v" - 6v + 9. Notice that, in general:

An expression
14 CHAKFER I FUNCTIONS AND LIMITS

EXAMPLE 5 Find an equation in the form x^ + y-^ + Ax + By + C = and an


equation in the standard form of the circle containing the three points (-2, 5),
(1, 4), and (-3, 6).

SOLUTION We substitute the and v coordinates of the three points into the
.v

equation x^ + y'^ + Ax + By + C = and thus obtain the three simultaneous equa-


tions
{-2A + 5B + C = -29

A + 4B + C = -17
-2A + 6fi + C = -45

Solving these simultaneous linear equations in the usual way, we get A = —2,
B = - 18, and C= 57. Thus, an equation of the circle is

x~ + r - 2jc- 18y + 57 =

Completing the squares, we have

.r - It + 1 +r - 18y + 81 = -57 + 1 + 81

(A -lf + (y- 9f = 25

Thus, the circle has radius 5 units and center (1, 9).

Problem Set 1.2

1 Plot each point and indicate which quadrant or coordinate axis 15 (-3, -5) and (-7, 16 (-i -i) and (-3, -I)
contains it.
17 (2, -I) and (5, t) 18 Ui. b + \) dnd (a + \. h)
(a) (1, 6) (b) (-2, 3)

(c) (4. 1) (d) (4, -2) [II In Problems 19 and 20, use a calculator to find the distance be-

tween the two points. Round off your answer to three decimal
(e) (-1, -4) (f) (0, 2)
places.
(g) (-3, 0) (h) (0, -4)
19 (-2.714, 7. Ill) and (3.135, 4.982)
] 2 Plot the four points A = {n. Vl), B = (-V3, V2),
C = (VS, -Vl), and D= (i -f ). 20 (tt, ^)3Lnd (,-Vn, H^)

In Problems 3 to 6, plot the point P and determine the coordinates of In Problems 21 to 24, (a) use the distance formula and the converse
points Q, R, and 5 such that (a) the line segment PQ is perpendicular of the Pythagorean theorem to show that triangle ABC is a right

to the .V axis and is bisected by it: (b) the line segment PR is perpen- triangle, and (b) find the area of U-iangle ABC.
dicular lo the y axis and is bisected by it; (c) the line segment PS
passes through the origin and is bisected by it. 21 A = (1, 1), B = (5, 1), C= (5. 7)

= (-4, -3) 22 A = (-1, -2), B = (3, -2), C= (-1. -7)


3 P = {3. 2) 4 P
A = B = (-3, C =
^ = (-1.
/ V3_ _ n 23 (0, 0), 3), (2, 2)

5 3) '' = \~r-~) 24 A = (-2, -5), B = (9, i), C= (4, ¥


25 Show that the points A = (-2, -3), fl = (3, -1), C = (1,4),
In Problems 7 to 18, find the distance between the two points.
and D= (-4. 2) are the vertices of a square.

7 (7, 10) and (I. 2) 8 (-1, 7) and (2, II) 26 Show that the distance between the points (x\.\\) and (.vj, yi) is
the same as the distance between the point (x\ - .vi, Vi - Vj) and
9 (7, -1) and (7. 3) 10 (-4, 7) and (0, -8
the origin.
11 (-6, 3) and (3, -5) 12 (0, 4) and (-4. 0)
27 If A = (-5. I), fi = (-6, 5), and C= (-2, 4), determine
13 (0, 0) and (-8. -6) 14 (/, 4) and U, 8) whether triangle ABC is isosceles.
THE CARTESIAN COORDINATE SYSTEM 15

28 Check the derivation of the distance formula (Theorem 1 ) for the 38 (x + 3)- + (y - 10)- = 100
case in which one point is in the second quadrant and the other
39 .r + y- + 2r + 4y + 4 =
point is in the third quadrant.

40 .r + y- - Jc - y - 1 =
29 Find all values of t so that the distance between the points
(-2. 3) and (r, t) is 5 units. 41 4x- + 4v= + 8.V - 4y + 1 = (Him: Begin by dividing by 4.)

30 If Pz and P3 are pwints i n the plan e, the n P 2 lies on the line


P) , -
42 3.r= + 3y- - 6.t + 9y = 27
segment PJP^ if and only if iPiP,! = l/'iP^I + {PiPil- Illustrate
43 Axr + 4r + 4.v - 4y + 1 =
this geometric fact with diagrams.

44 4.v2 + 4r + llv + 20y + 25 =


In Problems 31 to 33. determi ne whethe r P; ies o n the line segment
l

P^P, by checking whether \PiP,\ = |f iPjl + iPiPil (see Prob- In Problems 45 to 47, find the equation in the standard form of the
lem 30). circle or circles in the xy plane that satisfy the conditions given.

31 f, = (1. 2). P: = (0. i), P} = (-1. 3) 45 Containing the points (-3, 1). (7. 1), and (-7, 5)

32 P, = (-i. 0). P2 = (-1. 5), P3 = (2. 11) 46 Containing the points (1. 7), (8. 6). and (7. -1)

33 P, = (2. 3). P2 = (3, -3). P3 = (-1. -1) 47 Radius vT7, center on the x axis, and containing the point

(0, I). (There are two circles in this case.)


34 On a Cartesian coordinate grid, an aircraft carrier is detected by
radar at pwint A = (52, 7 1 ) and a submarine is detected by sonar 48 A point P = (.r, y) moves so that it is always twice as far from
at point 5 = (47, 83). If distances are measured in nautical the point (6. 0) as it is from the point (0, 3). (a) Find the equation

miles, how far is the carrier from a point on the surface of the of the cur\'e traced out by the point P. (b) Sketch a graph of this

water directly over the submarine? curve.

49 Salesperson A is allowed to sell encyclopedias anywhere within


In Problems 35 and 36, find the equations in standard form for the + y- = B allowed to
the circle .\r 100, while salesperson is sell
circles in the Cartesian plane that satisfy the given conditions. - =
them within the circle (.v 20)- + y- 144. Do their sales ter-

ritories overlap? Explain.


35 (a) Radius 3 and center (0, 2)

(b) Radius 2 and center (


— 1, 4) 50 Find conditions on the constants a. b, c, d, r, and R so that
the circle (x — a)' + (y — b)- = ir will intersect the circle
(c) Radius 5 and center (3,4)
(.V - f )- + (y - d)- = R-.
36 (a) Center at (1. 6) and containing the point (—2, 2)
51 Show that the equation of the circle in the standard form
(b) Radius 4 and containing the points (-3. 0) and (5, 0) -
(X - h)- + = r can always be rewritten in the
iy k)-
(c) Points (3. 7) and (—3. —1) are the endpoints of a diameter alternative + y- + Ax + By + C = 0. where A = -2h,
form x^
B = ~2k, and C = h- + kr — n. (Hint: Expand the squares and
In Problems 37 to 44. find the radius r and the coordinates ih. k) of collect the terms.)
the center of the circle for each equation, and sketch a graph of the
circle.
52 Show that the equation xr + y^+Ax + By + C = represents
a circle with center at the point (h. k) = {-A/2. -BID and ra-
37 (.r-i- 1)- + (V - 2)- = 9 dius r = iVA- + B- - 4C. provided that A- + B' > AC.

1.3 Straight Lines and Their Slopes


Perhaps the simplest curves in the plane are straight lines (called simply lines) and
circles. In Section 1.2, we derived equations for circles by using the distance for-
mula. In this section, we derive equations for lines by using the idea of slope.
In ordinary language, the word "slope" refers to a steepness, an incline, or a

deviation from the horizontal. For instance, we speak of a ski slope or the slope of

a roof. In mathematics, the word "slope" has a precise meaning.


16 CHAPTER I FUNCTIONS AND LIMITS

Figure 1 Consider the inclined line segment AB in Figure 1. The horizontal distance be-
tween A and B is called the run, and the vertical distance between A and B is called
the rise. The ratio of rise to run is called the slope of the line segment and is

traditionally denoted by the symbol in. Thus, by dcfinitiftn.

slope of AB = in

If the line segment AB is turned so that itbecomes more nearly vertical, then the rise
increases, the run decreases, and the slope m = rise/run becomes very large. When
the line segment becomes vertical, the slope m = rise/run becomes undefined since
the denominator is zero^In this case, we sometimes say that the slope is infinite.
If the line segment AB is horizontal, its rise is zero, so its slope m= rise/run is

zero. If AB slants downward to the right as in Figure 2, its rise is considered


negative; hence, its slope m = rise/run is negative. (The run is always regarded as
being nonnegative.)

Figure 2

negative
rise

Figure 3
SECTION 1.3 STRAIGHT LINES AND THEIR SLOPES 17

Consideration of the similar triangles in Figure 4 shows that two parallel line
segments AB and CD have the same slope. Similarly, if two line segments AB and
CD lie on the same infinite line, as in Figure 5, they have the same slope. The
common slope of all the segments of a line L is called the slope of L.
From two parallel line segments have the same slope, it follows that
the fact that
two parallel lines have the same slope. Conversely, it is easy to see that two distinct
lines having the same slope must be parallel, and we have the following theorem.

Figure 4 Figure 5

THEOREM 2 Parallelism Condition


Figure 6
Two distinct nonvertical lines are parallel if and only if they have the same slope.

Now, consider a nonvertical line L with slope m and containing a point A =


(.V|, \'i) (Figure 6). If P = (.v, v) is any other point on L, then, by Theorem 1,
m= (y - yi)/{x - .Vi); hence,

y ~ .Vi = m{.x - xj)

Note that this equation holds even if P= A, when it simply reduces to = 0. In


fact, we claim that it is an equation of the line L in the sense that not only do all
points (.V, v) on L satisfy this equation, but, conversely, any point (.v, y) that satisfies
this equation lies on line L. (The converse follows from Theorem 2.) The equation
Figure 7
_Vi = mix - .V|)

is called the point-slope form for an equation of L.

EXAMPLE 2 Let L be the line of slope 5 containing the point (3, 4). Write an
equation of form, determine where L intersects the v axis, draw a
Z, in point-slope

diagram showing L and the coordinate axes, and decide whether the point (4, 9)
belongs to L.

SOLUTION The point-slope form for an equation of the line L is y - 4 =


5(.v - This equation can be rewritten as y = 5,v - 1 1 If L intersects the v axis at
3). .

the point (0. b), then b = 5(0) - 1 1 = - 1 1 Since both (0, - 1 1) and (3, 4) belong .

(0.-11) to L, it is easy to draw L by drawing the line through these two points (Figure 7). If
we put .V = 4, y = 9 in the equation for L, we obtain 9 = 5(4) -11, which is true.
Therefore, (4, 9) does belong to L.
18 CHAPTER 1 FUNCTIONS AND LIMITS

Figure 8 Now suppose that L is any nonvertical line with slope w. Since L is not parallel to
the y axis, it must intersect it at some point (0. h) (Figure 8). The ordinate h of this
intersection point is y intercept of L. Since (0. h) belongs to L, we can
called the
write the point-slope equation y - h = m{.x - 0) for L. This equation simplifies to

nL\ + b

which is called the slope-intercept form for an equation of L.

EXAMPLE 3 In 1982 the Solar Electric Company showed a profit of $3.17 per
share, and it expects this figure to increase by $0.24 per share per year. Counting
the years so that 1982 corresponds to .v = and successive years correspond to
.r = 1,2,3, and so forth, find the equation y = nu + b of the line that will enable
the company to predict its profit y per share during future years. Draw a graph
showing this line, and find the predicted profit per share in 1990.

SOLUTION When .v = 0, y = 3.17; hence, 3.17 = m{0) + and so = 3.17. fc, /?

Thus, y + 3.17. When x increases by 1, y increases by 0.24; hence, m =


= ffix

0.24. The equation, therefore, is y = 0.24.V + 3.17. In 1990. x = 8 and y =


(0.24)(8) + 3.17 = 5.09. The predicted profit per share in 1990 is $5.09 (Figure 9).

Figure 9 Figure JO
SECTION 1.3 STRAIGHT LINES AND THEIR SLOPES 19

The equation of any line can be put into the form

Av + B\ + C =

where A. B. and C are constants and not both A and B are zero. This is called the
general form of an equation of a line. If B# 0, the equation Ax + By -'r C= Q can
be rewritten

y
= -X +

and therefore represents a line with slope m = —A/B and y intercept b = —C/B. On
the other hand, if fi = 0, then A ^ and the equation can be rewritten in the form
.V = —C/A. which represents a vertical line.

Perpendicular Lines
In Theorem 2 we have seen that two nonvertical lines are parallel if and only if they
have the same slope. Theorem 3 gives a condition for two nonvertical lines to be

perpendicular.

THEOREM 3 Perpendicularity Condition

Two nonvertical lines are perpendicular if and only if the slope of one of the lines
is the negative of the reciprocal of the slope of the other line.

Figure 12

\
20 CHAPTER I FUNCTIONS AND LIMITS

EXAMPLE 4 If A = (-1, 2) and fi_= (4. -5), find the slope m, of a line that is

perpendicular to the line segment AB.

SOLUTION The slope W: of the line containing the points A and B is given by

= yi - vi
= -5-2 = 7
m-,
JC2-.V, 4-(-l) 5

Therefore, by the perpendicularity condition (Theorem 3),

1 1 5

EXAMPLE 5 Let L be the line 3.v - v — = 0. Find (a) an equation of the line Li
1

that contains the point (-1,2) and is parallel to L; (b) an equation of the line L2 that
contains the point (—1, 2) and is perpendicular to L. Sketch the graphs of these
lines.

SOLUTION

(a) In slope-intercept form, the equation of L is v = 3.v — 1; hence, L has


slope w= 3. Since L| is to be parallel toL, the slope of Li must be/Hi =m = 3.

Because L, contains (—1, 2), its equation is v — 2 = 3[.y — (—1)] in point-

slope form, or y = 3.y -I- 5 in slope-intercept form.

(b) Since Li is to be perpendicular to L, it follows that the slope of Li must


be ott = -1/w = -3. Therefore, because L2 contains (-1, 2), its equation
in point-slope form is y — 2 = ~^[x — (—1)], or, in slope-intercept form,
y = -ix + § (Figure 13).

Figure 13

If two different lines in the plane are not parallel, they will intersect in a single
point. For instance, in Figure 13, the lines Z,| and Li meet in the point (- 1, 2). In

order to find the point at which two nonparallel lines intersect, it is only necessary to
solve the equations of the two lines simultaneously.
- ^^1 SECTION 1.3 STRAIGHT LINES AND THEIR SLOPES 21

Problem Set 1.3

\ In Problems 1 to 6, find the slope of the line that contains the two 26 L has slope m= i and intersects the x axis at (—3, 0).

points.
27 L intersects the x and y axes at (3, 0) and (0, 5).

1 (6. 2) and (3, 7) 2 (3. -2) and (5. -6) 28 L contains the points (5, f) and (§. -6).

3 (14. 7) and (2. 1) 4 (2, 2) and (-4, -1) 29 L contains the point (4, -4) and is parallel to the line

Ix - 5y -1-3 = 0.
5 (-5, 3) and (6. 8) 6 (1. 3) and (-1, -1)
30 L contains the point (-3, §) and is perpendicular to the y axis.
( In Problems 7 to 18. find an equation in point-slope form of the line
31 L contains the point (-3, §) and is perpendicular to the line
L.
5.V + 3y -1=0.
7 L contains the point (5. 4) and has slope m= 2. 32 L is the perpendicular bisector of the line segment AB. where

8 L contains the point and has slope m = -4. A = (3. -2) andB = (7, 6).
(6. 1)

9 L contains the point (3. 2) and has slope m= \. 33 Find a real number B so that the graph of 3x -1-
By — 5 = has y
intercept b = -4.
10 L contains the point (
— 5. —1) and has slope m= 0.

34 Suppose that the line L intersects the axes at (a. 0) and (0, b). If
11 L contains the point (7, —2) and has slope m= -3.
a 7^ and b 9^ 0. show that the equation of L can be written in
12 L contains the point (0. 2) and has slope m= -§. intercept form

13 L contains the point (5. §) and has slope m= 0.


= 1

14 L contains the points (7, 11) and (-1. 1).

15 L contains the points (3, 2) and (4, 8).


In Problems 35 to 38, suppose that m\ and mi are the slopes of the

16 L contains the point (7, 2) and is parallel to the line segment AS. distinct lines Z-i and Lt, respecfively. Indicate whether the lines are

where A = (i 1) and B = (-§. i).


(a) parallel, (b) perpendicular, or (c) neither parallel nor perpendicu-
lar. Then, supposing that Z,, contains the point (3. 2) and Li contains
17 L contains the points (
— 3. 4) and (-4, 4). (-2,
the point 5), draw L; and Z,i on the same diagram.
18 L contains the point (—1. 2) and is perpendicular to the line
segment AB, where A = and B = (-§,
35 mi = § and mi = 5 36 rri] and mi = f
(i, §) 3).

37 m, = f and mi = — 38 m. - 1 and mi = 1

19 Write an equation for each of the following lines, (a) The line

containing the point (-2. 3) and perpendicular to the y axis, A = (-5,


39 If -2), B = (1. -1), C= (4, 4), and D= (-2, 3),
(b) The line containing the point (-2. 3) and perpendicular to show that the quadrilateral ABCD is a parallelogram. (Hint:
the X axis. Show that opposite sides have the same slope.)

20 Show that if .Vi # .vt. then an equation of the line containing the
40 There are two circles of radius VTO that are tangent to the line
two points (.V|. yi) and (.Vi, Vi) is = 6 at the point (3, -3). Find the equations of these
3.r -(-
y
.V2 - -VaVi - Xjyi circles in standard form. (Hint: The tangent line to a circle is
y = .Vi
X
,

H
perpendicular to the radius at the point of tangency.)

41 (a) Determine d so that the line containing A = (rf. 3) and


Xln Problems 21 to 24, rewrite each equation in slope-intercept form, B = (—2, 1) is perpendicular to the line containing C = (5, -2)
find the slope m and the y intercept b. and sketch the graph. and D= (1, 4). (b) Determine k so that the line containing
E= (k, 3) and B = (—2. 1) is parallel to the line containing
21 - 2y = 6 22 5x - -10 =
3;c 2y C= (5, -2) and£) = (1, 4).

23 V -I- 1 = 24 jc = -iy + i 42 Prove that

In Problems 25 to 32. find an equation of the line L in (a) point-slope vi + X2 yi + y:


form, (b) slope-intercept form, and (c) general form.

25 L has slope m = —3 and y intercept b = 5. is the midpoint of the line segment between (.v, .y,) and (X2, yi).
22 CHAPTER 1 FUNCTIONS AND LIMITS

43 Use the midpoint foniiula in Problem 42 to find the midpoint of 47 If a piece of property is depredated linearly over a period of n

the line segment between each pair of points. years, then its value y dollars at the end of x years is given by
y = f[l - (x/n)\. where c dollars is the original value of the
(a) (8. 1) and (7, 3) (b) (9, 3) and (-5. 7)
property. An apartment building built in 1975 and originally

(-1. and (5. 3) (d) (1, -3) and (5, 8)


worth $400,000 is being depreciated linearly over a period of 40
(c) 1)
years. Sketch a graph showing the value y dollars of the apart-
44 If mi 7^ 1112. show that the line v = m^x + b, intersects the line ment building .v years after it was built, and determine its value

y = m2X + bn at the point in the year 1995.

( b2 - bi mi/>2 ~ 'n2bi \ 48 Show that the line Ax + By + C is perpendicular to the line


-Bx + A\ + D = Q.

49 In 1980, tests showed that water in a lake was polluted with 7


45 A car rental company leases automobiles for $22 per day plus
milligrams of mercury compounds per 1000 liters of water.
$0.20 per mile. Write an equation for the cost v dollars in terms
Cleaning up the lake became an immediate priority, and envi-
of the distance x miles driven if the car is leased for A^ days. If
ronmentalists determined that the pollution level would drop at
N= 3, sketch a graph of the equation.
the rate of 0.75 milligram of mercury compounds per 1000 liters
46 Find the distance (measured perpendicularly) between the point of water per year if all their recommendations were followed. If
(-4. 3) and the line y = 3.v - 5 by carrying out the following 1980 corresponds to .v = and successive years correspond to

steps: (a) Find the equation of the line through (-4, 3) which is x = 1,2,3, and so on, find the equation y = mx + b of the line

perpendicular to the line y = 3.v - 5. (b) Find the point (.vi, yO that allows the environmentalists to predict the pollution level y
at which the line obtained in part (a) meets the line y = 3.r - 5. in future years if their recommendations are followed. Sketch
(c) Use the distance formula to find the distance between (-4, 3) the graph of the equation and determine when the lake will be

and Ui, free of mercury pollution according to this graph.


yi).

1.4 Functions
Advances in our scientific understanding of the world often result from the discov-
ery that things depend on one another in definite ways. For instance, the gravita-
tional attraction between two material bodies depends on the distance between
them, and the pitch of a guitar string depends on its tension. The idea that a quantity
)• depends upon another quantity .v is nicely symbolized by the mapping notation

This notation indicates that to each value of .v there corresponds a uniquely deter-
mined value of y; or, as mathematicians say, each value of .v is "mapped onto" a

corresponding value of y. Another name for a mapping is di function.

DEFINITION 1 Function

A function
SECTION 1.4 FUNCTIONS 23

have special keys for some of the more important functions.


Scientific calculators
By number x and touching, for instance, the V.r key, you obtain a vivid
entering a
demonstration of the mapping .v > V.v as the display changes from x to its image

Va- under the square -root function. For instance.

25 1 >5
2 I * V2= 1.414

Programmable calculators and microcomputers have "user-definable" keys that


can be programmed for whatever function x * y may be required. The program \

for the required function is the actual rule whereby y is to be calculated from x. Each
user-definable key is marked with a letter of the alphabet or other symbol, so that,
after the key has been programmed for a particular function, the letter or symbol can
be used as the "name"" of the function.
The use of letters of the alphabet to designate functions is not restricted exclu-
sively to calculating machines. Although any letters of the alphabet can be used to
designate functions, the letters/, g, and /? as well as F. G. and H are most common.
(Letters of the Greek alphabet are also used.) For instance, if we wish to designate

the square-root function x i > Vjc by the letter /, we write

X I > vx or /: x i
> \^r

If/:.r I > V is a function, it is customar>' to write the value of y that corresponds


to.vas/(.v), read "/of.r." In other words, /(.v) is the image of .r under the function/.
For instance, if /:.v i > y/x is the square-root function, then

/(4) = V4 = 2

/(25) = V25 = 5

/(2) = \/2= 1.414

and, in general, for any nonnegative value of x,

fix) = Vx
lff:x 1 > y, then, for ever>' value of .v in the domain of/, we have

y=/(.v)

an equation relating the dependent variable y to the independent variable .x. Con-
versely, when an equation of the form

y = an expression involving x

determines a function /:.r i > y, we say that the function /is defined by, or given
by. the equation. For instance, the equation

y = 3.r2 - 1

defines a function /:j: i > y, so that

y=f{x) = 3x^- 1

or simply

fix) = 3x^-1
When a function/ is defined by an equation, you can determine, by substitution,
the image /(a) corresponding to a particular value x = a.
24 CHAPTER I FUNCTIONS AND LIMITS

EXAMPLE I Let/be the function defined by f(.x) = 5.v- + 3.v. Find the indicated
values.

(a)/(2) (b)/(-2) (c)/(r')

(d) [f(-\)\- (e) /(/ + /!) (f) n-x)


SOLUTION

(a) /(2) = 5(2)^ + 3(2) = 20 + 6 = 26


(b) /(-2) = 5(-2)2 + 3(-2) = 20 - 6 = 14

(c) /(r^) = 5(Pf + 3(P) = 5t^ + 3t^


(d) [/(-1)]2 = [5(-l)2 + 3(-l)]- = (5 - 3)2 = 2- = 4

(e) /(/ + h) = 5(t + hf + 3(/ + /i) = 5r + \Qth + 5h^ + 3/ + 3/i

(f) f(-x) = 5{-xf + 3(-;c) = 5;c2 - 3.v

If a function /:x i » y is defined by an equation, you may assume (unless


otherwise stated) that its domain consists of all values of x for which the equation
makes sense and determines a unique corresponding real number v. Then the range
of the function is automatically determined, since it consists of the set of all values
of V that correspond, by the equation that defines the function, to values of ;c in the
domain.

EXAMPLE 2 Find the domain of the function defined by

(a) f(x) = (b) g(x)


,
= V4-J: (c) h(x) = 3.v - 5
X — \

SOLUTION

(a) The domain of/ is the set of all real numbers except 1. In other words, it

consists of the interval (-«>, 1) together with the interval (1, «=).

(b) The expression V4 - x represents a real number if and only if 4 - jc > 0,


that is. A' ^ 4. Therefore, the domain of g is the interval (-=», 4].

(c) Since the expression 3.v — 5 represents a real number for all real values of
X, the domain of h is the set IR of all real numbers.

Functions that arise in applied mathematics may have restrictions imposed on


their domains by physical or geometric circumstances. For instance, the function
X I > TTjr that expresses the correspondence between the radius .v and the area ttv"

of a circle would have its domain restricted to the interval (0, ^), since a circle must
have a positive radius.
The particular letters used to denote the dependent and independent variables are
of no importance in themselves — the important thing is the rule by which a definite
value of the dependent variable is assigned to each value of the independent vari-
able. In appliedwork, variables other than .v and y are often used because physical
and geometric quantities are designated by conventional symbols. For instance, the
radius of a circle is often designated by r and its area by A. Thus, the function
f.r * I
A that assigns to each positive value of r the corresponding value of A is

given by

A =f(r) = ttP-
SECTION 1.4 FUNCTIONS 25

In dealing with a function/, it is important to distinguish among Xhe function itself

f:x H^ y
which is a rule, the image

fix)

which is a number depending on .v, and the equation

v=/(.v)

which dependent variable y to the independent variable x. Nevertheless,


relates the
people tend to take shortcuts and speak, incorrectly, of "the function /(.v)" or "the
function^ =/(j:)." Similarly, in applied mathematics, people often say that "y is a
function of a." for instance, "current is a function of voltage." Although we avoid
these practices when absolute precision is required, we indulge in them whenever it

seems convenient and harmless.

Figure 1

Figure 2

Figure 3
26 CHAPTER 1 FUNCTIONS AND LIMITS

Figure 4 It is important to realize that noi every curve in the Cartesian plane is the graph of
a function. Indeed, the definition of a function (Definition 1) requires that there be
one and only one value of y corresponding to each value of .v in the domain. Thus,
on the graph of a function, we cannot have two points (.v, Vi) and {x, Vt) with the
same abscissa x and different ordinates V] and y^- Hence, we have the following
test.

Vertical-Line Test

A set of points in the Cartesian plane is the graph of a function if and only if no
vertical straight line intersects the set more than once.

EXAMPLE 4 Which of the curves in Figure 4 is the graph of a function?

SOLUTION By the vertical-line test, the curve in Figure 4a is the graph of a


function, but the curve in Figure 4b is not.

Graph Sketching
A basic graph-sketching procedure is the following.

The Point-Plotting Method

To sketch the graph of y =/(.v), select several values of .v in the domain of/,
calculate the corresponding values of/(.v), plot the resulting points, and connect
the points with a smooth curve. The more points you plot, the more accurate your
sketch will be.

After you have learned some calculus, you'll find that you can sketch accurate
graphs by plotting relatively few points. This is a case of substituting knowledge for
tedious labor.

Figure 5
SECTION 1.4 FUNCTIONS 27

Figure 6

domain of /

(a)

The domain and range of a function are easily found from its graph. Indeed, as
Figure 6 illustrates, the domain of a function is the set of all abscissas of points on
its graph (Figure 6a) and the range of a function is the set of all ordinates of points
on its graph (Figure 6b).

For each function in Examples 6 to 10. determine the domain, sketch the graph, and
determine the range.

EXAMPLE 6 fix) = \x\

SOLUTION The independent variable .v can take on any value, so the domain is

the set !R of all real numbers. For .v > 0, we have/(.v) = x\ hence, the portion of the
graph corresponding to .v > is part of a line of slope 1 starting at the origin and
extending upward into quadrant I. For x < 0, we have f{x) = -.v, so the corre-
sponding portion of the graph is part of a line of slope - The graph
1 . is sketched in
Figure 7 Figure 7. Evidently, /(.v) cannot be negative but can take on any nonnegative value.
Thus, the range of/ is the interval [0, ^). We call/(.>:) = |.v| the absolute-value
function.

x^ - 4
EXAMPLE 7 g(x)

SOLUTION The fraction


jr -4
x-2
is defined for all values of .v except x = 2 (which makes the denominator zero).
Therefore, the domain of g consists of the two intervals (
— ==, 2) and (2, oc). Note
that

x^ -4 = (x + 2)(x - 2)
Figure 8
hence, for jc t^ 2,

x^ - 4 (X + 2)(.v - 2)
= x + 2
x-2
Therefore, provided that x ¥" 2,

gix) =x+2
It follows that the graph of g consists of all points on the line y = x + 2 except for
the point (2, 4), which is excluded (Figure 8). Evidently, the range of ^ is all real
numbers except 4; that is, the range consists of the two intervals ( - -t. 4) and (4, y^).
28 CHAPTER I FUNCTIONS AND LIMITS

Figure 9 El EXAMPLE 8 h(x) = 1 + V4 - A

SOLUTION Since V4 - x is defined only when 4 - .v s 0, that is, when x < 4,

the domain of /i is the interval (-5:. 4|. We select several values of .r in this interval

and calculate the corresponding values of

h(x) = I + V4-X
as in the table in Figure 9. Here, we have used a calculator and rounded off values

of h(x) to two decimal places. (If a calculator isn't available, the table of square
roots in Appendix D may be used.) Using the information in this table and the
point-plotting method, we sketch the graph as in Figure 9. Evidently, the range of /i

is the interval [1, ^).

if ;c<4
EXAMPLE 9 F(.V) =
-Zv-l-9 if x>4
Figure 10 SOLUTION A function such as F that is defined by using different equations in
different intervals is called a piecewise-defined function. Here x can take on any
value, so the domain of f is U. In sketching the graph, we must consider separately
the portions to the right and to the left of the vertical line .v = 4. To the right of

jc = 4, the graph is a portion of the line of slope -2, starting at the point (4, 1 ) and
extending downward to the right. To the left of .v = 4, the graph coincides with

the curve sketched in Figure 9. These parts, taken together, form the graph of F
(Figure 10). Evidently, the range of F is R.

IS EXAMPLE 10 G(.v) =
I
- X
SOLUTION The domain of G consists of the two intervals (-^, 1) and(l, ^). If

X is near 1 but a little smaller than 1, then 1


- .v is small and positive, so G(x) =
I/( 1 - .y) is very large and positive. Similarly, if .v is near 1 but a little larger than 1

then 1
— .V is negative and small in absolute value, so G(x) is negative and very
large in absolute value. Also, if .v is very large in absolute value, then so is 1
- .v,

and it follows that G(x) is very small in absolute value. Plotting a few points and
keeping these facts in mind, we can sketch the graph of G (Figure 11). From this

graph, we can see that the range of G consists of all real numbers except for 0; that

is, it consists of the two intervals (-^c, 0) and (0, ==).

Figure 11
SECTION 1.4 FINCTIONS 29

Problem Set 1.4

In Problems 1 to 14. let 18 Is the graph of the equation .ir + y^ = 9 the graph of a function?
Why or why not?

f(x) = 2x+ \ g(x) = JT - 3.x -4 h(x) = V3.t + 5


Problems 19 to 32. determine the domain, sketch the graph, and
F{x) = 'y\ G{x) = S/x'-A H(x) = |2 - 5.v|
In
determine the range of the given function.
3.V + 7

Find the indicated values. 19 fix) = 4v - 1

1 /(-3) 2 F(i) 3 /i(-i)

4 H(-A) 5 G(VlT) 6 [h(-\)]-

7 5(0) 8/(l/a) 9 «(c + 2)

10 /[U- l)/2] 11 F(a/3) 12 G(Vb)


013 g(4.718) 114 ;i(2.003)

In Problems 15 and 16. find the domain of each function.

15 (a) /(.v) = 1
- 4.\- (b) g(x) = (X + 2)"'

(c) h(x) = Vx (d) F{x) = V5 - 3.V


(e) G(.v) = 7/(5 - dv) (f) A:(a-) = (4 - 5*)'

16 (a) g(.v) = (9 + .v')^-' (b) hlx)


X + \x\

(c) f U) = —
.r^-8
(d) H(x) =
\.v

17 Use the vertical-line test to determine which of the curves in

Figure 12 are graphs of functions.

Figure 12
30 CHAPTER 1 KUNCTIONS AND LIMITS

shrabs at a distance .v kilometers from an industrial city is given shown to be a linear function of h. When h = 0, P =
by p(.r) = 32 - (3.r/50) for 50 < .v s 500. Sketcii a graph of the 1.013 X 10''
newtons per square meter. When h = 1 meter,

function/?, and find /7(50), /?( 100), p(200), p(400), and/7(500). P = 2.003 X 10^ newtons per square meter. Obtain the equation
that defines Z' as a function of h. and use it to find the pressure P
at a depth of h = 100 meters.

042 The period T{l) seconds of a simple pendulum of length / meters


if^mi lt^fitiy.k, ^^-f'f}'. <'**t*'''r^'''
=
swin ging alo ng a small arc is given approximately by r(/)
27rV//9.807. Using a calculator and rounding off your answer
to three decimal places, find 7(0.1), 7(1), 7(1.5), and
7(0.2484).

43 A baseball diamond is a square 90 feet on each side. A player is

running from home plate to first base at a rate of 30 feet per

second. Express the runner's distance .v from second base as a


function of the time / since he left home plate for < < f 3.

44 In Problem 43, suppose the runner continues around the bases at

a constant speed of 30 feet per second. Express his distance i


from second base as a function of r for s s / 12.

1345 A person sketches the graph of/(.r) = 4x* - H.r' + 20.r - 5x


for x > by plotting five points and connecting them with a
smooth curve, as shown in Figure 13. However, the graph is not

correct as shown. Find the error.

Figure 13 y

38 A closed box with a square base x centimeters by x centimeters


has a volume of 100 cubic centimeters. Express the total surface
area A of the exterior of this box as a function of .v. (Assume that
the thickness of the walls is negligible.)

39 An airline chart shows that the temperature T at an altitude of


h = 15,000 feet is 7 = 5° F. At an altitude of h = 20,000 feet,
7 = - 15° F. Supposing that 7" is a linear function of /i, obtain an
equation that defines this function, sketch its graph, and find the
temperature at an altitude h = 30,000 feet.

40 The speed v of a car is measured during a certain time interval

and found to vary according to the equation

_ |60r OS /<
for 5

^"UoO forf>5

where v is measured in meters per minute and the elapsed time t

is measured in seconds. If V is the speed measured in kilometers

per hour and T is the elapsed time measured in minutes, find V


as a function of T.

E41 In physics, the (absolute) pressure P in newtons* per square


meter at a point h meters below the surface of a body of water is
SECTION 1.5 TVPES OF UNCTIONS 31

1.5 Types of Functions


Grouping, or classifying, is a familiar technique in the natural sciences for dealing
with the immense diversity of things in the real world. For instance, in biology,
plantsand animals are divided into various phyla, classes, orders, families, genera,
and species. In much the same way, functions can be grouped, or classified, by
singling out important features possessed by some functions but not by others. In
this section we describe certain types, or classes, of functions that are considered in
calculus. Among these are even functions, odd functions, polynomial functions,
rational functions, algebraic functions, and transcendental functions.

Even and Odd Functions


Consider the graphs in Figure 1 . The graph of/ (Figure la) is symmetric about the
y axis; that is. the portion of the graph to the right of the y axis is the mirror image of
the portion to the left of it. Specifically, if the point (.v, y) belongs to the graph of
/. then so does the point-.v, y ). In other words, /( -x) = f(x). Similarly, the graph
(

of ^ (Figure lb) symmetric about the origin because if the point (.v, y) belongs to
is

the graph, then so does the point (— .v, — y); that is, g(—x) = —g{x).

Figure I
32 CHAPTKR 1 Fl'NCTIONS AND LIMITS

Figure 2 EXAMPLE 1 Determine whether each of the functions whose graphs are shown in
Figure 2 is even, odd, or neither.

SOLUTION In Figure 2a. the graph of/ is symmetric about the origin; thus.
/(— -v) = —/(.v) and/ is an odd function. In Figure 2b, the graph of ^ is symmetric
neither about the y axis nor about the origin, so ^ is neither even nor odd. In
Figure 2c, the graph of h is symmetric about the y axis; thus, h(-x) = h(x) and h
is an even function.

EXAMPLE 2 Determine whether each function is even. odd. or neither.

(a) f(x) = .V* (b) g(x) = x- \

(c) h(x) = Ir - 3|jc| (d) F(x) = X-'

SOLUTION

(a) /(-.v)= (-x)* = x^ =/(.r), so /is an even function.


(b) g(-.v) = -A - 1, while g(x) = x - \ and -g{x) = -.v + 1. Since we
have neither g(-x) = g(x) nor g(-.v) = — gU), g is neither even nor odd.

(c) h(—x) = 2(— .v)- — 3|— = 2ir — 3|.r| = h(x), so is an even function.
.r| /i

(d) F(—x) = (—xf = — .r^ = — F(.v). so F is an odd function.

There are many functions that are neither even nor odd; however, if a function is

found to be either even or odd. the job of sketching its graph becomes easier
because of the symmetry involved.

EXAMPLE 3 Sketch the graph of F(x) = x^.

SOLUTION The domain of F is the set U. Because

F(-.v) = (-v)- = .r' = fU)


the function F is even and its graph is symmetric about the y axis. We already
sketched the portion of this graph for x > in Section 1 .4 (Figure 5, page 26). The
full graph includes the mirror image of this portion on the other side of the y axis
and the point (0, 0) (Figure 3).

Figure 3
SECTION 1.5 TYPES OF FUNCTIONS 33

Figure 4
34 CHAPTER 1 FUNCTIONS AND LIMITS

Figure 6

a>0
SECTION 1.5 TYPES OF FUNCTIONS 35

Illuminated fountain, St. Louis, Missouri,


showing parabolic jets of water

Algebra of Functions
Sometimes functions are classified by the way in whicii they are formed from
simpler functions. For instance, new functions can be formed from old ones by
addition, subtraction, multiplication, or division. Thus, if f{x) = x- — 2 and
g(x) = -iv + 1, we can form the new function h{x} = f{x) + g(x) = x^ - ix - 1

simply by adding /(.v) and g{x). Naturally, we refer to the function h as the sum of
the functions / and g and write h =f + g (Figure 8). Notice that the graph of h is

obtained from the graphs of/ and g by adding corresponding ordinates; for instance,
h(-2)=f(-2) + g(-2).
It should be clear that any two functions with intersecting domains can be added
as shown.* In a similar way, subtraction, multiplication, and division of functions
can be accomplished. The following definition shows exactly how this is done.

Figure

;i(.v) = /(.v) + g(.v)

*This idea is crucial in applied mathematics, since the functions that describe natural phe-
nomena (for example, light waves or sound waves) often add when the phenomena are

combined.
36 CHAPIER 1 FUNCTIONS AND LIMITS

DEFINITION 2 Sum, Difference, Product, and Quotient of Functions

Let/
SECTION 1.5 TYPES OF FUNCTIONS 37

Figure 9

X
38 CHAFTER 1 FUNCTIONS AND LIMITS

Discontinuous Functions
Many functions considered in elementary calculus have graphs that are "con-
nected" in the sense that they consist of one continuous piece. Such functions,
which are said to be continuous, are discussed in detail in Section 1.9. In order to
fully understand and appreciate the nature of continuous functions, it is sometimes
useful to examine specific functions that are not continuous. One of the more inter-
esting discontinuous functions is the greatest-integer function . which, like the abso-
lute-value function, has its own special symbol.

DEFINITION 5 Greatest-Integer Function

If .V is a real number, the symbol Uxl] denotes the greatest integer not exceeding .v;

that is, II-vl] is the integer that is nearest to .v but is less than or equal to .v. The
greatest-integer function is the function / defined by/(.v) = H-vJ.

Note that Qa]] is the unique integer satisfying the condition

PB x< + 1

For instance, p. 71) = 3, IM =


= 3. [[-2.7]] = -3,
0, 113.234334]]
[1-2.343341] = -3, 1]-^ = -1, [lV3]l = I, Q21] = 2, and [1-2D = -2. A
table of values of Q.v]] for -3 ^ .v < 4 follows, and the corresponding graph of
f(x) = [].v]] is shown in Figure 10:

-3 for -3 <.x< -2
-2 for -2<.v< -1
-1 for - <X< 1

for < < .V 1

1 for 1 < < .t 2


2 for 2 < < a: 3
3 for 3 <X< 4

In Figure 10, we use small dots to emphasize that the left-hand endpoints of the
horizontal line segments belong to the graph, and we use small open circles to
indicate that the right-hand endpoints do not belong to the graph. The discontinuous
nature of the greatest-integer function is apparent from its graph.

Figure 10
SECTION 1.5 TYPES OF FUNCTIONS 39

Problem Set 1.5

1 For each function in Figure 1 1 . find the domain and range and
determine whether the function is even, odd, or neither.

Figure II
40 CHAPTER I FUNCTIONS AND LIMITS

2 Is there any function that is both even and odd? If not. why not? 30 Sketch the graph of fix) = Ir + k for (a) * = 0, (b) t = 1 , and
If so. give an example. (c) A- = -I.

In Problems 31 to 36, rewrite each function /in the form /(a) =


In Problems 3 to II. decide whether the function is even, odd, or
aix - h)' + k by completing the square.
neither.

31 fix) = AT + 2r - 4 32 fix) =.r - 12x + 5


3 f(x) = jr* + 3 4 g(x) = -X* + Ijr + \

33 fix) = 3.r - lOx - 2 34 fix) = 2xr + 3x - \


S f(x) =x* + x 6 g(t) = i' + \t\

35 fix) = -2x^ + 6.r + 3 36 /(.r) = ir^ - 6x - 7


1 F(x) = 5.r' + 7.r 8 /(/) = -t^ + lt

V8?T7 VTTT In Problems 37 to 40, sketch the graph of each function.


9 h(x) = 10 Av) =
\y\

37 fix) = 2(;( -1)^ + 3 38 /i(.v) = -iix +1)^-4


l./(.v) = 4--^' = -2U + 2)2-3 = Hx+ \)^-i
r+ 1
39 gix) 40 pix)

12 (a) Show that the function f U) = Vjlj is even, (b) Using the 41 Let fix) = -3.ir - lit - 1. (a) By completing the square,
rewrite/ in the form fix) = aix - h)- + k. (b) Sketch the
symmetry implied by the fact that F is even, and Figure 9,
sketch the graph of F. graph of/

42 Prove Theorem 1 . [Hint: Begin by rewriting

In Problems 13 to 20, decide whether the function is a polynomial =


fix) ax' + bx + c
function. If it is a polynomial function, indicate the degree (if any)
and identify the coefficients.
fix ) = a(x^ + ^x )
+
13 fix) = 6a- - 3.t - 8 14 f(x) = .t"' + 2x
Then complete the square.]
15 gix) = (X - 3)U - 2) - .x^ 16 /(.v) = 2-
43 In each case, evaluate if+g)ix), if-g)ix), if-g)ix), and
17 F(x) = VZt" - 5" V + 20 18 /(.«) = 210x"' - 1 l.r - 40 if/g)ix).

19 g(x) = (a) fix) = 2v - 5 and gix) = jr + I

(b) fix) = Vx and gix) = x^ + 4


20 h{x) = ^x-' - 6.r + 12v - 8
(c) fix) = 3a: + 5 and gix) = 1 - 4x
21 Explain in your own words the distinction between the constant (d) fix) = Vjc+ 3 and gix) = \/x
function / defined by the equation f(x) = 2 and the real num- = = -
(e) fix) |jc| and gix) \x 2\
ber 2.
if) fix) = ax + band gix) = ex + d
22 Is the function fix) = .v~' + [(.v - l)/.v] a constant function?
44 Suppose that /and
g are even functions. Show that/ + g,f - g,
f-g, ai\df/g are also even functions.
In Problems 23 to 26, determine a linear function/that satisfies the

condition. 45 Specify whether the algebraic function is a rational function.

3.V .V + 1
(a) fix) = .

gix)
(b) " =
23 /(2) = 5 and/(-3) = 7 24 /(2v + 3) = 2/(.v) + 3 .V - 1 S/l'C + 5

25 f{5x ) = 5/U 26 fix + 1) = fix) + /(7)


(c) fix) = AT + It + 1 (d) /(/) =
2r' + 5
27 A number r is called a zero of a function/ if /(r) = 0. Prove that
6f2
every nonconstant linear function has a zero. (e) /(/) = -T}
't+ 1

28 Let / be a function with domain IR. Prove that


46 Show that

/(fc + (l - t)d) = tfic) + il -t)fid)


fix)
holds for every three numbers c, d. and I if and only if/ is linear. I -.r I +JC

29 Sketch the graph of fix) = or for (a) a = 3, (b) a = -3, is a rational function by rewriting it as a ratio of polynomial
(c) a = i. and (d) a = -J. functions. What is the domain of/?
SECTION 1.6 TRIGONOMETRIC FUNCTIONS 41

In Problems 47 to 53, sketch the graph of the function and specify its (a) Find sgn (-2), sgn (-3), sgn 0, sgn 2, sgn 3, and sgn 151.
domain and its range. (b) Prove that |j:|
=x sgn .v is true for all values of x. (c) Prove
that sgn iab) = (sgn a)(sgn b) is true for all values of a and b.

47 H{x) = \x+ l\ 48 h(x) = -3\x\ +X (d) Sketch the graph of the signum function, (e) Find the domain
and the range of the signum function, (f) Sketch the graph of the
49 f(.x) = II3.VE 50 h(x) = \[x]\+ X
function / defined by/(.v) = sgn (x - 1). (g) Explain why the

51 fix) = dixl] 52 G(x) = IIWI) sgn function is discontinuous.

53 g(x) = lE.TllI
[356 In economics, the profit function P from the sale of goods is
related to the revenue function R from the sale and the cost C of
54 Let/ be any function with domain R. (a) Define a function g by producing the goods by the equation P = R - C. Suppose that

the equation g(x) = {f(x) + f(—x)]/2. Prove that g is even, the revenue function R (in dollars) and the cost function C (in

(b) Define a function h by the equation h{x) = [/(x) — f(—x)]/2. dollars) for a product are given by R(x) = 25.x + (.x-^/250)

Prove that /; is odd. (c) Prove that/(.r) = g{x) + htx) holds for and C(x) = 100 + 3x + (.x^/30), where x is the number of units
all Jr. Thus, conclude that any function with domain U is the sum of the- product manufactured and sold. Find the equation that

of an even function and an odd function, (d) Suppose that G is an describes the profit function P; also find f (350), ^(375), and
even function with domain R, that is an odd function with H /'(400). Sketch the graph of P, and find the smallest and largest

domain IR, = G{x) + H{x) holds for all x. Prove


and that/(.ic) values of the production level .x for which the profit is positive.

that G{x) = = h(x) for all values of x.


gtx) and that H(x) Also find the production level for which the profit is maximum.
(e) Show that/is even if and only if/(.x) = g(x) holds for all x.
057 The sum of all expenses, such as insurance, rent, and salaries,
(0 Show that /is odd if and only iff(x) = h(x) holds for all .v. incurred by a manufacturer even when no items are produced is

55 The signum function (abbreviated sgn) is defined by known as the fixed cost F. The part of the total cost of production
C that varies with the level of production x is called the variable
flvl
cost V. (a) Write an equation for C in terms of F and V. (b) If
sgn A- X F= 500 dollars and V (in dollars) is given by V(.x) = .x^ + 4.x,

if .X = find a formula for C(x) and sketch the graph of C.

1.6 Trigonometric Functions


Figure 1 This section is intended as a concise review of the basic trigonometry that you will
need in the rest of the book. To facilitate your review, much of the material is
arc length = f
presented in somewhat condensed, "outline" form.

Radian Measure of Angles


Although the degree measure of angles is used in most elementary applications of
trigonometry, more advanced applications (especially those that involve calculus)
require radian measure. One radian is the measure of an angle that has its vertex at

Figure 2 the center of a circle (that is, a central angle) and intercepts an arc on the circle
equal in length to the radius r (Figure 1).

A central angle of 2 radians in a circle of radius /• intercepts an arc of length 2r on


the circle, a central angle of | radian intercepts an arc of length |r, and so forth.
More generally, if a central angle AOB of 6 radians (0 is the Greek letter theta)

intercepts an arc AB of length 5 on a circle of radius r (Figure 2), then we have

s = rB
42 CHAPTER I FUNCTIONS AND LIMITS

It follows that the radian measure 6 of angle AOB is given by the foimula

Figure 3
For instance, if a central angle in a circle of radius r = 27 meters intercepts an arc of
length .9 = 9 meters, then the measure of the angle in radians is given by 6 = sjr =
^= 3 radian.

A central angle of 360° corresponds to 1 revolution; hence it intercepts an arc

5 = 2vr equal to the entire circumference of the circle (Figure 3). Therefore, if 8 is

the radian measure of the 360° angle.

27rr
Itt radians

that is. 360° = 2n radians, or

1
80° = 77 radians

You can use this relationship to convert degrees to radians and vice versa. In partic-
ular,

180 \°
1° = radian and 1 radian =
180

so we have the following rule:

Multiply degrees by 77/180° to convert to radians, and multiply radians by


180°/ 77 to convert to degrees.

For instance,

60° =
180°
(60°) radians = — radians
3

137T 180° / 1377


and radians = I
234°
10 77 ^ 10

Note that when radian measures are expressed as rational multiples of n. we may
leave them in that form rather than write them as decimals. Also, when angles are
measured in radians, the word "radian" is often omitted. For instance, rather than
writing 60° = 77-/3 radians, we =
Therefore, when no unit of angular
write 60° 7r/3.

measure is indicated, always understood that radian measure is intended.


it is

Table 1 gives the corresponding degree and radian measures of certain special
angles.

Table I

Degree measure
SECTION 1.6 TRIGONOMETRIC FUNCTIONS 43

Figure 4 A sector OPQ of acircle is th e region inside the circle bounded by the arc PQ and
the radial segments OP and OQ (Figure 4). If the central angle POQ has a measure
of radians and r is the radius of the circle, then the area A of the sector is given by
the formula

er

Indeed, the area of the circle is in^. and the sector occupies a fraction dj(lTr) of this
area, so

e Or-
A =
,
777- =
Itt 2

The Trigonometric Functions


Figure 5 In order to define the six trigonometric functions, we begin with any real number t

and measure out an angle of |r| radians, starting at the positive x axis and turning
counterclockwise about the origin if r > and clockwise if r < (Figure 5). Then
we choose any positive number r and draw the circle .ir + y- = r^ of radius r with
center at the origin. Let (.v, y) be the point at which the terminal side of the angle
meets the circle. Now the values at t of the six trigonometric functions are de-
fined by

sin f = —
r
44 CHAPTER I FUNCTIONS AND LIMITS

By using a little geometry, you can find the exact values of the trigonometric
functions at the special values of / shown in Table 2. (Dashes in the table indicate

that the function is undefined at the corresponding value of /.)

Table 2

Value of 1
SECTION 1.6 TRIGONOMETRIC FUNCTIONS 45

Figure 6

indefinitely. It is precisely the wavelike nature of the sine and cosine functions that
makes thein so useful in applied mathematics. Indeed, many natural phenomena,
from electromagnetic waves to the ebb and flow of the tides, are periodic, and so
these functions are indispensable in the construction of mathematical descriptions or
models for such phenomena.
46 CHAPTER I FUNCTIONS AND LIMITS

Trigonometric Identities
The six trigonometric functions satisfy certain identities that are routinely derived in

precalcuius mathematics courses. For your convenience, these standard identities


are listed on the inside front cover of this textbook.

In Examples 2 and 3, use the standard trigonometric identities to simplify the given
expression.

EXAMPLE 2 (sin X + cos .v)- - sin 2x

SOLUTION We use the Pythagorean identity

sin- .V + COS" .V = 1

and the double-angle identity

sin Iv = 2 sin .X cos .v

Thus,

(sin .V + cos ,v)- — sin Zv = sin- .v + 2 sin x cos x + cos" x — sin 2v


= sin- .V + COS" .V + 2 sin x cos x — sin Zx
= 1 + sin 2v - sin It
= 1

EXAMPLE 3 sin (.V + 77)

SOLUTION We use the addition formula

sin (x + y) = sin x cos v -I- sin y cos .v

with y = T. Recall that


cos TT 1 and sin 7J- =
Thus, sin (.V -I- tt) = sin x cos tt + sin ir cos x
= (sin.v)(-l) -I- (OKcosA-)
= —sin X

Right-Triangle Trigonometry
If we denote one of the two acute angles in a right triangle by 6. we can abbreviate
the lengths of the side opposite 6. the hypotenuse, and the side adjacent to 6 as opp.
hyp, and adj, respectively (Figure 7). Then

Figure 8
sin e = -^^
hyp

cos e = —adj
hyp

tan e = -^ opp
adj
SECTION 1.6 TRIGONOMETRIC FL'NCTIONS 47

Figure 9 EXAMPLE 4 Find the values of the six trigonometric functions of the acute angle i

in Figure 9.

SOLUTION For angle 0. we have


adj = 5 and hyp = 13

but opp isn't given. However, by the Pythagorean theorem,

adj- + opp- = hyp-


Figure 10
so that

opp- = hyp- - adj- = 13- - 5" = 144

Figure 11

Figure 12
48 CHAPTKR I FUNCTIONS AND LIMITS

Figure 13
SECTION 1.5 TRIGONOMETRIC FUNCTIONS 49

Figure 16
(see Figure 4). In right triangle OQS. we have

tan, = ^ MMadj
=
\0Q\
=
1

so that

\QS\ = tan ;

Therefore, the areaA^ of right triangle OQS (one-half height times base) is given by

AT=i\QS\\OQ\ =|(tan/)(l) = itanr


Evidently, the area Ac of the circular sector POQ is smaller than the area Ar of
triangle OQS\ that is,

Ac<At
or

I? < 5 tan t

Therefore,

r < tan /

It follows that

r cos r < sin ;

that is,

The inequality

< 1

follows from Theorem Combining


1. the inequalities we have obtained, we have
sm t
< cos / < < 1

as claimed in Theorem 2.

THEOREM 3 Comparison of 1 - cos t and r^/2

If < \r\ < 77-, then 1


1 - cos /I <—
'
1

PROOF From the half-angle formula

. . t 1

Sin-Y = y(l - COS t)

(see inside front cover), we have

|l - cos t\ = 2 sin--
50 CHAPTER 1 FUNCTIONS AND LIMITS

If < |/| < 77. then < |//2| < 7t/2; hence, by the remark following Theorem I

sin —
t \

<
\

—t

I 2 1 I 2

Therefore,

|sin-y| = |smy| <\-\ =-


and it follows that

— = ^ ' 1 .( '^\ ^
1 cos t\ 2 sin

Problem Set 1,6

In Problems 1 to 6, s denotes the length of the arc intercepted on a

circle of radius r by a central angle of radians. Find the missing

quantity.

1 r = 2 meters, S = 1 .65 radians, s = ?

2 r = 1.8 centimeters, 0=8 radians, s = ?

3 r = 9 feet, s= 12 feet. 0= ?

4 i = 477 kilometers, 6 = tt/1 radians, r = ?

5 r = 12 inches, S = Stt/IS radian. s = l

6 r = 5 meters, s = 1377 meters, 9 = ?

1 Convert each degree measure to radians. Do not use a calculator.


Write your answers as rational multiples of 77.

(a) 30° (b) 45° (c) 90° (d) 120°

(e)-150° (f) 520° (g) 72° (h) 67.5°

(i) -330° (j) 450° (k) 21° (1) -360°

51 8 each degree measure to an approxi-


U.se a calculator to convert

mate radian measure expressed as a decimal. Round off all an-


swers to four decimal places.

(a) 7°
SECTION 1.6 TRIGONOMETRIC FUNCTIONS 51

25 Use the standard identities (inside front cover) to simplify each


expression.

(a) (1 — cos /)(! + cos I) (b) 2 sin t cos r esc t

(c) sec" I (csc" / - l)(sin I + \) - esc i

1 + cot- /
(d) (e)
sec t sec / — 1

38
26 Show that

(a) cos
(f
(b) sin (?-') =

(c) tan (f + 77) = tan / (d) tan I f + -cot I

27 (a) Use the fact that 45° + 30° = 75° to find the exact values of
sin 75° and cos 75°. (b) Find the exact values of tan 75°.
39 If < < n/2 and x = 2 sin 6. find sec 6 in terms of .r.

cot 75°, sec 75°, and esc 75°. =


40 If < < 77/2 and .t 3 tan 0, find esc 6 in terms of x.

28 Using the standard trigonometric identities, prove that 41 < 8 < and r = 5 sec 0. find cos in terms of
If 77/2 z.
cos 3r = 4 cos^ / - 3 cos t. Then use this to show that

cos (7r/9) is a solution of the equation 8.v^ — 6ic — 1 = 0. 42 If < e < 77/2 and 3m = 2 tan S, find sin 6 in terms of u.

29 Simplify, 43 If Z, is a line with inclination and slope m, show that m= Ian 6

sin" 2t cos f - sin t in the cases where 6 = and where 7r/2 < < tt.

(a) .. ,, .... + 1 (b)'


(1 +
.

cos 2t)- '


"
sin 2r 44 An irrigation ditch has a cross section in the shape of an isosce-
(c) cos^ 2r - sin- t (d) tan f - esc f (1 2 cos- r) sec t les trapezoid that is wider at the top than at the bottom (Fig-

(e) cos {s - t) cos t — sin {s - t) sin t


ure 17). The bottom and equal sides of the trapezoid are each 2
meters long. If 6 is the acute angle between the horizontal and
30 Suppose that sin / = if and cos s = - i, where tt/2 < t <tt the side of the ditch, show that the cross-sectional area A of the
and IT12 < s <Tr. Find the exact values of ditch is given by A = 4 sin fl (1 -I- cos 6).

(a) sin (s - t) (b) cos (s + t) (c) cot (.s


- t)

In Problems 31 to 38, find the values of the six trigonometric func- Figure 17
tions of the acute angle 6.

31

]45 From a point on level ground 75 meters from the base of a


television transmitting tower, the angle of elevation of the top of
the tower is 68. 17°. Find the height h of the tower (Figure 18).

33 34

Figure 18

35 36

2v^
52 CHAPTER 1 FUNCTIONS AND LIMITS

E146 A high-altitude military reconnaissance jet photographs a missile panel and the ground be equal to the latitude of its location.

silo under construction near a small town. The jet is at an alti- Assuming that this recommendation has been followed, find the

tude of 57. 000 feet, and the angles of depression of the town and surface area of the panel. (Round off your answer to three deci-

silo are 60° and 30°. respectively (Figure 19). Assuming that the mal places.)
jet, the silo, and the town lie in the .same vertical plane, find the
048 Biologists studying the migration of birds are following a mi-
distance between the town and the silo.
grating flock in a light plane. The birds are flying at a constant
altitude of 1200 feet, and the plane is following at a constant
Figure 19 altitude of 1700 feet. The biologists must maintain a distance of
at least 600 feet between the plane and the flock to avoid disturb-
liorizontal line
ing the birds; therefore, they must monitor the angle of depres-
sion of the flock from the plane. Find the maximum allowable
angle of depression, rounded off to the nearest degree.

49 (a) Use the half-angle formulas


57,000 fcot

= —-( 1
- cos .v) and cos" —=— (1 -I- cos x)

to help show that

y<cos-y for < <— .r


2

(b) Using the result of part (a), show that < sin f < cos ; holds
E147 A rectangular panel to collect solar energy rests on flat ground
for < < f 7r/4. (Hint: Let x = It. ) (c) Use the result of part (b)
and toward the sun. The edge resting on the ground is
is tilted
to show that < tan / < 1 holds for < <
f 7r/4.
3.217 meters long, and the upper edge is 1.574 meters above the
ground. The panel is located near Chicago, and its latitude is 50 For small values of t, sin t = tan ;. Prove this by establishing
41.8°. Solar engineers recommend that the angle between the the inequality |sin ; - tan /| < r/2 for < |f| < Tr/4.

1.7 Limits of Functions

Figure I
SECTION 1.7 LIMITS OF FUNCTIONS 53

More generally, if/ is a function and a is a number, we understand the notation

lim f(x) = L

which is read "the limit oi f{x) as .v approaches a is L," to mean comes


that/(jr)

closer and closer to the number L as x comes closer and closer to the number a.
Although we give a more formal definition at the end of this section, you can
acquire a working understanding of limits by considering further examples and
geometric illustrations.

EXAMPLE 2 Determine lim (5x + 7).

SOLUTION As X comes closer and closer to 4, 5x comes closer and closer to 20,

and 5x + 1 comes closer and closer to 27. Therefore, lim {5x + 7) = 27.

Unfortunately, it isn't always possible to determine the limit of a function by


simple arithmetic considerations as in the preceding example. In the first place, the
function values may jump around so erratically that they never settle down and
approach a limit, inwhich case we say that the limit does not exist. (Later we give
some examples in which limits do not exist; however, in the present section, our
examples are chosen so that all required limits exist.) In the second place, the
function may be so complicated that the limit, even though it exists, is not evident

by superficial inspection.
For instance, let

-ir - Ax
/(.v)

and consider the problem of determining lim f{x). Here, it is not immediately clear

just how/(jf) behaves as .v approaches 2; however, we can gain some insight into
this behavior by using a calculator to find some values of /(x) as x gets closer and
closer to 2 but stays less than 2. These values are shown in the following table:
54 CHAPTER 1 FlINCTIONS AND LIMITS

This guess can be verified by some elementary algebra, as in the following


example.

3a- - 4.V - 4
EXAMPLE 3 Determine lim
.V - 2

SOLUTION Let

3.x- - 4.V - 4
fix)

Then the domain of/consists of all real numbers .v except for .v = 2 (which makes
the denominator zero). We are only concerned with the values of /(.r) as .v ap-
proaches 2 — what happens when .v reaches 2 is not in question here. For j: # 2,

3.V- - 4.V - 4 (3.V + 2)(.v - 2)


fix)
= = = 3.V + 2
.V - 2 .V — 2

Therefore, as .v comes closer and closer to 2,/(.v) comes closer and closer to 8; that
is,

3.x- - 4.V - 4
lim f(x) = lim = lim (3.v + 2) = 8
x->2-' .x-*2 x-2 x^2

The limit found in the preceding example can be illustrated geometrically by


sketching a graph of the function

3.x- - 4.V - 4
f(x) =
.V - 2
Figure 2
(Figure 2). Since /(.v) = 3.v + 2 holds for x ¥^ 2, this graph is a line with the point
(2, 8) excluded. Figure 2 makes it clear that the value of/(.v) can be made to come
as close to 8 as we please simply by choosing x to be sufficiently close to 2 (but not
x*2 equal to 2).
We see from Figure 2 that:

In finding the limit of/(j:) as a' approaches a, it does not matter how/ is defined
at a (or even whether it is defined there at all). The only thing that does matter is

how / is defined for values of .v near a.

Sometimes, you can use special algebraic tricks to simplify an expression so that

its limit becomes apparent.

V^-1
IS EXAMPLE 4 Find lim and check your result numerically with a cal
V — I
.V - 1

culator.

SOLUTION The desired limit, if it exists at all. is certainly not clear as things
stand. But, by multiplying both the numerator and the denominator of the fraction
by Vx + 1, we "rationalize the numerator" and obtain

Vx - 1 (Va - 1)(Va+ 1)

(.v-1)(Va+1) (.v- 1)(Va + 1) Va+1


SECTION 1.7 LLVllTS OF FUNCTIONS 55

Now, as .r comes closer and closer to 1 , Vjc comes closer and closer to 1 , V? + 1

comes closer and closer to 2, and \/(\G. + 1) comes closer and closer to s. There-
fore,

V^- 1
=
1
lim lim
..-1 X - 1 ^-1 V^ + 1 2

Using a calculator, we find several values of (\^v — \)/(x — 1) for values of a: less

than 1. but approaching 1. Thus.

X
56 CHAPTER I FUNCTIONS AND LIMITS

Consider, for instance, the problem of finding

sin X
lim
JT—O X

It's difficult to think of any algebraic trick that will simplify (sin .v)/.v so that its limit

as jc -* will become apparent. Using a calculator (in radian mode), you can verify

the entries in the following table:

v
SECTION 1.7 LIMITS OF FUNCTIONS 57

Geometrically, lim f(x) = L means that, for a; ¥= a, we can guarantee that /(a:) is in

any given small open interval around L if we make sure that jr is in a suitably small
open interval around a.
Now, we summarize these considerations in a formal definition of limit.

DEFINITION 1 Limit

Let/be a function defined on some open interval containing a, except possibly at


the number a itself. The statement lim /(.v) = L means that, for each positive

number e, there exists a positive number 5 such that |/(.v) - L\ < e holds when-
ever < l.v
— n| < 6.

EXAMPLE 5 Given e = 0.03, determine a positive 8 so that |(3.v + 7) - l| < e


whenever < |jc
— — 2)| < 5.
(

SOLUTION We have

|(3.x-l-7)- 1| = |3.r-l-6| = |3(.v + 2)1 = 3|x + 2| and |a:


- (-2)| = |a: + 2|

hence, we must find a positive 8 such that

3|.v + 2| < 0.03 holds whenever < |.v + 2| < 5

The condition 3|.v + 2| < 0.03 is equivalent to |.v + 2| < 0.03/3 = 0.01; hence, we
must determine a positive 8 such that

|.v + 2| < 0.01 holds whenever < |.v + 2| < 5

Obviously, 8 = 0.01 works. So does any smaller positive value of 5.

EXAMPLE 6 Use Definition 1 to prove that lim (3.v + 7) = 1.

SOLUTION Let e > 0. We must find 6 > such that |(3x + 7) - l| < e holds
whenever < |.v - (-2)| < 6, that is, whenever < |jr -I- 2| < 5. Just as in Exam-
ple 5, we have
|(3.v + 7) - 1|
= 3|jf + 2|

Therefore, the condition |(3.v + 7) - l| < e is equivalent to 3|.v + 2| < e, that is,

|x + 2| < e/3. Hence, we must determine a positive 8 such that

\x -I- 2 < holds whenever < Lv + 2 < 5

Obviously, 8 = e/3 works. So does any smaller positive value of 5. This proves
that

lim^ (3.V -^ 7) = 1

In the seventeenth and eighteenth centuries, the creators of calculus had to man-
age with something like the intuitive idea of limit that we first introduced — and they
had no electronic calculators with which they could quickly check their results. In
finding limits, they had to depend on geometric and numerical insights that were
developed and sharpened by laborious calculations. It wasn't until 1821 that the
great French mathematician Augustin Louis Cauchy (1789-1857) proposed a more
precise definition of limit. Definition 1 is a somewhat refined and perfected version
Augustin Louis Cauchy of Cauchy 's idea of the meaning of limit.
58 CHAPTER 1 FUNCTIONS AND LIMITS

Problem Set 1.7

In Problems 1 to 6, determine the limit and sketch a graph of each X


function to illustrate the limit involved.

1 lim ix 2 lim (3.r - 6)

3 lim^ (2 - 3x) 4 lim —2


x^i X

r -9 6 lim -2x\
5 lim |1
3 .V - 3

In Problems 7 to 14, use elementary algebra to simplify each expres-


sion so that its limit becomes apparent. Then IS check your result

numerically by using a calculator to fill in the blanks in the accom-


panying table.

*^ - 5.t -I- 6
7 lim
-2 x-2
SECTION 1.7 LIMITS OF FUNCTIONS 59

14 lim [Hint: Multiply numerator and denominator by


>r->i X-—
X 1

( Vx)- + V(v + 1

X
60 CHAPIER 1 FUNCTIONS AND LIMITS

1.8 Properties of Limits of Functions

Although graphs, "hunches," and numerical evidence generated by a calculator


can provide means for guessing the value of a limit, such a value must be viewed
with suspicion until it is confirmed. The formal e, 8 definition (Definition 1, Sec-
tion 1 .7) is a reliable means for confirming the true value of a limit, but it provides
few clues for finding such a value in the first place. Furthermore, direct use of the
definition can be difficult — it may not be easy to find a d that works for a given e.

In practical work, limits are often determined by using certain properties that can
be shown to follow deductively from the e, 8 definition. If you have managed a
good intuitive grasp of the idea of a limit, most of these properties should seem
quite reasonable, if not obvious. We recommend that you become familiar with
these properties and their use before studying their proofs (in Appendix B). At first,

in order to show you how the limit properties work, we have deliberately chosen
some very simple examples in which the limits themselves are more or less appar-
ent. Later in this section we use these properties to help prove some important
theorems about limits of trigonometric functions.

The Constant and Identity Properties

1 lim c = c
SECTION 1.8 PROPERTIES OF LIMITS OF FUNCTIONS 61

The Addition, Subtraction, Multiplication, and Division Properties

Suppose that lim /(.v) = L and lim ^(.y) = M. Then

4 Im [fix) + g(x)] = lim /U) + lim g{x) =L + M


5 |im [fix) - gix)\ = lim fix) - lim gix) = L- M
6 Im [fix) -gix)] = [lim/(jc)][Jim gix)] = LM

, ,. fix) J!iP/(^> L
' '™ ~7T ^ T TT = TT provided that A/ ^

EXAMPLE 3 Find:

(a) lim
v^3
ix+ \) (b) lim
.v^3
U- D (c) lim ix
.v^3
+ l)ix - 1) (d) lim ^^
v^3 X - 1

SOLUTION
(a) lim ix + 1) = lim a + lim 1 (Property 4)

= 3+1 (Properties 2 and 1)


= 4
(b) lim (jT - 1) = lim .v - lim 1 (Property 5)

= 3—1 (Properties 2 and 1)


= 2

(c) jim (x + 1)(a: - 1) = [jim ix + l)][lim (jc - 1)] (Property 6)

= (4)(2)

^^ J
lim^ ix + 1)
(d) lim
V— 3 X -
= ^j^
hm (.V - (Property
f J 7) '
I 1)

_ 4_

= 2

The Power, Root, and Absolute- Value Properties


62 CHAPTER 1 FUNCTIONS AND LIMITS

EXAMPLE 4 Find:
SECTION 1.8 PROPERTIES OF LIMITS OF FUNCTIONS 63

Therefore,
64 CHAPTER I FUNCTIONS AND LIMITS

EXAMPLE 8 Find lim x cos — 1

SOLUTION Because values of the cosine always lie between — and


1 1 inclusive,
we have

Lv cos —Mill
= cos — |.v|
1 1

< I

|jr|
I


1 = \x\
I .V I I X \

for all values of .v other than 0. Therefore,

— II ^ |.r| X cos —< 1 I

\x\
,

holds for all values of ^ other than 0. By Properties 10, 3, and 2,

lim l.vl = and lim (-U|) =

Hence, by Property 12,

lim A cos —=
1

The following properties can be used to rewrite limits in equivalent alternative


forms.

The Alternative-Form Properties

13 lim/(.v)
SECTION 1.8 PROPERTIES OF LEVUTS OF FIINCTIONS 65

SOLUTION By Theorem 3 in Section 1.6,

s |cos x — 1 1 < ijr


holds for every value of .r in the open inter\al (-77, tt) except for .v = 0. By
Properties 1, 3, 8, and 2, we have both

lim =

and

lim ic^ = i lim r^ = * • 0- =


JT^O x-*0

Therefore, by the squeezing property (Property 12),

lim Icos x — 1 1
=

Hence, by Property 13, it follows that

lim cos jc = 1

THEOREM 1 Limits of sin x and cos x


66 CHAPTER I FUNCTIONS AND LIMITS

In Section 1.7. our calculator work suggested that

sin X
lim = 1

Now we can prove this important tact.

THEOREM 2 Limit of (sin x)/x

sin .V
lim = 1

PROOF By Theorem 2 in Section 1 .6.

sin X
cos A < < 1

holds for < A- < tt/2. If -77/2 <x< 0, then < -x < tt/2. and we have

sin (-.v)
cos -a) < < 1

Because cos (— .v) = cos x and sin (—a) = —sin a, it follows that

sin A
cos X < < 1

also holds for -n/2 < x < 0. Therefore, it holds for every value of a in the open
interval (
— 7r/2, it/2), except for a = 0. Hence, by the squeezing property (Property
12) together with the facts that

lim cos A = 1 and lim 1 = 1

we can conclude that

sin A
lim = 1
V ^O A

sin 5a:
EXAMPLE 12 Evaluate lim

SOLUTION Let V = 5a, and note that v -^ when a -^ 0. Because a = v/5, we


have

lim
x^O
sin 5a
= hm,.

V—
—sm7^ = v^O
v ,,
lim
/^sin v\
5
X /5
v/5 \ V '

sin V
= 5 lim — (Property 3)

= 5(1) (Theorem 2)

= 5

The method used in Example 12 substituting y — for 5a — is generalized and made


more precise by the following property.
SECTION l.g PROPERTIES OF LIMITS OF Fl-NCTIONS 67

The Substitution Property*

Suppose that lim Av) exists and that lim g(.v) = c. Then, provided that/(g(.r))
is defined
fined and g(x) # c for all va
values of .t in an open interval containing a except
possibly
ibly for jr = a, it follows that
tl

15 lim/(g(.r))= lim/(.v)

EXAMPLE 13 Find lim cos r^ - —


SOLUTION Here, the idea is to make the substitution v = x- - (7r/3), so that
cos [.V- - (7r/3)] = cos v. Informally, we would proceed by notina that as .v -^ 0,
y -^ -~tt/1>: hence.

lim cos (a- --) = ^Jim^^ cos v = cos (- ~) (Theorem 1

This procedure justified by taking /O) = = - (V3)


is
cos v, g(x) .v^ a = and
c = -(-/3) in Property 15.

THEOREM 3 Limit of (1 - COS x)lx

1 — cos A
hm =

PROOF By the half-angle formula.


sin- —= 1 cos X
2
Therefore.

1 - cos A ^ 2 sin- (A-/2) _ sin- (.v/Z)

Now we use the substitution property (Property


15) with v = -^
v/2 Note that v
when A ^ Therefore.
0.

lim i-Zf^= H„ i!5l(V2l^ j^^


^iiry
(Property 15)
-0 a/2

= lim sin v ) (Property 6)


v-^o \ •
V /

,. / s
= dim .

sin V) lim -
y-»o • Vv—O
= (0)(1) = (Theorems 1 and 2)

*The function h(x) = f{g(x)) obtained by evaluating at g(x)


/ is called the composition off
and g. The composition of functions is studied in detail in Section
2.7.
68 CHAPTER 1 FUNCTIONS AND LIMITS

Problem Set 1.8

In Problems 1 to 66, find the limit by using the properties and theo-
rems ol" this section.

I lim 5

3
SECTION 1.9 CONTINUOUS FUNCTIONS 69

73 Prove that Property 3 follows from Properties 6 and 1. sold. The marginal profit at production level x is defined by

74 Prove that Property 11 follows from Property 12.


„. P(x + h)- P{x)
P ^
(.V) = ,.
lim
75 Prove that Property 5 follows from Properties 4 and 3. 'i-o h

76 Suppose lim f(x) = and that there exists a constant B such that where, in taking the limit as /; -^ 0. .v is to be held constant.
Suppose that the profit in dollars from the manufacture and sale
\g(x)\ < B for all values of a in some open interval containing a.
of x quartz travel alarm clocks is
except possibly for.v = a. Use the squeezing property (Property
12) to prove that lim [flx)g{x)] = 0.
P{x) = 20.r - 50,000 -
1000
77 In economics, the profit function P gives the profit Fix) dollars
to the manufacturer if x units of a commodity are produced and Find the marginal profit P'{x).

1.9 Continuous Functions


As we mentioned, there are many functions/for which lim f{x) is the same as /(a).

For such a well-behaved function, there's no problem in finding lim fix) — all you
have to do is evaluate /(a). A function that has this useful property is said to be
continuous at the number a.

DEFINITION 1
Continuous Function at a Number
A function /is continuous at the number a if the following conditions hold:

(i) /(a) is defined

(ii) lim fix) exists

(iii) lim fix) =f(a)

Figure 1 EXAMPLE 1 Is the absolute- value function /(x) = |.v| (Figure 1) continuous at 0?

SOLUTION

(i) Here, /(O) = |0|


= 0, so/(0) is defined,

(ii) By Property 10 in Section 1.8,

lim fix) = lim \x\ = |0| =

so lim fix) exists.

(iii) By (i) and (ii) above,

lim/(.v) = 0=/(0)

so /is continuous at 0.

EXAMPLE 2 Show that the absolute- value function /(.r) = \x\ is actually continu-
ous at every number a.
70 CHAPTER I FUNCTIONS AND LIMITS

SOLUTION Here. f{a) = \a\ is defined. Also, just as in Example 1

lim/(.v) = lim |.v| = \a\ = f(a)

so /is continuous at every number a.

If any of conditions (i). (ii), or (iii) of Definition 1 fail, we say that/ is discon-
tinuous at the number a. Condition (iii),

lim/(.v)=/(a)

means that/(.v) comes closer and closer to /(a) as .v comes closer and closer to a.
This condition will fail if the function value takes a sudden "jump" when .v = a.

EXAMPLE 3 Is the function

+ 3.V + 1

(^\- if .V 7^ -
.v+ 1

3 if .V = - 1

contmuous at — 1
.'

Figure 2
SOLUTION Here, /( - 1 ) = 3, so /( - 1 ) is defined. For .v r^ - 1

Ir^ + 3.V + 1 (Iv + 1)(.Y + 1)


fix) = ;
= = Iv + 1
.V + 1 .V + 1

Hence, by Property 11 of Section 1.8,

lim /(.v) = lim (Zv+ 1) = -1

so lim fix) exists. However, because lim fix) = -1 and /(-I) = 3, we have

lim _/{.v) #/(-!)

and it follows that /is discontinuous —1. The graph of/ (Figure 2) is the line at

y = 2x + 1 everywhere except at .v = -
1. where a "hole" appears. The defined

value of/ at —1 produces a point (-1. 3) on the graph of/ but above this hole.
Figure 3

Another example of a discontinuous function is provided by

f3.t-2 if.r<3
fix)
=
5- X if .V > 3

The graph of/(Figure 3) visually indicates a discontinuity at the number v = 3. In

order to study this apparent discontinuity, it is convenient to introduce the idea of a


one-sided limit, that is, a limit of/(.\ ) as .v approaches 3 through values entirely on
one side of 3. Figure 3 clearly shows that the numerical value /(.v) approaches 7 as
.V approaches 3 through values that arc always less than 3. We write this fact sym-
bolically as

lim /(.v) = 7

the condition that .v approaches 3 from the left being denoted by .v -^ 3~.
SECTION 1.9 CONTINL'OUS FXINCTIONS 71

Likewise, tlie condition that .v approaches 3 through values that are always greater
than 3. so that .v approaches 3 from the right, is denoted by .v -^ 3" From pfgure 3 .

we see that

lim fix) = 2

Because the two one-sided limits of/(.v) as .v ^3^ and as .v -^ 3* are not the same,
it follows that lim /(a) cannot exist. Therefore (as we have already guessed from

our inspection of Figure 3). / is discontinuous at the number 3.


A
formal definition of one-sided limits can be patterned after
Definition 1 in
Section 1.7 (Problem 44). The basic idea is simple. Indeed,
lim f(x) just =L
means that we can make |/(.v) - L\ as small as we please by taking x close enough to
a but less than a. Likewise, Hm^ f(x) = R means that we can make |/(.r)
- R\ as

small as we please by taking .v close enough to a but greater than a.\fL¥=R (as in
Figure 3). then jim cannot More
^^°
/(jc) exist. generally, we have the following
theorem.

THEOREM Limits and One-Sided Limits

The limit ^lini /(.v) exists and equals L if and only if both of the one-sided limits

hm f{x) and lim_/(jr) exist and have the common value L.

We leave the proof of this theorem as an exercise


(Problems 47 to 50).
One-sided limits
satisfy appropriate analogs of the properties in Section
1.8. For
instance, if lim f(x) = L and lim. ^(.v) = M, then

lim_ \f(x) + g(x)] =L+ M


lim, [fix) gix)] = LM

and so forth.

In Examples 4 and 5. (a) sketch the graph of the function, ih) find the one-sided
limits of the function as x -^ a' and as x^
a*, (c) determine the limit of the
fiinction as x -^ a (if this limit exists), and (d) decide
whether the function is
continuous at the number a.

-'"
Figure 4 EX.AMPLE 4 = \]~ '^•' - =
gix) \ « 1
U + .V- if .V > 1

SOLUTION

(a) A sketch of the graph is shown in Figure 4.


(b) lim gi.x) = lim (3 - .<^) = 2

lim, gi.x) = lim (1 -I- .v^) = 2

(c) Because both of the one-sided limits exist and have the same value, 2, it
follows that lim gix) = 2.
72 CHAPTER I FUNCTIONS AND LIMITS

(d) Here. ^'(1) = 3 - 1^ = 2, so ^ is defined at 1 . Also, lim ^(.v) exists and

hm ^(.v) = 2 = g(\)

hence, g is continuous at 1.

I.V
- 21 if X 9^2
EXAMPLE 5 /(a) ,a = 2
1 ifx = 2

SOLUTION

(a) A sketch of the graph of/ is shown in Figure 5.

Figure 5

(b) By Property 11 of Section 1.8,

lim fix) = lim |.v


— 2|

hence, by Property 10 of Section 1.8,

lim f(x) = I
lirn^ (.v - 2)| = |0| =

It follows from Theorem 1 that

Vm fix) = liin. f(x) = Hm_/(.v) =

(c) We have already seen in part (b) that lim /(,v) = 0.

(d) /(2) = 1, lim f(x) = 7^ 1; hence, /is discontinuous at 2.

Properties of Continuous Functions

Suppose that /and g are two functions that are continuous at the number a. Then
f(a) and g{a) are both defined, so that {/+ g)(a) = f(a) + g(a) is defined. More-
over, by Property 4 of Section 1.8,

hm^ (/+ g)(x) = lim^ L/(.v) + ^(.v)) = lim /(.v) + \\m g(.v)

= f(a) + g(ii) = {/+ g)(a)


SECTION 1.9 CONTINUOUS FUNCTIONS 73

Figure 6 y

Figure 7
74 CHAPTER 1 FUNCTIONS AND LIMITS

The following continuity properties of the trigonometric functions follow from


Theorem in Section 1.8 (Problem 46).
1

Continuity Properties of the Trigonometric Functions

6 The sine and cosine functions are continuous at every number.

7 The tangent, cotangent, secant, and cosecant functions are continuous at

every number for which they are defined.

InExamples 9 and 10, use the properties of continuous functions to determine the
numbers at which the given function is continuous.

EXAMPLE 9 /i;,v) = tan X

SOLUTION Because
sin .V

f(x) = tan .V
=

f(x) is undefined when cos x = 0; that is, when

A-= ±— , ±3- — :5- —


2 2

and so on. Therefore, by Property 7, /is continuous at all real numbers except for

odd integer multiples of 7r/2. The graph in Figure 9 clearly shows the periodic
discontinuities of the tangent function.

Figure 9 y

EXAMPLE 10 F(r) = 2 cos (^TT/ - — I

SOLUTION The polynomial function

g(t) = Trt - — 77

is continuous every number by Property 3, and the cosine function is continuous


at

at every number by Property 6; hence, by the substitution property (Property 5), the
function

h{t) = cos ( 7)7 - — ) = cos g{t)


SECTION 1.9 CONTINVIOUS FUNCTIONS 75

is also continuous at even' number. The constant function

fit) = 2

is continuous at even' number by Property 3; hence, by Property 1,

F(t) = 2 cos (777 - ^) = fit) hit)


6

is continuous at every number. (See Figure 10 for the graph of F.)

Figure 10 y

Continuity on an Interval
To say that a function /is continuous on an open interval means, by definition,
that /is continuous at every number in the intenal. For instance, the function

fix) = V'9 - x^

is continuous on the open interval (-3, 3) (Figure 11).


Similarly, to say that a function / is continuous on a closed interval [a, b]
means, by definition, that /is continuous on the open intenal {a. b) and that/
'
satisfies the following "one-sided" " continuity conditions at the endpoints a and b:

lim^/(jc) = f(a) and \\mj(x)=f{b)

Figure 11 For instance, the function f(x) = V9~ X- is actually continuous on the closed
interval [-3, 3] (Figure 11).

EX.\MPLE 11 Determine whether the rational function

x + 1
fix)
x^ - 36
is continuous on the following intervals: (-x, -6), [-6. 6], (-6, 6), and (6, x).

SOLUTION Here, fix) is defined for all real numbers x except for the values
jr = -6 and .v = 6 (which make the denominator zero). Thus, the rational function/
iscontinuous on (-x, -6), (-6, 6). and (6. ^) by Property 4. Since -b and 6 do
not belong to the domain of /, it follows that / is not continuous on the closed
interval [
— 6, 6].
76 CHAKIER 1 FUNCTIONS AND LIMITS

Problem Set 1.9

= x+ I

In Problems 1 to 14, (a) sketch a graph of the function; (b) find the 17 hlx) = X- 2\x\ 18 Fix)
one-sided limits of the function as v approaches a from the right and
X- 1

approaches
from the
a (if this
left; (c)

limit exists);
dctemiinc the limit of the function as
and (d) using the definition
v

of continuity, 19 C(A) =
I

20 fix) = —
decide whether the function is continuous at the number a.
A- - 4a + 3 \x^ if A <
21 ^(A) = 22 hix)
15 -l-.v if .V < 3 Ix' if A >
1 fix) a = 3
>
'

if .V 3 - 4a + - -
.V- 3 .r Iv 3
23 Fix) = + I

if v < A - I

2 F(.v) = i
if .V = ; o =
I
if .X > 24 Gix) 25 Hix) =
A - 2
13 + X if .V < I

a= 1
if .V > I 26 Tix) -
27 fix) = cot A

flv- I if .v< I

4 G(x) =
Ia-
, .^
if X s ,•.«=!
I 28 fix) = sec — 29 gix) = CSC x

= if .V # 5
5 W(.v) a = 5 I
- sin r
if .V = 5 30 hit) = tan |;t
31 fit)
=
cos t

x-2 if .V 5^ 2
6 Hlx) = - a = 2
<j
|.v 2| ;
if A ^
if X = 2 32 fix) = < X
if A =
\2- X i{x> \

7 fix) = a = 1 ( tan X if A < 7T/4


if .r < I
;

33 /;(A)
l^V^sinA if A > 77/4

if .V ^ 2
8 Q(x) = {
x-2 ;a = 2 I

sm if A 9^
if x = 2 34 fix) = < X
if A =
r - 9
''^ "^
9 Fix) = <! v - 3 * -\ a = 3 In Problems 35 to 40, determine whether each function is continu-
if .V = 3 ous or discontinuous on each interval.

3 + x^ if.r<-2
35 fix) = VA - .r on |-2, 2), [2, 3], (-2, 2), and (-1, 5)
10 mx) = i
if A = -2; u =
II -V- if A> -2
36 gix) on (-^, I), (-3. -I), (-X, -I), (-1, =0),

11 5(A-) = 5 + |6.r- 3|, a = i


[-1, =c). and [-2. 2]
12 «(A) = llAll + [15 - aH. « = 4
= -A ;-on(-oo, 61,(-oc, -4], (-6. o:),[-6. 9], and
13 /(A)
= A- -
A +

Iv
I
- 3
,«= -1
37 Fix)

[-7,00)
.r - 36

14 T{x) = 111 -aH + Ha- ID, a= 1 X - 5


38 fix) = on 1-1, l|, (-1. I). (-5, =c), {-X, 51. and
A - 5
1-8.61
in Problems 15 to 34. use the properties of continuous functions to
determine the numbers at which the function is continuous.
39 Gix) = ,'^\ ^„ on 1-i 01. 1-J, 01. (-i x). [-2. x), and
16a- -9
15 fix) = 2\x\ 16 8ix) = \\ -A-l (-1, -i)
REVIEW PROBLEM SET 77

44 Give a formal definition of (a) lim fix) =L and (b)


40 fix) = cot .V on (
- ^ ^ .
)
. - ^ y . I , (0. 17). and [0. v]
lim^ f(x) =R patterned after Definition 1 in Section 1 .7. {Hint:

41 The weight of an object is given by Note, for instance, that < |.v — a| < 5 and v < a if and only if

ax if ,v R -8<x - a <0.)

iv(.v) = ! b 45 Find lim^ V.x - 2 and explain why lim V.v — 2 does not exist.
if v >R

where .v is the distance of the object from the center of the earth. 46 Use Theorem 1 in Section 1 .8 and Properties 2 and 3 of continu-

R is the radius of the earth, and a and b are constants. What ous functions to prove the continuity Properties 6 and 7 of the

relation must exist between these constants if vc is to be a contin- trigonometric functions.

uous function? Assuming that w is continuous, sketch its graph.


47 Suppose that lim f(x) = L. Give an informal argument to ex-

42 If a hollow sphere of radius a is charged with one unit of static plain why it should follow that lim fix) = L.

electricity, the field intensity £ at a point P depends as follows

on the distance .v from the center of the sphere to P: 48 Suppose that lim /(.v) = L. Use the formal definition in Prob-

r if < .V < a lem 44 to prove that lim_ fix) = L.

if -V = a 49 Suppose that lim^ fix) = lim fix) = L. Give an informal argu-


E(x)
ment to explain why it should follow that lim fix) = L.
- if .V > a

(a) Sketch a graph of E. (b) Discuss the continuity of £. 50 Suppose that lim /(.v) = lim /(.v) = L. Use the formal defini-

tions to prove that lim fix) = L.


43 If g is defined by the equation

51 State the analog of Property 1 1 of Section 1.8 for limits from the
,^ = J^-^I-i if.v#l
1 V -
«(-^' right.
1

U( if .v= 1
52 State the analog of Property 1 2 of Section 1 .8 for limits from the
determine the value of o so that g will be continuous at 1. left.

Review Problem Set, Chapter 1

1 Assume that a and b are real numbers such that a < b. Which 10 >0 ll^i^<0
.x-l- 3
of the following are necessarily true and which could be false?
(a) a < 5b (b) -3a < -3b
(c) 5a < -i-Sb) id) a + 5 <b + 5
12^^.1
- V 2
13
x + 2
>3
(e) 1/a > l/b ii) a - 1 > b - 1

In Problems 2
on the number
to 13, solve the inequality

line.
and illustrate the solution
(a)
11,11
14 State the conditions under which each inequality

—<
.T
— 10
(b)
x
<
100
(c) -:;
x^-
1

\
is

<
true.

1000

(d) -.V < X (e) -.ir < T-


2 .V - 5 < 7 3 3.»: -I- 2 > 8
15 Give two different examples of real numbers a. b, c. and d with
4 3j: - 2 > 1 -I- 2x 5 5 > 8.V - 3 > -6
a < b and c < d but ac > bd.

t-
,
"2
1
2+ 7 .^ -I- .V - 20 < 16 For positive real numbers a and b. show that
3
lab a + b
8 -r' - 6.V - 7 s 9 2r -I- 9 < 5 Vab£
a + b
78 CHAPTER 1 FUNCTIONS AND LIMITS

17 A student on academic probation must earn a C in his calculus 35 Determine which equations or inequalities are true for all val-

course to avoid being expelled from college. Since he is lazy ues of the variables.

and has no interest in earning a grade any higher than neces- (a) \x'-4\ = \x- 2\\x + 2| (b) |jr - 4| = .r + 4
sary, he wants his final numerical grade for calculus to lie in - = - = -
(c) I
-tp ;r (d) |.t >•! |v .t|

the interval 170, 80). This grade will be determined by taking


three-fifths of his classroom average, which is 68, plus two- (e) |.r + ix\^x' + \3x\ (f ) \x-y\^ \x\ + \y\

fifths of his score on the exam. Since he has always had


final
36 It is predicted that the number N of cars to be produced by an
trouble with inequalities, he can't figure out the range in which automobile industry in the coming year will satisfy the inequal-
his score on the final exam should fall to allow him to stay in ity \N - 4,000.000| < 500,000. Describe the anticipated pro-
school. Can you help him? duction as an interval of real numbers.

El 18 A runner leaves a starting point and runs along a straight road at


InProblems 37 to 42, plot each pair of points and find the distance
a steady speed of 8.8 miles per hour. After a while, she turns
between them by using the distance formula.
around and begins to run back to the starting point. How fast
must she run on the way back so that her average speed for the
37 (I. 1) and (4, 5)
entire run will be greater than 8 miles per hour but no greater
than 8.5 miles per hour? {Caution: The average speed for her 38 (-1,2) and (5, -7)
entire run ciiniwt be found by adding 8.8 miles per hour to her
39 (-3, 2) and (2, 14)
return speed and dividing by 2.)
[g 40 (4.71, -3.22) and (0, tt)
19 Find the distance between the numbers in each pair and illus-

trate your answer on a number line. 41 (-2, -5) and (-2, 3)

—3 and 4 V2
(a)

(b) and -5
_
El 42 (Vtt,
r-
77) and
/
— 1 -I-

f)
31

[c)(c) -2.735 and -TT (-1,


El 43 What is the perimeter of the triangle with vertices 5),
(d) -3.2 and 4.1 (8, —7), and (4, 1)? Round off your answer to two decimal
(e) -§ and I
places.

[c](f) 1.42 and Vl 44 Are the points (-5, 4). (7, -11), (12, -ID. and (0, 4) the

vertices of a parallelogram? Justify your answer.


20 Use the absolute-value notation to write an expression for the
distance between the two given numbers. Simplify your an-
In Problems 45 to 48, find the radius r and the coordinates (/i, k) of
swer.
the center of each circle, and sketch a graph of the circle.

(a) —6 and 5 (b) — and


x+ 1
45 (.V - 2)- + (v + 3)- = 25

(O— and 3 .
— 4
(d)
.v-i- 1
and — 46 .r -I- 2v -I- y- + 2y + I =

47 .r -I- y- - 3.v -I- 4y -I- 4 =


In Problems 21 to 26, solve the absolute-value equation.
48 4.x- -h 4.V- -I- 4.V - 4y -H I =
21 Ix-t- 1| = 3 22 |2t - 3| = 5
In Problems 49 to 52, find the slope of the line segment AB and an
23 \2y+ 1| = 5 24 \2t H- 3| = |f -t-
2|
equation in point-slope form of the line containing -4 and B.

25 |2ir - M - 2i = 26 |5 - 3z| = 2z
49 A = (3, -5), B = (2, 2)

Problems 27 to 34, solve the absolute-value inequality and show


In
50 A = (0, 7). B = (5, 0)
your solution on a number line.
51 A = (1, 2), B = (-3, -4)
27 |2x + 5| s 6 28 |3.v -I- 4| < 2 = B = (i -I)
52 A (i, 5),

29 |1 -.t|<0.01 30 ll - 4.v| ^x
In Problems 53 and 54, sketch the line L that contains the point P

- > S3 and has slope m, and find an equation in point-slope form for L.
31 |7.r 6| .r 32
U - 1

1 1
53 P = (5, 2), m= -i
33 34 >5
|2v -1-31 4 |3^-1| 54 P = {-L i).m = ?
REVIEW PROBLEM SET 79

In Problems 55 and 56, (a) find an equation of the line that contains In Problems 77 to 82. find the domain of each function.

the point P and is parallel to the line segment AB. and (b) find an
equation of the line that contains the point P and is perpendicular to
=
1

77 fix)
line segment AB.

55 P = (7. -5). A = (1. 8). B = (-3, 2)

56 P = (i, I), A = (4. -f). S = (l, I)

In Problems 57 and 58, rewrite each equation in slope-intercept

form, find the slope m and the y intercept b of the graph, and sketch
the graph.

57 4.V - 3y + 2 =

58 f.Y - iv + 3 =
80 CHAPTER I FUNCTIONS AND LIMITS

84 Which of the graphs in Figure 2 are graphs of functions? ciency. For a certain wind-powered generator, the constant
it = 3.38 X lO""*. Sketch the graph of the function P for this
Figure 2 y generator, and determine how many horsepower are generated
when the wind speed is 35 miles per hour.

92 In economics, it is often assumed that the demand for a com-


modity is a function of selling price, that is,q = /(/>), where p
dollarsis the selling price per unit of the commodity and q is

thenumber of units that will sell at that price, (a) If p increases,

\kr\m^ would you expect q to increase or decrease? What does this


mean about the graph of/? (b) Write an equation for the total
amount of money F{p) in dollars spent by consumers for the
commodity if the selling price per unit is p dollars, (c) What
would it mean to say that there is a value p^^ dollars for which
/(Po) = 0?

93 In economics, it is often assumed that the number of units 5 of

a commodity that producers will supply to the marketplace is a


function of the selling price p dollars per unit, that is, 5 = g(p).
(a) If/) increases, would you expect i to increase or decrease?
What does this mean about the graph of ^?? (b) Write an equa-
tion for the total amount of money (in dollars) C(p) spent by
consumers for the commodity if the selling price per unit is p
dollars and all supplied units are purchased, (c) What would it

mean to say that there is a value p\ dollars for which g(/7| )


=
0?

94 A manufacturer finds that 100.000 electronic calculators are


sold per month at a price of $50 each, but that only 60,000 are
sold per month if the price is $75 each. Suppose that the graph
of the demand function /for these calculators is a straight line
in the pq plane (see Problem 92). (a) Find a formula for/(p),
where p dollars is the selling price per calculator and q = f{p)
is the number per month that will sell at that price, (b) Find the
selling price per calculator if the monthly demand is 80,000
calculators, (c) What price would be so high that no calculators
would be sold?

In Problems 85 to 90. find the domain of the function, sketch its


In Problems 95 to 100, determine without drawing a graph whether
graph, and find the range of the function. El If you wish, use a
the function is even, odd, or neither, and discuss any symmetry of
calculator to help sketch more accurate graphs.
the graph.

85 /(.v) = 5.V - 3 86 g(.v)


95 /(.v) = 5.x-'^ -I- 3.r^ +x 96 g(x) = (X* + X' + 1)-

88 G(.v) = x' 97 hlx) = tx+ l).r"' 98 Fix) = -x^\x\

89 /lU) = 99 G(.v) = x^° - 5.t'' -I- 9 100 Hix) =


if .V < 1 +x

if .V < In Problems 101 to 110, find


90 wu) = ; 1, if < .r < 1
(b)(/-g)(.r)
(a) (/-(-,<?)(.x) {c){J-g){x) (d) (f/g)(x)
y 3.x- - I if .r > I

m 91 The power delivered by a wind-powered generator is given by 101 f(x) = x + 2. g{x) = 3.r - 4

P{x) = kx' horsepower 102 f(x) =x^ + It. g{x) = -x- - It

where x is the speed of the wind in miles per hour and *: is a =


1
= 1

103 f(x) Six)


constant depending on the size of the generator and its effi- x+ 1
REVIEW PROBLEM SET 81

+ 3 X 131 Convert each degree measure to radian measure. Do not use a


104 /U) = jt
-, g(x) = -
X - 2 X - 2 calculator. Write your answer as a rational multiple of tt.

(a) 80° (b) 570° (c) -355°


105 fix) = .^^ g(x) = VTTT
(d) -810° (e) -310° (f) 765°
106 f(x) = X, gix) = \x-2\-x
132 Use a calculator to convert each degree measure to an approxi-
107 fix) = W, gU) = -X mate radian measure, rounded off to four decimal places.

108 /(x) = VVT7. gix) = 7r|jf|


(a) 5° (b) 27.7533° (c) -17.173° (d) 35.2819°

109 /U) =.r'-' + 1, g(A) = V^ 133 Convert each radian measure to degree measure. Do not use a
calculator.
110/U) = -U-,g(j:) = --
X 277 1377 777
\x\
(b)
- (c)
5
In Problems 111 to 122, find the vertex of the graph of the quadratic 3577 5177 1877
(e) (f)
function, determine whether the graph opens upward or downward,
3
sketch the graph, and find the domain and range of the function.
O 134 Use a calculator to convert each radian measure to an approxi-
Ill fix) = 4x^ 112 gix) = W mate degree measure, rounded off to four decimal places.

113 hix) = -ir 114 Fix) = 3.r + 2 (a) 5 (b) 3.9 (c) -7.63 (d) -21.403

115 Gix) = 3U - 2)- + 116 Hix) = i[ix + 3)- + 2]


1
In Problems 135 and 136, find the area A = ir-0 of a sector of a
117 fix) = ,r - 3x + 2 118 gix) = 6;r + 13x - 5 circle of radius r with central angle 0.

119 hix) = -6.r - 7.V + 20 120 Gix) = -2jr + X + 10


135 r = 25 centimeters, = 77/6 136 r = 3.5 meters, 6 = 60°

121 Fix) = 10.V - 25 - r^ 122 Hix) = 7;c + 2r - 39


137 The minute hand on a tower clock is 0.6 meter long. How far
El 123 Suppose a manufacturer of sports trophies knows that the total does the tip of the hand travel in 4 minutes'?
cost C (in dollars) of making .v thousand trophies is given by
138 In Problem 137, what the area of the sector swept out by the
C= 600 + 60-v, and the corresponding sales revenue R (in dol-
is

lars) is given by R = 300.v - 4.v-. Let Fix) be the manufactur- minute hand in 4 minutes?

er's profit if X thousand trophies are manufactured and sold, E) 139 Find the approximate diameter of the moon if its disk subtends
(a) Find a formula for P{x). (b) Find Pi 10) and P(35). (c) Find an angle of 0.5° on at a point the earth 240,000 miles away
the largest and smallest values of the production level x, (Figure 3). iHini: Approximate the diameter \DE\ by the length
rounded off to the nearest trophy, for which the profit is posi-
of the arc BC.
tive.

124 Suppose that/ is a quadratic function and that /(.vi) = Figure 3


fix2) = 0. Show that the .v coordinate of the vertex of the graph
of/ is 5(.lCi + Xi).

In Problems 125 to 130, i denotes the length of the arc intercepted

on a circle of radius r by a central angle of radians. Find the


missing quantity.

= = =
El 140 A satellite in a circular orbit above the earth is known to have a
125 r 5 meters, 6 0.57 radian, s 1
speed of 9.92 kilometers per second. In 10 seconds it moves
126 r = 40 centimeters, .s = 4 centimeters, 0=1 along an arc that subtends an angle of 0.75° at the center of the
earth. If the radius of the earth is 6371 kilometers, how high is
127 .J = 37r feet, 6 = tt radians, r = ?
the satellite above the surface of the earth?
37T
128 / = 13 kilometers, = ^— radians, i = ?
Elln Problems 141 to 158, use a calculator to find the approximate
values of the trigonometric functions.
129 r = 2 meters, s = tt meters, 0=1
141 sin 27.33° 142 sin 421.25°
5-77
130 i = 1777 micrometers, = radians, r = '?

6 143 cos 53.47° 144 cos(- 113.81°)


82 CHAPTER I FUNCTIONS AND LIMITS

145 ian(- 117.2583) 146 lan( -281. 5236) 180 Assume that « is in quadrant IV. cos a = ?, /3 is in quadrant I,

sin /3 = rV. y is 'i quadrant II, cos y = -M. is in


147 sec 16.43° 148 sec( -248.2°)
quadrant II, and sin fl = ^. Find the exact numerical value of

149 CSC —
47r
150 CSC 5.132
each expression.

(a) sin(a + fi) (b) cos(y + 0) (c) sin(/3 + 0)

(d) sin(a - y) (e) cos(/3- y) (f) sin(/3 - y)


151 cot(-3.18) 152 cot(-7.167)
(g) tan(/3 - y) (h) sec(/3 - y) (i) simO - y)
153 cos(- 19.213) 154 CSC 18.113
(j) cos(/3 - 0)
155 sin 3.015 156 cot V3
Problems 181 to 188, simplify each expression.
157 cos —-- 158 sin|tan(- 7 1.32)1
In

181 cos- 2v - sin- 2v 182 1


- 2 sin- —
2

In Problems 159 to 178. simplify each expression.


183 2 sin — cos — 184 cos-* 20 - sin-* 20
2 2
sin(-6») -sin(-a)
159 160 sin 4in
cos(-e) -cos(-a) 185 2 sin" -— + cos 186
2 4 sin TTt cos TTt

161 CSC .V - cos .r cot .r 162 sec e - sin e tan


cos-(r/2) - cos f
163 CSC" / tan- I - 1 164 (cot .V + 1)- - csc^ jc
187 188
sin-(iV2)

see' M + 2 tan u sec fi


165 166 189 If < e < 7r/2 and x = V7 sin 0. find cot in terms of
1 + tan M cot /3 + tan /3
.v.

sin'S + 2 cos-0
190 If < < r 7r/2 and .v = vTT tan f. find cos l in terms of .v.

167 2 cot
sin d cos El 191 A broadcasting antenna tower 200 meters high is to be held
vertical by three cables running from a point 10 meters below
1
168 the top of the tower to concrete anchors sunk in the ground
CSC V - cot y esc y + cot y
(Figure 4). If the cables are to make angles of 60° with the

169 cos(360° - 0) 170 tan(2Tr - /3)


horizontal, how many meters of cable will be required?

171 sin(270° + a) 172 cos(270° - <f>)

Figure 4
173 sin(27r + I) 174 cot (
-^ + xj
175 sin 37° cos 23° + cos 37° sin 23°

.,^ tan(7r/5) + tan(7r/20)


176 ; :

1
- tan(7r/5) tan(7r/20)

177 sin .V cos y - sinl.v + —I sin(-y)

178 cos(77 - t) - tan / cosi i]

179 Use the fact that 77r/l2 = (Tr/4) + (7r/3) to find the exact 192 An electronic echo locator on a commercial fishing boat indi-
numerical value of cates a school of fish at a slant distance of 575 meters from the
ItT boat with an angle of depression of 33.60°. What is the depth of
(a) sin
12 the school of fish?

(b) cos
In El In Problems 193 and 194. use elementary algebra to simplify each
12
expression so that its limit becomes apparent. Then check your re-

777 sults numerically by using a calculator to fill in the blanks in the


(c) tan
accompanying table.
REVIEW PROBLEM SET 83

193 Urn
r-2 1 - (2A)

A
84 CHAPTER 1 FUNCTIONS AND LIMITS

239 The air freight charge for shipping merchandise between two

231 g(.v) =-\ .\


- 2 ; lim s(.v), lim, ^U), lim g(.x) cities is given by the equation
'"* '"*• '""
if = 2
1 .V
0.5a if < .t < 10

5.V + 5 C(.v) 0.4a if lOs A<30


^ 0.3 a 30 s X
232/(.v)=-j |.v+l| '
"^
: lim^ /(.v), Im .
/(.v), lim /(.v)
if

lo if.v= -1 where a is the number of pounds being shipped and C(a) is the
charge in dollars. Sketch the graph of C, and find tho.se values
In Problems 233 to 236, sketcli tiie graph of the given function and of A for which C is discontinuous.
indicate whether the function is continuous at x = a.

240 A function / is defined by the equation

if .r ?^ 3 -X if A <
233 f(x) =< v - 3 a = 3 C
;
if < A s I

if A = 3 -
,A.v) A if 1 < A < 2

X- - 1
if A > 2
if V 7^ 1

234 i'U) =< A- 1 a = 1


:
(a) Sketch the graph of/, and discuss the continuity of/ at 0. 1

if .V = 1
and 2. (b) Determine the constants A, B,C, D. and E so that

v- 1
= Ax + B + C\x\ + D\x - l\+ E\x -
if X ^ 1
f(x) 2|
235 fix) : a = 1

:'A if A- = 1
241 Determine the values of the constants A and B so that the func-
tion/is continuous at every real number and sketch the graph
of the resulting function.
if A ?^ 2
236 /i(.x) : a = 2
3a if a s2
if A = 2
fix) Av B -I- if 2 <A< 5
—6a if a 2: 5
237 Taxi fare is 90 cents plus 50 cents for each quarter-mile or

portion thereof. If we let /(a) denote the fare for a ride of 242 Let [IaD denote the greatest integer not exceeding a. and con-
X miles, sketch the graph of/ and indicate where it is discon- sider the function /(a) = dl/Aj for a > 0. Sketch a graph of/,
tinuous. and indicate where /is discontinuous.

238 Assume that it takes 0.5 calorie of heat to raise the temperature 243 Determine whether each function is continuous or discontinu-
of 1 gram of ice 1 degree Celsius, that it takes 80 calories to ous on each of the indicated intervals.
melt the ice at 0°C, and that it takes 1 calorie to raise the
3
temperature of 1 gram of water 1 degree Celsius. Suppose that (a) fix) 1. 11. [-ill. (-1.1). [5.==)
Zx -
-40 s A < 20, and let Q{x) be the number of calories of heat
1

required to raise gram of water from -40°C to a°C. Sketch


1
'^'^'.(-oc.l).
(b) g(.r) = ^-^ ^ (1,2), [1.21
the graph of Q. and indicate where Q is discontinuous. '^
l
- X lf|SA£2
THE DERIVATIVE

The limit concept, introduced in Chapter 1, is used in this chapter to define a


mathematical procedure called differentiation. A variety of problems that cannot be
handled by strictly algebraic techniques— including problems involving the rate of
change of a variable quantity — can be solved by procedure. From
this a geometric
point of view, such problems can be interpreted as questions involving a tangent
line to the graph of a function.

2.1 Rates of Change and Slopes of Tangent Lines


In this section we use the concept of limit to solve two apparently unrelated prob-
lems; later, we same problem. The first problem is to find
see that they are really the
the rate of change of a variable quantity, for instance, the time rate of change of
distance (speed). The second problem is to find the slope of the tangent line to the
graph of a function at a given point.

Speed of an Automobile
Suppose you are driving along a straight road from city A to city B, perhaps at a

variable rate of speed r. The distance d of your automobile from city A depends on
the elapsed time t since the start of the journey (Figure I). Suppose that the func-

tions / and g give the distance d and the speed /• at time /, so that

d=f(t) and r = g(t)

Figure I For instance, if the rate of speed r is constant, say / = 55


position of automobile
^iles per hour, we have the familiar formula
at time t

city A / city B distance = rate x time

d = rt = 55f

85
86 CHAPTER 2 THE DERIVATIVE

Hence, in this case, /and g are given by

/(/) = 55f and g(t) = 55

In the more general case in which your speed is variable, the functions /and g are
more complicated.
We now find a relationship between the distance function /and the speed function
g. Let's choose and (temporarily) fix a value / of the time variable. Thus, at time /.

your automobile is d = fit) miles from city A. and the speedometer reads r = g(t)
miles per hour. Suppose a short additional interval of time h passes. At time / + h.

your automobile is at a distance /(/ + h ) miles from city A (Figure 2), and its speed
is git + h) miles per hour. Evidently, the automobile has gone/(r + h) —fit) miles
Figure 2 during the time interval h: hence its average speed (distance divided by time)
during the time interval h is

/(/ + /)) -fit)


average speed = miles per hour
-m h
—Al + h)-
If the time interval h is very short, the speedometer reading git) at time / will not

differ much from the speedometer reading git + /;) at the slightly later time / + h.

Furthennore. during this short interval of time, the speedometer readings should be
approximately the same as the average rate of speed.

fit + h) -fit)
.?(')

As the time interval /; becomes shorter and shorter, this approximation will become
more and more accurate. In other words, the speedometer reading at the instant / is
given by
/(/ + /() -fit)
ij(r) = lim

This equation expresses the promised relationship between the distance function/
and the speed function g. In fact, it shows that you can calculate or derive the
instantaneous speed git) from the distance function/by evaluating a suitable limit.

Instantaneous Rate of Change in General


The above considerations concerning the rate of change of distance with respect to
time can be generalized to any variable quantities whatsoever. Indeed, let x and y
denote variable quantities, and suppose that y depends on .v. so that y = fix), where
/ is a suitable function.
To find the rate of change of y per unit change in x. we naturally begin by
considering a change in .v, say from the value .V| to the value .v^. Now, let

.v, =/(.v,) and .V2=/(-V2)

so that as x changes from .Vi to .v^, y undergoes a corresponding change from


y\ to v..
It is traditional to denote the change in .v by the symbol A.v (read "delta x").* so
that

Av = V,

*The expression A.v must be regarded as a single symbol representing a change in the

.v; it must not be thought of as a product of A and .v.


variable
2.1 RATES OF CHANGE AND SLOPES OF TANGENT LINES 87

Similarly, the resulting change in y is denoted by the symbol Ay (read "delta y"),
so that

Aa- = V- /(v.) -fix,)

The ratio of thechange in y to the change in x that produced it is called the average
rate of change of y per unit change in x (or with respect to x). More formally, we
have the following definition.

DEFINITION 1 Average Rate of Change

If y =/lv). then the ratio


88 CHAPIKR 2 THK DKRIVATIVK

Hence, the average rate of change of y with respect to v over the interval A.r is

given by

—Av— = 0.120601
= 12.0601 cubic centimeters per centimeter of edge length
Ax 0.01

(b) More generally, if .v changes by an amount A.v from 2 to 2 + A.v centime-


ters, then y changes by a corresponding amount

Ay = (2 + A.V)' - 2' = 8 + 12 A.v + 6(A.O- + (Axf - 8

= 12 A.v + 6(A.v)- + (A.v)' cubic centimeters

Therefore, the required instantaneous rate of change of y with respect to x is

given by

Av \2 Ix + 6(Axf + (Ax)^
lim -^-= lim ^^ ^^
-= lim [12 + 6 A.v + (Ax)-]
A-V— Ax A-V— A.V Av— O
= 12 cubic centimeters per centimeter of edge length

Figure 3
In calculating the rate of change of one variable with respect to another, it is not
necessary to use the symbols y and .v. For instance, suppose that a particle P is

moving away from a starting point A along a straight line (Figure 3). It is traditional

to denote the distance between A and P at time i by the letter s. If .? = fU). the
speed* of the particle at the instant when / = /[ is given by

,. A.V /(f, +A/)-/(fi)


"
= lim
A(— At A(— At

EX.\MPLE 2 A particle is moving along a straight line in such a way that at the

end of / seconds, its distance 5 in meters from the starting point is given by 5 =
2r + t. Find (a) the average speed As/ At of the particle during the interval of time
from / = 3 to / = 5 seconds and (b) the instantaneous speed of the particle when
/ = 3 seconds.

SOLUTION Here s =/(/) = 2r + f.

(a) When / = 3 seconds, s = 2(3)~ + 3 = 21 meters. As increases by Af = 2 /

to 3 + 2 = 5 seconds, s increases to s + As = 2(5)- + 5 = 55 meters. Thus,


during these 2 seconds, the particle moves through a distance

As = 55 - 21 =34 meters

Its average speed during the interval of time from / = 3 to r = 5 seconds is

therefore

As 34
= = 17 meters per second
^
Af 2

(b) The change in s as the value of t changes (by an amount At) from 3 to
3 + At seconds is given by

As =/(3 + At) -/(3) = [2(3 + At)- + (3 + At)] - [2(3)- + 3] meters

* Actually, there is a difference between speed and velocity-, however, for the simple cases
considered in this section, it isn't necessary to distinguish between the two. (See page 143.)
2.1 RATES OF CHANGE AND SLOPES OF TANGENT LINES 89

The average speed during the interval of time Ar between 3 and 3 + Af seconds
is

A^ /(3 + Af)-/(3)
meters per second
Ar \t

Hence, the instantaneous speed of the particle when t = 3 seconds is, by Defi-
nition 2.

A5 /(3 + An -/(3)
hm = Imi
Af— \t A/^0 Af

Figure 4 [2(3 + At)- + (3 + A/)] - [2(3)- + 3]


= hm
A,^0 A?

[18 + 12 It + 2{Ar)- + 3 + Af] - 21


= lim
Ar-.0 A;
13 It + 2(Af)-
= lim
Af
— lim (13 + 2 Ar) = 13 meters per
'^ second
Ar->0

Slope of a Tangent Line to a Graph


Suppose you want to draw the tangent line to the graph of a function /at a point P
(Figure 4). Since the tangent line is the line that contains P and 'best approxi-
Figure 5
mates" the graph of/ near P. it's easy to sketch it roughly '"by eye."" However,
suppose you need to draw the tangent line accurately. Since a straight line in the
plane is completely determined once you know its slope and one point P on it, you
(x^ + A-i;. ,v, + 4i only need to find the slope m of the tangent line.

Q = (.r, + A.r. v, + Av) on the graph of/ near the point


Figure 5 shows a point
P = (.Vi Thus, A.Y and Av are the differences between the coordinates of Q and
. y, ).

the corresponding coordinates of P. A line segment such as Pg joining two points of


a cur\'e is called a secant, and the line containing P and Q is called a secant line.
Because both P and Q lie on the graph of/

.vi =/(.v,) and y, + Ay =/Ui + A.X)

Therefore,

Ay=/(.v, + A.v)-/(.v,

Figure 6 By the slope formula, the slope of the secant line through P = (.v,, /(.v,)) and
G= Ui + Ajc, /(.r, -I- Ax)) is given by

/(.Y| + A.v)-/(.v,) Ay
.Vi -I- A.V — .v. A.V

Now. if we let A.v approach 0. the point Q will move along the curve y = f(x) and
approach the point P: furthermore, the secant line will pivot about the point P and
approach the tangent line (Figure 6). Thus as A.\ approaches 0, the slope Ay/A.v of
the secant line approaches the slope m of the tangent line; that is.

Ay /(.vi + A.r)-/(.vi)
m = hm ,.
lim
A-v^o A.r Ax
90 CHAPTER 2 THE DERIVATIVE

The preceding considerations lead us to the following formal definition.

DEFINITION 3 Tangent Line to a Graph

Let /be a function defined at least in some open interval containing the number
.V|, and let V| =/(.Vi). If the limit

/(A, + A.V) -/Ui)


m = lim :

A> -II A.V

exists, we say that the line in the vv plane containing the point (.\ i
. y, ) and having
slope m is the tangent line to the ^iniph offiit (.V|, Vi).

In Examples 3 and 4, find the slope m of the tangent line to the graph of the given
function f at the indicated point P. Sketch a graph off showing the tangent line
at P.

Figure 7
2.1 RATES OF CHANGE AND SLOPES OF TANGENT LINES 91

Problem Set 2.1


I At a certain instant the speedometer of an automobile reads r 20 A projectile is fired vertically upward and is s feet above the
miles per hour. During the next \ second the automobile travels ground t seconds after being fired, where i = 256r — 16r. Find
20 feet. Estimate r from this information. (a) the speed of the projectile 4 seconds after being fired, (b) the

time in seconds required for the projectile to reach its maximum


~ Explain why the answer in Problem 1 is only an estimate for r
height (at which point its speed is feet per second), and (c) the
and need not be exactly the same as /•.

maximum height to which the projectile ascends.

In Problems 3 and 4. assume that y =/(.v) as given, (a) Find the 21 An equilateral triangle made of sheet metal is expanding because
average rate of change of y with respect to x as .v changes from .Vi to it is being heated. Its area A is given by A = (V3/4).r square
.V;. (b) Find the instantaneous rate of change of y with respect to x centimeters, where x is the length of one side in centimeters.
at the instant when .v = .vi. Find the instantaneous rate of change of A with respect to.v at the

instant when .v = 10 centimeters.


[S 3 y =/(A-) = X- + x+ I. .V, = 3, X2 = 3.5
22 A spherical balloon of radius R meters has volume V = lir/?'
4
4 V =/(.v) = — , xi = 5, .V, = 6
cubic meters. Find the instantaneous rate of change of V with
.V respect to R at the moment when R = 5 meters.

In Problems 5 to 8. a particle is moving along a straight line accord- 23 The pressure P of a gas depends on its volume V according to
ing to the equation given, where 5 is the distance in meters of the Boyle's law. P = C/V, where C is a constant. Suppose that
particle from its starting point at the end of r seconds. Find (a) the C = 2000. that P is measured in pounds per square inch, and
average speed ls/\t of the particle during the interval of time from that V is measured in cubic inches. Find (a) the average rate of
r = fi to f = fi and (b) the instantaneous speed of the particle when change of P with respect to V as V increases from 100 to 125
cubic inches and (b) the instantaneous rate of change of P with
respect to V at the instant when V = 100 cubic inches.
5 i = 6r, f, = 2. /2 = 3 6 i = 7r\ i, = I. h = 2
24 Ecologists studying acid rain determine that the concentration C
of sulfur dioxide in parts per million at a distance .v kilometers
from a coal-burning power plant is given by C= SOO/.i". Find
the instantaneous rate of change of sulfur dioxide concentration
In Problems 9 to 18, find the slope m of the tangent line to the graph in parts per million per kilometer at .v = 10 kilometers.
of each function at the indicated point, sketch the graph, and show
25 In respiratory physiology it has been found that a person's respi-
the tangent line at that point.
ration rate R in breaths per minute is related to the partial pres-

'^
fix) = Iv-A- at (1. 1) 10 /(.v) = (.V- 2)- at (-2. 16) sure P in newtons per square meter of carbon dioxide in the

lungs by an equation of the form R = AP - B, where A and B


11 fix) = r - 4v at (3, -3) are constants depending on the particular individual involved. If

12/(.v)=.v' at (-1, -1) A = 0.0044 and B = 10.4 for a certain person, what is the in-
stantaneous rate of change of R with respect to P when P =
13 fix) = i + Zx-x' at (0, 3) 8000 newtons per square meter?

26 Respiratory physiologists sometimes use the simplified mathe-


14/U) = -^at(^. 2)
matical model p = a — b iXn ct for the air pressure p (in new-
tons per square meter) in the lungs of a person t seconds after
3
15 fix) at ( 1 . 1 inspiration begins. Here a denotes ambient atmospheric pres-
.v+ 2
sure. 1.013 X 10" newtons per square meter at sea level, and b

16 fix) = Vx - 3 at (7, 2) and c are constants depending on the particular individual in-

volved. If i) = 100 and c = 1.26. find the instantaneous rate


17 fix) = V.v + 1 at (3. 2)
when =
of change of p with respect to time 1 at the instant 1

18 fix) = \9 - Ax at (-4. 5) (the beginning of inspiration). {Hint: Use the fact that

lim (sin .v)/.v =1.1


<—
19 An object falls from rest according to the equation s = I6r.
where s is the number of feet through which it falls during the 27 The current / in amperes in a certain electric circuit is given by
first t seconds after being released. Find (a) the average speed the formula / = 100//?. where R is the resistance in ohms. Find
during the first 5 seconds of fall and (b) the instantaneous speed the instantaneous rate of change of / with respect to R when
at the end of this 5-second interval. R= \Q ohms.
92 CHAPTER 2 THE DERIVATIVE

2.2 The Derivative of a Function


In Section 2. 1 we found that it' .x and v are two variables related by an equation
y = f(x), then the instantaneous rate of change of y with respect to x when .v has the
value Xi is given b.y

Ay /(.V| + A.v)-/(.vi)
,.
lim = lim
A<— A.V A 1-^(1 A.v

We also found that the slope m of the tangent line to the graph of /' at the point

Ui./Ui)) is given by the same limit m= lim (Av/Ax). Therefore, the problem of

finding the rate of change of one variable with respect to another and the problem of
finding the slope of the tangent line to a graph are both solved by calculating the
same limit.

Limits of the form lim Av/A.v arise so often in calculus that it is useful to
Av^O
introduce some special notation and terminology for them. If 3' =/(.v), then a quo-
tient of the form
Ay _ /(v+ A.V) -fix)
A.V Ax
is called a difference quotient. The limit of such a difference quotient as Av
approaches defines a new function/', read "/prime." by the equation

/(.V + A.V)-/(.Y)
fix) = lim
A.v^O A.V

Since the function/' is derived from the original function/, it is called the derivative
of/. Thus, we have the following definition.

DEFINITION 1 The Derivative

Given a function /, the function /' defined by

+
/(.v A.v)-/(.v)
lim —Av—
A.V Av— Ax
is called the derivative of /.

In the definition it is understood that the domain of the derivative function f is the
set of all numbers x in the domain offfor which the limit of the difference quotient
exists. In calculating this limit, you must be careful to treat x as a constant while

letting Ajc approach zero.

In Examples 1 and 2. find fix) for the given function by direct ilw of Definition I

EXAMPLE 1 fix) = .\-^

fix + A.V) - fix) ix + Ax)' - x'


SOLUTION fix) = lim : = lim
A.V A.i — A.v

v' + 3.Y- A.Y + 3.Y(A.v)- + (A.V)' - .v'


= lim
A.-*ll Ay
= lim (3a- + 3.Y A.Y + (A.y)-] = 3.y-
A.v—
2.2 THE DERIVATIVE OF A FUNCTION 93

EXAMPLE 2 fix) = Vx

SOLUTION For X > 0, we have

= hm ,. /(A- + A.v)-/(.v)
=
,. V.v + Ax-V^
f (A)
Ar^O
lim
A.X Ar^O Ax
(Vat + Ajc - Vx)(Vx + A.v + \/x) (,r + A.v) - a
= lim .
= lim
Av^o Aa(Vv + Aa + Va) A.v^o A.v(Va + Aa + Va)
Aa
= lim = lim
Aa^o Aa(Va + Aa + Va) '^-v^o Va + Aa + Vx

Vx+\G 2Va

The Derivative Notations


The derivative was invented independently by Isaac Newton and Gottfried Leibniz
in the seventeenth century. Newton used the notation i to denote the time rate of
change lim (Ai/AO of a variable quantity j, where 5 =/(?). Thus, Newton wrote i

for what we write as /'(/), the value of the derivative/' at the time t. Newton's
notation is still used in many physics textbooks.
Leibniz, on the other hand, realizing that the numerical value of a derivative is

the limit of A>'/Aa, wrote this limit as dy/dx; that is.

Av
—^
lim =
/'(.v)
A<-o Aa

Henceforth, we make extensive use of the Leibniz notation; however, until the
"differentials" dy and dx are given separate meanings (in Section 4.1), we do not
regard dy/dx as a fraction, only as a convenient symbol for the value of a derivative.
In the eighteenth century, the French mathematician Joseph Louis Lagrange
(1736-1813) introduced the notation/' for the derivative of the function/. Lagrange
is perhaps best known for his book Mecaniqiie which not only summa-
analytiqiie.
rized all previous work in mechanics since the time of Newton, but also contained
many of Lagrange's own highly original and important contributions to the field.
The Lagrange notation/' is the preferred notation for the derivative whenever preci-
sion and absolute clarity are demanded. Indeed, using this notation, you can easily
distinguish between the derivative/' (which is a function) and the numerical value
f'(x) of the derivative function at the number a.
The operation of finding the derivative/' of a function /or of finding the value
/'(a) is called differentiation. Thus, the incomplete symbol

jl_
~dx

Joseph Louis Lagrange


can be regarded as an instruction to differentiate whatever follows. For instance, the
result of Example 1 can be written in the Leibniz notation as

-x^ = S.r^
dx
94 CHAPTKR 2 THK DERIVATIVE

Popular alternative notation for the symbol d/dx is the simpler symbol D, (or
sometimes just D if the independent variable is understood), which is called the
differentiation operator. Thus, if v = a', then

Dj- = D^x^ = 3x'

In Example 2, we showed that if/U) = Va, then/'(.v) = l/(2V.v). This useful


result can be rewritten in Leibniz and operator notation as follows:

— yx^ = D.Vxr = —-
d \

2\ X
dx

Preference for one notation over another is often just a matter of taste and conven-
ience. In the remainder of this book, we use whichever symbolism seems appropri-
ate to the problem at hand. Also, as the following example shows, we can use
symbols other than y and .v to denote the dependent and independent variables.

KXAMPLK 3 A body dropped from rest will fall .v meters in i seconds according
to the equation s = 4.9r. Find the speed ds/di of the falling body at the end of
t seconds.

SOLUTION

—=
ds

di
Imi
A^—
\s
A/

4.9(/ -I- A;)- - 4.9r


lim
Af

4.9[r + 2t\t + (Al)-] - 4.9r


lim
A^^

9.8/ A? + 4.9(A/)-
lim
Af— A?
= lim (9.8/ + 4.9 A/)
Al^O
= 9.8/

Up to this point, we have taken care to distinguish between the derivative/' of a


function/and the value/'(.v) of this derivative function at the number .v. In practice,
however, it is customary to use the word "'derivative" to refer to both the derived
function/' and the value /'(a) of this function at a.

In summary, if

y=.nx)
then the insicintaiwous rate of change of y with respect to x. or, what is the same
thing, the sh)pe of the tangent line to the graph offat the point (x. y). is given by

+ Ax) - f(x)
dx
D,y = DJlx) = f'(x)
fix

Av
= ,.
lim
A>--0

Av
Aa
2.2 THE DERIVATIVE OF A FUNCTION 95

Differentiability and Continuity


Consider the function / defined by the equation

U.v-13 ifx>3
Figure 1 (See Figure 1.) Since lini /(.v) = -1 =/(3), it follows that /is continuous at the

number 3. However, if we form the difference quotient

/(3 + A.v)-/(3) f{3 + Ax)+ 1


_
Ax A.V

and calculate its limits as Ax approaches zero both from the right and from the left,*
we obtain

+ A.t)-/(3) + Ax) -
—4 —
,. /(3 [4(3 13] + 1 Ax
lim ^
= hm hm :
= 4
A.v-*0* Ax A.v^O+ Ax i.v-.o* Ax
whereas

,. /(3 + A.r)-/(3) [5 - 2(3 + Ax)] + 1 -2 Ax


lim = hm = lim = -2
A.v^O A.V A.v^O' Ax Ax
Since the right and left limits of the difference quotient are not equal, the limit of the
difference quotient cannot exist; that is, the derivative /'(3) cannot exist. The non-
existence of the derivative of/ at 3 might have been anticipated from the graph in
Figure 1. since this graph has no tangent line at (3, -1).
In general, we define the derivative from the right of a function /by

f(x+ Ax)-/(x)
/V(x) = lim
Aa-^O'

Similarly, the derivative from the left of/ is defined by

/'-(-
96 CHAPTER 2 THE DERIVATIVE

Here.

/(I + A.v)-/(1)
A(l)= lim

[2(1 + A.V)- 1] - 1 ,. 2A.V ^


= lirn = lim = 2
A.V -c A.V

Also,

/(I + Aa:)-/(1)
/'-(I) lim
Ax
(1 + Ax)^ - 1

lim = lim (2 + A.V) = 2


A;r

Since/'+(l) =/'-(l) = 2, we conclude that/'(l) exists and equals 2. This example


shows that a function defined "piecewise" can have a derivative at the boundary
number between the "pieces."

DEFINITION 2 Differentiable Function

A function/ is said to be differentiable at the number x if/ is defined at least on


some open interval containing v and /'(.v) exists.

Evidently, /is differentiable at x if and only if both of the one-sided derivatives


/V(.v) and/'_(.v) exist and are equal. A function/is said to be differentiable on the
open interval (a, b) if it is differentiable at each number in this interval. If a

function is differentiable at each number in its domain, it is called a differentiable


function.
Geometrically, to say that a function/is differentiable at a number .v is to say that
the graph of/has a tangent line with slope/'(.v) at the point (jr. f(x)). Obviously, if

a graph has a tangent line at a point, it cannot have a discontinuity at that point.

Theorem 1 confirms this analytically.

THEOREM 1 Continuity of a Differentiable Function

If a function / is differentiable at the number .v. then it is continuous at .v.

Assume that/is differentiable at the number .v. By Property 14 on page 64, /will be
continuous at x if lim f(x + A.v) = f(x). Since the limit of a product is the product

of the limits, we have


r f(x + Aa) - f(x)
— . 1
lim |/(.v + A.V) -f(x)] = lim A.v
A.t^O A.V— L A.V J

LA.V-.0 Ajr J VA.V— /

= /'(.v)-0 =

Therefore, since the limit of a sum is the sum of the limits, we have
lim fix + Ax) = lim \f(x + A.t) - f(x) + /(.v)]
A.r —'0 A.V —'0
= lim l/(.v + A.V) -/(.v)] + lim /(.v) = +/(.v) =/(.v)
2.2 THE DERIVATIVE OF A Fl'NCTION 97

Figure 3 Although, as Theorem 1 shows, a differentiable function is automatically contin-


uous, there are contimums functions that are not differentiable. The simplest exam-
ple is the function /defined by/(.v) = |.v| (Figure 3). Note that/is continuous at the
number 0. but it is not differentiable at since /V(0) = 1 and/'_(0) = -1. A
similar example is provided by the function graphed in Figure 1

Approximating Derivatives with a Calculator


A calculator or computer may be used
ed to find the approximate
apprc value of a derivative
by using the fact that
f{x + ^x)-f(x)
f'M

holds for small values of A.v. Up to a point, the approximation becomes more and
more accurate as A.v is chosen to be smaller and smaller. However, if A.v is chosen
to be too small, the calculator may not be able to resolve the difference between
fix + Ax) and/U), and the result of the calculation is useless. For most practical
purposes, the value A.v = .v/lO"' will provide a good approximation.

3 EXAMPLE 5 Use a calculator to find the approximate value of/'(77-/3) if/Cv) =


sin.r. Use A.v = (77-/3)/10-*.

SOLUTION With the calculator in radian mode, we find that

sin [(7J-/3) + A.v] - sin (7r/3)


/(!)-
3 / A.V
0.499956

[The exact value of/'(77-/3) happens to be O.5.]

Problem Set 2.2

In Problems 1 to 10, find/'(.v) by direct use of Definition 1. In Problems 17 to 20, find/'(.vi) for the given value of .vi by direct
calculation of
1 fix) = 4x + l 2 fix) = 13 - 7.x-

+ A.v)-/(.vi)
3 fix) = A 4 fix) = 3 -I- V^ ,. /(.v,

A.V
5 fix) = .r + 4x 6 fix) = 2r' - 1

x^ 3 17 fix) = - 2r=; = - 18 f(x) = sin x: .v, =


^ + y^
I .T,
7 fix) = Ir* - 4v 8 fix) =
(Use Theorem 2. page 66.)

9 fix) = — 10 fix) 19 fix) = -, -v, = 3 20 fix) = cos x\ =


X- 3 Iv- 1
-r,

(Use Theorem 3, page 67.)

In Problems 11 to 16, find the required derivative by direct use of


In Problems 21 to 23, find the value of the derivative for each func-
Definition 1.
tion at the number indicated.

ill ^ = —r-^=
/
3
- I
ds
dt
12 s =
1+ I
.D,s = ?
21 /'(4) \ffix) =
X- 1

13/(v) = Vv- l,/'(v) = ? 14 —- (Vl - 9m-) = ?


du 22 D,s at f = 3 if J = V'2r + 3

2 d\ 2
15 V D,y = 16 hit) h'it) 23 -^ at .V = 2 if V =
x+ I 't+ I dx Iv + 1
98 C HAPl ER 2 THE DERIVATIVE

24 Find D,P \( P = I'R and /? is a constant. x + 2 \fx> -2


34 /(.v) = ,x,^ -2
odd-numbered Problems
< -2
ifjf
25 Rewrite your answers to 1 to 15 in both
Leibniz and operator notation. |(.v- 1)' if .V >
35 fix) : .t, =
26 Given that i = f(t), write the value of the derivative /'(/) in as
iix- + 3.V - 1 if .r <
many different ways as you can. 36 fix) = I
- |.v - 3|; .V, = 3

In Problems 27 and 28, find the indicated derivative. 37 (a) Explain, in your own words, why it is geometrically reason-
able that a differentiable function must be continuous, (b) Is it

27 D,.v if i-
^
\br + 30/ + 10 geometrically reasonable to believe that every continuous func-
tion is differentiable? Why or why not?

28 —du- ..
it u 16v- + 30v + 10
dv 38 Suppose that

In Problems 29 to 36, (a) sketch the graph of /, (b) determine =


if a: < -
fix)
whether/is continuous at the number .V| and , (c) determine whether lav -I- b if -v s -
/is differentiable at .V| by finding /'+(.Y| ) and/'-(.V|).
Find values of the constants a and b so that/'(- 1) exists.

29 f(x) = X- - Ix; .vi = 3


In Problems 39 and 40, sketch the graph of the function and indicate
flv + 9 if.r<-l where the function is not differentiable.

39 fix) = |3.v 40 fix) k'-H


O In Problems 41 to 44, use a calculator to find the approximate value
of the indicated derivative.
.V, =4
41 /'(7r/3) for/(.v) = cos .v 42/' (77/4) for/(jc) = tan.r

43 /'(lO) for/(A) = V-v- 1 44 /'(7r/6) for/(jr) = Vsin x


if .V > 2

2.3 Basic Algebraic Rules for Differentiation

In Section 2.2 we differentiated functions by direct use of the definition of the


derivative as a limit of a difference quotient. Direct calculation of derivatives in this
way can be tedious, even for the relatively simple functions considered up to now.
Relief from this tedium is forthcoming — there are general rules for differentiation
which permit straightforward calculation of derivatives. In this section, we present
rules for differentiating sums, products, powers, quotients, and square roots. At
first, we simply state these rules informally and illustrate their applications in exam-
ples. Later in the section, we state them precisely and give rigorous proofs.

Constant Rule
!.3 BASIC ALGEBRAIC RILES FOR DIFFEREMTUTION 99

In using the constant rule, it is often convenient to deliberately confuse a constant


number such as 7 and the constant function / which it determines according to the
equation /(.v) = 7. Thus, we often write "D,(7) = O"" or dl/dx = 0" instead of
writing "/' is the function defined by the equation/'(.v) = 0." Since vom can differ-
entiate only functions — never numbers — an expression of the form D,(7) = could
only be interpreted sensibly as above.

EXAMPLE I Let/be the constant function defined bv the equation /(a) = 5 + tt.

Find/'.

SOLUTION By the constant rule,/' is the constant function defined by the equa-
tion /'(.v) = 0.

EXAMPLE 2 Find D,{5 + V3).


SOLUTION By the constant rule. D,(5 + Vl) = 0.

Identity Rule

EXA.MPLE 3 If/ is the function defined by f(x) = x, find/'.

SOLUTION By the identity rule,/' is the constant function defined by/'(.v) = 1.

Power Rule

The derivative of a positive integer power of .v is the exponent of .v times x raised


to the next lower power. In symbols, if n is a fixed positive integer, then

D,x'

EXAMPLE 4 Differentiate the function /(a) = x^.

SOLUTION By the power rule, fix) = 7a-''"' = 7.v*

du
EXAMPLE 5 If « = / , find
dt


By
,

the power rule,


du
dt
= — d
dt
t
.,
= 13f
,-,

Homogeneous Rule

The derivative of a constant times a function is the constant times the derivatl\e
of the function. In symbols, if c is a constant and u is a differentiable function of
X. then

Djcu) = cDji or —d
dx
(cu) = c—-
du
dx
100 CHAPTER 2 THE DERIVATIVE

EXAMPLE 6 Differentiate the function /(.r) = 5.x'*.

SOLUTION Using both the homogeneous rule and the power rule, we have

fix) = DASx"*) = soy = 5(4.x') = 20a:'

EXAMPLE 7 Find
"m
SOLUTION By the homogeneous and power rules,

IvM
DA
/
= D. I
V

2
.v^
,\
= — oy = —
2 ., 2
(7.v")
,
= 14 ,
.v"
= U.v"
\ 3 ' 3 / 3 3 3 3

EXAMPLE 8 If c is a constant and n is a positive integer, find —d


dx
(c.x").

(c.x") .x" = f(/u" ) = nc.x"


dx d.x

One important consequence of the homogeneous rule is that

D,(-ii) = -D,u
This follows from putting r = - 1 in Rule 4.
Many functions encountered in practice are (or can be rewritten as) sums of
simpler functions. For instance, the polynomial function /(.v) = Iv" + 5.v — 1 is a
sum of lv~, 5.x. and —1. Thus, one of the most useful differentiation rules is the
following.

Sum Rule, or Addition Rule

The derivative of a sum is the sum of the derivatives. In symbols, if u and v are
differentiable functions of .v, then

d dti dv
D,(M + I) = D,u + D,v (// + V) = +
dx d.x d.x

EXAMPLE 9 Find D,(3.v'^ + ll.v**).

SOLUTION We have

Dv(3.v-' + 1 l.v**) = Dv(3.v') + D,(l l.v**) (sum rule)


= ID^ + llD,.v'* (homogeneous rule)
= 3(5.v-*) + ll(8.v^) (power rule)
= IS.v-* + 88.v^

If II. V. and vr are three differentiable functions of v, then, by the sum rule,

DM + V + IV) = DAu + (V + w)]


= D,u + DAv + H')
= D,« + D,v + D,ir

More generally, the derivative of the sum of three or more differentiable functions
is the sum of their derivatives. This rule is also called the sum rule.
2.3 BASIC ALGEBRAIC RULES FOR DIFFERENTIATION 101

Using the sum, homogeneous, power, identity, and constant rules, you can dif-
ferentiate any polynomial function term by term. This is illustrated by the following
examples.

EXAMPLE 10 Given /(.v) = 3.v"^ 24.Y^ + 7.r- - .V - 2. find/'(.v).

SOLUTION

fix) = D,(3;c'°° - 24.V-' + 7.Y- - .r - 2)


= D,(3,r"») + D,{-24.v-') + D,(7.v-) + D,(-.v) + D,(-2)
= BOOa-^*^ - 72v- + 14.V - 1

EXAMPLE II Find -^if


du
v = Vlu^ - —+
4
5u^ + u + tt'.

—=dV
du
5V2m^
r- ,

(4«-) + 15m- +1+0


= 5V2M-* - i/' + 15«- + 1

The rule for differentiating the product of two functions is more complicated than
the rule for differentiating their sum. Early in the development of calculus, Leibniz
found that, in general, the derivative of a product is not the product of the deriva-
tives. He did manage to find the correct rule, which is the following.

Rt^'LE 6 Product Rule, or Multiplication Rule

The derivative of the product of two functions is the first function times the
derivative of the second function plus the derivative of the first function times the
second function. In symbols, if u and v are differentiable functions of .t, then

D^iuv) = u(D^v) + (D,u)v or —


dx
{uv) = u
d.\
1-

d.x
v

EX.AMPLE 12 Find D,[(3.v- + l)(7.v^ + .v)] by using the multiplication rule.

SOLUTION

DAOx^ + l)(7.v3 + X)] = Ox^ + 1)[D,(7a-3 + .x)] + [D,(3.v- + IWx^ + x)


= {3x^ + 1)(21a:- + 1) + (6.r)(7.v- + x)
= (63^" + 24.V- + 1) + (42v-* + 6.v-)
= 105.Y-* + 30.V- + 1

EXAMPLE 13 Suppose that/ and g are differentiable functions at the number 2 and
that/(2) = 1, g(2) = 10, /'(2) = h and g'a) = 3. If h=f-g, find h\2).

SOLUTION By the multiplication rule.

h'(x)=fi.x)-g'{x)+f'(.x)-g{.x)

so that

h'{2)=f{2)-g'i2)+fa)-g(2)

/!'(2) = (1)(3) + {*)(10) = 8


102 CHAPTER 2 THE DERIVATIVE

Reciprocal Rule

The derivative of the reciprocal of a function is the negative of the derivative of

the function divided by the square of the function. In symbols, if v is a differen-

tiable function of v, then

Da- d_ dv/dx
'\{
dx

EXAMPLE 14 Find D .(7)^

SOLUTION By the reciprocal rule.

D,x
<l)-
Quotient Rule

The derivative of a quotient of two functions is the denominator times the deriva-
tive of the numerator minus the numerator times the derivative of the denomina-
tor, all divided by the square of the denominator. In symbols, if u and v are
differentiabie functions of v, then
du dv
\ /(

I'D,;/ — uD^v d I u \ d\ d.\


DA V V /
dx

EXAMPLE L*; Find D>


.v-^ + 7 /

SOLUTION By the quotient rule,

\ (v-' + l)D,x^ - x~D,(x^ + 7)


D,
x' + 1 (A-' + 7)-
(.v' + 7)(lv) - .v-(3.v-)
(.v' + 1)-

14.V

(x^ + 7f
EXAMPLE 16 Suppose that/ and g are differentiabie functions at the number 3 and
that/(3) = -2, gO) = -5,/'(3) = 3, and g'{3) = 1. If /? =f/g. find the slope of

the tangent line to the graph of /; at the point (3, i).

SOLUTION By the quotient rule,

g(x)f'{x)-nx)g-{x)
h'ix)
[gix)f

so the required slope is given by

g(3)/'(3) -/(3)^'(3) (-5X3) - (-2H1) 13


h'0) =
U'(3)l- (-5)- 25
2.3 BASIC ALGEBRAIC RULES FOR DIFFERENTIATION 103

Since a rational function is a quotient of polynomial functions and you can now

differentiate any polynomial function, you can use the quotient rule to dijferentiate
any rational function. Example 15 above illustrates the technique.
In Example 2 in Section 2.2 (page 93), we obtained the following result.

RULE 9 Square-Root Rule

The derivative of the square root oix is the reciprocal of twice the square root of
X. In symbols.

_d_
0,V.v = r.-_L^
2\ dx 2\ T\

EXAMPLE 17 Find D,-


V.v

By the reciprocal and square-root rules,

1 DvV^ 1/(2Vt)
Vx (Vx)- ItVx

It"s interesting to note that because

Vx = .r'^- and
2V^
the square-root rule is just an extension of the power rule (Rule 3) to the case n = h.

The power rule can also be generalized to arbitrary integer powers as follows.

RULE 10 Power Rule for Integer Exponents

If n is any integer, then


104 CHAPTKR 2 THE DERIVATIVE

/ , V5 7j-\ d ^ /- ,
TTX'")
dx dx
= lx- iVlx-^ + 5i

Later we show that the power rule D,.v" = nx"' '


actually works for any constant
real number n.

Proofs of the Basic Differentiation Rules


We now give precise statements and rigorous proofs of the basic differentiation
rules. It is convenient to prove the theorems in an order other than that in which the
rules were stated.

Constant Rule

U r is a constant and / is the constant function defined by /(.v) = c, then / is

differentiable at every number v and /' is the function defined by /'(.v) = 0.

fix) = lim
fix +
— A.V)
—- ^— /(.Y)
^ = lim
r - f
= lim =

THEOREM 2 Identity Rule

If/ is the function defined by/(.v) = .v, then/is differentiable at every number .v

and/' is the constant function defined by /'(.v) = 1.

/ (.v) = hm
/U + A-Y)-/(.v)
: = lim
.V + A.V
— = lim 1 = 1

A.V— A.V A.V-.0 A.V

THEOREM 3 Sum Rule

Let/and g be functions both of which are differentiable at the number .V| and , let

'' ~
f ^ 8- Then /; is also differentiable at .v, and
/!'(.V|) =/'(.V|) + fi'iXi)

/?(.Y| + A.V) - /U.vi)


h'ix, ) = lim
Ax—O A.V

L/(.v, + A.V) + gix, + A.V)] - [/(.r, ) + g(.v,


Mm
Aa ^0 A.V

,. fix, + A.V) -fix, ) + gix, + A.V) - g(.V|


= hm
Aa ^o A.V

r /Ui + A.V) -fix,) ^


g(.V| + A.V) - gU,) 1
lin,
A.(-»o L Aa: A.v J

fix, + Ax) -fix,) gix, + Ax)- gix,)


= hm ,

1- Iim :

Aa^— A.V A.v-»o A.V

= /'(.v,) + ^'(.v,)
2.3 BASIC ALGEBRAIC RULES FOR
DIFFERENTUTION 105

THEOREM 4 Product Rule

Ley and g be functions both of which are differentiable


at the number v, and let
n ~J-g. Then /; is also differentiable at .v, and

^i^'i '
= /(vi )

g 'Ui ) + fix, ) gi.x, )

PROOF /,'(.v,)= l.m Jl^^-lA^LJli^

= /'-V|+A-v)-g(-V|+A.v)-/Ui)-^(.v,)
lim
AV-.0
^^

We now use
curious a
but effective algebraic trick-the expression
/U, + A.r)-^U,) subtracted from the numerator and then
IS
added back again
(which, of course, leaves the value of
the numerator unchanged). The result is

h'(xi) = lim
A-r—O
Ax

limUv, + A.t)g'''+-^'^)-^'-^i) ^ /Ui


,
+A.t) -/(.>-,) 1
^'-4 Ax ^^ S<'i»J

= .ti + A.X) - ,^(.V|


lim fix, + A-x)\-\ lim

+r ,:„ /(.v. + A.v)-/t .r,)1 r ,. ]

[^im /(.r, + A.r)j .g'u,) +/'(.x-,) |^^i,m g,.,. ,

Since/is differentiable at.v, it is continuous


. at.v, (Theorem 1, Section 2.2); hence.

2im/(.v, + A.V) = lim /(.v)=/(A,)

Also, since ^(.v,) is a constant.

2im^g(.v,)=^(.r,)

It follows that

/''(•vi)=/(.v,)-g'(.vi)+/'(.v,)-g(.v,)

THEOREM 5 Homogeneous Rule

Let 5 be a function that is differentiable at the number x, and let c be aconstant


,

Let the function h be defined by htx) = cg(x). Then h is differentiable at .t, and

^''(Vl) = Cg'i.Xi)

PROOF Let/be the constant function defined =


by/(.v) c. By Theorem l,/'(v)>
= Evi-
dently, j ^

'>M = cg{x)=f(x}-g(x)
• Therefore, by Theorem 4,

h'i.xi) =/(A-,)-g'(.v,) +/'(.v,)-g(.v,) = cg'(xi) + 0-g(xO = cg'{xi)


106 CHAPTKR I THE DERIVATIVE

THEOREM 6 Power Rule

Let n be an integer greater than 1 , and let/be the function defined by /(.v) = x"
Then / is differentiable at every nuinber x. and /' is the function defined by

fix) = /a" '

The proof proceeds by mathematical induction.* starting with n = 2. For /i = 2 we


have, by the product and identity rules.

/"(A) = D,.v- = D,(.v.v) = .v(D,.v) + (D,.v).v

= .v + .V = 2v = Ir-"'

Hence, the theorem holds when n = 2. Now. if we assume that n is greater than 2
and that the theorem holds for exponents less than n. Theorems 4 and 2 imply that

f'(x) = D,x" = D,(.v""' -A) = .r"~'(D,.v) + (D,a"'')a


= A-""' + ((/; - 1)a""-]a = a"~' + (n- Da"^'

THEOREM 7 Reciprocal
2.3 BASIC ALGEBRAIC RILES FOR DIFFERENTUTION 107

Since g is continuous at .V|

^im^ g(.vi + A.V) = Im g(.x) = g(.v,)

Thus,

1
g'Ui)
ll'{.Xi) = (-\)g'{.\l)'

THEOREM 8 Quotient Rule

Let /and ^? be functions both of which are differentiable at the number .v, , and
suppose that g(A , ) # 0. Then, if h = f/g. it follows that h is differentiable at Xi
and

g(.Vl)-/'(.Vl)-/(.V,)-g'(.V|)
/;'(-v,) =
[g(-v,)]-

PROOF Note that h =f- il/g): hence, by the product and reciprocal rules,

h\x,)=f{xi)- "^'-''i +/'(.Yi)-


[g(-xi)]- gixi)
-/(.V|)-g'(.Y,) ^ g(A-, ) -/'(.V, )

[g(-v.)]' [g(-v,)]-

(.vi)-/'(.vi)-/(.vi)-g'(,ri)

THEOREM 9 Power Rule for Integer Exponents

If the function /is defined by/(.v) = a", where /; is any fixed integer, then /is
differentiable and

fix) = nx"^

Here we understand that

(i) If ;; s 0. then x can be any number except 0.

(ii) If /; = 1. we interpret O" as being the number 1 (for purposes of this


theorem only).

Theorem 6 takes care of the case /! > 2. while Theorem 2 takes care of the case
/! = 1 .
= 0. a" = a" = (except for x = 0); hence, for x 7^ and n = 0,
For ;; 1

fix) = D,x" = D,l = = 0-.v~' = O-a"-' = nx"-\ as desired. Finally, sup-


pose that n < and that x ¥ 0. Note that -n is a positive integer; hence, by what
was already proved for positive exponents and by Theorem 7,

D,A""
/'(A-) = D,.r" = D,(^) =
108 CHAPTER 2 THE DERIVATIVE

Problem Set 2.3

In Problems
basic rules. Rules
1 to 40, differentiate

1 through 10.
each function by applying the
39 fix) =
/

Vr +
3a +
2
I \

'
(A + ^
7)
^„
40 p(x) = —Va
A + 1
-7=-

41 Find /'(2) in each case.


= yAr'
1 /(.v) = 3.r 2 g(x)
(a)/(.v) = W- 1 (b)/(A) = (l/.r')- 1

= - = (— +
3 h(x) = -S.r" 4 GU) = —' 8 ,, (c) fix) {X- + 1)(1 A) (d) fix)
2) ( 1^

4v^ (e) fix) = — (f) fix)


5 F{y) = 6 //(v) X- + 2 x+1
30
42 Suppose that/, g. and h are differentiable functions. Let k be a
7 //(r) = 5? - 7 8 G(vv) = V3(7 - 8vv)
= fix)
function defined by <:(a) • g{x) •
h(x). Use the product rule

to show that
9 f(.x) = a' - 3.r^ + I 10 fix) = —a'' - 9./
6 k'(x) = fix) g(x) h'ix) + fix) g '(A) •
h(x) + fix) g(x) h(x)

11 /(A) = — + ^+ 6 12 f (.t) = —-^+4 3


1
43 Use the
tions.
result of Problem 42 to differentiate the following func-

- = + 7/+ (a) fix) = (It - 5)(A + 2)(A- -


13 /(n = + + 3/- I
'" 2/^ 3/ 1 14/(/) 17

(b)/(.v) = (l - 3.v)=(2r + 5)

»5^^-^> = -^ +
i 16/0 = ^-^^+1
2r = (4- + - -
3r' (c) fix) 1) (3a 1)(A- 3.t)

17/(v) = 4--— 18/(«) = v;; (d) fix) = ax- + 1?

44 Using the sum and the homogeneous rules, show that the deriva-

19 g(x) = 3a"- - 7a"' + 6Va tive of a difference is the difference of the derivatives: that is,

D,(u - V) = D,ii - D,y.


20 G(A) = —A"-' a"- + -^^
3 2 Va 45 Let / and ^1; be differentiable functions at the number 1 , and let

/(I) = I, /'(I) = 2, g(l) = k. ands'(l) = -3. Use the differ-

21 fix) = - + -^ 22 fix) = \G{x^ - A) entiation rules to find


5a 3a^ Va (b)(/-s)'(l) (c) (2/+3g)'(I)
(a) (/+ sj'd)
23 F(x) = a-(3a' - = + +
1)

-
24 fix)

=
(a-

-
l)(2r' 5)

-
(d) {fg)'(\) (e) l-^l
(f)'
(1) (f) ^ g
(1)

25 G(A) = (A- + 3a)(a' 9a) 26 g(x) (3a a-)(3a-' 4)


46 Suppose that/, g. and h are differentiable functions at the num-
27 /(y) = V^(4y' + 7) 2% fit) = (6f- + 7)- ber 2, and let fil) = -2, /'(2) = 3, g(2) = -5, g'{2) = I,
h{2) = 2, and h'{2) = 4. Use the differentiation rules to find
= (a'-8)(y- 30/(A)=(i- +
29/(a) i)
3)(| + 7) (a) {f+g + /i)'(2) (b) (2/- g + 3/i)'(2)

(c) {/ghy{2) (d)


[y) (2)
31,(A) = (-i^ +
3)(Jr + .v)

47 Find the slope of the tangent line to the graph of the function /
at the point whose a coordinate is 4.

3
(a) fix) = X- - 4a- - I lb) fix)
= r-
It + 7 3a^ — I
33 fix) =
4a"
34 =
fix)
3a - 1 A- 2
48 Determine the rate of change of volume with respect to the ra-
2x- +A + 1
/-'
dius (a) of a sphere and (b) of a right circular cylinder with fixed
^^^''-^= 36 G{/) =
a^-3a.2 2/" + 5 height h.

3r + 7 A- - 19 49 Find the slope of the tangent line to the graph of fix) =


37 FU) 38 fix) = -
/- - 1 A- + 19 a/(a' 2) at the point (1, -1).
2.4 TANGE?>JT AND NORMAL LINES 109

50 For a thin lens of constant focal length p. the object distance .v is correct, find the rate of change dN/dt of the population 20
and the image distance v are related by (l/.v) + (1/y) = \/p years from now. {Note: Here N really isn"t a continuous varia-

(Figure 1 ). (a) Solve for y in terms of v and p. (b) Find the rate of ble, but we treat it as if it were, in order to obtain an approxima-
change of v with respect to x. tion to the rate at which the population will be declining in 20
years.)
Figure 1
56 The cost C (in dollars) to a refinery for refining .v million
gallons of gasoline in 1 month is given by the equation
C= 120.000 + 200.000.V - 16.00G.r. Find the rate of change
dC/dx of C with respect to x when .v = 4.5 million gallons.

1 57 Environmentalists determine that if untreated sewage is dis-

charged into a lake, it will decrease the amount A of dissolved


oxygen in moles per cubic meter of water in such a way that

t +
51 Criticize the following erroneous argument: We wish to compute 14.4
I- + 16/ -I- 65'
the value of the derivative of/(.v) + = 2v" 3.v — at 1 = 2. To
.v

this end, we put x = 2 and we have/(2) = 2(2)" + 3(2) - 1


= where i is the number of days the sewage has been flowing into

13. But D,13 = 0. so/'(2) = 0. the lake. Find the rate of change dA/dt of dissolved oxygen per
dav when / = 4 davs.
52 Show that the reciprocal rule is a special case of the quotient rule
when the numerator is the constant function /(.v) = 1

53 An object is moving along a straight line in such a way that at the

end of t seconds its distance s in feet from the starting point is

given by s = 8r + (2/0. with t > 0. Find the speed of the object


at the instant when i = 2 seconds.

3 54 In a certain electric circuit, the electric power P (in watts) is

related to the load resistance R P =


(in ohms) by the equation
100/?(0.5 + R)~- for /? > 0. Find the rate of change dP/dR of
the power P with respect to the resistance R when /? = 10 ohms.

55 Wildlife biologists predict that the population N of a certain en-


dangered species after t years will be given by the equation
N= {2,t+ 150)(50 - /) for </< 50 years. If this prediction

2.4 Tangent and Normal Lines


Suppose that the function/ is differentiable at .v, , so that/'(.Vi ) is the slope of the
tangent line to the graph of/ at the point (.r, ./(.Ti )). If y, = /(.Xi ). an equation of this
Figure 1 tangent line in point-slope form is

.*' - yt =/'(-vi)(.v --vi)

EXAMPLE 1 Find an equation of the tangent line to the graph of /(.v) = 4 - .v" at

the point (I. 3). Sketch the graph.


f(x) = 4-x-
SOLUTION Here,/'(.v) = -Iv, so that /'(I) = -2. An equation of the tangent
line is

y - 3 = -2(.\ - 1) or y = -Zv + 5

(See Figure 1.)


110 CHAPTER 2 THE DERIVATIVE

The normal line to the graph of /at the point (jC), Vi) is defined to be the line
through (.V|. yi ) that is perpendicular to the tangent line at (.V|, y, ) (Figure 2). Since
/'{.vi) is the slope of the tangent line to the graph of /at (.V), Vi) it follows from
Theorem 3 in Section 1.3 that — l//'(.Vi ) is the slope of the normal line at Ui, Vi).
Consequently, an equation of the normal line in point-slope form is

/ (-V| )

Figure 2

EXAMPLE 2 Find the equations of the tangent and normal lines to the graph of
= 1/x
fix) at the point (i, 2). Illustrate graphically.

SOLUTION Here,/'(jr) = -\/.x~, so that/'(2) = -1/(1)- = -4. Therefore, an


equation of the tangent line is

-4(.v - I) V = -4.V + 4

Since the tangent line at (1. 2) has slope -4, the normal line at this point has slope
— 1/(— 4) = 5. Hence, an equation of the normal line is

-i('-i) V = —+
4

(See Figure 3.)

Figure 3
2.4 TANGENT AND NORNUL LINES 111

It is sometimes useful to be able to find the point (or points) (.r, ,/(.t,)) on the
graph of a differentiable function /at which the tangent line has a prescribed direc-
tion. If the slope of a line in this direction is m. it is only necessary to solve the
equation /'(.v, )
= m for.v, in order to find the desired value (or values) of X]. This
technique is illustrated by the following example.

EXAMPLE 3 If/(.Y) = Zv- -


-v. find the point on the graph of/ where the tangent
line is parallel to the line 3.v - y - 4 = 0, find an equation of the tangent line at this
point, and sketch the graph.

SOLUTION Here, fix) = 4.v - 1 . The line 3.v - v - 4 = has slope 3: hence,
the .V coordinate .Vi of the desired point must satisfy the equation/'(.x, ) = 3; that is,
4.vi - 1 = 3. It follows that .Vi = 1: hence, the desired point on the graph of/ is

given by (.v,,/(.v,)) = (1,/(1)) = (1, 1). An equation of the tangent line at (1 , 1) is

>•- 1 =3(.v- I) y = 3.t - 2

(See Figure 4.)

Figure 4

Horizontal Tangents
A tangent line with slope zero is called a horizontal tangent.

EXAMPLE 4 Find all points on the graph of /(.i) = x^ - 3.\- + 5 at which the
tangent line is horizontal.

SOLUTION The slope of the tangent line at (.v, /(.v)) is given by

fix) = 3.r^ - 6.V

The tangent line at the point (.v,/(.v)) is horizontal if and only if/'(.v) = 0: that is. if

3.V- - 6.Y = or 3.x(.v - 2) =


The solutions of the last equation are .v = and x = 2. Since

/(O) = 0' - 3(0)- + 5 = 5

and /(2) = 2^ - 3(2)- + 5=1


the graph of/ has horizontal tangents at the two points (0, 5) and (2, 1).
112 CHAPTER 2 THE DERIVATIVE

In Figure 5. we have sketched the graph of /(.v) = x^ — 3.xr + 5 and drawn the
horizontal tangents at (0, 5) and (2, 1 ). Notice that the point (0, 5) is at the "crest of
a hill," and the point (2, 1) is at the "bottom of a valley" on this graph. Thus, the
point (0, 5) is higher than all its immediate neighboring points on the graph, al-

though the graph eventually climbs even higher. A point such as (0, 5) is called a
relative maximum point of the graph of/. Similarly, a point such as (2, 1) that is

lower than all its immediate neighboring points on the graph is called a relative
minimum point of the graph of/ (Figure 6). The following definitions apply to the
X coordinates (abscissas) of such points.

Figure 5 Figure 6
lion/ontal
2.4 TANGENT AND NOR.NLAL LINTS 113

Using Property 14 on page 64. we can rewrite this limit in the alternative form

/ (c) = hm

We must prove/'(c) = 0. If we prove that/'(c) can be neither positive nor negative,


then must be zero, and our argument will be complete. We prove that /'(c) is not
it

negative by showing that if it were, then, contrar)' to hypothesis. /could not have a
relative maximum at c. [That /'(c) is not positive can be shown similarly.] Thus,
suppose /'(c) is negative. Since we can make [/(.v) - f(c)]/(x - c) as close as we
please to the negative number/'(c) by taking .v sufficiently close to c (but not equal
to c). there is a small open interval / containing c such that \J(x) - f(c)]/(x - c) is

negative if x is different from c and belongs to /.

If .V belonas to / and v < c. then .v — c < and

f(x) -f(c)
<0
But, if a fraction and its denominator are both negative, then the numerator must be
positive; hence,

if X belongs to / and .v < c then /(.r) > f(c)


The last statement contradicts the hypothesis that / has a relative maximum at c,
since it says that the function values /(.v) are larger than /(c) for values of .v slightly
to the left of c. This is the promised contradiction, and the proof is complete.

We study relative extrema in more detail in Sections 3.2 and 3.3. As you can see,
horizontal tangents will have an important role to play in this coimection.

Problem Set 2.4

In Problems 1 to 14, find the equations of the tangent and normal H rlx) at (2, 3)
lines to the graph of the function at the indicated point. Illustrate
graphically in Problems 1 to 5. + + .r-
., „ = I .T
12 R(x) T at (1, 1)

1 /(.r) = 2r- 7 at (2, 1)

2 F(x) = 5 + 2v - .X- at (0. 5) 13 s(x) = at (4. 4)


Vx- 1

3 g(x) =.r^ + .X+ 1 at (1. 3)


14 5(.r) = arr + bx + c at (0. c)
4 G(x) = (.r- 1)- at (1, 0)
15 Find the point where the tangent line to the graph of/(.t) = 2Vx
5 ;i(.v) at (3. 1) 6 Hix) = xVx at (9, 27) at (1. 2) crosses (a) the .t axis and (b) the y axis.

16 Find the point where the normal line to the graph of/(.r) = 2/x at
7 pix) = x^ - 8.\- + 9.T + 20 at (4. -8)
(1, 2) crosses (a) the x axis and (b) the y axis.

2
8 P(x) =x + —^ at (4. 5)
17 At what point on the curve y = .ir + 8 is the slope of the tangent
Vx line 16? Write an equation for this tangent line.

9 q(x) = 4.r^ - 4.1^ - 25.r^ + v + 6 at (3. 0) 18 Suppose that the function / is differentiable at a and that
/'(a) # 0. Show that the tangent line to the graph of /at the
10 Q{x) at (0. II point {a,f(a)) intersects the x axis at the point « ith v coordinate
a - U(a)/f'(a)].
114 CHAPTER 2 THE DERIVATIVE

In Problems 19 to 24. find a point on the graph of the given function 31 ^(.r) = iVx - X + \

where the tangent or normal Hne satisfies the indicated condition,


.r + 2v - 1
and then write an equation for this Hne. 32 Q{x) =
.V - 3

19 The tangent line to /(.v) = x - .\- is parallel to the line

.t + V - 2 = 0.
33 r(.v) = —
20 The tangent line to f{x) = Zx^ - x~ is parallel to the line
34 «(.v) = av' + hx- + ex + d
4.V - V + 3 = 0.

21 The nomial line to = xG is parallel to the line 35 Determine a value of the constant b so that the graph of
fix)
4.t + y - 4 = 0. y = x'^ + bx + 17 has a horizontal tangent at (2, 21 + lb).

22 The nomial line to J\x) = .v - (l/.v) is parallel to the line 36 Complete the proof of Theorem I by considering the case in

X + 2v -3 = 0. which / has a relative minimum at c.

23 The tangent line to/(.v) = 5 + .v" intersects the v axis at the point 37 Let /(.v) = A" Show that the graph of/ has a horizontal tan-
. (a)

(2. 0). gent at (0, 0). (b) Show that/has neither a relative maximum nor
a relative minimum at 0. (c) Explain why the results in (a) and
24 The normal line to /(.v) = 3.r + Iv + 1 contains the point
(b) do not contradict Theorem I

(9, 5).
38 In Section 1.5,we showed that the vertex of the parabola

In Problems 25 to 34, find all points on the graph of the function at fix) = axr + hx + c is
which the tangent line is horizontal.

25 fix) = ix - 3)(.v - 2) 26 g(x) = x + .r'


(-i-'(-i))
Derive this same result by using Theorem I
27 F(x) = 3.V- + 5.r + 6 28 G(.v) = x' - 6.r + 9.r + 4
39 If the graph of a function / has a horizontal tangent at the point
29 /,(., + Ir - 5.V - I 30 Hix) = .r"' + x~~ (r, /(<)), what is the equation of the normal line at this point?

2.5 Rules for Differentiating


Trigonometric Functions
In Section 1.8, we showed that

sin A- 1 - cos X
lim and lim =

(See Theorem 2, page 66, and Theorem 3, page 67.) Using these two special limits,
we can prove the following theorem, which provides a rule for the derivative of the
sine function.

THEOREM 1 Derivative of the Sine Function

The sine function is differentiable at each real number, and the derivative of the
sine function is the cosine function. In symbols,

d
=
.

D, sin .V = cos v sin .V cos .V


dx

i
2.5 RILES FOR DIFFERENTUTING TRIGONOMETRIC FINCTIONS 115

sin (.V + Ax) - sin x


D. sin .V = lim
A.< -.0

We have sin (.v + A.r) = sin v cos Ax + sin A.v cos .v by the addition formula for
sine; hence.

sin X cos A.V + sin A.v cos x — sin x


D, sin X = Hm
Ax
(cos A.V — 1 ) sin A.v cos x
- +
A.r Aa
1 — cos A.V sin A.v
hm ,

( -sm .v) 1-
cos .v
Ax^U L A.V A.V

1
- cos A.V sin Ax
= (-sm .v) hm ; 1-
cos x
.

hm
Xx^o Ax Ajt^o Ax
= (-sin .v)(0) + (cos .v)(l) = cos .r

EXAMPLE 1 Differentiate /(.v) = a- sin x.

SOLUTION By the product rule, the power rule, and Theorem 1,

fix) = D,(.ir sin a) = (D,A-)(sin .v) + a-(0, sin x)


= Ix sin X + x^ cos .v

Figure 1 shows the graph of the sine function and its derivative, the cosine
function, on the same coordinate system. Because

cos X = sm (-")
A
I H

the graph of y = cos a ""lags behind" the graph of y = sin a by tt/2 units, although
both graphs have exactly the same shape. The fact that the cosine function is the
derivative (rate of change) of the sine function is nicely illustrated by Figure 1. For
instance, notice that when the sine function is increasing most rapidly — namely at
A = 0, ±277, ±477, and so on — its derivative, the cosine function, takes on its

ma.ximum value 1. Likewise, when the sine function is decreasing most rapidly
namely at a = ±77. ±377, ±577, and so on — its derivative, the cosine function,
takes on its minimum value — 1

Figure 1
116 CHAPTER 2 THE DERIVATIVE

According to Theorem 1 in Section 2.4, when the sine function taices on maxi-
mum or minimum values, its derivative, the cosine function, should have the value
0. This is also confirmed by the graphs in Figure 1. Indeed, when x = ±7r/2,

±37r/2, ±57r/2, and so on, sin .x takes on maximum or minimum values ± 1 and ,

cos X = 0.

The rule for the derivative of the cosine is established in Theorem 2, whose proof
is similar to the proof of Theorem 1 .

THEOREM 2 Derivative of the Cosine Function

The cosine function is differentiable at each real number, and the derivative of
the cosine function is the negative of the sine function. In symbols.

D, cos .V = —sin x cos .V = —sin X


dx

COS (.V + Ax) — cos X


D, cos X = lim

We have cos (.v + A.v) = cos x cos Ax — sin .v sin A.v by the addition formula for
cosine; hence,

cos X cos A.v - sin .v sin A.v — cos .v


D, cos .V = lim
Aa-^O A.V

r cos X (cos A.V - 1 ) sin .V sin A.v


= lim
A.v-^0 L A.V A.V

1
— cos A.v sin A.v
= lim ( — cos X )
:
sin .v
A.x-^O L A.v

1
— cos A.v sin A.v
= — cos
( x) lim sin x lim
A.v^o A.v A.v^o A.V

= (-cos.v)(0) - (sinx)(l)
= — sin .V

cos ;
EXAMPLE 2 Differentiate fiU)
r + 4
SOLUTION By the quotient rule and Theorem 2,

cos t (r + 4)(D, cos t) - (cos t)[D, (r + 4)]


g'U) = D,—
r + 4 (r + Ay
(?- + 4)(-sin t) - (cos t)(2t)

(f- + 4)-

-(f- + 4)(sin t) - 2t cos t

(t- + 4)-

Now the derivatives of the remaining trigonometric functions are easily obtained
by using the derivatives of the sine and cosine functions together with the differenti-
ation rules for quotients and reciprocals.
2.5 RULES FOR DIFFERENTIATING TRIGONOMETRIC FUNCTIONS 117

THEOREM 3 Derivatives of Tan, Cot, Sec, and Csc Functions

(i) O.
118 CHAPTKR 2 THE UERIVATIVE

KXAMPl.K 6 Find all values of .v with —it<x-^ it that are v coordinates of points
where the graph of v = sin .v — cos x has a horizontal tangent.

SOLUTION Here,

dy
dx
d
dx
,sin
_
X - cos a) = —
dx
sin x
dx
cos X = cos X sin x)

= cos .V + sin X

The condition for a horizontal tangent is dy/dx = 0; that is,

cos .V + sin .V = or sin .v = -cos x

Squaring both sides of the last equation, we obtain

sin" X = COS" X or sin" .v = 1 - sin" .v

that is.
2 sin- X = 1

Therefore,
V2
sin A- = ±
V2 2

The values of x between - n and tt that satisfy the last equation are

377 7T 77 377

4 '
4 " 4" 4

Because we squared both sides of an equation, we may have introduced extraneous


roots, so we must check each of these possible values of a in the original equation
cos X + sin A = 0. Doing so, we find that the only solutions are

3t7
and
4

The functions sin x and cos a are said to be cofunctions of each other. The same
terminology is applied to tan a and cot a as well as to sec a and esc a. As you can
see from Theorems 1, 2, and 3, the rule for differentiating a cofunction is obtained

from the rule for differentiating the corresponding function by changing function to

cofunction and introducing a negative sign. Thus, if you learn only the three rules

D, sin A = cos A

D, tan A = sec- a

D, sec V = sec v Ian v

you can easily recall the corresponding cofunction rules

D, cos
2.5 RULES FOR DIFFERENTIATING TRIGONOMETRIC FUNCTIONS 119

Problem Set 2.5

In Problems 1 to 30. differentiate the function

1 /(.v) = 7 sin A 2 g(.v) = —X cos X

3 hit) = 4 sin r - r cos / 4 F{r) = V7cos r

5 g(.v) = 3 tan x + sec x 6 G(i) = f' - 5 cot r

7 H(y) = 8 sec v - iy^ 8 f(z) = 4 CSC z - 3 sec z

9 g{r) r sin r + 4 esc r 10 H{u) = (\^ + 5)(cos u)

11 /(z) = cotz + Vi
120 CHAPIER 1 THE DERIVATIVE

42 Suppose that o is a constant, (a) Using the substitution property Figure 4


for limits, show that

sin (a A.v) cos (a A.v)


=
A.v

(b) Using the results of part (a) and arguments similar to those in

Theorems 1 and 2, show that

n. sin ax = a cos ax and /), cos ax = -a sin ax


1344 Because of seasonal variations in demand, the revenue /? to a
43 A tractor is pulling a heavy sledge weighing W newtons with a
swimsuit manufacturer on the .vth day of the year, starting with
cable making an angle with the horizontal (Figure 4). If /n is
.V = I on January I. is given by
the coefficient of sliding friction between the sledge and the
ground, then the tension T newtons in the cable is given by
R = 3800 - 1 800 cos dollars
365
^iW
T =
cos 6 + /Li sin f^ Find the rate of change dR/dx of revenue per day on June 1 when
X = 152. {Hint: Use the result in Problem 42b.)
Assuming that y, and W are constants, find a formula for the rate

of change dT/dO of the tension T with respect to the angle 6. 45 Complete the proof of Theorem 3 by proving parts (ii) and (iv).

2.6 Function Composition


We now introduce the idea of fiinciion composiiion; in Section 2.7 we use this idea
to develop an impoitant differentiation rule called the chain rule.

Suppose we have three variable quantities y. ii. and .v. If v depends on u and if

u. in turn, depends on .v, then clearly y depends on .v. In other words,


y is a if

function of u and w is a function of .v, then y is a function of .v. For instance, suppose
that

and u = 3.V

Then, substituting the value of ii from the second equation into the first equation,
we find that

Ox- \)~

More generally, if

y =/((/) and // = g(x)

then, substituting ii from the second equation into the first equation, we find that

.v=/(^(.v))

To avoid a pileup of parentheses, we often replace the outside parentheses in the last
equation by .square brackets and write

y=f\fiM\
If / and i; are functions, the equation y =/l,i;(.v)] defines a new function h:

hix) =f[g{x)]

The function h obtained by "chaining" /and g together in this way is called the
composiiion of /'and g and is written h =f°g- The idea of function composition is
made precise in the following definition.
2.6 FirNCTION COMPOSITION 121

DEFINITION 1 Composition of Functions

Let./ and be two functions satisfying the condition that at least


f;
one number in
the range of g belongs to the domain of/. Then the composition of/ and g. in
symbols /°g, is the function defined by the equation

(f°g)M=f[g(-x)]

Evidently, the domain of the composite function/" g is the set of all values
ofx
in the domain of g such that g{x) belongs to the domain off. The
range of f°g is
just the set of all numbers of the form/[^?(.v)] as v runs
through the domain of/° g.

E^^^'PLE 1 Let/(.r) = 3.V - 1 and g(.v) = a-\ Find

(a) {fog)a) (b) (gof)(2)

(C) (/°g)(.V) (d) {gof)(x)

(e) (/°/)U) [3(f) (/og)(3.007)

SOLUTION

(a) if°g)i2) =f[g(2)] =/(2^) =/(8) = 3(8) - 1 = 23


(b) (g°/)(2) = glfil)] = g[3i2) - 1] = g(5) = 5^ = 125
(c) (f°g){x) =flgix)] =/(.r^) = 3.r' - 1

(d) {g°f){x) = glfix}] = gax - 1) = (3.V - 1)-'


(e) (/°/)(.v) =/L/t.v)] =/(3.v - 1) = 3(3.v - 1) - = -
1 9.v 4
(f) (/°g)(3.007) =/[g(3.007)] = 3(3.007)^' - I
- 80.57

EX.4MPLE 2 Let/(.v) = - g(x) = and h{x) = V^.


3.V 1 , .r\ Find (/o (g <>
h)](x).

SOLUTION

[f°ig°h)](x) =fHgoh)(x)] =f{glhM]} =/[g(\^)] =/[(V^)^]


= /(.r3/2) = 3x^/2- 1

EX.AMPLE 3 Let fix) = 3.V - 1. g(x) = x\ and p{x) = + Show


i(.v 1). that
f°(g + p) is not the same function as (f°g) + {f°p).
SOLUTION On the one hand,

[/° (g + p)]{x) =fUg+ p){x)] =f[gix) + p(x)] =/[.r' + i(x + 1)]


= 3[a-' + ^(.v+ D] - = 3.v3 +.V
1

On the other hand,

[(J°g) + (f°p)](x) = {f°g){x) + (fop)(x) =f[g(x)] +f[p{x)]


= /(^)+/[3(.v+ 1)]
= 3x^-1 +3[Rt+ 1)]- 1

= 3.r' +x- I

Therefore, the functions/" (g + ;,) and (fog) + (fop) are not the same.

Although the symbolism /°g for the composition of/ and


g looks \aguely like
some kind of a "product," you must not confuse it with the actual product
f-g
of / and g. Whereas
f-g = g •/, note (in Example 1 ) that f°g¥^ g °/
122 CHAPTER 2 THE DERIVATIVE

Whereas f-(g + p) = if-g) + if-p). note (in Example 3) that /"(g + p) #


(f°g) + (f°P)- 't does turn out, however, that function composition is associative;
that is.

fo(goh) = {fof,)ol,

(Problem 12). Because of the associative property of composition, parentheses


aren't really necessary when three or more functions are to be composed. Thus we
simply write/° g ° h.f° g° h° p. and so forth. Composing a function/with itself is

called iteration. The successive iterates oi f dst f ° f. f ° f ° f, f ° f ° f ° f. and so forth.


For an interesting account of the use of function iteration in building mathematical
models for phenomena in physics, engineering, economics, and the life sciences,
see Douglas R. Hofstadter. "Strange attractors; Mathematical patterns delicately
poised between order and chaos." Scientific American, Vol. 245. No. 5. pp. 22-
43. November 1981.
Using programmable calculator, you can see a vivid demonstration of function
a
composition. Suppose that the/ and g keys are programmed with functions of your
choice. If you enter a number .v and touch the g key. you see the mapping

.V I > g{x)

take place, and the number g(.v) appears in the display. Now. if you touch the/key.
the mapping

g(.v)i >f\gix)]

will take place. Therefore, after you enter the number .v. you can perform the
composite mapping

-VI >(/°g)(.v)
Figure I

by touching first the g key. then the f key. This fact is represented by the diagram in
Figure 1.

In using a programmable calculator or a computer, it is important to be able to tell

when a complicated function can be obtained as a composition of simpler functions.


.'l.i;(vi|
jj^g same skill is essential in calculus. If

h(x) = {fog)(x)=f[g(.x)]

lefs agree to call g the inside function and /the outside function because of the
positions they occupy in the expression /[?(.v)]. In order to see that h can be ob-
tained as a composition /; = f° g. you must be able to recognize the inside function

g and the outside function /in the equation that defines h.

EXAMPLE 4 Express the function /i(.v) = (3.v + 2)- as a composition h =f°g of


two functions / and g.

SOLUTION Here we can take the inside function to be gi.x) = 3.v + 2 and the
outside function to be/(.v) = a". so that

(fog){.x) =f[gtx)] =/(3.v + 2) = (3.V + 2)- = hix)

The substitution property for limits (page 67) and the substitution property for
continuous functions (page 73) can be restated in terms of function composition.
2.6 FX'NCTION COMPOSITION 123

For instance, the siibstiiution property for continuous functions can be restated as
follows:

If/and g are functions, g is continuous at a, and/is continuous at g(a), then/° g


is continuous at a.

We leave it as an exercise for you to restate the substitution property for limits in
terms of function composition (Problem 44).

Problem Set 2.6

In Problems 1 to 10. let/(.v) = .v - 3 and gix) = .v" + 4. Find the 3.V - 1


23 fix)
indicated value.

24 fix) = 2. g(.v) = 7
1 (/°g)(4) 2 (/°g)(V2)

[a 3 (^°/)(4.73) E 4 (go/H-^.OS)
= = - = \
In Problems 25 to 32, let/U) 4jt, g(.r) .v" 3. and /j(.v) .x.

5 (/»/)( 3) 6 (g<'gK-3) Express each function as a comfwsition of functions chosen from/,


g, and h.
7 If^igo/m) 8 [(/°g)°/](2)

9 ifogKx) 10 (go/IU) 25 f (A) = V.Y= - 3 26 GU) = (V^)- - 3

27 Hix) = iVx 28 A:(.v) = 4.V- - 12


11 Let/U) = sin x, gtx) = .r". and /;{.v) = cos .v. Find a formula
for the given function. 29 Qix) = 4Vx 30 ^(.v) = 16.V- - 3

(a) fog (b)g°/ {c)gog 31 r{x) = V^ 32 six) = x^ - 6.V- + 6


(d) go(f+h) (e) g =
(/A) (f ) (//A) « (/i//)

(g)/o(g<'/!) (h) (/°g)°/i as a composition


In Problems 33 to 40. express each function /;

h = /° g of two simpler functions / and g.


12 Show that function composition is associative; that is. show that
fo(goij) = (jog)oij for any three functions/, g. and h (pro-
34 /!(.v) = sin (.v'' + 1)
vided, of course, that either side of the equation is defined).

36 /!(.v) = 5 CSC |x|

In Problems 13 to 24. find (a) (/°g)U). (b) (g°/)(.v). and (c)


1
(/°/)(-v).
38 /!(.v) =
(4.V + 5)'

13 /(,v) = .r, g(.v) = V^


39 /i(;r) = 40 hi v) = V] - 1 V.V - 1

14 fix) =x^ + I. gtx) = \/x - I

15 /(.v) = tan.v. g{x) = Vx


41 Suppose that user-definable keys / and g on a programmable
16 fix) =x + x-\ g{x} = V.v- I
calculator are programmed so that f:x i > 3.r - 2 and
17 = = esc X g.x I > 5.V - 3. Draw mapping diagram (see Figure 1) to
a
/(.c) |x|, gtx)
show the effect of entering a number v and touching (a) first the
18 /W = lA, g(.v) = lA
g key. then the /key; (b) first the /key. then the g key.

19 /U) = .r - I. g(.v) = 1 + cos -v


42 Give examples of (a) a function/ for which/°/is not the same
as /•/ and (b) a function g for which g° g is the same as g g.
20 fix) = ax + = •
b. g(.v) ex + d
1
43 Explain why the function fix) = sin |.r| is continuous at x = 0.
21 fix) = . g(.v) Iv- 3
2v- 3 44 Restate the substitution property for limits (page 67) in terms of
function composition.
22 /(.v) = Ax + B on: + fc

Cx + D Cr + rf 45 Explain why the function g(.v) = sin(sin .v) is continuous on R.


124 CHAFIKR 2 THE DERIVATIVE

46 A baseball diamond is a square 90 feet on each side. Suppose a 48 Show how to modify the procedure in Problem 47 to find the
ball is hil directly toward third base at the rate of 50 feet per values /(.v), (/»/)(.<;). (/°/°/)(.v), (/°/°/°/)(.v). and so on of
second. Let y denote the distance in feet of the ball from first the successive iterates of/ at x. The set consisting of .v and these
base, let .v denote its distance from home plate, and let / denote values is called the orbit of x under the iterates of /. (See
the elapsed time in seconds since the ball was hit. Here y is a Douglas R. Hofstadter's article, referred to on page 122.)
function of .v. say y = /(.v). and .v is a function of i, say .v = gU)-
E149 In the life sciences, a simple recursive model for population
(a) Find formulas for /(.v) and s(r). (b) Find a formula for
= growth is a function / that predicts the population /(p) of an
(/m;)(0. (c) Explain why y (/"g^t).
organism I unit of time later if the current population is p. (For
47 Show that ij'° )>)(.\) can be obtained graphically as follows: Start instance, the unit of time might be the gestation period of the
at the point (v, 0) on the .v axis. Move vertically to the graph of organism.) Thus, according to this model, if/) is the current
g. then horizontally to the graph of y = .v, then vertically to the population, then /(/>) is the population 1 unit of time later,
graph of/, and finally horizontally to the point (0. y) on the ,/l/(p)l - if°f)(p) is the population 2 units of time later.

y axis (Figure 2). Conclude that y = (f°g)(.x). f[(.f°f)ip)\ = (/".f °./)(/') is the population 3 units of time later,
and so forth. One of the most popular simple recursive growth
models is the discrete logistic model /(/)) = Bp - Ap-. where
Figure 2 i
A and B are nonnegative constants depending on the birth and
death rates of the organism. Suppose that 8 = 2.6 and A = 0.08
for a certain organism in a particular habitat. Starting with p = 4
organisms, use the discrete logistic model to predict the popula-
tion 1, 2, 3, 4. 5. and 6 units of time later according to the
discrete logistic model.

ISI50 Draw an accurate graph of the function /(/>) = 2.6p - 0.08p-


for < p < 32.5. (U.se a calculator to help plot points.) (a) Use
the graphical procedure obtained in Problem 48 to work Problem
49. (b) Use the same graphical procedure, starting with several
different values of the initial population p. Can you draw any
(tentative) conclusions?

2.7 The Chain Rule


Suppose that y = (x~ + 5xf and we wish to find dy/cLx. One approach is to expand
(.V- + 5xf and then differentiate the resulting polynomial. Thus,

y = (X- + 5jr)-' = .v'' + IS.r'^ + 75.V-* + 125.V-'

and so = 6.V-' + 75.V-* + 300.V-' + 375.V-


dx

Another approach is to let

u = X' + 5x

so that ,, _ ,,3

Then,
dy
3(r and
dll
-- = —d -(.v-
,
+ 5.v) = 2v + 5
dii dx dx

from which we might conclude that

dx d\ dll
-- = -^ -— = 3/r(2.v
,
+ 5) = 3(.v-
,
+
,
5.v)-(2v + 5)
d\ du dx
= 6.V-' + 75.v-^ + 30av' + 375.V-
2.7 THE CHAIN RULE 125

The second approach produced the correct result, but there's a catch to it! The
expressions dy/dii and du/dx are just symbols for derivatives which the "numera-
in
tors'" and ""denominators" have not yet been given any separate meanings, so we
weren't really justified in supposing that

^v _ dy dii

dx dii dx

In fact, the legitimacy of this calculation is guaranteed by one of the most important
differentiation rules in calculus — the chain rule. Although we give a precise state-
ment and proof of the chain rule later (see Theorem 1), we begin with the following
informal version.

The Chain Rule


126 CHAriER 2 THK DERIVATIVE

The following crude but effective memory device shows the basic pattern of the
chain rule:

D /(whatever) = /'(whatever) •
D(whatever)

For instance, using this device with /as the square-root function, we would have

D Vwhatcvcr = ,
• D(whatcver)
2Vwhatever

Thus, the calculation in Example 2 could have been abbreviated as follows:

D.VrTT = ,
,

D,(.v- + 1

2V.V- + I

2V.V- + 1

V.x" + 1

The chain rule is often used to calculate derivatives of the form D,m", where u is

a differentiable function of .v and n is an integer. Thus, letting /(i<) = u", so that

/'(//) = hh"~'. we obtain the important formula

D,ii" = mi"' 'D,ii

In other words, we have the pattern

D(whatever)" = ^(whatever)""' D(whatever)

In Examples 3 to 6, fiiul the derivative of each function with the aid of the chain
rule.

EXAMPLE 3 f(x) = {x- + 5.Y)'""

SOLUTION f'(x) = D,(.V- + 5.V)""


= 100(.v- + 5.v)^^Dvtv- + 5.v)
= 100(.v- + 5.v)''''(2.v + 5)

EXAMPLE 4 Fix) =
Ox- \f

SOLUTION F'(.V) = D,— -


—r =)"*
D,{}A - I )
"*

(3.V 1

= -4(3a 1)"-*^'
D>(.\v -
-
1)
= -4(3.v- l)-\3)
= -12(3.v- 1)^'

EXAMPLE 5 ;i;(.v) = Sin- X i


SOLUTION ^'(.v) = Dv(sin v)' = 3(sin x)-(D^ sin .v)

= 3 sin- X cos .v
2.7 THE CHAIN RULE 127

EXAMPLE 6 G(0 = (/- + 6/)'"(l - 3?)""

G'U) =D,[U- + 60"'(1 -3/)'*]


= [D,{r + 60'°](1 - 3?)-* + {r + 6/)'"[D,(l - ^if]
= 10(r + btf[D,{r + 6?)](i - 3tf + (r^ + 6?)'°[4(1 - ^'f AC 3/)]
= 10(f- + 6tf{2t + 6)(1 - 3/)'* + it- + 6ty°[m - 3f)3(-3)]
= mt' + ttfilt + 6)(1 - Sr)-* - 12(r + 6/)'°(l - 3?)^
= 2(/2 + 6f)''(l - 3/)^[5(2f + 6)(1 - 30 - 6(f2 + 6r)]
= 2(t- + 6/Al - 3?)\-36f- - 116/ + 30)
= -4(?- + 6/)'(l - 3r)'(18r + 58/ - 15)

Sometimes in calculating derivatives, you have to use the chain rule repeatedly.

EXAMPLE 7 Find D^(V.r' - \f.

SOLUTION A first use of the chain rule gives

D^(V.v-' - 1)5 = 5(VV - O^D^Vx-' - 1

Now. to work out DrV.v'' - 1, we use the chain rule again. Thus.

D,(Vjr^ - \f = 5(V.r' - l)^D,\/?~~

= 5(V.r'- 1)-* -0,(.v'- 1)


2V^
= f(V.r' - \fOx-)
= ^ .r^( V.v^ - 1
)-'

If (/ is a differentiable function of .v. we can combine the chain rule with the rules
for differentiating the sine, tangent, and secant functions to obtain the following:

D, sin u = cos it D,«


Dv tan u = sec- u D^u
D, sec II = sec ii tan u Djt

Similar rules hold for the corresponding cofunctions:

D, cos u = —sin ii Dji


Dj cot i( = —CSC" II D^ii
Dv CSC (/ = -CSC u cot M D,/<

/« Examples 8 to 10. find the derivative of each function with the aid of the chain
rule.

EXAMPLE 8 /(.v) = sin (5.\-)

SOLUTION /'(A) = D, sin (5.V-) = cos (5.v-) D^5,r


= [cos (5.x-)](iar)
= lO.r cos (5.ir)
128 CHAPTER 2 THE DERIVATIVE

EXAMPLE 9

SOLUTION
2.7 THE CHAIN RILE 129

Now, using function composition, we can give a formal statement and proof of
the chain rule.

THEOREM 1 The Chain Rule

If is a differentiable function
g at the number .t, and/is a differentiable function
at the number g(.x,). then/og is a differentiable function at the number
.r, and

(/°.?)'(-Ti)=/'[g(.r,)]g'(.v,)

Usmg the substitution property


for limits (page 67) with =
.v .v, + A.v, we have
=
g(.r, +A.r)-g(.v,) gW-g(.v,)
g'ixO lim _

lim
jLt^O xx X - Xi
Let M, - g(.Y,). Then we also have

Likewise,

a°5)'U,) = bm ^f°SKx)~(fo,)(xO ^ j^ /[gW]-/[g(.t,)]


"""*"
^ -^i ''-*'"
x - .r,

provided that the limit exists. Now define a function Q by

if ;< 5=^ H|
21") = -^ H - Ui

if M = i/i

Then

I^.^^"^ = .}^JJ!,,~^-nu.^=rui.^.n
Also, for /< 5^ «,,

/(«) -/(«,) = e((0((/ -;/,


Note that the last equation holds even =
.f u u, . when it simplv reduces to =
Now, we substitute u = g(.v) in the last equation and divide both
sides by v - v,' to
obtain ' '

/[g(-v)]-./'[g(-V,)]
= Q[gix)]
M - 8(xi )

for AT 5^.r,. Therefore, by the multiplication property for limits.

flsixn-flgixQ]
jjnj
lim e[.?(.v)] lim ^'"'"g^-^'V

(/°g)'(.v,) = ^hm^ Q[g(x)]-g'(xO

provided that jim^ Qlg(.x)] exists. The proof


will be complete if we can show that
130 CHAPTER 2 THE DERIVATIVE

this limit is equal to/'[g(.V|)). Because g is differentiable at .V|, it follows from


Theorem 1 on page 96 that g is continuous at .V|. Therefore,

Jim g(x) = g(A|) = W|

and it follows from the substitution property for limits that

lini QlfiU)] = lim Qdi) = flfUx^n

Problem Set 2.7

In Problems I to 4, find the required derivative by using the chain 21 ^(.t) = V.v- + 2\ - 1

rule.

1 y = Vm, u = X- +X + I , find dy/dx.

2 y = «' - 2«'/-, u = x' + Ix. find dy/dx.

3 y = u~\ u=x-* + \. find dy/dx.

4 y = H. M = (7 - .v-)(7 + A-) '. find D,y.

in Problems 5 to 62, find the derivative of each function with the aid
of the chain rule.

5 fix) = (5 - Iv)'"

6 fix) = {2x- 3)"

'^'•^"=(4y+l,'

8 Fit) = (2r^-/+ 1)"^

9 gix) = (3.v= + 7)=(5 - 3.xf

10 G(o = (5r + l)-(3r-' + 2)-'

11. /'(.v) = ('-tJ (6.V- I)'

12/(0 = (3/- l)-'{2r + 5)^'

13,?(y) = (7y + 3)--(2y- I)"*

14 /(H) = (6m +
^
—M )
'
(2m - 2)'

l9/(.r) = -!^ = (V^r' 20 F(x)


V.1C
2.7 THE CHAIN RULE 131

sec- 3f
59 git)
--
132 CHAPTKR 2 THK DERIVATIVE

2.8 Implicit Functions and


Implicit Differentiation

An equation such as

V = 3jt^ - 5.V + 12

that is already solved for y in terms of .v is said to give y explicitly as a function of


X. On the other hand, an equation such as

.ry + 1 = 2.V — >>

which can be solved for y in terms of x but is not solved for y as it stands, is said
to give y implicitly as a function of .v. (Shortly, we shall give a more precise
definition of an implicit function.) In this section we introduce a technique called
implicit differentiation which allows us to calculate the derivative dy/dx of an im-
plicitly given function without bothering to solve explicitly for y in terms of .v.

Implicit Differentiation

The equation 2v + 3y = 1 can be solved fory in terms of x to give y = 3 — f.v, and


so we have dy/dx = — § The same result can be obtained directly from the original
equation 2x + 3y = 1 simply by differentiating both sides, term by term, to get
2 + Mdy/dx) = and then solving for dy/dx = -ff The latter technique
. is called
implicit differentiation. More generally, we have the following.

Procedure for Implicit Differentiation

Given an equation that determines y implicitly as a differentiable function of x,


calculate dy/dx as follows:

Step 1 Differentiate both sides of the equation with respect to x\ that is, apply
d/d\ to both sides of the equation, term by term. In so doing, keep in mind that
y is regarded as a function of .v, and use the chain rule when necessary to
differentiate expressions involving y.

Step 2 The result of step 1 will be an equation involving not only .v and y but
also dy/dx. Solve this equation for the desired derivative dy/dx.

When the procedure for implicit differentiation is executed, the result is often an
equation that gives dy/dx in terms of both x and y. In this case, in order to calculate
the numerical value of dy/dx. it is necessary to know not only the numerical value
of X but also the numerical value of y.
The procedure for implicit differentiation can be used legitimately only if it is

known that the equation in question really does determine y implicitly as a differen-
tiable function of .v. However, in what follows we routinely apply the procedure and
simply assume that this requirement is fulfilled.

In Examples I to 5. use implicit differentiation to solve each problem.

EXAMPLE 1 If .v' - 3.vy -I- 4y^ = 6x + 1 , find dy/dx.


2.8 IMPLICIT FUNCTIONS AND IMPLICIT DIFFERENTIATION 133

SOLUTION We begin by differentiating both sides of the given equation, term


by
term, with respect to .v:

—x^
ax
- — 3.ry + —
dx dx
4v-^ = _6.x +
dx

dx
1

3x^ - 3(2v)v^ - 3.v2(4v^


^
^)
dx
+ 12v- ^= dx
6 +

3.V- - 6.n-^ - llv\-^ ^+


dx
12v- ^= dx
6

This completes step 1 of the procedure for implicit differentiation. To carry out
step 2, we must solve the last equation for dy/dx in terms of ;r and
y. Subtracting
3x - 6xy^ from both sides of the equation, we obtain

-llxy -j- + I2v- -- = -3.Y- + dn-* + 6


dx dx

Now, factoring dy/dx out of the left side and multiplying both
sides of the equation
by - 1 , we have

(\ZxY - 12v-) ^=
dx
3.V- - 6.n'-* - 6

"^^ 3x'- - 6.xy^ ~ 6 .r^ - In-^ - 2


Hence
dx llr-v^ - 12v- 4y\x-y - 1)

EXAMPLE 2 U(2x + yf - (3.V - .v)^ = y-\ find dy/dx.

SOLUTION Differentiating the equation term by term with respect to .v, we ob-
tain

3(2v + y)' —
dx
(Iv + y) - 3(3.r - v)-
'
-^{3x -
dx
v) = 5v''
^ dx

dy^

3<2..,,=(2.f)-3,3,^,,=(3-f).5,- dx
Thus,

6(2v + yr- + 3(2v + v)- -~ -


dr
9(3.v - v)- + 3(3.v - v)-
-
^= ^
dx
5v^
- dx
Rewriting the last equation so that the terms involving dy/dx are on the left, we have

3(2x + yf -^ + 3(3.v - yf -^ - 5y' -£ = 9(3.v - y)^ - 6(2v + yf

[3(2v + yf- + 3(3.r - yf - 5y*] -^ = 9(3.v - yf - 6(2r + yf

Therefore.

dy _ 9(3j: - yf - 6(2c + yf
dx 3(2x + yf + 3(3.r - yf - Sy"*
134 CHAPTER 2 THE DERIVATIVE

EXAMPLE 3 If X COS v + V sin A^ = 5, find dy/dx.

SOLUTION
—d
dx
(x COS v) H
d
dx
(v sin
'
.

.v)
d
= -—5^
dx

cos y. + X —
dx
d
cos V
'
H —^dx
dx
sin
.

x + v
'
— sm
dx
d .

.v = ^

dv dv
cos y — X sin v -^- H — sin
.

.v + v cos v =
dx dx
dv
(sin X — X sin v) —^ = —v cos x — cos y
dx

Multiplying both sides of the last equation by - 1 and solving for dy/dx. we obtain

dy y cos .v + cos y

dx X sin y — sin .v

EXAMPLE 4 Find the equations of the tangent and normal lines to the graph of
X IT
— + ^- = 6 at the point (2. 1).
y V'

SOLUTION Differentiating the given equation term by term with respect to x. we


obtain
2.8 IMPLICIT FUNCTIONS AND IMPLICIT DIFFERENTIATION 135

Consequently, the slope of the tangent line at (2, 1) is i , the slope of the normal line

is —1/5 = —2, and the corresponding equations are

Tangent line: y - I = ^(x — 2) or y = \x

Normal line: y — \ = —2{x — 2) or y = —2x + 5

EXAMPLE Given the equation x^ + xy + y^


5 = 2, considers: to be a differentia-
ble function of v and find dx/dy.

SOLUTION Applying d/dy to both sides of x' + rv + y^ = 2, we obtain

3x^
,
—+ +—
dx
dy
x
dx
dy
V + 3y-
T
=

Solving this equation for dx/dy, we have

dx _ —X — 7>y-
"
dy 2,x^-\-y

The idea of an implicit function is made more precise by the following definition.

DEFINITION 1 Implicit Function

A continuous function /, defined at least on an open interval, is said to be


implicit in an equation involving the variables x and >' provided that when y is

replaced by /(a) in this equation, the resulting equation is true for all values of .v
in the domain of/.

Figure 1 Thus, a continuous function /is implicit in an equation if and only if the graph of/
is contained in the graph of the equation (Figure 1).

graph of the equation


In Examples 6 to 8, if possible, find the functions implicit in the given equation by
solving for y in terms of x.

EXAMPLE 6 7 +X= y^ - 3y

SOLUTION We havey^ - 3y + (-7 - x) = 0. Using the quadratic formula, we


obtain

graph of an
implicit function/ =
3± V(-3f-4{l){-7-x)
y
2(1)

3± V4x + 37

The last equation does not give y as an explicit function of x because of the ambigu-
ous ± sign. However, the function / defined by

= 3 + y/4x + 37
fix)

is implicit in the given equation. But so is the function h defined by

3 - V4jc + 37
h(x) =
136 CHAPTER 2 THE DERIVATIVE

Figure 2 Figure 2 shows us geometrically just what's happening here. Indeed, the graph of
7 + j: = y^ — 3y is a parabola with a horizontal axis of symmetry. By the vertical-
line test (page 26), this parabola is not the graph of a function. However, it can be
broken up into two pieces, each of which is the graph of a function. The graph of
the implicit function/is the top half of the parabola, and the graph of /i is the bottom
half.

EXAMPLE 7 x- + V- + 1 =

SOLUTION This equation has no (real) solution, so it cannot define y implicitly


as a function of x.

Figure 3
2.8 IMPLICIT FUNCTIONS AND IMPLICIT DIFFERENTIATION 137

Problem Set 2.8

In Problems 1 to 26, find dy/dx by using implicit differentiation. 39

2 4xy^ + 3.r-y = 2

A xy'^ + x^ + y^ = 5

6 jrv' + 2v^ = X- — Ay-

i X- - Vxy - v =

10 vVr + Vv = 9

12 (Ax - \)^ = 5y^ + 2

14 (5;v-v + A)' = x^

16 + -=4
17 y = sin (Zr + y) 18 .V cos y = (.V + y)-

19 tan -vy + .vy = 2 20 tan" X + tan" \' = 4


21 sin" .t + COS" y = 1

22 sec (.V + y) + esc U + y) = 5

23 cos .vy + y- = 2 24 sin .V tan y = y'

25 cot Ox + y) = 5jy 26 sec (.V- + y-) = 5.v'

In Problems 27 to 32, find the equations of the tangent and normal


lines to the graph of the implicit function determined by each of the
following equations at the given point.

27 X- + xy + 2y- = 28 at (2, 3)

28 x^ - 3.ty2 + y^= 1 at (2, -1)

29 Vlx + V3y = 5 at (2, 3)

30 x^ - iVxy-y- = 52 at (8, 2)

31 sin .xy = y at (7r/2, 1)

32 (5 + tan xy)- = 36 at (7r/12, 3)

In Problems 33 to 36, consider.* to be a function of y and find dx/dy


by implicit differentiation.

33 3.r- + 5xy = l 34 x-y- = .V' + y''

35 r2 = v2 - V 36 Vxy + .rv" = 5

In Problems 37 to 46, find all functions implicit in each equation by


solving for y in terms of x.

37 5jc - 4y = 6

38 5jc- - 4y- = 6
138 CHAPTER 2 THE DERIVATIVE

53 A civil engineer is designing a curved access road leading from a Figure 5


point O on a state highway to a point P on an interstate high-
way. She sets up a Cartesian coordinate system with O as the
origin and the state highway along the .v axis (Figure 5). With
distances in meters. P = {150. 100). The access road is to be

tangent to the state highway at O and tangent to the interstate

highway at P. To provide for acceleration up to interstate high-

way speeds, the curvature of the access road is to decrease

uniformly from O to P. To meet these conditions, the access


road is designed to follow the graph of the equation
4.v= + 3(y - 200)- = 120.000. Find the x coordinate of the
point / where the interstate highway intersects the state highway.

2.9 The Rational-Power Rule


In Theorem 9 of Section 2.3, we established the power rule

d
dx

for integer exponents. In this section, we show that the same rule works for rational
exponents. We begin by considering the case in which the exponent is the reciprocal
Figure I of a positive integer.
Recall that if n is a positive integer, then

where Vx denotes the principal /nh root of Thus, if is even and x is nonnega- .v. /t

tive, Wx denotes the nonnegative number whose «th power is Similarly, if n is .v.

odd, Va denotes the real number whose nth power is but there is no restriction on .v,

the algebraic sign of .v.

If the positive integer n is even, then the graph of y = V.v has the general appear-
ance shown in Figure la; but if n is odd, the graph looks like the curve in Figure lb.
In either case, it is visually apparent that the graph of y = V.v has a definite tangent
line at every point, except perhaps at (0. 0).* If .v t' and there is a tangent line at
the point (x, Vx), then its slope must be given by the derivative dvx/dx. Thus,
Figure provides graphical evidence that dVx/dx exists, provided that Vx is de-
fined and
1

.V 7^ 0. In what follows, we shall assume this. A more formal argument I


can be given by using Theorem 1 on page 406.
Now, suppose that n is a positive integer, and let

y = Vx
Then,
i
*In fact, for n odd. the graph does have a tangent line at (0. 0). namely, the y axis (see page
201).
2.9 THE RATIONAL-POWER RLXE 139

Applying the implicit differentiation procedure to the last equation and assuming
that-v 7^ 0, we have

_d_ „__d_
dx ' dx

or ny"
^
,
' ^=
dx
dx
1

Therefore,

dy
dx m" ' nv"

Because y = \^ and \-" = x. the last equation can be rewritten as

dx nx

Using the alternative notation .v'"' for \^, we can rewrite this equation as

d
" l/„ _ ^r'^" _ J_ 1
l/n -1
dx rix n

^v'
140 CHAPTER 2 THE DERIVATIVE

Now we can prove the following theorem, which shows that the rule for differen-
tiating rational powers of v follows the same pattern as the rule for differentiating
integer powers of x.

THEORKM 1 Power Rule for Rational Exponents

Let r = m/n be a rational number, reduced to lowest terms so that n is a positive


integer and the integers m and n have no common factors. Then

d _,
y= rx'^
dx

holds for all values of .v for which x^ = (.v'^")'" is defined, except possibly for
A = 0. It also holds for .v = 0, provided that n is odd and m> n.

Suppose that x^ is defined and that x ¥= 0. Then, using the chain rule, the rule for

differentiating integer powers, and the rule obtained above for differentiating x^^",
we have

— dx
x' = —dx
(x''"r
,l/«.m-l
= m(x"")
dx
^\/n
X ' = mx /,.(l,n..-)
^^(m-1)/-

= _^_(m-l)/n + (l/n)-l = ^,(m/n)-l


n n

To complete the proof, assume that m> n and n is odd. Let/(.v) = .v'', noting that
r>\. Thus, when x = 0, rx"^' ' = 0, and so we must prove that/'(0) = 0. We have
/(0 + A.r)-/(0) [(^x)'/"^ -
/ (0) = lim : = lim
Av^o Ajc A-v—o Ajc

[(A.V)'
= lim lim [(^x)^'"]"'~" =
Aa^U [(A.V)'/"1" -i'-^O

If we combine the power rule for rational exponents in Theorem 1 with the chain
rule, we obtain the following important result:

If r is a rational number and u is a differentiable function of a, then

d du
—-U = ru —— or D^u"^ = ru^ '
D,m
dx dx

Of course, if u = 0, this rule may not apply, unless r = m/n. where m and n have
no common factors, m> n> 0. and n is odd.

In Examples 2 to 7, find the derivative.

EXAMPLK 2 fix) = X^''^

SOLUTION For A^ > 0.

/'(.,) = ^,.3/2 =
dx
|,.,,/2,-.
1 11
= l,./2 = |V]^
2.9 THE R^^TIONAL-POWER RULE 141

EXAMPLE 3 git) = r"'^^

SOLUTION For t ^ 0,

d 1'"-"
(,) = = -fl-7/3)-l = Lj-10/3
dt 3 3

EXAMPLE 4 h{x) = V Iv- + 3

d
SOLUTION h'{x) = Vlr- + 3 = (2v- + 3)'^^
(ir dx

= —3
(2V- + 3)"/''-'— (2X-- +
^v
3)

= Hlx- + 3)-"/\4r) = ir(lr- + 3)-^^^

EXAMPLE 5 Fix) = (1 - .v)-*/^(l + x^)~-^^

SOLUTION

F'{X)= (1 - .v)-*^-'(l + .V-)-2/3


dx

= [*(!- .r)-'/-'(-l)]{l + A-)--'-^ + (1 - .v)^'''5[-f (1 + x-r'^Mx)]


= (1 - x)-'/Hi + x-r^'\-i(i + .v^) - 3(1 - A-)(iv)]
= Ad - A-)^'''^(l + x-r^^Hlx- - 5x - 3)

This calculation illustrates a useful rule: To factor an algebraic sum in which each
term contains a rational power of the same expression, take as a common factor the
smallest rational power to which the expression is raised. For instance, in the
third line of the calculation, the expressions (1 - x)~^^^ and (1 - x)*^^ occur.
Since -i <, we factor out (1 - a) '/^. Likewise, since -§ < -f, we factor out
(1 + a:2)-5/3.

EXAMPLE 6 G(a) = sin Va^* + 5

G'(x) = —^v
sin \/x* + 5 = cos Va^* + 5 —dx
V'.r"* + 5

cos Va-* + 5 —
dx
U'* + 5)'

= cos Va-* + 5 -(A-' + 5)--^'3--(.v-' + 5)


dx J

= cos V7n liix'' + 5)--''\4x^)]

_ 4x^ cos ^x'* + 5

3(A^ + 5)-^-^

_ 4a-^ Va-* + 5 cos \^x-* + 5

3(x* + 5)

In the last step, we rationalized the denominat or by m ultiplying numerator and


denominator of the fraction bv (a-* + 5)''^ = Va"* + 5.
142 CHAPTER 2 THE DERIVATIVE

EXAMPLE 7 Hill) = (1 + tan (/')"''"*

SOLUTION If tan //-^-l,


H'(u) = -id + tan /r)' '*^'*'"'|(sec-«-)(2«))
= -JM(1 + tan u-)"'-^''*' sec- m"

Problem Set 2.9

In Problems I to 30. find the derivative of each function.

1 fix) = V^
2 /(.r) = V(?"

3 A'(.t) = 36Lr-'"''

4/(.v) = 2Lr'''

5 hU) = (\ - t)-^'^

6/U) =
^
7 /(«)= 1 + —

8 g(s) =
2.10 HIGHER-ORDER DERIVATIVES 143

2.10 Higher-Order Derivatives


As we have seen, a derivative can be interpreted as a rate of change. For instance,
the derivative dP/dt of the consumer price index P with respect to time / can be
taken as a measure of monetary inflation. When government economists say that the
rate of inflation is decreasing, they are talking about a rate of change of a rate of

Figure 1 change — a derivative of a derivative.


The idea of a "second derivative" also arises in the study of the motion of a
particle P along a linear scale (Figure 1). Call the scale the s axis, and denote the
variable coordinate of P by s. so that

s=f{t)

where/ is a function determining the location off at time /. The equation s -mis
called the law of motion, or the equation of motion, of the particle.
The velocity v of the particle P is defined to be the instantaneous rate of change of
its position coordinate s with respect to time:

The absolute value of the velocity is called the speed of the particle:

speed

In physics, the instantaneous rate of change of velocity with respect to time is

called the acceleration a of P:

Therefore, A ^'
-
~
A(^
dt dt\ dt

so the acceleration is the derivative of the derivative, or, as we say, the second
derivative of the position coordinate s with respect to time. If distances along the i
axis are given in meters and time t is measured in seconds, then the velocity v is

given in meters per second (m/s). Consequently, the acceleration a is given in

meters per second per second read "meters per second squared" (ni/s~).

EXAMPLE 1 Suppose that a particle is moving along the s axis according to the
law of motion s = +
lOOr^ lOf + 5, with s in meters and t in seconds. Find the
values of v and a when f = 0.5 second.

SOLUTION Here,

V = —=—ds

dt dt
d
(lOOr + lOr + 5) = 200/ + 10

= —= —
dv d
and a (200/ + 10) = 200
dt dt
144 CHAPTER 2 THE DERIVATIVE

Hence, when t = 0.5 second,

V = 110 m/s and a = 200 m/s"

EXAMPLE 2 Suppose that a particle is oscillating about the origin on the a axis

according to the law of motion .v = 10 cos [(7r/12)f - (7r/3)], with s in centimeters


and / in seconds. Find the values of v and a when i = 6 seconds.

SOLUTION Here,

V = — = —d
ds

dt dt
10 cos
IOtt

StT / 77
2.10 HIGHER-ORDER I»:RIVAT1\TS 145

If/ can be successively differentiated /; times in this way. we say that /is n times
differentiable, and we write its /ith-order derivative, or nth derivative, as

/" or /"

The parentheses around the n are to prevent its being confused with an exponent.
For instance.

and /"'=/'^'
f"=r-'

EXAMPLE 3 Find all the higher-order derivatives of the polynomial function


= 15.r^ -
fix) 8x^ + 3.r - Iv + 4.

SOLUTION = /'"(.v) = 60a-' - 24a- + 6x-2


f'(.x)

f"(x)=f'\x) = 180a- - 48a + 6


/"'W=/'^'W = 360a-48
f""(x)=f^\x) = 360
r"ix)=f'\x) =
Since/'"' is a constant function, all subsequent derivatives are zero; that is.

/""(a) = for n>5

Just as with first derivatives, we often deliberately ignore the distinction


between
the num-
the wth-order derived function/""' and the value /""(a) of this function at
ber a; both are referred to as "the nth derivative."
Operator notation for higher-order derivatives is self-explanatory:

D;/(a)=/"'(a)

The corresponding Leibniz notation is developed as follows: If

y=/(A)

d\
so that
dx

then the second derivative is given by

V dx '

= Dl fix) = fix)
dx

The symbolism
dy_'

dx
dx

second derivative cumbersome. Formal algebraic manipulation, as if ac-


for the is

tual fractions were involved, converts this to

idxf
146 CHAPTER 2 THE DERIVATIVE

In practice, the parentheses in the "denominator" are usually dropped, and the
second derivative is written as dry/dx'. Similar notation is used tor higher-order
derivatives (assuming these exist), as shown in Table 1.

Table I

V =/(A)
2.10 HIGHER-ORDER DERIVATIVES 147

EXAMPLE 6 Let/(.v) = v COS Iv. Find

(a) fix) (b) /"(A) (c) f'Xx)


(d)/'(0) (e)/"(7r/6) (f) /"'(-7r/4)

SOLUTION
(a) f'tx) = COS Ix - Ix sin 2x
(b) /"(.v) = -2 - 2 sin Iv - 4.x cos Zv
sin 2t
= -4 - 4.x cos Iv
sin 2a:

(c) /'"(.v) = -8 cos Iv - 4 cos Ix + 8.v sin Ix

= -12 cos 2v + 8.V sin It


(d) By (a)./'{0) = cos [2(0)] - 2(0) sin [2(0)]
= cos - = 1

,.,B,,.,,r(^) = -4.„[2(^)]-4(f)c„s[2(^)]

= -4 , .

sin
TT Itt
cos —n
3 3 3

-^(^)-f(l)
2

= -2V3

(f)By,c,./-(-f) = -,,cos[2(-^)].8(-f)s,„[2{-f

= -12cos(-y)-27rsin(-^)
2

= -12(0) -27r(-l)

Many of the differentiation rules developed earlier have generalizations for


higher-order derivatives. (Most of these are proved by mathematical
induction on n.
the order of the derivative involved.) For instance, the
addition rule and the homo-
geneous rule work for the ;!th-order derivative; that is.

D':U\x) + g(.x)] = D'iflx) + D'iglx)

^d D"Acf(x)] = cD", f(x) c constant

The product (or Leibniz) rule for higher-order derivatives (/• g)'">
is more compli-
cated. For example, if n = 2. we have

if-g)" = (f-g' +f'-gy = (f-g'Y + (J'-gY


= f-8"+f'-g'+f'-g' +f"-g
Hence,

U'-g}"=f-g" + 2f'-g' +f"-g


I

As Example 7 illustrates, higher-order derivatives can be found b\ implicit differ-


entiation.
148 CHAPTER 2 THE DERIVATIVE

EXAMPLE 7 If -r^ - 2y^ = 4, find D^y and Dly by implicit differentiation.

SOLUTION Differentiating both sides of r^ — 2y- = 4 with respect to .v, we have


Ix - 4vD,y =

2x X
hence, D,v =
4v 2v

Differentiating both sides of the last equation with respect to v and using the quo-
tient rule, we obtain

D:y = — - .vD,(2v)
—2vD,.v 2v - 2jfD,v
;- '— = — ^—^ = v - xD^y
r-^
{2y)- 4y- 2y^

Substituting D^y = .v/(2y) into the last equation and simplifying, we find that

y - A-[.r/(2y)] 2y^ - .r
D:v = :;
= ^
2y- 4y='

Finally, since x~ — 2y" = 4, we can rewrite this result as

-4
"-'^-^
1

or Dly = z-

Problem Set 2.10

In Problems I to 6, a particle is moving along the i axis according to 13 GU) = (.r - 3)(.v'' + 3.r + 9) 14 /(m) = (w" + 1)-'

the given law of motion i =f(i). Find v = ds/cli and a = civ/cli.


3
15 g{f) = r'vT- 5f 16 /(.v) = .V

1 s = r^ + 2r + 3i - 5. s in meters, / in seconds

2 .? = (r + 1) ', i in centimeters, i in seconds 17 /(jt) =^-\


X
18 gU) = (.V +
yj
3 .9 = 3 sin 777 - 2 cos 2m, s in meters, ( in seconds

4 s = tVr + 4, i in miles, / in hours ,9/,„, = ^^


2— M
20 F(v) = Vv +
1

5 5 = ir*^" + ir^''', s in kilometers, ; in hours


21 /(f) = Vr + 1 22 g(y) = V3y+T
6 i = igr + Vol + Sn, where g. vq. and sq are constants

23 F(r) = {\ - Vrf 24 h(x)


7 In Problems 1, 3, and 5, find i' and a at the instant when t = I
V.v- + 1

unit.
25 /(.v) = 7 cos ll.v 26/(/) = -6 sin (5 - 2t)

8 If air resistance is neglected, a dropped object will fall through


i = 4.9r meters during a time interval of; seconds, (a) Find the 27 F(B) = sin 20 + cos 3fl 28 /i(.v) = (.V + sin.v)'*

acceleration of the falling object, (b) Find a formula for the


velocity of the object after 29 H(t) = 2 CSC It 30 /J(y) = 5 tan' 4y
it has dropped through s meters.

In Problems 9 to 36, find the first and second derivatives of the 31 Q(e) = Scot 3e 32 Six) = VI + sin 5x

function.
33 G(.v) = sin —— - 34 V = Vl + sec.v
X + 1
9 f(x) = 5x^ + 4x + 2 10 g{x) = x^tr + 7)
36 y = r^ sin
11 fit) = li^ ' 23r + I + 9 12 F(x) = ^(x + 2f 35 V = -4 sec 5x
2.10 HIGHER-ORDER DERIVATIVES 149

37 In Problem 17. find /'(-I) and /"(-I).

38 In Problem 24, fmd h'{V2) and h"{V2).

39 In Problem 27. fmd F\tt/6) and F"(Tr/6).

40 In Problem 34. find the value of dy/dx and of d-y/dr when


X = tt/A.

41 Find all higher-order derivatives of the function


f(x) = 7.V-' - 5.r + 8a- - 3.v + 81.

42 If/U) = (VTTT)-'. fmd /'""(/).


43 Uy = s/jc - 1, fmd dy/dx, dy/dr, and A/atr^.

44 Develop a formula for D"(v!v), ,v 5^ 0.

45 Develop a formula for D"(l/.\). jr 5^ 0.

46 Find D™ sin .r.

47 If V = 3jr + 2.V, evaluate and simplify the expression


Ad-y/dx^) - Zx(dy/dx) + 2y.

48 Assume the Leibniz rule

{f-g)"=f-g" + 2f'-g' +f"-g


for second derivatives, (a) Derive the Leibniz rule

(f-g)'" =f-g"' + if-g" + ¥"-g' +f"'-g


for third derivatives, (b) Assuming that /(I) = -3, g(l) = i,
/'(1)=-1. g'(l) = 4. /"(I) =16, and g'U) = I fmd
(/•g)"(l).

49 Let g be a twice-differentiable function such that g(2) = 3,


g'(2) = 3, and g"(2) = 5. Define the function /by the equation
fix) = .r''g(.v). Find the numerical value of /"(2).
50 Find formulas for/'(.v) and/"U) if
150 CHAPTER 2 THE DERIVATIVE

2.11 Algebraic Signs and Zeros of Functions


If a function /satisfies the condition /(.v) > for all values of .v in an interval, we
say that /is positive on the interval. Likewise, we say that / is negative on an
interval if/U) < for all values of .v in the interval. To say that /is positive (or
negative) on an interval means that, over this interval, the graph of/ lies entirely

above (or entirely below) the .v axis. The x coordinates of points where the graph of
/touches or crosses the x axis are called the x intercepts of the graph. In other
words, the .v intercepts of the graph of /are the solutions, or roots, of the equation

/(-v)

For this reason, an x intercept of the graph of/ is also called a zero of/
For instance, from the graph shown in Figure 1 of the function

f(x) = Ax^ - 4.r-^ - 25.r= + .v + 6

you can see that/(j:) is positive on the intervals (-=<=, -2), (-2, 2), and (3, 00) and
negative on the intervals (-2, -5) and (A, 3). Evidently, the x intercepts of the
graph of/ are -2, -5, 5, and 3. Notice that the intervals on which/is positive are
separated by zeros of/ from the intervals on which /is negative.

Figure 1 y

In this section, we use the ideas of continuity and the derivative to study the
algebraic signs and zeros of a function. One of our basic tools is the following
important theorem about continuous functions.

The Intermediate- Value Theorem

Suppose that the function/is continuous on the closed interval \a, h\. Then if A'

is any number between /(a) and /(ft), there is at least one number r in the interval

\a. b\ such that /(<) = k.


2.11 ALGEBRAIC SIGNS AND ZEROS OF FUNCTIONS 151

Although the formal proof of this theorem is best left to more advanced courses,
Figure 2 should make it seem plausible. Indeed, since /is continuous, its graph
consists of one connected piece with no holes and no sudden jumps. The points
(a, /(a)) and (b,f(b)) lie on this graph, one below and the other above the horizontal
line y = i. It seems clear that the graph of/ must intersect the line v = k, say, at the

point (c, k). Therefore, /(c) = k. Note that the intermediate-value theorem just
guarantees the existence of such a number c, but it doesn't tell us how to actually

find such a number.

Figure 2

(b.fib))

EXAMPLE 1 Show that there is a number c between 1 and 2 such that

c^ - 2c^ = 9.

The polynomial function

fix) = . Ix'

is continuous on the interval = -1 and/(2)


[1, 2]. Also, /(I) = 16. Because 9 is

between —1 and 16, Theorem guarantees the existence of at


1 least one number c
between 1 and 2 such that /(c) = 9; that is,

c5 - 2c^ = 9

A function /is said to change sign on an interval / if there are numbers a and b
in /such that /(a) and fib) have opposite algebraic signs. As a consequence of
Theorem 1, we can state the following useful property of continuous functions.

Change-of-Sign Property

A continuous function cannot change sign on an interval unless it has a zero in


that interval.

To see how the change-of-sign property follows from Theorem 1 , let / be a


continuous function on an interval /, and suppose that a and b are numbers in / for
which /(a) and fib) have opposite algebraic signs. It follows that is between /(a)
and fib); hence, by Theorem 1 there is at least one number c between a and b such
,

that /(c) = 0.

EXAMPLE 2 Show that the function /(jr) X + X — 1 has a zero in the interval
[0, 1].

SOLUTION The polynomial function /is continuous on the interval [0, 1], and
because /(O) = -1 and /(I) = 1, it changes its sign on this interval. Therefore, by
the change-of-sign property, / has a zero on the interval [0, I].
152 CHAPTER 2 THE DERIVATIVE

The change-of-sign property justifies the following simple procedure.

Procedure for Finding the Intervals on Which a Function / is


Positive or Negative

Step 1 Find all the numbers at which /is undefined or discontinuous or takes

the value 0, and arrange these numbers in increasing order: xi. .v^, .^3, . . . ,

.v„. These numbers will divide the x axis into n + 1 open intervals: ( — oc, X\),
(.V,, as), U2, xj), . . . , U„_|, x„), and (x„, ^).

Step 2 By the change-of-sign property, you may conclude that / cannot


change sign on any of the intervals (X), xj), U^- f.i) Un-w -^n)- There-
fore, /is either positive or negative on each of these intervals. The same is true
for the unbounded intervals (-^, .Vi) and (x„. ^). provided that they are con-
tained in the domain of/.

Step 3 Select any convenient "test number" from each interval considered in

step 2, and evaluate the function /at each test number. The algebraic sign of/
at each test number determines the algebraic sign of /on the entire interval
containing that test number.

In Examples 3 and 4, determine the intervals on which the function is positive or


negative.

EXAMPLE 3 fix) = 4.x- -I- 8.v - 5

SOLUTION We follow the steps in the procedure above.

Step 1 The polynomial function/is defined and continuous at every real num-
ber X. The zeros of/ are the roots of the quadratic equation

4.x- -^ 8.V - 5 =

Factoring the left side of this equation, we have


(Iv-f- 5)(lv- 1) =
so the roots are

.V = —§ and .V = 5

These two numbers divide the .v axis into the three open intervals

(-=^, -f) (-1 i) (2. ^)

Step 2 The function / is either positive or negative on each of the intervals


obtained in step 1

Step 3 From the interval (


— ==, — i) we select, say, the test number -3. Be-
cause

/(-3) = 4(-3)2 -(- 8(-3) - 5 = 7 >


we conclude that/is positive on the entire interval (-^c, -f). From the interval

(-f, i) we select, say, the test number 0. Because

/(O) = 4(0)- + 8(0) - 5 = -5 <


we conclude that/ is negative on the entire interval (-1, 2). Finally, from the
interval (i, ^) we select, say, the test number 1. Because

/(I) = 4(1)' -I- 8(1) - 5 = 7>0


2.11 ALGEBRAIC SIGNS AND ZEROS OF FUNCTIONS 153

we conclude that/is positive on the entire interval (i, ^)- Figure 3 summarizes
our findings. I

Figure 3 algebraic sign of Hx = ) 4.y2 + 8.v - S

value is value is

zero zero

++++++++ +++++++-

(X + 3)(.v - 2)
EXAMPLE 4 /(.v) =
(x+ l)(.r- 1)

SOLUTION Here, f{x) is undefined when the denominator of the fraction

(X + 2,)(x - 2)

(x + l)(;c - 1)

is zero, that is. when x = — 1 and .v = Also,/(x) =


1 . when the numerator is zero,
that is, when .x = —3 and when .v = 2. We arrange these four numbers in the order
—3, —1, 1, 2 to determine the five open intervals

-3) (-3, -1) (-1, 1) (1. 2) (2, ^)

on which the function /is either positive or negative. Selecting a convenient test

number from each interval, say —4, —2, 0, f, and 3. and evaluating/(v) at each test

number, we obtain

/(-4) /(-2) /(O) /(i)


= /(3)

The algebraic signs of /(.v) at these test numbers determine


Figure 4 algebraic sign of fix) =

{x + I )(.t ~ the algebraic signs of /on the corresponding intervals, as


shown in Figure 4.
undefined undefined

value is value is
The change-of-sign property can also be used to help locate
zero zero
the zeros of a continuous function / by the following bisec-
++++++. tion method: By sketching a rough graph, or by trial and
*
i error, find a closed interval [a.b\ for which/(a andf(b have
) )

opposite algebraic signs. By the change-of-sign property.

/ must have a zero in this interval. Using the midpoint


c = \(a + b), divide the interval [a, b] into two subintervals [a. c] and [c, b]. If

/(a) and /(c) have opposite algebraic signs, then /must have a zero in the interval
[a. c]. If /(c) = 0, then c itself is a zero of/. The only other possibility is that /(a)

and /(c) have the same algebraic sign, which case /(c) and f{b) must have in

opposite algebraic signs (why?) and/must have a zero in the interval [c, b]. Con-
tinuing in this way, you can locate a zero of/as accurately as you wish by confining
it in successively shorter and shorter intervals.

El EXAMPLE 5 In Example 2. we used the change-of-sign property to show that the

continuous function i
^,
fix)
,
= .r
,

-I- .V - ,
1

has a zero between and 1. Starting with the interval [0. 1], use the bisection
method twice in succession to locate a zero of/ with more accuracy.
154 CHAPTER 2 THE DERIVATIVE

SOLUTION The midpoint of the interval [0. 1 1 is i(0 + 1) = 0.5. A simple cal-
culation reveals that

/(0)=-l and /(0.5) = -0.375

so/(0) and/(0.5) have the same algebraic sign. It follows that /(0. 5) and /(I) must
have opposite algebraic signs. Indeed, they do:

/(0.5) = -0.375 and /(I )


= 1

so /has a zero between 0.5 and 1. Now, for our second application of the bisec-
tion method, we start with the interval |0.5, 1|. The midpoint of this interval is

2(0.5 + 1) = 0.75. Here we have

/(0. 5) = -0.375 and /(0. 75) = 0. 171875

so /(0.5) and /(0. 75) have opposite algebraic signs. It follows that /has a zero
between 0.5 and 0.75.

Newton's Method
In the seventeenth century, Isaac Newton discovered an elegant method, using
derivatives, for estimating the zeros of functions. This method, which is usually
much more efficient than the bisection method, is especially well suited to modem
electronic calculators and computers. The basic idea of Newton's method is illus-

Figure S trated in Figure 5. Suppose that z is a zero of the differentiable function/ and that a
is an approximation to z. If we start at the point (a,f(a)) and follow the graph of/
to the point where it intersects the x axis, we obtain the exact value of z. If, instead,
we start at the point (a.fia)) and follow the tangent line to the point (b, 0) where it

intersects the x axis, we obtain an approximation b to the zero z. In many cases, b


is a better approximation to z than a was.
In Figure 5, an equation of the tangent line to the graph of/ at {a, f(a)) is

y-f{a)=f'(a)ix-a)
Because the point (b. 0) is on this line, we must have

0-f(a}=f'ia){b-a)
If /'(a) v^ 0, we can rewrite the last equation as

/(g)
^^
/'(«)

Thus, the basic idea of Newton's method is to replace an approximation a to a zero


of/ by what we presume is a better approximation b given by the equation above.
But then we can take this "better" approximation b and substitute it back for a in

the equation to obtain, we presume, an even better approximation;

c = b
2.11 ALGEBRAIC SIGNS AND ZEROS OF FUNCTIONS 155

This process can be repeated as often as we wish to obtain (we


hope!) better and
better approximations to a zero of
f.

The discussion above is summarized as follows.

Newton's Method

Let .V, be a first approximation to a zero of the differentiable function /. Then


successive approximations

X2, x^, . . . , jr„, .r„+

are given by repeated use of the equation

f'(x„)

which gives the {n + l)st approximation in terms of the Mth approximation.

In using Newton's method, the first approximation .v, can be obtained by an "edu-
cated guess. by sketching a rough graph of/, or by using the bisection method.
In more advanced courses in numerical analysis, conditions are developed to
guarantee that Newton's method "works" in the sense that the
successive approxi-
mations .V,, .V2, -V3. really do come closer and closer to a zero of/. Roughly,
. . .

these conditions are that [f'(x)\ is not too small and


\f"(x)\ is not too large for values
of .Y close to .vi. However, rather than bothering to check these
conditions, it's
usually easier just to use a calculator and see what happens.
When the method
works, you'll soon see the successive approximations "settling down"
and coming
closer and closer to the desired number. When the number
\n the display of the
calculator stops changing as you repeat the calculation, you have found an approxi-
mation to a zero of the function within the limits of accuracy of
the calculator.
When the method fails, you'll soon see the successive "approximations" behaving
erratically and not "settling down" (see Problems 53
to 56).

B EXAMPLE 6 In Example 5, we used the bisection method to show that the func-
tion

/(.v) = .r' -^ .V - 1

has a zero between 0.5 and 0.75. Using a calculator and Newton's
method, find this
zero to as many decimal places as possible.

SOLUTION Here,

f\x) = 3.v^ + 1

and the formula

f(x„)

/'UJ

- (x„? + x„ - 1
becomes .v„+ ,
= .v„
3(x„)- + 1

We're looking for a zero on the interval [0.5, 0.75], so let's take the nudpoint

-v, = i(0.5 + 0.75) = 0.625


156 CHAPTKR 2 THE DERIVATIVE

as our first approximation. Then, using a 10-digit calculator, we find that

(0.625)' + 0.625 - 1

X2 = 0.625 - -^^
; = 0.685251799
3(0.625)- + 1

Repeating the process with the value of X2 just obtained, we have

(X2? + X2- 1

•^3 = X2 = 0.682335090
3(X2)- + 1

Note that in the passage from X2 to x^. the first two decimal places haven't changed.
This is an indication that the successive approximations are settling down as de-
sired. Let's continue the process:

(.v.,)-' + x^- \
= 0.682327804
3(.V3)- + 1

(y4)-' + x^- \
= 0.682327804
l(x^f + 1

Figure 6

1
2.11 ALGEBRAIC SIGNS AND ZEROS OF Fl'NCTIONS 157

Problem Set 2.11

In Problems 1 to 12. use the intermediate-value theorem (Theo- 24 H(x)


rem 1 ) to show that there is a number c between the two given
numbers that satisfies the equation.

1 c between and 1 , 4c- — c' = 2

2 c between 1 and 2. 2c* - 4c^ -I- 8c- = 7.07

3 ( between 2 and 3, c' + 3c~ - 9c- = 10

V8c - 15 2
4 c between 2 and 3.

r' + 5
5 c between and 1.
V?+ 1

El 6 c- between 2.5 and 2.6, c"* - 8r- + c = -6

7 c between -2 and -1. 2f' - 3c-~ - 12f = 1

1
E 8 c- between 1.4 and 1.5, -^ V3
4c' + 4c-

9 c between 3Tr/4 and it. sin c + 2 cos 2c- = 1

EllO f between tt/6 and Tr/3, 2 esc c -I- cot c =4


Oil f between and 77/6, c + sin c = 1

sin c 1
12 c between 77/6 and 7r/2,
2 + cos r

In Problems 13 to 18. show that the function has a zero in the


interval by using the change-of-sign property. (You need not try to

find the value of the zero.)

13 /(jc) =.x^ - 2.r' - 1 in [1, 2]

1S114 g(x) =x* + 6.r' - 18.\-^ in [2.1, 2.2]

miSf(x) =;r*- Ir' - 1 in [1.5. 1.6]

EII6 gM =x* + 6.r' - 18.r^ in [-8.2, -8.1]

17 h(x) = sin .v + 2 cos 2x in [Tr/4, 7r/3]

EII8 f (.t) = sin- .V + 2 cos x in [1.9, 2]

In Problems 19 to 34. determine the intervals on which each func-


tion is positive or negative.

19 fix) = .r - 6.t + 8

20 g(.v) = 25.r - 20.x -I- 4

21 Fix) = lv(.v - 3)(.v + 5)

22 G(.v) = .V-* - 7.x- -I- 12

23 htx) = (Ix-h l)(3.x- l)(.x-2)


158 CHAPTER 2 THE DERIVATIVE

41 The function htx) = sin .t + 2 cos It of Problems 17 and 37;


.V| = midpoint of the interval [ir/4, ir/31

42 The lunction F(.x) = sin" x + 2 cos x of Problems 18 and 38;


A] = midpoint of the interval [1.9, 2)

43 G(.r) = .r' - 7.r + 7; .v, = -3.5

44 W(.v) =x^ - 4.r - Iv + 4; .r, = 4.5

Eln Problems 45 to 48, use Newton's method to solve the equation.

45 2r 4.r + 5.V = 7 46 15r' + I3.r' = 1

47 48 .r^/- + .V = 1
- Ix^'-

E149 The ancient Babylonians discovered the following rule for ap-

proximating the square root of a positive number k: Begin by


making a reasonable estimate ti of \k. Then b = i[a + {k/u)\

will be an even better estimate. Repeat the process to get better

and better estimates, (a) Use this method to obtain \/2 to as

many decimal places as you can, starting with 1 as a first ap-

proximation, (b) Show that this method of estimating square


roots is just a special case of Newton's method.

ElSO The function G(.r) = x^ - 7x + 1 of Problem 43 has two zeros


on the interval 11.3. 1.7]. Find these two zeros to as many deci-

mal places as you can.

51 Using Newton's method, derive a rule for obtaining successively


better and better approximations for the nlh root of a positive
number k.

El 52 In order to work Problem 56 on page 194 it will be necessary to


solve the equation (/ - a)\ i + a - b = Ofor t. where a = 324

and fc = 4.32 X 10^. (a) Use the change-of-sign property to


show that there is a solution of this equation on the interval from
t = 5000 to I = 6000. (b) Use Newton's method to find this
solution to as many decimal places as you can.

53 For the function/whose graph appears in Figure 7, .v, appears to


be a reasonable first approximation to the zero z. However, if

you use Newton's method, the second approximation

/Ui)
''
'" /'(JCi)

Figure 7 y
REVIEW PROBLEM SET, CHAPTER 2 159

Review Problem Set, Chapter 2


1 An object is thrown straight upward with such a speed that at Figure 1 iContinued)
any time its distance s in feet from the surface of the earth is

given by the equation j = 200( — 16r, where / is measured in

seconds. Find (a) the average speed during the third second
of motion, (b) the instantaneous speed when I = 2 and when
/ = 3, and (c) the highest point reached by the object.

2 The motion of a particle along the .v axis is given by the equa-


tion X = 2t - (r/2), where .v is the coordinate of the particle at
time /. Distance is in meters and time in seconds, (a) Determine
whether the particle is moving in the positive or negative direc-

tion when / = 0. (b) Find the instantaneous speed when / = 1.

(c) When does the particle change its direction of motion?

3 The side of a square metal plate is 20 inches when its tempera-


ture is 50°. If its temperature is raised to 75° in 10 minutes,
each side expands 0.2 inch. Find (a) the average rate of change
of the area of the plate per inch change in the length of a side
and (b) the average rate of change of the area per degree change
in temperature.

4 In optics, the magnification at .v of a lens that projects the point


X of one number line onto the point y = fix) of another number
line is defined to be the instantaneous rate of change of y with
respect to .v. Suppose that a lens projects x to y = .r\ What is its

magnification at x = 2?

In Problems 5 to 8, find the equations of the tangent and normal


lines to the graph of the function at the indicated point.

5 fix) = r - 4.V + 2 at (4, 2)

4
6 gix) at (2, i)
x+ 1

7 fix) = ix' at (i, 11)

8 fix) = 96.x - ir'' at (0, 0)

9 Indicate which of the functions in Figure 1 is (i) continuous on


(a, b), (ii) differentiable on (a, b), and (iii) continuous and
differentiable on (a, b)

Figure 1
160 C HAFFER 2 THE DERIVATIVE

11 f(.x) = 3.r - 2 12 /(.v) = x- x^ In Problems 31 to 80, use the differentiation rules to find the deriva-

tive.
13 fix) = .r + .V + 1 14 /(.v) = Ir^ - 1

31 fix) = 7.r^ 32 gix) = -4x-^


15 = - 4.x + 3) 16 fix) = —!—
f(.x) h(.x-
- .V 1
33 /;(.v) = ix'^ 34 Fix) = TTx^'-

17 f(.x) = 1 + — 18/U)= -n- 35 C(.v) = VZv^/' 36 Hix) = ^


il fix) = 5.V- -lx+ \l 38 gix) = TTi\ - it)
19 Verify your answers to part (b) of Problems 11, 13, 15, and 17
by using the basic algebraic rules for differentiation. 39 ^(.v) = -9.r' + Ix^ -x + 22'

20 We know that the derivative of a product is not, in general, the 40 Fix) = 7.r/' - iU"' + V43
same as the product of the derivatives. Are there any integers ;;
- 5r- +
41 Gil) = 5?^' + 15f* 8
and m for which D^{x".x"') turns out to be the same as
(D,x-")(D,x"')l 42 Hill) = 3.1m-'* -I- 1.2ir'*-

21 Let /be a function defined by/(.t) = |.t| + |.v + l|. (a) Sketch 43 fix) = i.x^ + 2x + l)(2xr' -I- 5)
the graph of /. (b) Determine the points at which / is not
44 git) = iV7+ l)(2f -I- V7- 2)
differentiable.

45 his) = (6J-' -t- 5r - s'')"


22 Suppose that the function g is continuous at 0, and/ is a func-
tion defined by/(.i) = xgtx). Is /differentiable at 0? If so, find 46 F(v) = (2V^ - 3v--)'"
/'(O) in terms of g.

23 Suppose that / and g are differentiable functions at 7 and that


., ^
47 Gix) =
A- +
X

+
-
3j:

2
1
48 Hit) = Vt
t-
+
1
3

/(7) = 10, /'(7) = 3, ^(7) = 5, and g'(7) = -^. Find


49 fix) = Vxix^ + Ix + 1) 50 git) = 2Vt + 1 (2/ + 1)
(a) (/+.i,')'(7) (b) (/-;s')'(7) (c) {JgYO)
5>'-l-4
/V (7) 51 /;(.v) = 1 52 Fiy) =
(d) (e) [^-~^)o) (f) (f+2gya) ^-1- 1

53 Gix) = V.x- + 12 54 Hiz) = y/z- + V?TT


(g)
+ e
'f+8
55 fix) = 8V^ - 56 gix)
24 Suppose that / is a differentiable function on IR such that Vx- 1 x-Vx
fix + y) = fix) + fiy) holds for all numbers x and y in IR.
Prove that/(0) = 0. (b) Prove that/'(0) =/'(.v) holds for 1 + (1/y)
(a)
57 /i(.v) = (.r -I- 7)-' V.V - 7 58 fiy) =
all numbers x in IR. 1 -d/y)
/ f^
- 3r -I- 2 f''-
In Problems 25 to 28: (a) Indicate whether the function/is continu-
ous at X = 2. (b) Find /'-(a) and /'+(«). (c) Is / differentiable at
X = a'l (d) Sketch the graph. 60 hiq) = \'q -I- Vq^Tv^

3.V <2
if -v 61 fix) = ax^ b
-, where a, b, c, and d are constants
25 fix) = a = 2 ex + a
6 ifx>2
I ax + bY
26 fix) = 2 -1-
|;r- 1|; a = 1 62 gix) = I where , a, b, c, d, and r are constants
\ ex + d '

\{x< -3
27 fix) = ;a= -3
10 if.»>-3 63 fix) = — sin 2v 64 git) = f sin

28/(.v) = [I.v- 2||;a = 2
65 hit) = -f cos 4r

El In Problems 29 and 30, use a calculator to find the approximate


66 Fix) = V.*: -I- 1 tan U -(-
1
value of the indicated derivative. Use A.v = a/10'*.

67 Gix) = § sin (3,r - 1) 68 Hi0) = cot (Ve-l- S)


29 f'ia) where /(.v) = VF+l and u = 3
cos f
69 /(/) = 70 g(y) = Vcos 5y
30 g'ia) where ^(.r) = cot Vx and a = 0.6169 1 -f esc /
RE\ lEW PROBLEM SET. CHAPTER 161

71 hid) = 3 sin cos 20 72 F{x) = at tan V3x 90 The normal line to F{x) = .v/V.r + 1 is parallel to the line

.V -I-
y = I

73 G(s) = § tan fj - I cot jj 74 //(at) = 2 sin (esc jt)

2 CSC .V - 3 cot .X In Problems 91 to 96. find all points on the graph of the function at
75 /(.r) = 76 gU) = cot"^'- 5,x
.v+ 1
which the tangent line is horizontal.

= 91 /(.v) = - 3.V 92 g(x) = ix-*'^ - lit"-' + 2


78 qiy) = \
.X-
77 ft(e) sin Vv
sec fl + tan
93 ;i(.v)

79 /(f) = a cos (o)/ - rf)) + k. where a, oj. <i). and k are constants x^ + 4

94 Fix) = xV3 - X
f 3 cos .V if .V a
Ua- + 3 it .r<0 95 G(.v) = sin- .v -i- cos x. — tt < .v < tt

El 81 Some people believe that we are all subject to f)eriodic varia- 96 Hix) = X + cot X. < .V < 277

tions, called biorhxthms. in our physical stamina, emotional


well-being, and intellectual ability. The physical cycle, for in-
97 Suppose that the interval (1. 3) is contained in the domain of
stance, is supposed to have a period of 23 days. Assume that,
the function/, that /is differentiable at 2. and that/(.v) </(2)
on a scale from to 10. a certain person's physical stamina p for all values oix in the interval ( 1 . 3). By quoting an appropri-
is given by p = 5 -i- 3 cos (6.5564? - 5.804) on day number t.
ate theorem, show that/'(2) = 0.
starting with f = 1 on January 1 . Find the rate of change of p
with respect to t on January 21. 98 Let g(.v) = x - sin x. (a) Show that the graph of g has a hori-
zontal tangent at (0. 0). (b) Does g have a relative extremum at
82 In the study of FM (frequency-modulated) radio transmission.
.V = 0? Why or why not?
the function f(t) =A cos (oj.j + bt cos io„t) represents the
strength /(f) of an FM signal at time r seconds. Here A is the
In Problems 99 to 102. suppose that h =f°g. Use the given infor-
amplitude of the carrier signal, (o^^/iln) is the frequency of
mation to find h'ia). iCaiition: There may be more information than
the carrier signal, b is the amplitude of the modulating signal.
you need.
and ojJ(1tt) is the frequency of the modulating signal. Find
/'(f) in terms of A. m^, b. (o„, and f.
= = -1
99 a = 2./(2) = 8. g(2) = 0./'(G) 12. g'(2)

83 If A- is a rational number and A > 1 . show that the tangent line at


= = = = = w
100 a TT.fiTT) 3, gin) 7r./'(7r) I/tt, g'iiT)
the point (1 . 1 ) to the graph of/(.r) = .v' intersects the y axis at

a point that is A: - 1 units below the origin. 101 a = \ 2./'lg(V2)] = Vl. g'iVl) = V2

84 Show that if n is an odd positive integer, the lines tangent to the 102 a= 1.732. g( 1.732) = 7.007. /' (7.007) = 0.2. g'( 1.732) = 5

graph affix) = x" at the points (1.1) and (


- 1 .
- 1 ) are paral-

lel. 103 Suppose that / gives the area of a square as a function of the
length of one of its diagonals. Express /as a composition of
85 Let y = (1 - (0/(1 + u) and h = (3 - x)/{2 + x). Use the
two other functions.
chain rule to find dy/dx.
104 In economics, a linear supply-demand-price model
86 The electric field intensity E on the axis of a uniformly charged
ring at a point .t units from the center of the ring is given by the s = Ap - B and ap
formula E = Qx/(a' + xrf^~, where a and Q are constants.
relates the number of units i of a commodity supplied by manu-
Find a formula for the rate of change of field intensity E with
facturers, the number of units q demanded by consumers, and
respect to distance .v along the axis.
the price p per unit of the commodity. Here A. B, a, and b are
positive constants. Of course, if the demand exceeds the sup-
In Problems 87 to 90. find a point on the graph of the function where
ply, the manufacturers will be inclined to increase the price. A
the tangent or normal line satisfies the indicated condition. Then
recursive pricing model is a formula
write an equation for this line.

new price =/(old price)


87 The tangent line to/(.v) = 3.r - 2v -I- 1 is perpendicular to the
line x + Ay - \ = 0. that gives the new price per unit of the commodity as a function

/ of the old price 1 unit of time earlier. Consider the recursive


88 The tangent line to g(.v) = x/(x - 1). .v > 1. intersects the y
pricing model given by
axis at (0. 4).

fip) =p + k(q- s)
89 The normal line to h(x) = cos x, s ,v < 7r/2, is parallel to

the line 2x —y+A= Q. where k is a positive constant, (a) If p is the current price per
162 CHAPTER 2 THE DERIVATIVE

unit of the commodity, what is the economic interpretation of 122 yVlr + I = y -I- 1 at (4, i)

the orbit of p.
123 v - cos y = at (i. 7r/3)

p. AP). (/"/)(/'). (/°/°/)(p) -


124 77 tan (.v y) = 2v at {7t/2, 7t/4)

under the iterates of/? (b) When p,, = fipo). we refer to po as

the equilibrium price of the commodity. Give the economic 125 Let (a, b) be a point on the circle r + y = r. Using implicit

interpretation of the equilibrium price, and find a formula for differentiation, confirm the fact (already known from elemen-
Pu in terms of ,4, B. a. and b. tary geometry) that the tangent line to the circle at (a, b) is

perpendicular to the radius at (a, b).


105 Let /(.v) = a + /3.t, where a and /3 are constants and /3 t^ I

Let -Vi be any real number and consider the successive images 126 The curve (x^ + y")" = ir - y", which was first studied by the
Swiss mathematician Jakob Bernoulli in 1694. is called a
.X2 =/(.V,), .tj = (/o/jUi). .V4 = (/°/°/)(.V,). . . .
lemniscate (Figure 2). Use implicit differentiation to find the

of X\ under the iterates of/, (a) Show that there exists ex- coordinates of the four points on the lemniscate at which the
actly one number .vn such that .Vo=/(.Vi)). (b) Show that tangent line is horizontal.

[f(x) - -Vol
= 1^1 |.v - .Vol holds for every real number .v.

(c) Show that |.r„+i - .Vo| = \^\"\xi — x„\ holds for every posi-
tive integer n. [Him: Use part (b) and mathematical induction.] Figure 2
(d) If |/3| < 1 , use the result of part (c) to show that the succes-
sive images of .ti under the iterates of/ come closer and closer
l.v- +.1-)- = .V- ->'-
to .v„.

106 In Problem 105. show that if |/3l


> I and .v, ^ .v,,. then the
successive images of .Vi under the iterates of / have absolute
values that become larger and larger without bound. What hap-
pens if ^ = - r?

E] 107 In Problem 105. let a = 2.7 and take .V| = 5. Use a calculator
to find X2. X, .Via for (a) /3 = 0.9 and (b) p= 1.1.
127 The curve .v' -I- y' = 3.vt. which was first studied by Descartes
108 The recursive pricing model in Problem 104 is said to be stable in 1638. is called the folium of Descartes (Figure 3). Use
if. as time goes on. the successive prices given by the model implicit differentiation to find the coordinates of the point P in

approach the equilibrium price po- Using the results of Prob- quadrant I where the folium has a horizontal tangent.
lems 105 and 106. find necessary and sufficient conditions in
terms of the constants A.B.a.b, and k for the recursive pricing
model to be stable. Figure 3

In Problems 109 to 116. use implicit differentiation to find D^\

109 3.r' + 2v- = I 110 v^ + 4o^ = 25

111 5.r^-t- 8y' = I6V7TT 112 4.r' - 5xf + >-' = 18

113 v - sin v = 1 14 sin y = cos .v

115 1 -I-
y = jr cos y 116 vlan.v- - xy* = 15

117 In Problem 109. find D\\ in terms of .v and y.

118 In Problem 1 10. find b\\ in terms of .v and y. 128 The curve xy + y- = 4 (an ellipse) intersects the
v" -I-
y axis in
two different points. Show that the tangent lines to the ellipse at
In Problems 1 19 to 124. find the equations of the tangent and nonnal these two points are parallel.
lines to the graph of the implicit function determined by the equation
at the point indicated. In Problems 129 to 136. find the second derivative of each function.

119 y- lx\ = 16 at (3, 2) -


-I-
129 fix) = 4.r' -1-
3.v^ .v H- 1 130 g{x) = x- O/x)
120 r' -I- 4xy r
-t- -(-
3 = at (2. -
—+
1

131 h{i) = ^- 132 F(s) = \'2s + 1


121 .v^ - xy^ +y = 8 at (2. 2) t I
REVIEW PROBLEM SET, CHAPTER 2 163

133 G(u) = W* + 3)^ 134 H{w) = ^V- 4>^1 + vv 151 J = 3 sin Itti

135 f(e) = e^ cos 3e 136 g(x) = ()r + 1) sin U/7) 152 =


i COS" t

In Problems 137 to 142, find the indicated higher-order derivative. 153 Find a formula for f"(x) if

if A s 1

137 f\\) if /(.v) = 8.rV- - 9x*'^


if A > 1

138 d\/dx^ if y = B.v'' - Ix- + l.r - 5x + 1


154 A particle moving along an .y axis according to an equation of
139 y" if y = (X- l)/(.v+ 1) motion of the form s =A cos (27rw - <^) + k, where A. v, <{>,

and k are constants, is said to be undergoing simple harmonic


140 /'"'(.r) if/(.v) = a„A-" + a„-iA^'"' + • • + a,A- + oo. where a„,
motion with amplitude A, frequency v, phase angle (p. and
o„- Oi. and (Jq are constants
equilibrium point k. For simple harmonic motion, show that

141 cf" (cos ax)/dx'', where a is a constant


d-s
—-T- + ^irvs = Airvk
142 §''"*"( S) if ^(6) = a sin bO, where a and b are constants dr

143 Find all higher-order derivatives of the function In Problems 155 and 156. use the intermediate-value theorem to
f(x) = 6a-' + 7a-^ - 8a-' - 9.r -H 10a + 11. show that there is a number c between the two given numbers that
satisfies the equation.
144 If /(A) = |.v|-, find formulas for/'(.v) and /"(a).

145 Suppose that/, g. and /; are functions which are differentiable 155 ( between 1 and 2. 4f^ - 7c- + 2c = Vs
at the number -2 and that /(-2) = 1, /'(-2) = -3,
/"(-2)=-4, g(-2) = 4, g'(-2)=-i g"(-2)=-3, 156 (• between and i 5 sin lire - 4 tan ttc = Q.lQl
/i(-2) = 6, h'{-2) = -8, and ;!"(-2) = 7. Find
In Problems 157 to 162, determine the intervals on which the func-
(a) (/g)"(-2) (b) (//!)"( -2) (c) (/-(-g)"(-2)
tion is positive or negative.

(d) (g-h)"{-2) (e) (/5/!)"(-2) (f) I — I (-2)


157 /(A) = 3.v(2v - 1)(3a- 2)

146 The curvature k of the graph of the function / at the point 158 g(x) = x^ 4- 8.r -I- I 1a - 20
(-V, /(a)) is defined to be
,.„ 8.r + 22a -I- 15
159 h{x)
,
= ^
/"(A) 6a- + 13a - 5
{1 + 1/'U)]-F''-
160 Fix) = 6a"/5 - 13.x'*''' + 6a'/'
The e quation o f a semicircle of radius r with center at (0. 0) is
^ — sin A
y = Vr^ - .r Find the curvature of this semicircle at the point
.
. ..
161 G(a) = 1

for -77 < A < 77


(a, \/r — X-), where -r <x< r.
1 + tan A

- = 162 H{x) = (A - 1)-/' tan a for -77 < a s 77


147 (a) If (a a}' + y- b'. where a and b are constants, show
that
In Problems 163 to 166. show that the function has a zero in the
yDly + 1 + (D_,y)- = interval by using the change-of-sign property.

(b) If .r -(-
v" = a~, where a is a constant, show that

[1 -I- (y')-]'
= a-(y")--
163 /(A) = .r' + r +X+ 5 in [-2. - 1]

148 Suppose = Vl -/{/), 0164 g(x) = 1 + 3a-'''' - IS.r-" in [4. 5]


that g is a function defined by g(t)
where /(-2) = -3. /'(-2) = 3, and /"(-2) = 5. Find EI165 h(x) = cos A - V7 in [0. 1]
«"(-2).
111166 Fix) = 3 sin A - 2 sin (.v/2) - 1 in [0.5. 0.6]

In Problems 149 to 152. a particle is moving along a horizontal El 167 In Problem 163. use the bisection method twice in succession,
s axis according to the given equation of motion. Find the velocity v starting with the interval [-2. -1]. to locate a zero of/ with
and the acceleration a of the particle. greater accuracy.

= bf - El 168 In Problem 166. use the bisection method four times in succes-
149 s 2r'
sion, starting with the interval [0.5. 0.6]. to locate a zero of F
150 s = Mr - Ibt* with greater accuracy.
164 CHAPTER 2 THE DERIVATIVE

Oln Problems 169 to 172. use a calculator and Newton's method to 1 176 In optics, it is necessary to solve the equation
find a zero of the function to as many decimal places as possible.
1 _ / sin a: \-
Start with the value of .V| as a first approximation.
T~ V X I

169 The function /of Problem 163; .r, = -1.5 in order to calculate the half-width of the central maximum for

= Fraunhofer diffraction by a single slit. Solve this equation to as


170 The function g of Problem 164; .vi 4.5
many decimal places as you can.
171 The function h of Problem 165; .v, = 0.5
177 Let
= t + 1
172 The function F of Problem 166; .v, 0.55 /(.v)
.r - 4
Sin Problems 173 and 174. u.se Newton's method to solve the equa-
Note that/(0) = -\ and/(3) = s have opposite algebraic signs.
tion. =
yet there is no number c- between and 3 such that/(f) 0.

Explain why this does not contradict the change-of-sign prop-


173 .r^ + .V = 17
erty.

174 .r^ + x = 3 - sin .t


178 Suppose that/ is a continuous function on an interval / and that

El 175 The graph of v = .v"* + 2r — 4.v + 1 has a horizontal tangent /takes on only integer values. Use the intermediate-value theo-

at exactly one point. Find the .v coordinate of this point to as


rem to conclude that / must be a constant function on the
interval /.
many decimal places as you can.
APPLICATIONS OF THE
DERIVATIVE

In Chapter 2 we saw that the derivative of a function can be interpreted as the slope
of the tangent line to its graph. In the present chapter we exploit this fact and
develop techniques for using derivatives to determine important geometric features
of the graph of a function. In Sections 3.2 through 3.5, computer- generated graphs
are used to illustrate these features. The chapter also includes graph sketching and
applications of derivatives to problems in such diverse fields as geometry, engineer-
ing, physics, and economics.

3.1 The Mean- Value Theorem


The driving distance between Kansas City and St. Louis is 250 miles. If you drive
this distance in 5 hours, then your average speed is 250/5 = 50 miles per hour. Of
course, your instantaneous speed — your speedometer reading — probably will not

remain constant during your 5-hour drive; sometimes it will be more than 50 miles
per hour, sometimes less. But it seems clear that at some point along the way, your
instantaneous speed must be exactly 50 miles per hour. You can't average m miles
per hour on a journey without attaining an instantaneous speed ofm miles per hour
somewhere along the way. This rather obvious fact corresponds to a mathematical
theorem, called the mean-value theorem, which we study in this section. (In mathe-
matics, the word "mean" is often used as a synonym for "average.")
The mean- value theorem (or law of the mean, as it is sometimes called) applies to

variable quantities in general, not just to distance, rate, and time for an automobile
trip. Indeed, let x and v be variable quantities, and suppose that y is a function of x,
say,

y=f(x)
Consider a closed interval [a. b] contained in the domain of/. As x changes from
X = aio X = b, the variable y will change from _y
= f(a) to y = f(b), and the aver-

165
166 CHAPTER 3 APPLICATIONS OF THE DERIVATIVE

Figure I

y
SECTION 3.1 THE MEAN-VALUE THEOREM 167

By the geometric version of the mean-value theorem, there is a point P = (c,f{c)) =


(f, Vc), with c between and 8, at which the slope of the tangent is equal to i;
that is.

/'(c) = i

Now,

fix) = D,V^ = D,.v'/^ = ^jc"2/3

so we want

that is, c'^^^ = f or c^^^ = i

If we cube both sides of the last equation, we get

, 64
27

so that

Since we require that c belong to the interval between and 8, we must reject the

negative solution and conclude that

8V3

/8V3 ,/8V3\
Therefore, P = (c..fic))
/( = {—^.
V 9
J—r~]

The mean- value theorem is one of the most important theorems in all of calculus.
We use it in the present chapter to help develop techniques of graphing and in

Chapter 5 in connection with the idea of an integral. For these purposes, we require
a more formal treatment of the theorem. In spite of the strong intuitive appeal of the
informal versions already given, it turns out that the proof of the mean-value theo-
rem is a bit tricky. The usual approach is to begin with the following theorem,
which is of interest in its own right.

THEOREM 1 Extreme- Value Theorem

If a function/is defined and continuous on a closed interval [«, b], then/takes


on a maximum (largest) value at some number in [a, b] and a minimum (small-
Figure 3 est) value at some number in [a, b].

Geometrically, the property of continuous functions expressed by Theorem 1 is

easy to accept since it just says that a continuous curve drawn from point A to point
fl, as in Figure 3, has a highest point C and a lowest point D. Surprisingly, a formal
proof of this obvious fact requires a rather sophisticated study of the structure of the
real number system IR and so is best more advanced courses in analysis.
left to

Using Theorem 1 , we can now prove mean- value theorem


a special case of the
which was discovered by the French mathematician Michel RoUe (1652-1719) in
1690.
168 CHAPTER 3 APPLICATIONS OF THE DERIVATIVE

THEOREM 2 Rolle's Theorem

Let/be a continuous function on the closed interval \a, h\ such that/is differen-
tiable on the open interval («, b) and /(a) = fib) = 0. Then there is at least one
number c in the open interval (a. b) such that/'(() = 0.

Figure 4
SECTION 3.1 THE MEAN-VALUE THEOREM 169

Now we are in a position to give a formal statement and proof of the


mean-value
theorem.

THEORE.M 3 The Mean- Value Theorem

If the function /is defined and continuous on the closed interval [a. b] and is
differentiable on the open interval (a. h). then there exists at least one number c
in {a. b) such that

PROOF The slope of the secant line through the two points (a. f(a)) and (b, f(b)) is

fib) ~f(a)

Figure 6

fix)
six)
170 CHAPTER 3 APPLICATIONS OF THE DERIVATIVE

KX.VMl'l.K 3 Let/(.r) = x^ + Zv" + 1. Verify the hypotheses of the mean-value


theorem for/on the interval |0, 3], and find a numerical value of c in the interval

(0, 3) such that/(3) -/(O) =/'(c)(3 - 0).

SOLUTION The hypotheses of the mean-value theorem are satisfied because the
polynomial function / is continuous and differentiable at every number. Here we
have
/(3) = 3-^ + 2(3)- -I- 1 = 46

/(O) = 0^ + 2(0)2 +1 = 1

and fix) = 3.V- + 4.v

so the condition /(3) — /(O) =/'(c)(3 — 0) is equivalent to

46 - 1 = (3c- + 4c)(3) or 9c- + 12c - 45 =

The last equation can be rewritten as

3c- -H 4c - 15 = or (3c - 5)(c -h 3) =


Thus, c = I or c = —3. Since we require that c belong to the interval (0, 3), we
Figure 7 must reject the solution c = —3. Therefore, the desired number is c = §.

In using the mean-value theorem, you must make certain that all the hypotheses
are satisfied. For instance, consider the function /(.v) = Vr^ = .v"^"^ on the interval
[—1, 1]. Figure 7 shows a computer-generated graph off. Notice that there is no
point on the graph of/ where the tangent line is parallel to the horizontal secant line
f(x) = VTi
containing the points ( — l,/(-l)) and (1,/(1)). This does not contradict the mean-
value theorem, since the hypothesis that /be differentiable on the open interval
(-1, 1) fails. Indeed, /is not differentiable at the number in this interval.

Problem Set 3.1


1 There are nt'o points, P and Q, between A and B on the continu-
ous curve^in Figure 8 at which the tangent line is parallel to the
secant AB. Does this contradict the mean-value theorem?

Figure 8
SECTION 3.1 THE MEAN-VALUE THEOREM 171

In Problems 9 to 16, verify the hypotheses of Rolle's theorem for 33 The functions whose graphs are shown in Figure 9 fail to satisfy

each function/on the indicated closed interval [a. b]. Then find all the hypotheses of the mean- value theorem on the interval from a
numbers c in the open interval (a. b) for which /'(c) = 0. to b. In each case, determine which hypothesis fails.

9 f(x) = X- - 3.r, [a, b] = [0. 3] Figure 9

10 f(x) = X- - 5.V + 6, \a. b\ = [2. 3]

11 fix) = x^ - 3.x:- - .V + 3, [a, /)] = [- 1 . 3]

12 /U) = VrCv-' - 1), [a, b\ = [0, 1]

13 fix) = A-V-* - 2v'^^ [a, b] = [0, 4]

14 fix) = x^ - 3.V, [a, fc] = I-V3, V3]

15 /W = sin .V, [a, b] = [0, 477]

16 /(.v) = Vl -cos A. [« ''I = -


y
[

y]

In Problems 17 to 26, verify the hypotheses of the mean-value


theorem for each function /on the indicated closed interval [a, b]
and then find all numbers c in the open interval (a, b) for which
f(b)-f{a)=f'{c){b-a).

17 f(x) = ix-. [a. b] = [2, 6]

18/U)=.r' +A- 1, [a. b\ = [0, 2]

19 f(x) = 2,v-\ [a, b\ = [0, 2]

20 /(A) = Va, [a, /)] = [1, 4]

1
21 /(A) = [a, fo] = [0. 3]
x+ 1

22 /(a) = Va+ 1, [a, b] = [3, 8]

23 fix) = V25 - a\ [a, /j] = [-3. 4]

= 2v- 3
24 /(A) [a, /.] = [-!, 3]
A + 4

25/(A)=A-COSA, =
[a,/,]
[y,^]
26 /(a) = 2a + 5 sin- 2v, [a. b] = [0. n]

In Problems 27 to 32, the conclusion of the mean- value theorem


fails for each function on the interval indicated. Sketch the graph of
the function, and determine which hypothesis of the mean-value
theorem fails to hold.

27 /(A) = VW, [-1, 1] 28 ^ix) ,[1,3]


A- 2

\x- +1 if A < 1
29 fix) [0, 3]
3 - A if A > 1
:

30 G(A a-EaD, [-1, 1]

31 fix) = tan A, [0, ir] 32 fix) = I


sin a|
[77 TT 1
172 CHAPTER 3 APPLICATIONS OK THE DERIVATIVE

34 Let /be a diffcrentiable funetion (and therefore continuous) at Rolle's theorem to show that /cannot have two different zeros

each number .v some open interval, and suppose that


in between 1 and 2. {Hini: If 1 <a < b<2 and /(a) = fib) = 0,

|/'U)| s 1 holds for each such .v. Use the mean-value theorem to then/' would have a zero between I and 2.|

show that \f(b) - fUi)\ s \b - a\ for any two numbers a and h in


42 Suppose that the function/ is diffcrentiable on the open interval
the open interval.

35 Complete the proof of Rolle's theorem by taking care of the case and are finite. Prove that there exists a number r in (a, b) such
in which / takes on a negative value somewhere on (a. h). - = -
that lim fix) lim,/(.v) ib u)f'(c).

36 Show that |sin x - sin y\ s |.r - y| holds for all numbers v and y
by using the result in Problem 34. 43 Let / be a second-degree polynomial function. Given any two
numbers a and b, find a formula for c strictly between a and b
37 In connection with the proof of the mean-value theorem, show - f(a) = - a)f'(c). [Hint: There
such that/(/7) {b are constants
that = Ax- + Bx + C.\
A. B, and C such that/(.v)

y =f(a) + — b ~ a
(.V - a) 44 Let F and G be two functions, both of which are continuous on
|o, b] and diffcrentiable on (a, b). Define a function/on [a, b\

is an equation of the .secant line containing the two points by the equation
{a. f(a)) and (b,f(b)). (Hint: Use the point-slope form of the
/(.v) = \G[a) - G(b)]F{x) - [F(a) - F(b)]G(x)
equation of a line.)
for a s .V < b. Verify that / satisfies the hypotheses of the
38 For all values oi x and y in the interval (-ir/l, 77/2), show that
mean-value theorem, and draw the conclusion that for some
|tan a: - tan y| s |Ar - y|.
value of c strictly between a and b.

3 In Problems 39 and 40, use a calculator and Newton's method to F'(c) F{b)-F{a)
find an approximation to a value off on the open interval (a, b) for G'U) G{b)-G{a)
which /{/7) -/(«) =f'{c)(.b - a).
(provided that the denominators do not vanish).

45 Give an alternative proof of the mean-value theorem using the


39f(x) = ,\nx.{u,b\=[j,j] function ifAx) = (.v - a)fib) - (.v - b)f{a) - {b - a)f(x) rather
than the function h in the proof of Theorem 3.
40 fix) = 2x^ + X* - 3x^ +x^ - X I. Ui.h] 1. II
46 Suppose that the function / is diffcrentiable on an open interval
41 Let/(.v) = x^ + - 5x —
2x^ 10. (a)Use the intermediate-value containing both .v and .v -I- ^x. Show that there exists a number c
theorem to show that / has a zero between and 2. (b) Use I
between ,v and x + Ax such that/(A; + A.v) = f(x) +/'(c) ^x.

Figure 1
3.2 Monotonicity and the First-Derivative Test
temperature T Many applications of mathematics involve finding where the graph of a function is

rising or falling and locating high and h>w points on the graph. For instance, sup-
pose the graph in Figure 1 represents the recorded temperature of a patient in an
intensive-care unit. In monitoring the patient's condition, the hospital staff is con-
cerned with whether the temperature is rising or falling and with the high and low
temperatures that occur over a given interval of time. On the graph, the patient's
temperature T is plotted as a function of the time I. Notice that T is increasing in the
time intervals [a, ci], Ici, c-.^l, and |c-4, c=.] and decreasing in the time intervals
Ui , C2I, [Ci, C4I, and [c's, b]. High temperatures (relative, or local, maxima) occur
at times Ci, C3, and c^; and low temperatures (relative, or local, minima) occur at
times (-> and ca.
SECTION 3.2 MONOTONICITY AND THE FIRST-DERIVATIVE TEST 173

The idea of an increasing or decreasing function is made precise by the following


definitions.

DEFINITION 1 Increasing and Decreasing Functions

Let the interval / be contained in the domain of the function /.

(i) /is increasing on / if for every two numbers a and bin I with a < bwe
have /(a) <f(b).

(ii) /is decreasing on / if for every two numbers a and bin! with a < bwe
have/(a)>/(fc).

DEFINITION 2 Monotone Function

Figure 2 A function / is said to be monotone on an interval / if it is either increasing or


decreasing on /.

To say that a function /is increasing on an interval / means that the graph of/
rises as you move to the right along the interval (Figure 2a). Similarly, to say that/
is decreasing on / means that the graph of ffalls as you move to the right along the
interval (Figure 2b).
Now. suppose that/ is a function with a positive derivative/'(.r) at each number .v

in some interval /. Then, for each value of .v in /, the tangent line to the graph of/
at the point (.v,/(.v)) is rising to the right since its slope /'(.v) is positive (Figure 3).
Each such tangent line is a good approximation to the graph of/ near the point of
tangency, so it stands to reason that this graph also must be rising to the right. For
similar reasons, a function with a negative derivative on an interval should be
decreasing on this interval. In fact, we have the following theorem.

Figure 3

THEOREM 1 Test for Increasing and Decreasing Functions

Assume that the function/ is continuous on the interval / and that/is differentia-
ble at every number in /, except possibly for the endpoints of /.

(i) If/'(.v) > for every number .v in /, except possibly for the endpoints of
/, then / is increasing on /.

(ii) If/'(.v) < for every number .r in /, except possibly for the endpoints of
/, then /is decreasing on /.
174 CHAPIKR 3 APPLICATIONS OK THE DERIVATIVE

Figure 4

zero zero
/' is positive negative

(a)
SECTION 3.2 MONOTONICITV AND THE FIRST-DERIVATIVE TEST 175

Figure 5 constant algebraic sign on the intervals (


— ^, 0) and (0, 3c). Using, say, test num-
bers — 1 and 1 in these intervals, wehaveg'(-l) = — §andg'{l) = §, which shows
undefined
'
is negative / positive that

g' is negative on (-'-^. 0)

and g' is positive on (0, ^)

Figure 5a displays this infomiation. Therefore,

g is decreasing on (
— ^, 0]

and g is increasing on [0, ^)

decreasing increasing
Figure 5b shows a computer-generated graph of ^iji.v) = .r^''''. The intervals on which
g is increasing or decreasing are indicated on the graph.
Figure 6
176 CHAKI KR J APPLICATIONS OF HIE DERIVATIVE

(a) Here, fix) = 6.v^ - 6.v - 12 = 6(.r- - .r - 2) = 6(.v + l)(.v - 2) is de-


fined for every real number v. and/'(.v) = Ofor.v = -1 and.v = 2. Hence, —1
and 2 are the critical numbers for/.

(b) Here g'(0) is undefined, so is a critical number for g. Because g'(x) =


Figure 7
-1 for .V < and g'(x) =1 for at > 0, there are no numbers x for which
^i,''{.v) = 0. Hence, is the only critical number for g.

(c) We have h'(x) = cos x for every real number .v. Now, cos a = for .v
=
± tt/2. ±377/2, ±577/2, and so on. Hence, the critical numbers for h are all the
odd integer multiples of 77/2.

After you have found all the critical numbers for a function, you must test each
such number to see whether it corresponds to a relative extremum. The simplest
tests for relative extrema make use of first or second derivatives.

The First-Derivative Test

Consider Figure 7a, in which the continuous function/has a critical number at c. a

positive first derivative on the open interval (a. c) just to the left of c, and a negative
first derivative on the open interval (c, h) just to the right of c. By Theorem 1,/
is increasing on \a. c] and decreasing on [c, b\, hence, //?«.? a relative maximum at
c. Similarly, in Figure 7b, the continuous function /has a critical number at c. a

negative first derivative on the open interval (a. c) just to the leftof c, and a positive
first derivative on the open interval (c. /?) just to the right of c. Again by Theorem 1.
/is decreasing on [a. c] and increasing on [c, b]; hence,/ has a relative minimum at

c. These observations are summarized in the following theorem.

THEOREM 3 First-Derivative Test for Relative Extrema

Let the function /be defined and continuous on the open interval (a. h), assume
that the number c belongs to {a. b), and suppose that/ is differentiable at every
number in (a, b) except possibly at c.

(i) If the derivative/' is positive on (a, c) and negative on (c, b), then/has
a relative maximum at c.

(ii) If the derivative/' is negative on {a. c) and positive on (c. b) then/


has a relative minimum at c.

In case (i) of Theorem 3, not only is there a relative maximum at <;, but also
/(c) s/(.v) holds for every x in the interval (a, b) (Figure 7a). Similarly, in case (ii)

of Theorem 3 (Figure 7b), /(c) s/(.v) holds for every x in the interval (a, b).

In Examples 4 to 7, use the first-derivative test to find all numbers at which the
function has a relative maximum or minimum.

EXAMPLE 4 fix) = .v' - Iv- + .V + 1

SOLUTION Here,/'(.v) = 3.t~ - 4.v + 1 = (3.v - l)(.v - 1), so the only critical
numbers /are the roots x = 3 and .v =
for of the equation /'(.v) = 0. To decide 1

whether /has a relative maximum or minimum at either of these numbers, we use


SECTION 3.2 MONOTONICITY AND THE FIRST-DERI\ ATI\ E TEST 177

the first-derivative test. The procedure for determining the intervals on which/' is

positive or negative shows that

/' is positive on (— ^c, §)

/' is negative on (3, 1)

and /' is positive on (1, =c)

By the first-derivative test (Theorem 3), we conclude that/has a relative maximum


at 3 and a relative minimum at 1 . Notice that/Cs) = (5)^ - 2(if + ^ + 1 = H and
/(I) = 1^ — 2(1)- -(- + = 1.
1 1 All this information is summarized in the follow-

in2 table:

Figure 8
x

f(.x)=-.x^-2x^ + x + I

relative maximum ^

relative minimum

J 1
178 t HAFfER 3 APPLICATIONS OK THE DERIVATIVE

SOLUTION Here, h'(.x) = 1


- (l/.v") for.v ^ 0. To find the critical numbers, we
set /I'W = 0, so that

—=
Thus. .V = - 1 and a = 1 arc the roots of h'(x) = 0, and it follows that - and
1 1 are
the only critical numbers for /;. Now we have the following:

Figure 10
X

relative
minimum
relative —1
maximum '^^
SECTION 3.2 MONOTONICITY AND THE FIRST-DERIN ATIVE TEST 179

Problem Set 3.2


1 For each graph in Figure 12. indicate the intervals on which the
function is increasing, decreasing, or constant.

Figure 12

1
180 CHAFIKR J APPLICATIONS OF THE DERIVATIVE

45 f(x) = 2Vt - X 46 ,i,'(.v) = .r - 3 \Xt ax' + bx^ + cx + d. Show that/ is an increasing function on IR

if and only if b^ s 3ac.


3 .V- 1

47 /i(.v) = 48 r(.v)
_ 2 .r" - 2v 58 Suppose that / is a continuous function on the closed interval

[a, b\ and that/is monotone on the open interval («, b). Prove
X- - 5 if .V < 4
49 fix)
= that /is monotone on the closed interval [a. h\.
10 - 3.V if .t > 4
-59 Let/(.r) = cos 2x + 2 sin .t for -27r < .x < 27r. (a) Find all the
V25 - .r- ifA:<4
numbers for/ Find the intervals on which
50 g{x) critical (b) / is in-
{; if x>4 creasing or decreasing, (c) Find all numbers at which /has a
Lv- + 4 if .V a 1
relative maximum or minimum.
51 lux)
1 8 - 3.V if .v< 1
El 60 In thermodynamics, the van der Waals i^as law

f.v- + lO.v + 21 if -4
.v< RT a
52 ^(.v)
Lv- + 2v - if^> -4

relates the pressure p in atmospheres of a gas. the temperature T


of the gas in kelvins. and the volume v in liters of 1 mole of the
In Problems 53 and 54. sketch a graph of a function having the gas. Here R = 8.206 x 10 ' is the universal gas constant, and a
indicated properties. and b are constants depending on the gas in question. For carbon
dioxide, a = 3.59 and b = 0.0427. For carbon dioxide at a tem-
53 /(I ) = 2,/(5) = 4, fix) < for.r < 1 .fix) > for 1 < .r < 5, perature T = 260 kelvins. consider p lo be a function of v for
and fix) < for a: > 5 \' > b. (a) Find all critical numbers for this function. {Hint: Use
Newton's method.) (b) Find the intervals on which p is increas-
54 /( 1 )
= 2,/(2) = 1 ./'(.v) > for A < 1 .fix) < for 1 < x < 2,
ing or decreasing, (c) Find all numbers at which/has a relative
and f'(x) > for .r > 2
maximum or minimum. (Caution: Don't forget that v > b. so

critical numbers less than b have no physical meaning.)


55 Suppose that / is a continuous function on the closed interval

\u. b\ and that/(«) = f(b). Show that/ has at least one critical 61 Suppose that the concentration C in milligrams per liter of a
number on the open interval (a, b). (Hint: Use the mean-value certain drug in the bloodstream t hours after it is administered
theorem.) orally is given by

56 Prove the following "converse" of part (i) of Theorem 1: If the It


function/has a continuous derivative/' on an open interval
CU) =
first + 3t + 4
/ and if/ is increasing on /, then/'(.v) > holds for ail numbers
(a) Find the interval in which C is increasing and the interval in
X in /.
which it is decreasing, (b) At what time I is the concentration C
57 Let a. h, c. and d be constants: suppose a > 0: and let/(.v) = maximum? (c) What is the maximum concentration C?

3.3 Concavity and the Second-Derivative Test


We saw in Section 3.2 that the algebraic sign of the first derivative of a function
determines whether the graph is rising or falling. Here we see that the algebraic sign

of the second derivative determines whether the graph is bending upward ("cup-
shaped") or bending downward ("cap-shaped"). We also show that the algebraic

sign of the second derivative can be used to test whether a critical number for a
function yields a relative maximum or minimum.
Figure la shows a cup-shaped graph. Notice that as the point P on this graph
moves to the right, the tangent line at P turns counterclockwise and its slope in-
creases. We say that such a graph is concave upward. Similarly, in Figure lb. the
graph is cap-shaped, and as the point P moves to the right, the tangent line at P turns
clockwise and its slope decreases. We say that such a graph is concave downward.
These simple geometric considerations lead us to the following formal definition.
SECTION 3.3 CONCAVITY AND THE SECOND-DERIVATI\T TEST 181

Figure 1

f decreasing

cap-shaped,
or
tangent line
concave downward
turning
f increasing
unterclockwise

DEFINITION 1 Concavity of a Graph

Let /be a differentiable function on an open interval /.

(i) The graph of/ is said to be concave upward on / if/' is an increasing


function on / (Figure la).

(ii) The graph of/is said to be concave downward on / if/' is a decreasing


function on / (Figure lb).

Using the test for increasing and decreasing functions (Theorem 1 . page 173). we
)'
see that if (/' is positive on an open interval, then/' is increasing on that inter\'al.
so the graph of/ is concave upward on the interval by Definition 1. Similarly, if

(/')' is negative on an open interval, then/' is decreasing on the interval and the
graph of/ is concave downward on the interval. Thus, we have the following
theorem.

THEOREM 1 Test for Concavity of a Grapli

Let the function / be twice differentiable on the open interval /.

(i) If /" is positive on /. then the graph of / is concave upward on /.

(ii) If/" is negative on /, then the graph of/ is concave downward on /.

EX.AMPLE 1 Find the intervals where the graph of/(.v) = .v-^ 9x- + 24.V - 20 is

concave upward or downward.

Figure 2 SOLUTION Here,

fix) = 3.V- - 18.V + 24

and f"(x) = 6.V - 18 = 6(.v - 3)

Therefore
/"(.Y) < for .V < 3

f"{x) > for .V > 3

Figure 2a shows this information. By Theorem 1. we may


conclude that the graph of / is concave downward (cap-
shaped) on the interval (
— ==, 3) and concave upward (cup-
shaped) on the interval (3, ^). Figure 2b shows a computer-
generated graph of /(.r) = x^ - 9x- + 24.v - 20. The inter-
vals where the graph is concave downward or upward are
indicated.
182 CHAPTER 3 APPLICATIONS OK THE DERIVATIVE

A point on a smooth graph that separates a portion that is concave upward from a
portion that is concave downward |for instance, the point (3, -2) in Figure 2b] is

called a point of inflection. At a point of intlection, a graph may "cut across its own
tangent line" (Figure 3), The notion of a point of inflection is made more precise by
the following definition.

DEFINITION 2 Point of Inflection

Figure 3
A point (r, /(c)) is called a point of inflection of the graph of a function /if
the graph has a tangent line at this point and if there is an open interval /

containing the number c such that for every pair of numbers ci and /; in / with
point of innection
CI < c < b, f"{ci) and /"(/;) exist and have opposite algebraic signs.

Intuitively, to say that {c,f(c)) is a point of inflection of/ means that the second
derivative/" "changes its algebraic sign" at c. This can happen only if either

/"(c) = or/"(r) is not defined. For convenience, we record this useful fact in the
following theorem.

THEOREM 2 Necessary Condition for a Point of Inflection

If (c, /(r)) is a point of intlection of the graph of the function /. then cither
/"(f) = or f "(c) is not defined.

Be careful! Theorem 2 does not say that the condition/"(c) = guarantees a point
of inflection at (c,f(c)) — for instance, see Example 3 below. The theorem does say
that if you are looking for a point of inflection, you need only check points (r ,/(r))

for which either /"(c) = or /"(c) is not defined.

In Examples 2 and 3, find all points of inflection of the graph of the function.

EXAMPLE 2 /(.v) = .V-' - 6.V- + 3.V + 13

SOLUTION Here,

f'(x) = 3a- - 12x + 3

and
Figure 4 -
/"(A) = 6a - 12 = 6(A 2)

Therefore, /"(a) is defined for all numbers a in IR, and the equation /"(a) = has
only one solution, namely, a = 2. Therefore, by Theorem 2, the only possible point
of inflection of the graph of/ is (2,/(2)). Because

/"(A) = 6(A - 2) < for A < 2


and

/"(a) = 6(A - 2) > for A > 2

it follows that/" really does change its algebraic sign at 2; hence, (2,/(2)) = (2, 3)
is the only point of inflection of the graph of/ Figure 4 shows a computer-generated
x' - 6.v^ + 3x+ 13 graph of /(a) = a"* — 6a- + 3a + 13. The point of inflection is indicated on the
graph.
SECTION 3.3 CONCAVITY AND THE SECOND-DERIVATIVE TEST 183

EXAMPLE 3

graph has no
point of inflection

Figure 6
184 CHAPTER 3 APPLICATIONS OF THE DERIVATIVE

THKORRM 3 Second-Derivative Test for Relative Extrema

Let the function/ be ditierenliable on the open interval /. and suppose that c is

a number in / such that /'(c) = and/'V) exists.

(i) If /"(c) > 0, then /has a relative minimum at c.

(ii) If /"(c) < 0, then /has a relative maximum at c.

(iii) If/"(c) = 0, then the test is inconclusive.

We prove (i) only, since the proof of (ii) is similar. [See Problem 48 for part (iii).]
Thus. assume/'(c) = and /"(c) > 0. By definition of/"(c) = (/' )'(c) and the fact

that /'(c) = 0, we have

(f ) (c) = lim = lim

Hence, we can make the ratio /'(.r)/(.v — c) as close as we please to the positive
number /"(c) simply by taking x sufficiently close to c (but not equal to c). In
particular, if .v is close enough to c (but different from c). then /'(a)/(.x — c) will
have to be positive. Therefore, there must be an open inters'al (a. b) containing c
such that if .V is different from c and belongs — c) > 0. to (a, b), then /'(.r)/(.r It

follows that if a< < c. then - c < and/'(.v)/(.x - c) > 0, so that/'(.v) < 0.
.v .v

Similarly, if c < a < b. we have — c > and f'(x)l(x — c) > 0. from which
.v

f'(x) > follows. Thus, slightly to the left of c the derivative /'(.v) is negative,
while slightly to the right of c it is positive. By the first-derivative test (Theorem 3.

page 176) we conclude that /has a relative minimum at c.

In Examples 4 and 5. use the second-derivative test to find all numbers at which the
function has a relative maximum or minimum.

EXAMPLE 4 /(.v) = x^ - 6x- + 9x

SOLUTION Here,

fix) = 3.V- - 12r + 9 = 3(.v - l)(.r - 3)


and
/"(A) = 6a - 12 = 6(A - 2)

To find the critical numbers for/, we set /'(a) = 0:

3(A- l)(A-3) =
Figure 8
Therefore the critical numbers are 1 and 3. Using Theorem 3. we can determine
whether these critical numbers correspond to relative maxima or minima by finding
relative the algebraic signs of the second derivative at 1 and at 3. We have
maximum
/"(I) = 6(1 - 2) = -6<0
fix) = x^ - 6.v2 + 9.V and

/"(3) = 6(3 - 2) = 6 >

relative
Hence. /has a relative maximum at 1 and a relative minimum at 3. Figure 8 shows a
'
minimum computer-generated graph of /(a) = a-^ — 6.v- -I- 9a. The relative extrema are indi-

cated on the graph.

EXAMPLE 5 g(x) = sin A


SECTION i.i CONCAVITY AND THE SECOND-DERIVATIVE TEST 185

SOLUTION Here,

g'ix) = cos A-

and g"(x) = —sin x

The critical numbers for g are the solutions of the equation g'ix) = 0. that is, of

cos .V =
Now, cos .V = if and only if a is an odd integer multiple of tt/I; that is,

x= (In
,77
+ D— = im + —
77

2 2

for « = 0, ±1, ±2, ±3, and so on. We have

g'yiTT + —) = -sin {^utt + —


But by the addition formula for the sine (inside front cover).

("^ + t) sm /!7r cos + sin — cos «7r = • + 1 •


cos hit

Therefore,

g [mr +

if n is an even integer
Now.
if n is an odd integer

- 1 if /; is even
Hence, .••(« 77 +
1 if n is odd
Therefore, by the second-derivative test, g(x) = sin x takes
on relative maxima when

Figure 9 =
niT + for n 0, ±2, ±4, ±6,

that is, when x is 77/2 plus an integer multiple of 277. Simi-


larly, g(x) = sin X takes on relative minima when

mr + /!77 -I- 77

= (n + 1)77 for M = ±1, ±3, ±5,

that is, when .v is —77/2 plus an integer multiple of 277. Fig-

ure 9 shows a computer-generated graph of g(x) = sin x.

Of course, we really didn't need to use calculus to obtain the results in the last
example — we already knew from elementary trigonometry when the sine function
takes on its maximum value of 1 and its minimum value of - 1 . However, the
technique illustrated in Example 5 can be applied to more complicated functions for
which the result isn't known in advance.
186 CHAPTER 3 APPLICATIONS OF THE DERIVATIVE

Problem Set 3.3

In Problems 1 to 34, find the intervals on which the graph of the In Problems 35 to 46, use the second-derivative test to find all num-
function is concave upward or downward. Also, find all points of bers at which the function has a relative maximum or minimum.
inflection. Sketch the graph for Problems I to 6 only. Sketch the graph for Problems 35 to 40 only.

1 fix) = x- -X- 6 2 gix) = -*- + 2t + 8 35 fix) = .V- - 5.V + 4 36 gix) = x\2x + 1)

i h(x) = (3x + 2)(1 - x) 4 Fix) = .t' + I 37 hix) = xix - 2)- 38 Fix) = 2x' + .r- + .t - 3

5 G(x) = (x+ I)' 6 Hix) = 5 + 12x - x^ 39 Gix) = x~ - x^ 40 Hix) = x^ - x~ + lx-2


7 fix) = .r' + 6.V- - 14 41 fix) = .V'' - 6.V- -I- 4

8 gix) = iv' + hx- - 2v - i 42 gix) = B.v-* -I- 8.x^ - 18.r= -H 12

9 hix) = Ir' - Ix' -lx+2 =


43 hix)
1
10 Pix) = 4.r' - 18.r- + \5x + 5

1 Qix) = .r' - 6.V- + 9.t + = X' + x+ \


1
44 Fix)

12 Rix) = x^ + 3a^- + 5
45 Gix) = sin x — cos .v

13 pix) =x'* + 4x' - 16


46 Hix) — X - sin x
14 qix) = 8.V - 2x- - x'*

47 In Figure 10, consider the graphs of the given functions and the
15 rix) = -.v-*
displayed interval [a, e]. In each case, the interval [a, e] is

16 u(.v) =jf-* + 2v' - llv- broken up into four subintervals [a. b], [b. c], [c, d], and [d, e].

Assume that the given functions are twice -differentiable on the


17 v(.r) =.r-' - 4.v' - 18.v- interior of each subinterval. Determine on which of these subin-

- tervals the given function increasing, decreasing,


18 wix) = ix-^ {x^
(i) is (ii) is (iii)

has a graph that is concave upward, and (iv) has a graph that is

19 fix) = 2x^ - 2ar' concave downward, (v) Find all inflection points.

20 gix) = 3.x' - 5x^

21 hix) = 2x + —2 Figure 10
X

22 Fix) = X + —pr
Vx
23 Gix) = X- + 5.x"-

24 Hix) = ix - l)"'^ + ix D-

25 pix) = x^'^

26 qix) = ix + IV'^-^'^

11 rix) = -r^
X' + X

28 six) = \
-\-
ix- 2)-/'

29 Pix) = -4^
X- + 4
30 Qix) = 1 + (^)'
31 Rix) = X- Vx 32/(.v) = |.v=- 1|

33 gix) = cos X 34 hix) = sin 2x


SECTION 3.3 CONCAVITY AND THE SECOND-DERIVATIVE TEST 187

if jc<2
60 g(x) = \^^l
8.V + 15 if X > 2

= if ;c< 1

61 /!(.V)
(.X - If if .V > 1

1 + -V-"- if X 7^
O h c d e 62 F{x} j

lo ifjf =

63 S{x) = sin x + esc x

64 T{x) = jf - tan x

65 If the function / is differentiable on the open interval /, some


textbooks define the graph of/ to be concave upward if/satisfies
the following condition: For every pair of distinct numbers a and
b in /, the point {b,f(b)) on the graph of/lies strictly above the
tangent lijie to the graph of/ at (a,f(a)). Show that this condition
holds if and only if, for every pair of distinct numbers a and b
inlj(b)>f(a)+f'(a)(b~a).

66 Assume that the function/ is differentiable on the open interval /


and that the graph of/ is concave upward on / in the sense of
Definition 1. If a and b axe distinct numbers in /, prove that
f(b)>f(a)+f'(a)(b-a).

67 Some textbooks define the graph of the function /to be concave


upward on the interval / provided that, for any two distinct num-
bers a and b in /, the portion of the graph between Ui.fia)) and
48 Show that if /'(c) = and/'V) = 0, as in part (iii) of Theo- (b.fXb)) lies below the secant between (a, f(a)) and (b, f{b)}.
rem 3, then the test is inconclusive. Do this by considering the Draw a diagram illustrating this condition.
three functions /(jr) = x'*, g(x) = x^, and h(x) = -x*, and take
c = 0. 68 Suppose that the function/is differentiable on the open interval /
and that the graph of / is concave upward on /. Prove that if

In Problems 49 to 64, use whatever method you wish to find all


a and b are two numbers in /, then < <
r 1 implies that
numbers at which the function has a relative maximum or minimum. flta + (l - t)b] < tfia) + a -t)f{b).

69 In educational psychology, learning curves are often used to


49 f(x) = Ix^ - 3jf- - 4
indicate a person's achievement or performance (as measured by
50 g{x) = ix^ - ix- - 6.V + 4 a standardized test) as a function of the total time spent studying
the subject in question. These curves often exhibit a point of
51 h{x) =x* -Ax diminishing returns (see page 183). Explain why study time is

52 F(x) = x* + Ax^ + 6.x:' + 4;c - being spent most efficiently at the point of diminishing returns.
1

53 G(.r) = 3.V'' - Ax^ - \lx- 70 As a result of a time-and-motion study, a manufacturer deter-


mines that between starting time (8 a.m.) and lunchtime
(12 noon), the average worker has produced p units of the prod-
54 H(x)
U+ 1)- uct after working t hours, where p = 22f f 9t^ - 3f'. (a) How
many units of the product does the average worker produce be-
55 p(x) tween 8 A.M. and 12 noon? (b) At what time in the morning does
,v + 2
the point of diminishing returns occur for the average worker?

56 q{x) = x^ - ix'^ (c) At what time in the morning is the average worker perform-
ing at maximum efficiency? (d) When the average worker is
57 r(x) = x'^^^
performing at maximum efficiency, how many units of the prod-
58 P(x) = 3x^'^ - x^'^
uct per hour is he or she producing? [c](e) Sketch the graph of p
as a function of ; for s < r 4. If you wish, a calculator may be
10-3;c iix>2 used to help determine the coordinates of points on the graph and
59
if ;t<2 thus improve the accuracy of the sketch.
188 CHAPIER i APPIJCATIONS OF THK DERIVATIVE

3.4 Absolute Extrema


People often find it necessary to determine the maximum (largest) or minimum
(smallest) value of a variable quantity. Thus, a wage earner is interested in maxi-
mizing after-tax income, an environmentalist wishes to minimize air or water pollu-
tion, a business person is concerned with maximum profit, and a conservationist
wishes to minimize energy consumption. Often problems such as these can be
translated into the mathematical question of finding the extreme values of functions.
In Sections 3.2 and 3.3, we used the first and second derivatives of a function to
locate its relative extrema. Notice, however, that such a relative maximum or mini-
mum need not be the actual largest or smallest value of the function. To emphasize
this fact, we often refer to the largest and smallest values of a function (if they exist)
Figure 1
as its absolute maximum and its absolute minimum.
Figure 1 illustrates the distinction between absolute and relative extrema. Notice

(r.Hr))
that the function /has relative maximum values at p and ;•; however, the function
value /(r) is larger than the function value /(p) — in fact,/(r) is the largest of all the
function values /(.v) for a < .v £ /?. Thus, we say iUaif takes on an absolute maxi-
mum value f(r) at r. Similarly, /has a relative minimum value at q\ however, the
function value /(fc) is smaller than the function value /(^) — in fact, /(fc) is the

smallest of all the function values/(A) for « < .v s b. Thus, we say i\\-d\. flakes on an
absolute minimum value f(b) at b.
This discussion leads us to the following general definition.

DEFINITION 1 Absolute Maximum and Minimum


Suppose that the interval / is contained in the domain of the function/, and let c
and d be numbers in /.

(i) If /(c) s/(.v) holds for all numbers a in /, we say that on the interval I
the function f takes on its absolute maximum value f(c) at the number c.

(ii) \ff{d) £/(.v) holds for all numbers .v in /, we say that on the interval I

the function f takes on its absolute minimum value f{d} at the number d.

In dealing with either absolute or relative extrema, it is essential to keep in mind


the distinctions among the number c at which the function / takes on the extreme
value; the extreme value itself, which is/(r); and the corresponding extreme point
(c,f{c)) on the graph of/.
Now, suppose you are given a continuous function/on an interval / and asked to
find the maximum and minimum values of/ and where they occur on /. To begin
with, if the adjective "relative" is not used, you are usually safe in assuming that
the absolute extrema are wanted. If / is a closed interval [a. b]. Theorem on 1

page 167 guarantees that these absolute extrema exist. An absolute extremum of/
may occur at an endpoint of the interval /, in which case it is called an endpoint
extremum. According to Definitions and 2 on page 12, an absolute extremum of
1 1

/ that occurs at a number other than an endpoint of / is automatically a relative


extremum of/. Therefore, by Theorem 2 on page 175, if/has an absolute extremum
at a number c, other than an endpoint of/, then c is a critical number for/. These

considerations provide the basis for the following.


SECTION 3.4 ABSOLITE EXTREMX 189

Figure 2
190 CHAPTKR 3 APPLICATIONS OF THE DKRIVATIVE

x- - Zr + 2 if r >
EXAMPLE 3 g(x) on [-1. il
.V- + Zv + 2 if .V <
SOLUTION To determine whether ^ is continuous on [— j, i), we examine the
one-sided limits at 0, the only number in question:

lim g{x) lim (.V- + Zv + 2) = 2

and lim, g(x) lim (A- - Zv + 2) = 2

Because these one-sided limits both have the same value, 2. we have

lim g(.v) = 2 = ,!;(0)

and so g is continuous at 0. Thus, g is continuous on |


and we can use our
procedure:

Step 1 We have

Ix-l if < .V < f


g'(x)
lx+2 if -i < A- <
To determine whether g is differentiable at 0, we calculate the derivatives from
the left and right at 0:

^-(0) and .? + (0)

Since these are unequal, g'(0) does not exist and is a critical number for g on
Figure 4 the interval (—{, i). Evidently, g'(x) = for x in the interval (-2, i) only
when V = I . Thus, in the open interval ( -i, i), the critical numbers for g are
and 1

>'=Jt2 + 2j: + 2^ Step 2 .^(0) = 2 and g(l) = 1

\ Step 3 g(-k) = (-5)- + 2(-i) -I- 2 = J and g(i) = (i)- - 2(1) + 2 = f

Step 4 The largest of the numbers g(-h) = f, ^^(0) = 2, ,i;(l) = I, and


g(i) = J is 2; the smallest is 1. We conclude that on the interval (-5, S] the
function g takes on an absolute maximum value of 2 at and an absolute
minimum value of 1 at 1. Figure 4 shows a computer-generated graph of ^t;.

2 -I-

EXAMPLE 4 /,(.v) -on (-2, 1]


A +X
SOLUTION Because the quadratic equation 2 —a -I- A" = has no real roots, the
denominator 2 — x + x^ is nonzero for all real values of a; hence, the rational
function /; is continuous on IR. In particular, it is continuous on |
— 2, 1], so we can
apply our procedure:

Step 1 Using the quotient rule and simplifying, we have

(2 - X + A-)( 1
- Zv) - (2 -I- A - a')( - 1 + Zv) 4( 1 - Zv)
h'iJc) =
(2 - A -I- A-)- (2 + x'f
Because the denominator (2 - a -I- a")- is nonzero for all real values of a, the
only critical number for /; is i. Thus, 5 is the only critical number for h in the
open interval (-2, 1).
SECTION 3.4 ABSOLITE EXTREMA 191

Figure 5 Step 2 iHh) = 7

Step 3 /;(-2) = -i and /;(!)= 1

Step 4 The largest of the numbers IH-2) = -i. h(i) = f, and /?(!)= is ?: 1

the smallest is —i. We conclude that on the interval [


— 2. 1] the function h
takes on an absolute maximum value of = at I and an absolute minimum value
of —i at —2. Figure 5 shows a computer-generated graph of /;.

r TT 2771
EXAMPLE 5 F(.v) = 2 cos x on

SOLUTION The function F is continuous on the closed interval, so we can apply


Figure 6 our procedure:

Step 1 We know that F'(.v) = -2 sin .v. Now. sin .v = ifand only if .v is an
integer multiple of tt. and the only integer multiple of 77 between —77/6 and
277/3 is 0; hence, is the only critical number of F on (
— 77/6, 277/3).

Step 2 F(0) = 2 cos = 2

Step3F(--j V3
'

iTT
F\ 2 cos
3 '

Step 4 The largest of the numbers f(-77/6) = \''3. F(0) = 2, and


F(277/3) = -1 is 2; the smallest is -1. We conclude that on the interval

[
— 77/6, 277/3] the function F takes on an absolute maximum value of 2 at and
an absolute minimum value of -1 at 277/3. Figure 6 shows a computer-
generated graph of F.

Figure 7
192 CHAPTKR 3 APPLICATIONS OF THE DERIVATIVE

In Examples 7 and 8. find the maximum and minimum vcdues (if they exist) of the
function, and indicate where they occur.

=
r - 1

EXAMPLE 7 fix)
.r + 1

Figure 8 SOLUTION Because the denominator .v" + 1 is nonzero for all real numbers, the
rational function/is defined and continuous on U. Hence, an absolute extremum of
/ would automatically be a relative extremum and could only occur at a critical
number for/. Using the quotient rule and simplifying, we have

4.x
/'(.v) =
(.r + 1 r
so is the only critical number for/. Since /'(jc) < for .t < and since /'(.v) >
for ,v > 0, it follows that / is decreasing on (-=c, 0] and increasing on [0, ^).
Therefore, /takes on an absolute minimum value of/(0) = -1 at 0, but it has no
absolute maximum value. Figure 8 shows a computer-generated graph of/.

B EXAMPLE 8 G{x) = x +

Figure 9

.
SECTION 3.4 ABSOLl'TE EXTREMA 193

(0, 64] and decreasing on [64, y-). Therefore, on the interval (0. ^). /takes on an
absolute maximum at 64. Consequently, a speed of = 64 \' kilometers per hour will
yield a maximum distance of /(64) = 256 kilometers.

In Sections 3.7 and 3.9, we undertake a more detailed and systematic study of
applied problems involving maxima and minima.

Problem Set 3.4

In Problems 1 to 32, find the absolute maximum and minimum


values of the function on the given interval, and indicate where they
occur. Sketch the graph in Problems 1 to 10 only.

1 /(.v)= -r on [-2, 1]

2 g(x) = 4.V + 3 on [0, 2]

3 h(.x) = -Ivon [-1, 2]

4 F(x) on [ 1 , 4[

5 G{x) = V4 - .V- on [-2, 2]

6 H{x) = -X- + 5.V - 4 on [0, 5]

7/(.v) = (.v+ l)-on [-2, 1[

8 g(x) = |.v- 2| + 3 on [1, 5[

if X < 1

''"^' =
U-^-\v
194 C HAPTER 3 APPLICATIONS OF THE DERIVATIVE

45/(.r)= -(.r+ Ip' 46 .uC.v) = I


- (1 +;c)'''- where P. E. and / are constants; .v is the distance in feet from the
left end; and s j: s 40. Find the maximum deflection and

48 f (.V) = ,-.,
where it occurs.

55 In medicine, it is often assumed that the reaction /? to a dose of


49 s{.x) = VI + sin- v 50 5(.v) = 1 + 2 sin .V
size x of a drug is given by an equation of the form
R = Ax-{B - .V), where A and B are certain positive constants.
51 One hundred animals belonging to an endangered species are The sensitivity of the body to a dose of size v is defined to be the
placed in a protected game preserve. Suppose that the mathe- derivative dR/d.x of the reaction with respect to the dose, (a) For
matical model what value of .v is the reaction R maximum? (b) What is the

/'
'
=

100
r +
-,
——
r + 25
5f + 25 maximum value of /f?
dR/dx maximum?
(c) For what value of .v is the sensitivity

is used to predict the population p of these animals in the pre- E156 A 4-ton weight is to be suspended from two identical cables
serve after I years. According to this model, after how many fixed to points A and B (Figure 10). The distance between A and
years is the population p a maximum? B is 36 feet, the perpendicular distance from the weight to the

Show maximum value of the function


line AS is x feet, and the cable weighs 3 pounds per running
52 that the absolute
foot. The resulting tension in the cable is given by

fix)
= 4000V324 -I- r + 972
+ |.v| \ +\.x - 4| T= + 3.V pounds
.V .V

is 3 and occurs when .v = 2.


Find the value of .t that minimizes the tension. {Hint: You will
53 The range R in meters of a projectile is given by the formula
have to use Newton"s method to find an approximation to the
R = (vg sin 20)/g. where Vn is the muzzle velocity in meters per
critical number.)
second, g is the acceleration of gravity in meters per second per
Figure 10
second, and 6 is the angle of elevation at which the projectile is

fired. Find the angle of elevation that gives the maximum range.

54 The deflection of a beam 40 feet long supported at the ends and


loaded at a point 30 feet from the left end is expressed by the
equation

P
200a-
3Er

3.5 Asymptotes and Limits Involving Infinity

In Sections 3.2 to 3.4, we discussed monotonicity, concavity, points of inflection,


Figure I
and extreme points of a graph. Information about these features can be used to help
y sketch the graph. Here, we introduce another useful concept — the idea of an asymp-
tote. This idea is illustrated in Figure 1, which shows a graph "approaching" a
straight line L in the sense that, as the point P moves along the graph, the distance
between P and L approaches zero. When this is so, we say that L is an asymptote of
the graph. Although an asymptote isn't really part of the graph, it helps us to
visualize how the graph behaves in distant regions of the xy plane.
As the point P in Figure 1 moves along the graph and approaches the line /., the
distance \0P\ between P and O becomes larger and larger without bound,
the origin
or, as we say, \0P\ approaches infinity. Thus, in order to study asymptotes, we
must introduce a new kind of limit — a limit involving infinity.
SECTION 3.5 ASYMPTOTES AND LLMITS IN\OLVTNG INFINITY 195

Figure 2
u Limits at Inflnity and Horizontal Asymptotes

fix) -
The computer-generated graph of/(.v) = l/.v in Figure 2 shows that as x becomes
larger and larger without bound, the function values
/(a:) approach 0. We express
this fact symbolically by writing

r —lim+=c —=
Y

Also. as decreases through negative values in such a way that |.t|


.V
increases without
bound, the function values /U) again approach
0; symbolically,

lim —=

Figure 3

B
196 CHAPTER i APPLICATIONS OF THE DERIVATIVE

Figure 4 In calculating limits at infinity, it is often useful to keep in mind that for any
positive integer p.
SECTION 3.5 ASYMPTOTES AND LIMITS 1N\0LV1NG INFINITY 197

SOLUTION Here the function is not rational: nevertheless, we try our trick of
dividing numerator and denominator by some power of .v. We divide by x itself, so
that the numerator simplifies to 2. Thus, we have

lim gix) = lim

Figure 6

tigure / J
198 C HAKIHR 3 APPLICATIONS OF THK DKRIVATIVE

Figure 8 also shows that as .v approaches i from the right. /(.v) increases without
bound. In fact,/(.v) can be made larger than any preassigned number simply by
taking v close enough to but greater than i. This is expressed symbolically by
'-l

writing
4
Figure 9 lim = +^
.-(-v:)- 2v - 3

In general, there are four possibilities for such one-sided innnite limits:
Urn /(.v) = -

1 The limit of/(.v) as x approaches a from the right is positive infinity: that is,

lim fix) = +3C (Figure 9a).

2 The limit as x approaches a from the


of/(jif) left is positive infinity; that is,

lim fix) = +3C (Figure 9b).

3 The limit of/(.v) as .v approaches a from the right is negative infinity; that is,

\imf(x) = -3c (Figure 9c).

4 The limit of/(.r) as x approaches a from the left is negative infinity; that is.

lim fix) = -3c (Figure 9d).

In all four cases, the vertical line x = a is an asymptote of the graph of the function
f If

lim^/(.v) = +^ and lim /(.v) = +==

we simply write

lim/(.v) = +00

Likewise, the notation

lim f(x) = -2c

is understood to mean that

\imfix) = -00 and lim /(.v) = -oc

In dealing with limits of functions of the form/(.v) = pix)/q(x). keep in mind that

if the denominator of a fraction is close to zero while the numerator is close to any
number other than zero, the fraction will tend to have a large absolute value. More
precisely:

pix)
lim pix) = L ^ and lim (/(.v) = then 1

-" 1
^U)

Naturally, the same thing holds for limits from the right or for limits from the left.

//( Examples 4 and 5. for the given function and the indicated value of a. find
(a) lim. /(.v). (b) lim /(.v), and (c) lim /(.v) // these limits are defined.

(d) Determine whether the graph of f has a vertical asymptote x = a.

EXAMPLE 4 f{x) = tan j:, a = tt/2


SECTION 3.5 ASYMPTOTES AND LIMITS INVOLVING INFINITY 199

We have tan x = sin .v/cos .v. Note that lim sin .v = 1 and
lim cos X = 0; hence.

lim

(a) For values of ,v slightly larger than 77/2, the numerator, sin x. is positive
and the denominator, cos .v. is negative: hence, the fraction sin .v/cos .v is

negative with a large absolute value. Therefore.

lim lim

(b) For values of .v slightly smaller than -/2. both the numerator and the
denominator are positive: hence, the fraction sin .v/cos .v is positive and large.
Therefore.

sin .V
lim tan .v = lim = +^
Figure 10 x^iTT/I) .v^(V2) cos JC

(c) From (a) and (b). we have

lim tan v = -^ whereas lim tan x


--( V2)-

Hence, tan x does not have a limit, finite or infinite, as


.V^ 77/2.

(d) From the result of either (a) or (b), we can conclude that
X = tt/2 is a vertical asymptote of the graph of the tangent
function (Figure 10). As a matter of fact, since tan .v repeats
itself periodically whenever .v is increased or decreased by tt,

it follows that the graph of the tangent function has periodic


asymptotes, each separated from the next by tt units.

EXAMPLE 5 fix)
4.V

-
— . a = 5
(.V 5)
Figure 11
SOLUTION The limit of the numerator is lim 4.t = 20. As .v approaches 5. the

denominator, (x — 5)~, approaches through positive values: hence, for values of v

near 5. the fraction is positive and very large. Therefore.

(a) lim fix) = +00


f(x) =
(x-sy-

(b) lim fix) = +=c

(c) lim fix) = +30

(d) The graph of/ has a vertical asymptote v = 5. As a matter of fact, the
graph of/ also has the .v axis as a horizontal asymptote (Problem 25). Fig-
ure 1 1 , shows a computer-generated graph of /.
200 CHAPTER 3 APPLICATIONS OF THE DERIVATIVE

Locating Asymptotes
As we have seen, horizontal asymptotes of the graph of a function/, if they exist,
can be located by calculating iim fix) and lim /(.v). To spot potential vertical

asymptotes .v = a of a function of the form p/q. just look for values of a for which
q{a) = 0. You must be careful, though — there may be such values of a that do not
give vertical asymptotes, nor does this procedure always yield all possible vertical
asymptotes (see Problem 63). However, and q are polynomial functions with no
if /j

common factors of degree 1 or more, so that the fraction p/q is in reduced form,
then the graph of/ = p/q has a vertical asymptote x = a if and only if « is a zero
of q.

EXAMPLE 6 Find all horizontal and vertical asymptotes of the graph of

2jr + 1

Ax)
2x2 -3a-

SOLUTION To find any horizontal asymptotes, we must calculate the limits of


f(x) as jf ^ —00 and as .v —> +==. For this purpose, we divide numerator and de-
nominator by x^, so that

2 + (l/.v-)

2 - (3A-)

This makes it clear that

lim f(x) = —2 = 1 and lim /(.v) = —2 = 1

Hence, y = 1 is a horizontal asymptote of the graph of/.


To find vertical asymptotes, we must find the zeros of the denominator. Factoring
the denominator,

2v' - 3.V = .v(2v - 3),

we find that the zeros are and 3. Now,


2v- + 1

x(lx - 3)

and for values of .v slightly smaller than 0, 2v- + 1 is positive, x is negative, and
2v — 3 is negative; hence, /(a) is positive. It follows that

lim fix) = +00

and this is enough to show that x = (the _v axis) is indeed a vertical asymptote of
the graph of/ Although it isn't needed for our present purposes, we notice that/(.v)
is negative for values of x slightly larger than 0; hence,

lim fix) = -00

For values of .v slightly smaller than S, 2v~ + 1 is positive, x is positive, and 2jr — 3
is negative; hence /(a) is negative and

lim fix) = -oo


SECTION 3.5 ASYMPTOTES AND LIMITS INVOLVING INFINITY 201

Figure 12 Therefore, the vertical line .t = f is also an asymptote of the


graph of/. We note in passing that

lim fix) = +-^


fix) = x-^onr

Figure 12 shows a computer-generated graph* of/.

y=i If a function / is given or can be rewritten in the form

fix) = m.x + b + gi.x)

where in and b are constants, m# 0, and

lim g(x) = or lim g{x) =

then the line _v = nix + /? is an asymptote of the graph of/.


Such an asymptote, which is neither horizontal nor vertical,
is called an oblique asymptote.

Figure 13
202 CHAPTER 3 APPLICATIONS OF THE DERIVATIVE

Figure 14 EXAMPLE 8 Show that the graph of/(.v) = (.v - 1


j'^"*
has a vertical tangent line at

(1. 0).

SOLUTION Here, /is continuous at 1 and

/(I + A.V) -/(I) 1(1 + A.V) - ll'/^-O


A.V A.V

(AaV
Ax
1

(A.V)-''

Therefore,

+A.v)-/(1)
.•
lim
A-v—
/('
: = I'm
A«-*o
— 1

riTl
,^
- +^
I A.V I I
(A.v)-'-^ 1

and it follows that the graph of/has a vertical tangent line x = 1 at (1 , 0). Figure 14
shows a computer-generated graph of/.

Problem Set 3.5

In Problems I to 24. evaluate the limit

1 +bx
1 1
- + « -2 +jr

5.v= - 7.V + 3
3 I
.--- 8.V- + 5.V+ I

,100 I ^99

,101 _ ,100

8/
7 lim ,, ,

9 lim (5.V- - 3.x)

II lim
— 1
^
I
.V- 1

13 lim
.1
—0

15 lim
<—

17 lim

19 lim
SECTION 3.5 ASYMPTOTES AND LIMITS INVOLVING INFINITY 203

x
204 CHAPTER .1 APPI.K ATIONS OF THE DERIVATIVE

3.6 Graph Sketching


Nearly all the graphs shown in Sections 3.2 to 3.5 were computer-generated. In
order to generate such graphs you need to be knowledgeable about graph-sketching
techniques. Ordinarily, it is not possible to generate the entire graph of a function,
so a program has to be developed to produce a suitable portion illustrating signifi-
cant features of the graph. For this purpose, methods of calculus may be used to

make a rough sketch of the desired graph. The techniques introduced in Sections 3.2
to 3.5 provide the basis for the following systematic procedure for sketching the
graph of a function /;

Step 1 Determine the domain of/, the numbers at which /is discontinuous,
and the zeros of/. Use this information and the procedure on page 152 to find
the intervals on which / is positive or negative.

Step 2 Check whether /is even. odd. or neither.


Step 3 Find all asymptotes of the graph of/.

Step 4 Calculate the derivative/', find all critical numbers of/, and apply the

procedure on page 152 to/' to find the intervals on which/' is positive or

negative. Thus, find the intervals on which / is increasing or decreasing.

Step 5 Calculate the second derivative /". and apply the procedure on
page 152 to find the intervals on which/" is positive or negative. Thus find the

intervals on which the graph of / is concave upward or concave downward.


Find all points of inflection of the graph.

Step 6 Locate all points on the graph of / at which a relative or absolute

e.xtremum occurs.

Step 7 Use the information gained in steps 1 through 6 together with the
point-plotting method to sketch the graph of/.

The steps in this procedure are more or less independent of one another, so if you
have difficulty with the details of any one step, it may be best to move on to the
ne.xt. For instance, in step 1 . if it proves difficult to find the zeros of/, it may be best
to carry out the rest of the procedure and thus obtain a preliminary rough sketch of
the graph. This rough sketch may enable you to find reasonable first approximations
to the zeros and use. say. Newton's method to
of/ Then you can go back to step 1

locate the zeros with greater precision. Finally, using this information, you can
improve the accuracy of your preliminary rough sketch.
The graph-sketching procedure given above does not exhaust cill the techniques

for sketching graphs. As you practice, you may discover some tricks of your own.
For instance, in carrying out step 1. it's usually a good idea, whenever possible, to

select as one of the test numbers for determining the algebraic signs of/. The
number /(O). which is often easy to calculate, is called the v intercept of the
graph because it is the y coordinate of the point (0./(0)) where the graph crosses
the y axis. Some additional graph-sketching methods are presented in Chapters 7
and 9.

E)/« Examples I to 5. sketch the graph of the function.

EXAMPLE I fix) = (X - 1 )-(.V - 4)


SECTION 3.6 GRAPH SKETCHING 205

SOLUTION We carry out the seven steps of the graph-sketching procedure.

Step 1 Expanding the product (x - \)-{x - 4), we see that

/(.v) = x^ - bx- + 9.V - 4

is a polynomial function; hence, it is defined and continuous at every real


number. The advantage of the factored form is that it quickly provides the
zeros of/, namely 1 and 4. Thus, /maintains constant algebraic signs on the
Figure 1 three intervals (-^, 1). (1, 4), and (4, ^). We select, say. the three test
numbers 0, 2, and 5 from these inter\'als and calculate: /(O) = -4,/(2) = -2,
{ is zero
and/(5) = 16. This information will be useful in step 7, when we are ready to
plot points. For now. it gives us the algebraic signs of/ on the three intervals
(Figure I ).

Step 2 Evidently, /(.v) = x^ - 6.v- + 9.v - 4 is neither even nor odd. so its

graph is symmetric about neither the y axis nor the origin.


Step 3 Since /is a polynomial function, its graph has no asymptotes.
Figure 2 Step 4 Here,

zero zero /'(.v) = 3.V- - llv + 9


= 3(.v- - 4.V + 3)
= 3(.v- l)(.v- 3)

Hence, the critical numbers for/ are 1 and 3. Again using test numbers, we

determine the algebraic signs of/' on the intervals (-^. 1),(1.3), and (3, ^).
as shown in Figure 2. Thus, /is increasing on (-^. I], decreasing on [1, 3],
and increasing again on [3, =c).
Figure 3
Step 5 Since /'(.v) = 3.v- - llv + 9. we have
/"is
f"(x) = 6.V - 12 = 6(.v - 2)

Hence,/" is defined at every real number, and x =


2 is the only solution of the
equation /"(.v) = 0. Evidently,/" is negative on the interval (-=c. 2) and posi-
tive on the interval (2, ^) (Figure 3). It follows that the graph of/ is concave
downward on (
— ^, 2). is concave upward on (2. x). and has a point of
inflection at (2,/(2)) = (2, -2).
Figure 4
Step 6 Applying either the first- or the second-derivative test to the critical
numbers obtained in step 4, we find that/ takes on a relative maximum value of
/(I) = at 1 and a relative minimum value of/(3) = — 4 at 3.

Step 7 Using the information in steps 1 through 6. we can already sketch a


rough graph of/ By plotting a few additional points, we can improve the
precision of our sketch until it becomes sufficiently accurate to suit our pur-
poses. Figure 4 shows our final result.

EXAMPLE 2 g(x) = X-

relative
minimum
point Step 1 Here g is defined and continuous at every real number except 0. To
find the zeros of ^?. we must solve the equation
206 CHAPTER 3 APPLICATIONS ()! THE DERIVATIVE

that is,

Thus, 1 is the only zero of ^. Using test numbers, say, -2, I, and 2, we find
Figure 5 that^(-2) = I, gii) = -J, and gil) = J. Hence, g is positive on (-x, 0),
undefined zero negative on (Oi 1), and positive on (I, ^) (Figure 5).

Step 2 The function g(x) = x~ - (l/.v) is neither even nor odd.


Step 3 We have

lim [x~ = -t-'X and hm


1-.0'

so the y axis is a vertical asymptote of the graph of g. Because

hm + 00

the graph has no horizontal asymptote.

Step 4 Here ^'(.v) = 2v + (l/.v"). To find the critical numbers of ^, we must


solve the equation

2x +

Figure 6 that is.

g' is zero undefined


lv =

Thus, the only critical number of g is


4^ "

-1 (V^)'

Figure 7
= -AV^== -0.79
g"is undefined zero
Using test numbers, we obtain the information about the algebraic signs of g'
shown in Figure 6. Thus, g is decreasing on {-x, -^\/4]. increasing on
[
— 2V4, 0), and increasing on (0, oo).
Step 5 We have
Figure 8
g"{x) = 2 - -^

so that g" is undefined at and its only zero is 1 . Using test numbers, we obtain
point of
inflection the information shown in Figure 7. Thus, the graph of g is concave upward on
(-00, 0), is concave downward on (0, 1), is concave upward on (1 oc), and has ,

2 V4 a point of intlection at (1, ^'(1)) = (1, 0).

Step 6 By either the first- or the second-derivative test, g has a relative mini-
mum value of g(-h\/4) ~ 1.89 at -i\/4.
Step 7 The graph of g(x) = x- - {l/x) is sketched in Figure 8.
SECTION 3.5 GRAPH SKETCHING 207

4a:
EXAMPLE 3 /;{.v)
1 +.V-

Figure 9
SOLUTION
Step 1 Because the denominator 1 + .v" is never zero, the rational function h
is defined and continuous at every real number. Evidently, the only zero of h
is 0. Figure 9 shows the algebraic signs of h on the intervals (-^c, 0) and
(0, =c).

Step 2 Evidently. h{-x) = -h(x). so /; is odd and its graph is symmetric


about the origin.

Step 3 We have

4.V 4/.V
lim = lim
'-=- 1 + .V- (l/.Y-) +1 0+1
SO the .V axis is a horizontal asymptote of the graph of h.

Step 4 Using the quotient rule and simplifying, we find that

Figure 10 h'(x)
4(1
2
(1 +.v-)=
zero zero
li is
so the critical numbers of /; are -1 and 1 . The algebraic signs of /; on the
intervals(-x, -i), (-1. i), and (1, =c) are shown in Figure 10. Thus, /; is
decreasing on (-x. -1], increasing on [-1, 1], and decreasing on [1, -x-).

Step 5 Again using the quotient rule and simplifying, we find that

8.Y(.v- 3)
/)"(.v) =
(1 +.V-)-'

so that h"lx) = for x = -\^. x = .v = V3.


0, and
Figure shows the 1 1

Figure 11 algebraic signs of/)" on the intervals (-=c, -Vs), (-V3, 0), (0, VS), and
(V3, ^). Thus, the graph of/; is concave upward on (-V3, 0) and (Vs, ^),
h" is zero zero zero
whereas it is concave downward on (-^. -V3) and (0, VS). Consequently,
+++ +! — +++++ the graph of /; has points of inflection at (-Vs, /!(-V3)) = (-VJ, -V3).
(0, h{Q)) = (0, 0). and (V3, /kVS)) = (V3, VS).
-V3 ^/J
Step 6 By either the first- or the second-derivative test. /; has a relative mini-
mum value of /i( - )= -2
1 at - 1 and a relative maximum value of /;( 1 )
= 2
at 1.

Step 7 The graph is sketched in Figure 12.

Figure 12
208 CHAPTER 3 APPLICATIONS OK THK DKRIVATIVE

EXAMPLE 4 F(.v) = .v'^'' (.V - 1)-/-^

SOLUTION
Figure 13 Step 1 The function F is defined and continuous at every real number, and its

zeros are and 1. Figure 13 shows the algebraic signs of F on the inter\'als
(-X, 0), (0, 1), and (1, x).
++++ +++++++ Step 2 The function F is neither even nor odd.

Step 3 The graph of F has no horizontal or vertical asymptotes. As a matter of


fact, can be shown that y = .v
it - i is an oblique asymptote of the graph of F:
however, because the proof is a bit tricky (Problem 38), we neglect this fact in
working the rest of the example.

Step 4 Calculating the derivative F' and simplifying, we obtain

3.V - 1

F\x) = 1/3
3.i-/^.v- 1)

Figure 14 Thus, F' is undefined at and 1 Also, F\x) =


. only for j: = 3, so 0, 3, and
I are the critical numbers for F Figure 14 shows the intervals on which F' is
.

F' is undefined undefined


Izero I
positive or negative. Thus, F is increasing on — x, 0], [0, 3), and [1, x),
(

++» i + ++++ ++ whereas it is decreasing on [3, 1]. Because F is increasing on the adjacent

intervals (
— =c. 0) and |0, 3), it is actually increasing on the entire interval
(— . ^].

Step 5 Calculating the second derivative F" and simplifying, we obtain

f"(-v)
9x^\x- \)'

Figure 15 Thus, F" is undefined at and 1 and has no zeros. Figure 15 shows the
/'"
intervals on which F" is positive or negative. Thus, the graph of F is concave
is undefined undefined
upward on ( — x, 0) and concave downward on (0, and ^c). Because the 1 ) ( I ,

+ + + + + + + +' graph is concave upward on -x, 0) and concave downward on (0, ), there is (
1

a possible point of inflection at the point (0, F{0)) = (0, 0). But, by Defini-
tion 2 on page 182, (0, 0) can be a point of inflection only if the graph of F
has a tangent line at (0, 0). Since F' is not defined at 0, the only possibility is

a vertical tangent line at (0, 0). Now F is continuous at 0, and we have

„ no + A.V) - F(0) (A.v)'/-'(A.v - 1)2/-'

Figure 16 lim
I

:
I

= lim
I I

A->-»o I A.V I
A.v—0 I Ajr I

= lim

relativc
maximum = +00
point
point ol
inncLlion Hence, by Definition 1 on page 201, the graph of F does have a vertical
tangent line at (0, 0). Therefore, (0, 0) is indeed a point of inflection of the
graph of F.

Step 6 By, say, the first-derivative test, the function F takes on a relative
maximum value o( F(k) = .t V4 == 0.53 at 3 and a relative minimum value of
F( = at
1 ) 1

Step 7 The graph is sketched in Figure 16.


SECTION 3.6 GRAPH SKETCHING 209

EXAMPLE 5 G(.r) = sin- t + 2 cos v for -tt < jr < tt

SOLUTION

Step 1 The function G is continuous on [


— 77, tt]. To find the zeros of G, we
must solve the equation

sin- X + 2 cos x =

for.v between —77 and tt. Using, say, Newton's method, we find that there are
two solutions, namely

.v, - -2.00 and 2.00

Figure 17 The algebraic signs of G on the intervals [-tt, .v,), (.X|, xj), and (.vi, tt\ are
shown in Figure 17.
G is zero zero
Step 2 BecauseG(-.v) = sin-(— .v) + 2 cos (— .ic) = sin"^ x + 2 cos x = G(x),
it follows that G is an even function and its graph is symmetric about the \' axis.

Step 3 The graph of G has no asymptotes.

Step 4 We have

G'{x) = 2 sin X cos x — 2 sin x


= 2 sin X (cos X — \)

Figure 18 Hence, the only critical number for G on the open interval ( - tt, 77) is 0. If we
consider the one-sided derivatives at the endpoints of the domain of G, we also
G' zero zero zero
is
have G + — 77) =
( and GL(7r) = 0. Evidently (Figure 18), G is increasing on
[-77, 0] and decreasing on [0, 77].

Step 5 Using the fact that 2 sin x cos x = sin 2x (see inside front cover), we
can rewrite G' as G'(.v) = sin 2x — 2 sin .v. Thus,

G"(x) = 2 cos 2x — 2 cos x

= 2(cos 2x — cos x)

Figure 19 Because cos 2v = 2 cos" x — 1 (see inside front cover), we can rewrite G" as

G" is zero zero zero G"(x) = 2(2 cos- .V — 1


— cos .v)

= 2(2 cos- X — cos .V — 1)

= 2(2 cos X -I- l)(cos X — \)

Therefore, G"(x) =
when cos x = —\ and when cos .v = 1 , that is (for
— 77 < jc < 77), when x = ±277/3 and when x = 0. The algebraic signs of G"
Figure 20
are shown in Figure 19. Thus, the graph of G is concave upward on
y absolute maxiiriLim (
— 77, —277/3) and (277/3, 77), whereas it is concave downward on (
— 27r/3, 0)
point
and (0, 277/3). Consequently, the graph of G has points of inflection at

(-277/3, G(-277/3)) = (-277/3, -\) and at (277/3, G(277/3)) = (27r/3, -\).


Notice that even though G"(0) = 0, the graph of G does not have a point of
inflection at (0, G(0)) = (0, 2).

Step 6 Checking the values of G at its critical number and at the two end-
points of its domain, we find that G( — 77) = —2, G(0) = 2, and G(77) = -2.
Hence, G takes on an absolute maximum value of 2 at and an absolute
minimum value of -2 at —77 and 77.

Step 7 The graph of G is sketched in Figure 20.


210 IHAPTER 3 APPLICATIONS OF THK DKRIVATIVE

Problem Set 3.6

Sin Problems 1 lo 36. sketch the graph of each function.

1 /(.v) = (.V + 1 )-(.v - 2)

2 s(.r) = .v' - 6.V- + 9.V - 4

3 /i(.r) = .v' - 3.V + 2

4 F(x) = 10 + llv - 3.V- - Ir'

5 G(.v) = 4.V- - .v^

6 H(x) = X* - .v= + 2

7 Pl\) = (3 + .v)-(l - X)-

8 Q(x) = iv' + i.v- - 2v + 1

9 /(A) = Mx' - bx' - 18.V-)

10 gix) = a' - ix"

9
1 1 /1(A) =
SECTION 3.7 APPLIED MAXIMUM AND MINIMUM PROBLEMS 211

3.7 Applied Maximum and Minimum Problems

In Section 3.4. we applied the methods of calculus to solve some practical problems
that required finding maximum or minimum values of certain quantities; however,
in all these problems, the function to be maximized or minimized was given in
advance. In we continue our study of applied maximum and minimum
this section,

problems, but now part of the problem will be to write an appropriate funcrion on
the basis of the information given in the problem. For working these problems, we
recommend the following systematic procedure:

Step 1 Attack the problem with determination! Just "have at it" and you will
often find that it is not as formidable as it appears. Begin by reading the
problem carefully (several times if necessary). Make certain that you under-
stand just which quantity is to be maximized or minimized.

Step 2 Select a suitable symbol for the quantity to be maximized or mini-


mized: for purposes of this discussion, call it Q. Determine the remaining
quantities or variables upon which Q depends, and select symbols for these
variables. If it is feasible, draw a diagram and label the various parts with the
corresponding symbols.

Step 3 Express the quantity Q whose extreme value is desired in terms of a


formula involving the variables upon which Q depends. If the formula involves
more than one of these variables, use the conditions given in the statement of
the problem to find relationships among the variables that can be used to
eliminate all but one independent variable from the formula.

Step 4 You now have Q =/(.v). where (for purposes of this discussion) .v

denotes the single variable upon which Q was found to depend in step 3 and/
is the function determined by this dependence. If there are constraints on the
quantity .v imposed by the physical nature of the problem or by other practical
considerations, specify these constraints explicitly. Apply the methods of Sec-
tion 3.4 to find the desired absolute extremum of/(A) subject to the imposed
constraints on .v.

Step 5 Reread the problem and make sure you have answered the specific
question or questions asked.

In practice, the elimination of all but one of the variables upon which Q depends
in step 3 is often the trickiest part of the procedure. Sometimes it cannot be carried
out at all because not enough relationships among these variables are given by the

statement of the problem. In this case, the procedure given above fails, and more
sophisticated methods must be used. (See Sections 14.8 and 14.9.)

E.x.wiPLE I A recreation department plans to build a rectangular playground en-


Figure 1 closed by 1000 meters of fence. What are the dimensions of the rectangular play-
ground of maximum area?

SOLUTION We carry out the suggested procedure:

Step 1 The quantity to be maximized is the area of the playground.

Step 2 Let A denote the area of the playground. Of course, A will depend on
the length / and the width w of the playground. Here the diagram is so simple
21+ :« = 1000 m that it may seem ridiculous to bother drawing it. We do so anyway (Figure I)
212 CHAPTER 3 APPLICATIONS OK THK DERIVATIVE

because it fixes ideas, displays the notation, and gives us a start on the prob-
lem.

Step 3 Evidently. A = Iw. Now, provided that we can find a suitable relation-

ship between / and w. we may be able to eliminate either / or w from the


formula A = Iw. But only 1000 meters offence are available, and the perime-
ter of the playground is 2/ -I- 2w meters; so

2/ -f- 2w = 1000 or I + w = 500


Solving the last equation for (say) /, we find that

/ = 500 - M'

Substituting / = 500 - w into A = Iw. we obtain

A = (500 - m')h' = 500h' - H'2

Step 4 We now have A =f(w), where

f{w) = 500w - w~

Since the dimensions w and / of the playground cannot be negative, we have


w> and / = 500 - H' > O; that is,

< H' < 500

(Actually, w and / must be positive, but with the closed interval [0, 500) we
can use the procedure on page 189, and no real harm comes from including the
endpoints.) We must find the value of vi- that gives the absolute maximum
value of /(m-) = 500m' - w' on the closed interval [0, 500]. Here,

f'(w) = 500 - 2w

so w = 250 number of/ on the open interval (0, 500).


gives the only critical
Calculating the valuesof/ at this critical number and at the endpoints and
500, we find that/(0) = 0./(250) = 62,500, and/(500) = 0. Hence, /takes
on an absolute maximum value of 62,500 square meters when w = 250 meters
and / = 500 - M' = 250 meters.

Step 5 The specific question was: "What are the dimensions of the rectangu-
'

lar playground of maximum area? The answer is: The playground should be '
'

a square with length and width both equal to 250 meters."

After some experience in solving maximum-minimum problems, you may de-


velop an intuition that suggests a somewhat informal treatment of the details of the
suggested procedure. In fact, people often just "set the first derivative equal to
zero" for a solution, perhaps "checking with the second derivative" to see
whether a maximum or minimum is obtained. Of course, informal solutions must be
viewed with skepticism since the desired extremum (if one exists at all) might
occurat an endpoint and the second derivative test (when it works) indicates only

a relative extremum. Thus, although we proceed more informally in the remainder


of this section, we advise you to fill in all details whenever you have the slightest
doubt.

Icj KXAMi'LF. 2 Equal squares are cut off at each comer of a rectangular piece of
cardboard 8 inches wide by 15 inches long, and an open-topped box is formed by
turning up the sides (Figure 2). Find the length .v of the sides of the squares that mu.st
be cut off to produce a box of maximum volume.
SECTION i.l APPLIED MAXIMUM AND MINIMUM PROBLEMS 213

Figure 2

SOLUTION The quantity to be maximized is the volume V of the box. From


Figure 2. we see that the height of the box is x inches, the width is 8 - Iv inches,
and the length is 15 - 2v inches. Thus,

V= A(8 - Zv)(15 - Iv) = 4.y' - 46.V- + 20.V for < .V < 4

dV
Here, 9Zv+ 120 = 4(.v- 6)(3.v- 5)

Setting dV/dx equal to zero and solving for.v, we find that a = | or.v = 6. Because
6 does not belong to the interval (0, 4), the only critical number in this interval is

f. Evidently, V takes on a maximum value of 4(f)-''


- 46(§)- + 120(1) == 90.74
cubic inches when .v = § inches.

EXAMPLE 3 A cylindrical stainless-steel boiler with an open top has a copper


bottom. If the price of the copper per square unit is 5 times the price of the stainless
steel per square unit, find the dimensions of the most economical boiler of fixed
volume V cubic units.

Figure 3 SOLUTION Let C be the total cost of the materials for the boiler. Our problem is

to find the radius r and the height h of the boiler of volume V for which C is

minimum (Figure 3). Suppose the stainless steel costs k dollars per square unit.
Then, the copper costs 5k dollars per square unit. The cost of the stainless steel for
the boiler is k times the lateral surface area:

cost of stainless steel = k •


Iirrh = 2A-7rr^ dollars

Likewise, the cost of the copper for the base is 5k times the area of the base;

cost of copper = 5k • tti" = Skirr- dollars

Thus, the total cost is given by

C = Ikirrh + 5kTrr~ dollars

Now, we must eliminate one of the two variables r or /; from the last equation. This

is accomplished by using the fact that the volume V of a cylinder is the area of the
base times its height:

V= TTT-h

Therefore,

h =
214 tHAPlER J APPLICATIONS OF THK DKRIVATIVF.

Substituting this into the equation for the total cost C. we find that

V , 2kV
C= 2kTTr\ + Skirr- = + 5kTTr-

Now. keeping in mind that V is constant, we have

dC = IkV
—+ \QkiTr
dr r-

Setting dC/dr equal to zero, we obtain the equation

2kV
iOA-Trr =

Hence, the critical value of r is


SECTION 3.7 APPLIED MAXIMUM AND MINIMUM PROBLEMS 215

under the radical in the equation for 5. Here, r > and

clQ . , , 2(3000)-
= 477-/-
dr

Setting dQ/dr equal to zero, we find only one positive critical number, namely,

3000-
277-
8.77 m

The corresponding value of /; is given by

3000
12.41 m

Again, if desired, it isn't difficult to supply a conclusive argument that these dimen-
sions correspond to an absolute minimum amount of canvas (Problem 36).

Figure 5 E] EXAMPLE 5 Ship A is 65 nautical miles due east of ship B and is sailing south at

-65 nautical miles-


15 knots (nautical miles per hour), while ship B is sailing east at 10 knots. If the

B -
ships continue on their respective courses, find the minimum distance between them
R \0t 65 10^
and when it occurs.

SOLUTION In Figure 5, P shows the original position of ship A, and R shows the
original position of ship B. After / hours have elapsed, B will have moved 10/
y 'v nautical miles while A will have moved 15/ nautical miles. By the Pythagorean
theorem, the distance v between A and B at time t is given by

v = V(15/)- + (65 - lOf)- = V325f2 - 1300/ + 4225


nautical miles. Clearly, y is minimum when the quantity

325/- - 1300/ + 4225 = 325(/- - 4/ + 13)

is minimum, that is, when the quantity

e= /- - 4/ + 13

is minimum. Here,
dQ
= 2/-4
dt

and the only critical number is / = 2. Since dQ/dt < for < < 2 and dQ/dt > ?

for / > 2, it follows that Q is decreasing for s < 2 and increasing for a 2.
/ /

Therefore, Q, hence also y, takes on its absolute minimum value when / = 2 hours.
The absolute minimum distance between the ships is therefore given by

= V30- + (65 - 20)- = V'2925 « 54.08 nautical miles

many laws state or can be reformulated so as to


In engineering and physics, —
state — motions or transformations take place in such a way that certain
that physical
quantities are maximized or minimized. For instance, in optics, the principle of
Fermat states that light follows a path that minimizes the time of transit. Pierre de
Fermat (1601-1665), a French jurist for whom mathematics was a hobby, discov-
ered this "principle of least time" in 1657. He also did important pioneering work
in analytic geometry, number theory, and the theory of probability. The following
Pierre de Fermat example shows how Fermat's principle can be used to solve a problem in optics.
216 THAPTER 3 APPLICATIONS OF THE DERIVATIVE

Figure 6
SECTION 3.7 APPLIED MAXIMUM AND MINIMUM PROBLEMS 217

that is, T=
218 CHAPTER 3 APPI.lt ATIONS OF THK DKRIVATIVE

7 A small college is planning to build a jogging track and a field Figure II


house. The jogging track is to be I kilometer in circumference
and will consist of two parallel, straight segments connected by
two semicircles (Figure 9). The field house will be built on the
rectangular plot of land enclosed by the two parallel segments of
track and the diameters of the two semicircles. Find the area of
the largest possible rectangular plot of land for the field house.

Figure 9

Held
house
13 A rectangular box with a square base is to be constmcted so as to
have a volume of I cubic meter. Find the height h and the length
.V of an edge of the base of such a box if the amount of material
used for its construction is to be minimum and (a) the box has an
open top. (b) the box has a closed top. (Neglect the thickness of
8 A wire 24 centimeters long is to be cut into two pieces. A circle
the material and extra material needed for making seams.)
will be formed from one piece and a square from the other. How
much wire should be used to form the square if the total area 14 In Problem 13, suppose that the material for the bottom and (if
enclosed by the square and the circle is to be (a) a minimum and there is one) the top of the box costs c times as much per square
(b) a maximum? meter as the material for the sides. Find the dimensions of the

A most economical box.


9 child's sandbox is to be made by cutting equal squares from
the comers of a square sheet of galvanized iron and turning up 15 A rod-shaped bacterium has the fomi of a circular cylinder of
the sides. If each side of the sheet of galvanized iron is 2 meters length / and radius r. closed at the ends by two hemispheres
long, what size squares should be cut from the comers to maxi- (Figure 12). Find the dimensions / and r of a rod-shaped bacte-
mize the volume of the sandbox? rium of fixed volume V if its total surface area is to be a mini-

10 A right circular cone is to be inscribed in a sphere of fixed radius


mum. {Hini: The volume of a sphere of radius r is sTrr"* cubic
units, and its surface area is Anr- square units.)
a units. Figure 10 shows a vertical cross section through the
center O of the sphere. Find the height h = \AC\ and the radius
r = \AB\ of the base of the cone with maximum volume V. Figure 12 L /

{Hint: Of course, your answer will depend on the radius a of the


sphere.)

Figure 10
d
16 The logo for a world's fair is to consist of a transparent right
circular cone circumscribing a sphere (Figure 13). For a sphere
of fixed radius a. find the height h and the base radius r of the
circumscribing cone of minimum volume.

\0C\= \OB\ = a Figure 13

El 11 A standard "55-gallon'" oil drum is a cylinder, with closed top


and bottom, of radius 1 1 inches and height 34 inches. Do these
dimensions minimize the amount of metal in the drum for a
given fixed volume V =
Mtt cubic inches? If 41 not, what di-
mensions would minimize the amount of metal in the drum?
(Neglect the metal used in the crimped seams of the drum.)

12 The Norman style of architecture is characterized by massive


constmction, carved decorations, and rectangular windows and
doors surmounted by semicircular arches. Find the height h and
the width w of a Norman door of maximum area if its perimeter
is 7 meters (Figure 11).
SECTION 3.7 APPLIED MAXIMUM AND MINIMUM PROBLEMS 219

17 A cable television company has its master antenna located at 21 If a resistor of R ohms is connected across a battery of E volts

point A on the bank of a straight river I kilometer wide (Fig- with an internal resistance of /• ohms, a current of/ amperes will

ure 14). It is going to run a cable from A to a point P on the flow and generate P watts of power. Here P is given by P = /-/?,

opposite bank of the river and then straight along the bank to a where / = E/(R + r). Find the resistance R if the power P is to

town T situated 3 kilometers downstream from A. It costs $15 be maxiinum.


per meter to run the cable under the water and $9 per meter to
]22 Ornithologists have determined that some species of birds prefer
run the cable along the bank. What should be the distance from
to fly over land, when possible, rather than over water. It is
P to r in order to minimize the total cost of the cable?
believed that this may have to do with unfavorable air currents

that are more prevalent over water. Suppose that a certain spe-
Figure 14 A
cies of bird, released on an island A which is 5 kilometers from
the nearest point D along a straight shoreline, flies straight to the
point B on the shoreline and then flies along the shoreline the
remaining distance \BC\ = 1 kilometers to its nesting area C,
which is 13 kilometers from point D (Figure 15). Assume that

the bird instinctively chooses a flight path that minimizes its

total energy expenditure. If the bird uses iv joules of energy per


18 In Problem 17, suppose there is an increase in the cost of laying
kilometer to fly over water and / joules of energy per kilometer
underwater cable, but the cost of laying cable along the bank
to fly over land, find the ratio iv//.
remains $9 per meter. Is there a cost for laying underwater cable
for which it is most economical to run the cable straight across

the river from point A to point Ql


Figure 15
119 An offshore oil well is located at point /4, which is 13 kilometers
from the nearest point g on a straight shoreline. The oil is to be
piped from /\ to a terminal at a point T on the shoreline by piping
it straight under water to a point P on the shoreline between Q
and T and then to T by a pipe along the shoreline. Suppose that

the distance \QT\ is 10 kilometers, that it costs $90,000 per kilo-

meter to lay underwater pipe, and that it costs $60,000 per kilo-

meter to lay the pipe along the shoreline. What should be the
distance from P to 7" in order to miniinize the cost of laying the 23 A farmer is planning to construct a silo in the form of a circular
pipe? cylinder capped by a hemisphere. Material for the hemisphere
will cost twice as much per square unit as material for the cylin-
der. Find the ratio of the total height of the silo to its radius that
provides the most economical structure for a given fixed vol-
ume. Neglect the thickness of the silo and the waste in construc-

tion.

24 Suppose that the concentration, in parts per million, of particu-

late matter in the atmosphere caused by an industrial plant is

proportional to the rate at which the plant discharges the particu-

late matter and inversely proportional to the square of the dis-


tance from the plant. If two industrial plants A and B are located

12 kilometers apart, and if plant A emits particulate matter 8


times as fast as plant S, at what distance x from A along the line

segment AB will the atmospheric concentration of particulate

matter be minimum?

25 In sociology, a simple model for the spread of a rumor predicts


that the rate R at which the rumor is spreading in a population of
size N is number of people .v
proportional to the product of the
in the population who have already heard the rumor and the

number of people N - x who have not yet heard it. According to


El 20 Ship A is sailing north at 20 knots, while ship S is sailing west at this model, what fraction of the population will have already
30 knots. At the start, ship B is 50 nautical miles east of ship A. heard the rumor when the rumor is being spread the fastest?
If they continue their respective courses, find the minimum dis- (Although X represents a whole number here, you may treat it as

tance between the ships. a continuous variable.)


220 CHAPTER 3 APPLICATIONS OF THK DKRIVATIVE

26 The strength ol' a rectangular beam is proportional to the product 030 An iron pipe of length / is to be carried horizontally around the

of its width and the square ol its depth. Find the dimensions ot corner from a corridor of width 1 meter into a corridor of width

the strongest beam that can be cut Irom a circular cylindrical log 1 .5 meters (Figure 19). Find the largest value of / for which this

(with bark removed) of radius 10 centimeters. can be accomplished without bending the pipe. Neglect the
thickness of the pipe. (Hint: Use the results of Problem 29.)
27 Light travels with speed c in air and with (slower) speed v in

water. Figure 16 shows the path by which a ray of light travels

from a point A in the air to a point B on the surface of the water Figure 19
and then to a point W in the water. Here, a, the anf;le of inci-
dence, is the angle between AB and the normal to the surface of

the water, while ft. the angle of refraction, is the angle between
BVV and this normal.

(a) Show that the total time required for the light to go from A
to W is

\/a- + r- Vfo- + (k - X)-


T= +
c V

where a = \AD\. x = \DB\, h = \EW\. and k = \DE\.

(b) Show that when x has the value that makes T minimum, then 031 A steel cable to guy an electric pole is to pass over a 3-meter
(sin a)/sin /3 = c/v (Snell's law of refraction). wall (Figure 20). If the wall is 8 meters from the pole, what is

the length / of the shortest cable that will serve this purpose?

Figure 16 {Hint: Use the results of Problem 29.)

Figure 20

28 A plastic cup in the shape of a right circular cone is to have a


slant height of 8 centimeters (Figure 17). Find the vertex angle
for which the volume of the cone will be a maximum.

Figure 17 Figure 18
032 A spherical ball of radius r settles slowly into a full conical glass
of water, causing the water to overflow. The line segments con-
necting the vertex of the cone to the points on the upper rim each
make an angle of 6 radians with the central axis of the glass. If
the conical glass has height a, find the value of r for which the
amount of overflow is a maximum.

33 A rain gutter with a cross section in the shape of an isosceles

trapezoid (Figure 2 1 ) is to have sides of length 8 centimeters and


a horizontal bottom of length 8 centimeters. If the sides make
equal angles with the horizontal, find the value of that pro-

duces a trapezoid of maximum area.

29 In Figure 18. assume that a and h are positive constants and that
0. X. and y are variables.
Figure 21
(a) Express x and y in terms of 9, a. and b.

(b) Show that x + y is minimum when tan t) = Va/h.


(c) Using the result of part (b). show that the minimum value of
x + yh (a^/' + b-''^)^'-.
SECTION 3.8 RELATED RATES 221

34 Figure 22 shows a first circle of radius a and part of a second will be proportional to the sine of a and inversely proportional to
circle of radius r whose center is on the first circle. Let / be the r: thus. I = c (sin a)/r. where c is a constant. Find the value
length of the arc of the second circle that lies inside the first of .V that maximizes /.

circle. Show that / is a maximum when the angle satisfies the


8=6. (Here a
Figure 23
equation cot is a constant, while 6 and r can
vary.)

Figure 22

38 In Figure 24, a light beam starts at the point (p, 0) on the x axis,
strikes the circular mirror x^ + y^ = 1 at the point R, and is

reflected to the point (0, q) on the y axis. Use Fermat's principle


to show that the angle of incidence a is equal to the angle of
reflection p.

Figure 24

35 Find the dimensions of the right triangle ABC with hypotenuse


\AC\= c. a given constant, if the area of the triangle is a maxi-
mum.

36 In Examples 3 and 4, pages 213 to 215, complete the formal


analyses to show that the given solutions are correct.

37 A sodium vapor lamp L is to be placed on top of a pole of height


X meters to furnish illumination at a busy traffic intersection T
(Figure 23). The foot P of the pole must be located 30 meters
from T. If r = \LT\ is the distance from the lamp to the point T
and a is the angle PTL, then the intensity of illumination / at T

3.8 Related Rates


In this section we use derivatives to solve problems involving the rates of change of
quantities that vary in time. In a typical related-rates problem, the variable quanti-
ties are related by an equation that either is given or can be derived from information
supplied in the statement of the problem. The rates of change with respect to time of
all but one of these quantities are also given, and the problem is to find the rate of
change with respect to time of the remaining quantity. This is accomplished by
differentiating the equation implicitly (see Section 2.8) with respect to time, substi-
tuting the known values of the variables and their time derivatives into the resulting
equation, and then solving for the value of the unknown time derivative. The
method is illustrated in the following examples.

E EXAMPLE 1 Petroleum from a leaking offshore well forms a circular oil slick on
the surface of the water. If the area of the oil slick is increasing at the rate of 5
square kilometers per day, at what rate is the radius of the oil slick increasing at the

instant when the radius is 3 kilometers?


222 ( 1IAP1KR 3 APPLICATIONS OF THE DERIVATIVE

Figure I SOLUTION Let A be the area of the oil shck. and let r be its radius (Figure 1).

Then

A = irr

Here the area A and the radius r are functions of time / in days since the beginning of
ihc leak. Wc want to find drlclt when r = 3. Differentiating both sides of the equa-
tion A = m~ with respect to we find that /,

clA

dt
= 2nr —
cir

dt

We are given that

dA
5 km-/day
dt

Hence, at the instant when r = 3 kilometers, we have


dr
5 = 277(3)
dt

so that — = 6v ^0.265
(//•

dt
5
km/day^

KXAMPLE 2 A guard 6 feet tall is walking away from a lamppost that is 20 feet

high. If he is walking at the rate of 4.4 feet per second, at what rate is the length of
his shadow increasing when he is 12 feet away from the base of the lamppost?

SOLUTION In Figure 2, let x denote the length in feet of the guard's shadow at

Figure . / seconds, and let v denote the distance in feet of the guard from the lamppost at t

seconds. We want to find d.x/dt when v = 1 2. In Figure 2, triangle ACE is similar to

tE
'^-5
triangle BCD; hence,

V + .V ,v

20

Therefore,
-^c = =
6y + 6x 20.V or 7.x 3y

Differentiating both sides of the last equation with respect to t. we find that

dy^
We are given that = 4.4 ft/s
dt

Hence, = 3(4.4)

3(4.4)
and it follows that 1.89 ft/s

Note that we did not need to use the information that the guard is 1 2 feet away from
the lamppost. In other words, the rate d.\/dt at which the guard's shadow is length-
ening is a constant, independent of his distance from the lamppost.
SECTION 3.8 RELATED RATES 223

Figure 3

(
224 CHAPTER 3 APPI.K ATIONS OF THE DERIVATIVE

Differentiating both sides of this equation with respect to i. we find that

dx , d\ \
-(.V- + V-) =

Figure 5 airplane

observer
SECTION 3.8 RELATED RATES 225

At the instant when 8 = ir/'i. we have esc 6 = Z/VS,


_±de___ ^00^ de_ _ 200
°^ ' rad/h
i di
~ 9 dt

(about 63.66° per minute).

[ciProblem Set 3.8


1 A stone thrown into a still pond causes a circular ripple to directly toward the pole at the constant rate of 2 meters per

spread. If the radius of the circle is increasing at the rate of 0.85 second, (a) At what rate is the person's shadow shortening?
meter per second, how fast is the area enclosed by the ripple (b) At what rate is the tip of the person's shadow moving?
increasing when the radius is 5 meters?
10 A person 6 feet tall is walking toward a wall at the rate of 4 feet
2 A culture of bacteria occupies a circular region in a petri dish. per second. Directly behind the person and 40 feet from the wall
An experimental bactericide, placed in the petri dish, causes the is a spotlight 3 feet above ground level. How fast is the length of
area of the circular region occupied by the bacteria to decrease at the person's shadow on the wall changing at the moment when
the rate of 3 square centimeters per hour. At what rate is the the person is exactly halfway between the spotlight and the wall?
radius r of the circular region changing at the instant when r = 2 Is the shadow lengthening or shortening?
centimeters'?
11 At 2:00 P.M. a ship, which is steaming north at 20 knots, is

3 A circular metal plate expands when heated so that its radius 4 nautical miles west of a lighthouse. At the same time, a motor
increases at the constant rate of 0.02 centimeter per second. At launch, traveling at 10 knots, is proceeding south from a point
what rate is the surface area of one side of the plate increasing 4 nautical miles east of the lighthouse. Find the rate of change of
when the radius is 4 centimeters'? the distance between the ship and the launch at 3:30 p.m., as-
suming that they maintain their respective speeds and courses.
4 The length of each side of a square is increasing at the constant
rate of 2 meters per second. Find the rate of increase of the area 12 Suppose that a particle is moving in the xy plane along the line

and of the perimeter of the square at the instant when its side is
y = mx + b. Show that the speed of the particle at tmie i is given
3 meters long. by V/n- + \dxldt\. 1

5 Each side of a closed cubical box is decreasing at the rate of 10


13 A television camera is located on the ground 16 kilometers from
centimeters per minute, (a) How fast is the volume of the box the point where a rocket is launched vertically. At what rate is

decreasing at the instant when its side length is 20 centimeters'? the distance from the camera to the rocket increasing at the in-
(b) How fast is its total surface area (four sides plus top and
stant when the rocket is 14 kilometers high and rising at the rate
bottom) decreasing at the instant when its side length is 20 centi- of 3 kilometers p)er second?
meters'?
14 An airplane is being pulled into a hangar by a rope from a pulley
6 Show that if each side of a cube is increasing at the constant rate
pulley 3 meters higher than the
to the nose of the plane. If the is
of 0.1 unit f>er second, then, at the instant when the side length
nose of the plane and the rope is being pulled in at a constant rate
of the cube is 10 units, the volume will be increasing at the rate
plane coming into the
of I meter per second, how fast is the
of 30 cubic units per second. Does this mean that during the next
hangar when the length of the rope from the pulley to the nose of
second exactly 30 cubic units will be added to the volume? Ex-
the plane is 12 meters?
plain.

15 A girl on a bike rides north 8 kilometers and then turns east. If


7 If the base of a triangle is increasing at the constant rate of
she rides at a steady rate of 16 kilometers per hour, at what rate
6 meters per minute and the altitude is increasing at the constant
is the distance, measured directly between the girl and her start-
rate of 2 meters per minute, what is the rate of change of the area
ing point, changing 2 hours after she leaves that point?
of the triangle at the instant when its base is 8 meters long and its

height is 10 meters? 16 In a baseball game (which is played on a square-shaped diamond

90 feet on each side) between the Brewers and the Cubs, Joe
8 The area A of a rectangle is decreasing at the constant rate of
(who is playing left field for the Brewers) hits the ball while at
9 square centimeters per second. At any instant, the length / of
bat and runs toward first base at the rate of 20 feet per second.
the rectangle is decreasing twice as fast as the width w. At a
Just as Joe starts toward first base, his teammate Tony, who had
certain instant the rectangle is a 1 -centimeter by 1 -centimeter
taken a 10-foot lead off second base, starts running toward third
square. At this instant, how rapidly is the width decreasing?
base at the same rate of speed. How fast is the distance between
9 A streetlight is on top of a pole 4 meters high. A person 1.8 Joe and Tony changing at the instant when Tony reaches third
meters tall is 6 meters from the base of the pole and is walking base?
226 ( HAnKR i APPLICATIONS OF THK DKRIVATIVE

17 A boat is nuxircd lo a dock by a rope. A person on the dock meters, and the depth of the reservoir from top to vertex is

begins pulling in the rope at the rate of 72 feet per minute. The 5 meters. The water in the reservoir evaporates at the rate of
person's hands (holding the rope) are 5 feet above the level of 0.0(X)05 cubic meter per hour for each square meter of its sur-

the bow of the boat (where the rope is fastened). How fast is the face exposed to the air. Show that, because of evaporation, the
boat moving toward the dock at the instant when 13 feet of rope water level will drop at a uniform rate (independent of the depth
is out? of the water), and find this rate.

18 A spherical snowball is melting so that its volume is decreasing 25 Sand is falling at the rate of 2 cubic meters per minute upon the
at the rate of 0. 17 cubic meter per minute. Find the rate at which tip of a sandpile, which maintains the form of a right circular
its radius is decreasing at the instant when the volume of the cone whose height is the same as the radius of the base. Find the
snowball is 0.4 cubic meter. rate at which the height of the pile is increasing at the instant

when the pile is 6 meters high.


19 A ladder 4 meters long is leaning against the vertical wall of a
house. If the base of the ladder is pulled horizontally away from 26 A cylindrical tank with vertical axis has a radius of 10 inches.

the house at the rate of 0.7 meter per second, how fast is the top The tank has a circular hole in its base with a 1-inch radius
of the ladder sliding down the wall at the instant when it is through which water pours out with a velocity of v = sV/i feet

2 meters from the ground? per second, where h is the height in feet of the surface of the
water above the base. How rapidly is the water level dropping at
20 A spherical weather balloon is being inflated with helium so that
the instant when h = 5 feet?
its volume is increasing at the rate of 0.75 cubic meter per min-
ute. At the moment when the radius of the balloon is 0.25 meter, 27 A water tank has the shape of an inverted right circular cone with

(a) how fast is the radius of the balloon increasing and (b) how a radius of 5 meters at the top and a height of 12 meters. At the
fast is the surface area of the balloon increasing? instant when the water in the tank is 6 meters deep, more water
is being poured in at the rate of 10 cubic meters per minute. Find
21 A spherical tank of radius R filled with liquid to a height h the rate at which the surface level of the water is rising at this
contains V = riTrlrOR - h) cubic units of liquid (Figure 6). A instant.
spherical gasoline tank has a radius of/? = 20 feet. Gasoline is

being pumped out of this tank at the rate of 200 gallons per 28 A water tank has the shape of an inverted right circular cone with

minute. (One gallon is appro.ximately 0. 134 cubic foot.) At what a radius of R units at the top and a height (from vertex to top) of

rate is the level of gasoline in the tank dropping at the instant


H units. A small hole with cross-sectional area k square units

when h = 5 feet?
allows water to leak out of the tank at the vertex with a velocity
of \'2gh units per second, where the depth of the water in the

tank is h units and ,? is the acceleration of gravity in units per


Figure 6
second squared. If water is being pumped into the tank at a

uniform rate of c cubic units per second, find a formula for the
rate at which the water level is changing.

29 A certain microorganism has a spherical shape and a density of


1 gram per cubic centimeter. It grows by absorbing nutrients
through its surface at the rate of A( 10 *) gram per hour, where A
is its surface area. If its radius is now 10 '
centimeter, what will
be its radius after 2 hours?

30 If the volume of 1 unit of mass of a substance at the temperature


22 Water isbeing pumped at the rate of .5 cubic meters per minute 1

into a swimming pool 20 meters long and 10 meters wide. The


6 is V. then dV/dO is called the coejficieni of cubical expansion
of the substance. The volume of I gram of water is given by
depth of the pool decreases uniformly from 7 meters at one end
V = I 4- (8.38)(IO ")(0 - 4)- cubic centimeters, where O is
to 1 meter at the other end. How fast is the surface level of the
the temperature of the water in degrees Celsius. Find the coeffi-
water rising at the instant when its depth at the deeper end is 6
cient of cubical expansion of water at IO°C and the rate of
meters?
change of the volume of 1 gram of water at 10°C if its tempera-
23 A horizontal eaves trough is 20 feet long and has a cross section ture is decreasing at the rate of 1.5°C per minute.
in the shape of an isosceles triangle 8 inches across at the top and
31 Boyle's law for the expansion of a gas held at constant tempera-
10 inches deep. Because of a heavy rainstorm, the water in the
ture states that the pressure P and the volume V are related by the
trough is rising at the rate of 0.5 inch per minute at the instant
when 5 inches deep. How fast is the volume of water
equation PV = C. where C is a constant. Suppose that a sample
it is in the
of 1000 cubic inches of gas is under a pressure of 150 pounds per
trough increasing at this instant?
.square inch, but that the pressure is decreasing at the instantane-

24 Water is stored in a reservoir in the .shape of a right circular cone ous rate of 5 pounds per .square inch per second. Find the instan-
with an open base facing upward. The radius of the base is 20 taneous rate of increase of the volume.
SECTION 3.8 RELATED RATES 227

32 Wildlife biologists studying the predator-prey relationship be- 40 A weight is lifted by a rope that passes over a pulley and down to

tween owls (the predator) and field mice (the prey) in a certain a truck that is moving at 5 feet per second. If the pulley is 100
habitat have determined that the owl population v and the field feet above the level of the truck, how fast is the weight rising
mouse population v obey the predator-prey model when the rope from the pulley to the truck makes an angle of
- 800)- - 50)- = 100,000 7r/4 radian with the ground?
(.V -I- 400(,v

When the owl population is 55. the field mouse population is 41 In an isosceles triangle, whose equal sides are each 3 centimeters
1 100, and the field mouse population is increasing by 40 field long, the vertex angle increases at the rate of 77/90 radian per
mice per month, how fast is the owl population changing ac- second. How fast is the area of the triangle increasing when the
cording to this model? vertex angle is 7r/3 radians?

42 A man is walking along a straight sidewalk at the rate of 2 me-


In Problems 33 to 37. consider the triangle ABC (Figure 7). Assume
ters per second. A searchlight on the ground 12 meters from the
that 6 is decreasing at the rate of ^ radian per second. Find the
sidewalk is kept trained on him. At what rate is the searchlight
indicated rate of change by using the given information.
revolving when the man is 7 meters away from the point on the
sidewalk nearest the light?
Figure 7
43 An airplane is flying horizontally at a speed of 400 miles per
hour at an elevation of 10,000 feet toward an observer on the
ground. Find the rate of change of the angle of elevation of the
airplane from the observer when the angle is 7r/4.

44 The hands of a tower clock are 4.5 and 6 feet long. How fast are

the tips of the hands approaching each other at four o'clock?


33 dy/dt when 6 = 7r/3 given that .v is a constant equal to 1 2 centi-
45 The angle of elevation of the sun is decreasing at the rate of 14
meters
degrees per hour. What is the rate of increase of the length of the
34 dz/dr when d = 7r/4 given that y is a constant equal to lOVz shadow cast by a tree 4 meters tall when the angle of elevation of
centimeters the sun is 7r/6?

35 dx/di when y = 20 centimeters given that r is a constant equal to 46 The crankshaft of an engine is turning at a constant rate of N
40 centimeters revolutions per second (Figure 9a). Suppose that the arm OA is a
units long and the connecting rod AP is b units long. Let 8 be the
36 dz/dt when y = a if v remains 1 .6 kilometers at all times
angle through which the crankshaft has turned from the top-
37 dz/dt when .v = 1 meter and z = 2 meters if .vand y are both dead-center position of the piston, (a) By considering Figure 9b,
changing and \' is increasing at the rate of A meter per second show that the distance .v = \OP\ units between the center of
the crankshaft O and the wrist pin P is given by the equation
38 A parachutist is descending at a constant (but unknown) velocity .V = a cos + Vb- - a- sin" 0. (b) Using the resuh of part
from an unknown height directly toward the center of a horizon- (a), obtain a formula for dx/dt in terms of a, b, 0, and N.
tal circular target of unknown radius (Figure 8). She carries a (Hint: dO/dt = 27rA' radians per second.)
small optical device that measures the angle 6 (in radians) sub-
tended by the circular target and calculates the quantity T given Figure 9
by r = (sin e)/{de/dt). Show that she will hit the target in T
seconds.

Figure 8 parachutist

/ circular target

39 A ladder 3 meters long leans against a house. The upper end


slips down the wall at the rate of 1.5 meters per second. How
fast is the ladder turning when it makes an angle of Tr/b radian
with the ground?
228 C H APIKR 3 APPLICATIONS OF THK I)KRI\ ATIVE

47 A television crew is televising a 100-mctcr dash I mm a position If the target is moving, a Doppler radar can also determine the
10 meters from the track and in line with the finish line (Fig- rate of change dr/cll of the range by measuring the frequency
ure 10). At the instant that the winner of the race is 10 meters change of the reflected signal caused by the Doppler effect. As-
from the finish line, the camera, focused on her. is turning 0.3 suming that the target T is moving along a straight line (called its

radian per second. How fast is she running? ira(k), show that by using two Doppler radars located in known
positions, it is possible to determine the speed of the target and

Figure 10 finish its track.

Figure II
^•'target

48 A radar station R can determine the ran^e r = \RT\ of a target T /j^'radar


and the bearing angle 6 of the target from due north (Figure 11). station

3.9 Applications to Economics and Business

Although calculus was originally developed by people interested in astronomy,


mechanics, physics, and engineering, it is currently finding more and more applica-
tions in the worlds of economics, manufacturing, advertising, and marketing. Econ-
omists and business managers routinely use the methods of calculus to study rela-
tionships among such variables as advertising costs, demand for goods and senices,
intlation rates, interest rates, inventories, prices, production costs, production lev-
els, profits, revenues, supplies of goods, supplies of raw materials, taxes, unem-
ployment, and wages. Here we can give only a brief survey of one aspect of the
application of calculus to economics and business.
We consider a producer supplying a particular commodity (goods or services) to
consumers. We understand that the producer could range from a small manufactur-
ing plant to an entire industry. As our basic independent variable, we choose the
number .v of units of the commodity produced in a given period of time by the
producer in question. We assume that the variable .v. which is called the production
level, is under the control of the producer and that

< .V < M
where M is the maximum possible production level. In order to be able to apply the
methods of calculus, we treat .v as if it were a continuous variable.
We denote by C(.v) the cost in dollars to the producer for producing .v units of the

commodity in the given period of time. The portion of the total cost of production
C(.v) that is independent of the production level .v is called the fixed cost. This fixed
cost may include capital expenses for new equipment, depreciation on existing
equipment, insurance premiums, salaries of permanent employees, taxes, advertis-
ing costs, and so forth, as well as overhead expenses for rent, lighting, and heating
of the plant. The remaining part of the total cost of production — the part that
SECTION 3.9 APPLICATIONS TO ECONOMICS AND BUSINESS 229

depends on the production level — is called the variable cost. This variable cost
may include the cost of raw materials, the cost of fuel and electricity actually used
in the manufacturing process, costs of packaging, wages for part-time workers,
overtime wages for permanent workers, and so forth. Thus,

C{x) = the fixed cost + the variable cost

If we assume that the variable cost is zero when the production level is zero, then we
have

C(0) = the fixed cost

and therefore.

C{x) — C(0) = the variable cost

To specify a realistic cost function C, it is necessary to determine both the fixed


and the variable costs. Usually, the fixed cost can be found by consulting suitable
accounting records. The variable cost may be more difficult to determine — past
production records must be studied; changing costs of raw materials, energy, and
labor at various production levels must be considered; and differences in the effi-
ciency of plant operation at various production levels must be taken into account.
Often, an exact formula for variable cost cannot be found, in which case a reason-
able approximation, based on experience and insight, must be used.
In economics, the derivative of a function is usually referred to as a marginal.
Thus, the derivative C of the cost function is called the marginal-cost function,
and C'(.v) is called the marginal cost at production level .v. Hence, the marginal
cost is the rate at which the production cost changes per unit change in production.
In practical situations, the production level x is usually a rather large number — so
large that, in comparison to x. the number 1 can be considered to be very small.
Then we have

C(.v + A.Y) - C(.v) Clx+\)-C(x)


C'(x) = hm == '— = C(x + \)~ C(x)
d.v^O \x 1

In other words, when the production level x is rather large, the marginal cost C'(x)
can be regarded as a good approximation to the cost C(x + \) — C(x) of producing
one more unit. Under ordinary circumstances, an increase in production level will
result in an increase in cost; that is, C is an increasing function, and the marginal
cost C'(.v) is always positive.
Often, as the production level increases, the manufacturing process becomes
more efficient and, at higher production levels, it costs less to produce one more
unit of the commodity than at lower production levels. When the most efficient
production level is reached, the marginal cost will have a minimum value. Above
this most efficient production level, the advantages of mass production begin to be
offset by such factors as stress put on workers and production machinery, scarcity of
raw materials in large quantities, the necessity of paying overtime wages, and so
forth.

1 EXAMPLE 1 Decibel Records is planning to produce a new album featuring re-


cording star Jason Dean. The accounting department estimates the fixed cost (in-

cluding the recording session) on a daily pro rata basis over the projected production
lifetime of the album to be $500 per day. On the basis of experience with previous
230 CHAPTER .1 APPLICATIONS OF THE DERIVATIVE

albums, management projects that the variable cost for the production of v albums
per day (including royalty payments) is given by

tir' - b.x~ + ex dollars for < .v s 8000

where a = 1.25 x 10"^ fo = 1.5 x 10~\ and c = 7.

(a) Find the cost function C(.v).

(b) Find the marginal-cost function C'(.v).

(c) Find the exact cost of manufacturing the 1000th album.

(d) Use the marginal cost to find the approximate cost of manufacturing the
1000th album.

(e) Find the most efficient production level.

.SOLUTION

(a) C(.Y) = 500 + ax^ - bx- + ex


= (1.25 X 10"'').r' - (1.5 X 10~-').r + 7.x + 500
for < .V < 8000
(b) C'(.v) = (3.75 X 10"^).r - (3 x 10^').v + 7 for < .v < 8000
(c) The exact cost of manufacturing the lOOOth album is given by
C(IOOO) - C(999) = 4.376125 dollars.

(d) C'(999) « 4.37725 dollars

(e) To find the minimum value of C'(.v), we find its derivative: C"(x) =
(7.5 x 10"^).v - (3 x 10"'). Thus, the only critical number for C on the
interval (0. 8000) is

3 x 10"
4000
7.5 X 10'

For < .V < 4000. C"(.v) < 0. whereas for 4000 < .v < 8000. C"(x) > 0.

Therefore. C is decreasing on the interval (0, 4000) and increasing on the


interval (4000, 8000). It follows that C'(.v) has an absolute minimum value
when the production level is 4000 albums per day.

The revenue R(q) is the total income to the producer from the sale of q units of
the commodity. For instance, if each unit of the commodity sells forp dollars, then
the revenue from the sale of q units is given by

R{q) = pq dollars

Often q. the number of units sold, will depend on the price /; per unit. Ordinarily,
the higher the price p. the smaller the number q of units sold, and vice versa. A
demand equation relates the price p per unit of the commodity and the number of
units (/ that will be demanded at that price by consumers in a given period of time. If

we assume that this demand is met by the producer, then can take q = .v, the
we
production level. For the remainder of this section, we make this reasonable as-
sumption.
By means of suitable market researeh. it may be possible to determine a demand
equation relating the price p per unit and the demand .v units of the commodity. In

this equation, we may regard p as a (perhaps implicity defined) function of .v. The
revenue function R is then detemiined by the formula

R(.\) = p\
SECTION 3.9 APPLICATIONS TO ECONOMICS AND BUSINESS 231

EXAMPLE 2 Market research conducted by Decibel Records indicates that 3000


albums per day will be sold at a price (to dealers) of $8 per album. (Dealers will add
their own markup to arrive at the retail price.) For each $0.50 increase in the
wholesale price, it is projected that the resulting increase in the retail price will
result in 500 fewer album sales per day.

(a) Find the demand equation for the albums.

(b) Find the daily revenue R{.x) dollars to Decibel Records at production
level X.

SOLUTION

(a) Let n denote the number of $0.50 increases in the price per album above
the price of $8 per album. Then the price p per album is given by

p = 8.00 + 0.50« dollars

and the daily demand .v for albums is given by

.V = 3000 - 500«

From the last equation, we have

3000 -X
n =
500

Substituting this value of n into the equation for p, we obtain the demand
equation

3000 - .V \
p = 8.00 + 0.50
500
X
= 11
1000

(b) Since R{x) = px, we have

R(x) = ll.v
1000

The marginal revenue R'(x) is the rate at which the revenue changes per unit
change in x. Again, for large values of .v, the marginal revenue R'{x) is a good
approximation to the additional revenue R(x + 1) - R{x) generated by the sale of
one more unit of the commodity.

1 EXAMPLE 3 Let R{x) = 1 U - (jr-yiOOO) dollars be the daily revenue to Decibel


Records if x albums per day are produced and sold, < .v < 8000.

(a) Find the marginal revenue R'(x).

(b) Approximately how much additional revenue per day would be generated
by increasing the daily production level from 3000 to 3001 albums?

(c) For what daily production level x will the revenue be maximum?

(a) R'ix) =11- —— for < .v < 8000

(b) R'{30m =11 = $5


232 CHAPTER .1 APPLICATIONS OK THE DERIVATIVE

(c) Setting R'(x) = 0. we find that the only critical number for R on the inter-
val (0, 8000) is 5500. Checking the values ot'Ri.x) at this critical number and at
the two endpoints of the interval [0, 8000], we find that R{.x) takes on a

maximum value of $30,250 per day when 5500 albums per day are produced
and sold to retailers.

Actually, it's usually not the revenue that a producer wants to maximize — it's the
profit, that is, the revenue minus the cost of production. If C(.v) is the cost of
producing .v units of a commodity and R(x) is the revenue from the sale of these x
units, then the profit P{,x) from the sale is given by

Pix) = Rix) - C(.v)

Again, if the production level x is large, the marginal profit P'(.v) can be regarded
as the approximate additional profit P(x + 1) - P{x) generated by the manufacture
and sale of one more unit of the commodity.

[SEXAMPLE_4 For Decibel Records, suppose that the daily cost and revenue func-
tions for Jason Dean albums at a production level of .v albums per day are given by

C(x) = (1.25 X 10~^).r^ - (1.5 X 10"-^).v^ + 7.v + 500 dollars

and Rix) = 1 l.v - ( 10 ').v- dollars for < .v s 8000

(a) Find the daily profit P(x] for the manufacture and sale of .v albums.

(b) Find the marginal profit P'(x).

(c) For what daily production level x will the profit be maximum?
(d) What price per album should be charged by Decibel Records when the
production level is set to yield maximum profit?

SOLUTION

(a) P(x) = ;?(.v) - C(x)


= -(1.25 X 10"^).v^ + (5 X 10"'').v- -I- 4x - 500 dollars
for < .V < 8000
(b) P'(.v) = -(3.75 X 10-').x- + (lO^-').v -1-4 for < .v < 8000
(c) Using the quadratic formula and a calculator, we find that the roots of the
quadratic equation P'(x) = are (approximately) -2194 and 4861 Rejecting .

-2194 because it does not belong to the interval (0, 8000), we find that 4861 is

the only critical number for P in the interval (0, 8000). Checking the values of

P(x) at this critical number and at the endpoints of the interval [0, 8(X)0], we
find that a maximum profit of $16,400.89 per day is achieved by manufactur-
ing and selling 4861 albums per day.
(d) Since R(x) = px. we have

p = —^^ = 1 1 - ( 10~').v dollars


.V

Hence, when .v = 4861, the price per album is

11 - (10--')(4861) = 6.139 dollars

A negative profit is called a loss. At a production level for which the profit is

zero, the producer is said to break even.


SECTION 3.9 APPLICATIONS TO ECONOMICS AND BUSINESS 233

3 EXAMPLE 5 Using the profit function for Decibel Records obtained in Exam-
ple 4a, find the production levels for which the producer breaks even.

SOLUTION Using Newton's method to find the (approximate) zeros of the profit
function P in the interval [0, 8000], we find that production levels of 123 and 7958
albums per day cause the producer to break even. Production levels of fewer than
123 albums per day or more than 7958 albums per day will result in a net loss for the
producer.

A study of cost, revenue, and profit (illustrated for Decibel Records in the previ-
ous examples) is called a marginal analysis because of its reliance on marginals
(that is, derivatives) to investigate changes in cost, revenue, and profit at various
levels of production. In conducting a marginal analysis, you should consider the
following questions:

1 What period of time is involved when we speak of production level xl Is it

units per day, per week, per production run, or what? In what units is the
commodity measured? What is the maximum possible production level?

2 What is the fixed cost (prorated over the period of time involved in the
specification of the production level)? What is the variable cost at production
level x't What is the cost C(x) dollars? What is the marginal cost C'{x)l What
is the most efficient production level?

3 What is the demand equation for the commodity? Whai is the rcenue R(x)
dollars? What is the marginal rcenue k (x)'l For w.iat production le-.cl will

the revenue be maximum?


4 What is the profit P(x) dollars at production level xl What is the marginal
profit f'(.v)? For what production level will the profit be maximum? According
to the demand equation, what price per unit should be charged by the producer
when the production level is adjusted to yield maximum profit? At what pro-
duction level or levels does the producer break even?

Problem Set 3.9

]In Problems 1 to 4, assume that C(x) is the cost in dollars of produc- per unit if x units are produced. In Problems 1 and 3, find
ing x units of a certain commodity during a particular interval of C(900).
time, (a) Find the fixed cost, (b) Find the variable cost, (c) Find the
6 Under normal economic circumstances, it often works out that
marginal cost C'(.v). (d) Find the exact cost of producing the 901st
unit of the commodity, (e) Use the marginal cost to estimate the cost
the production cost per unit of a commodity is minimized by
setting the production level so that the production cost per unit is
of producing the 901st unit of the commodity, (f) Find the most
equal to the marginal cost. Explain why.
efficient production level.

7 Under normal economic circumstances, it often works out that


1 C(x) = -.V- + 2500.V + 10,000, < .v < 1000
the most efficient production level is the abscissa of a point of

2 C(x) = 150 + 8V^, s jc < 1500 inflection of the graph of the cost function. Explain why.

3 C(x) = 0.00003.r' - 0. 18.r + 500,v + 5000, < .x < 2500 8 Under normal economic circumstances, it often works out that
the production level for which minimum production cost per unit
26,000.v + 150.000
4 C(.v) 0: 1500 is obtained somewhat
is larger than the most efficient production
3.Y + 1000 level. Explain why.

5 If C is a cost function, the corresponding average-cost function 9 A production-cost analysis conducted by the Mattoon Steel
C is defined by C(x) = C(x)/x. Thus, C(x) is the average cost Company, Inc., shows that the fixed cost for a production run of
234 CHAP! KR i APPLICATIONS OF THE DERIVATIVE

a new alloy is $100,000 and the variable cost for a production El 14 Market research conducted by Gainesburg Tire and Rubber indi-
run of .V tons of the alloy is tire it will sell 3000 dirt-bike tires
cates that at a price of $24 per
per week; however, each $1 increase in the price per tire will
- 3.r + 4()()0a dollars forO: 1000 result in the sale of 500 fewer tires per week, (a) Find the de-
900 mand equation for the tires, (b) Find the revenue Rix) in dollars

(al Find the cost function C(.v). (b) Find the marginal cost C'(.v). if .V tires per week are manufactured and sold, (c) Find the mar-

(c) Find the exact cost of producing the 500th ton of the new ginal revenue R'tx). (d) Approximately how much additional

alloy during a production run. (d) Use the marginal cost to find weekly revenue would be generated by increasing the production
the approximate cost of producing the 500th ton of the new alloy level from 1000 to 1001 tires per week? (e) For what production

during a production run. (e) Find the most efficient production level will the revenue be maximum?
level for the new alloy.
El 15 Market research conducted by Solar Electronics indicates that at
110 The Gainesburg Tire and Rubber Company, Inc.. manufactures a price of $60 per unit, it will sell 2000 personal walkaround

tires for dirt bikes. The fixed cost for production of these tires stereo cassette players per day; however, for each $5 increase in

per week is $15,000 and the variable cost for production of .v


price per unit, 250 fewer units will be sold per day. (a) Find the

tires per week is


demand equation for the walkarounds. (b) Find the daily revenue
^(.v) for the manufacture and sale of x walkarounds per day.
(2 X 10'^).r' - (3 X 10 -').r + 20.V dollars how
(c) Find the marginal revenue R'ix). (d) Approximately
for 10.000 much additional daily revenue would be generated by increasing

(a) Find the cost function Clx). (b) Find the marginal cost C'{x). the production level from 200 to 201 walkarounds per day?

(c) Find the exact cost of manufacturing the 1001st tire, (d) Use (e) For what production level will the revenue be maximum?
the marginal cost to find the approximate cost of manufacturing E 16 Market research conducted by the Molar Brush Company indi-
the 1001 St tire, (e) Find the most efficient production level for cates that 6000 toothbrushes per day will be sold at $0.90 per
the tires. toothbrush, but that each $0.25 increase in the price per brush
will result in 1000 fewer sales per day. (a) Find the demand
mil Solar Electronics manufactures personal walkaround stereo cas-
sette players. The fixed cost per day for producing these equation for the toothbrushes, (b) Find the daily revenue R(x) if

.V toothbrushes per day are manufactured and sold, (c) Find the
walkarounds is $4000 and the variable cost for the production of
.V walkarounds per day is
marginal revenue R'ix). (d) Approximately how much addi-
tional daily revenue would be generated by increasing the pro-
(5 X 10"^).r' - (1.5 X 10 -).v- + 40.v dollars duction level from 5000 to 5001 toothbrushes per day? (e) For
for < .V < 3000 what production level will the revenue be maximum?
(a) Find the cost function C{.x ). (b) Find the marginal cost C'(.v).

(c) Find the exact cost of manufacturing the 201st walkaround. Eln Problems 17 to 20. use the indicated information to find (a) the
(d) Use the marginal cost to find the approximate cost of manu- profit P{x) at production level .v, (b) the marginal profit P'(x).
facturing the 201st walkaround. (e) Find the most efficient pro- (c) the production level for which the profit ismaximum, (d) the
duction level for the walkarounds. price per unit that the producer should charge when the production
level is adjusted to yield maximum profit, and (e) the production
12 An analysis of production costs conducted by the Molar Brush
level(s) for which the producer breaks even.
Company shows that the fixed cost per day for manufacturing
toothbrushes is $500 and the variable daily cost for manufactur-
17 The information in Problems 9 and 1 3 concerning the production
ing .V toothbrushes is 30V\v dollars for < j.- < 10.000. (a) Find
of the new alloy by Mattoon Steel.
the cost function C(x). (b) Find the marginal cost C'(.v). (c) Find
the exact cost of manufacturing the 5001st toothbrush, (d) Use 18 The information in Problems 10 and 14 concerning the produc-
the marginal cost to find the approximate cost of manufacturing tion of dirt-bike tires by Gainesburg Tire and Rubber.
the 5001st toothbrush, (e) Find the most efficient production
19 The information in Problems II and 15 concerning the produc-
level for the toothbrushes?
tion of personal walkaround stereo cassette players by Solar
13 Market research conducted by Mattoon Steel indicates that for Electronics.
each production run of its new alloy, it will sell 600 tons at
20 The information in Problems 12 and 16 concerning the produc-
$4000 per ton; however, for each $500 increase in the price per
tion of toothbrushes by Molar Brush.
ton, 100 fewer tons will be sold, (a) Find the demand equation
for the alloy, (b) Find the revenue R{x) in dollars for the alloy at

production level .v. (c) Find the marginal revenue R'ix). Eln Problems 21 and 22. use the given information to conduct a
(d) Approximately how much additional revenue would be gen- marginal analysis.
erated by increasing the production level from 500 to 501 tons

during a production run? (e) For what production level will the 21 Rolar Bicycle Company is planning to bring out a new, light-

revenue be maximum? weight, and relatively inexpensive 10-speed bike. Management


SECTION 3.9 APPLICATIONS TO ECONOMICS AND BUSINESS 235

estimates a daily fixed cost of SI 700 and a variable cost of defined by

23.r 69a- dp
+ 159-\ dollars per day q
10.000 100
is called the elasticity of demand with respect to price.
for a daily production run of a bikes. Market research shows that (a) Explain why under normal economic conditions you might
at SI 50 per bike. 75 bikes will be purchased per day. but each expect that £> 0. (b) Assuming that the demand is met by the
SIO increase in price per bike will result in 15 fewer bikes per producer, so that q = x = the level of production, show that the
day being purchased. Because of limited production facilities, rate of change of revenue R with respect to price p is given by
no more than 125 bikes per day can be manufactured.
dR
-— = x{\ - E)
22 Decibel Records is planning to produce an album featuring the dp
trumpet virtuoso Adrian Scott. The recording will be made by (c) Economists say that the demand is elastic if £> 1. If the

using a state-of-the-art digital process that is considerably more demand is elastic, explain why a slight increase in price p will

e.xpensive than ordinary recording methods. Management esti- result in a loss of revenue.

mates a daily fixed cost of $800 and a variable cost of


27 A linear cost function has the form C(a) = Cn + Ax for

(6.5 X 10"^).r - (3.9 x 10 'U" -i- 9.8.V dollars per day < A < M, where Co & and A > 0. Here. Co is the fixed cost,
M is the maximum possible production level, and each addi-
for a daily production run of .v albums. Market research shows
tional unit of the commodity costs the same amount, A dollars,
that 500 albums per day will be sold to distributors at SIO per
to produce. Such a linear cost function applies to situations in
album, but each SI increase in price per album will result in 50 which production efficiency is the same at all possible levels of
fewer albums being purchased. Because of limited production
production, so there is no advantage in mass production. For a
facilities for these high-quality records, no more than 1000 rec-
commodity with demand
a linear cost function, suppose the
ords per day can be produced.
equation has the form aq + p = (i with o > and )3 > (see
Problem 25). Assume that the demand is met by production, so
23 A cable television company plans to begin operations in a small that q = X. (a) Show that the profit function P has the form
town. It foresees that about 600 people will subscribe to the Fix) = -ax- + ip- A)x - Co for < .v < M,. where A/, is

service if the price per subscriber SIO per month; but for each
is the smaller of the two numbers M and p/a (or their common
SO. 10 increase in the price per month, 4 of the original 600 value if they are equal), (b) If < fi
— A ^ laM. show
people will decide not to subscribe. The fixed cost to the com- that maximum profit is obtained at the production level
pany is S2000 per month, and each subscription costs the com- .V = (;8 - A)/(la).
pany S3 per month for maintenance, (a) What price per month
per subscription will bring in the greatest revenue to the com- 28 In Problem 26. show that the elasticity of demand E is unaf-
pany? (b) What price will bring in the greatest profit to the com- fected by a change in the units in which the commodity is meas-
pany? ured (for instance, kilograms rather than pounds) and that it is

unaffected by a change in monetary units (for instance, cents


24 An automobile leasing agency rents cars tomembers of a teach- rather than dollars per unit of the commodity).
ers' credit union and discounts its members by
total bill to these

2 percent for each rented car in excess of 12. For how many cars 29 A quadratic cost function has the form C(.v) = Co + bx — cvr

rented to the members would the receipts to the agency be maxi- for < .V < M, where Co s and a t^ 0. Of course, is re- it

mum? quired that C(x) a for s X s M. (a) Explain why the condi-
tion a > corresponds to the situation in which production be-
25 For a certain commodity, suppose that at a price of p] dollars per comes more efficient as the production level increases.
unit there is a demand for q^ units, but for each i-dollar increase (b) Under normal economic conditions, the marginal C'(a) will
in the price per unit, r fewer units will be sold, (a) Show that the be positive for < .v < M. Show that this is so if and only if
demand equation for the commodity has the form aq + p = fi, b > Q and a < b/(lM). (c) Show that the average production
where a = kir and fi = aq\ + p^.Qi) Show that unless the price cost per unit of thecommodity at the maximum production level
p per unit of the commodity is less than jS dollars, there will be M is given by (Co/M) + b — aM. (d) If a > 0, what is the most

no demand for the commodity, (c) Explain why the demand efficient production level? (e) If < a < b/(2M). show that the
q units for the commodity must satisfy < ^ s jS/a. condition C{x) s for s .r s Af holds automatically.
(d) Assuming that the demand is met. so that q = x = the pro-

duction level, write a formula for the revenue R(x) at production 30 For the quadratic cost function of Problem 29, assume that

level X. (e) Assuming that q = x. find the production level that < a < b/(2M) and that the demand equation has the form

maximizes the revenue. aq + p = p with a > and /3 > (see Problem 25). Suppose
that the demand is met by production, so that q = x. (a) UP is
26 Suppose that q units of a certain commodity are demanded when the profit function, write a formula for P{x). (b) What produc-
the producer charges a price p per unit. The quantity E tion level will generate maximum profit?
236 CHAP! KR 3 APPLICATIONS OK THE DERIVATIVE

Review Problem Set, Chapter 3


In Problems 1 to 6, verify the hypotheses of Rollers theorem lor
each runction/on the indicated closed interval [a. h\, and then find

all numbers f in the open interval (a, b) for which /'(c) = 0.

1 fix) = .x- + 6.V - 7, [a. h] = \-7. 1|

2 fix) = x^-x, [a, b] = 10. 1

3 f{x) = 4jc' - 21x^ + 25, [a. /;] = - | 1 . 5]

4 fix) = It' - 27.t- + 25.V. [a. b] = |(). 1

77 377
5/(.v) = cos.v. [a. b\ =

6 fix) = sin-.v, [a. b] = \0. Itt]

7 Which of the following functions, for one reason or another,


fail to satisfy the hypotheses of Rolle's theorem for the closed
interval [a, b]'! In each case where a failure occurs, indicate the
hypothesis that fails.

(a) /(jc) = Jt^ - 25, [a.b] = \-5, 5\

(h)j\x)= I
- W, [u.b] ;-i, "

(c)/(.v)= I -(x-2)-^\ \a.b] = \i. 31

f4 - X- if .V <
(d) fix) = [«. I>\
2v if .V > 1

8 Use Rolle's theorem to prove that if /'U) > for a < x < b,
then there is at most one x for which a < x < b and /(at) = 0.

9 Which of the functions whose graphs are shown in Figure I fail

to satisfy the hypotheses of the mean-value theorem on the


interval [a, h\'! In each case where a failure occurs, indicate the
hypothesis that fails.

Figure 1
REVIEW PROBLEM SET 237

10 Find all values of c such that - 1 < c < 1 and the tangent line

to the graph of /(.v) = .Iv* + Iv' - a" + .v - I at (c. /(<;•)) is

parallel to the secant between the points (-1, /'(-I)) and


(1,/(1)).

In Problems 11 to 16, verify the hypotheses of the mean-value theo-


rem for each function/on the indicated closed interval [a. b]. Then
find all numbers c in the open interval Ui. b) for which
f(b)-f{a)=f'(c)(b- a).

11 /U) = V^. [a, b] = [l. 4]

.r-4
12 fix) -. [a. b] = [0. 4]
.v + 4

13 f(x) = .v' - Iv- + 3.V - 2. [a. b] = [0, 2] (c)

f 3 - A-
if A < 1

14 fix) . [a, b] = [0, 2]


— if A > 1
L A

15 /(a) =A -I- sin- A. [a. b] = [0. 27rl

= X + Vl -cos = - ^.
16 fix) A. [a, /)]
^
El In Problems 17 and 18, use a calculator and Newton" s method to

find an approximation to a value of c in the open interval (a. b) for

which /(fc) -/(a) =f'(c){b -a). 22 Suppose that/and g are increasing functions on the interval /.
(a) Is 3/ increasing on /? Why? (b) Is -3/ increasing on /?

Why? (c) Is/ -I- g increasing on /? Why? (d) Is/*g increasing


17 /(a) = cos A, [a. b] on /? Why?

18 /(A) = A--* -I- A-' + 2a- -A + 3, [a. b] -1. 1] In Problems 23 to 30, indicate the intervals on which each function
is increasing or decreasina.

19 Use RoUe's theorem to prove that if/" exists on the open inter-
23 /(A) = A-' + 3a- - 2 24 g(x) = a' -t- 6a- -t- 9a -H 3
val (a, b) and if/" maintains a constant algebraic sign on this

interval, then /has at most one critical number in the interval

(a. b). 25 gix) = ^'a(a - 4)- 26 h(x) = a' + —


20 Use the mean- value theorem to prove that Vo < {a + I )/2 for
'a -1-1 if A <
< a < 1 . [Hint: Let /(a) = Va and apply the mean-value 27 fix) =
if A >
theorem on the interval [a. 1], Use the fact that if < c < 1.

then 0< V^< 1.] 1/a if A <


28 gix) = \ (A- 1)= if <A< 2
21 For each graph in Figure 2, indicate the intervals on which the
1/A-' if A >2
function is increasing or decreasing?

29 Fix) = cos- A, —277 SAS 277

Figure 2
30 G(A) = —- sin A, < A s 477

In Problems 31 to 52, find all the critical numbers of the function.

Then use either the first- or the second-derivative test to find all
numbers at which the function has a relative maximum or minimum.

31 fix) X- + 1
238 CHAPTER 3 APPLICATIONS OF THE DERIVATIVE

32 s(-v) = 2r' + 3.V- - 12;t


- 2 If too many people are put on the work crew, they get in each
other's way and the efficiency of the crew decreases, (a) For
33 /i(A) = Ir' - 9.r^ + lit + 1 34 Fix) = J.r' + i.v" - 2x
what value of v does the point of diminishing returns occur? (b)

35 G(.v) = i.v' + .V- - av + 1 36 H(.x) = .V* - 6.v- + 8.r For what value of v will the ma.ximum number of trucks per day
be unloaded'.' (c) Sketch the graph of y as a function of x.
37 fix) = (.t- - 9)- 38 g(x) = (X - 3)=(.v + 1)=
(Although v can assume only integer values, you may treat it as

39 h{x) = t- - r- 40 F(x) = x(x +1)"' a continuous variable for this problem.)

= + '
= - 16)"' 68 Suppose that / and g are continuous functions, both of which
41 G(.v) .V- Iv 42 H(x) (x-
have relative minima at .vi Given that/Ui ) > and g(Xi ) > 0, .

16 show that the product function /g has a relative minimum at x^


43 pix) 44 q(x) = .rV.t - 1

X- + 4
69 Let a, b, c. and d be constant numbers such that ad ¥ he and
45 = Vlv- + 9 46 Fix) = + 2)""- Show defined by the
r(.v) 10(.r c ¥= 0. thai the graph of the function /
equation /(.v) = iux + b)/icx + d) has no points of inflection.
47 Q(x) = X + sin x, -27r < .v < Itt

70 Suppose that the function /is defined by the equation /(.r) =


48 R(x) = X + cos 2r, < .v s 27r
ax^ + bx' + ex + d, where a. b, c. and d are constants with
49 f(x) = COS" .t + 2 cos .V, -2it s .v < 2 a > 0. Show that the graph of/ has exactly one point of intlec-
tion, the point
50 ^(.v) = sin Ix + 1 cos .r, £ .r s 2Tr

[ Iv + 1 if .t > 1
=
51 lux)
4 - A- if .Y < 1
(-i-4-i))
fV.v+ 1 if.r<0 Also show that/' is increasing when x > -fo/(3o).
52 f (.V) =
U + Vx if A^ >
In Problems 71 to 78. find the absolute maximum and minimum
values of the function on the given interval and indicate where they
In Problems 53 to 66, indicate the intervals on which the graph of occur. O Sketch the graph of the function.
each function is concave upward or downward. Also find all points
of inflection of each graph. 71 fix) = x' - 6.V- + 9.Y + I. 10. 4]

= - = - 72 gix) =.v' - 2v + 1, [-1, 1]


53 fix) .v' 8.V 54 gix) x^ 6x- + 9x + 1

73 /,(.v) = U - 1)'. (-1, 2]


55 hix) = -2x^ + 4.V- + 5 56 Fix) = x'ix- - 6)
74 Fix) = xix- + 2)"''-, [0. »)
57 Gix) = 2v' + 4.t- + Iv + 1

.V + 1
58 Hix) = x^ - 8.v^ + 64.V + 8 75 Gix) = -, [-3, 3]
X - 1

59 pix) = Iv-* + 4.r' - 24.r^ + .r - 3


76 H(x) = (X- + x)''\ 1-2. 3]
Ix
60 qix) = 77 fix) = sin' X + cos x, [-v. it]
U+ 3)-

= —3x x+ 1 78 gix) = V3 sec .r - tan x. 0.


y
61 rix) 62 six)
9 X- + 1

63 Pix) = 2 cos Iv, <As 27r In Problems 79 to 88. find the maximum and minimum values (if

64 Qix) = 8 sin x + < < they exist) of the function and indicate where they occur.
sin It, .v 27t

65 Rix) = X + tan .v 79 fix) = 1 + .V - .v^ 80 gix) = -v-' + .V - 1

= 1 + sin 2r if -irSxKO 81 /i(.v) = .V* - 2v' + 1 82 Fix) = X* + 4.V - 3


66 fix)
if S j: s TT
+ •V
= -^
1

83 Gix) 84 Hix)
.T- +
67 As a result of a time-and-motion study, management at a fac-
1

tory determines that a crew of .r persons can unload y trucks per


day, where 85 pix) = .V + —
Va
87 rix) =1-2 sin- .v
REVIEW PROBLEM SET 239

In Problems 89 to 102, evaluate the limit

3.V- + -v - 3
89 iim 90 Iim
4x- + 71 5;c + 3

5f 3/-= + 7r'
91

liin
- r + 1
92 Iim

h- - 3h
93 Iim 94 Iim
VS/i"* + 7/i- + 3

95 Iim (4v- - ly)

97 Iim
240 CHAPTKR 3 APPLICATIONS OK THK l)KRl\ ATIVE

134 A long rectangular sheet of tin is 8 centimeters wide. Find the experimental electric car. The motor has a resistance of
depth of the V-shaped trough of maxinium cross-sectional area R ohms, while each cell has an internal resistance of r ohms
that can be made by bending the plate along its central longitu- and an electromotive force of E volts. In the arrangement, x
dinal axis. cells are to be connected in series so that the battery will have a
net electromotive force of .v£ volts and an internal resistance of
135 Find the dimensions of the rectangle of greatest area that can be
x'r/n ohms. The current delivered to the motor is given by
inscribed in (a) a circle of radius 5 meters, (b) a semicircle of
radius 5 meters, (c) an isosceles triangle of base 10 meters and xE
/
altitude 10 meters, and (d) an isosceles trapezoid with bases of R + (x~r/n)
10 and 6 meters and an altitude of 8 meters.
Solve for the value of .v that will maximize the current /. (Al-

136 A long sheet of paper is 8 inches wide. One comer of the paper though X represents a whole number here, you may treat it as a

is folded over (Figure 4). Find the value of v that gives the right continuous variable.)
triangle ABC the least possible area.
142 If air resistance is neglected, it can be shown that the stream of

water projected from a fire hose satisfies the equation


Figure 4

v = ;n.x -^(1 + »r)(—


where m is the slope of the nozzle, i' is the velocity of the
stream at the nozzle, y is the height of the stream x units from
the nozzle, and g is the acceleration of gravity (Figure 5). As-
sume that V and g are positive constants, (a) For a fixed value of
m. find the value of .v for which the height y of the stream is

maximum, (b) For a tlxed value of m, find the distance (/ from


the nozzle at which the stream hits the ground, (c) Find the
value of m for which the stream hits the ground at the greatest
distance from the nozzle, (d) Find the value of m for which the
137 The range of a projectile is given by the formula R = water reaches the greatest height on a vertical wall x units from
(IT, sin l(l)/g. where iq is the muzzle velocity, g is the gravita-
the nozzle.
tional acceleration, and is the angle of elevation. Find the
angle of elevation that gives the maximum range.
Figure 5 i

138 A cylindrical can without a top is to be formed from sheet metal


of uniform thickness and is to weigh 100 grams. What is the
relationship between its height h and its base radius r if the
volume V of the can is to be maximum?

139 A newly independent country designs a flag that consists of a


\^^
red rectangular region divided by a green stripe. The perimeter
of the flag is to be 8 meters, and the red part is to have an area
143 Ship A is anchored 3 kilometers directly from a point B off the
of 2 square meters. What are the dimensions of the green stripe
shore of a lake (Figure 6). Opposite a point D. 5 kilometers
of largest possible area?
farther along the shore, another ship £ is anchored 9 kilometers
140 In the theory of probability, it is shown that the function L directly from point D. A boat is to take some passengers from
defined by ship A to a point C on shore and proceed to ship E. Find the
shortest course of the boat.

Up) /(I -p)" for < /? s 1

k\(n - ky.
Figure 6 U 5

has a value equal to the probability of exactly k successes in n


independent trials when the probability of success in each trial
is p. Here, of course, n and k are integers and Q s k s n. An
important problem in statistics is to p on estimate the value of
the basis of experimentally determined values of n and k. The
method of maximum likelihood uses, for such an estimate.
the value of maximizes Up). Find
p that this value of p in

teims of the constants n and k.

141 Suppose that n identical nickel-cadmium cells are to be ar-


ranged in series-parallel to furnish current to the motor of an
REVIEW PROBLEM SET 241

144 A weight of 1000 kilograms hanging 2 meters from a point A at For what angle 6 will the carr\'ing capacity of the gutter be
one end of a lever is to be lifted by an upward force F from the maximum?
other end B. Suppose that the lever is to weigh 10 kilograms
per meter (Figure 7). Find the length of the lever if the force at
B Figure 10
end is to be a minimum.

Figure 7

rp
10;tkg 1000 kg
150 The arithmetic
defined by
mean

.V, +
.v

X2
of the numbers x,

+ Xi + + X„
. X2, x^,

145 A sector ABC of central angle d is cut out from a circular sheet
of metal of radius a (Figure8), and the remainder is used to Show that X is the value of .v that minimizes the function
form a conical shape. Find the ma-\imum volume of such a
fix) = (.V - .V,)' + {x - .r,)- + (x - X})- + + (.V - .v„)-
cone.
El 151 A heavy wooden crate is being dragged along a horizontal sur-
Figure 8 face by a chain making an angle 6 with the horizontal. The
force F exerted on the chain is given by the equation

400
0.4 sin e -I- cos e

Find the angle 6 for which the force F is minimum.

152 A real estate company manages an apartment building contain-


ing 80 units. When $250 per month, all
the rent for each unit is

apartments are occupied. However, for each $10 increase in


146 A right circular cone whose generators make an angle 6 with its
monthly rent per unit, one of the units becomes vacant. Each
central axis is inscribed in a sphere of fixed radius a. For what
vacant unit costs the management $15 per month for taxes and
value of 6 will the lateral area of the cone be greatest?
upkeep, and each occupied unit costs the management $65 per
147 A projectile tired from the foot of a 30° slope will strike the month for taxes, service, upkeep, and water. What rent should

slope at a horizontal distance .v given by the equation be charged for a maximum profit?

153 A concession at a large city park rents bicycles on a daily basis.


sin 26 -(1 + cos All 100 of its bikes are rented at a rate of $10 per bike per day.
g ^ V3
but for each $1 increase in the daily rental rate per bike, 10
where vq is the muzzle velocity, 6 is the angle of elevation of
fewer bikes will be rented per day. What daily rental rate per
the gun. g is the acceleration of gravity, and air resistance is
bike will yield the maximum revenue to the concession?
neglected (Figure 9). For what value of 8 will the projectile
reach the farthest distance up the slope? 154 A manufacturer of sports trophies knows that the daily cost of
manufacturing x trophies is 60 -I- 6;r dollars and that the corre-
sponding sales revenue is 30.v - Iat' dollars. Find the daily
Figure 9
production level for trophies that will maximize the profit.

1155 The manufacturer of a certain microcomputer finds that in

order to sell x microco mputers each week, he must price them


at Vs. 000, 000 - Zx- dollars each. How many microcom-
puters per week will bring the largest total revenue?

] 148 A ladder 8 meters long leans against a building. At what angle


156 A garage owner finds that she can sell x tires per week at
should the ladder be inclined to give maximum headroom
p where p = ^(375 - 5.v) and the cost C in
dollars per tire,
under it at a point 2 meters from the building?
dollars of obtaining x tires per week to sell is expressed by

149 A gutter is to be made out of a long sheet of metal 30 centime- C= 500 -I- 15.V -I- {x-/5). Find the number of tires she must sell
ters wide by turning up strips 10 centimeters long on each side per week and the price she should charge per tire to maximize
so that they make equal angles 6 with the vertical (Figure 10). the profit.
242 CHAPTER 3 APPLICATIONS OF THE DERIVATIVE

157 The labor force > required by a tlrm to manufaclure x units of a ing directly under the pulley, grasps the rope 2 meters above

certain product is given by the equation y = iV.v. Find the the ground. If the boy, holding fast to the rope and keeping his
instantaneous rate at which the labor force should be increasing
if. at present, there is a demand for 40. ()()() units of the product, Figure 11 .pulley

but the demand is increasing at the constant rate of 10,000 units


per year.

158 The demand </ in thousands of boxes per week for a detergent is

given by q = ( 1000//? )
- .^0, where /) is the price in dollars per
box of detergent. The current price is p = $0.83 per box, but
inflation is increasing the price at the rate of $0.01 per month.
Find the current instantaneous rate of change of the demand.

159 The price of apples in a certain marketing area is given by the


equation /> = 2 + 60(30 + .r)' ' , where x is the supply in thou-
sands of bushels and p is the price in dollars per bushel. At
hand 2 meters above the ground, walks away at a constant rate
what rate will the price per bushel be changing if the current
of 1 meter per second, how fast is the weight rising 2 seconds
supply of 10,000 bushels is decreasing at the rate of 200 bush-
after he starts to walk?
els per day?
166 From a point 0.5 kilometer from a straight road, a searchlight is
El 160 A boat sails parallel to a straight beach at a constant speed of
kept trained upon a car which travels along the road at a con-
19.2 kilometers per hour, staying 6.4 kilometers offshore. How beam
stant speed of 80 kilometers per hour. At what rate is the
fast is it approaching a lighthouse on the shoreline at the instant
of light turning when the car is at the nearest point of the road?
it is exactly 8 kilometers from the lighthouse?
167 A balloon rises from the ground 1000 feet from an observer and
161 A 10-meter ladder leans against a vertical wall. The lower end
ascends vertically at the rate of 20 feet per .second. How fast is
is pulled horizontally away from the wall at a constant rate of 2
the angle of elevation changing at the observer's position at the
meters per second, and the top slides down the wall. Find the
instant when the balloon is 2000 feet from the ground?
rate at which the top moves (a) when the lower end is 3 meters

from the wall and (b) when the upper end is 8 meters above 168 A liquid nitrogen tank has the shape of a horizontal cylinder

ground level. with a radius of 2 meters and a length of 10 meters. The tank is

vented to allow the evaporating liquid nitrogen to escape. At


162 Water is flowing through a hole in the bottom of a hemispheri-
the instant when the tank is half full, the surface level of the
cal bowl of radius 10 centimeters. At the instant the water is 6
liquid nitrogen is dropping at the rate of 2 centimeters per hour.
centimeters deep, the rate of flow is 5 cubic centimeters per
Find the rate of evaporation, in cubic meters per hour, at this
minute. Find the rate at which the surface of the water is falling
instant.
at this instant.

E] 169 Conduct a marginal analysis using the following information:


163 A highway crosses a railroad track at right angles. A car travel- Polar Oil Company is looking into the possibility of setting up a
ing at the constant rate of 40 kilometers per hour goes through
plant to manufacture synthetic fuel (synfuel) from coal. It pro-
the intersection 2 minutes before the engine of a train traveling
jects that by using a new process, up to 14(X) barrels per day
at the constant rate of 36 kilometers per hour goes through the
could be produced at a fixed cost of $1500 per day and a vari-
intersection. At what rate are the car and engine separating 10
able cost of
minutes after the train goes through the intersection?
(3.5 X IQ-V' - (1.05 X 10"-).v- -I- 40.5.V dollars
164 The height of a right circular cylinder is increasing at the rate of
3 centimeters per minute, and the radius of the base is decreas- where -v is the number of barrels produced per day. Market
ing at the rate of 2 centimeters per minute. Find the rate at research indicates that at a price of $40 per barrel. 700 barrels

which the volume of the cylinder is changing when the height is per day can be sold; but for each $1 increase in the price per
10 centimeters and the radius of the base is 4 centimeters. barrel. 100 fewer barrels per day will be sold.

165 A boy is going to lift a weight W by means of a rope mounted 170 In Problem 169. calculate the elasticity of demand with respect
on a pulley (Figure II) that is 12 meters above the ground. At to price when the price is $40 per barrel. (See Problem 26,
the start the weight is resting on the ground, and the boy. stand- page 235.)
ANTIDIFFERENTIATION AND
DIFFERENTIAL EQUATIONS

In the previous chapters we have seen that differentiation has applications to prob-
lems from geometry and physics to business and economics. In this
in areas ranging
chapter we introduce antidifferentiation, which reverses the procedure of differenti-
ation and allows us to find functions that have a given function as their derivative.
We show how antidifferentiation permits us to solve simple differential equations,
and we relate antidifferentiation to a number of practical problems. However, be-
fore going into these matters, we introduce the idea of differentials, since differen-
tial notation is useful in connection with antidifferentiation.

4.1 Differentials and Linear Approximation


In Section 2.2 we introduced the Leibniz notation dy/dx for the derivative; however,
we were careful to point out that dy/dx should not be regarded as being a fraction
until the "numerator" dy and the "denominator" dx are given separate meanings.
Shortly, we provide such meanings for the so-called "differentials" dx and dy.
Leibniz himself regarded dx and dy as being "infinitesimals," that is, quantities
that, although they are nonzero, are smaller in magnitude than any finite quantity.

He imagined that in the limit. Ax and Ay somehow become "infinitesimal quan-


tities" dx and dy, respectively, so that the difference quotient Ay/ Ax becomes
the derivative dy/dx. Leibniz's point of view has persisted, and even today some
mathematicians and most engineers and scientists prefer to think of dx and dy as
infinitesimals.
Part of Leibniz's concept can be salvaged by regarding dy/dx as being an honest
ratio, if not of infinitesimals, then of differentials dy and dx and by rewriting the
equation dy/dx = f'{x) as dy = f'(x) dx. The last equation will serve as a definition
of the differential dy, provided that an appropriate meaning can be given to the
differential dx. This is accomplished as follows.

243
244 CHAPTER 4 ANXroiFFERENTIATION AND DIFFERENTIAL EQUATIONS

DEIINITION 1 Differentials

l.ci / be a lunction. and let v and y be variables so related that y = fix). Then the
differential dx is a quantity that can take on (or be assigned) any value in U. If
v is any number in the domain of/for which/'(.v) exists, then the differential dy
is defined by

Jy = f'(.x) d.x

EXAMPLE I If y = f(x) = 3.t- - 2jt + 1 , find dy.

SOLUTION Here/'(.v) = 6.t - 2, so dy = (6.v - 2) dx.

Notice the distinction between the differential dx of the independent variable x


and the differential dy of the dependent variable y. Whereas dx can be given any
value whatsoever, dy depends for its value on x and dx (not to mention/). For
instance, in the example above when .v = 1 and dx = 0.002, we have
dy = (6 - 2)(0.002) = 0.008

Because of Definition 1 , from now on we can regard the derivative dy/dx as an


actual fraction with numerator dy and denominator dx. and we can manipulate such
fractions according to the usual rules of algebra.* In particular, an equation involv-
ing derivatives expressed in Leibniz notation may be converted to an equation
involving differentials by "clearing fractions'" as usual. For instance, if

Table 1

Derivatives
SECTION 4.1 DIFFERENTIALS AND LINEAR APPROXIMATION 245

EXAMPLE 3 Let V = V3 - A ^ Find dy.

-
SOLUTION d\ = dO ,
x^)
= -5a-*
, dx
2V3-.r5 2V3^r?

EXAMPLE 4 Let z = tan ^. Find dz.


2

^z = sec-
-,

-A d[-) =/ .r \
sec-
^
-X —
dx
=y
1
sec-
,
-A dr

EXAMPLE 5 Let a and y be differentiable functions of a third variable /. and


suppose that a-*+ Ax-y + y^ = 2. Find the relationship between dx and dy.
SOLUTION "Taking the differential" on both sides of the given equation, we
obtain

d{x^ + 4a^>' + y^) = d(2) or ^(a') + Adix-y) + d(y^) =


Therefore,

3a- dx + 4[a- dy + y(lx dx)] + ?>y- dy =


or

3a- dx + ^x- dy + 8at dx + 3.v^ dy =


By collecting terms involving like differentials, we may rewrite the last equation in
the equivalent form

(3a- + 8at) dx + (4a- + 3y-) dy =

Beginning calculus students often carelessly confuse derivatives and differen-


tials. Derivatives and differentials are closely related but they are not the same. —
The differential of a variable « is always written with a small d as du; the derivative
J-
. ,
du du
of u with respect to a is a ratio of differentials. Of course, the derivative
dx dx
can also be written in operator notation with a capital D as D^u. Beware of equa-
tions with a differential on one side and a derivative on the other side — an equation
such as du = D^u simply makes no sense! A correct equation would be

—du— = D^u or du = (DjK) dx


dx
Suppose that u is a function of v and. in turn, v is a function of a. According to
the chain rule, we have

du du dv
dx dv dx

provided that the derivatives du/dv and dv/dx exist. We pointed out in Section 2.7
that the Leibniz notation makes the chain rule — a rather deep and important fact
look obvious. Now that du/dx. du/dv. and dv/dx are actual fractions, can't we
regard the chain rule as an algebraic triviality? The answer is still no. since the
dv in du/dv is the differential of v regarded as an independent variable
differential
(upon which u depends), while the differential dv in dv/dx is the differential of v
246 CHAFrKR » ANTIDIFFERENTIATION AND IMFKERENTIAI. EQUATIONS

regarded as a dependent variable (depending, in fact, on.v). Because oftiiis distinc-


tion between the dv in du/dv and the dv in dv/dx, we cannot conclude that

du _ du dv
dx dv d.x

on purely algebraic grounds. It is the chain rule that justifies the algebraic manipula-
tion rather than vice versa.
In casual calculation with differentials, one does not bother to distinguish be-

tween differentials of dependent and independent variables. Miraculously, such


carelessness rarely causes any difficulty. The "miracle." of course, is really the

chain rule!
A geometric interpretation of differentials can be obtained as follows: Let v =
f(x), and assume that/is differentiable at.v, . If we calculate the differential dy when
X = x\, we have

dy =/'Ui) dx

Recall that dx can take on or be assigned any value in U. Thus, let

dx = A.V

denote an increase in the value of .v from .vi to .vi -I- A.y (Figure 1). Then

A_v =/(.v, + A.V) -/(A-,)

is the corresponding increase in the value of y from the value /(.v, ) to the value

/(xi -I- determined by moving up along the graph off. However, since/' (vi
A.v) as

is the slope of the tangent line to the graph of/ at (xi, /(ai)), it follows that

dy =/'(a-i) dx

gives the corresponding increase in the value of y as determined by moving up along


the tangent line.

Figure 1

Because the tangent line to the graph of/ is a good approximation to the graph
itself near the point of tangency, it shouldn't make very much difference whether
we move up along the graph of/or up along the tangent line, provided that we don't
move very far from the point of tangency. In other words, ^v should be a good
approximation to Ay provided that dx = Ax and Ajc is sufficiently small. Of course.
SECTION 4.1 DIFFERENTIALS AND LINEAR APPROXIMATION 247

Figure 1 shows a rather special situation in which A.v is positive, the graph of/
is
concave upward, and /is an increasing function. However, by sketching graphs
showing other possibilities,* you can quickly convince yourself that

Ay == c/v if d.\ = A.v and A.v is "smair

B EXAMPLE 6 Let v = f(.x) = 3.X- - 2.V + 4, and put .v, = 1 and dx = A.v = 0.02.

(a) Calculate A>' = f(xx + A.v) -/(.Vi) exactly.

(b) Find an approximation for Av, using dy = f'(x^) dx.


(c) Determine the error Ay - dy involved in the approximation Av = dy.

SOLUTION

(a) Ay =/(.v, + A.V) -/(.v,) =/(1.02) -/(I) = 5.0812 - 5 = 0.0812


(b) Since /'(.v) = 6.v - 2, follows that it

Ay « dy =f'(x^)dx = |6(1) - 2](0.02) = 0.08

(c) The error is given by Ay - dy = 0.0812 - 0.08 = 0.0012.

El EXAMPLE 7 Use differentials to estimate V35.


SOLUTION We know that V36 = 6, so \/35 will be a little smaller than 6. To
estimate the numerical value of V35, let

y = Vx .v, = 36 dx = A.v = -
Then

Ay = V.v, + A.v -V7,= V35 - V36 = VSS - 6

hence, \/35 = 6 + Ay
If we knew the exact numerical value of Ay, we could determine the exact numeri-
cal value of V35 by using the last equation. Now the idea is to use the fact that
Ay ~ dy. so that

V35 =- 6 + dy

Here

dy = dV-X = ^dx
iVx
= ^2V^
Thus, putting x = .v, = 36 and dx = A.v = - 1, we find that

2V36 12

It follows that

\'^ «6 + £/y = 6-T^ = 5.9167

(Incidentally, the correct value of V35 rounded off to four decimal places is
5.9161, so our estimate using differentials is fairly accurate.)

*See the proof of Theorem 1 on page 249 for a more conclusive argument.
248 CHAPIF.R 4 ANTIDIITERENTIATION AND DIFFERENTIAL EQUATIONS

Figure 2 (B EXAMPLE 8 Use ditTerentials to find the approximate volume of a right circular
cylindrical shell (Figure 2) 6 centimeters high whose inner radius is 2 centimeters

and whose thickness is 0. 1 centimeter.

SOLUTION The volume of a right circular cylinder is its height times the area of
its base. If V denotes the volume of a (solid) cylinder of height 6 centimeters and
radius r centimeters, then V = bnr- cubic centimeters. Figure 2 shows a cylinder of
height 6 centimeters and radius r = 2 centimeters inside a larger concentric cylinder
of height 6 centimeters and radius r + Ar = 2 + 0. =2.1 centimeters. The differ- 1

ence AV in the volumes of the two cylinders is the required volume of the cylindrical
shell. We put t/r = Ar = 0.1 centimeter and use the approximation

AV^dV = d(6TTr-) = 127rr dr

Hence, for r = 2 centimeters and dr = 0.1 centimeter, we have

AV= 127n2)(0.1) = 2.477- = 7.5 cubic centimeters

] EXAMPLE 9 The radius of a spherical steel ball bearing measured to be 1.5


is

centimeters, and it is known that the error involved in this measurement does not
exceed 0.1 centimeter. The volume V of the ball bearing is calculated from its
measured radius by using the standard formula V = ^vr. Estimate the possible
error in the calculated volume.

SOLUTION The true value of the radius is 1.5 + Ar, where Ar is the measure-
ment error. We are given that |Ar|<0.1. The true value of the volume is

s77( 1 .5 + Ar)-\ while the value of the volume calculated from the measurement of
the radius is i|77-(1.5)-\ The difference AV = |7r(1.5 -I- Ar)'' - ^7r(1.5)' represents
the error in the calculated volume. We put dr = Ar and estimate AV by dV as

follows:

AV = JV = ^(^Trr-') = 4nr- dr = 47r(l .5)' Ar = 9it Ar

Therefore,

|AV| = 1973- Ar| = 97r|Ar| < 97r(0.1) = 0.97r

so the possible error is bounded in absolute value by about 0.9i7 = 2.8 cubic centi-
meters

The Linear-Approximation Theorem


The approximation Ay = dy is called a linear approximation because it is based on
the geometric idea that the tangent line to a curve is a good approximation to the
curve near the point of tangency (Figure 1 ). In scientific work, an approximation of
any kind is of limited use unless there is some way to specify bounds on the error

that might be involved. The error E in the linear approximation Ay == dy is given by

E = true value — approximate value


= Ay - dy
= \fi-x\ + A.v)-/(.V|)] - L/'(V|)<ivl
= /(.v, + Ax) -/(.v,) -/'(.v,) A.V
SECTION 4.1 DIFFERENTIALS AND LINEAR APPROXIMATION
249

If we solve the last equation for/(.v,


+ Ax), we can rewrite the linear approximation
Av == dy in the equivalent form

/(.vi + Ax) =/U,) +/'U,) Aa- + E

In other words,

f(x, + Ax) = /(A, + /'(.v, Ax ) ) with error E


Here we do not discuss the question of putting bounds
on the numerical value of
the error £; such matters are dealt with
in Section 10.5. Instead, we prove the
following theorem, which shows formally that
for "small" values of Ax, the linear
approximation is indeed very accurate.

THEOREM I
Linear-Approximation Theorem

If the function /is differentiable at the number .v,, then for all values of Av for
which xi + Ax is in the domain of/,

/(.v, + Ax) =/(x,) +/'(x,) Ax + (Ax)e


where the quantity e satisfies the condition that

lim e =

PROOF For each value of Ax such that x, + Ax is in the domain of/ we define

/(xi + Ax) -/(x,)


J^
/'(x, ) if Ax ^
-0 ifAx =
Then for A.v ^ we have

/(x, + Ax) =/(x,) +/'(x,) Ax + (Ax)e


If Ax = 0, the last equation still holds, since both sides reduce to/(x,
) Therefore
the equation holds for all values of Ax for which x, + Ax is in the domain off. We
'
also have

Ax
=
/'Ui)-/'U,) =
and the proof is complete.

In Theorem 1 , note that

E= (Ax)e
ZJ
This formulation of the error E as a
product emphasizes the fact that E approaches
zero very rapidly as Ax approaches
approaches zero.
zero, since both factors approach zero
as Ax ^
250 CHAP1KR 4 ANTIDIFFERENTIATION AND DIFFERENTIAL EQLATIONS

Problem Set 4.1

In Problems 1 to 20, find </v in terms of ,v and dx.

I V = 3.V- 2 \= ] - X

= -' -
3 y t' 4.r + 5

7.V - 2
5 v =
3.V + 1

7 y = (3.V- + 2)'

9 y = /'

II v =

13 y =

15 v =
X- + 1

17 y = 3 sin Iv

19 y = 2 tan- .v

21 Find a formula for (/(cot u).

22 Find a formula for rf(see u).

23 Find a formula for^^lcse u).


SECTION 4.2 A\TIDERIVATI\'ES 251

E163 Use differentials to find approximately how many cubic centi- function of .v. (b) Find dP in terms of .v and dx. (c) When the
meters of chromium plate must be applied to coat the lateral production level changes from .r = 350 to .v = 355. what is the
surface of a cylindrical rod of radius 2.34 centimeters to a thick- approximate change in P'l

ness of 0.01 centimeter if the rod is 30 centimeters long.


67 The law for adiabatic expansion of a certain gas is fV'^ = C,
[364 The period o f osc illation of a pendulum of length L units is given where V is the volume of the gas, P is the pressure of
by r= liT\Llg. where g is the acceleration of gravity in units the gas. and C is a constant. Derive the following equation:
of length per second squared and T is in seconds. Use differen- {dPIP) -I- (1.7 dViV) = 0.
tials to find the approximate percent that the pendulum in a

grandfather clock should be lengthened if the clock gains 3 min-


68 If/ is a differentiable function at the number .vi, show that there
utes in 24 hours. is a linear function g such that

65 The attractive force between unlike electrically charged particles - g(x)


fix)
is expressed by f= kjx', where x is the distance between the lim =
particles and k is a certain constant. If x is increased by 2 per-
(Him: Let A-v = .r - xi in Theorem 1
cent, use differentials to find the approximate percent decrease
.

in F
E69 An angle is measured to be 31 .4° with an error that is known not
66 The total cost in dollars of producing .v toys is to exceed 0.05°. The sine of the angle is then found by using a
calculator to be sin 31.4° = 0.521009632. Use differentials to
3.r-
C= -I- 1 l.v + 75 estimate the maximum possible error in the calculated value of
15,000 100
the sine of the angle. (Neglect any error caused by inaccuracies
and each toy is sold at SIO. (a) Find the total profit P as a inherent in the calculator.)

4.2 Antiderivatives

In applied mathematics it often happens that we know the derivative of a function


and would like to find the function itself. we might know the velocity
For instance,
ds/dt of a particle and wish to find its equation of motion 5 = fit), or we might wish
Figure I to find the revenue function for a certain product when we know the marginal
revenue. The solutions of such problems require us to '"undo"" the operation of
differentiation: that is. we are required to a/iffdifferentiate.
Suppose g and /are functions and that the derivative of g is/. Then
that we say
that ^1,' is an cmtiderivative of/. For instance, since

D,(.v-) = Iv

we say that g(x) = x- is an antiderivative of/(.v) = 2v. If C is any constant, then we


also have

DJx- + C) = lx

and it follows that .v" + C is another antiderivative of Ix. This is geometrically clear
(Figure 1 ). since the graph of >• = .v" + C is obtained by shifting the graph of y = .v"

vertically by C units, and this does not change the slope of the tangent line for a
given value of the abscissa .v.

In general, we define antiderivatives as follows.

DEFINITION 1 Antiderivative

A function g is called an antiderivative of a function / on a set of numbers / if

g'(x) = fix) holds for every value of v in /. The procedure of finding antideriva-
tives is called antidifferentiation.
252 CHAPTER 4 ANTIDIFFERENTIATION AND DIFFERENTIAL EQUATIONS

If we say that g is an antiderivative of/ without exphcitly mentioning the set /


in Definition 1. then it is to be understood that / is the entire domain of/, so that
g'{.x) =/(.v) holds for ail values of .v in the domain of/.

x + 1

EXAMPLE I Show that g{x) is an antiderivative of /(.v)


A- - 1 (.X- 1)-

SOLUTION By the quotient rule.

U- l)-(-V+ 1)
gM (.V- 1)- (.V - 1
)-

Since the derivative of a constant function is the zero function, it follows that any
constant function is an antiderivative of the zero function The following important .

theorem shows, conversely, that the constant functions are the only antiderivatives

of the zero function.

THEOREM 1 Antidifferentiation of the Zero Function

Let 5 be a function such that ^''(.v) = holds for all values of .v in some open
interval /. Then g has a constant value on /.

It is enough to prove that the value of g at a number a in / is the same as the value of
g at every other number b in /. By the mean-value theorem (Section 3.1), there
exists a number c between a and b such that

gib) - g(a) = g'(c)(h - a) = 0(b-a) = Q

Thus. g(a) = g(b). and the theorem is proved.

The following theorem, which is a direct consequence of Theorem 1 , tells us how


to find all the antiderivatives of a function on an open interval, provided that we
know one such antiderivative.

THEOREM 2 Antidifferentiation on an Open Interval

Suppose that g is an antiderivative of the function/on the open interval /. Then a


function h with domain / is an antiderivative of/ on / if and only '\i h = g + C
for some constant C.

If /! = ^ + C, then h' = g' =/. so that /; is an antiderivative of/ on /. Conversely,


suppose that h is an antiderivative of /on /. Then, the function h — g satisfies

{h - g)' = h' — g' =/-/= on the open interval /. It follows from Theorem 1

that there exists a constant C such that h -g = C; that \s, h = g + C.

EXAMPLE 2 Given that the function ^(.v) = iv" is an antiderivative of the func-
=
tion /(jc) X, find all antiderivatives of/.

SOLUTION Here, the interval / is IR. By Theorem 2. the antiderivatives of/ are
all functions /; of the form /i(.v) = |jr" -I- C, where C is a constant.
SECTION 4.2 ANTIDERTV ATHTS 253

Notation for Antiderivatives


Antiderivatives are traditionally written by using a special symbolism which has
some of the same advantages as the Leibniz notation for derivatives and which, in
fact, was used by Leibniz himself. This symbolism can be understood by thinking
of the differential dy as an "infinitesimal bit of y"" and imagining that y is the
"sum" of all these infinitesimals. Leibniz used a stylized letter 5, written /, for
such "summations," so that

y = dy
J
would symbolize the idea that "y is the sum of all its own differentials." Johann
Bernoulli, a contemporary of Leibniz, suggested that the process of assembling
infinitesimals so as to get a whole, or complete, quantity should properly be called
integration rather than summation. People accepted Bernoulli's suggestion; hence,
the symbol / is referred to as the integral sign.
Now suppose that g is an antiderivative of/, so that

8'=f
If we let

y = g(x)

then

dy = g'{x) dx =f(x) dx

and we have

y = dy = jfix) dx
J
that is.

g{x) = fix) dx
J
For this reason, the notation

fix) dx

is used to denote an antiderivative of the function /. If C is any constant,


then g(x) + C is also an antiderivative of/; therefore, we make the following
definition.

DEFINITION 2 Integral Notation for Antiderivatives

The notation J fix) dx = g{x} + C. where C denotes an arbitrary constant,


means that the function g is an antiderivative of the function /, so that
g'(x) = fix) holds for all values of v in the domain of /'.

In Definition 2, the constant C is called the constant of integration, and


the function / [or the expression fix)] is called the integrand* of the ex-
pression J fix) dx. We often say that the integrand / [or f(x)] is under the
integral sign.

*Sometimes the entire expression /(x) dx, including the differential dx, is referred to as the
integrand.
254 CHAKIKR 4 ANTIDIFFERENTIATION AND DIKFERENTIAL EQUATIONS

In order to verify a statement of the form

J/(.v) dx= i;{.x) + C

it is only necessary to check that D^gix) =/(.v).

In Examples 3 and 4. verify the given equation.

EXAMPLE 3 jr dv = W+C
SOLUTION Dv(ir') = .r

EXAMPLE 4 dx = .V + C

SOLUTION dx = 1 <iv = v + C, since D>(.v) = 1

The procedure of evaluating

/(.v) dx
J

that is, the procedure of finding an antiderivative g of/ and writing

dx = g(x) + C
I /(.v)

is called indeflnite integration. The adjective "indefinite" is presumably used


because the constant C can have any value and therefore is not definitely determined
by the function/. Because of the arbitrary nature of the constant C.

J/(.v) dx

which is called the indefinite integral of the function/, does not represent a particu-
lar quantity or function: hence, care should be taken in manipulating this expres-

sion. Since the problem of evaluating an indefinite integral //(.v) dx is essentially


the same as that of finding an antiderivative ^i; of the integrand /, we use the terms
"indefinite integration" and "antidijferentiation" interchangeably from now on.
The fact that differentiation and antidifferentiation "undo"" each other is expressed
by the following formulas:

D, /(.vl (/.v =/(.v) and \ f\\) dx = t\x) ~ C

Basic Algebraic Rules for Antidifferentiation

Since antidifferentiation (or indefinite integration) "undoes" differentiation, each


rule or formula for differentiation will yield a corresponding rule or fomiula for
antidifferentiation when "read backward." Some of these rules are as follows:
SECTION 4.2 ANTIDERIVATIMiS 255

Lety, .i;../i.y2 jm tie liinctions that have antiderivativcs; let n, fli, a2.
a,„ be constants; and let n be a rational number with « # - 1 . Then:

A-" </.v = -^ + C
/I + 1

2 «/(.v) dLr = « j/(.v) ^.v


J

3 l/(.v) + ^(.v)] cLx = /(A) ^/x -f ^(.v) dx


J J J

4 [a,/, (A) + a./,(A) + • • • + «,j;„(A)] dx =


J

(/,
I
/|(A) <:/a + a. /:( V) d\ + + a,„ \
JJ.x) dx
J

Each of these four basic algebraic rules may be verified simply


by differentiating the
quantity on the right side of the equation and showing
that the result is the expres-
sion under the integral sign on the left side
(Problem 41),

EXAMPLE 5 Evaluate I (5a- - 3a + 1) dx.

SOLUTION

(S.v^ - 3.V + 7) dx = 5 A- dx + (-3) I A dx +


J I 1 j ^A (Rule 4)

= ir' - ix- + 7a + 5C| - 3C2 + 7C3


= ir' - f.r^ -I- 7a + C
where C= 5C, - 3C2 + 7C,.

In practice, when the basic rules are used to


antidifferentiate (that is, to evaluate
mdefinite integrals), the individual constants of integration
that arise are immedi-
ately combined into one constant. Thus, the
solution given above would ordinarily
be condensed as follows;

(5.v^ - 3a + 7) dLv = 5 x" dx -3 j xdx + 7 dx = jr^ - + 7x +C


J J j Ix^

Just as the derivative of a product is not the


product of the derivatives, neither is
the antiderivalive of a product the product of
the antiderivativcs. In fact, there is no
simple general formula for an antiderivative of a
product, nor is there such a for-
mula for an antiderivative of a quotient. However,
by aclually performing the indi-
cated multiplication or division, you may be able to
rewrite a product or quotient in
such a form that it can be antidifferentiated by using
the basic rules given above.
This is illustrated in the following examples.

In Exatnples 6 to 9, evaluate the antiderivative.

EXAMPLE 6 (A + 2)(3a - \) dx
J
256 CHAPTKR 4 ANTIDIFFERENTIATION AND DIFFERENTIAL EQl!ATIONS

(.V + 2)(3.v - \) dx = (3.V- + 5.r - 2) dx


I

= 3 A- d.x + 5 I .X dx- 2 \ d.x


J

-(4 -Iv + C
(4) +5(
3(y) — )

= .r* + |a~ -Ix + C

EXAMPLE 7
/^
,

r
..

.V* +
_..
;

3.r
+

+
5

5
dx

r / -v^ A- 5 \

J
; dx=
J
-r + 3-^ + -^ dLv
A- ^ A- A- A" '

= (A- + 3 + 5a'-) d.x

= \ JT dx + ?> \ d.x + 5 \ .x'~

^+ 3
3a + 5^
-1
+ C

—+ x"

3
3a
5

X
+ C

EX AMPLE 8
J
(v^++
(vVv 1)- ^V

SOLUTION (3'Vy + 1)- dy = {/^' + 1)= dy = (/'" + 2/''-' + I) dy


J J J

= y^/' dy + 2 y^' ^ dy + <v


I I J
yll/3 7^.7/3
^ + ^ + V + C
11/3 7/3
3,,n/3 6^,7/3
-^ + -^-^ + V + C

EXAMPLE 9 (
— 4?^ ] du

du

= 2 1/ ' £/«- 3 « " du


J J

= 2 3^— + C = -M^- - 4m-'/'' + C


-2 " 3/4

Antidifferentiation of Trigonometric Functions

The formulas obtained in Section 2.5 for the derivatives of the trigonometric func-
tions can be reversed to obtain the following formulas for antiderivatives;
SECTION 4.2 ANTIDERIVATIVES 257

1
258 CHAPTER 4 ANTIDIFFERENTIATION AND DIFFERENTIAL EQUATIONS

(li^ - At- - 5f + 6) dt 8 (2 + 3v^ - 8.v^) dy (2 sec .r + csc .r)(2 sec .r - csc .r) dx
J

(3i/ - 2m' - m - \) du 10 (SJ' - ?r + h - h) dz tan- v dx (Hint: 1 + tan" .v = sec^ x)


I

(Ax~- - 3.r
•*
+ I) tiv 12/(4 + 5.-3)^. Cr^' - 4 sin .V + 5 cos x) dx

2/" + 5r^ + 7
- 4 cot" u du
dt 14 (2r' l)(.r + 5) dx
J

{2\G + 3 sec" .r - 4 csc x cot x) dx


(r + 3r + j) dt 16 dx

cos"
.
— dx-x \
Hint: cos"
,
—=—
-x 1

(1 + cos .c)
ax" - i)(A- + 5) dx dx

41 Verify the basic algebraic rules for antidifferentiation on page


(4r + 3)- dt dy 255.

42 Explain why an antiderivative of a polynomial function is again


\Av-(w - 2) dw \/m(3m + W) du a polynomial function.
I

43 Verify the rules for antidifferentiation of trigonometric functions


25.r' - 1

dx on page 257.
VTx
44 Let /, g, and h be defined by the equations fix) = -2/.r,
g(x) = 1/jr, and
(Vlr + IxVx + —^) dx
- 2r
if .t <
{\Cx- 1) r' + 2r - 3 /i(.v) =
dx 26
if .V >

(2 cos M + 4 sin u) du 28 I (3 sec" m - 4 csc" «) du respectively. Show that both g and h are antiderivatives of/but
that there is no constant such that h = g + C. Does this contra-

dict Theorem 2? Explain.


sec A (3 tan x - 2 sec .v) d^t

45 (a) Give an example to show that f f(x) dx ^f{x) / dx.


(b) Give an example to show that / f(x)g{x) dx ¥"
(2 sin f - 3 cos f + \\) dt
i//(.v)dT][;g(.t)d:x].

csc y (2 cot y - 7 csc y) dy



32
r ^^
5
(c) Give an example to show that '—
dx ^
J/(-v) dx
J cos JC J six
g(x) S g(x)dx

f , , , • r 5 cos u
— + 46 Prove that on an open intenal. two antiderivatives of the same
33 (5 csc^ t + 1 sec= f + 4)
.

<// 34 ^M
J J sin" 1/ function must differ by a constant.

4.3 The Method of Substitution,


or Change of Variable

In order to evaluate / .v(.v" + 5)'"" </.v using only the basic algebraic rules, it would
be necessary to expand (.v" -I-
5)'"" by the binomial theorem, multiply through by .v.

and then antidifferentiate the resulting expression term by term. The required calcu-
lation could be quite tedious, to say the least. Fortunately, there is a simpler way to
SECTION 4.3 THE METHOD OF SLBSTITLTION. OR CHANGE OF VARIABLE 259

proceed, namely, by "changing the variable"" from.v to a new variable u = .^r + 5.

Notice that if m = .r^ + 5. then we have du = Iv dx. or x dx = i dii. and so

j
4-r + 5)"» dj(= \ (.x^ + 5)'""j«: dx

= I
«"*' — du = —\ u'"^ du = — ^^^ + C
2 101
Substitution of « = .\~ + 5 into the last expression gives

J 202

The method illustrated here is called change of variable, or substitution.


There is more going on than meets the eye in the calculation above, since the
integral notation has compressed what could otherwise be a complicated argument
(using the chain rule) into a few simple calculations. In the same way that the
basic rules for antidifferentiation are obtained by reading the basic rules for differ-
entiating backward, the method of substitution (or change of variable) is just the
chain rule "read backward."" However, we defer the formal justification of this
method to the end of the section and concentrate here on seeing just how it works.

Procedure for Evaluating f fix) dx by Change of Variable (Substitution)

Step 1 Find a portion of the integrand /(.v) that is especially "prominent."" in


the sense that if it were replaced by a single new variable, say u. then the
integrand would be noticeably simpler. Set ii equal to this portion. The result-
ing equation will have the form

It = g{x)

Step 2 Using the equation it = g(x) obtained in step 1. find the differential

du = g'ix) dx

Step 3 Using the two equations u = g{x) and du = g'{x) dx obtained in steps
1 and 2, rewrite the entire integrand including dx. in terms of u and du only.

Step 4 Evaluate the resulting indefinite integral in terms of u.

Step 5 Using the equation u = g(x) of step 1 , rewrite the answer obtained in
step 4 in terms of the original variable x.

There is never a guarantee of success when the method of substitution is used

one can only try and see what happens. After the algebraic manipulations required
in step 3 are carried out, it may happen that the resulting integral (involving u) is
more complicated than the original integral (involving .v). However, if the proce-
dure fails for one choice of u. it may still work for another — just try again.

In Examples 1 to 5, use the procedure for change of variable {substitution) and the
basic rules to evaluate the antiderivative {indefinite integral).

EXAMPLE 1 V7.V + 2 dx

SOLUTION We carry out the steps in the procedure.

Step 1 Let"s try w = 7.r -I- 2.

Step 2 du = 1 dx
260 CHAPTER 4 ANTIDIFFERENTIATION AND DIFFERENTIAL EQUATIONS

Step 3 Since du = 1 dx, it follows that dx = ^ du; hence,

J
V'7.r + 2 dx= \ \^-l du= \ \Vu du

Step 4
J
V7.V + 2 dx =
j
y V^ du = — j u''- du = — -^ + C

Step 5 Since h = Ix + 2. we have

I V7.V + 2 civ = A(7^ + 2)^/2 + c

dx
EXAMPLE 2 I
— (x^ + 4)^
;

SOLUTION

Step 1 Let's try u = .r* + 4.

Step 2 t/M = 3.x^ dv


Step 3 Notice the .v" dx in the numerator of the integrand and the same expres-
sion multiplied by 3 in step 2. Solving the equation in step 2 ior x^ dx, we find

that
x^ dx = h du

Thus,
J

[x^
—r =
+ 4f J
—^ = —
u^ J 3
H ' du

Step 4
r
—A- dx r = —u _, du = ^-If u _, du
^ f 1

1
«-•* 1 ,
+ C = M"-* + C
3 -4 12

r A- fix 1 ,
Step 5 -^ r = (A-' -I-
4)--' + C

EXAMPLE 3 .V-V3 - Ix dx

Step 1 Let's try « = 3 — 2v.

Step 2 du = -2 dx
Step 3 The integrand contains three factor s: .r-, V 3 - 2x, and dx. The substi-
tution 1/ = 3 - Iv will change the factor V3 - Zx into Vu. We can solve the
equation in step 2 for the differential dx. so that dx = -2 du. It only remains
to rewrite the factor a" in terms of u. To this end, we solve the equation

M = 3 - 2v in step 1 for x, so that

A = i(3 - u)

and

X- = i(3 - «)- = i(9 - 6m + M-) = I - Im + W


SECTION 4.3 THE METHOD OF SUBSTITUTION. OR CHANGE OF VARIABLE 261

Therefore, we have

jrV3 -Ix dx = (f - Im + W)Vii(-\ du)


J J

= (- IVm + iuVu - iu-Vu) du


J

= \(-%ii"- + W~-W~)du
Step 4 A--V3 - Ix rf.v
= (- |m'/2 + |„3/2 _ 1^^5/2) ^„
J J

= - I m'/2 ^„ + 3
„3/2 J„ _ 1 „5/2 rfw
I J I

~ 4
"*"

8 3/2 5/2 8 'ijl


= - Im^/^ + A„5/2 _ J^„7/2 + (.

step 5 X-V3 - 2x dx
J
= - f(3 - Ix?'- + U^i - 2x?'~ - A(3 - Iv)^/- + C

EXAMPLE 4 sin 9.T dx


J
SOLUTION

Step 1 Let M = 9.V.

Step 2 du = 9 dx

Step 3 From step 2, we have dx = i du; hence,

I sin 9x ^v = I sin u (g tfi/) = 5 sin u du

Step 4 sin 9.x dx = j


h s\n u du = i \ sin u du = ^(-cos j/) + C
J
= — 5 cos M + C

Step 5 I sin 9.v dx = - i cos 9x + C

f sin 7j: <ic


EXAMPLE 5
(1 + COS 7x)^

SOLUTION

Step 1 Let u = \ + cos Ix.

Step 2 (/m = —7 sin Ix dx

^i"7..^v r__^j^
Step3 f
J ( 1 + COS 7xf J 1 u*

Step 4
f

J
—+ sinlx dx
cos

Ixr
^=
f
- \ du
= _ 1
f
K,-4
^
^j^
(1 J 1 u 1 J

= - — —-3- + C = — M"^ + C
7 21
262 CHAPTER 4 A^^TIDIFFERENTUTION AND DIFFERENTIAL EQUATIONS

r sin Ix dx
Step 5 -(1 + cos 7.V)"' + C
J (1
( 1 + cos IxT 21

After you have become familiar with the method of substitution, you may wish to

abbreviate the step-by-step procedure suggested above. The following example il-

lustrates how this can be done.

sec VTtan VJ dt
EXAMPLE 6 Evaluate

SOLUTION Let M = V7, so that du = 1/(2V7) dt and dt/Vt = 2 du. Then

f sec v7tan ^/i dl


r- sec vTtan V7 —^ = sec u tan u (2 du)

= 2 sec u tan m c^m = 2 sec m -I- C


= 2 sec VT-I- C

Justification of tlie Method of Substitution

The following theorem can be used to justify the method of substitution, or change
of variable.

THEOREM I Substitution
SECTION 4.3 THE METHOD OF SliBSTITUTlON. OR CHANGE OF VARIABLE 263

In step 3, we use the last equation to rewrite /(a) and dx in terms of ;/ and du.
Suppose the result is

fix) dx = hUt) du

where h is a suitable function. What the last equation really means is that

du
f(x) = h(u)--
dx

or
f(x) = h{g(x)]g'(x)

Now, step 4 produces an equation of the form

h(u) du = H(u) + C

and step 5 yields the result

fix) dx = I
/![g(A-)]g'U) dx = H[g{x)] + C

in conformitv with Theorem

Problem Set 4.3

In Problems 1 to 50. use substitution to evaluate each antiderivative ,, f 2v" — 1 ., f \ 1 + V.x


some cases
13 r^
_ ^-^?rdx
,

14 |-
,

dx
(indefinite integral). (In a suitable substitution is sug- I
J ^(,^-}
(6.t-' - g-^
9.t +1)''^*
i)3/2 J Va
gested.)
^
15
J
(.r + — ) (
'^
. ) d^v 16
J
(49.r - 42r + 9)*'^ dx
1 I (4a + 3)" dx. u = 4.r + 3

17 xV5 - .X dx 18 .r>\'l +X dx
J I

2 f(4r + 7)'' dt. u = Ar + 1


J
f t
' dt r V + 2
19 20 ;'
dx
J Vt+ 1
J V2 - V
3 .vV'4.x- + 15 dx, w = 4.r + 15
J

21 22 J(A: + 2)^Vl+.rdx
'''-' J (^_^,:/3
4 \
J (4 - 3x-f
23 j "^Sa^ + 5 x^ dx 24
J
VV + 1 x^ dx
.
I
_ s ds ..
=r. W _
= cj;
5.r + , ,^
16 6
^ r (8f + 2)dr —
-.,

\'5r +
,
16 J
;
(4r^ + 2t + 6)'^ 25
J
(
^^^
r- rll

V> + 4
26
C

J V3
\-

-J^
->-
Hv

7 ( 1 - a^/-)5^'Va iv, i/ = 1 - x^'-


I
35a dx
.J:2 sin

8 (.r - 6.V + 9)"'-' dx.


u = x-3 f
J 28 I (7 sin 5a + 3 cos 7a) dx

„ r r dx [ x^ + I

J (4.x- + 1)^ J V?+3x 29


JJ
8 cos (16a- 1) dx

.. r n ^^-T r ^'l + 1/(20 r


11 (5r + l)V5r' + 3/ - 2 dr 12 t^ dt
-
J J ^^
30 j
5 cos (8 3a) dx
264 CHAPTER 4 ANTIDIFFERENTIATION AND DIEFERENTIAL EQUATIONS

sec- 1 l.t dx 32 -csc^ 5x dx 49


J J Vl - 5 sec 3fl

dt__ ^ f dy
50
f cot (Vx/2) csc^ (Vx/2) £ic

sin^ 3f J cos^ 5y '"JJ y^


sec (2v + I) tan (2y + 1) dy 36 tan" CSi + 7) dt 51 (a) Evaluate / sin x cos x dx using the substitution u = sin x.

(b) Evaluate / sin at cos x dx using the identity sin x cos at =


/ /
j"
J sin 2x. (c) Show that the answers to parts (a) and (b) are
-sec
y tan -j dt 38
J
esc 10: cot 10: dz
consistent with each other. \Hint: Use the identity sin^ jt
=
id -cos It).

cos A cos (sin .V) dx. u sm x


52 Evaluate / sin mx cos ;u dx, where m and n are constants, by
, ,„ 4 ,„ J ,
using the identity sin a cos b = ^ sin {a + h) + i sin (a - b).
X sec 10a tan lOv dx
53 Evaluate
, , ,

=
, . r sin Va + 1 dx r J
x dx
.r' CSC- Ta-* dx. u Ta-* 42 ;

J Va+ 1
J
J VA-XT
,
A + 1

sin A dx f sec a (it by two different methods, and show that the answers are consis-
(2 + cos a)- J (3 + 2 tan a)' tent with each other, (a ) Use t he substitution u =x + 1. (b) Use
the substitution u = Va + 1.

cos 2y VS + sin 2y dy
54 Given that / is a function with domain (-1, ^o) such that
/'(a) = 2(1 + A) - and/(0) = 0, find/.
tan 3/ sec 3f V4 + sec 3t dt
55 Evaluate / atVSa - 1 dx by two different methods, and show
^'^^^ '^^ answers are consistent with each other, (a) U se the s ub-
cot Va CSC V^ dx
/- , stitution « = 5a - 1. (b) Use the substitution u = VSa - 1
Va
56 Suppose that g'{x) = (1 -I- a)"' holds for all values of a except
(sin I):)"'" cos 2x dx for a = -1. Given that lim g(x) = lim g(x) = 0, find g.

4.4 Differential Equations

Mathematical models for real-world phenomena often take the form of equations
relating various quantities and their rates of change. Since the rate of change of a
quantity is represented mathematically by a derivative, such equations often involve
derivatives or differentials. For instance, in the biochemistry of digestion, the en-
zyme which helps break proteins into amino acids, is produced in the small
trypsin,
intestine from an inactive proenzyme called trypsinogen. If y is the amount of
trypsin in the small intestine at time t. it is known that the rate dy/dt at which trypsin
is produced is proportional to the product of the amount of trypsinogen present and

the amount of trypsin present. Thus,

dv
-j- = A:(A - y)(B + y)
dt

where k is the constant of proportionality, A is the amount of trypsinogen present


when / = 0, and B is the amount of trypsin present when / = 0.
SECTION 4.4 DIFFERENTIAL EQUATIONS 265

An equation such as
dv
-j- = k{A - v)(B + y)
at

that involves derivatives or differentials of an unknown function is called a


difTerential equation. To solve a differential equation is to find the unknown func-
tion or functions that satisfy the equation.*
The simplest type of differential equation has the form

-— = JM or dy = f(x) dx
dx

Here /is a given function, and v is an unknown function of x. Evidently,

3' = g(x)

is a solution of this differential equation


and only if g is an antiderivative of/. Any
if

one solution v = g(x) is called a particular solution of the differential equation.


Given any such particular solution v = g(.v), we can write

V = g(x) + C
where C an arbitrary constant, to obtain other particular solutions.
is

Conversely, by Theorem 2 of Section 4.2, any particular solution of the differen-


tial equation
d\ ,
-7-=/(.v)
dx

on an open interval / has the form

y = g{.x) + C
where g is an antiderivative of/ and C is a suitable constant. In this sense,

y = g(x) + C
where C
an arbitrary constant and g is an antiderivative of/, represents the
is

complete solution of the differential equation. Since

f(x) dx = g{x) + C
J

the complete solution of the differential equation dy/dx = f{x) can be written as

y = jf(x) dx
Thus:

The complete solution of the differential equation


266 CHAPTER 4 ANTIDIFFERENTIATION AND DIFFERENTIAL EQUATIONS

EXAMPUK I Find the complete solution of the differential equation Jv = 6.v- dx.

SOLUTION y = I
\ dx
6x^(t(
6x^ = 6 \
\ X- dx
.
= 6— + C = 2x^ + C

The circumstances that give rise to differential equations often entail additional
conditions called side conditions, initial conditions, boundary conditions, or
constraints. For instance, in the differential equation for the formation of trypsin,
we have the initial condition that v = A when t = 0. Such additional conditions can
be used to single out the relevant particular solution from the complete solution.

EXAMPLE 2 Find the complete solution of the differential equation dy/dx =


cos 2x. Then find the particular solution satisfying the side condition that y = 1

when X = tt/\2.

SOLUTION The complete solution is given by

V = cos Ix dx = k sin 2x + C

where we evaluated the antiderivative by using the substitution u = 2x. Putting

y = 1 and x = tt/II into the complete solution, we obtain

1 = — 1

sin 2
( IT \
+ C= — 1

sin —+C=—+C
77 1

2 M2/ 2 6 4

from which it follows that

Substituting this value of C into the complete solution, we find that the particular
solution satisfying the side condition is

y = 5 sin Ir + f

The differential equation

ax-

is said to be separable since it can be rewritten in the form

dy=f{x)dx
in which the variables x and y are "separated" in the sense that all expressions
involving x are on one side of the equation and all expressions involving y are on the
other side. More generally:

A differential equation that can be rewritten in the form

G(y) dy = Fix) dx

where F and G are functions, is called separable. The general solution of such a
differential equation, obtained by separating the variables as shown and then
antidifferentiating both sides of the resulting equation, is given by

G(y) dy = F(.v) dx
J J
SECTION 4.4 DIFFERENTIAL EQUATIONS 267

In solving a separable differential equation by the indicated method, you can com-
bine the two constants of integration corresponding to / G( y) dy and / F(.x) dx into
a single constant C. If a side condition is given, it can be used to determine the value
of C.

EXAMPLE 3 Find the general solution of the differential equation y' = Ax-y-.
Then find the particular solution satisfying the side condition that x = 1 when
y=-\.
SOLUTION Here y' is used as an abbreviation for dy/dx. so the given differential
equation has the form

—^ = 4a:"v"
'
or d\ = 4.v''V" dx
dx

We can separate the variables if we divide both sides of the last equation by y" to
obtain
"V -,

-^ = Ax' dx (provided y 5^ 0)

Thus, y~- dy = 4x- dx


I

so
V-'
^
-]
+ Ci = 4 —+
x'

3
C-,
'

or — = Co
1

4x^

y 3

where Cq = C^ — Cj. The last equation may be rewritten as

3
^'~
3Co-4x^
Therefore, letting C= 3Co, we have the general solution

3
^ ~ C- 4x'

Putting X = I and y = — 1 in the general solution, we obtain

3
- 1
= or C= 1
C-4
Hence, the desired particular solution is

3
>'
1 - 4x'

When you solve a separable differential equation by separating the variables and
you can lose certain solutions. For instance, in the preceding
antidifferentiating,
example, when we divided by y- to separate the variables, we assumed that y # 0.
However, as you can check, the constant function given by y = k a solution of the
original differential equation y' = 4x~y~. Notice that the solution y = cannot be
obtained from the general solution
_ 3
'~
C- 4x'
268 CHAPTER 4 ANTIDIFFERENTIATION AND DIFFERENTIAL EQUATIONS

by assigning a value to the constant C. This shows that the general solution of a
separable differential equation might not he the complete solution. A solution (such

as V = in the example above) that cannot be obtained directly from the general
solution by assigning a value to the constant of integration is called a singular

solution.
The general solution of a separable differential equation involving x and y might
be an equation that only implicitly determines y as a function of v. This is illustrated

by the following example.

EXAMPLE 4 Find the general solution of the differential equation a d\ + y dy = 0.

SOLUTION Separating the variables and antidifferentiating, we have

X dx = -y dy so a dx = \ { -y) dy

hence, ^+ C, = - ^+ C. or —+^= C. - C,

Therefore, x~ + y^ = 2(C2 - C,), that is, .v' + y' = C, where we have put C =
2(C2 - Ci ). Thus, the general solution of x dx + y dy= is

x~ + y- = C

Second-Order Differential Equations


Up to now, we have considered only "first-order" differential equations, that is,

equations involving only first derivatives. By definition, the highest order of all the

derivatives involved in a differential equation is called the order of the equation.


For instance,
d'y ^ . dy
r- + 2 = X
dx^ dx

is a second-order differential equation, and

is an Mth-order differential equation.


Sometimes, the general solution of a second-order differential equation can be
obtained by two successive antidifferentiations. The resulting solution will involve
two arbitrary constants that cannot be combined into one constant. Two side condi-
tions are required to determine these constants.

EXAMPLE 5 Find the general solution of the second-order differential equation


y" = -2x + 1, and then find the particular solution that satisfies the following two
initial-value conditions: y = - 1 when .v = and y' = 1 when .v = 0.

SOLUTION Since

d
>•
^Yx'
we can write the differential equation as


dx
y' = -2v+l or d{y' ) = {-Zx + \) dx
SECTION 4.4 DIFFERENTL\L EQI'ATIONS 269

Antidifferentiating both sides of the last equation, we obtain

y' =
J
(-2v + I) dx= -x- + .x: + d
Therefore, we have

dx
= -x^ + .t + d dy = (-.r- + x + CO dx

Again, we antidifferentiate both sides of the last equation and obtain

y = I (-X- + x + Ci)dx + —+ Ci.x- + C,

Thus, the seneral solution is

V =
3
+ —+ 2
Ci.v + d
Notice, here, that there is no way to combine the two constants C\ and Ci into a
single constant since Ci is a multiplier of the variable x.
To find the particular solution corresponding to the given initial conditions, we
begin by substituting y = - 1 and x = into the general solution to obtain

-1 =C2
Then we substitute y' = I and a = into the previously obtained equation

y' = -X- +.V + C,

and find that 1 = C,

Substituting these values of Ci and C2 into the general solution, we obtain the
desired particular solution,

y=
x^
+ — + A--1
X-

Linear Motion
Differential equations are important in the study of the motion of physical objects
(cars, projectiles, balls, planets, electrons, and so forth). Often we disregard the
size, shape, and orientation of such objects and think of them as particles. Here we
Figure 1
consider the motion of a particle P along a linear scale, which we call the s axis

(Figure 1). The equation of motion of F,

s=f{t)

gives the coordinate 5 of P in terms of the elapsed time t since some arbitrary (but
fixed) initial instant. The instantaneous velocity v and the instantaneous acceleration
a of P are given by the equations
270 CHAPTER 4 ANTIDIFFERENTIATION AND DIFFERENTIAL EQUATIONS

If V or a is a known function of /, subject to suitable initial conditions, it may be


possible to solve the resulting differential equations for the law of motion s =/(/).
This is illustrated in the following examples.

In Examples 6 and 7, find the law of motion s = f(t) fi'om the information given.

EXAMPLE 6 a = It - t^\ V = Q when t = and s = when / = 0.

SOLUTION We have dv/dt = a = 2t - I', so dv = (It - /-) dt. A first anti-

differentiation gives

V = L/v = at - t~)dt = t^
-'— + C|
J

Since v = when / = 0, it follows that C\ = 0. Also,

—=
dt
V = f
3
so ds = \t-
^ 3/
\ dt

and a second antidifferentiation gives

Since s = Q when / = 0, it follows that C^ = and the equation of motion is

*
3 12

EXAMPLE 7 A car is braked to a stop with constant deceleration. The car stops 8
seconds after the brakes are applied and travels 200 feet during this time. Find the
law of motion of the car during this 8-second interval. Also, find the acceleration
and determine the speed of the car at the instant the brakes are first applied.

SOLUTION Represent the car by a particle on the s axis moving, say, to the right.

Start reckoning time / from the instant when the brakes are first applied, and place
the origin at the position of the car when t = 0. Here the acceleration a is constant,
and we have
5 =
SECTION 4.4 DIFFERENTIAL EQUATIONS 271

Now we substitute C, = -8a into the equation 5 = iat^ + C\t to obtain

s = iar - Sat

Finally, we substitute 5 = 200 and / = 8 into the last equation and find that

200 = ia{Sf - 8a(8) or 200 = 32a - 64a = -32a


from which it follows that during the entire 8-second interval,

a = - W = -¥ ft/s-

Here the negative sign indicates deceleration. By substituting this value of a into the
equation s = kat' - Sat. we obtain the law of motion

25/2
= = 50'
t("T^)''-K^)' °^ ^

The speed of the car at the instant when r = is obtained by putting f = in the
equation v = at + Ci to obtain

v = C, -8a = -8(-¥) = 50ft/s

Figure 2 An object falling near the surface of the earth experiences some force because of
air resistance; however, in many situations this force is negligible, especially when
" streamlined," and has not attained a
k the object has a high density,
very high velocity. Such an object
is more or
falls
less

with a constant acceleration g. called the


acceleration of gravity. The value of g is approximately 32 feet per second squared
or 980 centimeters per second squared, or 9.8 meters per second squared.

//////////////////////////////////// If such an object P is projected straight upward, it is usually convenient to take


the s axis pointing straight upward with the origin at the surface of the earth (Fig-
ure 2). Then the acceleration a of P is negative (it will slow down, stop, then begin
to fall back down): hence.

Figure 3 d-s

dt-

On the other hand, if the object P is dropped or thrown straight downward from
an initial height /;, it is usually convenient to take the s axis pointing downward with
the origin h units above the surface of the earth (Figure 3). Then the acceleration a
77777777777777777/77777777777777777/ of P is positive; hence.

d-s

dt-

(3 EXAMPLE 8 An iron ball is thrown vertically upward, starting 2 meters above the
ground, with an initial velocity of 8 meters per second. How many seconds will
elapse before the ball strikes the ground?

SOLUTION We set up the coordinate axis as in Figure 2. Here, dv/dt = -g, so


dv = —g dt and

/-/ (-g)dt= -g\dt= -gt + C^


272 CHAPTER 4 ANTIDIFFERENTIATION AND DIFFERENTIAL EQUATIONS

When f = 0, V = 8 meters per second, so

8 = -g(0) + C| or C| =

Therefore

V = -gr + 8

The last equation can be rewritten as

ds
= -8' + or ds={-gt + ^)dt
dl

Antidifferentiating, we find that

^ = {-gt + 8)dt= -igr + 8/ + C2


J

When t = 0. s = 2 meters; hence, substituting these values into the last equation
yields

2= -ig(0)' + 8(0) + Cj or C2 = 2

Therefore, the equation of motion of the ball is

s= -igt' + 8? + 2

where g = 9.8 meters per second squared. Thus,

s = -4.9r + 8r + 2

When the ball strikes the surface of the earth, ^ = 0, so that

= -4.9r + 8r + 2 or 4.9r - 8f - 2 =
Solving the last equation by the quadratic formula, we find that

8 ± V64 - 4(4.9)(-2) 8 ± Vl03.2


2(4.9) 9.8

Because t is positive when the ball lands, we reject the negative solution and con-
clude that

8 + V103.2
t
= = 1 .85 seconds
9.8

Problem Set 4.4

In Problems 1 to 8, find the complete solution of each differential 7 —=


dt
/ " + sin f 8 DnM = ^ - CSC- 50
equation.

In Problems 9 to 14, find the particular solution of the given differ-


1 -t- = S.v'' + 3.r= + 1 2 -f^ = 2at'' -6x^+ n ential equation that satisfies the indicated side condition.
dx dx

dy 6 (x- - 4f
3 -T" = ^r + 15.r- + 10 4 v' 9 —^ = 5 - 3.v: V = 4 when .v =
dx X' Ix- dx

6 dy = (5/ + 12)' dl 10 -i- = 3.t" + -v; V = — 2 when .v = 1


dx dx
SECTION 4.4 DIFFERENTIAL EQUATIONS 273

11 ^= dt
f' + —r : V = 1 when t = -2 37-^ =
dt'
2,^ + 3 38 y" = {X + if

12 >•' = V^ + 2; y = 5 when a = 4
40 Dly = 1

13 v' = 3 sin- 1 when .V = —


3
41 -—r = cos Zx 42 v" = sin 3.r
ds dx~
14 —i^ = sec" 6r. s = - 1 when r =

In Problems 43 to 50, find the particular solution of the given differ-


In Problems 15 to 26. find the general solution of each separable ential equation that satisfies the indicated side conditions.
differentia] equation.

15_v' =.v(.r'-3)'' 43 —dx^ = 6.V -t- 1; V = 2 and v' = 3 when .r =

16 (3.r- + lx+ \f dy = {6.x + 2) dx


44 —d-\ r
-V = Vx; V = 3 and v' = 2 when .v = 9
ds dx-
17 V.r' + 7 dy = x' dx 18 -- = (r + 1)V
dt
d's
45 —-J- = 2; 5 = when t = and 5 = when = -3
19 \/2v+ 1 dy = y- dx dt^
1 t

^
20 (
v= - W) dy = (.V- + \G)dx 21 4^ = ""^
^' ,"^
46 —d-\
-V = 3.r-: v = - 1 when x = and \ = 9 when .r = 2
at 5v dx-

rfv rfv sin 2x 47 y" = 3(2 + 5.r)': y = 2 and y = - when =


v-f = .vWl(h-+
' 1 .r I
22 1 23
(iv cos 3>'
.o d-s >,
48 --T- = V5r - 4; i = 2 and 5 = -3 when t = 4
24 CSC .T cos y dx + tan .v tan > dy = dr
25 y- (iv — esc .V dy =
49 y" = sin—; v' = when x = tt and v = 2 when .r =
26 COS" 3/ sin 4s ds - cos" 4s dt =

In Problems 27 to 32, find the particular solution of the given sepa- 50 —^r
dr
= 2 sec- f tan f: i = and —=
dt
when t =
rable differentia! equation that satisfies the indicated side condition.

27 — = ^2-x'
dx
— dx
y-, : y = 2 when x = 9 51 A
tion s
particle
= fit),
moving along a straight line has the equation of mo-
where t is in seconds and j is in meters. Its velocity
V satisfies the equation v = f- - 8r -i- 15. If i = 1 when t = 0,
ds
28 s = k when t = 2 find i when r = 3.
dt Vf- -I- 1

52 A particle, starting with an initial velocity- of 25 meters per sec-


29 /- -'
dW=(l - t^^^f dt\W=-\ when t
ond, moves in a straight line through a resisting medium which
decreases the velocity of the particle at a constant rate of 10
30 —j- = ,/— : V = 4 when x = 1
meters per second each second. How far will the particle travel
dx V .V '

before coming to rest?

31 sec 3t ds + esc 2s dt = 0: s = — when t E 53 The brakes on a certain car can stop the car in 200 feet from a
speed of 55 miles per hour. Assume that when the brakes are

32 CSC V dx + COS" .v d\ = 0: v = — when x = — applied, the car has a constant negative acceleration, (a)
much time in seconds required to bring the car to a stop from
How
2 4 is

55 miles per hour? (b) If the car is brought to a stop from 55


In Problems 33 to 42, find the general solution of each second-order miles per hour, how far will it have moved by the time its speed
differential equation. is reduced to 25 miles per hour? (Recall that 1 mile is 5280 feet.)

d\ EI54 From the top edge of a building 20 meters high, a stone


33 —dx'V = 3.V- -I- 2v -t- 1 34 v" = (5.x + 1)"
thrown vertically upward with an initial velocity of 30 meters
is

per second, (a) In how many seconds will the stone strike the

35 v" = vV+5 36 S" = 5 ground? (b) How high will the stone rise? (c) How fast will the

it + 7)' stone be falling when it hits the ground?


274 CHAFIER 4 ANTIDIFFERENTIATION AND DIFFERENTIAL EQUATIONS

I
55 A balloon is rising at the constant rate of 10 feet per second and straight upward if they are released from the assistant's hand at

is 1(30 feet from the ground at the instant when the aeronaut an altitude of 2 meters?
drops her binoculars, (a) How long will it take the binoculars to
strike the ground? (b) With what speed will the binoculars strike 58 Suppose that a particle P moves along the s axis with a constant

the ground? acceleration a. Let vo be the velocity of the particle when


t = 0, and let So be its coordinate when I = 0. Show that
56 A projectile is fired vertically upward by a cannon with an initial
(a) V = ar -I- Vo and (b) s = iar + vni + .vo.
velocity of I'o meters per second. At what speed will the projec-
tile be moving when it returns and strikes the hapless cannoneer? 59 A stone is dropped from a height of h meters with zero ini-

(Neglect air resistance.) tial velocity, and it hits the ground T seconds later. Show that
h = 4.97"-.
I 57 A balloon is rising vertically at the constant rate of 1 meter per
second and has reached an altitude of 8 meters at the instant 60 Consider the differential equation dy/dt = k(A - y)(B + y) for
when an assistant on the ground directly under the balloon at- the formation of trypsin in the small intestine. Assuming that
tempts to toss the aeronaut's binoculars up to her. What is the A > B, determine the time I at which trypsin is being formed
minimum velocity with which the binoculars should be thrown most rapidly.

4.5 Applications of Differential Equations


In Section 4.4, we saw an application of differential equations to linear-motion
problems. Fuither simple applications to geometi^, physics, and economics are
given in this section.

A first-order differential equation of the form

gix)
dx
where g is a given function, can be interpreted geometrically as a condition on the
slope dy/dx of the tangent line to the graph of v =/(-v), where / is an unknown
The complete solution of the
function. differential equation gives all functions/
whose graphs satisfy this condition. A side condition amounts to an additional
requirement that the graph of /contain a specified point.

EXAMPLE 1

(a) Find all curves in the plane satisfying the condition that the slope of the
tangent at each point is 3 times the abscissa of that point.

(b) Sketch graphs of several such curves on the same .xy coordinate system.

(c) Find the equation of that particular curve satisfying the condition in part (a)

which contains the point (1, 2).

(a) The differential equation expressing the condition on the slope of the tan-
gent is

dy_
= 3.V
dx

The complete solution of this differential equation is

= I 3xdjc = 3 I xdx =
^ 3 —+C
2
SECTION 4.5 APPLICATIONS OF DIFFERENTIAL EQUATIONS 275

Figure 1 Thus, an equation of a curve satisfying the given condition is of the form

where C is a constant. There is a different curve for each different value of C.

(b) The curves corresponding to C = —2, C = 0, C= 5, C= 2, and C= 4


are sketched in Figure 1.

(c) Here we have to impose the side condition that v = 2 when .v = 1 . Setting
y= 2 and jc = 1 in the equation y = ix' + C and solving for C, we find that
C = 2 — i = i. The desired equation is, therefore.

Work Done by a Variable Force


Suppose that a constant force F, having a direction parallel to the s axis, acts on a
particle P which moves along this axis from an original position with coordinate Sq
to a final position with coordinate S] (Figure 2). Then, by definition, the force does
an amount of work W on the particle given by

H' = F-(5, -So)

A positive (respectively, a negative) force is understood to act in the positive


(respectively, the negative) direction along the i axis.
We now take up the problem of calculating the work done when the applied force
is not necessarily constant but still acts in a direction parallel to the s axis. We
suppose that P starts at an initial position with coordinate sq- and we denote by W
the net work done by the variable force F in moving P from its original position to
the position s (Figure 3). Since F may not be constant, we cannot calculate W
simply by multiplying F by s — Sq.

Figure 2 In this case, suppose that P moves from 5 to i + As, caus-

original position of P final position of P


ing the net work done by the force to change from W to
W + AW AW
r I

D= "^
moving P from
change as
very small. Thus,
(Figure 4). Here,

P is
s to s

so moved,
if F
+

is
it
is the
As. Although the force
should not change
the force acting
work done by

on P
much
F

at position 5,
if
F
might
As
in

is

we should have

AW
Figure 3 AW == F As
As
original position of P
As As approaches 0, this approximation should become bet-
ter and better: hence, we should have
I
f \
I

o-- dW =
ds
lim
As-»o
AW =
As
F

Therefore, the net work W done by the variable force F satis-


fies the differential equation
Figure 4

O 0-' with the initial condition that


cM' = F

W=
ds

when s = sq.
276 CHAPTER 4 ANTIDUTERENTIATION AND DIFFERENTIAL EQUATIONS

EXAMPi.K 2 The F acting on a particle P is given by F = 1/s^, where s is


force
the coordinate of P. How much work is done by this force in moving P from 5 = 1

to s = 9?

SOLUTION Let W denote the net work done by the force in moving P from the

point with coordinate 1 to the point with coordinate s. We want to find the value of

W when 5 = 9. Since .

dW = F ds = ^ds
r 1 -1
it follows that W= J
\
—;- ds = + C
S'' s

When 5 = 1, we have W = 0, so that

-1
= + C
1

hence, C= 1 and
-1
W= + 1

When 5 = 9, we have
W= - 5 + 1 = f unit of work

EXAMPLE 3 Consider the apparatus shown in Figure 5, in which a particle P that

is attached to a stanchion by a perfectly elastic spring can slide without friction


along the horizontal s axis. The particle starts at 5 = and is pulled by a force F to
a final position s = b. At the start, when 5 =
assume 0, that the spring is relaxed

and F= 0. Find the work done by F in pulling P from s = to s = b.

SOLUTION Because the spring is perfectly elastic. Hooke's law requires that the

force F be proportional to the displacement 5; that is,

F= ks
Figure 5
where Jt is a constant called the spring constant. (The stiffer the spring, the larger
initiai^position
^^^^ ^^^^^_^^
^j^^ ^3,^^ of /t. ) Let W be the net work done by F in pulling P from the origin to the
of/" position 5, so that dW = F ds - ks ds. Antidifferentiation gives

W= ksds = k\
\ksds
\ k sds = \ k—- + C

When 5 = 0, we have W = 0, so that = k{Qr/l) + C, and it follows that C=


and W= ks^/2. Thus, when s = b, we have
kb'
W=
2

EXAMPLE 4 A perfectly elastic spring is stretched from its relaxed position


through 2 meters. When it is extended these 2 meters, the stretching force on the
spring is 3 newtons. How much work is done?

SOLUTION Reasoning as in Example 3, we have F = ks, so k = F/s. Because


F= 3 newtons when 5 = 2 meters, it follows that

k = —F = —3 newtons per meter


SECTION 4.5 APPLICATIONS OF DIFFERENTL4L EQUATIONS 277

Using the result of Example 3. we have

W= k — ~3 2-
T~ ~ 3 newton-meters

In the international system (SI) of units, one newton-meter 1 N m) is defined to be


(

onejouled J). (One pound of force is approximately 4.45 newtons.) Therefore, the
work done in stretching the spring is 3 joules.

Remarks on Setting Up Differential Equations

As we mentioned, many engineers and scientists like to think ofdx as an "infinitesi-


mal bit of .V." and they prefer to regard / dx as a • summation" of all the infinitesi-
mal bits of X to give the quantity x. That is, apart from an additive
constant of
integration, .r = / dx. They persist in this point of view, in spite of its possible lack
of "mathematical rigor," because it allows them to set up differential equations
quickh' and easily. Thus, for instance, in the textbook Physics by Paul Tipler,
Worth Publishers. Inc., New York. 1976, on page 175. we find, "This definition
is equivalent to saying that the work dW done over a very short interval d\ is
dW ^F^dx.-
la 1960, the mathematical logician
Abraham Robinson (1918-1974) discovered
a way to make
Leibniz's infinitesimals mathematically rigorous and thus to justify
the kind of reasoning that engineers and scientists have indulged in for more
than
three hundred years. In the remainder of this book, we use arguments involving
infinitesimals whenever it is convenient to do so. In every case, rigorous arguments
can be made to justify the result in question.

Newton's Law of Universal Gravitation

The following example illustrates a typical physical application of the infinitesimal


point of view.

EXAMPLE 5 According to NcHlon's law of universal gravitation, two particles


with masses m, and w^ kilograms which are separated by a distance of s meters
attract each other with a force of

WjOTi
newtons

where G is a constant given by

G= 6.672 X 10"^' N-mVkg-


If 1 kilogram of lead is distributed uniformly along the .r axis between the origin and
the point with .v coordinate 2 meters, how much gravitational force does it exert on
a 1 -kilogram particle P situated on the .x axis 1 meter to the left of the origin?

SOLUTION Let F denote the total gravitational force of attraction on P of the


portion of the mass lying on the inter\al [0. (Figure 6). The linear densitv of lead
Figure 6 .v]

on the interval [0, 2] is

5 = 0.5 kg/m
hence, the infinitesimal mass dm of the portion of lead on the subinterval +
[.v, x dx]
is given by
dm = 0.5 dx kg
278 CHAPTER 4 ANTIDIF-FERENTIATION AND DIFreRENTIAL EQUATIONS

Since the distance between P and dm is \ + x meters (Figure 6), this infinitesimal
mass exerts an infmitesimal gravitational force dF on the unit mass P given by

1 dm dx
dF = G ^ = 0.5G T- newtons
(1 + x)- (1 +.V)-

r r dx dx
Therefore, F= \
dF = \ 0.5G ^ = 0.5G 7
J J (\ + X)- J (1 +.v)^

Making the change of variable m = 1 + x. so that du = dx, we find th

f dx f , «-'
F = 0.5G = 0.5G // - du = 0.5C + C
J (\
(1 + xf J -
^ 0.5G 0.5G
= C = C-
1 +a:

Here we have the side condition that F= when .r = 0, so that

0.5G
= C j — and
SECTION 4.5 APPLICATIONS OF DIFFERENTIAL EQUATIONS 279

EXAMPLE 7 The marginal revenue for a digital watch is expressed by

dR
60.000 - 40.000(1 + x)~- dollars per 1000 watches
dx

where x represents the demand in thousands of watches. Express the total sales

revenue in terms of x. given that for x = 1 (thousand watches), the total sales

revenue is $38,000. If the demand increases to .v = 4 (thousand watches), what is

the total sales revenue?

R = \ [60,000 - 40.000(1 + x)-'] dx = 60,000 dx - 40,000 1(1+ x)"- dx


J

The change of variable m = 1 + .v in the second integral gives

(1 + xy- dx = \ u~- du = -«"' + A") = - + K,


1 + X

Hence,

R = 60,000.r + —+
40,000
\ X
+ K where K= -40,000^,

Putting X = 1 and R = 38,000 in the last equation and solving for K, we find that
K= -42,000; hence,

40,000
R = 60,000.v + 42,000
1 +.V

When X = 4, we have R = 240,000 + 8000 - 42.000 = $206,000.

Problem Set 4.5

In Problems 1 to 6, the slope dy/dx of the tangent line to the graph of 1 A curve in the plane contains the point (0, 1) and satisfies the

a function y = /(.t) is given by a differential equation. Find the func- condition that at each point on the curve, the product of the
tion/if its graph contains the indicated point (a, b). ordinate and the slope of the tangent is the abscissa. Find an
equation of the curve.
dv
1 -;-= 1 - 3x, (a, fe) = (-1, 4) 8 The tangent to a certain curve at (0, 3) makes an angle of Tr/4
dx
with the positive .v axis, and at each point P on the curve, the

dv normal to the curve at P intersects the x axis at a point Q in such


2 -^ = X- + \. {a. b) = (-3, 5) a way that the vertical projection of the segment PQ on the x axis
dx
has constant length (independent of/*). Find an equation of such
a curve.
3^=(y-)\,a.b) = a.l)
dx \ .r
'

dy in Problems 9 to 14. the force F acting on a particle P moving along


4 -;- = 2X-V-. (a. b) = (0, 1)
dx the i axis is given in terms of the coordinate s oi P. Find the work
done by F in moving P from j = io to j = i| . In each case, assume
r-
5 —dy— = —3, .,

cos 3x, (a, o)


,
=
/ TT
I — F is in newtons and 5 is in meters.
dx \ 3

9 F = 2i, 5o= 1, -Si


= 5
6 -^ = 2 tan- x sec .v, (a, b) = (0, 1)
dx 10 f = 400jVi + j^ 5o = 0, ii = 3
280 CHAPTER 4 ANTIDIFFERENTUTION AND DIFFERENTIAL EQUATIONS

11 F = Vs - I. S(,= \, s\ = \0 est velocity v'escapc for which the particle will continue to move
forever away from the sphere is given by i'„,.apf = \2GMjr.
12 F = {l + sf\so= -7,5, = 7
This is called the escape velocity, (c) The radius of the earth is

approximately 6.371 x lo"" meters, and its mass is approxi-


13 F = sin—, So = 0. S] = IT
mately 5.983 X lO""* kilograms. Neglecting air resistance, find
the escape velocity for the earth, (d) The radius of the moon is

14 F = sin s cos i, so = 0. Si = — approximately 1.738 x 10^ meters and its mass is approxi-
mately 7. .347 X 10" kilograms. Find the escape velocity for the

15 A perfectly elastic spring is stretched from its relaxed position moon.


through 0.5 meter. When it is stretched 0.5 meter, the stretching
21 Using the result of Problem 19, determine how many joules of
force on the spring is 40 ncwtons. How much work is done?
work are required to transport I kilogram of mass from the sur-
16 Consider the apparatus shown in Figure 5, but suppose that the face of the earth to the orbit of the moon. Neglect the gravita-
spring is relaxed when the particle is at the position x = c. As- tional attraction of the moon, take the radius of the earth to be
sume that P is = a with a > c and then to
pulled to the position s 6.37 X 10'' meters, take the mass of the earth to be 5.98 x lO"''

the position s = b with b> a. Let Fa be the force on P when kilograms, and suppose that the orbit of the moon is approxi-
s = a. and let f be the force on P when s = b. (a) Show that the
/,
mately a circle of radius 3.80 x 10* meters with center at the

spring constant is given by center of the earth.

I 22 A homogeneous sphere so dense that the escape velocity from its

b — a surface exceeds the .speed of light c is called a black hole. Using

- = aF^ - the result in Problem 20b, show that a homogeneous sphere of


(b) Show that ci.Fi, Fa) bFa- (e) Show that the work
Wa,h done in stretching the spring from 5 = a to i- = fc is given
mass M will form a black hole if its radius is less than the

Schwarzschild radius 2GM/c'.


by 'Wa.t = Hb - a)(Fa + F,).

17 A perfectly ela.stie spring with spring constant k = 150 pounds


23 If a mass of M kilograms is distributed uniformly along the .v

axis between the origin and the point x = a meters, a > 0, show
per inch is stretched through 6 inches. At the start of the stretch-
that the gravitational force of attraction F of this mass on a
ing, the spring is not relaxed, and the force on the spring is 300
1 -kilogram particle P situated at the point x = —b, b > 0, is
pounds. How much work in foot-pounds is done in stretching the

spring through the 6 inches'?


given by F = MG/[b(a + b)].

1 18 A bucket containing sand is lifted, starting at ground level, at a lA Two thin homogeneous rods, each of mass M and each 1 meter
constant speed of 2 meters per second. The bucket itself weighs long, are placed end to end along the x axis so that they meet at

12 newtons and at the start is filled with 30 kilograms of sand. the origin. Let F{n) denote the gravitational force of attraction

As the bucket is lifted, sand runs out a hole in the bottom at a of the right-hand rod on the portion of the left-hand rod that lies

constant rate of I kilogram per second. How much work (in on the interval [—1, w — 1] for sws 1. Show that F satis-

joules) is done in lifting the bucket up to the height at which the fies the differential equation (1 - m)(2 — vv) (/F = A/"G (^vf

last of the sand runs out? (Hint: One kilogram weighs approxi- with the initial-value condition F(0) = 0.

mately 9.8 newtons.)


25 The marginal cost C of manufacturing .v nylon backpacks is

19 A homogeneous sphere of mass M and radius r exerts a gravita- given by C'U) = 5 -I- (8/Va-) dollars per backpack, (a) Find the
tional attraction on a particle P of mass m that is the same as if cost C of manufacturing .r backpacks if it costs $1200 to manu-
all the mass of the sphere were concentrated at its center facture 100 backpacks, (b) If each backpack sells for $21, find
provided that the distance from P to the center of the sphere is the manufacturer's profit P if .v backpacks are manufactured and
not less than r. If G
Newton's universal constant of gravita-
is sold.

tion, show that the work W(R) done against the force of gravity
26 For the production of a certain commodity, suppose that the
by carrying P from the surt'ace of the sphere to a point R units
from the center of the sphere is given by
marginal cost C is a quadratic function of the production level
x. Co is the fixed cost, / is the most efficient production level. A
is the average production cost per unit at production level /. and
= GMm
W(/?)
W R' B = C'U) is the marginal cost at production level /. (a) Find the
cost function C in terms of .v. Cq. /, A. and B. (b) What is the
120 (a) In Problem 19, show that the work required to move the
P
economic significance of the condition A> B + (Co//)?
particle infinitely far from the sphere, starting from the surface
of the sphere, is given by CMm/r. (b) If the particle P is shot 27 The marginal cost dC/dx of producing x thousand cans of dog
away from the surface of the sphere with an initial velocity v, food is given by dCjdx = 30.v'"'' dollars per 1000 cans. Given

then its initial kinetic energy is given by kinv'. If all this energy that 8000 cans can be produced for $2600: (a) What is the fixed
is used in work against the force of gravity, show that the small- cost? (b) How much will it cost to produce 125.000 cans?
SECTION 4.6 THE H.4RMOMC-OSCILLATOR EQUATION 281

EI28 Supfwse that the marginal cost C of producing v electric razors 31 The cost of production of a newspaper in a small town is

per week is a quadratic function of v and that the fixed cost is C= 100 + 3.5.V dollars per month, where .v is the number of
$5000 per week. Assume that the most efficient production level subscriptions to the paper. The marginal revenue is ^'(.v) =
is 20.000 razors per week and that at this production level the 13 — (V40) dollars per subscriber per month, and R{0) = 0.
average production cost per razor is $15. Finally, suppose that (a) Find R as a function of .t. (b) Find the monthly profit f as a
when operating at the most efficient production level, it costs function of .r. (c) What value of .r will maximize the profit? (d) If
$10.75 to manufacture one more razor. Thus, taking the mar- the value of .v is that which maximizes the profit, what price does
ginal cost C'CO.OOO) to be $10.75, find a formula for the cost each subscriber pay per month for a subscription?
C{x) of manufacturing .v electric razors per week.
i£]32 In Problem 28, find the production level for which the produc-
29 The cost C of manufacturing .v thousand cassette tapes is tion cost per razor is minimum.
C(x) = 600 + 60.V dollars. The corresponding marginal revenue
33 The Frostbite Frozen Foods Company can manufacture thou-
is R'{x) = 400 — 8.V dollars per 1000 tapes. Find the number of .v

sand frozen macaroni and cheese dinners for 70O.v -•-


5000
tapes (in thousands) that will ma.\imize the manufacturer's profit.
dollars. The marginal revenue is R'[x) = 1030 - .x dollars
EI30 In Problem 28, suppose that the marginal weekly revenue is per 1000 dinners. How many macaroni and cheese dinners
given by R'lx) = 40 — (.v/500) dollars per razor. Find (a) the should be manufactured to maximize (a) the revenue and
production levels for which the manufacturer breaks even and (b) the profit? (c) When the profit is maximized, what price
(b) the production level for which the manufacturer's weekly does Frostbite charge for a single macaroni dinner? [Assume
profit is maximum. [Assume that /?(0) = 0.] that R(0) = 0.]

4.6 The Harmonic-Oscillator Equation


There are a number of real-world quantities that oscillate or vibrate in a unifoiTn
manner, repeating themselves periodically in definite intervals of time. E.xamples
include alternating electric currents; sound waves; light waves, radio waves, and
other electromagnetic waves; pendulums, mass-spring systems, and other mechani-
cal oscillators; tides, geysers, seismic waves, the seasons, climatic cycles, and other
periodic phenomena of interest in the earth sciences; and biological phenomena
ranging from a human heartbeat to periodic vaiiation of the population of a plant or
animal species. The simplest mathematical model for such quantities is

v =A cos (wf — <t> I

where v is the oscillating quantity


t is time '

A is a positive constant called the amplitude of the oscillation


a; (the Greek letter omega) is a positive constant called the angular
frequency of the oscillation
d) (the Greek letter phi) is a constant called the phase angle
The equation v =A cos (wf — rf)) gives the value of y at time /, where f is meas-
ured from an arbitrary but fixed initial moment when f = 0. As we shall see, the
constants A. w, and (t) determine certain specific features of the oscillation. We
define the positive constant p (the Greek letter nu) by

and we refer to v as the frequency of the oscillation. If r is measured in seconds,


282 CHAPIER 4 ANTIDII'TERENTIATION AND DIFKERKNTIAl. EQUATIONS

then V represents the number of complete oscillations, or cycles, per second. One
cycle per second is called a hertz (abbreviated Hz) in honor of the German physicist
Heinrich Hertz (1857-1894). who discovered radio waves in the late 1880s. The
quantity 7" defined by

7=

is called the period of the oscillation.


The graph of v =A cos (a>/ — <}>) (Figure 1) resembles the graph of the cosine
function, except that it is stretched vertically by a factor of A (the amplitude) and
makes one complete oscillation (cycle) whenever / increases by 7" = \/v units
(Problem 8). A quantity v oscillating in accordance with the equation
y =A cos (tu/ - </)) is said to be undergoing simple harmonic oscillation. Any
physical device capable of producing a quantity undergoing simple harmonic oscil-
lation is called a harmonic oscillator.
In order to derive a differential equation for a harmonic oscillator, suppose that
Heinrich Hertz =A
V cos (w/

d\
Figure I Then = —0}A sin (wf - (^)

d^ = —
and —urA cos {wt (t>)

d-v d-\
Therefore, + w-v =
dt- dr
The second-order differential equation

d-\
-+ co-y =

is called the harmonic-oscillator equation. The calculation


above proves the following theorem.

THEOREM 1 The Harmonic-Oscillator Equation

UA.
SECTION 4.6 THE HARMONIC-OSCILLATOR EQUATION 283

The Superposition Theorem

(i) The superposition of any two solutions of a harmonic-oscillator equation


is again a solution of that equation.

(ii) Any constant multiple of a solution of a harmonic-oscillator equation is

again a solution of that equation.

We prove (i) here and leave the proof of (ii) as an exercise (Problem 23). Thus,

suppose that Vi and V2 are solutions of the harmonic-oscillator equation

d-y
-rr + ury =

so that
d'V]

df
w~Vi = and —^ +
d-y.

dt-
w-\n =

Adding corresponding sides of the last two equations, we obtain

dyi d~y2
+ oj'Vi + w'Vt =
df ~df
d-(y, +y.)
+ a;-( Vi -I- V2) =
dt-

Therefore, the superposition vj -I- v^ satisfies the original differential equation.

By combining parts (i) and (ii) of Theorem 2, we obtain the following result: If
^1 and At are constants and if vj and vt are solutions of the harmonic-oscillator
equation

d^y
-~ + or\ =
dt-

then y = A'l vi + k-,\2

is also a solution of the equation. In particular, taking jti = 1 and /tj = - 1 we


, see
that

y = y\- yi

is a solution of the equation. Thus, the difference of any two solutions of a har-
monic-oscillator equation again a solution of that equation.
is

If y is a solution of the hamionic-oscillator equation

d-\'

dt'

then
(f)"^-'-^
can be considered to be a measure of the total energy stored in the oscillator. In
various special cases, S may have to be multiplied by a suitable constant K to obtain
the actual energy

E = KZ
in appropriate units(Problems 17 and 21). Thus, Theorem 3 can be regarded as a
conservation-of-energy theorem for harmonic oscillators.
284 CHAPTER 4 ANTIDIFFERENTIATION AND DIFFERENTIAL EQUATIONS

THEOREM 3 Conservation of Energy for a Harmonic Oscillator

Let V be a solution of the harmonic-oscillator equation

-PT + w-v =
dt~

and let

+ (oY
-(^r
Then S is a constant.

dt
dy
dt
d-y
dr
,
-V ^=
d\
dt
2
dy
-^
dt ^
d\
—^
df
+ to'Y
-, \
= 2 ^
d\
(0) =

Therefore. by Theorem 1 in Section 4.2, <S is a constant.

Now. suppose that y = g{t) is a solution of the harmonic-oscillator equation

—- + w-v =
dt-

and let

S = [-yj + w-y- = [g'iDY + oj-[g(t)]-

By Theorem 3, S is a constant, independent of the value of t. In particular, we can


find t, by setting t = 0:

& = [g\Q)]- + io-[g(Q)]-

Therefore, if both ^(0) = and g'(Q) = 0, the energy of the oscillation is 0. Intui-

tively, this would mean that there is no oscillation. Theorem 4 confirms our intui-

tion.

THEOREM 4 The Case of Energy Zero

Suppose that y = gU) is a solution of the harmonic-oscillator equation

d'\
-+ io~\ =
dt-

Then, if ^i,'(0) = and g'{0) = 0, it follows that g(t) = for all values of i.

Since both g(0) = and g'(0) = 0, we have

S= |,i,''(0)l- + a^-|i'(0))- =

Because ^ is a constant (Theorem 3), it follows that

0= ig'U)]- + u}~lg{t)f

holds for all values of/. Because neither [g'(f)]^ nor (o-[g(t)]- can be negative, both
must be for all values of /. In particular,

^'(/) = for all values of i


SECTION 4.6 THE HARMONIC-OSCILLATOR EQUATION 285

Hence, by Theorem 1 in Section 4.2, g is a constant function. Therefore,

git) = g(0) = for all values of /

According to Theorem I, y =A cos (o)t — </>) is a solution of the harmonic-


oscillator equation

In Theorem 5 below, we show that y =A cos (wr - (f>) is, in fact, the complete
solution of this equation; that is, all possible solutions can be obtained by assigning
suitable values to the constants A and </>. To begin with, suppose that/ is a differen-
tiable function and that w t^ 0. Consider the point

>' = {m,m
Figure 2 in the xy plane (Figure 2), let

A = \dP\= JlfiO)f +
n]
and take <j) to be the radian measure of the smallest positive angle from the positive x
axis to the line segment OP. [If P= (0, 0), just take </> = 0.] Then, if A # 0,

/(O) f'(Q)lw
cos (/>
= and sin <f)
=

so that even if A =0, we have


A cos </) = /(O) and (i}A sin ^ =/'(0)

THEOREM 5 Complete Solution of the Harmonic-Oscillator Equation


286 C HAriER 4 ANTIDIFFERENTIATION AND DIKKKRENTIAI. EQUATIONS

Now
^(0) =/(0) -A cos (-0) =/(0) - A cos<t> =
and

g'{t) =f'U) + loA sin {tot - (f))

so

^'(0) =/'(0) + cjA sin (-(^) =/'(0) - wA sin <}> =


Therefore, by Theorem 4.

g(t) = for all values of /

Consequently,

J\t) = A cos (u)t - (f)) for all values of t

EXAMPLE 1 Solve the differential equation

Figure 3
SECTION 4.6 THE HARMONIC-OSCILLATOR EQUATION 287

Naturally, the mass bobs up and down between Aq and -Aq. At any time /, the
velocity of m is given by

dt

and its acceleration is

_ dv _ d-y
"~ dt
~ ~dt^

By Hooke's law (see page 276). the unbalanced force F exerted on the mass m by
the spring is given by

F = -ky
(The minus sign is present because when y is negative, the spring is pulling up on
the mass.) According to Newton's (second) law of motion — force equals mass
times acceleration — we have
F = ma
It follows that

it
ma = —kv or a -\
v =
m
In other words, the vibrating mass in Figure 4 satisfies the differential equation

^-v k
-pr + -y =
dt- m
This is just a special case of the harmonic-oscillator equation

— -- + (D-\ =
dt-

with
.
J" = —k or (o = Ik

EXAMPLE 2 For the mass-spring system in Figure 4. find (a) the equation of
motion and (b) the frequency of oscillation.

SOLUTION In Theorem 5 we take w = vk/m and seek a solution y = f(t) satis-


fying the initial conditions

/(O) = An and /'(O) = Vq =


Thus, in Figure 2 we have

/'(O)
P = (...
i/(0), j
= (Ao, 0)

and it follows that

A = V(Ao)^ + 0- = Ao and 4) =
288 CHAP1ER 4 ANTIDIFFERENTIATION AND DIFFERENTIAL EQUATIONS

Therefore, /(/) = A cos (wt - (f))


= A() cos J— t

Hence:

(a) The equation of motion is

y = A() cos ,/
V
—m /

(b) The frequency of oscillation is given by

CO 1 fk
Itt 2tt V m

Problem Set 4.6

In Problems 1 to 6, solve the differential equation with the indicated 15 r = I second and Aq = 0.8 meter
initial- value conditions.
16 m = 0.5 kilogram; when m is suspended from the spring, it

stretches 0.2 meter. (Hint: A mass of m kilograms weighs mg


= -^
1 4t
di^
+ 9v 0: V = and
(it
3 when t = newtons, where g is the acceleration of gravity.)

d-y 17 For a mass-spring system (Figure 4), the potential energy stored
2 —pr + 2v = 0; V = 2^3 and -^ = 2V2 when / =
dr dt in the spring is given by iky', and the kinetic energy of the mass
m is given by \m(dy/dt)'. The total energy E is the sum of the
3 f"(i) + 3/(/) = 0;/(0) = - 1 and/'(0) = potential and kinetic energies. If i', is the quantity defined in
Theorem 3, find a constant K such that E = KS.
4 —d'\
-V + 5v = 0: 'V
dv
= -4 and -^ = -4V5 when
/—
jr =
dx' ' dx 18 Consider a simple pendulum of length / (Figure 5). When the

5 D]y + = = and D,y = when = bob of mass m is displaced 6 radians from its equilibrium posi-
v 0; v I /
tion O, the restoring force F is the component of its weight mg in
El 6 y" + 4y = 0; y =4 and y' =6 when a: = the direction tangent to the circular arc s\ hence.

7 In Problems I and 3, find (a) the frequency v of the oscillation d^s


and (b) the period T of the oscillation.
—mg sin 6
dr
8 Account for the general features of the graph in Figure 1. d~s
-I- g sin =
dt-

O In Problems 9 to 12, sketch the graph of a quantity y oscillating in a


simple harmonic manner and satisfying the conditions given.
Figure 5

9 /I = 1 , i^ = 50 hertz. 4> = tt/I

10 A = 5, V = ——cycle/day, <i>
= -377/14
28

11 A = 220, v = 60 hertz, </> =

12 A = 5, 1/ = 88.5 megahertz, <^ = 0.8

ID In Problems 13 to 16, find (a) the equation of motion of the mass-


spring system and (b) its frequency by using the information given.

13 /r = 19.6 newtons per meter, m= 0.1 kilogram

14 A force of 40 newtons stretches the spring 0.5 meter, and m = 1


kilograms.
SECTION 4.6 THE HARMONIC-OSCILLATOR EQUATION 289

If the pendulum is swinging through a small arc, then 21 In Problem 19, the electrostatic energy stored in the capacitor at

any given time is given by Q-/(2C). and the magnetic energy


sin „
= 1 so sm ~ 6 stored in the inductor is given by LI'/l. The total energy E
e stored in the LC-circuit is sum of these two energies. If S is
the
and we can write (approximately) the quantity defined in Theorem 3, find a constant K such that
E = KS.
d-s
+ ^9 =
dr 22 By Theorem 1, if fi and C are any two constants, then B cos t

(a) Using the fact that s = 16, show that the last differential
and C sin t are solutions of the harmonic-oscillator equation

(d'y/dt') + (o-y — 0. By the superposition theorem (Theo-


equation can be rewritten in the form of the harmonic-oscillator
equation, (b) Assuming that the pendulum is swinging through a rem 2), B cos f -t- C sin / is also a solution of this equation.
Therefore, by Theorem 5. there must exist constants \ and <^
small arc, find a formula for the (approximate) period of its
such that S cos f -I- C sin / =A cos (f — <i>) holds for all values
oscillation.
of t. Find A and <i> in terms of S and C.
19 An electric circuit consisting of a coil with inductance L henries
23 Prove part (ii) of Theorem 2.
and a capacitor with a capacitance of C farads is called an LC-
circuit (Figure 6). If the capacitor charged with Qo coulombs
is 24 If F{t) is a given function, then the differential equation
and the switch is closed, a current will begin to flow in the (d-yjdt-) + ory = Fit) is called the inhomogeneous harmonic-
circuit and the capacitor will begin to discharge. If Q is the oscillator equation with driving function Fit). Suppose that
charge in coulombs on the capacitor at time t. then the voltage yp=fp(t) is any particular solution of this inhomogeneous
drop across the capacitor at this time is given by Q/C. The cur- equation. Show that y =A cos (wt - 4>) + fpd) is also a solu-
rent / in amperes flowing in the circuit at time I is given by tion of the inhomogeneous equation for any value of the con-
/ = dQ/dt, and the voltage drop across the inductor is given by stants A and </>.

L(dl/dt). From KirchhofTs (first) law, we have


25 If S is a constant, find a solution of the second-order differential
dl Q equation id-y/dt-) + coy = B.
dt C
26 In Problem 24, show that y = A cos (wf - (f>) + fp{t) is the
complete solution of the inhomogeneous harmonic-oscillator
equation.
Figure 6
27 In Figure 7, suppose that the point P is moving counterclockwise
around a circle of radius A with a constant angular speed of co

radians per second. Let y be the coordinate of the projection of P


on a vertical y axis with the origin at the level of the center of the
circle. Show that y undergoes simple harmonic motion with an-

gular frequency o).

Figure 7 y

(a) Show that the current in the LC-circuit satisfies the har-

monic-oscillator equation

d-l
+ lo'l = with u)
dr
(b) Show that the charge Q on the capacitor satisfies the har-

monic-oscillator equation with the same (o as in part (a), (c)

Assuming that Q = 2o and / = when r = 0, write an equation


for Q at time I. (d) Using the initial conditions in part (c), write
an equation for / at time t. (e) Find a formula for the frequency c
with which the LC-circuit oscillates.
28 If B and oi are constants, find the complete solution of the inho-
20 An astronaut on the surface of the moon finds that a certain
mogeneous harmonic-oscillator equation (d'y/dt') + (xi'y = B.
pendulum has a period 2.45 times as long as it does on the
surface of the earth. From this information, she is able to calcu- 29 In Problem 27, explain the geometric meaning of the phase
late the acceleration of gravity on the moon. How does she do angle </> for the simple harmonic oscillation of y. Assume the
this, and what is her answer? (See Problem 18.) initial condition d = do when r = (Figure 7).
290 CHAFIKR 4 ANTIDIKKERENTIATION AND DIFFERENTIAL EQUATIONS

4.7 Areas by the Method of Slicing


Figure I One of the most important applications of antidifferentiation is finding the areas of
regions in the plane. If finite number of
such a region has a boundary consisting of a
line segments, it number of nonoverlapping triangles
can be subdivided into a finite

(Figure la), and its area can be found simply by summing up the areas of these
triangles. However, if the region is bounded by curves (Figure lb), it may not be
immediately clear how to calculate its area. One clever method is to cut out the
region, weigh it with a chemist's analytical balance, and divide its weight by the
weight of 1 square unit of the same paper.
In this section, we present a method for calculating areas that is simple, can give
exact results, and does not require access to an analytical balance. Here we develop
this method, called the method of slicing, on the basis of Leibniz's idea that differ-
(?
entials represent infinitesimals. This will set the stage for the more rigorous and
formal treatment of areas given in Chapter 5.

To find the area of a region in the plane by the method of slicing, begin by
choosing or setting up a convenient reference axis. This could be the .v axis, the v
axis, or another number scale of your own choosing. For the sake of generality, let's

call it the s axis (Figure 2a). At each point along this axis, construct a perpendicular
line intersecting the region in a line segment of length /. Notice that / is a function of
s. Suppose that the entire region lies between the perpendiculars al s = a and at

s = b, where a < b.

Figure 2

Now let A denote the area of that portion of the region between the perpendiculars
at a and at s (Figure 2a). Notice that A is a function of i and A = when s = a. If

s is increased by an infinitesimal amount ds. then A increases by a corresponding


infinitesimal amount dA (Figure 2b). Apparently, clA is virtually the area of a small
rectangle of length / and width els; that is.

dA = / ds

Therefore, A can be obtained as a function o\' s by solving this differential equation


with the initial condition

A = when s = a

The value of A when s = b is the desired area of the region.

I
SECTION 4.7 AREAS BY THE METHOD OF SLICING 291

Figure 3 In Examples 1 to 4. find the area of the region by the method of slicing.

EXAMPLE 1 The region in the first quadrant of the .rv plane bounded above by the
parabola v = .v". below by the x axis, and on the right by the vertical line x =
I.

SOLUTION Figure 3 shows the region in question. Let's take the x axis as the
axis of reference. Then the differential equation for the area A is

dA = Idx
where, by Figure 3,

/ = V

Thus, A= \
I dx= \ x- dx = ^—^C
Since A = when .v = 0. we have C = and

A =

Hence, when x = 1,

square unit

Figure 4

Figure 5
292 CHAPTER 4 ANTIDIFFERENTIATION AND DIFFERENTIAL EQUATIONS

and therefore

A-- I I d.x= \ (Vx - .r') ±x=\ x^'- dx- \ x^ dx

= —x^'~ + C
3 4

When .V = 0. A=0. and so C= 0. When ;c = 1,

A = i(l)'^^" -1=12 square unit

Figure 6 EXAMPLE 4 A triangle with base 5 meters and height 8 meters

SOLUTION In Figure 6, we take the reference s axis perpendicular to the base of


the triangle with the origin at the level of the base, so that dA = I ds. From Figure 6
and similar triangles,
/
= —
5
hence / = —
5
(8 - s)
8 -s 8 8

The differential equation dA = I ds can now be solved to obtain

A = I I ds = \ —(8 - s)ds = 5 \ ds i ^5- = 55 s" + C


J

Since A = when s = 0. we have C = 0; hence,

A = 5s- ^s^
The area of the entire triangle is obtained by putting 5 = 8 and calculating

A = (5)(8) - (-re)(8)- = 20 square meters

(This, of course, corresponds to the result obtained by the usual formula of one-half
the height times the base.)

If/ is a function and fix) > for a s .v < fe, then the region bounded by the
graph of V =/(-v), the a axis, and the vertical lines .v = a and x = b is called the
region under the graph of/ between v = a and x = b (Figure 7a). You can find
the area of such a region by using the method of slicing with the .v axis as the
reference axis and with / = y = f{x) (Figure 7b). Thus, in Figure 7b, we have
dA = I dx=f(x) dx
and so

fix) dx

Figure 7
y=Ax)
SECTION 4.7 AREAS BY THE METHOD OF SLICING 293

where the constant of integration is determined by the condition

A = when .v = a

Then, the area of the region is the value of A when x = b.

Figure 8 EXAMPLE 5 Find the area of the region under the graph of y = x^ — 2x + S be-
= —2 and x = 4 (Figure 8).
tween X

SOLUTION Since the graph lies above the .v axis between .v = -2 and x = 4, we
can use the method suggested above. We have

A =
r
(x^
,
-2x+^) dx = ^ x^
.
+ 8x + C
J

Since A = when .v = -2, we have

(-2)-^ 68
= - (-2)' + 8(-2) + C = --- + C

Therefore, C=^ and


68
A = .r + 8.r +
3 3

Thus, when .v = 4, we have

— 4- + 8(4) + —- = 60 square units

Remarks on the Method of Slicing and the Idea of Area


Ahhough our '
"derivation" ' of the differential equation

dA = I ds

for area (Figure 2) might make it seem as if an approximation is involved, neverthe-


less, it is possible to prove that the differential equation holds exactly, and, with the
appropriate choice of the constant of integration,

A= \ Ids

gives the area exactly. Also, it can be shown that the reference axis can be chosen
arbitrarily — the answer will always be the same. Of course, skillful selection of the
reference axis may simplify the actual details of the calculation.
The proof of these facts requires a formal definition of area. In spite of the fact
that the intuitive idea of area seems reasonably clear, it is surprisingly difficult to
define it formally. One has to begin by making clear just what is meant by a region

in the plane and then define exactly what is meant by the area of such a region. To
do this in general by modem standards of rigor is beyond the scope of most calculus
textbooks, including this one. However, in Chapter 5 we make a start in this direc-
tion by studying the special case of the area under the graph of a function by using
the important idea of a definite integral. In Section 5.6 we show how to use the
definite integral to find more general areas by the method of slicing.
294 CHAPTER 4 ANTIDIFFERENTIATION AND DIFFERENTIAL EQUATIONS

Problem Set 4.7

In Problems 1 to 8. sketch the indicated region and use the method


of slicing to find its area.

1 The region in the first quadrant of the vy plane bounded above by reference

the parabola y = v". below by the v axis, and on the right by the axis

vertical line .v = 2

2 The region bounded above by the graph of y = -.v~ + 3.r — 2


and below by the .v axis

3 The region bounded below by the parabola y = I + 2r and


above by the horizontal line y = 9

4 The region between the curve y = 4 - .ir and the v axis

5 The region in the first quadrant of the xy plane bounded above by


y = 2v', below by the x axis, and on the right by the vertical line

x = 2

14 Calculate the area of the shaded region in Figure 9 by the


6 The region bounded above by one arch of the sine curve
= sin X and bounded below by the axis
method of slicing in two ways: (a) using the .r axis as the refer-
y ,v

ence axis; (b) using the y axis as the reference axis.


7 The region bounded below by one branch of the curve
y = sec' X and bounded above by the horizontal line y = 2
Figure 9
8 The region bounded by y = .v\ the x axis, and the vertical lines

X = —2 and x = 2

In Problems 9 to 13, find the area of the shaded region in the figure

by the method of slicing, using the indicated axis as the reference

axis. In each case, compare your answer with the area obtained by
using elementary geometry.

reference
axis

In Problems 15 to 28. calculate the area of the given region by the


method of slicing using the given axis as the reference axis. Sketch
the region in the \y plane.

15 The region bounded above by y = V.x - 2, on the left by .v = 2,

on the right by .v = 6, and below by y 0. Take the x axis as the


reference reference axis.
axis
16 Same as Problem 15, but take the y axis as the reference axis.

17 The region between y" = x and y - .v + 2 = 0. Take the y axis


as the reference axis.

18 The region between y = x^ - 6x + & and y -.r + 4x - 3.

Take the x axis as the reference axis.

19 The triangular region bounded by the lines y + 2.r — 2 = 0,

V - .V - 5 = 0, and v = 7. Take the v axis as the reference axis.

20 The region between .v = y - 4 and x V' . Take the v axis


as the reference axis.

I
REVIEW PROBLEM SET 295

21 The region between y- = 1 - v and v = x + 5. Take the y axis 25 The region between y = r and x = y". Take the x axis as the
as the reference axis. reference axis.

22 The region under the curve y = j: v25--? between x = -5 26 The region between y = x, y = 2 - x, and y = 0. Take the y
and X = 0. Take the x axis as the reference axis. axis as the reference axis.

23 The region between y = 4 - .ir and y = .r - 2. Take the x axis 27 The region between y = 4 - .r and y = -2. Take the .r axis as
as the reference axis. the reference axis.

24 The region between \' = .ir and y = 2v. Take the x axis as the 28 The region between y = .v'* + 1 and y = 17. Take the y axis as
reference axis. the reference axis.

Review Problem Set, Chapter 4


In Problems 1 to 6,
296 CHAPIER 4 ANTIDIFFERENTIATION AND DIFFERENTIAL EQUATIONS

] 30 Suppose thai the mathematical model In Problems 61 to 76, find the general solution of each differential
equation.
p = (1.013 X 10') - 115 sin (1.24/)

is used to represent the air pressure p in newtons per square dy


meter t seconds after inspiration begins in the lungs of a human ''^
volunteer during a study of the respiratory elTecls of air pollu-
tion. Using differentials, find a formula for the approximate
change dp in lung air pressure during the first Ar seconds after
inspiration begins, assuming that A/ is small.

In Problems .11 to 60, evaluate each antiderivative.

31 Cix* + 4.V- + \\)dx 32 I (4.r'-(-3.v--.v + 91)(iv

33 3i\/7di 34 (1 +2t)^ dt
J

35 V'3/ + 9dt 36 jrix^ - \f^dx


J

37 .r(x' + 8)'^ dx 38
J
4^ + 4) '/-'atx

x^ dx f (VJ - 3)-" dx
39 40
A>7TT3 v:^

f 3i di
41 x\/l + X dx 42

r X* dx
43 x^y/x^ + 1 dx 44

45 (3 cos x - 2 sin x) dx

46 sec Zx{2 tan 2x + sec 2v) dx

47 2 sin Ix dx 48 u sin 3i(- du

49 (2-3 cos 4r) dt 50 .v cos (4.r - \) dx


J

51 x sec .r tan .r dx 52 Vtan 6 sec" d6

53 (2 + 3 cot B)^^- CSC-


,/-,•>
fidp 54
J
r
—sm"^r- ^
cos .V

.V

cos V dv
55 56 CSC" (sin x) cos .r rfjc

(1 - sin v)"

sec 3x tan 3x
57
(sec 3.V + 8)' J V5 + cot u

+ b cos
a .t ,„ f i/'
59 r
,

fi*; 0" ~ ' 7


(ax + b Sin xf J sm /(sin r + cos /)
REVIEW PROBLEM SET 297

85 The slope of the tangent line to the graph o f y = fjx) at the (a) If a* < a. show that the two particles will eventually col-
point with abscissa .v is given by dy/dx = x\'.\~ + 5. Find the lide, (b) If a* = a, show that the two particles will eventually
function /if its graph contains the point (2. 6). collide if and only if vg < vq. (c) If a* > a. show that the two
particles will eventually collide if and only if vq < vq and
86 The slope of the tangent line to the graph of the function/at the
— -
2{a* a)k < (ij vo)'. (Him: Let s and i* denote the
point (.v,/(.v)) is x csc~ .r. Find the function/if its graph con-
P and P*, respectively, at time F^it
position coordinates of i.
tains the point (vtt/2. 3).
y = — s, and show that y satisfies the differential equation
s*
87 (a) Show that there is no function / satisfying the differential d^yldf' = a* — a with the initial conditions that y = k
equation f'(x) = 3.r + I such that /(O) = and /(I) = 3. and dyjdt = vq — vo when / = 0. Collision occurs if y =
(b) Show that there is a function / satisfying the differential for some positive value of t.)

equation/"U) = 3.r + 1 such that/(0) = and /(I) = 3. Ex-


99 A 2000-newton weight is suspended 50 meters below a
plain, contrasting part (a) with part (b).
windlass by a cable weighing 10 newtons per meter. Neglect-
88 Solve the differential equation dy/dx = |.v| -f |.v - 1 1 -I- |.v - 2| ing friction, determine how many joules of work will be re-

subject to the initial-value condition that v = 1 when v = 0. quired to lift the weight through the 50 meters.
(Hint: / dx = x\x\/l + C.)
|.r|
100 Let W be the work done in charging a capacitor having (con-

In Problems 89 to 92, the force F in newtons acting on a particle P stant) capacitance C with Q coulombs. If £ is the voltage drop

moving along the i axis is given in terms of the coordinate ^ of P. across the capacitor and / is the current in amperes flowing

Find the work (in joules) done by F in moving P from i = io to


into the capacitor, then dW = E dQ, Q = CE. ax\A dQ = 1 dt.

s = s\ meters.
where t denotes time in seconds, (a) Show that the instantane-

ous power dW/dt required to charge the capacitor is given by


89 F = 3i - 1, .So = i -Si = 6 dW/dt = EL (b) Assuming that W= when £ = 0, prove that
W = iCE-.
90 F= cos Is, sq = 0, ji = 7r/4
101 Suppose that m kilograms of mass are distributed uniformly

F= = = along the s axis between s = a and s = b meters, where


91 sec- ^- -^0 0, .?, 27r/3
< a < b. An M-kilogram particle P is placed on the s axis at

= = the origin. Find the net gravitational force exerted on P by the


92 F= V'l -h V7, 5o 0. 5, 1

distributed mass.

93 A perfectly elastic spring is compressed 6 inches from its natu- ! 102 In the study of the transport of oxygen (Ot) by the blood from
ral length because a weight of 2 tons is placed on it. How much the lungs to the tissues of the body, it has been found that the
work (in foot-pounds) is done? conversion of hemoglobin to oxyhemoglobin is governed by a
differential equation of the form 1 -I- kx")~ dy — nLx"~ '
dx =
94 What constant negative acceleration is required to bring a train
(

0, where x is the partial pressure in newtons per square meter of


to rest in 500 meters if it is initially going at 44 meters per
O2 dissolved in the plasma and y is the ratio of the amount of
second?
O2 combined with hemoglobin to the maximum amount of Oi
13 95 An athlete running the 100-meter dash maintains a constant that could combine with the hemoglobin. For normal human
acceleration for the first 1 5 meters and thereafter has zero ac- blood, the constants n and k are given approximately by n =
celeration. What must the acceleration be if the athlete is to run 2.7 and k = 2.4 x 10"'°. (a) Using the initial-value condition
the race in 10 seconds? that y = when x = 0, solve the differential equation for arbi-
trary positive values of the constants n and k. (b) Sketch a
E] 96 A certain type of motorcycle can be brought to a stop from a
graph of y as a function of x for normal human blood,
speed of 72 kilometers per hour in 4 seconds. How long will it
s X s 1.5 X 10" newtons per square meter, (c) Find the co-
take to bring it to a stop from a speed of 96 kilometers per
ordinates of the point of inflection of the graph in part (b).
hour? Assume the same constant deceleration in both cases.

97 A stone thrown down with an velocity of 96


103 A company manufacturing jogging shorts expects a marginal
is straight initial
rewtnue of dR/dx - (.v/12, 500) dollars per pair manufac-
= 10
feet per second from a bridge 256 feet above a river, (a) How
tured. Assume that R = when x = 0. The marginal cost is
many seconds elapse before the stone hits the water? (b) What
dC/dx = 6 dollars per pair manufactured, and C = $400 when
will be the velocity of the stone when it strikes the water? (Take
= X = 0. (a) Find the total revenue expected if .v pairs of jogging
g 32 feet per second squared.)
shorts are manufactured, (b) Find the total cost of manufactur-
98 Two particles P and P* are moving on the s axis with constant ing X pairs of jogging shorts, (c) How many pairs should be
accelerations a and a*, respectively. When = / 0, particle P* is manufactured to maximize the total revenue? (d) How many
k units to the right of particle P. where k > 0; particle P is pairs should be manufactured to maximize the profit? (e) How
moving with a velocity vq. and particle P* is moving with a much does the company charge per pair when the profit is max-
'
velocity vq. The question is, "Will the two particles collide? imized?
298 CHAPTER 4 ANTIDIFFERENTIATION AND DIFFERENTIAL EQUATIONS

104 The expected revenue for the sale of x sweaters is given by with coordinate y = 0.2 meter and is released at time t =
with initial velocity dy/di = 0. Find (a) the equation of motion
/?=.v(27-—^) dollars of the mass-spring system and (b) its frequency.
\ 1000/
while the marginal cost is given by dC/dx = 700/Vt dollars In Problems 111 to 120, sketch the region and use the method of
per sweater. When x = O, then C= $500. (a) Find the cost as a slicing to find its area.
function of .v. (b) Show that the maximum profit is achieved by
selling 10,000 sweaters, (c) What would be the price per 111 The region in the first quadrant of the xy plane bounded above
sweater if 10.000 sweaters were .sold'.' by the parabola y = x^ + 1 below by the .v axis, on the left by
,

the y axis, and on the right by the line .v = 3. Use the x axis as
In Problems 105 to 108, solve the differential equation with the the reference axis.
indicated initial-value conditions.
112 The region in the first quadrant of the xy plane bounded above

d'y by the graph of y = \/x and to the right by the line .v = 4. Use
105 + 25v = 0; V and -7- = 5 when t = the .r axis as the reference axis.
df dt

1 13 The region bounded above by y = 1 , on the left by y = x^, and


106 —^ + 7v = 0; 'V = and -f- = V? when x = on the right by y = x. Use the y axis as the reference axis.
dx- dx
1 14 The region under the graph of y = 1
— |.v| between .r = - and
1
107 v" + 36.V = 0; v = iVl and v' = 12 when =
.r
jc = 1. Use the y axis as the reference axis.

E 108 D;s + 9i- = 0; i = 9 and D,s = 11 when / =


115 The region bounded above by y = 4x — x- and below by the x
El 109 A car with faulty shock absorbers is approximately a mass- axis.

spring system, and so it vibrates more or less in a simple har-


116 The region bounded below by one branch of y = csc" .v and
monic manner. Suppose that the equation for the displacement
above by y = 2.
y meters of the body of such a car is {Syjdr) lOOy 0, -I- =
where >> = 0.02 meter and dyjdt = 0.24 meter per second 117 The region between the x axis and one complete cycle of
when t = seconds. Find (a) a formula for the displacement v
meters of the car body at time t seconds, (b) the frequency of
118 The region between y = x* and y = xr.
the vibration, and (c) the period of the vibration.

119 The region in the second quadrant under the graph of


ElIO A mass of 0.5 kilogram is attached to a spring with constant
y = V'.v + 1 Use the .v axis as the reference axis.
= 4.5 newtons per meter. A vertical y axis is set up so that
.

k
when the mass and spring are hanging in equilibrium, the y 120 Same region as in Problem 119, but use the y axis as the refer-
coordinate of the mass is zero. The mass is lifted to the position ence axis.
THE DEFINITE INTEGRAL

In Chapter 4, we introduced the antiderivative, or indefinite integral, and indicated


how it could be used to help solve differential equations. In particular, we showed
how to use the indefinite integral to calculate areas by the method of slicing. There
our study of area was preliminary and exploratory because it was based on the

intuitive idea that a differential can be considered to be an infinitesimal.


In the present chapter, we introduce, by means of a formal definition, the impor-
tant idea of a definite integral, and we show how to use the definite integral to

calculate area. A profound connection between definite and indefinite integrals


Ihc fundamental theorem of calculus — is proved in Section 5.4. Finally, in Sec-
tion 5.6, we indicate how the definite integral and the fundamental theorem of
calculus can be used to provide a more formal justification for the method of slicing.

5.1 The Sigma Notation for Sums


The formal definition of the definite integral involves the sum of many terms and
thus calls for some special notation. In mathematical symbolism, the capital Greek
letter sigtna, which is written

1
and corresponds to the letter S, stands for the words

"the sum of all terms of the form ..."

For instance, rather than writing 1+2 + 3-I-4-I-5-I-6, we can write 2^ k, and
1^ -f 2^ -I-
3*^ + 4' -I- 5-^ + 6" can be written as Zj k~. provided that we make the
convention that k is to run through all integer values from 1 to 6. If we wish to

299
300 C HAPTER 5 THK DEFINITE INTEGRAL

include the range of values of A: as part of the sigma notation, we write, for instance,

2 or X or X
Thus, 2*= I
^' means "the sum of all terms of the fomi k~ as k runs through the
integers from 1 to 6."

In Examples 1 to 3, write out the given sum explicitly and then find its numerical
value.

EXAMPLE I Ik^-

SOLUTION S k- = 1- + 2- + 3- + 4- + 5- + 6- + 7-

= 1 + 4 + 9 + 16 + 25 + 36 + 49 = 140

EXAMPLE 2 2 2*

SOLUTION X 2* = 2' + 2- + 2' + 2^ = 2 + 4 + 8 + 16 = 30

EXAMPLE 3 2 (3* -3*"')

SOLUTION

X (3* - 3*-') = (3' - 3") + (3' - 3') + (3-' - 3") + • •


<.= i

+(3""' - 3""-) + (3" - 3""')

If we rewrite this sum in the form

2 (3* - 3*-') = -3" + (3' - 3') + (3- - 3-) + • • • + (3""' - 3"^') + 3"

then the terms within the parentheses cancel, and we have

E (3* - 3*"') = -3" + 3" = 3" - 1

In Examples 1 to 3 above, the variable k. which runs from 1 to 7 in ^^= i


k'. from
to 4 in X*=i 2*. and from 1 to n in Xa'=i <3* - 3*"'), is called the summation
SECTION 5.1 THE SIGNU NOTATION FOR SIMS 301

index. There is no particular reason to use k for the summation index — any letter of
the alphabet will do. For instance.

4 4 4

E 2* = 2 2' = E 2^ = 2' + 2- + 2-' + 2'*

*=i 1=1 j=i

Note that if/ is a function and if the integers from 1 to n belong to the domain of
/. then

5 Ak) =/(l) +/(2) +/(3) + • • •


+/(/!)

For instance, if / is a constant function, say fix) = C for all values of .v. then

2 fik) =/(l) +/(2) +/(3) + • • •


+f{n) = C^C-r + C = nC
i=I '
V

n terms

The fact expressed by this equation is often written simply as

2 C = nC
k=l
Thus.
^ 1 100

2 5 = (7)(5) = 35 2(-2)=-2« S 1 = 100


'=1 t=i ,= 1

and so forth.
The Sigma notation handy for indicating the sum of the terms of a
is especially
sequence of numbers. A
sequence of numbers consisting of a first number a, a ,

second number a2. a third number a,, and so on, is written as ai. 02, ot.
Thus, the Ath term in this sequence is a^. and the sum of. say. the first six terms can
be written
6

Zj fl* = fli + Ot + a? + ^4 + as + fle

Similarly, the sum of the second, third, and fourth terms of the sequence can be
written
4

Zj Clk = ^2 + ^3 + ^4
k=2

Sometimes,
in dealing with sequences, it is convenient to start with a zeroth term

flo-For such a sequence, the sum of the terms a^. with k running from to n. can
be expressed as

Zj a^, = Qq + ai + a2 + • • • + a„

Some basic properties of summation, which are easy to establish either by inspec-
tion or by using the principle of mathematical induction, are as follows:
302 CIIAKIKR 5 THE DEFINITE INTEGRAL

Basic Properties of Summation


SECTION 5.1 THE SKJMA NOTATION FOR SUMS 303

which is obviously true. Now, we assume that (i) holds for a particular value of w,
and try to show that it holds for the next value, /; + 1. Thus, we assume that

n{n + 1)
I^ =
Adding /; + 1 to both sides of the last equation we find that

2j k + (n + 1)
= + (/z + 1)
«.= i
2

^ "v n(n + I) 2(« + 1) «(« + 1) + 2(/; + 1)


.

that IS. A, k
,
= 1 =
->
^ 1 ->

k=\

Therefore, we have /.-i


n
y (/I + ])[{ii +!)+!]
i,, .

and our proof by induction of formula (i) is complete.

In Examples 5 to 9, use the basic properties of summation and Theorem 1 to


evaluate each sum.

20

EXAMPLE 5 Z (2k- - 3k + 1)

20 20 20 20

SOLUTION 2 (2A- - 3A- + 1) = 2 2k- + S (-3/t) + X 1 (Property 3)

20 20
= 2 X A- - 3 2 A- + 20 (Properties 2 and 1)
k=[ k=\

(20)(21)(41)
= 2 3-y- + 20(20)(21)

= 5130 [Theorem 1 (ii) and (i)]

EXAMPLE 6 2 k\5k + 1)

SOLUTION S k-{5k + 1) = 2 (5k^ + k-) = 5 E A-' + S k-


k=l k=] k=l k=[

5 , , /((/( + l)(2n + I)
= —/)-(/( + 1)- +
4 6

r5«(/i +1) 2/; + 1 1


304 I HAPTKR 5 IHK DKHNH K INIK(;RAI.

EXAMPLK 7 ' +
Ill
y + T * IT + + "^
1

+ — + — + •••+
1 1

^=ii=i(ir
1

2"
_
", „, 2'
,

4 8 - ,_„ V 2

" ITheorem 1 (iv)]


1 - i 2"

EXAMPLE 8 The sum of the first 100 odd positive integers

SOLUTION As k runs through the integers from 1 to 100, 2k - 1 runs through


the first 100 odd positive integers. Thus.

^" r-^' \ i^' [100(101)1


Z (Ik - \) = 2[2l k) - 2^ 1 = 2[
J
- 100 = 10.000

EXAMPLE 9 2 c* C#0, 1

SOLUTION Formula (iv) in Theorem 1 cannot be used directly here because k


starts with the value 1 rather than 0. However, by "splitting off the zeroth term'" in
part (iv) of Theorem 1. we see that

-C""'
C'=^ C
1

C'^+L
1 -c
Therefore.
SECTION 5.1 THE SIGMA NOTATION FOR SUMS 305

Figure I [k/n)-. and its area is {\/n)(k/n)-. An estimate of the area /I

can now be obtained by adding the areas of the n circum-


scribed rectangles:

V 1 /A:

,,_ , n ^ n

Now w is a fixed number (a constant) in the summation, so


we can use Property 2, page 302, to write

S -(-) = 2 -^k'- = \t. k'-

So, by part (ii) of Theorem 1. we have

n^ ^^ «n 6 J

(n + 1)(2/? + 1) _ 2n- + 3w + 1

A = — + — + —-
1 1 1

3 2n 6n-

Moreover, since the approximating rectangles are circum-


scribed,

A s — + — + —-
1 1 1

3 2n 6m"

In Figure lb, we again subdivided the interval [0, 1] into n


subintervals of equal length, but we formed the inscribed
rather than the circumscribed rectangles. Evidently, the
height of the Ath inscribed rectangle is [{k - 1 )/«]-, and its

area is {l/n)[(k - l)/n]-. Again, we obtain an estimate of the


area A by adding the areas of the n inscribed rectangles:

1 / k- \ \-
1

1 / A- - 1
Now, 2j 2 (— ) (k -\)- = [—)
"
E (k- - 2A- + 1)
k=l " k=[

1
I
n(n + \){2n + 1) 2n{n + 1)

.M

A =
306 C HAnKR 5 IHK DKFINITE INTEGRAL

Moreover, since the approximating rectangles are inscribed,

I 1 1

+ 7</l
3 2« 6/r

The above considerations show that for every positive integer n,

11 +
1

T^A < —
11
+ +
1

3 2/1 6«- 3 2/1 6//-

As n becomes larger and larger, both

111—
3 2/!
+
6/1
- and
111—
—3 + 2n
+
6/r
r

approach s as a limit. Since the constant number A is "'trapped"' between two


quantities, both of which can be made to come as close to 3 as we please, it follows
from the squeezing property for limits* (page 63) that A = 3.

*Actually, the squeezing property applies to limits as a contiiuwus variable v approaches a


real number u — and here we have limits as the integer-valued variable n becomes larger and
larger without bound. However, the basic principle is the same.

Problem Set 5.1

III Problems 1 to 8. write out each sum explicitly and then find its In Problems 1 1 to 20, use the basic properties of summation and
numerical value. Theorem I to evaluate the sum.

1 2 (2A- + I) 2 2 7/t- 11 2 (2* + 3) 12 2 /(/- I)


1=1

4 100

3 2v sin — 13 SS* 14 S (5*^' -5')


^Sj^
6 ""^ /I I \

5 1
,=2 kik- 1)
6 E 3' 15 ^ A(A- + I

7S-:t
r + 3
8 1 k + 2
17 S A-2 18 2 («i - "t-
,=n

In Problems 9 and 10. assume that ^[l a* = 23, 21- ''* = '^'^•
and 2^1, Q=
,

-14. Find the numerical value of the indicated


1

19 S ik - D- 20 E 10^
sum.

21 Using the method of circumscribed and inscribed rectangles,


find the area under the parabola y = .v" between x = I and
9 X (5«j + 3/^;

22 Using the method of circumscribed and inscribed rectangles,


10 2 i2a, - 3b, + 4(> - 5) find the area under the graph of y = .v' between .v = and
JC= I.
SECTION 5.2 THE DEFINITE INTEGRAL 307

23 Give an alternative proof of part (i) of Theorem 1 by completing 26 Use the principle of mathematical induction to prove part (iii) of
the following argument: Let 2_,l^ ^
k = S. Then Theorem 1

S=\+2 + 3 + --- + (n-l) + n 27 Use the result in Problem 25 to show that the sum of the first n
Writing the same sum we have odd positive integers is n". [Hint: Let fcj = k'.]
in the opposite order,

= 28 Use the principle of mathematical induction to prove part (iv) of


S /i + («-l) + --- + 3 + 2+l
Theorem 1

Adding the last two equations and combining corresponding


terms, we obtain 29 Use the result in Problem 27 to give an alternative proof of

part (i) of Theorem 1

2S = (n + 1) + (« + 1) + • • + (/7 + 1)

from which the desired formula follows. 30 Use the triangle inequality (Theorem 1. page 7) and the prin-
ciple of mathematical induction to prove the generalized triangle
24 Use the principle of mathematical induction to prove part (ii) of inequality:
Theorem 1

25 Prove the telescoping series property:

S (/'A
- bk-t] = h„ -bo 31 Give an altemative proof of part (iv) of Theorem 1 by complet-
ing the following argument: Let C# 0, 1 and put 2t=o C* = S.

(Hint: Notice that part of each term cancels with part of the next Then SC = 2^n C*""', so 5 - 5C = C" - C"*', and the de-
term.) sired formula follows from the last equation.

5.2 The Definite Integral

Figure 1 The method of circumscribed and inscribed rectangles, used in Section 5.1 to find
the area under the parabola y = .v" between x = and .v = 1 , is easily generalized.
Indeed, suppose that/is a continuous function such that/(.v) > fora < .v < b. and
we wish to find the area A under the graph of/between x = a and x = b (Figure 1 ).

By a partition of the interval [a. h\. we mean a sequence of n subintervals

[Xq. X|], [.V|, .v.], [.V2, .V3], [v„-,,.v„l

where .vq = a and .v„ = b (Figure 2). The subinterval (.Vi_|. x^ is called the kth
subinterval in the partition, and its length is denoted by

A-v, •Va Xk- 1

Thus, the length of the first subinterval is A.v; = jcj — aq, the length of the second
subinterval is A.vi = .Vt — Xi , and so on. There no requirement
is that all subinter-
vals in the partition have the same length. Denote the partition

[.Vo. .V|], [.V|, .V2], fc, .V3],

by the symbol fP.

Figure 2 -i-v«
308 CHAPTER S THE DEFINITE INTEGRAL

By the norm of the partition IP, in symbols

we mean numbers Ajti, Axt, Ajts,


the largest of the Ajt„ representing the . . . ,

lengths of the n subintervals in !P. Note that two or more of the numbers A-Ti A-Vi, ,

A.V3, .... Aj:„ may have the same largest value H.^H. In particular, if all the
subintervals in the partition [P happen to have the same length, then

= =
b- a
Ajci = A.V2 = Ajcj = Ajr„

b- a
so that

in this special case.


We now choose a number from each subinterval in the partition .7. Let C\ denote
the number chosen from the first subinterval [.Vo, .Vi], let C2 denote the number
chosen from the second subinterval [x\. .r^]. and so forth. Thus, q denotes the
number chosen from the Ath subinterval (Figure 3). The partition !P. together with
the chosen numbers C\, fi. • • • . c„, is called an augmented partition and is
denoted by ff*.
On each subinterval of the partition >'P, we now construct a rectangle as in Fig-
ure 4. Notice that the hh rectangle has the k\h subinterval [.v;._ 1 , .v^.] of length A-v^ as
its base and extends to the point (Ck,f(Ck}) on the graph of/. Since the height of the
kxh rectangle isfick), its area AA* is given by

Aytt=/(Q) A.V,

Figure 3 Figure 4

The sum

of the areas AA^ of the rectangles determined by the augmented partition if* is
called the Riemann sum corresponding to .'f * for the function/. This terminology is
used in honor of the German mathematician Bemhard Riemann (1826-1866), who
did some of the early definitive work on the problem of giving a precise mathemati-
cal formalization of the integral of Newton and Leibniz.
SECTION 5.2 THE DEFINITE INTEGR.4L 309

Evidently, the Riemann sum provides an approximation for tiie area A under the
graph of /between x = a and x = b. that is,

A - Z AA* = E /(a) A.t,

If an exceedingly large number of very skinny rectangles are involved in the Rie-
mann sum. this approximation will be quite accurate.

EX.4MPLE 1 Find the Riemann sum for the function /(.r) = 1 + x^ on the interval
[—1. 2]. using the augmented partition ff* consisting of the six subintervals

[-1. -*]. [-*, 0]. [0.1]. [i. 1]. [l.f]. [1,2]

with C| = -J C2 = f3 = 2 Q = 1 Cs = I Ce = 2

Sketch a graph showing the six rectangles corresponding to this Riemann sum.

Bernhard Riemann SOLUTION For the given partition, each of the six subintervals has length \ unit,

that is.

-Iv, 1X2 = -^^3 = A.V4 = Aa5 = A.V6

Also,

/(f,)= 1 +(-!)-' = J /(C4) 1 + 1-' = 2

/(r2)= 1
+0-'= 1 /(C5)=l +(§)' a\3 =

/(C3) = 1 + (i)' = I /(Ce) =1 + 2-^ = 9

Therefore, the required Riemann sum is given by

E f(Ck) A.V,
«.= !

= /{c,) A.V, +f(C2) 1X2 +fiC3) A.V3 +/(C4) A.Y4 +/(c-5) AXs + f(Cf,) AXf,

= iM) + (l)(i) + (l)(^) + (2)(^) + (f )(i) + (QKi)


Figure 5 —
— 147
16

fU)= 1
+<^^ Figure 5 shows the six rectangles whose areas are the terms in the Riemann sum.

As you can see from Figure 5. the Riemann sum

Z /(Q) Ax, = W
*=i

provides an approximation to the area under the graph oi fix) = 1 + x^ between


V = — 1 and v = 2. For a better approximation, we could take an augmented parti-

tion with a smaller norm (and hence with more rectangles).


In general, if/ is a continuous function and/(.T) s for a s .r < fe. the approxi-
mation

A= X /(c*) AX;t

for the area A under the graph of /between x = a and x = b becomes better and
310 C HAPIER 5 THE DEFINITE INTEtJRAl.

better as the number of rectangles increases and their widths decrease. Since this
can be accomplished by taking the norm |l.y||
of the partition smaller and smaller,
we might expect that

lim 2 /(c*) Ax,,

should exist and give the exaci area A.


As the discussion above indicates, the area under the graph of a continuous
function may be found by evaluating the limit of a Riemann sum. However, nothing
in the construction of the Riemann sum

E /(Q) A.V,

corresponding to an augmented partition for a function/ requires that the function be


continuous, that its values be nonnegative. or that there be any particular asstKiation
with area all that is required is that fix) be defined for every .v in the inter\al

[a. b]. As you will see in Chapter 6. there are many quantities other than areas that
can be obtained as limits of Riemann sums. Thus, we make the following defini-
tion.

DEFINITION I The Definite (Riemann) Integral

If the limit lim A /(q) A.Vf exists, then the function/ is said to be inteerable

()/; |(/. b\ in the sense of Riemann . If/is integrable. then the definite (Riemann)
integral of/ on the interval [a. b] is defined by

fix) d.x = lim 2 /(a) A.V4

The limit indicated in Definition 1 to be understood in the following sense: To


is

say that a number L is the limit of the Riemann sum 2*=i /("^a* ^* ^^ ^^e norm
||'»^|| approaches zero means that for each positive number e (no matter how small),
there exists a positive number 5 (depending on e) such that

E/(Q) A.V, -L < €

holds for every augmented partition !P* with ||.'*"'|| < 8.

The notation Saf(x) dx for the definite integral is deliberately chosen to be similar
to the notation / /(.r) dx for the indefinite integral (antiderivative). The advantage of
this is seen in Section 5.4. Because of the similarity of notation, you must be careful
not to confuse the definite integral /^ fix) dx with the indefinite integral //(.v) dx.
Note that the definite integral is a number — a limit of Riemann sums — whereas the
indefinite integral represents aU functions that are antiderivatives of the integrand.
SECTION S.2 THE DEHNITE INTEGRAL 311

We call /^/(.v) dx the definite integral from a to ft of the


function /. The
interval [a. b] is called the interval of integration, and
the numbers a and b are
known as the lower and the upper limits* of integration. The function/ or the
expression /(x), is called the integrand, just as it is for the indefinite integral:

upper limit
of integration

lower limit integrand


of integration

If the
function/is integrable on the closed interval
[«. b] in the sense of Riemann
we simply say that/is integrable on [a, b] or
that /2/(.v) dx exists. If
f'^f(x) dx
exists. It can be shown that it is a uniquely
determined real number (Problem 38)

Existence of the Definite Integral

Virtually every function encountered in


practical scientific work is integrable on any
closed interval contained in its domain. This includes not only all continuous func-
tions and
monotone (increasing or decreasing) functions, but also
all
all functions
that are bounded and --piecewise continuous"
or "piecewise monotone" as well
The proofs of the integrability of such functions can
be found in more advanced
textbooks on analysis; here, we content ourselves
with precise statements of some
of the existence theorems for definite integrals.

THEOREM 1 Existence of the Definite Integral of a Continuous


Function
If/ is a continuous function on the closed interval
\a. b\. then /is integrable on
^
[a, b].

Figure 6
For instance, a polynomial function is integrable on any closed interval since
polynomial functions are continuous.
f(x)dx The function whose graph
(
is shown in Figure 6 is not continuous, but it is piece-
wise continuous on the interval [a. b] in the sense that [a. b] can be partitioned into
number of subintervals such that /is continuous on each
a finite
subinterval The
function /is also
bounded on the interval [a.b]m the sense that there are
fixed
numbers A and B such that A £f(x) < B holds for all
values of x in [a. b] The
integrability of any such function is ensured by the following theorem.

THEOREM 2 Existence of the Definite Integral of a Piecewise


Continuous,
Bounded Function

It,/ IS a bounded and piecewise continuous function on the closed interval !</. b
then /is integrable on [a, h\.

*This use of the word "limit" must not be


confused with the limit of a function or with the
limitof Riemann sums in Definition 1
312 CHAPTER 5 THE DEFINITE INTEGRAL

Figure 7 The function defined by

for .V >
fix) =
for v <
is piecewise continuous but not bounded on the interval [-1. 1] (Figure 7). A
glance at Figure 7 suggests that great care should be taken in talking about the
"area" of the region under the graph of an unbounded function. Theorem 3 gives
1 1
further evidence for the necessity of such care.

THEOREM 3 Boundedness of Integrable Functions

If/ is defined and integrable on [((. b], then / is bounded on |((. h

For instance, the function / whose graph appears in Figure 7 is not integrable on
(—1. 1], since it is unbounded on this interval.

The following theorem can be useful in deciding whether certain functions are
integrable.

THEOREM 4 Change of a Function at Finitely Many Points

It/
satis

also
SECTION 5.2 THE DERMTE INTEGRAL 313

EXAMPLE 2 d\ (left endpoints)


-'n

SOLUTION Let tP,, denote the partition of the interval [0. 2] into n subintervals of
equal lengths
=
2-0
A.v

Figure 8 (Figure 8). Thus, the augmented partition (P* consists of the n subintervals

Ax Ax Ax [0. A.V], [A.V. 2 A.V]. [2 A.v. 3 ^.x] [2 - A.v. 2]

with C| = 0, r: = 1.x. c, = 2 A.v. . . . , c„ = 2 - A.v


A.V 2A.V 3A.V

Evidently, for ^ = 1. 2. 3 n. we have


2 2{k- 1)
Q. = {k- 1 ) A.V = (A- - n— = /; /)

Consequently, the Riemann sum corresponding to 'P* for the function /(x) = x^ is

given by

^ ^ , ^ r 2(A - 1) 1' 2
V 16

Using the properties of summation in Section 5.1. we have

E /(a) A-Xk = S -^(fc - 1)' = ^E (A- - 1)^

= ^- Z (A- - 3)t- + 3A'- 1)

= ^( S /t-' - 3 E '^•' + 3 S A- - 2 1

i=l k=l /t=l *=]

16 r »r(« + 1)- _ n{n + l)(2n + 1) n(/! + 1)


~ " ^ ^ "]
/i4 4 6

16 r «* - 2^r' + «- 1 / 2 1

Figure 9 Therefore.

x^dx= lim S /(q) A.v^. = lim 4(l + -^) = 4


Jo "-*" "-'
"-""i^i ^ «

Since .v' ^ for < .v < 2, we may interpret the result

.r- dx = 4

as stating that the area under the graph of /(.v) = x^ between .v = and .v = 2 is

4 square units. Because fix) = x^ is an increasing function on the interval


[0. 2], inscribed rectangles are obtained by forming augmented partitions .'P,t for
which Ci, C2, C3, .... c„ are the left endpoints of the subintervals; for instance.
i i :
1
Figure 9 shows the case where n = 4.
314 CHAPTER 5 THK DEKINITE INTEGRAL

Figure 10
A dx
A.v A.V A.X
J J
(right endpciints)

soi.irioN Let .'P,, denote the partition of the interval


-2 + Ax -2+2AX -2 + 3A.V ••• 1 - A.v 1
[-2, IJ into n subintcrvals of equal lengths

^
1 -(-2) 3

(Figure 10). Thus, the augmented partition ,'f* consists of the n subintervals

1-2. -2 + A.v], [-2 + A.V, -2 + 2 A.v], [-2 + 2 A.v, -2 + 3 A.v],


. . . , 1 1
- A.V, 1
]

with f| = -2 + A.V, (-2 = -2 + 2 A.v, o = -2 + 3 A.v c„ = 1

Evidently,

Q = -2 + k A.v = -2 + ki
/
—3 \

j
=
3k
2
^ n ' n

Consequently, the Riemann sum corresponding to 0'* for the function /(.v) = .v is

given by

i/(c-j A.V, = s c, A.V = i (— - 2) (-) = s (^ - -)

V 9fc V 6 9 V 6

"'";" ]-6
= -^[ 4(-^)-6
= l(..^l-.
Therefore,

.V dx = lim E /(c>) A.V, = lim — (l+— )-6= — -6=- —


Since the function /(.v) = .v has negative values on the interval |-2, 1), we cannot
interpret the integral

1:
v dx = — 3

as an area.

The Deflnite Integral /' /(x) dx for a > 6


J
In the definition of the definite integral f'',f{.x) dx, it was assumed that a < b, so
that \a, b\ is a closed interval. For certain purposes, it is convenient to be able to
deal with expressions such as /';/(.v) d.x without worrying about whether a < b.

This is accomplished by the following definition.

I
SECTION 5.2 THE DEFINITE INTEGRAL 315

DEFINITION 2 The Definite Integral


J' I f(x) dx ior a ^ b

(i) If/ is any function and a is a number in the domain of/, then we define

f(x) dx =
J

(ii) \i a> b and/ is integrable on [h. a], then we define

fix) dx = /(A) dx
J -J

EXAMPLE 4 Evaluate

(a) xdx (b) xdx


-'-2 •'l

SOLUTION

(a) By part (i) of Definition 2, x dx = 0.

(b) By Example 3 above, .v dx = - f. Therefore, by part (ii) of Defini-

tion 2,

xdx= -\ xdx= -(-§) =


J, J_2

Problem Set 5.2

In Problems 1 to 4, find the Riemann sum for each function on the intervals of equal lengths, and augment the partitions by using the
prescribed interval, using the indicated augmented partition. Also. left or right endpoints of the subintervals as indicated.
sketch the graph of the function on the interval, showing the rectan-
gles corresponding to the Riemann sum. ri (-3

5 Iv dx (right endpoints) 6 2x dx (left endpoints)


•'0 ^
1 f(x) = 3a: + 1 on [0, 3]. (P* consists of [0, i], [L 1], [1, i],

[i, 2], [2, i], and [|, 3] with c, = i, c, = I, cj = f, C4 = 2,


7 {Ix - 6) dx (left endpoints)
C5 = i, and c^ = 3. -'4

2 /(x) = 2x- on [0, 3]. S"* consists of [0, i], [i, 1], [I, f], [f, 2],
~ 8 (9 - X-) dx (right endpoints)
[2, t], and [|, 3] with c, = i, C2 = i, C3 — 4 , C4 ~ 4 ,
C5 •'1

and C6 = ¥
f"' .
— -
= 9 t-v" .V 2) dx (right endpoints)
3 /U) l/.r on [1, 3]. IP* consists of [1, i], [f, 2], [2, §], and
[i, 3] with ci = J, f2 = h C3 = I, and C4 = t-
+ 10 (.v' -I- 2) dx (right endpoints)
4/U)= 1/(2 a) on [-1, 2], a** consists of [-1, -*], [-i, 0], -'

[0,5], [i, 1], [l,i], and [f, 2] with ci = -l.cj = "icj = 0,

Q= 2, C5 = 1 , and C6 = i
11 (.v' -H 2) iv (left endpoints)
•'0

[''
In Problems 5 to 14, evaluate each definite integral directly by cal- -
12 (4 X') dx (right endpoints)
culating a limit of Riemann sums. Use partitions consisting of sub-
316 (IIVPIKR 5 THK DKFINITF. INTKCRAI.

13 (1 - Iv") d.x (left endpoinls)

14 (a) (v- - 4.V + 2) dx (left cndpoinb)


J

(b) (A- - 4.V + 2) </.v (right endpoints)


i:

In Problems 15 to 28, indicate whether each definite integral exists


and give a reason for your answer.

(.(XKI 1

15 — (/v
SECTION 5.3 BASIC PROPERTIES OF THE DEFINITE INTEGRAL 317

and the same geometric interpretation applies to a rectangle of height K units and
width a — b. Finally. '\i a = b. the equation

\> dx = K(a - a)

just says that = 0.

Two special cases of Theorem 1 are of interest:

\'>-[ dx = b — a and = Q{b - a) =


\
J"0..v

EXAMPLE 1 Find PI2 (-7) dx.

SOLUTION By Theorem 1,

(-7)dLr = (-7)[33-(-2)]= -245


J

THEOREM 2 Homogeneous Property

If / is a function, a: is a constant, and /*/(jc) dx exists, then /* Kf(x) dx exists


and

Kf{x) dx = k\ fix) dx
J

Figure 2 shows the underlying geometric reason for the homogeneous property;
namely, when /is multiplied by K. the rectangles whose areas are involved in the
Riemann sum have their heights multiplied by K, and so their areas are multiplied
by K.

Figure 2

EXAMPLE 2 Given that /t VW dx= -\l, find Jf^ 52/(.r) dx.

SOLUTION By Theorem 2,

j 52/(.x) ^t = 52 f(x) dx = 52(- 13) = -676


J
318 CHAPTER 5 THE DEFINITE INTEGRAL

Figure 3 THEOREM 3 Additive Property

Let/and g be functions, and suppose that /,';/(.v) d\ and fa i'(v) d\ exist. Then
area = /(c) Av
/a I fix) + g{x)] dx exists and

l/W + ^(.v)] ^.v = fix) d.x + g(x) dx


J J J

The underlying geometric reason for the additive property of the definite inte-
gral can be seen in Figure 3, which shows corresponding rectangles involved in

Riemann sums for/, g, and f+g. The area of such a rectangle for/+g is

[/(t) + g{c)] A.V = f{c) A.V + g(.c) A.v; hence, its area is the sum of the areas of the
corresponding rectangles for /and for g.

EXAMPLE 3 Given that pi-, f(x) dx = 2.11 and /L\ g(.v) dv = - 1,32, find

;'-S (/(.v) + gix)] dx.

SOLUTION By Theorem 3,

[f(x) + g(.x)\ dx = fix) dx + gix) dx


/:
= 2.77 + (-1.32) = 1.45

The homogeneous and additive properties of the definite integral can be com-
bined to yield the following property.
area =(/!:<.-) + g(c)|Av
(c, /(() + ««•))

* THEOREM Linear Property


4

Let A and B be constants, and suppose that /and g are functions for which
if, fix) dx and /S gix) dx exist. Then /^ [A/(.v) -I- Bgix)] dx exists and

•M
[Afix) + Bgix)] dx = A fix) dx + B gix) dx
J J j
(0

EXAMPLE 4 Given that fl .x^* dx = ^ and Ji x dx = i. find Js ( lO.v'* + 16.v) dx.

-SOLUTION By Theorem 4,

(10.Y-' + 16.V) dx = 10 .v-" dx -(-


16 .V dx = lO(^) + 16(f) = 462
J J J

By using the principle of mathematical induction, the linear property can be


extended to more than two functions. Thus, for example, we have

(4.v' - 3.V- + 7.V - 8) dx


-'o

= 4 x^ dx + {-3) \
x-dx + l\ xdx + (.-S)\ dx M
= 4 .v' dx - 3 .V- dx -1-7 .V ^v - 8 dx
-'(I -'n M -'n
SECTION 5.3 BASIC PROPERTIES OF THE DEFINITE INTEGRAL 319

THEOREM 5 Additivitj with Respect to the Interval of Integration

Suppose that the function /is integrable on a closed bounded interval / and that a.
b, and c are three numbers in /. Then /a/(.T) dx. 5 a fix) dx. and /^/(.y) dx exist
and

fix) dx = fix) dx + fix) dx


J J J

Figure 4 For the special case in which a < b < c and/(.v) > for a < .v < c. Theorem 5
has the following geometric interpretation: The area under the graph of y = fix)
from .V = a to .r = r is the sum of the area from x = a to x = b and the area from
X = btox = c (Figure 4). From this special case, the result for other orderings of a,
y=Kx) b. and c can be derived by using Definition 2 of Section 5.2. For instance, suppose
that a < c < b. Then
fA.lvT
fix) dx = I
fix) dx + I
fix) dx

Solving the last equation for Jafix) dx. we obtain

fix) dx = fix) dx - fix) dx = fix) dx + fix) dx


'a 'a 'c 'a 'h

We leave it as an exercise for you to check the remaining cases (Problem 59).

E.XAMPLE 5 Given that /-_, /(.v) aLv = 7 and that J^fix) dx = -5. find
P-ifi.x)dx.

SOLUTION By Theorem 5,

fix) dx = fix) dx + fix) dx = 7 + (-5) = 2

Although we illustrated Theorems 1 through 5 geometrically for only those cases


in which the definite integrals can be interpreted in terms of area, we must empha-
size that these theorems hold in all cases, even if no such interpretation is possible.
In the next section, we method for evaluating definite integrals.
present a general
Meanwhile, the following theorem gives some special integrals to aid in the illustra-
tion of the basic properties.

THEOREM 6 Two Special Integrals

.r .V dx ib- - a-) (u)


f',.
dx = —ib' - a-)

PROOF We prove (i) and leave the proof of (ii) as an exercise (Problem 60). To begin with,
suppose that a < i>. Let iPn denote the partition of the interval [a,b] into n subinter-
vals of equal length

A.v =
320 CHAPTER S THE DEFINITE INTEGRAL

Thus, f?„ consists of the n subintervals

\a. a + A.t], \a + A.v, a + 2 A.v], . . . , [a + (/! - 1 ) A.v, Z;]

and the A:th subinterval is

\a + (k- 1) A.Y, « + k Aa]

for A: = 1, 2, 3 n. To obtain an augmented partition ff* . we choose the


numbers

(|. C2 c„

to be the right-hand endpoints of the corresponding subintervals. so that

i\ = a + k A.V

for A = 1, 2, 3, . . . , n. Thus, the Riemann sum for/(.v) = .v corresponding to ,'P*

is

E /(Ci) A.v^- = S Q A.V = 21 (fl + <: A.V) A.v

b — a \ I b — a
= i(«
ya + k
k-
n ' "^
n

a(b - a) V' / '' ~ ii \' sr


= Zl + Z.k
t? \ n I

a(b ^-a^^^lb^
— a) b — a ( \' ii{n + 1)

n ^ n '

= a(b - a) + (b - iiY

a{a + - «)(| +
(/,
^)]
Therefore,

xdx= lim S /(a) A-v^ = lim (/? -«)« + (/>-«)(— + -—


Jy "~*"^*
^=1
n^ + -^
L \2 2«/J

= (/? - a)\a + {b - a)(h)\ = i(b - (i)(h + a) = i(/;- - «')

This proves (i) for« < b. If « = b. then (i) is obviously true since both sides of the
equation are zero. Finally, suppose b < a. From what has already been proved, we
have

v d\ = kUr - b")
SECTION 5.3 BASIC PROPERTIES OF THE DEFINITE INTEGRAL 321

Figure 5

i
322 CHAPTER 5 THE DEHNITE INTEGRAL

in spite of the fact that

f{\)^\- \

(Figure 5). This is justified by Theorem 4 of Section 5.2, since

/(.v) = A- - 1

for all values of .v in [0. I] except .v = 1.

THEOREM 7 Nonnegative Property

U/is an integrable function on the interval \ci, h\ and if/(.v) > for all values of

X in [a. b]. then fix) dx > 0.


J

Figure 6 Theorem 7 is geometrically evident since if /(.v) > for a ^ .r s fo, then

j\x)dx
jV(x) dx>0 J

is supposed to represent the area under the graph of y = fix) between x = a and
.V = b (Figure 6).

EXAMPLE 9 Show that /i .v'' iLx < /', x dx.

SOLUTION For < .V < 1, x^ < .V holds; hence, .v - .v'' > 0. By Theorem 7,

(x - x^) dx^O

that is.

xdx- \ x^ dx>0

From the last inequality.

v' dx

that is.

x^ dx^ \
X dx
•'o A)

The argument given in the last example is quite general. Indeed, suppose that/
and g are integrable functions on the interval [a. b] such that/(.v) < g(x). Then,
g(x) - f(x) > 0, so that by Theorem 7

\g(x)-f(x)\dx^Q
f
that is.

g(x) dx -
g{x)dx-\ fix) dx>0 or /(A)dv< gi-x) dx
J J j
SECTION 5.3 BASIC PROPERTIES OF THE DEFINITE INTEGRAL 323

Therefore, we have the following theorem.

THEOREM 8 Comparison

If/ and g are integrable functions on the interval [a, b] and if/(A) < g(x) holds

for all values of .v in [«, b\. then I /(.v) dx < j


g(x) dx.

Theorem 8 provides the basis for the following theorem.

THEOREM 9 Absolute- Value Property

A Theorem 9 would require an argument to show that |/| is


rigorous proof of
integrable, but this beyond the scope of this book. However, assuming that /and
is

I/I are integrable functions on the interval [«, /?], we can derive the inequality in
Theorem 9 as follows: Applying Theorem 8 to the inequality

-|/(.v)|</(.v)<|/(.v)|

we obtain

- |/(.v)| dx < f(x) ^^ < |/U)| dx


J J J

that is,

- |/(.v)| dr < f(x) dx^\ |/U)| dx


J J

It follows from Rule (iii) on page 7 that

f* r*
I

J f(x)dx\
I

< \f(x)\dx
I J

The following property of the definite integral plays an important role in our
forthcoming proof of the fundamental theorem of calculus.

THEOREM 10 Mean- Value Theorem for Integrals

Suppose that/ is a continuous function on the interval [ci,b]. Then there exists a
number c \n [a, b] such that

f(c) (b «) = fix )dx


I

By Theorem 1, page 167, the continuous function /on the closed interval [a, b]
takes on a maximum value, say, B, and a minimum value, say, A. Thus,
A ^f{x) <B holds for all values of j: in [a, b]. By Theorem 8, then.

J
Adx^j f(x) dxsj B dx
324 ( HAPTER 5 THE DEFINITE INTEGRAL

By Theorem 1

A dx = A(h - a) and B dx = B{h - a)


I

Hence, A(h - «) s fix) dx < B(b - a)


I

But h — a > Q, so the latter inequahty can be rewritten as

1
<B
-r f(x) d.x

Now the intermediate-value theorem for continuous functions (Theorem 1,

page 150) implies that /takes on every value between any two of its values. Thus,
since A and B are two such values of/ and since \\/(b — a)] /^/(.v) dx lies between
these two values, there must exist a number c in [a, b] such that

= fix) dx
i\c)
7f
that is. j\c) (b a) = /(.v) dx
J

If fix) > for fl < X < fe, the condition f{c)ib - a) = J!;/(.v) dx (Figure 7)
means that the area under the curve v = fix) between x = a and .v = b k the same as
the area of a rectangle whose base is the interval [a, b] and whose height is /(c).

Figure 7 y

Ac)
SECTION S.3 BASIC PROPERTIES OF THE DEFINITE INTEGRAL 325

The mean-value theorem for integrals just says that a continuous Junction f on an
interx'al {a. b] takes on its own mean value at some number c on this interval.

EXAMPLE 10 Find the mean value of the function/(.v) = .v" on the interval [1,4],
and find a value of c on this interval such that /(c) gives this mean value.

SOLUTION The desired mean value is given by

-
v- A- = —— (4-'
- h') = 7
4 1 J, 3 L 3 J

We need to find a value of c with 1 < c < 4 such that /(c) = c" = 7. Evidently.
c = V7.
In a definite integral

f(x)dx
J

no particular reason
there's to use .x as the variable of integration — any other letter
would do. In fact.

f{x)dx
J

is a number depending on a. b. and the function/; it really has nothing at all to do


with .V. In other words, the variable of integration in a definite integral is a '
'dummy
variable." so that, for instance,

.V- dx= \
t- dt= \ s- ds= \
y- dy = ^(1^ - 0^) = ^
^0 ^ -'o
-^0

More generally.

fix) dx = fit) dt
J J

Often it is necessary to consider a definite integral in which one or both of the


limits of integration are variable quantities. For example, if the upper limit of
integration is a variable quantity and the lower limit of integration is fixed, then the
value of the integral will be a function of the upper limit. Since it is customary to
use the symbol .v for the independent variable whenever possible, we might be
tempted to write such a function as

g(.v) fix) dx

however, the .v in the expression /(.v) dx is the (dummy) variable of integration and

must not be confused with the independent variable .v in the expression gix) and in
the upper limit of integration. Therefore, in such a situation we use another symbol
for the variable of integration and write, for instance.

gM f(t)dt
326 CHAPTER 5 THE DEFINITE INTEGRAL

Problem Set 5.3

In Problems 1 to 30. use the basic properties of the definite integral


to evaluate the expression.
32 COS" .r dx (Hint: sin" x + cos" .r = 1.)
^

2dx + dx 33 3 cos X dx
4: (7 TT)
4/2
(0.75 rW2
i-dx) 4 +V2-Vi)dx 34 cos 2r dx {Hint: cos 2t = I
- 2 sin" x.)
(\
•'05

In Problems 35 to 38, evaluate the definite integral.


dx 6 (-4) dx
J
Zx- for < .r < 2
35 fix) dx. where /(.v) =
2dx 8 2dx •"o Ax for 2 < .X < 4
J

1 for .V 5^
36 fix) dx. where /(.v) =
Sxdx 10 {-3x)dx '-1 for .r =

1 -X for -2 s.rsO
37 /(A) dx. where /(.v) =
(-2v) dx 12 (4 - 3x) dx J 1 + .r for <X< 3
J

for .r =
(2v+ l)dx \]dx
'2/3 ^ - 38 f(x) dx. where /(.v) = -^ .v + 1 for < .r < 1

for X = 1

(.r + X-) dx 16 (.V- - l)dx


J
39 Use the nonnegative property or the comparison property to de-

(3.r- -Zx + \)dx 18 (-Zx' + 4x + 5) dx


cide whether each inequality holds. Do not evaluate the definite
j ^
integrals involved.

Ox - l)(2v + 3) dx 20 lx+
(2v If dx (a) X dx s dx (b) .V- dx < \
X dx
I

4x(lx - 7) dx 22
J,
r-25
X
.r - 5
(c) £
Jo
sin X dx (d) <
^ 1 +
^ .V-

(-1 r\ rn/2 ri,/2

f (e) .v' dx < .r* dx (f) sin .v dx < x dx


X dx + X dx 2 .V d:v •'o ^ •'o -'n

-'o

40 If < A" sf(x) holds for all values of x in [a. b], prove that

{2x- \)dx + (2r- l)dx < SafM dx. [Hint: Use the comparison theorem and the fact
j
that f^, K dx = Kit - a).]

.t" it A In Problems 41 to 48. find the mean value of the function /on the
inter\al [a. b] and find a value of c in [a, b] such that /(c) is equal to
|.v| dlr 28 |.t - 1| d[r this mean value.
I
'-1

41 /(x) = .v-l- 5 on [1, 3]


l.v'l dx 30 [I.rD</v
I

42 /(jc) = .v" on [-3, -1]

In Problems 31 to 34, evaluate the given expression. You may 43 /(.v) = .t- - 2r + 3 on [- 1 . 5]
as-
sume that fo^- cos X dx = and Jo^' sin" x dx =
\ 7r/4.
44 /(jc) = U- 2)(.T -I- 3) on [-3, 2]

45 f(x) = \x\ on [-2, 5] 46 f{x) = x on [a. b]


(3 COS JT - 2 sin^ .v) dx
47 /(A) = Ax + B on \a. b] 48 fix) = a" on \-a. a]

I
SECTION 5.4 THE FUNDAMENTAL THEOREM OF CALCULUS 327

In Problems 49 to 58, use the properties of the definite integral to stant. Find the mean value of F if the spring is stretched from
justify each statement. s = alo s = h. Also find a value of c between a and b such that

the value of F when 5 = r is the same as this mean value.

49 < (1 + cos .V) dx 50 .V* dx < X dx


J„/4 •'o •'o 62 Suppose that /is a continuous function such that jafix) dx =
for even- closed interval [a. b]. Prove that/is the zero function.
ran rnx r\n rW2
51 tan v d.x = tan f di 52 x" dx < rdi
\

'o A)
63 Is it true that when a perfectly elastic spring is stretched, the
Ai -'n

work done is equal to the product of the mean force (see Prob-

= - lem 61) and the distance through which the spring is stretched?
53 sin .V dx sin .v dx sin j: d!x
Jtt ^ 'o Justify your answer.

riooo
64 Suppose that/and g are integrable functions on [a. b] such that
54 U'™ + VW^)rf.v>0 - g(x)\ <K for every number x in [a. b], where A: is a
-"-lOOO l/U)
positive constant. Prove that

55 V.v- + 1 dx = Va- + 1 rf.v - V.v- + 1 dx


A Jq 'o
<
fMdx- g(x) dx\ K{b - a)
\j j

56 \/5x- + 3 rf.v - V^5.v- + 3 dx + V^5.v' + 3 rf.v


=
•'4 -'4 •'- I {Hint: Use the absolute- value property.)

65 Prove Theorem 10 continues to hold even a S: b; that is,

57r^^-r^^=r^^+r^^
+ J,+ + +
1 .V- J5 \ X- Jft 1 ?- Jj 1 v- show
that

that if/ is continuous on the closed interval


if

between a and
b, then there exists a number c on this interval such that
f(c)-(b-a) = {tf{x)dx.
J„ Vl + .V- 4 Vl + V- J, vrrp prove that
66 If/ and g are integrable functions on [a, b].

59 Assuming that Theorem 5 is correct for the special case in which


a < b < c, and considering that the case a < c < b has J [fix) + gtx)] <iv| < |/(.v)| dx + \g{x)\ dx
I
J J
already been discussed, check the result for the remaining cases;
b<a<c,b<c<a, c<a<b, and c < b < a. Also, check 67 Find formulas for (a) Ji dt. (b) /; I dl. and (c) J; /" dt.

the result for the cases in which two of the numbers a. b, and c
are equal.
68 If /(.v) = Ax- + Bx + C. find a formula for

60 Prove part (ii) of Theorem 6.


/(f) dt

61 The force required to stretch a spring from its relaxed position


through J units is given by F = ks, where A: is the spring con- (Hint: Use the results of Problem 67.

5.4 The Fundamental Theorem of Calculus


In this section, we state and prove a theorem that establishes a profound connection
between antidifferentiation and definite integration — a theorem so useful and so
important that it is known as the fundamental theorem of calculus. We begin by
giving an informal argument that leads to this theorem; then we illustrate the useful-

ness of the theorem in several examples; and finally we give a formal proof of the
theorem.
You might already suspect that a connection exists between the indefinite and the
definite integrals. After all, in Section 4.7, we used indefinite integration to find
areas by the method of slicing, whereas our definition of the definite integral in
Section 5.2 was suggested by the idea of the area under a curve. Here is an informal
argument to make this connection more explicit:
328 CHAPTER 5 THE DEFINITE INTEGRAL

Figure 1
SECTION 5.4 THE FUNDAMENTAL THEOREM OF CALCULUS 329

DEFINITION 1 Special Notation

If g is any function and if g(a) and g(b) are defined, then

\b

gix) = gib)- g(a)

The notation g{.x) is read "g(.x) evaluated between x = a and .x = b." For i

stance, in the previous example with ^(.v) = a"*, we can write

4.x^ dx = .x* = 2-*- !*= 15


1

Using this special notation, we can rewrite the fundamental theorem of calculus
the form

1 fi.x) d.x
=
330 CHAPTER 5 THE DEFINITE INTEGRAL

rr/3

EXAMPLE 3 I
sin .V dx

sin -x civ = I sin x dx = —COS


( .x)
I

= (-COSj) -(-COSO)=
3
(-— ) -(-l)= —

Note that the fundamental theorem of calculus.

/(.v) d.x = /(.v) dx\


J [ J

works even if a s fe (Problem 69).

+
EXAMPLE 4 Evaluate
f ' f-'
; —9
dt by using the fundamental theorem of
calculus.

SOLUTION

J
— di = \ (t + 9t -) dt

= [/(,. 9,-=,
,*][ = (f + ^)[
=
(4-7)l>(T-T)-(f-T)-'»
Sometimes it is useful to split the interval of integration into two (or more)
subintervals before using the fundamental theorem of calculus.

EXAMPLE 5 Use the fundamental theorem of calculus to evaluate /fill- ^1 dx.

SOLUTION For.v < 1, we have - .v > 1 0; hence. |l - .v| = 1 - .v. On the other

hand, if .v > 1. we have 1 - < 0; hence,


.x: |l ~ x\ = -(1 - x) =x- 1. This sug-
gests that we split the interval of integration [0, 2] into the two subintervals [0, 1]

and (1, 2]. Thus, by Theorem 5 in Section 5.3,

\\-x\dx=\ \\-x\dx+\ \\-x\dx=\ (\-x)dx+\ (x-\)dx

=(-'-4)l>(T-')l>(|-»)4''-(-i)]-

Change of Variable in a Definite Integral

Suppose we want to evaluate the definite integral

r- .V- dx
[ (.V-' + 1?
SECTION 5.4 THE FUNDAMENTAL THEOREM OF CALCULUS 331

Since we propose to use the fundamental theorem of calculus, we begin by using the
change of variable u = x^ + 2 to evaluate the corresponding indefinite integral.
Thus,
du = lix- dx so x~ dx = I du

and we have

J (x^ + 2)-
2) J u- 3 J 3-1
+ C = -. + C
3« 3U' + 2)

Therefore,

n x^dx _ -1 1^^ -1 -1 ^ 7
" + + +
J, (x^ + 2f 3(x^ 2) 1 , 3(8 2) 3(1 2) 90

The calculation above can be shortened by changing the limits of integration in


accordance with the change of variable. Indeed, the limits of integration in the

original definite integral refer to the variable x, a fact that is sometimes emphasized
by writing

J,=
f""-^
— X- dx
I
+ 2)-
T rather than
(~

J,
— x^ dx
^

(.x-' +

2)-
,
(.v^

To make the change of variable u = x^ + 2, we notice that

when X = 1 we have « = 1^ + 2 = 3

and
when X = 2 we have j< = 2^ + 2 = 10

Thus, we can abbreviate our previous calculation as follows:

J,= i
(x-' + 2)- J„=, M- 3 Jj 3 V M

= -(—)--(—)=-
3 V 90 10 / 3 V 3 /

More generally, you change the variable from, say, x to, say, u = g{x) in a
if

definite integral, not only must you change the integrand just as you do for indefi-
nite integrals, but also you must change the limits of integration. Thus

f[g{x}]g'{x)dx = f(u)dii
j
J

In Examples 6 and 7, use a change of variable to evaluate the integral.

EXAMPLE 6 xV9 - 5jc- dx


^
SOLUTION We make the change of variable u = 9 - 5x', noting that du =
- \0x dx, or X dx =- TO du. Also, u = 9 when x = 0, and u = 4 when x = 1
332 CIIAPIKR 5 THK DEHNITE INTEGRAL

Therefore,

.V V9'^^~5p dx = V9 - 5.V- .V dx
^ -'v=o

Vui ]du = V^idu


10 ^
i=9 ^ 10 X,

= If',/.
h'/' du =
I
"'^'
—r
10 J4 10 i I4

1 V. 1 ,n _ 19
9-V2 4-V2
15 15 15

EXAMPLE 7 COS" sin dt


r / t

SOLUTION We make the change of variable u = cos t. noting that du =


-sin / dt, or sin t dt = -du. Also, // = 1 when i = 0, and « = 5 when / = 7r/3.

Therefore,

riT/3 ^ ri/2 ^ n/2 ^ ri ^

I cos' t sin t dt = u-[—du) = - I ir du = u' du


\) '1 -'1 -'1/2
SECTION 5.4 THE FUNDAMENTAL THEOREM OF CALCULUS 333

In Examples 8 lo 10. find the indicated derivative.

EXAMPLE 8 dt
dx Jq 5 + r

SOLUTION ({{ =
dx Jq 5 + r 5 + r'-

EXAMPLE 9 -^ I
csc.vo^v
dy 4/2

SOLUTION -- CSC .V ^V = CSC V


dv J-n

EXAMPLE 10
334 CHAn KR 5 THE DEHNITE INTEGRAL

Proof of the Fundamental Theorem of Calculus


Wc now give a formal proof of the fundamental theorem of calculus. Here we begin
by giving a formal statement and proof of the alternative version involving the
derivative of an integral. U.sing this as a basis, we can then prove the fundamental
theorem itself.

THEOREM 1 The Derivative of an Integral

Let /be a continuous function on the closed interval \h. c]. and suppose that a
is a fixed number in this interval. Define the function g with domain [b. c] by

g(-x) = { findt
for X in [b. c]. Then g is differentiable on the open interval {b, c), and

g'(x)=f(x)

holds for all x in {b. c). Furthermore, g is continuous on [b. c]. and at the
endpoints h and c, we have

gUh)=f{b) and g'-(c)=f(c

Suppose that .v belongs to the open interval (b. c) and that A.v is small enough that
.V + A.v also belongs to {b. c). Then

gi\) = fit) dt and g(x + A.v) = f(t) dt


J J

Therefore, by Definition 2 in Section 5.2 and Theorem 5 in Section 5.3,

^(.v + A.V) - g(x) = /(/) dt - j\t) dt = fU) dt + fit) dt


J ^ J ^

= f{t)dt + fU)dt = f{t)dt


J j j

Since / is continuous on the interval \b. c\. it is also continuous on the closed
subinterval between .v and .v + A.v. By Theorem 10 in Section 5.3, there exists a
number v* on the closed interval between .v and .v + A.v such that

f(t) dt =/(.v*)|(.v + A.V) - .v] =/(.v*) A.V

Consequently,

g(x + A-v) - ^(.v)


g{.v + A.v)-g(.v)=/(.v*) A.V = /(.v*)

Since .v* lies between x and .v + A.v, it follows that .v* approaches x as A.v ap-
proaches zero. Therefore,

g(x + A.V) - g{x)


^''(.v) = lim lim fix*) =/(.v)
A^.— (I

where we have used the continuity of/ in the last equation. This establishes the
desired result for values of v on the open interval {b. c). The proof that one-sided
derivatives of .s; at the endpoints b and c give the values of/ at these endpoints is
SECTION 5.4 THE FUNDAMENTAL THEOREM OF CALCULUS 335

similar and is left as an exercise (Problem 68a). Because g is differentiable on the


open interval {b. c). it follows from Theorem 1, page 96, that g is continuous on
(b. c). Using the fact that the one-sided derivatives of g exist at the endpoints b and
c. and arguing as in Theorem 1. page 96. we can easily show that g is continuous
on the closed interval [b. c] (Problem 68b).

In Theorem 1 . notice that the function g is an antiderivative of the function /on


the open interval (b, c). This fact is often phrased informally as follows: A continu-
ous function f always has an antiderivative g.

THEOREM 2 The Fundamental Theorem of Calculus

Let/ and g be continuous functions on the closed interval [a,b], and suppose that

g'(-v) =/(.r)

holds for all values of .v in the open interval {a, b). Then

fix) dx = gib) ~ g(a)

Define a function G with domain [a, b] by

G(.v) = fit) dt
J

for .V in («, b]. By Theorem 1, G is continuous on [a, b]. and

G'{.v)=/(.v)

holds for all x in [a. b). On the open interval (a. b). we have G' = f= g': hence,

(G - g)' =0
By Theorem 1, page 252, there exists a constant C such that

G(.v) - g(x) = C
for all values of .v in (a. b).

Since G and g are continuous from the right at a, we have


C= lim. C= Hm, [G(.v) - g(.v)]

= lim^ G(.v) - lim. g{x) = G{a) - g{a)

G(fl) =
336 Cll AIMER 5 TIIK DEFINITE INTEGRAL

Therefore,

G(b) = gib) - g(a)

that is.

J\i)dl = gib) - g{a)


J

Consequently,

fix) dx = /(/) dt = g(b) - g(a)


J J

and the theorem is proved.

Problem Set 5.4

In Problems 1 to 45. use the fundamental theorem of calculus to


evaluate each definite integral.

1
SECTION 5.4 THE FUNDAMENTAL THEOREM OF CALCULUS 337

In Problems 47 to 58. use the alternative version of tiie fundamental 70 The operator of a trucking company wants to determine the opti-
theorem of calculus to find the derivative. mal period of time T (in months) between overhauls of a truck.
Let the rate of depreciation of the truck be given by /(/ ), where i

47
is the time in months since the last overhaul. If K represents the
\)dt
dx Jo fixed cost of an overhaul, explain why T is the value of t that

minimizes gU) = i K+ fix)dx\.


48 Iw + \) dw I
'0 J

71 The marginal cost of processing a certain grade of tuna fish is

49 given by 200 - 30Va-, where


number of cans in thou- -v is the
dx J_, 1 + r sands and the cost is in dollars. What would be the total increase
in cost if production were increased from 4000 to 25,000 cans?

72 Find the maximum value of the function

/(.v)= [ (\t\-\t- \\)dt

on the interval [-1, 21.

73 (a) Show that /J \t\ dt = x\x\/2 by considering the separate cases


X < and x > 0. (b) Use the result in part (a) and the alternative
+ 1 ds
version of the fundamental theorem of calculus to prove that

d /.v|.vh

dx \ 2 )

74 Use the method suggested by Problem 73 to find an antideriva-


tive of the function /(.v) = |.v|".

75 Find the mean value of each function on the interval given,


In Problems 59 to 66, use the alternative version of the fundamental
theorem of calculus together with the chain rule to find dy/dx. (a) fix) = .r -t- 1 on [ 1 , 4] (b) fix) = x^ - 1 on [ 1 , 3]

(c) fix) = \/v on [1, 9] (d) fix) = \x\ + 1 on [a, b]

60 V
76 Show that Jo '/(/) dt + fofi-t) dt is a constant by taking the
derivative with respect tox. (Assume that/is continuous.) What
is the value of this constant?
62 V

In Problems 77 to 80, sketch graphs of/ and g.


64 V

f 1
- r for f < {' ,

^^^"' =
ll+r for,>0-^'-^' = I/^'»'''
66 V

1
- r for

67 Given that u and v are differentiable functions of .v and that /is r for

a continuous function, justify the equation


79 fit) = gt]\ for -3 < < 3, ^(.v) = fit) dt for -3 < < 3
—-
/ .v
d {^\. , „ dv du
f(t) dt = f(v) j(u
•'o

dx J dx dx
for f <
68 Complete the proof of Theorem 1 by attending to the following 1-2
details: (a) Prove that g'+{b) = fib) and that g'-{c) = fie). / for < < / 5, g{x) = fit) dt
J-i
(b) Prove that g is continuous on [b. c].
2/ for f > 5
69 Show that the fundamental theorem of calculus works even if

a > b; that is, show that if/ and g are continuous functions on 81 Assume that /is a differentiable function and that/' is continu-

the closed interval [b, a] and if g'(.v) = fix) for .v in the open = + fia).
ous. Explain why D, I /(/) dt D,fit) dt
interval (b, a), then J*/U) dx = g(b) - g(a).
338 CHAPTKR 5 THE DEFINITE INTEGRAL

5.5 Approximation of Definite Integrals


The Trapezoidal Rule and Simpson's Rule
The fundamental theorem of calculus enables us to determine the numerical value of
a definite integral J'',, fix) dx provided that we can find an antiderivative g of the
function/. However, in practical applications of calculus, it is sometimes difficult

or impossible to find an antiderivative g of/ such that i>(b) and g{a) can be calcu-
lated explicitly. In these cases, numerical methods of approximation can be used to
estimate the value of the definite integral to within acceptable error bounds.
In this section, we present some methods for estimating the value of a definite
integral /,';/(.v) dx by formulas that use the values of/t.v) at only a finite number of
points on the interval |a, b]. These methods involve only simple computations and
thus lend themselves well to the use of calculators or computers.
Perhaps the most obvious way to approximate the definite integral

/(.v)^.v
J

is to use its very definition as a limit of Riemann sums. Thus, we can select a
partition W with a small norm, augment fP to obtain .'f *, evaluate the corresponding
Riemann sum, and observe that

" rb
E/(q)Ajc,-J f(x)dx

Figure I Figure 1 illustrates how this procedure, with /i = 5, is used to estimate

Jo 1 + ^'

The areas of the five rectangles are shown in Figure 1 , and the sum of these areas,
which is approximately 0.834, provides an estimate for the integral above. The true
value of this integral, rounded off to four decimal places, is 0.7854, so the estimate

dx == 0.834
r

is not particularly good. To obtain a reasonably accurate estimate by this method,


we would have to partition the interval [0, 1] into a very large number of sub-
intervals.

The Trapezoidal Rule


Figure 1 makes it clear why the above estimation of

f 1

dx
1 +.v^
SECTION S.S APPROXIMATION OF DEFINITE INTEGRALS—THE TRAPEZOIDAL RULE AND SIMPSON'S RULE 339

is so crude — the five rectangles overhang the region under the graph of

1 + .V-

so that their total area is considerably larger than the desired area.
Figure 2 Evidently, a much more accurate estimate is obtained by adding the areas of the

five trapezoids shown in Figure 2. Each of these trapezoids has the same width,
namely, 5 unit. Since the area of a trapezoid is given by one-half the sum of the
lengths of its two parallel edges times the distance between these edges, the total

area of the five trapezoids in Figure 2 is

25 1
+ + lf\ T
—+ —+
,

1 1 1 1 41 2

5 5

-{ — + 1 25
+
25
+
25
+
25
+ — — 1 \ 1

2 26 29 34 41 4/5
The numerical value of this total area, rounded off to four decimal places, is

0.7837. Therefore, by this "trapezoidal method.""

f 1

dx « 0.7837
1 + .V-

The trapezoidal rule for estimating definite integrals in general is given by the
following theorem.

THEOREM 1 Trapezoidal Rule

Let the function /be defined and integrable on the closed interval [a, b]. For
each positive integer n. define

where

=
b-a
A.x and Vi =/(a + k A.v) for ^' = 0, 1,2,

Then
T„ - f(x) dx
J

The approximation becomes better and better as n increases in the sense that

lim T„ = /(.v) dx

PROOF Let ffn be the partition of [a, b] consisting of n subintervals of equal length

A =
^-^
A.V
n

Thus, the subintervals in ff^ are

[,Vo. -Vi], [.Vi, .v.], [X2, -Vj], . . . , [.V„_|, .v„]


340 CHAPTER 5 THE DEFINITE INTECRAL

where

xq = a x„ = b and x/, — x/,- 1


= Ajc

for A: = 1, 2 n (Figure 3). Evidently,

.V| = a + A.V, .V2 = a + 2 A.v, .Vj = a + 3 A.r, ... , x/, = a + k Ax. . . . ,

and x„ = a + n A.v = b
Figure 3
SECTION 5.5 APPROXIMATION OF DEFINITE INTEGRALS— THE TRAPEZOIDAL RULE AND SIMPSONS RULE 341

If/(.r) > for fl < .V < fe, then Theorem 1 admits a simple geometric explanation
since the quantity T„ represents the total area of all the trapezoids shown in Figure 3
(Problem 16).

E/m Examples 1 and 2, use the trapezoidal rule, 7",, = S^fM d.x, with the indicated
value of n to estimate the given definite integral.

EXAMPLE 1 V 1 + .x^ dx\ w = 4

SOLUTION Here [a, /;) = [0, 1] and « = 4, so

=
b- a
=
1-0 —
1
A.v =
n 4 4

Also,

Vk =/(0 + k Ax) = Vl + (k/4f

for A- = 0, 1, 2, 3, 4; hence,

^65
.vo Vl +0' = 1 V, = vTTW
V9T
>'2 = Vl + (if = Vl V3 = VTTW = -—— 8
ja = VTTP = V2
Therefore,

7"4 =
(
Y+ >'i + y^ + >'3 + -y) ^-^•

—+
1 V65
+ \ —+
[9 Vol Vim 1

2 V
8 8 8 2 / V 4 ^

== (0.50 + 1.01 + 1.06 + 1.19 + 0.71)(0.25) = (4.47)(0.25) == 1.12

Thus,

Vl + x^ dx=- 1.12
'n

EXAM PLE 2 :-; « = 5


A 1 + X-

SOLUTION Here

--;
A
A.V = —— = 5
'
—-
^

5
^
and v*
1 +(1 +
1

k/5)- 25 +
25
(5 + k)-

for A=0, I, 2, 3, 4, 5. Therefore,

T
I5 = fl
U 4_ 25 25
T^ 61 T^ 74
1 I 25 1 _25_
+ i _25_^ /Is.
1 89 1^ 106 250M5)
= (0.2500 + 0.4098 + 0.3378 + 0.2809 + 0.2358 + 0.1000)(0.2) == 0.323

hence,

dx
-=^0.323
r 1 1 + jc-
342 CHAPTKR 5 THE DEFINITK INTEGRAL

The following theorem, whose proof can be found in more advanced textbooks,
provides an upper bound for the error involved in using the trapezoidal rule.

THEOREM 2 Error Bound for Trapezoidal Rule

Suppose that/" is defined and continuous on \a. b] and that M is the maximum
value of |/"(.v)| for a in \a. b\. Then, if T„ is the approximation to Ju/U) dx
given by the trapezoidal rule.

- af
-r /(.v) d.x\ < M
ih

\2n-

I EXAMPLE 3 Use the trapezoidal rule to estimate Jf d\/.x with n = 6, and use
Theorem 2 to find a bound for the error of the estimate.

SOLUTION We have

2-1
Aa- = = — 1

and .V*
6 6 ] + k/6 6 + k

for k = 0. 1.2, 3, 4. 5, 6; hence,

V ~ 6 ~ 1 .> 1 ~ " .2 — 8 .13 — 3 >4 ~ 1 V5 — 1 1 Vf, — 1

Therefore.

n= {i + ^ + i + l + u> + Ti + M)
== (0.5 + 0.8571 + 0.75 + 0.6667 + 0.6 + 0.5455 + 0.25)(0.1667)
= 0.6950

so that

= 0,695

Here,

/(A) ./'(A) = and /"(a) = —


Since/" is a decreasing function on [1, 2], its maximum value on this interval is

taken on at the left endpoint 1. Hence, in Theorem 2,

M =/"(!) =-^ = 2

In Theorem 2 we also put b = 2. a = 1 . and n = 6, to conclude that the error in the

estimate above does not exceed

(/; - «)' 1' 1

M ^- = (2) =
12/r 432 216

Since ^< 0.005, the estimation /t dx/.x ~ 0.695 is correct to at least two decimal
places.
SECTION 5. 5 APPROXIMATION OF DEFINITE LNTEGR.ALS— THE TRAPEZOIDAL RULE AND SIMPSON'S RULE 343

Simpson's Rule
A third method for approximating the value of a definite integral is known as
Simpson's rule, or the parabolic rule, and is usually more efficient than either the
The method is based on the use of
direct use of the definition or the trapezoidal rule.
a number of adjacent regions having shown in Figure 4 to approximate the
the shape
area under the graph of/. (See Problem 33.) The result is the following theorem,
whose proof is analogous to the proof of Theorem 1 and is therefore omitted.

THEOREM 3 Simpson's Parabolic Rule

Let the function/be defined and integrable on the closed interval [a. b]. For each
Figure 4 positive integer n. define

: of a parabola
S2n = — A.V
( Vo + 4y, + 2y2 + 4y3 + 2y^ + + 2y2„-2 + 4j2«-i + ym)

, V = Ax^ +Bx + D
where

b- a
A.V = — 2n and v* = f(a + k Ax) for k = 0, \. 2, . . . , 2n

Then
Sin = fix) dx
J

with the approximation becoming better and better as n increases in the sense that

lim 52„ = fix) dx

min Examples 4 and 5. use Simpson's parabolic rule. S2„ ~ iaf(x) dx, to estimate
the definite integral, using the indicated value of n.

EXAMPLE 4 (1 +.V)"' dx; /! = 4

SOLUTION Here, the interval [0. 1] must be subdivided into 2n = 8 parts, each
of length Ajc = (1 - 0)/8 =i Thus,

y, =f(a + k Ax) =/(o + Aj = j^i + M

y6 = (i+f)-'=fi .V7 = (l + 3-8= (1 +!)"

Therefore,

Sm = 58 = —
Ax
( Vo + 4y, + 2v2 + 4v3 + 2y4 + 4ys + ly^ + 4y^ + yg)

— 24(1 -^9 +IO+II+T2+T3 + T4 + + 2)


T5
= ^(1 + 3.5556 + 1.6000 + 2.9091 + 1.3333 + 2.4615
+ 1.1429 + 2.1333 + 0.5000)
= A(16.6357) = 0.6932
344 CHAPrKR 5 THE DEFINITE INTEGRAL

Consequently, /,', (1 + jt) '


ctv ~ 0.6932. Incidentally, the correct value, rounded
off to five decimal places, is J,', (1 + .v)"' dr = 0.69315.

ri

EXAMPLE 5 Vl - x^ dx; n = 3
'o
SECTION 5.5 APPROXIMATION OF DEFINITE INTEGRALS—THE TRAPEZOIDAL RULE AND SIMPSONS RLLE 345

value of « satisfying the latter inequality is n = 2. Thus, we estimate /f dx/x. using


5-.„ = 54. We have

A.v =

Vo = 1

Hence,

^2n I2U T^ 5 ' 6 T^ 7 ^ 1) — 2520

It follows that /i dx/x ~ 552^ with an error not exceeding jifeo- In fact, if we round
off to five decimal places, =
0.69325, while the correct value of /^ dx/x.
MI5
rounded off to five decimal places, is 0.69315.

Theorem 4 has an interesting consequence: Simpson's parabolic rule gives


the exact value of fafW ^x iffis a polynomial function of degree not exceeding 3.
The reason for this is just that for such a polynomial function, /'^' is the zero
function; hence, in Theorem 4, N= 0. Thus, taking /! = 1 in Theorem 3, we obtain
the following theorem.

THEOREM 5 Prismoidal Formula

Let /be any polynomial function whose degree does not exceed 3. Then

b
fix) dx = /(«) + 4/(^4^) +f(b)

EXAMPLE 7 Use the prismoidal formula to evaluate Js (.v"* + \) dx.

SOLUTION (a:^ + l)rf.v = |[(0^ + 1) + 4(1' + 1) + (2' + D] = 6

Problem Set 5.5

lln Problems 1 to 12, use the trapezoidal rule, T„ = /a/(.x) dx. with
= =
the indicated value of n to estimate the integral.
7
I
(4 + .v-)"'/Vv; n 6
f VI + .V- dx: 11 1

r- dx r " sin .V

/ •
« 10 dx: H = 4
Jo 1 + -V- J, X J, .vVl + .V K'^ X

^^
3 ; /! = 6 4 \ 9 - .V- ±x\ n = 6 tan .V dx: n = 4 dx: n = 6
4 1 + .V Jo

5 : n = 5 6 7;/! = 4 ] 13 Use Theorem 2 to find an error bound for the estimate in Prob-
,
,
Jq Vl + .r-*
Jq 1 +^ lem 1.
346 CHAPTER 5 THE DEFINITE INTEGRAL

El 14 Use Theorem 2 to find an error bound for the estimate in Prob- estimate /i 5 dx/x. Give an upper bound for the error of

lem 2. estimation.

15 Suppose that/U) a for all x'm[a, b] and that the graph of/ EI29 On geometric grounds, 4 Jo Vl - x" dx = n Why?
. Using
Simp son's p arabolic rule, S2,,~ /i "V/l - x^ dx. to estimate
is concave downward on [a, b]. Explain geometrically why
/o Vl - x^ dx, give an estimate for tt. Use n = 2.
T„ s St fix) dx in this case.
030 Use the procedure of Problem 29 to estimate tt taking n = 5.
16 U fix) ^ ioT a £X^ b. show that the quantity T„ in Theo- Compare the result with the correct value of tt. which is

rem represents the total area of all the trapezoids in Figure 3.


1
3.14159, . . .

31 (a) Use the prismoidal formula to prove that


Elln Problems 17 to 26, use Simpson's parabolic rule, 52,,=
S'^ifix) dx. with the indicated value of n to estimate the integral.
(Ar* + Bx- + Cx + D) dx = -^ {2Ba- + 6D)

X + \ dx; n = 4 (b) Prove the formula in part (a) directly using the fundamental
theorem of calculus.

32 Prove the prismoidal formula directly using the fundamental the-

orem of calculus.

33 Prove that by a suitable choice of the three coefficients A. B.


and D, the graph of v = Ax- + Bx + D can be made to pass
through any three points of the form (c. p), (c A.v, q). and -I-

(( -t- 2 A.V, r) (Figure 4).

034 A loaded freighter is anchored in still water. At water level, the

cos X dx; n = 3 boat is 200 feet long and for each A- = 0, 1, 2 20 has
breadth V; at a distance 10/t feet from the bow. Assume that

Vo = and Vio = 0. (a) Use Simpson's parabolic rule to write a

EI27 Find the smallest value of n for which the error involved in the formula giving the approximate area of the water-level section of
estimation 52„ = /i dx/x does not exceed 0.0001. (Use Theo- the boat, (b) Recalling an exploit of Archimedes, write a for-

rem 4.) mula for the approximate number of tons of freight that should
be removed to raise the level of the boat by foot. (Assume that 1

E)28 Use Simpson's parabolic rule, S2„ ~ it fix) dx. with /i = 1 to the water weighs 64 pounds per cubic foot.)

Figure 1
5.6 Areas of Regions in the Plane

We already used the definite integral to calculate areas of regions under the graphs
of continuous functions. In this section, we develop methods for using the definite

integral to find areas ofmore general regions in the plane.


Suppose that/is a continuous function on [a. h] and that/(.v) < for a < .r < fo.
y=-m Thus, the graph of y =/(-v) lies under the .v axis between .v = a and x = b (Fig-
ure 1). Let A denote the area of the region under the .v axis and above the graph of
y = fix) between .v = a and x = b. Notice that the graph of v = -fix) is the mirror
image above the x axis of the graph of y =/(.v); hence, A is also the area under the
graph of y = —fix) between x = a and .v = b. Therefore,
= Iix)
SECTION 5.6 AREAS OF REGIONS IN THE PLANE 347

Figure 2 More generally, suppose that /is a continuous function on [a, b\ and that /(.v)

takes on both positive and negative values for a <x< /? (Figure 2). If you wish to
calculate the area A of the region between the graph off and the x axis from x = aio
X = b, you can begin by dividing the interval («, /;] into subintervals over which/
is either positive or negative. If [c, d] is a subinterval of [a, b] and/is positive over
(c, d), then

/(.v) dx

gives the area under the graph of/ between c and d\ however, if/ is negative over
(c, d), then the area above /between c and d is given by
the graph of

Ax)dx
-J

The total area A is obtained by adding the areas corresponding to all such sub-
intervals.

EXAMPLE Find 1 the area A between the graph of /(.v) = iv' and the x axis from
= -I to.v = 2.
-v

Figure 3 SOLUTION A sketch of the graph of/ (Figure 3) shows that it falls below the x
axis on the interval [—1, 0]. Therefore, we cannot find A simply by calculating

//(A) = 1.V3
f " iv^ dx

since the area below the .v axis provides a negative contribution to this integral.
However, by splitting the interval [— 1 , 2] into two subintervals. we obtain the area
A as follows:

r° r- 1° I-
A = - ix' dx + ix' dx = -[t^.v^] + [t^.v^]
'-1 •'o 1-1 '

= A + H=H square units

Areas by Slicing Using the Definite Integral


The method of slicing, introduced in Section 4.7, enables us to find areas of regions
by solving a differential equation of the form dA = I ds. We now show how such
areas can be calculated by using the definite integral.
In this section we consider only regions R in the plane that satisfy the following
two conditions:
Figure 4
1 The boundary of R consists of a finite number of straight line segments or

smooth arcs which can meet in a finite number of "comers," or "vertices."


2 The region R is bounded in the sense that there is an upper bound to the
distances between the points of/?.

A region R satisfying conditions and 2 will be called an admissible region. The


1

region R shown in Figure 4 is an example of an admissible region. Notice that such


a region is permitted to have a finite number of "holes," provided that the bound-
aries of these holes satisfy condition 1. The boundaries of the holes, if any, are
regarded as part of the boundary of R.
348 CHAPTER 5 THE DEFINITE INTEGRAL

We assume that any admissible region R in the plane has an area A square units
associated with it. The following theorem shows how to calculate A in terms of a
definite integral.

THEOREM 1 Areas by Slicing Using the Dennite Integral

Let R be an admissible region, and choo.se a convenient reference axis called,


say, the .v axis (Figure 5a). At each point along the s axis, erect a perpendicular
line, and suppose that the region R is entirely contained between the two perpen-
diculars at .V = a and s = b. a < b. Let the perpendicular at i intersect R in one
or more line segments of total length lis). Then the area A of the region R is given
by rb
A= \ lis) ds

Figure 5

Figure 6
SECTION 5.6 AREAS OF REGIONS IN THE PLANE 349

SOLUTION We use Theorem 1 , taking the y axis as the reference axis. To deter-
mine the points of intersection of the two graphs, we solve the two equations
y~ = Iv and y = x — 4 simultaneously. Substituting y = a - 4 into y" = 2x, we
have

{.V - 4)- = 2y

.V- - lO.v +16 =


Factoring to solve the latter equation, we obtain

(X - 2)(x - 8) = so .V = 2 or .v = 8

When -V = 2, then

y = .V -4 = 2 - 4 = -2
When .V = 8, then

y = .r -4=8-4 = 4
Therefore, the two graphs meet at (2, -2) and at (8, 4).

The point on the graph of y" = 2x with ordinate y has abscissa .v = y"/2, and the
point on the graph of y = .v - 4 with ordinate y has abscissa .v = y + 4 (Figure 6).

Therefore,

= - -2 < <4
/(y) (y + 4)
Y for V

By Theorem 1,

A = /(y) dy = V + 4 dv
J J


V 2 • 6 / _

= ^ — (—^ = ) 18 square units

The Area Between the Graphs of Two Functions


Figure 7 Let's consider the problem of finding the area A of an admissible region bounded
above by the graph of y =/(.v), below by the graph of y = g{x), on the left by the
vertical line x = o, and on the right by the vertical line x = b (Figure 7). If we take
the X axis as the reference axis, then /(.v) is the distance between the points (.v,/(x))
and (x, g(.v)); hence.

lix) = fix) - g{x)

Therefore, by Theorem

-[ [fix) - g(x)] dx

In Examples 3 to 5, find the area A of the indicated region.

EXAMPLE 3 The region between the graphs of y = .v + 2 and y = x'


350 ( HAPTKR 5 THK DEFINITE INTEGRAL

Figure 8 SOLUTION To determine the points of intersection of the two graphs, we solve
the two equations y = x + 2 and _v = -v" simultaneously and find that the intersec-
tion points are

(-1.1) and (2. 4)

(Figure 8). Notice that the graph of v = .r + 2 lies above the graph of _v = x^ be-
tween X = -\ and .V = 2. Therefore, using the formula above with/(.v) = x + 2 and
g{x) = x~, we find that

-r, |(.v + 2) - .v-| dx = (4—4)


= 10 /
I
7 \
1
= —9 square units

Figure 9

EXAMPLE 4 The region bounded by y = .v and y = .v'.

SOLUTION The two graphs are sketched in Figure 9. By symmetrN. the portion
of the region in quadrant III has the same area as the portion in quadrant 1. There-
fore, the area A of the entire region is twice the area of the portion in quadrant I.

Consequently,

r(.v-.vv.v=2(f-:^)l' =2(1-0)4 square unit

EXAMPLE 5 The region bounded above by y = cos .v, below by y = sin .v. on the
left by .V = 0, and on the right by .v = 7r/4 (Figure 10).

(cos X — sin x) dx = (sin .v -I- cos a)


^
Figure 10 Vi V2\ r-
(
(— - + ^-(0+l) = V2- 1 square unit

Areas Bounded by Graphs of "Piecewise-Defined" Functions


The following example illustrates the method for finding the area between the graph
of a "piecewise-defined" function and the .v axis.

EXAMPLE 6

(a) Find the area between the graph of y = /(.t) and the x axis from a- -2 to

X = 5, where/ is the function defined by

2 -^ ^- if A <
/(.v)
if <A< 3

1 16 - 4a if 3 <A

(b) Find fix) dx.


/:
SECTION 5.6 AREAS OF REGIONS IN THE PLANE 351

Figure 11

(a) Figure 1 1 shows a sketch of the graph of/ with the desired area A shaded.
From the figure, we see that

A=\J-2 [2 + ~)dx-\Jq {.x--.x-2)dx+ \ (.V- Y - 2) ^.v


^
4 J2

+ I (16- 4.V) dx -
(\6-4x)d.x- I (16 - 4x)dx

2.V + lx]\ +{ 2x)\


V 16/ 1-2 ^ 3 2 / lo ^ 3 2 / U
14

+ (16.v- Iv-) - (16.\- Zv-)

— 3 -t
—— ——
10
- I-
11
- t-
2 + 2 = 73
square units
3 6 6

(b) [md.x = £(2 + ^)^v + [\.v^-.v- 2) d.x

+ I {l6-4.x)dx

Iv + (16.V - 2v-)
V 16^ L, V 3

= 3 —3.3
2
+ =—
7

Again, in the example above, notice that

fix) d.x = —
f.
does not give the area between the x axis and the graph of y = f(x) from .v = -2 to
.V = 5 because the area below the x axis provides a negative contribution to the
intearal.

Problem Set 5.6

In Problems 1 to 14. sketch the graph of each function, and find the 8 g(x) = x' - 6a- + 8a; a = 0, ^ = 4
area between the graph and the a axis from v = o to x = b.
9 /(.v) = i(A - .r'); a = -l.b = 2

1 /(.v) = 1 - .V-; a= -\.b= \


10 /(A) = a"; a = 0. fo = 1. where n > 1

= - l:a = =
2 g(.v) X- Q. b \
11 /(a) = sin Zx: a = 0. b = —
3 h(x) =.v'-.r; a = -1. fo= i

4 f (A) = A- - 9: a = -3. fc = 3 12 g(x) = cos —-; a = 0. fc = 77-

5 G(A) = A-': a= -l.b = l

= - + = = 13 /i(a) = T
sec" a; a = II

. b = — 41

6 //(a) a- 6a 5: fl 1 . /) 3 4 4

7 /(A) = A-'' - 4a- + 3a; a = 0, i) = 2 14 F(.r) = sin a cos a; a = — tt, fc = tt


352 CHAPTER S THE DEFINITE INTEGRAL

In Problems 15 and 16. find the area A of the shaded region in the 29 X = 6v- - 3 and x + 3.v =
indicated figure in two ways; (a) by using the v axis as reference axis
30 X = 4.V- - 1 and 8.v - 6y + 3 =
and (b) by using the y axis as reference axis.
31 /(.t) = x^ and g(x) = V^
15
32 y = 2x' - .r' and y = Ix — x'

33 y = -v" and y = x*

34 y = |.v| and y = x*

35 -v = y' — 2 and .v = 6 - y"

36 X = v" — y and x =y - y"

37 fix) = x\x\ and g(.r) = .v'

38 fix) = -X and gix) = Iv - 3.t-

39 fix) = cos .X and ^(.v) = 1


- cos .v for —— < j: < —
16

40 fix) = 3 - |.v| and gix) = sec" —


In Problems 41 to 44, find the area of the region bounded by the
graphs of the given equations.

41 y = .V + 6, y = iv", .v = 1 . x = 4

42 y = A-\ y = 12 - .v". .v =

43 y = 2 - .V, y = .V", and above y = Vx

44 y = .v\ y = 0, and y = 3.y - 2 (first quadrant)

In Problems 45 to 50. sketch a graph of the function/, find the area


In Problems 17 to 40, (a) sketch the graphs of the two equations, (b)
between the graph of f and the v axis from v = a to x = b. and find
find the points of intersection of the two graphs, and (c) find the area
ft fix) dx.
of the region bounded by the two graphs.
for -2 <x< 1

17 f(x) = X- and gix) = Ix + i


V.v
45 fix)

18 V = ^ 4
and 7.v - 2v = 20

19 fix) = -X- - 4 and g{x) = -8 46 fix)

20 y = V^ and y =x

21 y = X' — X and y = x
47 fix)
22 fix) = x^ and g(x) =x

23 fix) = -X' + 4x and ^(.r) = .t"

24 fix) = jc- + I and gix) = 4x + \ 48 fix) =

25 fix) = I Da- - .1- and g(.v) = 3.v


-

26 fix) = X- - 8.t and gix) = -.r'

27 jf = ( y - 2)" and x = y 49 fix)

28 v^ = 3-t and v =x
REVIEW PROBLEM SET 353

—— r for -3 < .V <


51 Use the method of slicing to find the area of the triangle whose
vertices are the points (1, 1), (2, 4), and
(.V- + 1 )- (4, 3).

50 /(.v) \^ for < .V < 4 ; -3,b 52 Use the method of slicing to find the area of the trapezoid whose
A- - 111- + 32 for 4 <.v£ 10
vertices are the points (1, 1), (6, 1), (2, 4), and (5, 4).
5 for 10 < .V < 12

Review Problem Set, Chapter 5


In Problems 1 to 6, write out each sum explicitly and then find its 16 (a) Suppose that / is an integrable function on the interval
numerical value. [0, 1]. Use the definition of the definite integral to express

1 S (5k + 3) 2 2 5(/ + 1
lim 2/(
as a certain integral.
4

3S k+ +
1
;=0 i- 1
(b) Write lim 2^ —^ as a definite integral.

5 Zj cos — 6 Zj 'an
kir
In Problems 17 to 20, find the Riemann sum corresponding to each
augmented partition for the indicated function, sketch the graph
of the function, and show the Riemann sum as a sum of areas of
In Problems 7 to 10. evaluate the sum by using the basic properties
rectangles.
of summation.
17 f(x) = a' - .V, ,9^ is the partition of [1, 3] into four equal sub-

7 S k(2k- 1) 8 S (6-'*' - 6^) intervals with Ci , C2, C}, and c^ chosen so that all approximat-
7=1 ing rectangles are inscribed

18 f{x) = .V- + 1 , ^4 is the partition of [ 1 . 4] into four equal sub-


9 E (3^ + 3^+') 10 S (k + D' intervals with C| , Ci , Cj , and C4 chosen at the midpoints of these
*=o subintervals

19 Same as Problem 17. except that all approximating rectangles


11 Find the sum of the first 1000 even integers:
are to be circumscribed.

2 + 4 + 6+8+ ••• +2000 20 fix) = 16 — -t", fl'4 is the partition of [0, 4] into four equal

12 Use the principle of mathematical induction to show that


subintervals with C[, o, Cj, and C4 chosen so that all approxi-
mating rectangles are circumscribed
El, ak- \) = n-.

In Problems 21 to 24, evaluate each definite integral directly by


13 For doing a certain job, you are offered $0.01 the first day,
$0.03 the second day, $0.09 the third day, $0.27 the fourth
calculating a limit of Riemann sums. Use partitions consisting of
subintervals of equal lengths, and augment these partitions by using
day, and so on, so that your daily earnings are tripled each day.
left or right endpoints of the subintervals as indicated.
(a) Using the S notation, write an expression for your total
earnings after 14 days of work, (b) Use the basic properties of
summation to evaluate the sum in part (a), and write the answer 21 (.V- + I) dx (left endpoints)
-'1

in dollars.

22 (.v' + X) dx (left endpoints)


14 Evaluate Zi -^ Hint: — ^
'1
r^, k- + k V k- + k

15 Evaluate the Riemann sum


23 (.V" + I) dx (right endpoints)
2* = i A^k) ^xi, corresponding to -"i

the augmented partition [0, i], [i, i], [i, f], [f, 1] with c, = i,
Q = 8, cj = i, and C4 = I if/is given by/(A:) = x'. Interpret 24 I (.V- - 2r + 3) dx (right endpoints)
this Riemann sum as an approximation to a certain area.
354 IHAPIKR 5 THK DEKINITK INTE(;RAL

In Problems 25
and give a reason for your answer.
to 32, indicate whether each definite integral exists
47
r (4 - 3 cos- .x) dx

'

1 + fM for I < ,t £ 3
26 48 ix) dx. where Hix) =
h 11. vn .i;(.v) - 1 for 3 < .V s 5

Vl -x- 49 True or false'.'


27 dx 28
'1 Jo \ +X-
f" =
(a) fix) dx fix) dx fix dx )

,„ ("'' d-x -'i


29
J() cos X
(b) f;ix)dx-j ^M.v)(/.v=J ,i;(.v) dx
:.v- for X < J
30 /(,v) tlx. where /(.v)
-' Lv-* for X a
I

50 Suppose that / is an integrable function on \a. h\ and that


|/(.v)| SK holds for all values of .v in \u. h\. where K is a
31
J
cot
( V (/.V 32
J„
CSC —2 rf.v
constant. Prove that |/';/(.v) (/.v| s K\b ~ a\.

In Problems 33 and 34. use the indicated previous result to evaluate


In Problems 51 to 55, without evaluating the integrals involved,

the definite intecral.


show that the inequality holds.

33 + 1) dx (see Problem 21) .V dx s .f d\


Jo 1 + -v- Jo I + J

+ dx (see Problem 22) 53 X dx


v)
['•"-I
(-"/'
^ )-' sin.v
35 Evaluate cot v dx. 54 cos .V dx s ^.v
4/6 4/6 X

36 Is tan v dx defined? If so, what is its value? 55 sin .V (/.v cos .V dx


4/:
56 Assume that /and g are continuous functions on \a, b] and that
37 h fix) = \\\\] continuous on |0, 3|? Does | J\x) dx exist? A" is a constant. Show that

Why or why not?


2ArJ ,/U),i;(,v)(y.v<
/(.v),i;(,v) (/v < |/(.v)l- ^/.v + A.-- J l^'(.v)]- civ
J J
38 By using the definition of the definite integral, prove that if K
{"
is a constant, then K dx = K(b - a).
iHinl: 0£ [/(.v) - A'.i;(.v)l-.)

57 Verify the inequality in Problem 51 by using the fundamental


theorem of calculus to evaluate the integrals.
In Problems 39 to 48, assume that /i/(.v) dx = 6, J:,/(.v) dx = 7.

/a gix) dx = 8, and J,"^" cos' .v dx = 7t/4. Use the properties of 58 Use Problem 56 to derive the Cauchy-Bunyakovski-Schwarz
definite integrals to evaluate each expression. inequality: If/and g are continuous functions on [a. b], then

39 (-5)/(.v)(iv 40 7/(.v) dx
•'i
I /(.V)^i,'(.V)(/v| sj |/(.v)|-(/.v Li,'(.v)|- (/.V
I
J

41 |/(.v) + dx 42 - dx [Hini: First take care of the case in which JJ; |,i,'(.v)l" dx = 0.
3«(.v)l \4kIx) 2/(.v)|
Then, assuming that /fj [;?(.v)|- dx 9^ 0, let K in Problem 56 be
the ratio of J,';/{.v),?(.v) dx to /,'; \f;{.x)]- dx.\
43 j\x)dx 44 COS" X dx
In Problems 59 to 64, find the mean value of the function /on the

4/(.v) for 1 < A- < 3 indicated interval, and find a value of c in the interval tor which/(c')
45 /!(.v) dx. where /i(.v) =
I
-2s(,v) {ot3£x^5 is equal to this mean value.

59 f{.x) = 3.V + 1 on 1 0, 4
for v = 1

46 F(.x) dx. where Fix) = < < 60 fix) =x- + 2 on |(), 3]


f
I
{ fix)

52
for

for .V
1

=
.v

3
3

61 fix) = 4 - A- on 10. 2]
REVIEW PROBLEM SET 355

62 /U) =4 + 3.v-on [-1. 1]

63 /(.v) = |.v| on [-1. 3)

64 /(.v) =Ai.v| on l-h. 1|

In Problems 65 to 88. use the fundamental theorem of calculus and


the properties of definite integrals to evaluate the integral.

65
5 (4.r + 3) dx 66 3y- ^:
I
'-4

67 6\Gdx 68
'(1 h

69 (2r + 3/-) (// 70 I


(3m - \)Ur + \) du
J -'n

71 I {:- + 2)- dz 5(.v - \ ^v)- (iv


-'n

73 (.V + |.t|) dx 74
-'-I

75
J
V7T1 dt

77
(1 +.V-)-

79 r,,.
I
(|.v- ll + |.v-2|)dLr

81 sin 4.V dx

83 sin ^ cos \' d\


I

85 Isin .vl dx
356 CHAPTKR 5 THE DEFINITE INTEGRAL

06 fix)g{.x) dx = [f(x)h(x)\\ - /i(.v)/'(.v) dx, where 118 \''\6-x-dx:n = 2


J J J
/i'(.v) = g(x)

07 I f(x) dx =0, if/ is an odd function

dx
120 /I = 3
08 f(kt) dt ^— fit) di. where k IS a constant V 1 + .v'

121 -;« = 3
09 J g{x)dx\ \g{x)\dx
I =J
rV^
10 If / is a decreasing function, then the function g defined by 122 cos iv; « = 4
J
g{x) = fl)fit) di is also decreasing.

In Problems 123 to 134. (a) sketch the graphs of the equations, (b)
n Problems 111 to 116, use the trapezoidal rule. T„ ~ fa fix) dx. find the points of intersection of these graphs, and (c) find the area
with the indicated value of n to estimate the integral. of the region bounded by these graphs.

11 .vVl6 - .v' dx: n = 4 123 v = iv' and v = x. .r >


^
124 y = -t'. the y axis, and y = -27
12 V'4 + .v' dx: « = 4 125 y = 9 - X- and y = x-
'i

10 , 126 2y- + 9.V = 36 and 3.v + 2y =


f
13 Vl25 +ji' dx: /I = 5
'n 127 y = 2v- and y = .v" + It + 3

128 y= = -4(.v - 1) and y- = -2(.x - 2)


14 V64 - .V- dx:n = S

129 Iv + 3y + 1 = and .V + 3 = y -{ 1
)-

15 sec .V dx: n = 6 130 .v-y = A- - 1 . y = 1 . .r = 1 . and .v = 4


^
131 y = 4 - .V- and y = 4 - 4.v

sin' .V iv: n = 7 = - = -
132 X iy- 1 and y 4.v 16

133 y' = 9x, y- = -3(.v - 6), and y = -3


E In Problems 117 to 122, use Simpson's parabolic rule, Sz,,
~
Saf(x) dx. with the indicated value of n to estimate the integral.
134 y- = - 16(.v - 1 ) and y- = ¥ (.v + 3)

r 3.t
135 Find the area of the region bounded by the curve y =-
117 ^dx: n = 4 A-t'
L 1 + .v'
and the tangent to this curve at (3, -2).
APPLICATIONS OF THE
DEFINITE INTEGRAL

In Chapter 5 we used definite integrals to find the area of plane regions. Here we
apply definite integrals to the problem of finding the volume of three-dimensional
regions. In addition, we consider applications of definite integrals to the calculation
of arc length, surface area, consumer's surplus, blood flow, work, and energy.

6.1 Volumes of Solids of Revolution


Figure I The techniques used in Section 5.6 can be modified to express the volume of a
three-dimensional region S as an integral. We assume that any three-dimensional
region S which has a "reasonable shape" has a definite volume V{S) cubic units
associated with it. We refer to such a region as an admissible three-dimensional
region, or simply as a solid. In particular, any three-dimensional region having the
following two properties is a solid:

1 The boundary of S consists of a finite number of smooth surfaces that can


intersect in a finite number of edges. These edges, in turn, can intersect in a
finite number of vertices.

2 5 is bounded in the sense that there is an upper bound to the distances


between the points of S.

For instance, a solid spherical ball, a solid right circular cone, a solid cube, or the
Figure 2 base
solid region between two coaxial right circular cylinders would satisfy the above
conditions (Figure 1).

A solid consisting of all points lying between an admissible plane region Bi and a
second admissible plane region St obtained by parallel translation of 6i is called a
solid cylinder with bases Bi and B2 (Figure 2). All the line segments joining points
on the base Bi to corresponding points on the base Bt are parallel to one another. If

all these line segments are perpendicular to the bases, then the solid cylinder is

357
358 t HAri ER 6 APPLICATIONS OF THE DEFINITE INTEGRAL

Figure 3 called a solid right cylinder (Figure 3). The distance, measured perpendicularly,
between the two bases of a sohd cylinder is called its height. Henceforth, we
assume that the volume of a solid right cylinder is its height times the area of one of
its bases.
If one solid Si is contained in a similar but slightly larger solid S2, then the

three-dimensional region S^ consisting of all points in 5^ that are not in Si is some-


times called a shell (Figure 4). Notice that

solid nglu cylinder V(S,) = V(S2)- V(S,)

that is. the volume of the shell is the difference between the volumes of the larger
Figure 4 and the smaller solids. For instance, in Figure 4, the volume of the right cylindrical
shell is given by
inner cylinder 5|

V(Si) = ViS.) - V(Si) = TTrlh - Tir-ih

The Method of Circular Disks


In this section and the next we develop methods for finding the volumes of solids
called solids of revolution. Solids of revolution are formed as follows: Let R be an
admissible plane region, and let / be a line lying in the same plane as R but touching
R. if at all, only at boundary points of R (Figure 5a). The solid S "swept out," or
generated, when R is revolved about the line / as an axis is called a solid of revolu-
tion (Figure 5b).
Consider the special case in which R is the region under the graph of a continuous
nonnegative function /between .v = a and .v = b (Figure 6a). Denote by S the solid
of revolution generated by revolving/? about the .v axis (Figure 6b). Figure 6c shows
outer cylinder 52 an " infinitesimal" portion dV of the volume V of S consisting of a circular disk of
infinitesimal thickness d\. perpendicular to the axis of revolution and intersecting it

at the point with coordinate .v. Evidently, the radius r of this disk is given by
r=/(.v).
Figure 5 The disk in Figure 6c can be thought of as a solid right
cylinder whose base is a circle of radius r and whose height is

dx. The area of its base is vr--. hence, its volume dV is given
by

dV = irr- d.x = 7r[/(.v)]- dx

The total volume V of the solid S should be obtained by


'summing" — that is to say. integrating — all the infini-

tesimal volumes dV of such disks as v runs from a to b.


(a)

Figure 6

i-'" T\

1^^ (b)
SECTION 6.1 VOLUMES OF SOLIDS OF REVOLUTION 359

Therefore, we should have

V= dV = j TT[f{x)]- dx=7rj [fix)]- dx


J

The calculation of volumes of solids of revolution about the .v axis by the formula

V = 77- [/(.v)]- dx
J

The shape of a solid of revolution is

found in these elevators

Figure 7
360 CHAPTER 6 APPI.K ATIONS OF THE DEFINITE INTEGRAL

Figure 9 y a-^ y ^ b (Figure 9a). Figure 9b shows the sohd of revolution S generated by
revolving R about the y axis. In Figure 9b,

dV = -nr- dy = 7r|^(y)j" dy

1^ hence,

Use of the formula


V=
J
JV =
J
77lg( v)l- dy = 77
J
[^( v)]^ dy

V= 77 |,i;(.v)l-</v

to calculate the volume of a solid of revolution about the y axis is also called the
method of circular disks.

EXAMPLE 3 Find the volume of the solid S generated by revolving the region R
bounded by the y axis, the line y = 4, and the graph of y = a" with .v s about the
y axis. Use the method of circular disks, and sketch both R and 5.

SOLUTION Solving the equation y = .v" for .v in terms of y and using the fact that
> 0, we have x = Vy (Figure 10). By the method of circular disks.
.t

V= 77 I {\y)~ dy =77 y dy = 77 = 877 cubic units

Figure 10

The Method of Circular Rings


Volumes of solids of revolution more general than those considered above can be
found by using the method of circular rings. This method works as follows: Sup-
pose that /and g are nonnegative continuous functions on the interval [a, b] such
that/(jc) > g(x) holds for all values of .v in [a. b], and let R be the plane region
bounded by the graphs of /and between .v = a and x = b (Figure 11a). Let S be
^i;

the solid generated by revolving R about the .v axis (Figure lb). Here we consider 1

an infinitesimal portion dV of the volume V of 5 consisting of a circular ring, or


washer, of infinitesimal thickness dx, perpendicular to the axis of revolution and
centered at the point with coordinate .v. The base of this circular ring is the region

Figure II
SECTION 6.1 VOLUMES OF SOLIDS OF REVOLUTION 361

between two concentric circles of radii/(.v) and g(.x) (Figure lie); hence, the area of
this base is v[fix)]- - TT[g{.x)]- square units. It follows that

dV = {nlfix)]- - 77[g(.x)]-} dx

Therefore,

V= j
dV=j {TTlf(.x)]--Tr[g{x)]'-}dx=Trj {[f(.x)]--lg{.x)]^}dx

Figure 12 Thus, we have the formula

V=7TJ {[fix)]- - [g(x)]-} dx

for finding the volume of a solid of revolution about the x axis (Figure 11 ) by the
method of circular rings.

EXAMPLE 4 Using the method of circular rings, find the volume V of the solid S
generated by revolving the region R about the .v axis, where R is bounded by the
curves y = x- and y = .r + 2.

SOLUTION The points of intersection of the two curves are (2, 4) and (-1, 1)
(Figure 12a). By the method of circular rings (Figure 12b), we have

V= TT
j
[(x + 2)- - (.V-)-] dx = tt\ {x- + 4.r + 4 - x*) dx

''-'=-->-T)l>4^-(-f)]
3

7277
cubic units

Naturally, the method of circular rings works for solids 5 generated by revolving
plane regions R about the y axis rather than the x axis. Thus, in Figure 1 3a, the plane
region R bounded on the right by the graph ofx = F(y), on the left by the graph
is

of X = G( y), above by y = b, and below by y = a. where a < i>. If ^ is revolved


about the y axis, then a solid of revolution S is generated (Figure 13b). The circular

Figure 13
362 CHAPIKR 6 APPLICATIONS OF THE DEFINITE INTEGRAL

Figure 14
SECTION 6.1 VOLUMES OF SOLIDS OF REVOLUTION 363

Figure 15

(4. -4)
I

axis of revolution

(a)

EXAMPLE 7 In Figure 16, the curve OP has the equation v = x^. Find the volume
of the solid of revolution generated by revolving the region

(a) OBP about the line y = 8

(b) OAP about the line x = 2

(c) OAP about the line v = 8

(a) When the region OBP is revolved about the axis y = 8, the infinitesimal
rectangle of height 8 - x^ and width ^.v shown in Figure 17 sweeps out a
circular disk of thickness dx and radius 8 - .v'. Its volume is given by

t/V = 77(8 - x^f dx = 7r(64 - 16.v' + .v*) dx

Therefore,

V= dV = \
77(64 - 16.v^ + x^) dx
-'.v=o 'n

= 77-I64.V 4.v^

Figure 18 57677
cubic units
/

(b) When the region OAP is revolved about the axis = 2, the infinitesimal
.v

rectangle of height dy and length 2 - Vj shown in Figure 18 sweeps out a


circular disk of thickness dy and radius 2 - \/y. Its volume is given by

^= 77(2 - Vv )2 dy = 77(4 - 4y'/^ + >•-/-')


dy

Therefore,

V= \
^= (
77(4 - 4y'/3 + r/3) dy

3
= 77 (^4y- 3//^ +-

= —1677
-— cubic units
364 t HAFIER 6 APPLICATIONS OF THE DEFINITE INTEGRAL

Figure 19 (c) When the region OAP is revolved about the axis v = 8. the infinitesimal

axis of rectangle of height .v' and width dx shown in Figure 19 sweeps out a circular
' = 8 P revolution
ring of thickness dx with inside radius 8 - .v\ outside radius 8, and intlnitesi-
mal volume

dV = 7r|8- - (8 - .v')-] dx = -n-(16.v-' - .r") dx

Therefore,

V= dV= \
77-(16.v' - x^) dx
-' V ^n •'f 1

32077
= 77 4.r'* cubic units

Problem Set 6.1

In Problems 1 to 8. find the volume of the solid generated by revolv- 21 y = x^ , X = 2, and the x axis about the y axis
ing the region under the graph of each function over the indicated
22 y = 2x, y = x, and x -¥ y = 6 about the x axis
interval about the .v axis.

23 y" = Ax + 16, and the y axis about the y axis


1/U) = 3.r;[-1,3] 2 gU) = 3\^;[l,4] = = =
24 y ix, y .v, and .v + y 8 about the y axis

3 hlx) = V9 - .v^; [-1.3] 4 G{x) = \x\; [-2, 1]


25 y = x- and y' = .v about the line jc = -
5 F(x) = V2 + X-; [0, 2] 6/(.x) = |.v|-.v;[-3. 2]
26 y = .v\ X = 0, and y = 8 about the line y = 8

7 g{x) = sec .v; 8 fix) = tan v; 0, 27 y = 4.V - X' and y =x about the line x = i

28 y = x^ - X and y = 3 - jc" about the line y = 4

In Problems 9 to volume of the


16, find the solid generated by 29 y = cos X, y = sin x, x = 0, and .v= Tr/4 about the x axis [Hint:
revolving the region bounded by the graphs of the given equations = -
COS" .V 5(1 + cos 2x) and sin" x = i(\ cos 2v).]
about the v axis.
30 X = cos y + sin y, x = cos y - sin y, y = 0, and y = 7r/2

9 y = x', y = 8. and .v = 10 V" = j:, V = 4, and .v = about the y axis

11 y' = 4.V, y = 4, and x = In Problems 31 to 38, find the volume of the solid generated when
the given region in Figure 20 is rotated about the axis indicated. In
12 y = .v~ + 2, y = 4, and .v = (first quadrant) = s
Figure 20, the curve OP has the equation y 3.r* for :£ j: 2.

13 y^ = x\y = 8, and .v = 14 y = lv-\y 2,and;c =

15 X = Vcos (7ry/4), y = 0, and y = 1


Figure 20 p = C-.n)
16 .V = esc (7ry/6), y = I . and y = 2

In Problems 17 to 30, find the volume of the solid generated by


rotating the region bounded by the given curves about the indicated
axis. Use the method of circular disks or the method of circular
rings.

= -V" and y = 2x about the .v axis

= t' and y' = x about the x axis

= x~ and y = X about the y axis

20 V = j:^ + 4 and v = Ix' about the v axis


SECTION 5.2 THE METHOD OF CYLLNDRICAL SHELLS 365

31 OAP about the .v axis 32 OBP about the x axis by the method of circular rings, you will encounter the integral
ft^ \ a' — x~ dx. Notice that this integral represents the area
33 OBP about the y axis 34 OAP about the y axis
of a semicircle of radius a; hence its value is iTza'.)

35 OAP about the line AP 36 OBP about the line AP


Figure 21
37 OBP about the hne BP 38 OAP about the line BP

39 Find a general formula for the volume of the solid generated

when the region OAP in Figure 20 is revolved about the axis


v = a. for 2 2.

40 A torus, or anchor ring, is a doughnut-shaped solid (Figure 21


generated by revolving a circular region R about an axis in its

plane that does not cut the region R. Find the volume V of such a
torus if the radius of R is a and the distance from the center of i?
to the axis of revolution is b. {Hint: In working this problem

6.2 The Method of Cylindrical Shells

In this section we present an alternative method for finding the volume of a solid of

revolution based on cylindiical shells rather than on circular disks or rings. We


begin by considering the volume V of a solid right circular cylinder of fixed height
h. but with \ariable radius .v (Figure la). Here

Figure 1 V= TTX-h

'x If the radius .v is increased by a small amount A-v = dx. then


the volume V is increased by a corresponding amount AV
(Figure lb). Evidently. AV is the volume of a thin cylindrical

shell of height /; with inner radius .v and thickness A.\- = dx.


Using the differential to approximate AV. we have

AV == £/V = d(TTX-h) = iTT.xh dx

As A.V approaches zero, this approximation becomes better

and better.Thus, a cylindrical shell of height h, inside radius


X, and infinitesimal thickness dx has an infinitesimal volume

d\' \-xh dx

You can remember the formula dV = lir.xh dx by imag-


ining that the cylindrical shell (Figure lb) is cut vertically and
"unwrapped" to form a rectangular slab of height /; and
thickness dx (Figure Ic). The length of this slab is approxi-
mately the inner circumference of the shell. 2ttx units: hence,
its volume is approximately

{2'rr.x){h){dx) = l-rrxh dx cubic units

Now, let /? be a region in the .xt plane bounded above


by the curve y = fix), below by the curve y = g{x). on the
left by a: = a. and on the right by x = b. where <a <b
and/W > g{x) for a < a; < fc (Figure 2a). Let 5 be the solid
366 CHAKFER 6 APPLICATIONS OF THE DEFINITE INTEGRAL

Figure 2

Figure 3
SECTION 6.2 THE METHOD OF CYLINDRICAL SHELLS 367

Figure 4

y
368 CHAPTER 6 APPLICATIONS OF THE DEFINITE INTEGRAL

Figure 7
SECTION 6.3 VOLUMES BY THE METHOD OF SLICING 369

10 R is bounded by the graphs of y = cos .r. y = sin .\~. x = 0, and 23 R is the same region as in Problem 21 but the axis of revolution
,

.V = Vtt/I. is X = 20.

24 /? is bounded by the graphs of y = 2x^ - 9x' + llv, y = 0,


In Problems 1 1 to 16, use the method of cylindrical shells to find the
-v = 0, and x = 3; about the x axis.
volume V of the solid of revolution generated by revolving each
region R about the .x axis. 25 R is the region in the first quadrant above the graph of y = x^ and
below the graph of y = .v + 2; about the line x = —2.
11 /? is bounded by the graphs of y = .r^, y = 27. x = 0.
26 R is the region in the second quadrant above the graph of y =
12 R is bounded by the graphs of y = vx. y = 2, and x = 0. -.r' and below the graph of y = 3x^; about the line y = -3.

13 R is bounded by the graphs of a = 4Vy, y = 4. and x = 0.


27 Use the method of circular rings to rework Example 3 on page
14 R is bounded by the graphs of x = y'VT+y?, x = 0, and 368.
v= 1.
28 Use the method of cylindrical shells to show that the volume of a
15 R is bounded by the graphs of .v = y- + 1 , -v = yv 1 + y', y = right circular cone is one-third of its height times the area of its

0. and y = 1 base.

16 /? is bounded by the graphs of y = .v", y = x".x = 0, and x = 1 29 Find the volume of the solid generated by revolving the right

where n < m. triangle with vertices at (a, 0). (i>, 0), and (a, h) about the y axis.
Assume that Q < a <b and /; > 0.

In Problems 17 to 20, use the method of cylindrical shells to rework


30 Let V be the volume of the solid of revolution generated by
the indicated problem.
revolving a region R. which lies to the right of the y axis, about
the y axis. If /j is a positive constant, show that the volume of the
17 Problem 5 in Problem Set 6.
solid of revolution generated by revolving R about the axis .v
=
18 Problem 22 in Problem Set 6. — fc is given by V + 2ttM, where A is the area of R.

19 Problem 25 in Problem Set 6. 31 Let R be the region bet ween x = —a and x = a bounded above
by the graph of y = fcVl — (x/a)^ and bounded below by the x
20 Problem 28 m Problem Set 6.
axis, where a and b are positive constants. The football-shaped
solid 5 generated by revolving R about the x axis is called an
In Problems 21 to 26, use either the method of circular rings or the
ellipsoid of revolution. Find a formula for the volume V of 5 in
method of cylindrical shells to find the volume V of the solid of
terms of a and b.
revolution generated by revolving each region about the indicated
axis. 32 Rework Problem 40 in Problem Set 6. 1 , using the method of
cylindrical shells.
21 R is bounded by the line y = 16 and the parabola y = .\-; about
33 A cylindrical hole is bored through the center of a sphere of
the X axis.
unknown radius. However, the length of the hole is known to be
22 R is the same region as in Problem 1 1 . but the axis of revolution L units. Show that the volume of the portion of the sphere that
is V = 27. remains is equal to the volume of a sphere of diameter L.

6.3 Volumes by the Method of Slicing

Figure I In Sections 4.7 and 5.6, we studied the method of slicing for determining the areas
of admissible plane regions. In this section we use an analogous method, also called
the method of slicing, for finding the volumes of solids. Actually, this method is

just a generalization of the method of circular disks or circular rings presented in


Section 6. 1

In order to calculate the volume V of a solid S by the method of slicing, select a


convenient reference axis, and let A{s) be the area of the cross section of S inter-
cepted by the plane perpendicular to the reference axis at the point with coordinate s
b reference
axis (Figure I). Suppose that the entire solid S is contained between the plane at 5 = a
and the plane dX s = b. where a < b.
370 CHAPIER 6 APPLICATIONS OF THE DEHNITE INTEGRAL

Figure 2 In Figure 2, let dV denote the infinitesimal volume of the portion of the solid 5
between the plane perpendicular to the reference axis at the point with coordinate .v

and the corresponding plane at the point with coordinate .v + ds. Here ds represents
an infinitesimal increase in the reference coordinate. Evidently, dV is the volume of
an infinitesimal solid cylinder of height ds with base area A(s): hence,

dV = A(s) ds

The total volume V of S should be obtained by summing


the solid that is,

integrating — all the infinitesimal volumes dV as s runs from a to h. Therefore, we


should have

A(s) ds
=r
This argument in which we use Leibnizian infinitesimals, although perhaps not
mathematically rigorous, produces the correct result. A more conclusive argument,
based directly on the definition of the definite integral, can be made as follows:
Figure 3 We partition the reference axis into n subintervals

^/Ifs^ As^
\S„, .S|], |.S|, i'2]. [Sk-\- Sk]- |.^„-:.^,,1
volume of A'th slab )

(Figure 3). Above each subinterval |.v^_|, i^] of this partition we have constructed a
circumscribed cylindrical slab of thickness A^;;. The cross-sectional area of such a

slab is approximately Ais/^), so its volume is approximately Ais^) As^. Therefore,

V« S A(s,) Ai,

As the norm of the partition becomes smaller and smaller and the number of slabs
increases while the slabs get thinner and thinner, the approximation becomes better
and better. In the limit, as the norm of the partition approaches zero.

" rh

Zj A{si,) A.v^ approaches A(s) ds

by definition: therefore, we have

A{s) ds

EXAMPLE 1 Find the volume of a solid whose base is a circle of radius 2 units if
Figure 4
all cross sections perpendicular to a fixed diameter of the base are squares.

SOLUTION Figure 4 shows the square cross section of the solid at a distance s

units from the center O along the fixed diameter. The reference axis is taken to lie

along this diameter. If .v is the length of one side of the square, then, from Figure 4
and the Pythagorean theorem.

s' +
(I)'
= 2- — = A-s'
4

Therefore, the area of the square is

A(s) =x- = 4(4 - S-) = 16 - As-


SECTION 6.3 VOLUMES BY THE METHOD OF SLICING 371

Figure 5

reference
axis
372 CHAPTER 6 APPLICATIONS OF THE DEFINITE INTEGRAL

Notice that the surface of the gasoline is at level ^ = -3. Therefore, by the method
of slicing, the volume of the portion of the sphere occupied by the gasoline is

given by

V=
'-10
A(s) ds =
'-10
n( 100 - s') ds = J lOOi- - —
3 ' I
-10
(-3 (-10)'
=4 100(-3) q-4 lOO(-lO)

2000 1127
= -29177 + 77 = 77= 1180.19 cubic meters

Figure 7 EXAMPLE 4 A solid right circular cylinder has a radius of 3 units. A wedge is cut

from this cylinder by a plane through a diameter of the base and is inclined to the

base at an angle of 30°. Find the volume of the wedge.

SOLUTION Let the reference axis lie along the intersection of the plane and the
base of the cylinder, with the origin O at the center of the base (Figure 7). The
cutting plane for the method of slicing intersects the wedge in a triangular cross

section TPQ, whose area


A(i) = h\PQ\ \PT\

we must find in terms of the coordinate s of the point T. Triangle OTP is a right
triangle, and \0T\ = \s\ while \0P\ = 3; hence,

\dp\- = \df\- + \pf\-

so \pf\' = \0P\- - \df\- = 9 - |^|- = 9 - ^'

Referring to right triangle TPQ. we have

\PQ\ = 1^1 tan 30°

Therefore,
A(s) = i\PQ\ \PT\ = i\PT\ tan 30° \PT\

= i\pf\- tan 30°

Since \Pf\~ = 9 - s^

A(s) = -(9
1

i-)tan30° =
1 .
-(9-5-^)(^-
-> (^\
By the method of slicing,

, V3
V= A{s) ds (9 s-)^ds
J- 3 2
Figure 8 Ik'ing plane
V3
(9s - -
[18 -(-18)]
6 3/1-3 6

(36) = 6\/2i cubic units


6

If B is an admissible plane region and A' is a point not lying in the same plane as

B. then the three-dimensional solid consisting of all points lying on straight line
segments between P and points of B is called a solid cone with vertex P and base B
(Figure 8). The perpendicular distance /; between the vertex P and the base B is

called the height of the cone.


SECTION 6.3 VOLUMES BV THE METHOD OF SLICING 373

If a reference axis is chosen perpendicular to the base 6. then a slicing plane at

a distance s from the vertex P will intercept the cone in a cross-sectional region
which is similar to the base B. Furthermore, the linear dimensions of this cross
section are proportional to its distance 5 from P. Since the area A{s) of the cross
section is proportional to the square of its linear dimensions. A{s) is proportional to
S-. Thus.
A(i) = Ks-

where A^ is a constant. When s = h. then

A(i) = A{h) = area of base B


Therefore,
Mh)
A(h) = Kh- so K = -—Y-
Figure 9
Hence,
A{.h)
A{s) =

By the method of slicing,

—h'V-5- ds = —/r—
{'' {'• A(h) A(h) {'•
, ,
V= A(s) ds = s^ ds
Jn Jn Jq

h- 3 3

Hence, the volume of a solid cone is given by one-third of its height times the area of
the base.

EXAMPLE 5 The Great Pyramid of Cheops at Al Giza, Egypt, has a square base
230 meters on each side and a height of 147 meters (Figure 9). Find the volume V
of the Great Pyramid.

SOLUTION The pyramid is a solid cone with height 147 meters and base area
(230)" square meters. Hence, its volume is

V= |(147)(230)- = 2,592,100 cubic meters

Problem Set 6.3

1 A certain solid has a circular base of radius 3 units. If cross 5 Find the volume of a spherical shell whose inside diameter is Yi
sections perpendicular to one of the diameters of the base are units and whose outside diameter is Ki units.
squares, find the volume of the solid.
6 The base of a solid is the region in the .r>' plane bounded by the
2 Find the volume of a solid whose base is a circle of radius 5 oval-shaped curve (.i/4)- -I- (\/3)- = 1 (an ellipse). Every cross
centimeters if all cross sections perpendicular to a fixed diameter section of the solid perpendicular to the .v axis is a semicircle.

of the base are equilateral triangles. Find the volume of the solid.

3 A monument is 30 meters high. A horizontal cross section .v 7 The base of a solid lies in a plane, and the height of the solid is

meters above the base is an equilateral triangle whose sides are 5 meters. Find the volume of the solid if the area of a cross

(30 — ;r)/15 meters long. Find the volume of the monument. section parallel to the base and s meters above the base is given
by the equation (a) A{s) = 35^-1-2, (b) A(s) = s^ + s.
4 A tower is 24 meters tall. A horizontal cross section of the tower
X meters from its top is a square whose sides are nt.v + 1.5) 8 Use the method of slicing to find the volume of a solid right
meters long. Find the volume of the tower. circular cylinder of radius r and height h. taking the cross sec-
374 CHAPTER 6 APPLICATIONS OF THE DEFINITE INTEGRAL

tions perpendicular to a fi xed diameter of the base. You may 13 Show that the volume of a solid cylinder (not necessarily a cir-
assume that /I, vr-r ds = Jirr. cular cylinder and not necessarily a right cylinder) is given by its

height limes the area of one of its bases.


9 The Department of Public Works of East Mattoon intends to cut
down a diseased elm tree 4 feet in dian\;ter. They first cut out a 1 14 The reentry vehicles used during Projects Mercury, Gemini, and
wedge bounded below by a horizontal plane and bounded above Apollo were each shaped roughly like a frustum of a right cir-

by a plane that meets the horizontal plane along a diameter of the cular cone (Figure 12). Find the volume of such a reentry vehicle
tree at a 45° angle. What is the volume of the wedge? if its base is a circle of radius 2.7 meters, its top is a circle of
radius 1.4 meters, and its height is 2.8 meters.
10 A solid right circular cylinder has a radius of r units. A wedge is

cut from this cylinder by a plane through a diameter of the base Figure 12
circle, and the plane inclines to this base circle at an angle 6.

Find a formula for the volume of this wedge.

11 The plans for a wave-guide antenna are shown in Figure 10. All
dimensions are in meters. Each cross section perpendicular to
the central axis (the x axis) is an oval-shaped region whose area
is Tn'"/2 square meters, where y is the distance from the center
of the oval to its upper edge. The upper and lower boundaries
of these cross sections are parabolas y = iW + 1 and
y = -A.r - 1. Find the volume enclosed by the wave guide if

its length is 1.5 meters.

Figure 10

15 Find the volume of the spherical segment of one base shown in

Figure 13 if the radius of the sphere is r units and the height of


the segment is h units.
12 The portion of a cone (not necessarily a circular cone) formed by
cutting off the top by a plane is called a frustum. If the cutting Figure 13
plane is parallel to the base, then the distance between the cut-

ting plane and the base is called the height h of the frustum.
Show that the volume V of the frustum shown in Figure 1 1 is

given by V = (/i/3)(A + VAa + a) cubic units.

Figure II
16 Find the volume of a solid whose cross section made by a plane
perpendicular to a reference axis at the point with coordinate s
has area given by

ias' + bs + c for < i < /i


A(s) =
lo otherwise

Express this volume in terms of .4() = A(0), A\ = A(h/2). and


At = /4(/?). (Hint: Use the prismoidal formula. Theorem 5 on
page 345.)
SECTION 6.4 ARC LENGTH AND SURFACE AREA 375

17 A tent is made by stretching canvas from a circular base of 20 Find the volume of the spherical sector shown in Figure 16.
radius a to a semicircular rib erected at right angles to the base
Figure 16
and meeting the base at the ends of a diameter. Find the volume
enclosed by the tent.

18 Find the volume of the spherical segment of two bases shown


in Figure 14 if the sphere has radius r. the height of the segment
is /i2, and the lower base of the segment is hi units from the
bottom of the sphere.

Figure 14 Figure 15

21 A tower is 60 meters high, and every horizontal cross section is


a square. The vertices of these squares lie in four congruent
parabolas whose planes pass through the central axis of the
tower and which open outward away from this central axis. Eiach

of the four parabolas has its vertex in the upper square base of
the tower, and each of these parabolas has a horizontal axis. The
diagonals of the upper and lower bases of the tower are 2 and 12
meters long, respectively. Find the volume enclosed by the
tower.

19 A regular octahedron is a solid bounded by eight congruent 22 Two right circular cylinders of radius r have central axes meet-
equilateral triangles (Figure 15). Find the volume of an octahe- ing at right angles. Find the volume V of the solid common to
dron if each of its eight bounding triangles has sides of length /. both cylinders.

6.4 Arc Length and Surface Area


In this section, we give two further geometric appHcations of the definite inte-
gral: the calculation of arc length and the calculation of the area of a surface of
revolution.

The Arc Length of a Curve

Figure 1
If a straight piece of wire of length s is bent into a cui^e C we understand that the
curve C has arc length 5 (Figure 1). For instance, the arc length of a circle of
wire of length s diameter D is known to be given by j =
Here we do not attempt to give a
ttD.
formal definition* of arc length, but we assume that you have an intuitive under-
standing of this concept.
Now. let/be a function with a continuous first derivative on some open interval /
containing the closed interval [a. b]. We give an informal derivation of a formula
for the arc length of the portion of the graph of /between the points (a, f{a)) and

*See Problems 41 and 42 (pages 382 and 383) for an indication of a formal development.
376 CHAPTER 6 APPLICATIONS OF THE DEHNITE INTEGRAL

(h. f(h)) (Figure 2). Let s denote the arc length of the portion of the graph of/
between the points (a,f(a)) and (x,f(x)), where a < .v < fc. If x is increased by an
Figure 2
infinitesimal amount dx, then v =/(.v) will change by a corresponding infinitesimal
amount dy, and likewise s will increase by an infinitesimal amount ds (Figure 3).

Notice the infinitesimal right triangle with legs \dx\ and \dy\

and with hypotenuse ds. By the Pythagorean theorem,

(ds)- = (dx)- + (dy)-

=
-(i (dx)-

Hence,

ds= + Vl +
\ \
'iU \j'(x)f dx

Integration of the differential equation above gives the arc length


Figure 3

5 = ds

of the graph of/ between the point with abscissa a and the point with abscissa b.
Thus,

= Vl + [/'(.v)]- dx
J

E EXAMPLE 1 Find the arc length of the graph of the function f(x) = .r^^ I be-
tween the points (8, 3) and (27, 8).

SOLUTION Here /'(jc) = i.v~'/-^, so the desired arc length is given by

= Vl + \f(x)f dx = \
Vl + ix~
J

dx
3.v'/-'

Making the change of variable u = 9a~''' + 4 and noting that du = 6.r


'''^
dx,
M = 40 when j: = 8, and u = 85 when .v = 27, we have

du
=
J
V9P^T4--^ = J
v^
18
„3/2|85 853/2 _ 403/2

27
= 19.65 units
27

El EXAMPLE 2 Set up an integral representing the arc length of the portion of the
parabola v = .v" betwe en the origin and the point (2, 4). Then use Simpson's para-
bolic rule, 52„ ~ Jo Vl + [f'(x)]- dx, with /i = 2 to estimate this integral and thus
the arc length.

SOLUTION Here dy/dx = Zx, so the desired arc length is given by

5 = Vl + (Zv)- dx = Vl + 4x- dx
SECTION 6.4 ARC LENGTH AND SURFACE AREA 377

To find the quantities needed for Simpson's parabolic rule with /; = 2. we subdivide
the closed interval [0, 2] into 2« = 4 equal subintervals by means of the points
Xk = k/2 for A: = 0, 1, 2, 3, 4. Now let

v* = Vl + A(Xk)- = VTTF
so that

vo = 1 \\ = V2 v. = V5 V3 = VlO and .V4 = V\l

1/2,
Then Sn„ -(vo + 4v, + 2_V2 + 4V3 + y4)

= — { 1 + aVi + 2V5 + 4V10 + VT?)


6

Evaluating ^4 and rounding off to two decimal places, we have

54 « 4.65

hence, the desired arc length is approximately 4.65 units. (Incidentally, the correct
value of the desired arc length, rounded off to four decimal places, is 4.6468 units.)

If we express the equation of the curve between two points in the form x = g(y),
where g' is a continuous function in the closed interval [c. d\, then the arc length of

the graph of g between (g(c), c) and (g(d). d) is given by the formula

= { Vl + [g'(v)]- ^y = yi
I d\'

EX.4MPLE 3 Find the arc length


" of the graph
^ of the equation 8.y = y"* + (2/y~)
from (1. 1) to (ft, 2).

SOLUTION Here x = ky^ + iy"-, so

^ _~ 2T
dy
,3 _ ^4-^'
,-3 _ 2I
" 2
L
2y^

The desired arc length is given by

f- r 7 i
]~ ^ r- h" 1 1

dy

= [\^'i^'y = \[^^i^'y 2 2v
1 \P 33
8 4v- 16

Figure 4

surface
The Area of a Surface of Revolution

Once again, we do not attempt to give a formal definition of the "surface area"" of
5 (Figure 4). Suffice it to say that if the solid 5 has surface area A. then the
a solid
same amount of paint required to apply a uniformly thin coat to the surface of 5
would be required to apply a uniformly thin coat to a flat surface of area A.
378 CHAPTER 5 APPLICATIONS OF THE DEFINITE INTEGRAL

Figure 5 To find the surface area A of the right circular cone shown in Figure 5a, we cut
the cone along the dashed line and flatten it out to form a sector of a circle, as in
Figure 5b. The slant height a of the cone is the radius of the sector, and the arc
length of the sector is the circumference lirr of the base of the cone.
The sector in Figure 5b is part of a circle of area na' with circumference Irra.
The ratio of the area A of the sector to the area ira- of the circle is the same as the
ratio of the arc length Inr of the sector to the total arc length Ina of the circle; that

A Ivr
A = TTar
Tra" lira

Now, consider the frustum of a right circular cone shown in Figure 6. The surface
area Af of this frustum is evidently the difference between the surface area of the
large cone with slant height a and base radius /t ^^^ the surface area of the small
cone with slant height a - s and base radius ri (Figure 6). Thus, by the formula
previously obtained for the surface area of a cone,

Af = TTarj — 77(a — s)ri = 7ra(/'2 — r\) + -nsr^

By similar triangles, a/r2 = (a — s)/rx, so

ar-i — sr-,

Figure 6
Therefore,

Af= 77
rn-
— r^
(r-,
"
— rj) + TTsri = TTsr-,
" + Trsr, = 7T5(ri + r-,)
'
= 2tt
2
—s

Notice that (r, + r2)/2 is the radius of the cross section of the frustum midway
between its two bases and 27r(r| + r2)/2 is the circumference of this midsection.
Hence, the surface area Af of a frustum of a right circular cone is given by

2tt-

the circumference of its midsection times its slant height.


We now consider the problem of finding the surface area A of the surface of
revolution generated by revolving the portion of the graph of the nonnegative con-
tinuous function/between the lines x = a and x = b about the x axis (Figure 7). Let

Figure 7
.
SECTION 6.4 ARC LENGTH AND SURFACE AREA 379

ds denote the infinitesimal arc length of the portion of the graph of /above the
interval of infinitesimal length dx. as shown in Figure 7a, and let .v denote the
coordinate of the center of this interval. When the infinitesimal arc of length ds is

revolved about the x axis, it generates an infinitesimal frustum of a cone of slant


height ds whose midsection has radius /(.v) (Figure 7b). The surface area of this
infinitesimal frustum is

dA = Ivfix) ds

Assum e that the function / has a continuous first derivative, so that


ds = Vl + [f'(x)]- dx; hence,

dA = 27r/(.v)Vl + [/'(.v)]- dx

The desired surface area A can now be obtained by integrating ^, so, for revolution
about the x axis.

A = j
27r/(.v)Vl + [,/"(.v)l- dx

that is.

A = 27r/(A) ds
380 CHAPTKR 6 APPLICATIONS OF THE DEFINITE INTEGRAL

Figure 9
SECTION 6.4 ARC LENGTH AND SURFACE AREA 381

Figure 11

;-
382 CHAPTER 6 APPLICATIONS OF THE DEFINITE INTEGRAL

(c)23 v = .v' between (0. 0) and (2, 8) length of the astroid and (b) the area of the surface of revolution
obtained by revolving the portion of the astroid in the first and
, /I V3 \ / 3 V3 \
y— ——j
,

24 v = Vlv - .V- between . and y—. ——-) second quadrants about the x axis.

36 Suppo.se that the function /is nonnegative and has a continuous

[325 V = 4- +
8
—^
4.V-
between ( 1, ^) and (2, -^— derivative on |u, h\. Let So denote the arc length of the graph of
/between (a,fU')) and (b,f(h)), and suppose that A^i is the area
of the surface of revolution generated when this graph is re-

26 V =
V2
4
.vVl - .V- between (0, 0) and (-—
\2 16 /
volved about the .v axis. Let k
= /(.v) +
be a positive constant, and define a
function ^ by ^'(.v) k. Express the area A of the surface
generated by revolving the graph of ,1; between (a. gUt)) and
In Problems 27 to 30, find the area of the surface of revolution
[h, fiih)) about the .v axis in terms of .S'd and -4|i.
obtained by revolving each curve about the v axis.
37 One cubic centimeter of a certain substance is formed into n

[327 y = 4x- between (0. 0) and (3, 36) equal spheres, (a) What is the total surface area of all these
spheres? (b) What is the limit as n grows larger and larger with-
\£}2S y^ = x^ between (0. 0) and (4, 8)
out bound of the total surface area in part (a)? (c) Assuming that
the rate at which this substance dissolves in a certain solvent is
29 v' = 9x between (0. 0) and (— ,
2J proportional to its surface area, explain why it dissolves more
rapidly if it is first ground into a fine powder.

1330 •'^ = 7:^ + -7 between (--^, l) and y— .


2j 38 An organism in the shape of a sphere with a semipermeable
surface is suspended in a tluid having the same density, (/ grams
= per cubic centimeter, as the organism. Nutrients diffuse through
Elln Problems 31 and 32, use Simpson's parabolic rule with n 2
the surface from the fluid into the organism at the rate of k grams
(that is, wiih four subintervals) to estimate the area of the surface of
per second per square centimeter of surface, and waste products
revolution obtained by revolving each curve about the v axis.
diffuse into the surrounding fluid at the same rate. To sustain

= itself, the organism requires at least b grams of nutrients per


31 V sin -v between (0, 0) and (v, 0)
second per gram of its own total weight. Find an upper bound R
32 y = iV9 -.V- between (0, 2) and (2, iVS) for the radius r of the organism.

33 Find a formula for the total surface area (lateral surface plus
39 A sphere of radius r is inscribed in a right circular cylinder of

base) of a right circular cone of height h with base radius r.


radius r. Two planes perpendicular to the central axis of the
cylinder cut off a spherical zone of area A on the surface of the
34 (a) If the linear dimensions of a right circular cone are multiplied sphere. Show that the same two planes cut off a region on the
by a positive constant k. how is its total surface area affected? cylinder with the same surface area A.
Why? (b) Complete the following sentence; If the linear dimen-
El 40 The Torrid Zone is the region on the surface of the earth between
S are multiplied by
sions of a solid a positive constant k, then the
the Tropic of Cancer and the Tropic of Capricorn (Figure 13).
surface area of 5 is
The Tropic of Cancer is the parallel of latitude that is 23.45°
35 The astroid, or tetracuspid, .r^^ + y^^^ = 1 (Figure .12) was north of the equator, and the Tropic of Capricorn is the parallel
first studied by Johann Bernoulli in 1691-1692. Find (a) the arc of latitude that is 23.45° south of the equator. Find the area of
the Torrid Zone if the radius of the earth is 6.37 1 x 10' kilome-

Figure 12 ters. (Hint: Use the result of Problem 39.)

Figure 13

41 Let/be a continuous function on the interval [a. h\. A partition

IP of [a, b\ consisting of the subintervals l-Vy, .vi), \x\, X2],

<
SECTION 5.5 WORK. FORCE. AND ENERGY 383

[.X2. xi.] l-v„-|, A„] determines points P„ = (.ro. /(.Vq)). Figure 14


P, = Ui./(.v,)). P^ = (.v:,/(.V2)) P„ = (.v„./(.v„)) on the
graph of/ above the endpoints of the subintervals in the partition

ff. (Figure 14 sho ws the case n = 4.) The pol ygon co nsisting of
segments PoP\. P\Pi. P2P3,
the line • • • . P„-\P„ is called a
polygon inscribed in the graph of/. Let

uif) = 2 \Pk-iPk\
k=l

denote the total length of the inscribed polygon corresponding to


the partition IP. If

i = lim LifP)
IMI-o 42 In Problem 41 assume that/has a continuous
. first derivative on
[a. b]. Using the mean-value theorem, show that for each k =
exists, then the graph of/ is called a rectifiable curve, and j is

called arc length. If A-r^ = - show that


1. 2, 3 71. there exists a number q in the interval
its xt a*-i .

U*-i, -v<] such that/(.Vi) -/(.vj_,) =f'(ct) A.r*. Using this re-

sult, rewrite Z.I//"') as a Riemann sum. and thus obtain the for-
US') = S \ (Avt)- + [/(.Vj)-/(A>_,)|= mula i = JS \ 1 + l/'(-v)]- dx for arc length.

6.5 Work, Force, and Energy


In Chapter 4 we used antidifferentiation to calculate the work done by a variable
force. Here we show how the definite integral can be used in calculating work,
force, and energy.

Work Done by a Variable Force


In Section 4.5, we showed that if a particle P moves along the s axis under the
influence of a possibly variable force F acting parallel to the .s axis (Figure 1), then
the net work W done by F on P satisfies the differential equation cW = F ds. If a
and b are coordinates of two points on the s axis, we can take the definite integral
Figure 1 from s = a io s = b oi both sides of the differential equation to obtain

dW =\ F ds
J

Since dW =W\

represents the difference between the value oi W aX s = b and aX s = a. then

1
F ds=W
384 CHAP1F.R 6 APPLICATIONS OF THE DEFINITE INTEGRAL

ION The work done is given by

215
= 53.75 ft -lb

EXAMPLE 2 A spring has a natural length of 0.25 meter. If a force of 30 newtons


is required to stretch the spring 0.05 meter, how much work is done in stretching the

spring 0.15 meter?

SOLUTION By Hooke's law, the force F on the spring is proportional to its

displacement: F= ks. When .v = 0.05 meter, we have F = 30 newtons; hence,


k = 30/0.05 = 600 newtons per meter. Therefore. F = 600s, and the work done is
Figure 2 given by
0.15 10.15

Fds= I
600i- (is = 300s- = 6.75 joules

Work Done in Pumping a Liquid


A container C contains a liquid weighing w units of force per cubic unit of volume
(Figure 2a). It is desired to pump some of this liquid up above the rim of the
container to a certain height and then discharge it. To calculate the work done by the
pump, we establish a vertical axis with its origin.? at the level up to which the liquid
is to be pumped. For simplicity, let the positive direction on the s a.xis be down-
ward.
Assume that the level of the liquid at the start of the pumping is s = a and that its

level at the end is s = ^. Suppose that the cross-sectional area of the surface of the
liquid at level s is A(s) square units. The infinitesimal slab of liquid between level s
and level s + ds has a volume of A{s) ds cubic units and weighs wAis) ds units of
force (Figure 2b). The infinitesimal work done in raising this slab through s units to

the level of the origin is given by dW = swA(s) ds. so the total work required to
surface area^(i) pump the liquid from level s = a lo level s = b \s given by
roluine/l(j)<fc

dW= swA{s) ds
J
I

»f sA{s) ds

Figure 3
SECTION 6.5 WORK. FORCE. AND ENERGY 385

Therefore, the work done is given by


MO
sAis) ds = 9800 stt[25 - [s - 6)^] ds

= 980077- + 45-'
^ 4 2 /,
= (5.9584 X 10'')77= 1.8719 x 10^ joules

Compression or Expansion of a Gas


Figure 4 It requires work to compress a gas; conversely, when a gas expands, it does work
upon its surroundings. Consider, for instance, a quantity of gas in a cylinder of
pressure P
radius r closed by a movable piston (Figure 4). Set up a reference axis parallel to the

r central axis of the cylinder, denote the coordinate of the piston


the coordinate of the end of the cylinder.
Denote the pressure of the gas by P units of force per unit area, and let V
represent the volume of the gas in cubic units. The force F on the piston is given by
by 5. and let c be

F= TTT-P

the product of its cross-sectional area and the pressure of the gas. The volume V of
the gas is the volume of a cylinder of radius r and height c — s\ hence.

V= 77r"(c ~ S) = TTT'C — TTT'S

Therefore,
-dV
dV = — 77T~ ds or ds
Trr~

The work done on the gas by the force F in moving the piston from j to 5 + t/j is

g'venby
dW = F ds = {nr-P){
^^^
-) = -P dV

Consequently, if the gas is compressed from an initial volume V„ to a final volume


Vi . the total work done is given by

rv = r, rv, rv,
W= dW = \
{-P)dV=-\ P dV

By switching the limits of integration, we can remove the negative sign, so that

Vo
W P dV

Notice that if the gas is being compressed, its volume decreases, so that V, < Vq
and W is positive.
If the gas is expanding, it does just as much work on its surroundings as would be
required to compress it back to its original volume. Hence, if a gas expands from an
original volume Vq to a final volume V| , it does an amount of work given by

W PdV
i:
386 C HAP1KR 6 APPLICATIONS OF THE DEFINITE INTF.tJRAL

IB EXAMPl.K 4 One cubic foot of air at an initial pressure of 50 pounds per square
inch expands adiabatically (that is. without transfer of any heat energy) to a final
volume of 3 cubic feet according to the adiabatic gas law P = kV ' "*.
where k is

a constant. Find the work done.

SOLUTION When V = 1. P = 50 pounds per square inch = (50)(144) pounds


per square foot. Therefore, (50)( 144) = k( "'•*). so k = (50)( 144) = 7200. Hence,
1

P = 7200V"' •',
Vn = 1, V, = 3, and

W PdV= \
7200V- ''^V

\ -n 4
-0.4 / ,

= -18,000(3" r"-*) = 6400.9 ft -lb

Figure 5

unit cross-sectional
area

Figure 6
SECTION 6.5 WORK, FORCE, AND ENERGY 387

Figure 7 SOLUTION From Figure 7 and the Pythagorean theorem,

[—j +s- = 3- or / = 2V9 - s-

The desired force is given by

fS r3 13
^

F = w\ si ds = 62.4 2sV9 - s- ds = 62.4[-§(9 - s-)^^-]\


•'o ^ '

= (62.4)(18) = 1123.2 lb

Energy
If a particle P of constant mass m moves along a linear scale because of an unop-
Figure 8 posed (possibly variable) force F (Figure 8), then Newton's (second) law of
motion force equals mass times acceleration — can be written

'O-- F =
388 CHAPTER 6 APPLICATIONS OF THE DEFINITE INTEGRAL

The equation

= imvl - |wiu

which expresses the fact that the work done by the force F in moving the particle P
from s = a lo s = b \s equal to the resuhing change in the kinetic energy of P, is

called the work-energy theorem.


The work that would be done by the force F in moving the particle P from its

present position .v to an arbitrary but fixed reference position — say, the point with
coordinate h — is called the potential energy V of P. Thus,

=/""'=-!' F ds
By the alternative version of the fundamental theorem of calculus (Theorem 1

page 334) it follows that


dv _ __j_ r
F ds = -F
ds ds Jh

that is.

dV
ds

The last equation can be taken as an equivalent definition of potential energy V.


The kinetic energy hnv' of the particle P represents the capacity of P to do work
owing to its motion. The potential energy V of P also represents the capacity of P
to do work, not because it is moving, but because of where it is located. For

instance, if the particle is connected to a spring and the spring is stretched, then

work can be done by letting the spring return to its natural length. The total energy
£ of the particle is defined to be the sum of its kinetic energy and its potential energy

E = hm- + V

EXAMPLE 6 Prove the law of conservation of energy for the particle P discussed
above; that is, show that the total energy E remains constant as the particle moves
along the i axis.

SOLUTION It will be enough to show that dE/dt = 0. We have

dE d I
, ,,
dv dV
dt dt\2 I
SECTION 6.5 WORK. FORCE. AND ENERGY 389

Vertical Motion in a Gravitational Field

An object of mass m kilograms near the surface of the earth is acted upon by a
constant force of gravity F newtons and if allowed to fall, accelerates with a con-
stant acceleration ^ = 9.8 meters per second per second. We refer to the gravita-
tional force F as the weight of the object. According to Newton's law,

F = mg
the weight of an object (near the surface of the earth) is
obtained by multiplying its
mass by the acceleration of gravity. In particular, one kilogram weighs approxi-
mately 9.8 newtons.
Figure 9 In the following examples, suppose that a projectile P of mass m is fired verti-
cally upward from the surface of the earth at time t = with an initial velocity \o
(Figure 9).* Take the .? axis pointing straight upward with its origin at the surface of
the earth. Then the (constant) force F of gravity on the projectile acting down-

f
is
ward; hence F is negative. The acceleration n of P is also negative: hence, a = -g
and
F = ma = -mg
'/////////////////y
We take 5 = 0, the surface of the earth, as the reference position for the calcula-
tion of the potential energy V of P. Thus.

V {V.. =
f (~mg) ds = {-mgs)\ = - (~mgs) mgs

EX.AMPLE 7 Find the equation of motion s =f(t) of P.

SOLUTION We have

dv
a - -g or dv = -g dt
dt

Hence, v= f (-g)dt= -gt + C^

When r = 0, we have v = vq, and it follows that C, = vq. Therefore,

'gt + Vo

Because v = ds/dt. we can rewrite the last equation as

ds_
-gt + VQ or ds = (- gt + Vq) dt
dt

Hence, s = \ (-gt+ Vo) dt = ~igt- + Vo? + C.

Since s = when r = 0, it follows that C. = 0. and the equation of motion is

s = -ig'- + Vof

*In these examples we assume that the air resistance can be neglected and that the decrease
in the force of gravity as the particle moves away from the center of
the earth can also be
neglected.
390 C HAITER 6 APPLICATIONS OF THE DEFINITE INTEGRAL

EXAMPLE 8 Find the speed |v'| in terms of s.

SOLUTION From Example 7,

V = -gt + Vf) and i = -igr + vv


Eliminating t from these two equations, we find by simple algebra that

v^ = vg - 2gs

Hence,

|v| = Vvg - 2gs

EXAMPLE 9 Find the maximum height /; to which the projectile climbs in terms of
the initial velocity vo.

SOLUTION To maximize s. we set

ds_
= v =
dt

In Example 8, we found that


V- = V5 - Igs

so, setting v = and s = h. we obtain

= V5 - 2gh

h =
^8

EXAMPLE 10 Find the kinetic energy and the potential energy of P as functions of
time /.

SOLUTION By the result of Example 7, we have

imv' = im(-gt + v'o)- = irng't^ - mg\\)t + hm\i


Similarly, the potential energy is given by

V = mgs = mg(-hgr + v^) = -kmg't- + ingvot

EXAMPLE 11 Find the (constant) total energy E off, and explain what happens to

the kinetic energy and to the potential energy off as it climbs from s = Q\o s = h.

SOLUTION By the results of Example 10, we have

E = imv' + V = 2/ni'5

As P climbs from 5 = to .v = li, there is a trade-off between the kinetic and


potential energies. At the start, when t = and ^ = 0, all the energy is kinetic, and
the potential energy is zero. As P climbs from = i- to .y = //, the kinetic energy
decreases and the potential energy increases. Finally, when s = h. the kinetic en-

ergy is zero (because \' = 0), and all the energy is potential.
SECTION 6.5 WORK, FORCE, AND ENERGY 391

Problem Set 6.5

E] 1 How much work in foot-pounds is done in stretching a spring of El 1 1 The piston in a cylinder compresses a gas adiabatically from 50
natural length 12 inches from 15 to 18 inches if the final stretch- to 25 cubic inches. Assuming that f V' ' = 100, where P is the

ing force is 25 pounds'? pressure in pounds per square inch and V is the volume in cubic

inches, how much work in foot-pounds is done on the gas?


El 2 A spring has a natural length of 30 centimeters. A force of 200
newtons is required to compress the spring by 3 centimeters. 12 Gas being compressed in a cylinder satisfies the equation
How much work is done in compressing the spring from its PV^ = constant, where y is a constant greater than I, if the

natural length to a length of 25 centimeters? compression takes place adiabatically (that is, without heat pass-
ing into or out of the cylinder). If the compression takes place
O 3 A reservoir is in the form of a hemisphere of radius 3 meters. If
isothermally (that is, at constant temperature), then the gas satis-
it is filled with salt water weighing 10.110 newtons per cubic
fies the equation PV = constant. If a gas is compressed from an
meter, how much work (in joules) is required to pump all the salt
initial volume Vq to a final volume V,, will more work be re-
water out over the rim of the reservoir?
quired for adiabatic or for isothermal compression?
El 4 Water is pumped directly up from the surface of a lake into a
El 13 Steam expands adiabatically according to the law FV' ''

water tower. The tank of this water tower is a vertical right


constant. How much work is done if 0.5 cubic meter of steam at
circular cylinder of height 20 feet, with radius 5 feet, whose
a pressure of 2 x lO"* newtons per square meter expands by 60
bottom is 60 feet above the surface of the lake. The pump is
percent?
driven by a 1.5-horsepower motor. Neglecting friction, how

long will it take to fill the tank with water? Assume that water El 14 A small compressor is used to supply pressurized air for filling

weighs 62.4 pounds per cubic foot and that 1 horsepower is tires. The compressor is driven by a 1500-watt electric motor,
33,000 foot-pounds per minute. and tank has a volume of 0.02 cubic meter. Suppose we
its air

start from a normal atmospheric pressure of 1.013 x 10'' new-


El 5 A conical cistern is 6 meters across the top and 4.5 meters deep
tons per square meter. How long will be required to bring the
and is filled to within 1.5 meters of the top with rainwater
pressure in the air tank up to 5 x lO' newtons per square meter?
weighing 9800 newtons per cubic meter. Find the work in joules
Assume that the compression takes place adiabatically, so that
done in pumping the water over the top to empty the tank.
PV' * = constant, and recall that one watt is one joule per sec-
El 6 An elevator weighing 10,000 newtons is lifted through 60 me- ond.
ters by winding its cable onto a winch. Neglecting friction, how
much work is done if the cable weighs 150 newtons per meter?
El In Problems 15 to 21, use the fact that water weighs 9800 newtons
El 7 A water trough is 6 feet long and has a cross section consisting
per cubic meter (approximately 62.4 pounds per cubic foot).
of an isosceles trapezoid with altitude 4 feet, upper base of 3
feet, and lower base of 2 feet. If the trough is filled with water
15 What force must be withstood by a vertical dam 30 meters long
weighing 62.4 pounds per cubic foot, how much work will be
and 6 meters deep if the water level is at the top of the dam?
required to pump all this water up to a level 20 feet above the top
of the trough? 16 A water main in the shape of a horizontal cylinder 1 .9 meters in
radius is half filled with water. Find the force on the circular gate
El 8 An object immersed in water is buoyed up by a force equal to the
that closes the main.
weight of the water that the object displaces (Archimedes' prin-
ciple). Find the work required to completely submerge a spheri- 17 A rectangular oil can is filled with oil weighing 9114 newtons
cal float of negligible weight if its diameter is 0.3 meter. As- per cubic meter. What is the force on one side of the can that is

sume that the water is in such a large reservoir that its level rises 0.2 meter wide and 0.4 meter high?
negligibly as we submerge the float and that the water weighs
18 A tank in the shape of a horizontal right circular cylinder of
9800 newtons per cubic meter.
radius 0.5 meter is half filled with water and half filled with oil.

El 9 A cable that is 25 meters long hangs vertically from a winch. If The oil, which weighs 91 14 newtons per cubic meter, floats on
the cable weighs 20 newtons per meter, how much work is done top of the water. Find the total force caused by these liquids on
in winding the cable onto the winch? the circular end of the tank.

10 A uniform girder of length L meters and weight w newtons lies 19 The face of a vertical dam has the shape of an isosceles trapezoid
horizontally on the ground. How much work in joules will be of altitude 16 feet with an upper base of 42 feet and a lower base
required to pivot the girder about one of its ends into a vertical of 30 feet. Find the total force exerted by the water on the dam
position? when the water is 12 feet deep.
392 CHAKIKR 6 APPLICATIONS OF THE DEFINITE INTECRAL

20 A horizontal tank in the shape of a right circular cylinder of Given that -Fi = Fj. prove that the sum m^vi + m2V2 of the
radius 2 meters is sealed, and water is forced in until the tank is linear momentum of F| and the linear momentum of P2 remains
exactly half full. Find the total force on one end of the tank constant during the motion.
caused by the resulting air and water pressure. Take normal at-

mospheric pressure to be 1.013 x 10^ newtons per square


29 A particle P is moving along the positive s axis under the influ-

meter.
ence of a force F. The total energy £ of F is zero, and its poten-
tial energy is given by V= -\/s. (a) Find the velocity v = ds/di
21 A water trough has a cross section in the shape of an isosceles of P in terms of s. (b) If s = 25 meters and v is negative when
triangle with its apex pointing downward. If the trough is 0.6 / = 0, find the equation of motion of F. (c) If j = 25 meters and

meter wide and 0.4 meter deep, find the force on one end when V is positive when t = 0, find the equation of motion of F and
it is full of water. show that the velocity v of F is positive for all / > 0. (d) Given
the information in part (c), find lim v.

22 Show that for a region R submerged as shown in Figure 10, the


force F caused by a liquid weighing w units of weight per unit 30 Consider the mass-spring system in Figure 1 1 in which the mass
volume is given by m is suspended by a perfectly elastic spring with spring constant

k. The mass of the spring itself and air friction are to be ne-
ds
yf \h(s)]- glected. The origin on the vertical s axis is chosen so that when
the mass and spring are hanging in equilibrium, the s coordinate
of m is zero. The mass m is lifted to the position with coordinate
Figure 10 s = Aq and released at time / = with an initial velocity vq = 0.

surface of liquid
Figure II
1 y

'yyy/yy/yyy»
gp

E123 A 5-cent piece has a mass of about 5.2 x 10 '


kilogram,
(a) How many newtons does a 5-cent piece weigh? (b) How
many
(c) If
joules of work
5 cents will buy 1.8
proximately how many
5 cents' worth of electricity?
are required to
x 10''
lift a 5-cent piece
joules of electric energy, ap-
5-cent pieces could be lifted 1
1 meter?

meter for I
24 An elastic spring with a spring constant of k newtons per meter is

stretched from its relaxed position through L meters. How much


(See the discussion at the end of Section 4.6.) (a) Given that the
(potential) energy in joules is stored in the stretched spring?
potential energy V is zero when the mass passes through the
25 (a) Is thework necessary to stretch a spring from 21 to 22 centi- origin, find a formula giving V in terms of 5. (b) Find the (con-
meters the same as the work necessary to stretch it from 22 to 23 stant) total energy E of the mass-spring system, (c) Find the
centimeters? Explain, (b) Is the work necessary to lift a 5-cent velocity of the mass when it passes through the position i =
piece from 21 to 22 centimeters above ground level the same as the first time after it is released, (d) Discuss the trade-off be-
the work necessary to lift it from 22 to 23 centimeters? Explain. tween the potential and the kinetic energies as the mass oscil-
lates.
1S126 A weight of 1 metric ton (1000 kilograms) is dropped from a
height of h feet, and it hits the ground with the same kinetic 31 Two positively charged particles with charges of 1 coulomb
energy as that possessed by a car of mass 1 .2 metric tons travel- apiece repel each other with a force equal to kjr newtons, where
ing 88 kilometers per hour. Find h. r is the distance in meters between the particles and the constant
k is given by k = 8.99 x 10''. Suppose that one of the particles
E127 Given that the potential energy of a relaxed spring is zero and
is fixed at the origin and the other particle, call it F, is free to
that the spring constant is 500 newtons per meter, how far must
move along the positive s axis. Find the potential energy V of F
the spring be stretched so that its potential energy is 200 joules?
in terms of .v, given that lim V = 0.
28 Two particles Pi and P2 with masses mi and mi are moving on
the s axis with variable velocities V| and vt. A variable force of 32 Figure 12 shows an .v axis with its origin at the center of a homo-
attraction exists between the two particles, so that the force ex- geneous solid sphere of radius r meters and with constant density
erted on /"i by P2 is F, and the force exerted on P2 by P, is Fi- of vv kilograms per cubic meter. A particle F of mass m kilo-
SECTION 6.6 APPLICATIONS TO ECONOMICS AND THE LIFE SCIENCES 393

grams at the point with coordinate s on the axis will experience a Figure 12
gravitational force F given (in newtons) by

r 4Gnnn^w
-4Gini
for 5 > r
= J 3?
4
Gnnnvs for < 5 < r

where G= 6.672 x 10~" N •


m"/kg" is Newton's universal
constant of gravitation, (a) Find the potential energy V of P in

terms of s if lim V= 0. (b) Find the escape velocity for P


starting on the surface of the sphere.

33 Suppose that a projectile P of mass m is fired vertically upward


from the surface of the earth at time t = with an initial velocity
1334 Work Problem 33, continuing to neglect air resistance, but tak-
vq. Neglecting air resistance and neglecting the decrease in the
ing into account the decrease in the force of gravity as the pro-
force of gravity as the projectile moves away from the center of
jectile moves away from the center of the earth. Take the radius
the earth, find a formula for the idnetic energy of P in terms of
of the earth to be 6.371 x 10* meters and the mass of the earth
its height .j above the surface of the earth.
to be 5.983 X lO"'' kilograms.

6.6 Applications to Economics and


the Life Sciences

We now present a few simple, but typical, applications of the definite integral to
economics and the life sciences.

Consumer's Surplus
Suppose that the demand equation

relates the price p per unit of a commodity and the number of units q of the com-
modity that will be demanded at that price by consumers in a given period of time.
Usually, fewer units of the commodity are demanded as the price per unit increases,
and when the price reaches a sufficiently large value, say c dollars per unit, none of
the commodity is demanded. Thus, we assume that

c=/(0)
Now suppose that the actual market price, po dollars per unit, is less than c dollars
per unit and that the corresponding demand, qo units, is positive. Thus,

Po = fiqo)

Clearly, those consumers who are willing to pay more than po dollars per unit
benefit from the only po dollars per unit. Economists measure
fact that the price is

this benefit, the consumer's surplus, by the integral

consumer's surplus -[ qdp


394 CHAPrER 6 APPLICATIONS OF THE DEFINITE INTEGRAL

We now derive an alternative formula for the consumer's surplus in terms of the
function/. To begin with, notice that the revenue R dollars to the producer for the
sale of </ units of the product at p dollars per unit is given by

R=pq
If we take the differential on both sides of the last equation, we obtain

dR = p dq + q dp or ^ dp = —p dq + dR
Therefore,
re rp=c rp=c
consumer s surplus = \ q dp = \
( -p) dq + dR
Pi) P=l>o P=Pn
rp=c \p=c
= - pdq + R\
I
P=Ph 'p=Po

Now, since q = qo when p = po and ^ = when p = c. we can rewrite the last


equation as

consumer's surplus = - p dq + Rl =1 p dq + pq\


<?=*i P=Pu P=Po- 9=90

= p dq + c-0 - poqo
'o

= p dq - poqo
'o

Therefore, since p =f(q). we have

consumer's surplus = f(q) dq


— f(qo)q(]

EXAMPLE 1 The demand equation for coal in a particular marketing area is p =


200(600 - 10^ - q-), where q is the demand in thousands of tons per week and p
is the price in dollars per thousand tons. If the current price is $80,000 per thousand
tons, find the consumer's surplus.

SOLUTION Here, f{q) = 200(600 - \0q - q-) and po = $80,000. Solving the
quadratic equation

80,000 = 200(600 - lOc/o - <75)

for qo, and recalling that qo cannot be negative, we find that qt,
= 10. Therefore,

consumer's surplus = f(q) dq ~ f{qo)qo


-'n

200(600 - 10^ - q-) dq - (80,000)(10)


=r
^3 \ I
10

= 20o(600^ - 5q 800.000

700,000
$233,333.33
APPLICATIONS TO ECONONOCS AND THE LIFE SCIENCES 395
SECTION 6.6

Production over a Period of Time


often
When new production process is set in motion, the rate of production is
a
'bugs" in the production techniques, unfamiliarity
rather slow at first because of
out and the personnel
with new methods, and so forth. As the bugs are worked
increases
become accustomed to the new methods, the rate of production ordinarily
value.
and, after a suitable period of time, approaches a steady
produced in the
Thus, let X denote the number of units of a certain commodity
represents the
first t of working time by a new process, so the derivative dx/dt
units

rate of production at time t. Then, according to the fundamental theorem of calcu-


number of units of the commodity produced during the inter\'al of time
lus, the total

from r = a to r = i> is given by

X
396 CHAPTER 6 APPLICATIONS OF THE DEFINITE INTEGRAL

SOLUTION Making the change of variable u = t/\0. we have

25 J di = 250 «'/- du = 250 —


^3
m'''" = 4500 lb
4 V 10 4 /l„

Blood Flow in the Circulatory System


Provided that certain factors (such as pressure and viscosity) are held within pre-
scribed limits, blood will flow smoothly through a cylindrical blood vessel in such a
way that the velocity v of flow increases continuously from a value close to zero at

Figure I the wall of the vessel to a maximum value at its center. Figure 1 shows a cross
section of the blood vessel perpendicular to its central axis and an infinitesimal
circular ring of width dr at a distance r from the center. Assume that the velocity v
of blood flow depends only on r; then the infinitesimal volume of blood dV flowing
across the circular ring in unit time will be given by

dV = vil-nr dr)

the product of the velocity of flow across the circular ring and the area of this ring.
If R denotes the radius of the blood vessel, the total volume of blood passing the

entire cross section in unit time will be given by


REVIEW PROBLEM SET 397

Problem Set 6.6

1 Consider the demand equation q = 100(4 - V^). (a) Solve the of 30\ t automobiles per week at the end of / weeks. How many
demand equation for p in terms of q. (b) Calculate the con- automobiles do they expect to produce during the first 36 weeks
sumer's surplus when the price is po = $1 per unit, using the of production?
definition of consumer's surplus as the integral q dp.
/J,,
6 Suppose that the rate of production of a new product is
(c) Calculate the consumer's surplus, using the alternative for-
mula Stf(q) dq -f{qo)qo, where p =f{q).

2 Sketch a graph showing an arbitrap,' demand curve p = fiq) and


a point iqn. po) on this curse. Shade in an appropriate region on units per week at the end of r weeks, where A and k are positive

your figure whose area corresponds to the consumer's surplus constants and p is a constant greater than 1 . Find a formula for
when the price is po dollars per unit. the number of units produced during the nth week of production.

3 On the basis of market sur\eys. a manufacturer determines that


7 A factory is dumping pollutants into a lake at the rate of r /600
tons per week, where ; is the time in weeks since the factory
5000 souvenirs will be sold per week in a certain reson area if
commenced operations. After 10 years of operation, how much
the price is SI per souvenir, but that for every 5-cent increase in
pollutant has the factory dumped into the lake?
price per souvenir. 500 fewer souvenirs per week will be sold.

Find the demand equation for the souvenirs, and calculate the 8 In Problem 7. assume that natural processes can remove up to
consumer's surplus if the souvenirs are priced at SI. 25 each. 0.015 ton of pollutant per week from the lake and that there was
no pollution in the lakewhen the factory commenced operations
4 Suppose that the supply equation p = giq) relates the price per
10 years ago. How many tons of pollutant have now accumu-
unit of a commodity and the number of units q of the commodity
lated in the lake?
that the producers would be willing to supply per unit of time at

that price. Supposing that the producers would be unwilling to 9 A cylindrical blood vessel has a radius R = 0.\ centimeter, and
supply any of the commodity if the price drops as low as c blood is flowing through this vessel with a velocity
dollars per unit, so that c = g(0). and that the current selling V = 0.30 — 30r^ centimeters per second at points r centimeters
price of thecommodity is po dollars per unit, economists define from the center. Find the rate of flow of the blood.

theproducer's surplus as the value of /f q dp. If pn = giqo)-


10 If blood is flowing in a blood vessel according to Poiseuille's
show that the producer's surplus is given by poqo ~ /o" S*<?) dq.
equation v = k{R- - r). show that the volume V of blood that
5 Heron Motors has set up an assembly line for their new steam- flows across a fixed cross section of the vessel in unit time is

powered automobile and expects to be producing them at the rate given by V = Uk/DR-^

Review Problem Set, Chapter 6


In Problems 1 to 12, use any convenient method to find the volume 8 y- = 4.V and .v = 4 about the line .v = —2
of the solid generated by revolving the region bounded by the given
9 y = .X- \ y = 1 . and y = 4 about the y axis
curves about the indicated axis.
10 The loop of y- = .v"'(.x + 4) about the y axis

1 y = Vx, y = 0. and .v = 8 about the .v axis 11 y = X. X = 4, and y = about the line y = -2

2 y = .r, y = 0. and .v = 1 about the y axis 12 V = V.v and y = 18 - .x". to the right of .x = 1 , about the y axis

3 jr -I- 4 = 4y, .v = 0, and y = .v about the x axis 13 Find the volume of the solid generated by revolving the region
bounded by a circle of radius 3 centimeters about a line in its
4 y = .r'. .\ = 1 . and y = about the line .r = —2
plane that is 7 centimeters from its center.

5 y = -V and y = Iv about the y axis


14 A solid is generated by revolving the region bounded by the

= — = = graph of y =/(.v), the lines x = Oand.v = a. and the .x axis about


6 .x~ 4(1 y) and y about the line y 3
the X axis. Its volume for all values of a is given by the equation
7 y = 1/x and y = 3 - x about the y axis V= a- + la. Find a formula for/(x).
398 CHAPTER 6 APPLICATIONS OF THE DEFINITE INTEGRAL

15 A solid paraboloid of revolution is ge nerat ed by revolving the 23 A solid is generated by a variable regular hexagon which moves
region bounded by the graph of v = b\'x/a, the v axis, and the so that its plane is always perpendicular to a given diameter of a

line .V = a about the v axis, (al Sketch the paraboloid of revolu- fixed circle of radius a. with the center of the hexagon lying in

tion, (b) Find the volume of the solid by the method of circular this diameter and two opposite vertices lying on the circle. Find
disks, (c) Find the volume of the solid by the method of cylindri- the volume of the solid.

cal shells.
24 Liquid in a reservoir evaporates at a rate (cubic units per unit

16 A form generated by revolving the graph of y =


vessel has the time) proportional to the surface area of the liquid exposed to the
\.x^ about the y axis. If 4 cubic units of liquid leak out of the air. Show that no matter what the shape of the reservoir, the

bottom of the vessel per minute, at what rate is the depth of surface of the liquid drops at a constant rate.

liquid changing at the instant when the depth is 4 units?

17 Sketch the graph of the portion of the parabola Aat = B-y lying
in the first quadrant, where A and B are positive constants. Find
the volume of the solid generated when the region bounded by
this parabola, the y axis, and the line y = A is revolved about the
y axis. Compare the answer with the volume of a cylinder of
height A and base radius B.

18 For each positive integer ^i. let U„ denote the volume of the solid
generated by revolving the region under the graph of y = x"
between .v = and .r = 1 about the y axis. Let V„ be the volume
of the solid generated by revolving the same region about the .v

axis. Evaluate

(a) lim U„ (b) lim V„ (c) lim

19 Find the volume of a solid whose cross section perpendicular to

the .r axis is a square .x units on a side for < j: £ 2.

20 Find the volume of a solid whose cross section perpendicular to


the .V axis is an equilateral triangle \.x\ units on a side for
- 2 < .V s 3.

21 A spire is 26 feet high. A horizontal cross section x feet above


the base is a rectangle whose long side is (26 - .r)/13 feet long

and whose short side is (26 — .t)/20 feet long. Find the volume
of the spire. 25 A football-shaped solid is swept out by a variable square that is

perpendicular to the .v axis and whose center lies on the x axis. In


22 A conoid is a wedge-shaped solid whose lateral surface is gener-
sweeping out the solid, one vertex of the variable square follows
ated by a line segment which moves so as always to be parallel
the curve y = 5V49 — x- in the .vy plane between .v = -7 and
to a fixed plane and which has one endpoint on a fixed circle and
x = 1 . Find the volume of the solid.
the other endpoint on a fixed line, with both the circle and the
straight line being perpendicular to the fixed plane (Figure I ). If 26 A horn-shaped solid is generated by a variable circle whose
the distance from the fixed line to the plane of the circle is h plane turns about a fixed line^ The point of the circle nearest the

units and the circle has radius a units, find the volume enclosed line describes a quadrant AS of a circle of radius a. and the
by the conoid. radius of the variable circle is cd. where 6 denotes the angle
between the variable plane and its initial position when it passes
through A. Show that the volume of the horn is given by V=
Figure I Ti57rV-(8a -I-3m).

In Problems 27 to 38, find the arc length of the graph of each equa-
tion between the points indicated.

27 (y - 8)- = .X-' from (0, 8) to (1, 9)

28 y = 4jr from (4. 4) to (32, 16)

29 y = §jr»/- from (0, 0) to (4, ^


REVIEW PROBLEM SET 399

30 y = M-V + (2A-)] from (1. i) to (2. fi) 51 y = ir from (0, 0) to (4, 4) about the y axis

31 y- = i.r' from (0, 0) to (1, §) 52 y" = .r' from ( 1 , 1 ) to (4, 8) about the y axis

v' 1 ,,., ,
53 y- = 9 - A from (0, 3) to (9, 0) about the x axis
32 v = "7" + Tyx *''"°'"
<™- i> '° *^- '

33 y = (.V + 1)-'''-
+ 2 from (3. 10) to (8, 29)
54 v = 1

12a-'
- from (1, M) to (2, ^ ) about the x axis

34 V = Vr + 2i di from (0, 0) to ( 1 . Vr + 2f rfr) 55 Find the work done in pumping all the water out of a full cistern
in the shape of a hemisphere of radius r feet surmounted by a

+ )- = 4y' from (- right circular cylinder of radius ; feet and height h feet.
35 (jc 1 1 , 0) to ( 1 . 1

36 X = i(y + 1)-'/- from (¥ , 3) to (18, 8)


56 A tank is made in the shape of a right circular cylinder sur-
mounted by a frustum of a cone as shown in Figure 2. How
= V2f' + - c//from(1.0)to(2. V2?'* + - much work is done in pumping the tank full of water /row the
37 y r' 1 f'' 1 rff]
I J bottoml

38 V = V4-.r from (-2. 0) to (2. 0)

Figure 2

Eln Problems 39 to 42. set up an integral representing the arc length

of each curve, and use Simpson's parabolic rule with n = 2 (that is,

with/owr subintervals) to estimate this arc length.

39 y = iVx from (1, 2) to (4, 4)

40 y = 2(1 + x-)^'- from (0. 2) to (1, iVl)

41 y = A- from (0, 0) to ( 1 , 1

42 ^• from (0, 0) to 2
-'o 1 +r J,, 1 + r /

43 Use the arc length formula to verify that the straight-line dis-
tance d between the points Pi = (x\, yi) and Pi = (.vt. Vi) is SI A cable that is 30 meters long and weighs 4 newtons per meter
given by d = V(.r2 - .vi)" -I- (ys - y\)'. hangs vertically from a winch. Find the work done in winding
half of the cable onto the winch.
44 Suppose that the function / has a continuous first derivative on
an open interval containing the closed interval [0, 1 ] and that / 58 A cable that is / units long weighs ir units of force per unit of
is monotonically decreasing on [0. 1]. Assume that/(0) = 1 and length and hangs vertically from a winch. If a weight W hangs at
/( 1 )
= 0. Write an integral for the amount of time T required for the end of the cable, how much work is done in winding k units
a particle P of mass m to slide frictionlessly down the graph of of the cable onto the winch?
y = f(x) from (0, 1) to (1, 0) under the influence of gravity.
59 How much work is done compressing 400 cubic feet of air at 15

pounds per square inch to a volume of 30 cubic feet, provided


that the compression is adiabatic (FV' * = constant)?
In Problems 45 to 54, find the area of the surface of revolution
obtained by revolving each curve about the a.xis given. 60 A vertical masonry dam in the form of an isosceles trapezoid is

70 meters long at the surface of the water, 50 meters long at the


45 y = 3Vv from (1, 3) to (4, 6) about the .v axis bottom, and 20 meters high. Find the total force that it must
withstand if the water is level with the top of the dam.
46 y = 3.r from (0, 0) to (2, 12) about the y axis
61 A right triangle ABC is submerged in water so that edge AS lies
47 y = .r' from (1, 1) to (3, 27) about the axis
.v
on the surface and edge BC is vertical. Find the total force on a
48 y = 4a- from (0, 0) to ( 1 , 4) about the y axis face of the triangle caused by the pressure of the water if

\BC\ = 6 meters and |A6| = 2 meters.


49 y = iv^ from (0, 0) to (3, 9) about the .v axis
62 A mass m is falling freely near the earth's surface. At a certain
A^ =
= -—
1
50 V
4
-t- —-^
8.x-
from (1, i) to (3, ^) about the a axis
instant, / seconds, the mass is jq units above the ground and
is falling with a velocity vq. Take the s axis positively upward
400 CHAKIKR 6 APPLICATIONS OF THE DEFINITE INTEGRAL

with its origin at ground level. (Thus, v^ is nef;aiivt'. and the 66 Rebecca. Joshua, and Miriam have grown 100 pumpkins, which
acceleration of gravity is —g.) (a) Find the velocity of the mass they are planning to sell. They believe that they can sell all their

in tcmis of its distance s above the ground, (b) Find the increase pumpkins at a price of 75 cents each, but (hat each 25-ccnt
in the kinetic energy of the mass between .v = .v,, and .v = s^. increase in price per pumpkin will result in the sale of 20 fewer
where ^ S\ ^ S(,. (c) Find the potential energy V of the mass in pumpkins. What is the consumer's surplus if they sell the pump-
terms of i if the potential energy is zero when .v = so. kins at the price that brings them the maximum revenue?
(d) Compute the decrease in the potential energy of the mass
between s = sq and s = s^. where < :r| < i(|. (e) Find the ki- 67 A former mathematics professor is going into business as an
netic energy K and the potential energy V of the mass in terms of income tax consultant and expects to be receiving 1 00 Vr dollars
the time I. per week in consulting fees I weeks after opening a new office.

The fixed expen.ses for office rental, telephone, newspaper ad-


63 The velocity of a l(K)-kilogram sled is 20 - 2i meters per sec-
vertisements, and so forth will amount to $200 per week. What
ond at time / seconds. Find the change in its kinetic energy
is the consultant's net profit after 49 weeks in this new job?
between / = and ? = 10 seconds. How much work is done in

stopping the sled? 68 Suppose new employee at Solar Electric Company Jan
that a

solder 50 - 50/V'/ + connections per hour at the end of /


64 A rod of length h is spinning n times per second about one of its
I

working hours. How many connections does the employee sol-


ends. The material of which the rod is composed has a density of
der during the first 8 working hours?
fix) mass units per unit of length at a distance of .v units from the
stationary end. Find an integral representing the kinetic energy
69 Blood is flowing smoothly through a cylindrical artery of radius
of the spinning rod.
R in such a way that its velocity r units from the center is given
65 Joe, Jamal, and Gus have set up a lemonade stand. The demand by V = K(R- - r) units per second. Here A" is a constant de-
equation for lemonade is /j = -((/-/lOO) - (Iq/IQ) + 30. pending on blood pressure, viscosity, and so forth. A drug is

where p is their asking price in cents per glass and q is the administered which increases the radius of the artery by 5 per-
number of glasses demanded. If the asking price is 15 cents per cent. Calculate the percentage increase in the rate of flow of
glass, find the consumer's surplus. blood through the artery.
TRANSCENDENTAL FUNCTIONS

Algebraic and trigonometric functions, useful as they are, are not sufficient for the
applications of mathematics to physics, chemistry, engineering, economics, and the
life and earth sciences. In this chapter, we add the inverse trigonometric, logarith-

mic, exponential, and hyperbolic functions to our repertory. In order to better un-
derstand the inverse trigonometric functions and the relationship between logarith-
mic and exponential functions, we begin by considering the general idea of the
inverse of a function.
All the functions that can be built up from algebraic, trigonometric, logarithmic,
and exponential functions by addition, subtraction, multiplication, division, compo-
sition, and inversion are called elementary functions. Although nonelementary

functions sometimes have to be used in applied mathematics, the elementary func-


tions are sufficient for our purposes in this textbook.

7.1 Inverse Functions

Many calculators have both a squaring key (marked .v") and a square-root key
(marked Vx). For nonnegative numbers .v. the functions represented by these keys
"undo each other" in the sense that

squaring function square-root function


xh

and

square-root function squaring function


x\- ^ V.V h -^ (\G)- = x

Two functions related in such a way that each "undoes" what the other "does" are
said to be inverse to each other. Not only do most scientific calculators have keys

corresponding to a number of important functions, but they also can calculate the
values of the inverses of these functions.

401
402 I HAP-IKR 7 TRANSCENDENTAL FlNfTIONS

In order that two functions / and ^' be inverses of each other, we must have the

following; For every value of a in the domain of/, fix) is in the domain of g and

.V ^/(.v) ^^ glfM] = x
Likewise, for every value of .v in the domain of g, g(..x) is in the domain of/ and

.V^Ug(.V)^/[g{.V)]=.V

In other words, /and g are inverses of each other if and only if

(,?°/)(.v) = gl/(.v)|

for every value of a in the domain of /and

(f°gH-x) =f[g(.\)] = X

for every value of a in the domain of g. A function /for which such a function g
exists is said to be invertible.

hi Examples 1 and 2. show that the functions f and g are inverses of each other.

X + I

EXAMPLE I /(a) = 5a - 1 and g(x)


5

SOLUTION The domain of/ is the set U of all real numbers. For every real

number x, we have
(5a - I ) + I 5a
=
ig °/)(.v) = g[fM] = g(5x - 1) ^ "
T" "

The domain of g is also the set IR of all real numbers, and for every real number a,
we have

(/°g)W =f[g{.X)] =/(^4~^) = 5(^^) - 1 = A + I - 1 =A

2v- I
= _J
EXAMPLE 2 fix) and g{x)
2 -A
SOLUTION The domain of/ is the set of all nonzero real numbers. If a # 0, we
have

(g°f)(x) = g\f(x)] = g[ — = -
Ix - 1

[-(^)]-'

Ix - (Iv- 1) I

The domain of g is the set of all real numbers except 2. If a ^ 2, we have

'";
(/=.)W =/l.(.v)l =/(7^) = ^
= Ky^) - ']<2

2 -a/
= 2 - (2 - A) = A
SECTION 7.1 INVERSE FUNCTIONS 403

Figure I Geometrically. /and g are inverses of each other and only the graph of
if if g is

the mirror image of the graph of/ across the straight line v = .v (Figure 1 ). This fact
will be clear if you note first that the mirror image of a point {a. b) across the line

y = .V is the point (b. a). But if/ and g are inverses of each other and if (a. b)
belongs to the graph of/, then

b=f(a) so gib) = g[f(a)\ = a

that is, (b. a) belongs to the graph of g. Similarly, you can show that if [b. a)
belongs to the graph of g, then (a. b) belongs to the graph of/ (Problem 42).
Not every function is invertible. Indeed, consider the function / whose graph
appears in Figure 2. The mirror image of the graph of/ across the line y = x isn't the
graph of a function, because there is a vertical line / that intersects it more than
once. (Recall the vertical-line test, page 26.) Notice that the horizontal line L ob-
tained by reflecting / across the line y = x intersects the graph of/ more than once.
These considerations provide the basis for the following test (see Problem 43).
Figure 2
Horizontal-Line Test
mirror image of .

graph of/ y" A function / is invertible if and only if no horizontal straight line intersects its
graph more than once.

E.X.AMPLE 3 Use the horizontal-line test to determine whether the functions


graphed in Figure 3 are invertible.

_l
Figure 3

(a)

Figure 4
(a) Any horizontal line drawn above the origin will intersect the graph of/
twice. Therefore. / is not invertible.

y>'=^ (b) No horizontal line intersects the graph of F more than once; hence, F is

invertible.

If a function /is invertible. there is exactly one function g such that /and g are
inverses of each other: indeed, g is the one and only function whose graph is the
mirror image of the graph of/ across the line y = .v. We call g the inverse of the
function/, and we write g =/"'. The notation/"' is read "/inverse." If you
imagine that the graph of/ is drawn with wet ink. then the graph of/~' would be
the imprint obtained by folding the paper along the line y = x (Figure 4).
The fact that the graph of/" ' is the set of all points (.v. y) such that (y, x) belongs
to the graph of/ provides the basis for the following procedure.
404 C HAKI KR 7 TRANSCF.NDKNTAL Fl'NtTIONS

'
Algebraic Method for Finding /
Step 1 Write the equation _v = /(.v) that defines /.
SECTION 7.1 INVERSE FUNCTIONS 405

The Inverse-Function Rule


Now we give a rule, called the inverse-function rule, that enables us to find the
derivative of the inverse of a function. First we give an informal derivation of this
rule; then we state the rule more formally as a theorem (Theorem 1).

Suppose that / is an invertible and differentiable function and that

.v=/U)
Then,

-V =/-'[/«] =/-'(.y)

hence, using Leibniz notation, we can write the derivative of/~'( v) as

dx d

assuming that this derivative exists. However, by the chain rule,

dx dy dx
dy dx dx

Solving the last equation for d.x/dy. we obtain the following:

Inverse-Function Rule Using Leibniz Notation


406 CHAKI KR 7 TRANSCENDENTAI, FUNCTIONS

The Inverse-Function Theorem


Now we give a precise statement of the inverse-function rule in the form of a
theorem. In order to understand this theorem, suppose that/ is a differentiable and
invcrtible function and

>'=/(A)

so that

as we saw above. Thus, assuming that dy/dx ¥" and that/ ' is differentiable. we
have the inverse-function rule
SECTION 7.1 IN\'ERSE FlINCTIONS 407

image — the graph of/~' — also has a tangent line at each point (v, u) with u —
/" '(i')- But to say that the graph of/" ' has a tangent line at each point is to say that
it is a differentiable function.
Use of the inverse-function rule in the form

(./ ')'(.v)
l'l/"'(.v)]

is illustrated in the following examples.

EXAMPLE 7 Let / be the function defined by /(.v) = .v^ + x + 1

(a) Show that/"' exists.

(b) Find(/"')'(1).

SOLUTION

(a) Since /'(.r) = 3.r^ + 1 t^ for all real numbers .v, it follows from Theo-
rem 1 that/"' exists.

(b) Evidently. /(O) = 1. so/"'(l) =/"'[/(0)] = 0. Hence, by Theorem 1,

(/^')'(l)
/'[/-'(I)] /'(O) 3(0)- +1
EX.AMPLE 8 Suppose that the hypotheses of the inverse-function theorem are sat-

isfied by the function/, that/(3) = 7, and that/'(3) = 2. Find (/~')'(7).

SOLUTION Since /(3) = 7, it follows that/" '(7) = 3. Hence,

(/-')'(7)
1
_ 1
._ 1

/'[/"'(7)] "/'(3) " 2

Problem Set 7.1

In Problems 1 to 6. show that the functions / and g are inverses of Figure 7


each other.

.v + 3
1 fix) = Iv - 3 and g{x) =

2 /(.v) = x^ and g(x) = Vx

3 /(.v) = — and g{x) = —

4 f(x)
2v
ix -
— and
1
g(x)
Iv-
-
3.V
3

5 f(x) = V'.v-H 8 and g{x) = .v' - 8

6 fix) = X- + I for .V > 1 and gix) = V.v - 1

7 Use the horizontal-line test to determine whether the functions


graphed in Figure 7 are invertible.
4U8 CHAPTER 7 TRANSCENDENTAL FUNCTIONS

8 Under what conditions is a linear function fix) = nvc + h


invertible?

9 The three functions graphed in Figure 8 are invertible. In each


case, obtain the graph of the inverse function by reflecting the
given graph across the line v = x.

Figure 8 y
SECTION 7.2 INVERSE TRIGONOMETRIC FUNCTIONS 409

42 If g is the inverse of the function / and if (b. a) belongs to the time since the anesthetic was administered. Assuming that /is
graph of g, show that (o, b) belongs to the graph of f. invertible, write an equation for C in terms of t.

43 Complete the argument to justify the horizontal-line test by


46 If/ is an invertible function, show that ibe range off is the same
showing that if no horizontal line intersects the graph of/ more
as the domain off '
and that the domain offis the same as the
than once, then /is invertible.
range off^^.
44 A function/ is said to be one-to-one if whenever u and b are in
the domain of/and/(a) = f(b), it follows that a = b. Using the 47 Sketch the graph of any invertible function/; then turn the paper
over, rotate it 90° clockwise, and hold it up to the light. Through
horizontal-line test, show that /is invertible if and only if it is
the paper you will see the graph of/ '. Explain why.
one-to-one.

45 Suppose that the concentration C of an anesthetic in body tissues 48 If/ is aninvertible function, show that/"' is also an invertible
satisfies an equation of the form t = f(C), where t is the elapsed function and that (/"')"' =/.

7.2 Inverse Trigonometric Functions

Figure I

Figure 2

^\. '
410 CHAPTER 7 TRANSCENDENTAL FUNCTIONS

DEFINITION

Figure 4
SECTION 7.2 INVERSE TRIGONOMETRIC FCNCTIONS 411

DEFINITION 2 The Inverse Cosine, or Arccosine

The inverse cosine,


412 CHAPTKR 7 TRANSCKNDKNTAI. FINCTIONS

SOLUTION

(a) If cos'' {-V2/2) = V. then -\/lll = cos v and < y < tt. Therefore,

y = 37r/4.

(b) Using a 10-digit calculator in radian mode, we find that

arccos 0.6675 = 0.839950077

(c) If arctan Vs = y. then Vs = tan y and -7r/2 < y < v/2. Therefore,
y = -n-/3.

(d) Using a 10-digit calculator in radian mode, we find that

tan ' (-2.498) = -1.190013897

you enter a number x between - 1 and in a calculator, take sin"


If 1 ' .v, and then
take the sine of the result, you will get x back again. Thus,

-rl-

Do you see why? Therefore, we have the identity

sin (sin"' x) = x for - 1 < .v < 1

However, if same thing the other way around, first taking the sine and
you try the

then taking the inverse sine, you might not get your original number back. For
instance, starting with x = 2, we get (on a 10-digit calculator in radian mode)

21- -> 0.909297427 I .141592654

The reason is simply that sin"' 0.909297427 is the number benveen -tt/2 and ir/2
whose sine is 0.909297427, and 2 does not lie between -tt/2 and it/2. However, if
you start with a number between -77/2 and 77/2, take the sine, and then take the
inverse sine, you will get your original number back. In other words.

(sin x) = X for

Similar rules apply to cos and cos"' and to tan and tan"' (Problems 41 and 42).
It sometimes necessary to find the exact values of expressions such as
is

sin (tan"'i). (Notice that if you used a calculator, you would obtain only an ap-

proximation to the value of this expression.) The following example shows how to
find the exact values of such expressions by sketching an appropriate right triangle.

EXAMPLE 3 Find the exact value of sin (tan '


S).

SOLUTION We begin by sketching an acute angle in a right triangle such that


6 = tan"' |: that is, tan fl = §. This is accomplished simply by letting opp, the side
opposite 0. be 2 units long and adj, the side adjacent to 0. be 3 units long (Figure 7).

Then, by the Pythagorean theorem, the length of the hypotenuse is given by

hyp = Vopp- -1-


adj- = V2- -I- 3" = Vl3
It follows that
adj = 3
2VT3
(-I) sin
hyp VB 13
SECTION 7.2 IVS-ERSE TRIGONOMETRIC FlfNCTIONS 413

The method illustrated in Example 3 above works more generally, as in the


following example.

EXAMPLE 4 Show that for -1 < .V < 1. cos (sin"' x) = Vl - .r.

SOLUTION Again, we begin by letting = sin '


.v. so that sin 9 = x. We con-
sider first the case in w hich 6 is an acute angle: that is. < 6 < tt/I. or < <
.v 1

This allows us to rig up a right triangle with vertex angle 6 simply by taking
hyp = 1 and opp = .v, so that

sin 6
hyp 1

Figure 8 (Figure 8). Bv the Pvthasorean theorem.

hyp" = opp" + adj"

and it follows that

adj = \ hyp- - opp- = V I- — x- = VT


Therefore, for < v < 1 , we have

adj Vl -.v^
cos (sm .v) = cos 6 = vr
hyp

That cos (sin ' x) = Vl — .r^ w hen .v = and when .v = 1 can be verified by
direct calculation. For instance.

cos (sin ' 0) = cos = 1 = \ 1


- 0-

and a similar calculation confirms the equation for .v = 1 . Thus, for s .v < 1 . we
have _, , r,
^
cos (sin
.

.v) = V 1 — .V-

Now. let's consider the case in which — 1 < .v < 0. Then < — < .v 1 . and by what
has already been proved.

cos [sin"' (-.v)] = Vl - (-V)- = Vl -.r^

But cos [sin"' (-.y)] = cos (


— sin"' .v) = cos (sin"' .v)

and it follows that


cos (sin"' x) = \ 1
- -X-

holds for - 1 < .\ < as well as for < < .v 1 . Therefore, it holds for - 1 < .\ < I

The standard trigonometric identities can be used to help simplify expressions


involving inverse trigonometric functions. The method is illustrated in the following
example.

EXAMPLE 5 If —1 :S.v < 1, simplify the expression cos (2 sin"' .v).

SOLUTION Let y = sin"' x. By the double-angle formula (see inside front


cover),

cos (2 sin"' x) = cos 2y = 2 cos" y — 1 = 2[cos (sin"' .v)]- — 1

Therefore, using the formula cos (sin '


.v) = Vl — .ir from the preceding exam-
ple, we have
cos (2 sin"' X) = 2(Vl - .r= )- - 1 = 2(1 - .r^) - 1 = 1 - Ir^
414 CHAPTER 7 TRANSCENDENTAL FUNCTIONS

The Inverse Cotangent, Secant, and Cosecant


Because the remaining inverse trigonometric functions — cot"', sec "'. and csc~'
(that is, arccot, arcsec, and arccsc) — are easily calculated by using tan ', cos^',
and sin"', you'll rarely find special keys for these functions on scientific calcula-

tors.

Figure 9a shows the portion of the graph of the cotangent function that is used, by
reflection across the line y = v. to form the graph of the inverse cotangent function
(Figure 9b). Thus, the inverse cotangent function is defined as follows.

DEFINITION 4 The Inverse Cotangent, or Arccotangent

The inverse cotangent, or arccotangent, function, denoted by cot '


, or arccot,
is defined by

cot~' .V = y if and only if x = cot y and < y < tt

The following identity is useful when you are using a scientific calculator to
evaluate cot" ' .v;

For all values of .v.

Figure 9
SECTION 7.2 INVERSE TRIGONOMETRIC FUNCTIONS 415

1 EXAMPLE 6 Evaluate cot"' (-2.334).

SOLUTION Using a 10-digit calculator in radian mode, we obtain

cot"' (-2.334) = — - tan"' (-2.334) = 1.570796327 - (-1.166007964)

= 2.736804291

Unfortunately, two different definitions for the inverse secant are in wide use; the
same is true for the inverse cosecant. One definition makes it relatively easy to
evaluate these functions on a scientific calculator, but requires absolute values in
certain calculus formulas. The other definition does not require the absolute values,
but makes it more difficult to evaluate the functions on a calculator. Because most
people these days routinely use a calculator to evaluate trigonometric and inverse
trigonometric functions, we prefer the definition that simplifies these calculations.
If you use formulas from other sources, it is essential to check which definitions of
sec"' and csc~' the author chooses to employ.
In this textbook, we choose the portions of the graphs of the secant and cosecant
functions shown in Figure 10 for purposes of defining the inverses of these func-
tions. Thus, we make the following definitions.

Figure 10
416 ( IIVIIKR 7 TRANSCENOEMAl. FUNCTIONS

Figure II
SECTION 7.2 INVERSE TRIGONOMETRIC FL7SCTIONS 417

15 16 cot'' V3

18 sec"' Vl

20 CSC"' V2
22 arccsc (— V2)

] In Problems 23 to 40, use a calculator (or Appendix D. Table 1 ) to

evaluate each expression. Give answers in radians.


418 CIIAKIKR 7 TRANSCENDKNTAI. FUNCTIONS

a Figure 13
74 cos (2 sec '
.t) for l.vl 1

75 Solve tor v:

tan '
^ + tan '2

76 (a) We know that tan' ' x does not mean 1/tan .v, but there is a
y"-'
value of v between and I for which the equation tan"' x =
1/tan X does hold. Use graphs to show this. El(b) Use a calcula-
tor to find the value of .v between and for which tan .v
= 1
'

1/tan X. Round off your answer to four decimal places.

77 A picture a meters high hangs on a wall so that its bottom is h


meters above the eye level of an observer. If the observer stands
.V meters from the wall (Figure 13), show that the angle ti sub-
Figure 14
tended by the picture is given by

= arctan
a + b
arctan —h
X X

78 Two parallel lines, each at a distance a from the center of a circle


of radius r. cut off the region of area A shown in Figure 14.
Show that

A = 2aVi
r^
cr
•! 1
+ 2r arcsm —O

7.3 Derivatives of and Integrals Yielding


Inverse Trigonometric Functions

Rules for differentiation of the inverse trigonometric functions can be obtained


informally by implicit differentiation. For instance, suppose that u is a differentiable

function of x and that — 1 < w < 1 . Let

V = sin II so that

Assuming that y is a differentiable function of .v, we implicitly differentiate the last


equation with respect to .v to obtain

dy du ~" d\
dy_ ^ 1 du
(cos y) ^
dx ~dx dx cos y dx

Therefore, using the identity

cos (sin '


(/) = VT
(see Example 4 on page 413), we have

d\ du du du
sin u
dx dx cos y dx cos (sin ;/) ^.v Vl - u~ dx

Thus, we have the differentiation rule


d
dx
sin
_,
(/ = —
Vl -
,
1

«-
du
dx
SECTION 7.3 DERIVATIVES OF AND INTEGRALS YIELDING INVERSE TRIGONOMETRIC
FUNCTIONS 419

(For a formal proof of Theorem 2 on page 423.)


this rule, see
A similar argument,
which again uses implicit differentiation, yields the rule

-1
\ 1
- //- dx

(Problem 22).

In Examples 1 and 2, differentiate each function.

EXAMPLE 1 /(.v) = sin"' Iv

SOLUTION f'(x) = sin^ ' 2v = ^


— Ox)
dx VI - (2v)- dx Vl - 4.\-

EXAMPLE 2 g(0 = COS~'?-'

-D,t^ -At'
SOLUTION g'{t) = D, COS"' ;* =
Vl - (/^)2 \/rr7
To obtain the rule for differentiating the inverse tangent,
let

v = tan" ' M so that tan v = u

Again, assuming that v is a differentiable function of .v, we implicitly differentiate


the last equation with respect to x and obtain

(sec- v)
^ .
—=
dy
dx dx
du dx
dx
1 du
sec- V dx
Using the identity + tan= y = see- y (see inside front cover),
1
we have
sec- \- = 1 + tan- y = 1 + ir

Therefore,

~~ tan /(
= :;

'/-v 1 + //- dx

A similar argument yields the rule

-1 du
dx -+-
u- dx

(Problem 22).

//( Examples 3 and 4. differentiate each fimct ion.

EXAMPLE 3 F(.v) = tan"' —

SOLUTION F'(.x) = —dx


tan" ' —=
1+
^ —
5 )-
(x/5f dx\5)
dx\5

1 + (.1-/25) '5 25 + .X-


42(> C HAKIER 7 TRANSCENDENTAL HiNCTIONS

EXAMPLE 4 G(l) = cot '


r

SOLUTION G (/) = —
d
dt
cot
_,
'
,
r =
-d(r)/dt
1 +
7-^
(t-)-
=
1
-It
+
T
/•*

The rule for differentiating the inverse secant can be obtained by using the iden-
tity

sec u = cos

and the rule for differentiating the inverse cosine. Thus, for \u\ > 1 , we have

—d
dx
sec
_,
M = —d
dx
cos
,1 =

u Vl -
,
-1

(l/«)-
^
d I
1

dx^u^
1

I
- ,

V\ -
-^
(\/ir)^ u-
\
1

-,
\
I
^
du
,

dx

dii du

u~V{ir - \)/ii- dx uWu' - \/Vir dx

Now, to simplify the denominator in the last expression, recall that Vtr = |h| and
u' = ImP; hence,

M-Vm- - 1 H-V«- - 1
= "V'-Vm- - 1 = |«|Vir - 1

Therefore,

d ..-, _ 1
SECTION 7.3 DERIVATIVES OF AND INTEGRALS YIELDING INVERSE TRIGONOMETRIC FUNCTIONS 421

SOLUTION We use implicit differentiation. Differentiation of both sides of the


given equation with respect to .v yields

1 ^ dy/dx dv
= I + V-
T = -^— '-—
7 "I or
I +.X- 1 + V- dx 1 + yr

EXAMPLE 8 Let/(.v) = .V sin^' .v for - < .v s 1 . Find/' and/", and sketch the
graph of/.

For - 1 < .r < we have

/'U) = sin-'
Figure I vr
and f"(x) =
<-''?' Vl - x^ (Vl -.f=)-' (Vi -.r=)-'

Now
/(--v) i-.x) = -(-.V sin"' .V) =f{x)
so, / is an even function. Also, /'(O) = and /"(O) = 2 > 0, so the graph of/
shows a relative minimum at (0. 0). Since /"(.v) > for - 1 < .v < 1, the graph
of/ is concave upward (Figure 1).

] EXAMPLE 9 A high tower stands at the end of a level road. A truck driver ap-
proaches the tower at the rate of 50 miles per hour. The tower rises 500 feet above
the level of the driver's eyes. How fast is the angle subtended by the tower
increasing at the instant when the distance .v between the driver and the base of the
Figure 2 tower is 1200 feet (Figure 2)?

Since tan d = 500/.Y, we have = tan"' (500/.Y). Therefore,

_^/^0()^\ /500\
dd dt^ .X ' ^ X-
'

dx 500 dx
.\- + 500- dt

ft/s == -73.33 ft/s


422 CHAPIKR 7 TRANSCENDENTAL Fl'NCTlONS

dll
= sin ' M + C, for Iwl < 1

Vl - ir

du
—= tan « + C, for u in
1 + u

du
= sec '
|h| + C, for Iwl > 1

wVm- - 1

Formulas 1 and 2 follow directly from the rules for the derivatives of sin '
u and
tan"' u. To establish Formula 3, recall that for u # 0,

du

Hence,

d
\u\
d -1 _ du ' '

_ ^ U
u ^
I
1

^^^ I I

du |w|V|h|- - 1 |w|Vi/- - 1 //Vir - 1

holds for |(/| > 1. It follows that

du
= sec \u\ + C for \u\ > 1

uVu- - 1

Formulas 1, 2. and 3 can easily be generalized as in the following theorem.

THEOREM 1 Integrals Yielding Inverse Trigonometric Functions

(i)
(
^ ,
dx
= sin"
,

'
—X + C for a > and x
, ,

< «
J V«- - A- a

(ii)
(
— dx = — tan"' —X + C
1 ,

J a~ + X' a a

(iii)
{
J

jcV.r-
dx
,
— «"
= — sec"' -^ + C
a
1
,
|.v|

a
for Lv > a >

We prove (i) and leave the similar proofs of (ii) and (iii) as exercises (Problems 73
and 74). To prove (i), we make the substitution .v = au. so that dx = a du. and we
use Formula i above. Thus, since a > 0,

f dx _ f a du r a du
J Va- - .r J Vcr - cru- J v'^Vl - «-

- " ''" du
i {
J aVl - u- J Vl - u-

= sin"' u + C = sin"' —+C

dx
EXAMPLE 10 Evaluate
/ V9 - A-
SECTION 7.3 DERIVATIVES OF AND INTEGRALS MELDING INVERSE TRIGONOMETRIC Fl'NCTIONS 423

dx
SOLUTION By part (i) of Theorem 1.
\ 9 -
-^ sin ' — + C.
.r^ 3

dx
EXAMPLE 11 Evaluate
\ 1
- 16.V-

SOLUTION Put u = 4.V, so (/- = 16.V- and d.x = du/4. Then

r dx f du
J Vl - ~ J 4V'l - "
1

sin ' tt + C = — sin"' 4.v + C


16.v^ ;/- 4 4

f'^~' dx
IPLE 12 Find ;-

A. 25 + A-

SOLUTION By part (ii) of Theorem 1.

dx _ v\|-'^^-^

I 25 + .r'
/
\— tan'—
^5
1

5'lo
= — tan"'\/3
1

5
1

5
tan"'

5 ^3^ 15

EXAMPLE 13 Evaluate - d.x.


1 +A-
SOLUTION Put /( = tan '^v. and note that du = dx/(l + .x-). it = when x =
0. and II = tt/3 when v = \ 3. Therefore.

tan '
A-
-dx = (/ du =
1 + A- 18

dx
EXAMPLE 14 Evaluate
aVa- - 9

SOLUTION By part (iii) of Theorem 1,

dx I
— 1
s = — sec 2 sec"' V'2
M
'

•'3V2 •'^v.v- - y . 3^l3\2 3 3

~
3 ^ 3 '
3 'T 36

A Proof of the Differentiation Rule for Inverse Sine


The formal proofs of the rules for differentiating the inverse trigonometric functions
provide nice illustrations of the use of the inverse-function theorem (Theorem 1
page 406). As an example, we give the formal proof of the rule for differentiating
the inverse sine. Similar arguments (see Problems 75 and 76) will provide formal
proofs of the rules for the remaining inverse trigonometric functions.

THEOREM 2 The Derivative of Sin"*


424 CHAFIER 7 TRANSCENDENTAL FUNCTIONS

PROOF We have

f'(x) = cos 7^ for in the interval


.r .v
(-T-l)
Hence, by the inverse-function theorem, /is invertible, / '
is differentiable, and

But, by Definition 1 on page 410,

/"'U) = sin"' X

Therefore, using the identity

cos (sin~' x) = Vl — x^

we have
1 1 1

(/"')'U)
/'[/-'(a:)] cos (sin"' x) Vl - jt

and the proof is complete.

The differentiation rule

—dx
d
sin
_,
M = —
Vl -
,
I

M^
du
dx

as originally stated, is obtained by combining Theorem 2 with the chain rule.

Problem Set 7.3

In Problems I to 2 1 , differentiate each function 15 Gir) = sec '


r + esc ' r 16 Fix) = sec"' VPTg
17 g(.v) = .V' cos"' 3.V 18 /i(/) = f(sin"' /)' -3/
1 fix) = sin^' 3.V 2 g(x) = cos 7.V

— Vv
3 h{x) = tan"' 4 Hix) = cot"' — 19 Hix) = ^ tan"'
X
20 Fix) =
sec"'

JC-+
y X' 1

CSC"' (.Y' + 1)
5 G(f) = sec""' P 6 fix) = csc~' x~ 21 gix)

2x
7 fit) = cor' (r + 3) 8 Fix) = tan"'
1 -.v^ 22 U.se implicit differentiation to derive the rules for differentiating

9 gix) = esc '


—2x
10 fir) = tan"'
r

1
+
-
2
2r
(a) cos"' u (b) cot"' u (c) esc"' m

In Problems 23 to 26, use implicit differentiation to find D,y.

=
11 hiu) sec^
VT^ 23 .V sin '
y = jf + y 24 cos '
ry = sin '
(.v + y)

25 tan"' .v + cot"' y = 7r/2 26 sec"' X + esc"' y = Tr/2


= a;V4 - X' + 4
12 /U) sin" '

y
dy 4(1 -l-y^)
27 If y = tan show that
13/W = sin"-' — + cot"'4 A+x"-
i 2 "
Ian ' Ix
Jr
14 gii) = t cos"' 2/ - Wl - 4f^ 28 (5 + uY° du, find D,y
n
SECTION 7.3 DERIVATIVES OF AND INTEGRALS YIELDING INVERSE TRIGONOMETRIC FUNCTIONS 425

In Problems 29 to 48. evaluate each integral dti


60 Evaluate
4m + 5
"'
30 61 Sketch the graph of v = sin '
(cos x) + cos '
(sin .v) in the in-
J V16-9f^ terval [0. 27r].

62 Find the acute angle between the tangents to the curves y =


V-
J V 25 - 1 1 tan -x and y = cot"' x at their point of intersection.
'

063 A picture 2. 13 meters high is hung on a wall in such a way that

J X- + 9 its lower edge is 2.74 meters above the level of an observer's


eye. How far from the wall should the observer stand in order to

maximize the angle subtended by the picture?

64 Solve Problem 63 for the case in which the picture is h units high
and its lower edge is a units above the observer's eye level.
38
J x\x- - 4
Ic]65 A ladder 15 meters long is leaning against a vertical wall of an

r du office building. A window washer pulls the bottom of the ladder


40 horizontally away from the building so that the top of the ladder
J u\9ir- IC
slides down the wall at the rate of 2 meters per minute. How fast

is the angle between the ladder and the ground changing when
Jo Vl -/2 the bottom of the ladder is 6 nieteis from the wall?

r_^^fi_ E 66 A missile rises vertically from a point on the ground 10 miles


44 rising at the rate of 4000
from a radar station. If the missile is
Jo V8 - M-
feet per minute at the instant when it is 2000 feet high, find the

rate of change of the angle of elevation of the missile from the


radar station at this instant.

du El 67 A police officer in a patrol car is approaching an intersection at

80 feet per second. When she is 210 feet from the intersection, a
car crosses it. traveling at right angles to the police car's path at
60 feet per second. If the officer trains her spotlight on the other
In Problems 49 to 58. evaluate each integral by substitution. (In car. how fast is the light beam turning 2 seconds later, provided
some cases, an appropriate substitution is indicated.)
that both vehicles continue at their original rates?

r cos X (B 68 Romeo needs a ladder to elope with Juliet. However, for protec-
49 dx. II = sin X 50 tion, the Capulets" palazzo is surrounded by an 8-foot-high wall
J V36 - sin- J 1 + tan- I
located 4 feet from the outer wall of the palazzo (Figure 3).

r -v What is the shortest ladder that Romeo must bring?


51 r dx. u = X- 52
J 4 + x^ J 7 + {3.x - 1)-
Figure 3
cos (.t/2
53
r
dx. M = sin —
J 1 + sin' U/2)
(a: 2

r sec- t dt
54
J V 1
- 9 tan- /

'^'
f CSC r cot / ,

55 ;— di. M = 3
4/6 1 + 9 CSC- /

V3/2 ^„,-l

1
\2/2 Vl -.V'-

r- cof' (.y/2)
58 p'"' du
Jo 4 4- .V-
d^r
69 If < .V < — . calculate
2 Jcos, Vl - U-

b^O. 70 Find the area of the region under the graph of y = 3/(9 + x~)
59 Evaluate "
J \'a- - fc-.v- between .t = and x = V3.
426 t HAPTER 7 TRANSCKNDENTAL FUNCTIONS

71 Find the volume of the soli d genera ted when the region bounded 73 Prove part (ii) of Theorem 1.

by the graphs of y = l/Vl + x'. x = 1 , .r = 0. and y = is


74 Prove part (iii) of Theorem 1

rotated about the v axis.


75 Use the inverse-function theorem to give a formal proof of the

72 (a) Show that tt I


(b) Use Simpson's para- rule for differentiating tan '
v.
'„
V 1
- X-
bolic rule with n = 2 (four subintervals) to estimate the defmite 76 Use the inverse-function theorem to give a formal proof of the

and thus estimate n. rule for differentiating sec"' x.


integral in part (a)

7.4 The Natural Logarithm Function

Figure 1

1
SECTION 7.4 THE NATURAL LOGARITHM FUNCTION 427

1 EXAMPLE 1 Use a calculator to estimate e = lim

SOLUTION The following table shows values of [1 + (l/«)]", rounded off to

four decimal places, for selected values of u.

l<

Leonhard Eider
428 CHAPIER 7 TRANSCENDENTAL EUNCTIONS

Figure 2 Evidently, the domain of In is the interval (0. ^), and for .r > 1, In .r can be
interpreted geometrically as the area under the graph of _v = 1/f , / > 0, between
/ = 1 and / = .r (Figure 2). From Definition 1. we have

In 1 = f —dt1
= Q
h t

Also, for < .V < 1

In .V =
J\
—di=-\ J,
— di
t t

(Figure 3). Therefore,

1 ln.v<0

Figure 3
SECTION 7.4 THE NATURAL LOGARITHM FINCTION 429

EXAMPLE 4 Given /(.v) = 5.v In Vcos .v. find/'U).

SOLUTION Using the product rule, we have

1 — sin.v
/'(A) 5 In V cos + .V 5.x 5 1 In V cos .V ^tan.v)
' cos .V Z V COS .V

EXAMPLE 5 If In (.XT) + 3.V + y = 5, use imphcit differentiation to find dy/dx.

SOLUTION Here, we have

d dOx) dy d
'

dx d.x d.x d.x

d\ \ d\

Therefore,

—+
X
1 1

y
d\
dx
^+ 3 + ^=
dx
dx
dy_

dx
3

1
+ (l/.v)

+(l/y)
3-tT

.IT
+
+
y
.V

Integrals Yielding the Natural Logarithm

For ;/ 7^ 0, we have
111
DM = D„VIr iVir
hence, by Theorem 1.

I
, 1 11 1m It u 1

l"l l"l l"l l"l~ "" "

Rewriting the latter equation in terms of antidifferentiation. we obtain the following


theorem.

THEOREM 2 Integration of l/«

For /( ^ 0. I
— di, = In IhI + C.

Ill Examples 6 to 10, evaluate the given integral.

EXAMPLE 6 dx
.V- + 7

SOLUTION We make the change of variable ii = .x' + 7, so that dii = Ix dx and


X dx = 2 du. Then, since .v" + 7 > 0. Theorem 2 gives

J
—;
.X-
X
+ 7
dx = —2 —u ^M = —
1 r

J
1

2
1

In
,

'
(/
,

'
+ C

= —2 1
In Lv-
,

'
.
+ 7
,

'
+ C = — 1

In (.v- + 7) + C
2

EXAMPLE 7 dx
430 C HAPIER 7 TRANSCENDENTAL FUNCTIONS

SOLUTION Put M = In x, SO that du = dx/.x. Thus,

xY u (In A)'
+ C =
,
=
I

= + C
/^ '
dx u- du ;

EXAMPLE « tan v (/.v

SOLUTION Noting that tan .v dx = (sin .v/cos .v) J.v. we put u = cos x. so
J
that du = -sin
;in .v dx. ""
' Thus

tan v J.v = - = -In |«| + C = -In Icos x\ + C


J
dx
EXAMPLLE 9
J-9 JC +

SOLUTION Put M = A- + 1 . so that du = dx. When x = -9, u = -8; and when


X = -5, u = -4. Thus,

J_9
r' ^^ =
X + 1
f
J-8
' — "
= (In |(/|)|
I
-8
' = In I
- 4| - In I
- 8| = In 4 - In 8

'"^'^
sin Ix
EXAMPLE 10 dx
r 3 + 5 cos 2v

SOLUTION We put « = 3 + 5 COS Ix, so that du = -\0 sin Ix dx, or

sinlx dx = -^ du. When .v = 0, m = 8; and when x = 7r/4, m = 3. Thus,

"^^ sin 2v _ P -TDdu _ __}_r du_ ' I'

-'n 3 + 5 cos Zv
'"" "4 (( ~ 10 Js « 10
In \u\

(In 3 - In 8) = (In 8 - in 3)
10 10

Problem Set 7.4

In Problems to 22, differentiate each function. In/


I
19 hit) = 20 Hix)
r' + 5 x + 2
I fix) = In (4a:- + I) 2 g(A) = In (cos .v")
In (tan- a) In |cot (a/3))
21 ^(A) = 22 fix)
3 fix) = sin (In .x) 4 /(r) = tan ' (In

5 g(.x) = A - In (sin 6a)

= + - a
In Problems 23 to 26, find dy/dx by implicit differentiation.
6 H{x) In (A cos A) tan '

7 = + 8 G(A) = In (4a + A- + 5)
fit) sin / In (/- 7)
23 In - + -^ = 5
- y A
9 Fiu) = in (In «) 10 fix) = In (esc A cot a)

24 V = In sec a + tan a| - esc y


A
I

sec
1 1 /i(A) = A In 12 fir) = In N^I + 5P
3 25 y In (sin a) - .vy- = 4

13 giv) = In (v-Vv+ 1) 14 f-U) = In (/' In /^) 26 In > - cos (A + y) = 2

15 /i(a) = In (cos- a) 16 sir) = r- CSC (In r) =


27 (a) Find D, J',"^ cos t' dt. (b) Evaluate dy/dx if y
8a- J'j'"'
" In (tan t*) dt. iHint: Use the alternative version of the
>^^<^' = -^'"4a'+I 18 j?(A) = In '

v- + I
fundamental theorem of calculus, page 332, and the chain rule.)
SECTION 7.5 PROPERTIES OF THE NATURAL LOGARITHM FUNCTION 431

dx dx
28 If (/ > and fc > 0. find

In
'"
.xVa - \1) \
'
39
.vVl - (In.r)-
40
"'
h
J .r[l + (In .v)-]

2\^ xVa + Vb In Problems 41 to 46. evaluate each definite integral.

and simplify your answer.


'/5
dx
41 42
i ^ 01 '

xdx iv
In Problems 29 to 40, evaluate each integral by using a suitable 43 P" 44
J, .V- + 3 v(v(l + V^)
substitution. (In some problems, an appropriate substitution is sug-
gested.) cos (In
rv
.v) civ
45 46 tiv
f
29 f dx
+
— . « = 7 + 5.V 30
\ +
J 7 5.V 9 cos X 47 A particle moves along the s axis in such a way that its velocity

at time / seconds is \/U + I ) meters per second. If the particle is


f dx
31 I cot .V dx. u = 32 at the origin at f = 0, find the distance that it travels during the
J (.V + 2)) In (.V + 2)
= =
time interval from r to f 3.

33 fi!£lilii d.x. (/ = In 4v 48 Find the arc length of the graph of v = (.v-/4) - (In .v)/2 be-
-'
.V
J 3VF(1
"(1 + Vx)
tween (1.5) and (2. - (In 2)/2). 1

M =
35
J
[4^
+ .V- 7
49 Find
(ln.v~)/.v
the average (or
on the interval
mean) value
[1, 4).
of the function /(.v)

+ =
^^ Jo^s^ 38
f sec X sec .v tan .v
dx
50 Find the arc length of the graph of/(.v)
points (77/4, /(7r/4)) and (77/3. /(7r/3)).
In (cos .v) between the
J sec X + tan .v

7.5 Properties of the Natural Logarithm Function

Since the natural logarithm function is defined by an integral (Definition 1,

page 427), we can use properties of the integral to derive properties of In. For
instance, we have already shown that In is a differentiable function on (0, 5=) with

D;t In jc = l/.v, from which it follows that In is a continuous function on (0, ^)


(Theorem 1, page 96). The following theorem establishes one of the most important
properties of In.

THEOREM 1 The Natural Logarithm of a Product

If fl > and ii > 0. then In ab = In « + In b.

Using Theorem 5. page 319, we have

\nab=
{"" dt
\
—= —+ {" di {"" dt
—= Infl+
f"" dt

In the last integral, we make the substitution / = ait. so that dt = a du. Also u = 1

when t = a. and u = b when = ab. Thus, t

a du du
=
rf= In fo

It follows that \n ab = \n a + \n b.
432 CHAriER 7 TRANSCKNDKNTAI. FUNCTIONS

EXAMPLE 1 Given that In a ~ 0.6931 and In h ~ 1.0986, estimate In uh.

SOLUTION By Theorem 1 , we have

\nab = ln« + info = 0.6931 + 1.0986 = 1.7917

THEOREM 2 The Natural Logarithm of a Quotient

It" rt > and fc > 0. then In —= \n a - \n b.


b

Since a = (a/b)h, it follows that In u = In [(a/b)b] = In (a/b) + In /? by Theo-


rem 1. Solving the last equation tor In {ci/b). we obtain In (a/b) = \n a - \n b.

EXAMPLE 2 Given that In a ~ 1.0986 and \n b ~ 1.3863, estimate In —a


b

SOLUTION By Theorem 2, we have

In —= In a - In fo = 1 .0986 - 1 .3863 -0.2877


b

If we put o = 1 in Theorem 2 and use the fact that In 1 = 0, we obtain the useful
identity

-Info forfo>0

EXAMPLE 3 Given that In fo = 1 .6094. estimate In —


fo

SOLUTION In -lnfo== -1.6094

Since log/, .v^ = k log;, .x. you might expect that In a*" = k In .v. However, as we
pointed out, there may be some question about the meaning of .v* when k is an
irrational number. Thus, the following theorem establishes the desired identity only
for the case in which k is rational.

THEOREM 3 The Natural Logarithm of a Rational Power

If A > and k h a rational number, then In a* = k In .v.

We have

D, In A*- = —r DA-X ,

)
= —r Ctv*
1 , ,

')

and

D,(k In A) = k D, In a = /t • — = —k
1
SECTION 7.5 PROPERTIES OF THE NATVRAI. LOGARITHM FINCTION 433

Therefore,

D,(ln.v^ - Aln.v)=--- =
and it follows from Theorem 1. page 252. that

in A* - k In A = C
for some constant C. Putting x = 1 in the last equation, we find that

C= In 1' - A- in 1 = In 1
- A- In 1 = - ^ • =
Consequently.

In .V* - k In A- = or In .v^ = k In .v

and our proof is complete.

EXAMPLE 4 Given that In r « 0.6931. estimate In c".


SOLUTION By Theorem 3, we have
Inc" = (11) In c^(ll)(0.6931) = 7.624

THEOREM 4 Monotonicity of the Natural Logarithm

(i) If < < b. then In


fl a < In b.

(ii) If A > 0. y > 0. and In a = In y. it follows that a

(i) Since D, In a = 1/a > for a > 0. it follows from Theorem 1 . page 173.
that In is an increasing function on (0. y--).

(u) Suppose that a > 0. y > 0, and In a = In v. Then it cannot


be that .r < y,
smce this would imply that In a < In y. which contradicts
In x = In y. Like-
wise, cannot be that y < a. since this would implv that
In v < In x. again
it

contradicting In a = In y. The only remaining


possibility is a = y.

EX.\MPLE 5 Solve the equation In (5 - 3a) + + =


In ( 1 a) In 4 for a.

SOLUTION By Theorem 1 . we can rewrite the given equation as

ln[(5 - 3a)( 1 + A)] = In 4


Therefore, by part (ii) of Theorem 4.

(5 - 3a)( 1 + A) = 4
that is. 5 + 2v - 3a- = 4 or 3a- - Iv - 1 =
Factoring the last equation, we obtain

(3.V-I- 1)(A- 1) =
from which it follows that x = -^ or a = 1

Notice that In (5 - 3a) and In (1 +


when 5 - 3a
a) are defined only
and >
1
-I- A > 0, so we must take the trouble to check that these'inequalities
hold for our
alleged^ solutions a = -i and a = 1. If .v = ~l then 5 - 3a = 6 > and
1
-I- - s > 0;
A hence, a = -5 is indeed a solution. Similarly, if a = 1 . then
5 - 3a = 2 > and 1+ A = 2 > 0; hence, a = is also a solution. I I
434 ( HAPTKR 7 TRANSCENDENTAL Kl'NCTIONS

The Graph of the Natural Logarithm Function

To evaluate In .v directly from the definition

in.v=ri dl

it is necessary to evaluate the integral on the right. Of course, by using (say)


Simpson's parabolic rule, we can obtain an approximate value of the integral and
thus an approximate value of In .v. For instance, using the result in Example 6 on
page 344, we have

= 0.69325
fl dl

Figure I
SECTION 7.5 PROPERTIES OF THE NATURAL LOGARITHM FUNCTION 435

This shows that In x can be made as large as we please by taking x large enough; for
instance, to make In x larger than 1000, it is only necessary to take x larger than
4'""^ It follows that

so the graph of y = In x climbs without bound as a grows larger and larger. In


particular, the graph of In x has no horizontal asymptote.
Using the limit just established and the identity In (\/t) = -In t. we find that

lim In .V = lim In = lim (-In t) lim In t = -oo

In other words. In x can be made smaller than any given number simply by taking x
close enough to 0. The fact that
Figure 2

lim In A

shows that the graph of y = In x has the y axis as a vertical asymptote.

EXAMPLE 6 Sketch the graph of the equation y = In |a|.

SOLUTION Since In |a| = In |


— a|, the graph of y = In |.v| is symmetric about the
y axis. For positive values of a, thegraph coincides with the graph of y = In x in
Figure 1. By reflecting this portion of the graph across the y axis, we obtain the
entire graph of y = In |a| (Figure 2).

THEOREM 5 Range of the Natural Logarithm

The range of In is the entire real number system

We must prove that given any real number y, there exists a positive real number x
such that y = In .v. Since lim In x = -\-<^, there exists a positive real number b so

large that \n b> \. Since lim In a = — «=, there exists a positive real number a so

small that In a < y. Therefore, because In a < y <\n b and In is a continuous


function, the intermediate-value theorem (Theorem 1, page 150) implies that there
exists a value of x between a and b such that In a = v.

Evaluation of Natural Logarithms


Until now, in dealing with the natural logarithm, we have deliberately avoided
using a calculator or tables so that you could see clearly how the properties of In
follow directly from its definition as an integral. Approximate values of In a can be
found almost effortlessly by using the LN key on a scientific calculator.*

*If a calculator isn't available, you can use Appendix D. Table 2 to evaluate natural loga-
rithms.
436 CHAPTER 7 TRANSCENDENTAL FLNCTIONS

E EXAMPLE 7 Evaluate

(a) In 7124 (b) In 0.05319

SOLUTION Using a 10-digit calculator, we obtain

(a) In 7124 = 8.871224644 (b» In 0.05319 = -2.933884870

The following examples illustrate the use of the natural logarithm function.

[SEX.\MPi.K 8 Find the exact volume V of the solid generated by revolving the
region bounded by the graphs of y = x~^^', x = \, x = 4. and y = about the x
axis. Also find the numerical value of V, rounded off to two decimal places.

SOLUTION Using the method of circular disks, we have

V=
J,
77(.v''/-)-rf.v= TT
J|
—.V
civ

Therefore, by Theorem 2 on page 429.

14

V= 77-(ln |.v|) =7r(ln |4|


- In |l|) = tt In 4
'
1

Using a calculator or Appendix D, Table 2. we find that

V= TT In 4 = 4.36 cubic units

B EXAMPLE 9 To the nearest joule, how many joules of work are required to com-
press air isothermally* according to the equation PV = C from an initial volume of
Vo = 10""* cubic meter to a final volume of V| = 10""^ cubic meter if the initial

pressure is Pq = lO'' newtons per square meter?

SOLUTION Here

C= PoV'o = (10')(10-^)= 10

and the work done during the isothermal compression is given by

W= \ P(/V= —dV = c\ — ^V=10 —dV


(see page 385). Therefore.
•*
10
I

W= 10(ln |V|) = lOdn |10^^| - In |10--'|) = 10(ln lO^* - In 10"^)


'
lO''

By Theorem 3. we have

In 10 * = -4 In 10 and In 10"' = -5 In 10

so that

W = 101-4 In 10 - (-5) In 10] = 10(5 In 10 - 4 In 10) = 10 In 10

Using a calculator (or Appendix D. Table 2). we find that

W = 10 In 10 « 23 joules

*lsothcrnial compression is compression with the temperature held constant.


SECTION 7.5 PROPERTIES OF THE NATlTtAL LOCJARITRM FtNCTION 437

Problem Set 7.5

In Problems 1 to 6. assume that In a = 0.6931. \n b = 1.0986. and In Problems 25 to 27. find the exact volume of the solid generated
In c ~ 1.6094. Use the properties of the natural logarithm to esti- by revolving the region bounded by the given curves about the x
mate each quantity'. axis. Write your answers in terms of logarithms.

1 In, 3 In aV 25 (1 + .i:-)y- = .r. .v = 1 , .r = 4. y =

26 (.t - 6)y- = ;c. .v = 7.x= 10. y =


ab
4 In 5 In Vfc 6 In c-'/2
27 (.t- + 2t)y- = .V + 1. .r = 1, .r = 4. y =

In Problems 7 to 10. solve each equation for x.


28 Given that In 10 » 2.3025851. use the differential to estimate

hi 10.007.

7 In (.V - 1) + In (.V - 2) = In 6

8 In (X- - 4) + In (.v - 2) = In 3 El In Problems 29 to 34. evaluate each logarithm.

9 2 In Cv - 2) = In .v
29 In 4126 30 In 2.704
10 In (6 - X - X-) - In (.v + 3) = In (2 - .v)
31 In 0.040404 32 In (7.321 x 10*)

In Problems 1 1 to 17. sketch the graph of each function. Indicate the 33 In (1.732 x 10"'') 34 In n
domain and the range of the function. Also indicate the extreme
points and the inflection points of the graph and any horizontal or El 35 Find numerical approximations to (a) the area in Problem 21 and
vertical asymptotes. (b) the volume in Problem 25. Round off your answers to four

decimal places.
11 g{x) = In (2 - x) 12 /;(.v) = In (-.r)
36 Show that In ( 1 + .v) = .v if |.t| is small and that the approxima-

13 Fix) = In |.v + 1| 14 G(.v) = In


—4 tion becomes more and more accurate as |.v| gets smaller and
.V
smaller.

15 H(x) = X In X 16 fix) = -^ 37 Determine the coordinates of the pwint at which the function
In X
fix) = (In .V -I- l)/.r takes on an extreme value. Is the extreme
17 Ux) = .V - In .1
value a maximum or a minimum? Is it an absolute or only a
relative extremum?
18 Sketch the graph of the equation x = In y fory > 0. Use implicit
differentiation to find the slope of the graph at the point 38 Find a general formula for the work done during the isothermal

(In 2. 2).
compression of a sample of gas from an initial pressure of Po and
an initial volume of Vq to a final pressure of P|.
19 Find the equations of the tangent and normal lines to the cur\e
= X' In the point (2. 4 39 The speed of the signal in a submarine telegraph cable is propor-
y .V at In 2).
tional to X' In ( l/.r), where x is the ratio of the radius of the core
20 A particle is moving along a straight line according to the equa- to the thickness of the winding. For what value of this ratio will
tion of motion 5 = In [8r/(4r- -t- 5)]. Find the velocity ds/dt and the speed be maximum?
the acceleration d's/dr.
40 A product sells for S25 per unit. The production cost C for .r
units is expressed by the equation C = 250 + x{6 2 In x). -I-
In Problems 2! to 24. find the exact area of the region under each
Find the output level that yields maximum profit, assuming that
curve in terms of logarithms.
all manufactured units are sold.

21 V = — between x = 5 and
1

.r = 7 41 A lake polluted by colifomi bacteria is treated with bactericidal


agents. Environmentalists estimate that t days after the treat-

ment, the number iV of viable bacteria per milliliter will be


22 V = between .v = 2 and .v = 3 given by
.V — 1

23 between x = 3 and .v = 4 A'(/) = 100 In - 30 for 1 Sf < 12

= — = = How many days after the treatment does the pollution reach a
24 V between x 2 and .v 3
Zx — 1 minimum?
438 CHAHTKR 7 TRANSCENUKNTAI. FUNCTIONS

7.6 The Exponential Function


Because the natural logarithm is an increasing function (Theorem 4, page 433), its

graph can intersect a horizontal line y = k only once (Figure la). Therefore, by the
horizontal-line test. In is an invertible function. Later we prove that In .v is exactly
the same as log,, x. from which it will follow that the inverse of In is the function
/(.v) = e\ In anticipation of this result, we refer to the inverse of In as the exponen-
tial junction.

DEFINITION 1 The Exponential Function

The inverse of the natural logarithm function is called the exponential function,
denoted by exp.

Thus, the graph of exp is obtained by reflecting the graph of In about the line y = .v

(Figure lb).
Since the domain of In is the interval (0, ^), it follows that the range of exp is

(0, ^). Likewise, since the range of In is the set IR of all real numbers, it follows that
IR is the domain of exp. From the graph of y = exp .v (Figure lb), we see that exp is

an increasing function and that exp .v > for all values of .v in R. Also,
SECTION 7.6 THE EXPONENTIAL FlINCTION 439

Properties of the Exponential Function

Because exp is the inverse of In, we have

exp ( In A )
= A for a >
and In (exp A) = A for all a in R

These relations can be used to establish many of the basic properties of the exponen-
tial function. For instance, we have the following theorem.

THEOREM 1 Basic Properties of the Exponential Function

If A- and y are any two real numbers and if k is a rational number, then

(i) exp (.V + y) = (exp A)(exp y)

(ii) exp ikx) = (exp a:)*

(iii) exp (a - y)
= exp X
exp y

Figure 2
440 CHAPIER 7 TRANSCENDENTAL FUNCTIONS

Put X = 1 in part (ii) of Theorem 1 to obtain exp k = (exp 1 )*. By Definition 2.


exp 1 = e; hence, exp k = <>*.

By Theorem 2, e' = exp v holds for all rationalnumbers x. so it is reasonable to


ask whether it also holds when x is irrational. However, as we originally pointed
out, if .V is irrational, it is perhaps not clear just what is meant by e' anyway. To
clarify matters once and for all, we simply define e' to be equal to exp .v when x is

irrational.

DEFINITION 3 Definition of e" for Irrational Values of x

U.x
SECTION 7.6 THE EXPONENTIAL FIWCTION 441

SOLUTION Using a 10-digit calculator, we find that

(a) e^ = e = 2.718281828
(b) e^ - = e'^'^213562 = 4.113250377
(c) e-5 0321 = 6.525093476 x 10"'' = 0.006525093476

Differentiation of the Exponential Function

From now on. we can use either the notation e' or the notation exp x for the inverse
of In evaluated at .v. In the following theorem, we obtain a most remarkable result:
The exponential function is its own derivative.

THEOREM 3 The Derivative of the Exponential Function

D,e' = e'

Let

y = e^

so that

X = In V

Differentiating both sides of the last equation with respect to x, we obtain

Ox.v

that is.

DxV

D^e" = e'

Combining the result of Theorem 3 with the chain rule, we see that if u is a
differentiable function of .v. then

D,e" = e" D,u

EXAMPLE 3 If y = e'^*\ find D^y.

SOLUTION DvV = Dve"^"^-" = e'^*''D:,(x- + x) = e'^^^lx + 1)

EXAMPLE 4 Find/'(jr) if/W = e™^\


SOLUTION f'(x) = e™^ "^D, cos X = e"^"^ '(-sin .v) = -sin .v e""

EXAMPLE 5 If xe^ = tan ' .v, use implicit differentiation to find dy/dx.

*A more rigorous proof may be given by using the inverse-function theorem (Theorem 1.
page 406).
442 C HAPTKR 7 TRANSCENDENTAL FUNCTIONS

SOLUTION —d
dx
{xe^) = —d
dx
tan ' x e* + -ve*
d\
dx 1 + .V-

Solving for dy/dx. we obtain

d\ I e

dx ( 1 + x~)xe^ xe^ ( 1 + .V) tan .r x

EXAMPLE 6 A medical team predicts that the percent P of a city's population that

will be infected with a communicable disease t days after the disease is first detected
is given by
PU) = 10/e"""' for 1 s,< 15

How many days after detection will the maximum percentage of the population be
infected?

SOLUTION We have

P\t) = \Qte- •"''(- to) + lO^"-'/'" = ^-'/'°(10 -

Recall that e^ is positive for all values of .v; hence.

for all values of /. It follows that / = 10 is a critical value and that

P'(t) > for 1 <f < 10

whereas P'U) < for 10 < < f 15

Therefore, by the first-derivative test. P takes on a maximum value 10 days after the
disease is first detected.

Integration o{ e"

Theorem 3 can now be used to obtain the following important integral formula.

THEOREM 4 Integration of e"

:" (/(( = e" + C

By Theorem 3, D„c'" = e"\ that is. e" is an antiderivative of e".

EXAMPLE 7 Evaluate / e'*" dx.

SOLUTION Put M = Ax, so du = 4 dx and dx = \ dii. Thus, by Theorem 4,

I e^' dx = j e"(-i
du) = ^ j e" du = \e" + C = ie"*' + C

EXAMPLE S Evaluate / x-^e"' dx.

SOLUTION Put u = .v\ so du = 3.V- dx. or .v- dx = ^ du. Thus,

I x'e" dx = ^ \ e" du = ^e" + C = 3^' + C

e' dx
EXAMPLE t Find
V \
- e-'
SECTION 7.6 THE EXPONENTIAL FUNCTION 443

SOLUTION We put u = e\ noting that du = e^ dx and that e-" = {e'^f = u^.


Therefore.

du
sin u + C = sin ' e' + C
J VT^T^ J Vl - ir

EXAMPLE 10 Find the volume V of the soUd generated by revolving the region
bounded by the curves y = e~'*\ .v = 0. -v = 2. and y = about the x axis.

SOLUTION By the method of circular disks.

V= TTie^-^')- dx = TT e~^' dx
'0 A)

We put u = — 8.V. so that du = -8 dx and dx = — I du. When x = 0. « = 0, and


when .V = 2. = — 16; hence.
!/

f-'" / 1 \ rr (-'" n h'^


V = 77 e" I duj = e" du = e"\

= (e"'" - 1)
= — (1 - e"'") cubic unit

Problem Set 7.6


444 t IIAITKR 7 TRANSCENDENTAL FUNCTIONS

30 Show that v = 5xe *' satisfies the differential equation second, find the rate at which the y intercept is changing when
(d-\/dx-) + Udy/dx) + 16v = 0. the X intercept is 10 units.

EI59 If the value of a certain piece of property at time / years is given


In Problems 31 to 48, evaluate each integral. (In some cases a suit-
by the equation V = $20,000 - $10,000<'"" ", find the rate of
able substitution is suggested.)
change of V with respect to t when t = 5 years.

31
J
<'' d\ O 60 The electric current / in amperes flowing in a series circuit hav-
ing an inductance L henrys, a resi.stance R ohms, and a constant

electromotive force E volts (Figure 3) satisfies the equation


33 e'"^' dx
J
E E I

35 xe^'' dx, u = 5x'


J
where ( is the time in seconds after the current begins to flow. If
£= 12 volts, R = 5 ohms, and L = 0.03 henry, sketch the
„ f e'dx
graph of / as a function of /.

39/-^!^
+ 4
J Ve'
Figure 3 R ohms
^coi _ ^2
41
J
,
< j,^j,- ^, ^^.^ j^ j,jj| ^. (-"•'^^ '
sec .V tan .v dx -AAAr-
I rlnS
43 I e-'dtt 44 c ''^v
/ amperes
•'o

45 .r'(c'' + 1 ) dlr
1-
^ 1 1 1

47 '
cos 2v dx
-'n

El In Problems 49 to 5 1 . sketch the graph of each function and indicate 061 Atmospheric pressure at altitude h feet above sea level is given
the intervals where the function is increasing, decreasing, concave by f = i5<.~o "00^'' pounds per square inch. A jetliner is climb-
upward, and concave downward. Also, locate all extreme points, ing through 10,000 feet at the rate of 1000 feet per minute. Find
points of inflection, and asymptotes. the rate of change of external air pressure as measured by a
49 = e'^' 50 fix) = e"^ gauge on board the plane.
fix)

51 fix) = xe~' E 62 Carbon 14 decays radioactively according to the equation y =


^,^-0 oooi2i2f
y g^ams is the amount left after I years and
^^,|,g[.g

52 Find the area under the curve = - between x = and vo grams is the initial amount, (a) Sketch a graph of y as a
y c'~' .v 1

X = 5.
function of r for < s r 10,000 years, (b) Of a 10-gram sample
of carbon 14, how much will be left after 10,000 years?
53 Find the area bounded by the curves y = e~\\ = e '\ and x = 1

El 63 The audience for an advertising campaign num-


total potential
54 Find the volume of the solid generated by revolving the region bers 10,000. The average revenue per response is $3, and the
under the curve v = e^ between x = and x = 2 about the x cost of the campaign is $300 per day, plus a fixed cost of $500.
To maximize profit, find the number of days that the campaign
should continue if the number of responses y is given in terms of
55 Finish the proof of Theorem 1 by demonstrating that
the number of campaign days i by y = (1 - e~''-'')(10,000).
exp (.V - y) = exp jt/exp y.
El 64 The concentration C of a drug in a person's circulatory system
56 Find the arc length of the graph of y = ie' + e'')/2 from (0, 1 decreases as the drug is eliminated by the liver and kidneys or
to (1. ie + e'')/2). absorbed by other organs. Medical researchers often use the

57 Let fix) = - - Show > > equation C= Co? *'


to predict the concentration C at a time ;
e' \ x. that fix) if .r and that
hours after the drug is administered, where Co is the initial con-
fix) s if .r s 0. Use these facts to prove that e' S: I +x and
centration when = and k is a constant depending on the type
that e~' 2 1
-X for all values of .v.
t

of drug. If C|i = 3 milligrams per liter and <: = 0.173, (a) sketch
58 If the X intercept of the tangent to the curve y = e '
at the vari- a graph of C as a function of / for s < / 4 hours and (b) find C
able point ip. q) is increasing at the constant rate of 5 units per when / = 4 hours.
SECTION 7.7 EXPONENTIAL AND LOCARITHMIC FUNCTIONS WITH BASE OTHER THAN 445

7.7 Exponential and Logarithmic Functions with


Base Other Than e
In this section we use the natural logarithm and the exponential function to define /?*

and log ft
X for values of the base b other than e. Of course, fc* is already defined
when b> Q and k is a rational number. In fact, we have the following theorem.

THEOREM 1 ft* for 6 > and it Rational

If fc > and k is a rational number, then

By part (ii) of Theorem 1 in Section 7.6,

[exp (In /j)]^ = exp (k In b)

that is,

Since
e'"" = b

it follows that
l,k = gk In h

Theorem 1 gives us the key to the proper definition of b^ when a is irrational.

DEFINITION 1 Dennition of 6^ for Irrational Values of x

If /? > and A- is an irrational number, we define

As a consequence of Theorem 1 and Definition 1, if the base b is positive, then

holds for all real numbers a. Notice that

for all values of the exponent x.

El EXAMPLE 1

\/2
(a) Using a calculator with a v" key, evaluate 7r

(b) Evaluate e^-'" ^, and thus confirm that tt^'- = e^ -'" '^.

SOLUTION

(a) TT^- = 5.047497266


(b) e^'"'^= 5.047497268
Notice the discrepancy in the last decimal place caused by inaccuracies inherent in

the calculator.
446 CHAPIER 7 TRANSCENDENTAL FUNCTIONS

Basic Properties of b'

Using the equation fc' = f"'"'' and the i^nown properties of the exponential func-
tion, we can now derive the basic properties of b\

THEOREM 2 Laws of Exponents


SECTION 7.7 EXPONENTIAL AND LOGARITHMIC FINCTIONS WITH BASE
OTHER THAN . 447

In applying the general power rule, it is important to realize that the base is the
variable and the exponent is constant. On the other hand, if the base is constant and
the exponent is the variable, you can find the derivative by using the formula

For instance.

D^2' = D^e"^"- = r"^'"-D^(.v In 2) = 2'(ln 2)

More generally, we have the following theorem.

THEOREM 4 Derivative of b"

Iffcis^a positive constant, then D^fc' = b" In h.

PROOF D,b^ = D.e''"'' = e""*D,(.v In b) = bmn =


b) ft' In b

Of course. Theorem 4 can be combined with the chain rule to obtain the formula

DJr' = h" D,u


In b
2
EX.4MPLE 4 If v = 2'-. find dy/dx

d\
SOLUTION = 2>^"(ln 2)(lv) = 2x2'^ In 2 = a-2''^' In 2
dx

EXAMPLE 5 Let /(A) = 3'^'. Find /'(A).

SOLUTION /'(A) = 3'^" "


In 3 sec- x

EXAMPLE 6 Find D,(5''V''" '^).

SOLUTION DA5-'e""') = 5-; In 5(3x^)e''"' + 5^>"' cos x


= 5"^>*'" "^(3a- In
5 + cos x)

The differentiation formula D,b' = /,' In b yields a corresponding


r t integral for-
mula as follows: g

b' dx
b'
,
Inb
,
d.x = -—-
If b'' In b dx = ]

b' + C
in * In b J In b
Thus, replacing .v with u. we have

J
+ C provided that b > and b 9^
\nb ]

In Examples 7 and 8. evaluate the given integral.

EXAMPLE 7 7^ dx
I
']

SOLUTION T dx = —— "
= ^^ ~ ^
^ ^-
-"i In 7 I
, In 7 In 7
448 C HAPIER 7 TRANSCENUKNTAl, KUNCTIONS

EXAMPLE 8 I
5"'"-' COS Iv clx

SOLUTION Put u = sin Iv, so that dii = 2 cos Iv dx. Thus,

f 5"'"-'
,
cos 2v dx = —If 5" \
(/// = —-— + C = ——- + C
5" 5^'"-'

J 2 J 2 In 5 2 In 5

Logarithmic Differentiation
The properties of the natural logarithm (as developed in Section 7.5) are useful for
finding the derivatives of functions involving products, quotients, and powers of
other functions. The technique, called logarithmic difTerentiation, works as fol-
lows.

Procedure for Logarithmic Differentiation

To find the derivative of a function /. carry out the following steps:

Step 1 Write the equation v =/(.v).

Step 2 Take the natural logarithm on both sides of the equation.


Step 3 Differentiate the resulting equation implicitly with respect to .v.

In Examples 9 and 10. use logarithmic differentiation to find dy/dx.

EX.AMPLE 9 y = .\

SOLUTION Taking the natural logarithm on both sides of the equation y = .v', we
obtain
In y = In .v' = .v In .v

Differentiating the equation In y = v In .v on both sides with respect to .v, we have

1 dx I
'— = .V 1- In .V = I + In ;«:

y dx X

Therefore,
dv
-^ = v(l + In.v) = .v'(l + In.v)
dx

{X- + 5)(5.v + 2)-'^-

EXAMPLE 10 V
V'(3.v-I- DU' + 2)

In y = In (x' + 5) + In (5a: + 2f^- - In N/{3.v + l)(.v' + 2)


= In {x~ + 5) + 2 In {5x + 2) - illn (3.v + 1) + In (.v' + 2)]

so that

.1
1 rfv
'-
dx
= —^
x~
1

+ 5
(2v) + —
/ 3 \

V 2 /
1

+ 2
(5)
4
1
r

L 3.v
1

+
(3) + —7
1

+ 2
(3.V-)
J
y 5.V 1 .v'

or

dy^ _ r Iv 15 3 3.V- 1
~ -^
dx L .V- + 5 2(5.v + 2) 4(3.v + 1) 4U' + 2) J
SECTION 7.7 EXPONENTIAL AND LOtJARITHMIC FUNCTIONS WITH BASE
OTHER THAN . 449

Therefore,

jiy^
^ (.v- + 5)(5.Y + 2)^'''-
r Zv 15 3 3a:-
dx \/(3.x+ + + ^ ~ 1
l)(.v'' 2) I X- 5 2(5.v + 2) 4(3.v + 1) 4(
4(A-' + 2) J

The Function logo


The base =
2.71828 ... is preferred in all work with logarithms or
e
exponents in
calculus because of the relative simplicity of
the resulting differentiation and inte-
gration formulas. However, other bases are used
in some applications. For mstance
logio IS used m chemistry for calculating the pH of
acids and bases; in computer
science, log, and logs are used in reckoning with
"bits" and "bytes" of informa-
tion.

Recall the definition of a logarithm to the base a: >


If a 0, a 5^ I, and .v > 0. then
we say that y is the logarithm of x to the base
a and write

log^.r

to mean that

In particular, v = log, .v means that e> = x: that is. means that v =


it In x. In other
words.

In X = log, X
The following useful theorem can be used to convert logarithms
from one base to
another.

THEOREM 5 Base-Changing Formula

If a > 0, fc > 0, a 5^ 1, ^ 5^ 1. and .V > 0, then

log* AT
log„Ar =
lOgfofl

PROOF We have

Hence, by the definition of logarithm,

logo X logft a = logft X


and it follows that

log/, .V
lo&, .V =
log/, a

If we put /? - e in Theorem 5 and use the fact that =


log,, .v In .v. we obtain the
important formula

In.v
log„ X =
In a
450 CHAPIER 7 TRANSCENDENTAL FUNCTIONS

Using this formula and the properties of the natural logarithm, you can verify the
properties of log,,, such as the following:

log,, .vv = log,, A + log,, y ii)g„ —A = log,, A - log,, y log,, a' = y log,, a

(Problem 59). Since scientific calculators ordinarily have keys only for In and logio,
you must use the base-changing formula if you want to calculate logarithms to other

bases.

B EXAMPLE II Use a calculator and the base-changing formula to find log2 3.

SOLUTION log. 3 = -^2 =


In
1.584962501

By using the base-changing formula, we can derive the following rule for differ-
entiating logarithmic functions.

THEOREM 6 Derivative of Logarithmic Functions

Let u be a differentiable function of x with u > 0, and suppose that a > with
a ¥= \. Then

D, log,, u = D,u
(/ In a

PROOF D, log,, u = D,
In M
= 1

D, In u =
1 D,u
— = -— 1
D,u
In a In a In a u u In a

EXAMPLE 1 2 Find —dv- if V = logio (x~ + 5).


dx

SOLUTION By Theorem 6,

^v 1

(.V- + 5)
dx {.V- -I- 5) In 10 dx (a- -I- 5) In 10

EXAMPLE 13 Find D,(log2 sin a).

cos .r cot X
SOLUTION D, log2 sin A =
sin X In 2 In 2

Problem Set 7.7


El 1 Use a calculator with a y' key to find the value of each quantity (a) a'a^ = a'*>' for a = 3.074, a: = 2.183, y = 1.075
as accurately as you can.
(b) a'*> = aV for a = 2.471. x = 5.507, y = 0.012
(a) 2^ (b) 2-^- (c) 2" (d) 2"
(e) Vl"^ {() 77" (g) (V3)-^' (h) a.OlS?-""" (c) (a'V = a" for a = 1,777, a = -2.058, y = 3.333

[c] 2 Use a calculator with a y" key to verify each equation lor the
(d) a'-' = -^ for a = V2, A = VS, y = V3
indicated values of the variables.
SECTION 7.7 EXPONENTIAL AND LOGARITHMIC FUNCTIONS WITH BASE OTHER THAN e 451

(e) (abY = a'b^ for a = VV. h = tt. x = \'tt


dx. u = In .V

(f)
("t) = tt for a = 2 + 77, ft = Vl - 1 . .V = Vs - Vs *'^^'
44 " sec .V tan x dx
J

'"'"'
In Problems 3 to 24. differentiate each function. '
CSC" X dx

4 g(t) = r"-= 46 ''"'(1 + \nx) dx


J
= 6"'' 6 f(x) = 2'^
5 h(x)
5'-' dx

7 gU) = 3^+' 8 h(i) = (r= + D^""


-I '0

3"'^'"
9 G(f) = 5''"'
10 H(x) = ymx j,Qj ^ ^,,

11 h{x) = (.V- + 5)2-''^ 12 gU) = (sin r)3"'

049 Using a calculator and the base -changing formula, evaluate


13/(-v) = 4^ 14 /i(.v) = 2'' cot x
(a) Iog2 25 (b) log3 2 (c) logg e

15 g(.v) = In (5" + 5"') 16 h(t) = s/r (d) Iog„ 5 (e) logv2 0.07301

y - 1 50 Show that for .v > 0. In .v = M logm x. where M = In 10.


17 /U) 18 Fix) = 4"-''
In (.V- + 8)
3> + 1
E151 Which number is larger, e^or tt'?

2t
= 20 = logj (In cos
52 Find the maximum value of (In x)/x for j: > 0, and use the result
19 h(i) logui /(/) /)
1 + f
to answer Problem 51 without evaluating e" or tt'.

21 Fiu) = 3'^" logs M 22 g(t) = Vlog5 t 53 Prove that b^b^ = f > for b>0.
logj U- + 5) 54 Prove that b^' = \/b' for > 0.
23 fix) = 2A Fix) = CSC
ft
.V log, (A--* + 1

x + 2
55 Prove that In ft' = x In ft for ft > 0.
In Problems 25 and 26, use implicit differentiation to find dy/dx.
56 For what value of .v is/(.v) = .v"", .v > 0. a maximum?
25 .v(2') + y- = 5 26 3" = x^ 57 Prove that if a > and ft > 0, then log,, ft = l/logj, a.

58 Use Problem 57 and Theorem 6 to show that


In Problems 27 to 38, use logarithmic differentiation to find dy/dx.

/ log„
^ e \

27 .v = :t" 28 y = (cos x)' DJogaM = I D,u.

29 .V = (sin x-)^" 30 y = (.v+ D' 59 Prove that if a > 0, a 5^ I, .v > 0, and y > 0, then

31 y = (.v' + 4)'"' 32 y = (sin x)'"^'


(a) logo .rv' = logo X + log^ v. (b) log^ —X = log^ x - log„ v,
y
33 y = (X- + 7)-{6x^ + D* 34 y = -v" sin .v' cos (3.v + 7)
and (c) loga x^ = y log^ x.

sin .V V 1 + cos X
= tan' .r
60 Suppose that a > and a # is a given constant. Find D^ log^ a
35 >' 36 v 1

/cos X Vl -4 for A- > 0, .V # 1

x-\/x- + 7 / sec jc + tan jc 61 Find the volume of the solid generated by revolving the region
37 >'
= 38 y
\/ll.v + 8 \ sec X — tan .r
under the graph of y = 3' between x = and x = 2 about the
X axis.

In Problems 39 to 48, evaluate each integral. (In some cases an 62 If y is a differentiable function of .v, then the (point) elasticity of
appropriate change of variable is suggested.)
y with respect to x is defined to be the limit

39 I
3-
' dx. u = 5x 40
/^ A-.-0 A.V/.V

41 7''^-'"'(.r-'' + 3.V-) dx, u = x^ + 4.v of the proportional change in y to the proportional change in .v.
J
Show and y are positive, then the elasticity
that if .V is given by
42 3"^ ' sec" X dx dY/dX, where Y = logj, y and X = log^ .v.
452 CHAPTER 7 TRANSCENDENTAL UNI TIONS

7.8 Hyperbolic Functions


Certain combinations ot exponential functions, which are related to a curve called a
hyperbola* in much the same way as trigonometric functions arc related to a circle,
prove to be important in applied mathematics. These functions arc called the
hyperbolic functions, and their similarity to the trigonometric functions is empha-
sized by calling them hyperbolic sine, hyperbolic cosine, and so on. The detlni-

tions of the hyperbolic sine and the hyperbolic cosine, abbreviated sinh and cosh,
are as follows:

sinh .V
=
SECTION 7.8 HYPERBOLIC FUNCTIONS 453

Figure I
For large values of -v, e~" small, and so the graph of y =
is ver>' cosh .v is very close
to but above the graph of y = e^/2 (Figure lb). Unlike the trigonometric sine and
cosine, the hyperbolic sine and hyperbolic cosine functions are not periodic.
The remaining hyperbolic functions are defined by analogy with the correspond-
ing trigonometric functions as follows:

sinh .V £'' — e
*
454 ( HAKIKR 7 TRANSCENDENTAL Kl'NCTIONS

By inverting the differentiation rules for the hyperbolic functions, we can obtain
integration formulas. Thus;

sinh 14 (III = cosh u + C cosh II dii = sinh ii + C

sech" u dii = tanh u + C

EXAMPLE 5 Evaluate / sinh 5.x dx.

SOLUTION Put It = 5a, so that dx = ^ du and

sinh 5.V dx = sinh u (5 du) = 5 sinh u du = 5 cosh u + C

= 5 cosh 5x + C

The Inverse Hyperbolic Functions


By analogy with the six inverse trigonometric functions, there are six inverse hy-
perbolic functions. Unlike the analogous trigonometric functions, the graphs of
sinh. tanh. coth. and csch satisfy the horizontal-line test, so there is no problem
about the existence of sinh ', tanh"'. coth"'. or csch"'. The graphs of sinh. tanh.
coth, csch. and their inverses are sketched in Figure 2. In each case, the graph of the
inverse function is obtained by reflecting the graph of the function about the line
y = X.

The hyperbolic cosine and its reciprocal, the hyperbolic secant, are not invertible
because their graphs do not satisfy the horizontal-line test. However, the portions of
these graphs that lie to the right of the y axis do satisfy the horizontal-line test;

hence, their reflections across the line y = v are taken, by definition, to be the
graphs of cosh"' and sech"' (Figure 3).

Since D, sinh .v = cosh x > 0, it follows that sinh '


is a differentiable function
by the inverse-function theorem (Theorem 1, page 406). Now, suppose that m is a
differentiable function of .v, and let

V = sinh" so that u = sinh V

Implicit differentiation of the last equation gives

D^u = cosh y D^
1
Hence, D,v = D,u
cosh V

Since cosh" y - sinh" y = 1 and cosh y > 0, we have

cosh y = V 1 -I- sinh" y = Vl -I- u~

and it follows that

D,v = ,
D,u
\\ + u-

D. sinh =- D,u
V 1 -H M
SECTION 7.8 HYPERBOLIC FLNCTIONS 455

Figure 2
456 tlUPTKR 7 TRANSCENDENTAL FUNCTIONS

Similar differentiation rules may be derived for the remaining inverse hyperbolic
functions; for instance.

Dj cosh '
u = D,M if u > 1

Vm- - 1

D, tanh '
ii - D,« if \u\ < 1

1
- //

(Problem 50).

EXAMPLE 6 If V = tanh ' (sin 2v). find

dv d
SOLUTION ^- =
1

—T^ — sm 2.V
dx 1
- sin~ Iv dx
2 cos Zx
2 sec 2x

Problem Set 7.8

In Problems 1 to 8, prove each identity. In Problems 19 to 22, use implicit differentiation to find dy/dx.

1 sinh (-.r) = -sinh .v 2 cosh (—x) = cosh x 19 .x~ = sinh y

3 sinh (x + v) = sinh .v cosh y + sinh y cosh x 20 sinh x = cosh y

4 cosh {x + >') = cosh x cosh y + sinh x sinh y 21 sin X = sinh y

5 1 - tanh" x = sech" x 6 coth" .v - 1 = csch" r 22 tanh" .v — 2 sinh y = tanh y

1
7 e' = sinh .r + cosh x 8 sinh (In x) =
Ix In Problems 23 to 30, evaluate each integral,

El 9 Use a calculator with an e' (or EXP) key to evaluate the follow- cosh 7.V dx
ing quantities as accurately as you can.

(a) sinh 1.2 (b) cosh (-1.4) (c) tanh 0.7 sinh -j- dx

(d) coth 1.3 (c) sech 0.6 (f) csch (-0.9)


sech" 3.V dx
El 10 Sketch the graphs of (a) y = tanh .v, (b) y = coth .(, (c)

y = sech .v, and (d) y = csch .v. sech" vx


dx
VTx
In Problems 11 to 18, differentiate each function.
sinh 5.x
dx
11 fix) = sinh (3.t- + 5) 12 g(x) = cosh (In x) cosh"' 5.r

13 fU) = In (sinh /') 14 f(u) = e'" tanh u


sinh'" 3.V cosh 3.r dx
15 hii) = tanh e^' 1

cosh' X sinh .v dx
16 F(r) = sin '
r tanh (3r + 5)

ruinh.,
jj
17 G(s) = sin '
(tanh .v) 18 fix) ~ sinh-* V cosh X dx
J, \ + r J
SECTION 7.8 mPERBOLIC FINCTIONS 457

31 Evaluate / cosh (In r) d\. {Hint: Use the definition of cosh be- 52 Use the formula in Problem 47 to find t he area of the region
fore integrating.) bounded by the curve y = 16/V 16 -i- .v" and the lines x = 0.

y = 0. and x = 3V'2.
32 Derive differentiation rules for (a) cosh ii. (b) coth ii. (c) sech ii.

and (d) csch u. 53 When a flexible cord or chain is suspended from its ends, it

hangs in a curve called a catenary whose equation has the form


33 Show that if A. B, and k are constants, the function
=
y a cosh (x/a). where a is a positive constant (Figure 4). Find
V =A sinh Lx + B cosh kx is a solution of the differential equa-
the length of the catenary between (-b. a cosh (b/a)) and
tion v" — k-y = 0.
(b. a cosh (b/a)).
34 Find the area under the curve ^ = sinh .v between .v = and
.v= 1.
Figure 4

y =a cosh —
In Problems 35 to -14. differentiate each function.

35 f(.x) = sinh^' .r' 36 g(.v) = cosh"' (sec .x)

37 hi.x) = cosh '


— 38 Fit) = tanh"' (sin ;)

39 G(i) = tanh"' 5r

40 H(t) = In Vr - 1 - r tanh"' t

41 fix) = .X cosh"' e'

42 glx) = x sinh '


.v - V 1 -I- .v

43 h{u) = u tanh"' (In w)

54 The famous Gateway Arch to the West in St. Louis has the shape
cosh"' w
44 f(»f) of an inverted catenary (see Problem 53). The equation of any
II- -I- 4 such arch takes the formy = li + all - cosh {x/a)].where a is
a positive constant, h is the height of the arch, the two bases of
45 Let V = sinh"' .v, so that .v = sinh y = 2.(e^ — «"-'). (a) Show on the y
the arch lie on the x axis, and the apex of the arch lies
that (e')' — 2re' -1=0. (b) Recalling that e^ is always posi- axis. Show that the distance between the two bases of such an
tive, solve t he quad ratic equation in (a) for e^ and thus show that
arch is 2a cosh"' -i- {h/a)] units.
[1
e' =X+ V.v" -t- 1. (c) Using the result in (b). show that

sinh"' .r = In (x + V.v" -i- 1).

46 Proceeding as in Problem 45, show that

tanh"' .V = ^ In
'—
for \x\ <

In Problems 47 to 49, justify the given integral formula by differen-


tiating the right side.

47 , / , = sinh" '
—+C for a >
J Va' + X- a

48
(
—— d-x
r = —a tanh"' —a
1 . , -T
-I- C

for
I

|.r|
I

< a
J a' X'

r dx
cosh ' — -(-
C for .V > a >
J \x- - a- a

50 Derive the differentiation rules for (a) cosh"' and (b) tanh"

51 Use the formula in Problem 47 to evaluate —


J V9+7,
458 CHAPTER 7 TRANSCENDENTAL FUNCTIONS

7.9 Exponential Growth and Decay


One of the most important applications of exponential functions occurs in the study
of a quantity that grows or decays at a rate proportional to the quantity itself. For
instance, money compound interest increases at a rate that is propor-
invested at

tional to the amount of money earning interest. We discuss such growth and decay
in this section. We begin by establishing some useful technical results.

The Exponential Function as a Limit

On page 426 we provisionally defined e to be the limit as « ^ +^ of [ 1 + 1/m)]".


(

However, at that stage of the game, this was not really legitimate since we didn't
have Definition 1 on page 445 at our disposal, so the meaning of [ 1 + ( l/u)]" when
M is irrational was in doubt. Now we can prove the following theorem.

THEOREM I e as a Limit

lim (l +— ) = lim ( 1 + —

In the expression [1 + (l/u)]", we put A.v = 1/h. so u = 1/A.v and

(l +— )" = (1 + A-v)'/^'

Notice that « — +^ as A.t -* 0^ and that u —> -^ as A.y -^ 0~. Hence, the equa-
tions to be established can be written

e= lim (1 -I- A,v)"-^> = lim (1 -F A.x)''''^^

or simply as the single equation

e= lim
^0
(1-1- A.t)'''-^"^
A-i

To prove this, we use Definition 1 on page 445 to write

(1 -I- A.v)''^> = eii-i""""^-^"' = exp In 1 1 -(-


A.v)
L A.v

The proof will be complete once we show that

lim In ( 1 -H A.V) I
= 1
Av--o L A.V J

for then

lim (1-1- A.v)'/-^"^ = exp lim In (1 -^ Av) = exp 1


=
I

will follow from the continuity of the exponential function. Thus, let

/(.v) = In A- for X >


SECTION 7.9 EXPONENTIAL GROWTH AND DECAY 459

SO that /(I) = In 1=0 /'W = - and /'(I) = 1

r 1 /(I + Aa) _ ,.
/(I +A.r)-/(1)
]

and our proof is complete.

The Exponential Function as a Limit


THEOREM 2

•i"^
<i+J ^^'^-A'^T

= Thus, we can assume that a ^ 0. First


PROOF The equations obviously hold when a 0.

we consider the case in which a > 0, Put u =hja, noting that « -^


+oc as /i ^ +0°.
Therefore,

= lim [(l+-)

= by Theorem By Theorem on
We put V = [1 + (1/m)]". noting that _
lim^ v e 1
.
3

a continuous function of
a differentiable function of v; hence,
it is
page 446, v" is

V. Therefore,

lim 1+— = lim v° = ( lim v)" = e"

Similar arguments take care of the cases in


which a < and in which h^ -oo

(Problem 50).

EXAMPLE 1 Evaluate each limit:

(a) lim (l+5;f)'/' (b) liin (l+—

SOLUTION

(a) Let h = l/.v, so that .v = l/h, and note that h-^+^ as .x ^0^ Thus,

^'
lim (I +5x)'/>= lim (l + Y =

(b) ^lim_ (l + y) -, lim. [('+7)1

Poisson
El EXAMPLE 2 Telephone calls coming into a certain switchboard follow a
probabilin distribution, averaging c calls per minute.
The theory of probability
one call comes into
gives P= lim c[ 1 - (c/n)]"" as the probability that exactly '

the switchboard during any given 1-minute period.


Find F if c = 4 calls per minute.
460 CHAPTER 7 TRANSCENDENTAL FUNCTIONS

-—] <• lim



(l + —^) ^
/(
' /, • ' \
n '
ce
=
SOLUTION P=
—lim
^
(1--)
=
1- (l--)
, = ce

by Theorem 2. For c = 4. we have P = Ae'* = 0.073 = tooo. and we could expect


exactly one incoming call during approximately 73 of 1000 one-minute periods.

Compound Interest and Present Value


Suppose that a principal P dollars is invested for a term of t years at a nominal
annual interest rate r compounded n times per year. Thus, the interest is peri-

odically calculated and added to the principal. The time interval between successive
conversions of interest into principal. (l/«)th of a year, is called the conversion
period.
At the end of the first conversion period, an interest of P{r/n) is added to the

principal, so that the value of the investment is P + P(r/n) = P[\ + {r/n)\ dollars.
At the end of the second conversion period, an interest oi P[\ + (r/n)](r/n) is
added, and the value of the investment is

p{\ + —] + p{\ +—)( — = p{\ +— )(l +—n =p(l +—n dollars


n ^
) )
\ / V ' ' '
„ II ^ ;i '

At the end of the third conversion period, an interest of P[ 1 + (r/n)\-(r/n) is added,


and the value of the investment is

p[\ +-)' + pi \ +-)'[-]= p{\ +-)"(l +-)=p(l +-) dollars

and so on. Evidently, at the end of K conversion periods, the value of the invest-
ment is

p(l -I-—) '


dollars
^ II

Over the term of / years, there will he K = nt conversion periods, so the final value
5 dollars of the investment will be given bv

S =P

IB EXAMPLE 3 If you invest P = $500 at a nominal annual interest rate of 8 percent


(that is, r = 0.08), what is the final value S of the investment after a term of f = 3

years if the interest is compounded (a) quarterly, (b) monthly, (c) daily?

SOLUTION
(a) Here m = 4. so 5 = 500(1 + (0.08/4)]^'" = 500(1.02)'- = $634.12.

(b) Here n = 12, so S = 500(1 -H (0.08/12)1'-'" = $635.12.

(c) Here n = 365, so S = 500(1 -I-


(0.08/365)l'''-"" = $635.61.

When a bank offers compound interest, it usually specifies not only the nominal
annual interest rate r, but also the effective simple annual interest rate R. that is, the
SECTION 7.9 EXPONENTIAL GROWTH AND DECAY 461

rate of simple annual interest that would yield the same final value over a 1-year
term as the compound interest. Thus.

)"
Pd +/?) = p(l +— or l+^ = (l+—
Therefore.

«= 1+-

B EXAMPLE 4 A bank offers savings accounts at 8 percent


nominal annual interest
compounded quarterly. Find the corresponding effective simple annual interest
rate R.
r\" 0.08 \-*

SOLUTION

/?=ll-l-—
/,
I -1=1+ (
- 1 = 0.0824 = 8.24%

Money that you will receive in the future is worth less to you than the same
amount of money received now, because you miss out on the interest you could
collect by investing the money now. For this reason, economists use the idea of the
present value of money to be received in the future. If there is an opportunity to
invest P dollars at a nominal annual interest rate r compounded n times a year, this
principal plus the interest it earns will amount to 5 dollars after t years, where

5 = p(l +—n
Thus. P dollars in hand right mnv is worth S dollars to be received t years in the
future. Solving the equation above for P in terms of S. we obtain the equation

P = S\\ +

for the present value P dollars of an offer of S dollars to be received t years in the
future.

E EXAMPLE 5 Find the present value of $500 to be paid to you 2 years in the future
if investments during this period are earning a nominal annual interest of 10 percent
compounded monthly.

SOLUTION Here S = $500, r = 0.10, n = 12, ? = 2, and

^= 5 1 + — 500 1 $409.70

Some banks offer savings accounts with interest compounded not quarterly, not
weekly, not daily, not hourly, but continuously. The formula for continuously
compounded interest is obtained by letting «-^ +x in the formula for compound
interest. Thus, using Theorem 2. we have

S = lim P
(-ir=.-''[(-i)T=i-(-i)T
= Pie')' = Pe"
462 CHAPTER 7 TRANSCENDENTAL FUNCTIONS

Therefore, if a principal of P dollars is invested for a term of t years at a nominal


annual interest rate r, the final value S dollars of the investment is given by

S = Pe"

The corresponding effective simple annual interest rate R, obtained by solving the
equation P{\ + R) = Pe"^'" for R, is given by

B EXAMPLE 6 The New Mattoon Savings Bank offers a savings account with con-
tinuously compounded interest at a nominal annual rate of 8 percent.

(a) Find the corresponding effective simple annual interest rate R.

(b) What is the final value S dollars if f = $500 is deposited in such an


account for a term of 3 years?

SOLUTION Here P = $500, r = 0.08, and / = 3.

(a) R = e' - \ = e" ™ - 1 = 0.0833 = 8.33%


(b) 5 = Pe" = 500^""*"" = $635.62

Solving the equation

S = Pe"

for P in terms of S, r, and t, we obtain the formula

Se'

for the present value P dollars of an offer of 5 dollars to be received / years in the
future if investments during this period are earning a nominal annual interest rate r
compounded continuously.

B EXAMPLE 7 Find the present value of $6000 to be paid to you 3 years in the future

if investments during this period are earning a nominal annual interest rate of
9 percent compounded continuously.

SOLUTION Here 5 = $6000, r = 0.09, t = 3, and

P = Se-" = 6000f "" °^'-'' = $4580.28

Exponential Growth and Decay


Suppose that a positive quantity q is changing in time at a rate dq/dt that is propor-
tional to q itself; that is,

dq_
= kq
dt

where k is the constant of proportionality. If A: > 0, then dq/dt> 0, so q is increas-


ing as time goes on; whereas if k < 0, then dq/dt < and q decreases as time
passes. The constant k is called the growth constant.
SECTION 7.9 EXPONENTIAL GROWTH AND DECAY 463

Figure 1 The differential equation dq/dt = kq is separable, so it can be rewritten as

dq
-^ = kdt

Integration of both sides of the last equation yields

In q = kt + C
where C is the constant of integration. It follows that

gin ci ^ ^k,+c ^ ^k,^c


Q^ q = e'^e'"

Suppose that qo is the value of q when f = 0. Then

qo = e^V"' = e^e° = e^

Therefore, the general solution of the differential equation dq/dt = kq is

9 = Qoe

A positive quantity q whose value at time t is given by q = q^e^' is said to be


increasing, or growing, exponentially if k > 0, and it is said to be decreasing, or
decaying, exponentially if A- < 0. Graphs of <? as a function of t for e.xponential
growth and for exponential decay are shown in Figure 1

EXAMPLE 8 Solve the differential equation dq/dt = 2>q with the initial-value con-
dition that q — 2 when r = 0.

SOLUTION Separating variables and integrating, we have

dq
3 dt so that \nq= 'it + C

Therefore,
,ln<;
or q = e e

Substituting q = 2 and / = into the last equation, we find that

2 = e^e-^<0' = e^gO = e^

and it follows that


q = 2e^'

When one-celled organisms reproduce by simple cell division in a culture con-


taining an unlimited supply of nutrients, the rate at which the population of
organisms increases is often proportional to the number of organisms present;
hence, the growth is exponential.

B EXAMPLE 9 Bacteria grown in a certain culture increase at a rate proportional to


thenumber of bacteria present. If there are 2500 bacteria present initially and if the
number of bacteria triples in i hour, how many bacteria are present after t hours'?
How many are present after 2 hours? How long will it take before there are 1 million
bacteria present?

SOLUTION Let q be the number of bacteria present after t hours, so that

dt '
464 CHAFIKR 7 1 RANSCENDKNTAL FUNCTIONS

where k is the constant of proportionality. Separating variables and solving this

differential equation, we find that

q = qae'"

where ^o = 2500 is the number of bacteria present when t = 0. When / = |, we


know that q = 3qo = 7500; hence,

7500 = 2500e*"'^-* or 3 = e*''-

From the last equation, it follows that

—k = In 3 or A- = 2 In 3
2

Therefore.

q = qoe'" = 2500e"-'"-'*' = 2500(e'" ')-' = 2500(3)-' = 2500(9')

Thus, after t hours, there will be 2500(9') bacteria in the culture. After 2 hours, the
culture will contain

2500(9-) = 202.500 bacteria

The time t required for the number of bacteria to increase to 1 million can be found
by solving the equation

1 .000.000
1.000.000 = 2500(9') or 9'
,
= = 400
2500

Taking the natural logarithm on both sides of the last equation, we find that

In 9' = in 400 or / In 9 = In 400

Hence,

In 400
t
= = 2.73 hours
In 9

Radioactive substances decay at a rate proportional to the amount of the sub-


stance present; hence, the quantity q of such a substance satisfies a differential
equation

dt ^

with a negative constant k. Separating variables and solving this differential equa-
tion, we find that

q = qoe'"

where ^o is the amount of radioactive material present when / = 0. Although the


decay constant k gives a measure of how rapidly the substance is decaying, the rate
of decay of a radioactive material is usually expressed by its half-life — the length of
time T during which exactly half of it will decay.

El EXAMPLE 10 Polonium, a radioactive element discovered by Marie Curie in 1898


and named after her native country. Poland, has a half-life of 140 days. Find the
decay constant k. and sketch a graph showing the amount q grams of polonium
remaining after / days if the amount when t = i$ q^^ = 5 grams.
SECTION 7.9 EXPONENTIAL GROWTH AND DECAY 465

SOLUTION The differential equation for q is

'
dt

where it is the decay constant. Separating variables and solving this differential
equation, we have

where ^o = 5 grams. The half-life isT = 140 days, so when t = T.q = 2^0; that is,

= or
„kT
e
— i
2^0 loe

Therefore,

kT=\x\ — =-\n2
1

so that k=
In 2
=- -ttt
In 2
== -0.005
140

The graph of
q = 5e

is sketched in Figure 2

Figure 2

Problem Set 7.9

In Problems 1 to 8. evaluate each limit follow a Poisson probability distribution, averaging c patients
per hour. According to the theory of probability.
6\"
1 lim 1 + 2 lim 1
,
,,^ + x \ „ /
P(k) =
n(n - l)(rt - 2)
3 lim ( 1
- 5.V) 4 lim (1 )
k(k- \)[k - 2)
... 1
V H / V n'

\V5
lim (1 + — 6 lim 1 + •
gives the probability that exactly k patients will arrive at the
emergency room during a given 1-hour period. Evaluate this

limit and thus find P{k).

lim(l+^)'
1,^0
V 7 /
]In Problems 1 1 to 16. assume that you have invested a principal P
dollars at a nominal annual interest rate r compounded n times per
E] 9 Using a calculator, evaluate [1 + (6//!))'' for successively larger
year for a term of t years. Calculate (a) the effective simple annual
and larger values of h. and thus confirm your answer to Prob-
interest rate R and (b) the final value S dollars of your investment.
lem 1.

10 Patients arriving at the emergency room of a certain hospital \\ P = $1000, r = 1 percent, n = 1, ( = 13 years
466 C HAHTER 7 TRANSCENDENTAL FUNCTIONS

12 P = $1000. r = 7 percent, n = 12. / = 13 years would result if that monthly rate continued for 12 months. The
understanding is that, P dollars
on the average, if an article cost
13 P = SIOIX). r = 7 percent. « = 52. / = 13 years
at the beginning of the month, then P + Pr dollars it would cost
14 P = $1000. r = 12 percent, n = 12. / = 13 years at the end of the month; similarly, if an article cost P dollars at

the beginning of the year, would cost P + PR dollars at the it

15 P = $50,000. r = 13.5 percent. n= \2. I = i year


end of the year, (a) Show that /? = (!+ r)'" - 1. (b) Find the
16 P = $25,000. r = 15.5 percent, n = 52, r = 5 year
annual rate of inflation corresponding to a monthly rate of 1

percent.
El 17 Suppose that a bank offers to pay a nominal annual interest rate

of 8 percent on money left on deposit for a 2-year term. Assume


In Problems 27 to 32, solve each differential equation with the given
that a principal of $1000 is deposited. Find the final value S
initial-value condition.
dollars after the 2-year term if the interest is compounded
(a) annually, (b) semiannually, (c) quarterly, (d) monthly,
da
(e) weekly, (f) daily, (g) hourly, and (e) continuously. 27 -r- = 5q. q = 2 when / =
E18 Find out the nominal annual interest rate r offered by your local
savings bank for regular savings accounts and the number of 28 -^ = 2v. V = 10 when v =
times n per year that the interest is compounded. Using this ax
information, calculate the effective simple annual interest rate R.
29 = -4^. N = 40 when / =
El 19 Suppose someone owes you $100 and offers to pay you this
that dt
money 6 months from now. What is the present value to you of
this offer if you currently have an investment opportunity paying 30 ^=
dx
-2v, V = - 10 when .v =
8 percent nominal annual interest compounded (a) quarterly,
(b) monthly, (c) weekly, (d) continuously?
31 -~- = W - q. q = 3 when I =
E120 If a sum of money is invested at a nominal annual interest rate of dt

7 percent, how long will it take for the investment to double in


value if the interest is compounded (a) quarterly, (b) monthly. 32 —-= -0.2(80 - .V). .V = when t =
dt
(c) continuously?

E121 A savings bank with 328 million in regular savings accounts is E33 Bacteria grown in a certain culture increase at a rate proportional
paying a nominal annual interest of 5.5 percent compounded to the number of bacteria present. If there are 1000 bacteria
quarterly on such accounts. The bank is contemplating offering initially and if the number of bacteria doubles in 15 minutes,

the same rate of interest, but compounding continuously rather how long will it take before there are 2 million bacteria present?
than quarterly. If this plan is implemented, (a) what effective
E34 Bacteria in a culture have a natural tendency to increase by 25
simple annual interest rate R can the bank advertise on its sav-
percent each hour; however, an experimental bactericide present
ings accounts and (b) how much more interest will the bank have
in the culture kills 20 percent of the bacteria each hour. How
to pay out per year on these accounts'.'
much time is required for the number of bacteria to double under
El 22 On Jay's 16th birthday, his father promises to give him $50,000 these conditions?
when he turns 21 to help .set him up in business. Local banks are
[335 Ecologists have determined that the rate at which the population
offering savings accounts at a nominal annual interest rate of
of bears in a protected forest area increases is proportional to the
7 compounded continuously. Jay. who has
percent studied
number of bears. If there were 225 bears in 1977 and 239 bears
the mathematics of finance, says, "Dad, I'll settle for
in 1980. how many bears will inhabit the region in 1990?
dollars right now!" Fill in the blank appropriately.
36 If a quantity q is growing exponentially in time with a growth
El 23 Money deposited in a certain savings account doubles every 9.9
constant k. show that the percentage of increase of ^ in 1 unit of
years with the interest compounded continuously, (a) What is
time is given by («* - 1) x 100 percent.
the nominal annual interest rate? (b) What is the effective simple
annual interest rate? [S37 The number of bacteria in an unrefrigerated chicken salad triples

in 2 hours. In how many more hours will the original number of


El 24 The inflation rate in a certain country has been 1 1 percent per
bacteria be multiplied by a factor of 50?
year for the past 5 years. If in that country a loaf of bread now
costs $1, how much did it cost 5 years ago? 38 Suppose that a quantity q is growing or decaying exponentially
in time and that k is the growth or decay constant. Assume that
El 25 How much money should you invest at 8 percent compounded
= =
q q\ when / ;, and q = q2 when / = ;;. If r, 7^ n. prove that
continuously in order to have $40,000 after 15 years?

El 26 Government economists, monitoring the rate of inflation, often


=
; In^
report a monthly rate r and the corresponding annual rate R that ^1
SECTION 7.9 EXPONENTIAL GROWTH AND DECAY 467

EI39 The bacterium Escherichia coli is found in the human intestine.


When E. coli is cultivated under ideal conditions in a biological
laboratory, the population grows exponentially and doubles in

20 minutes. Suppose that t/o = 10 E. coli cells are placed in a


nutrient broth at time / = minutes, (a) Write an equation for
the number q of E. coli bacteria in the colony I minutes later,

(b) Find q when / = 60 minutes.

40 Prove that the solution q = go^*' of the differential equation

dq/dt = kq is actually the complete solution. To do this, assume


that q = f(t) is a solution and that qo =/(0) and then calculate
D,[f{t)e''%

[341 At a certain instant, 100 grams of a radioactive substance is

present. After 4 years, 20 grams remains, (a) How much of the


substance remains after 8 years? (b) What is the half-life of the
substance?

42 If ^ X 100 percent of a radioactive substance remains after t


years, show that the half-life of the substance is T years, where
7= (r In 2)/(-ln/0.

E43 If the half-life of polonium is 140 days, how long does it take for
2 grams of polonium to decay to 0. 1 gram?

044 In the chemical processing of a certain mineral, the rate of


change of the amount of mineral remaining is proportional to
this amount. If 100 kilograms of mineral has been reduced to 70
kilograms after 8 hours, what quantity of the mineral remains
after 24 hours?

E145 In 1921, President Warren G. Harding presented Marie Curie a


gift of 1 gram of radium on behalf of the women of the United
States. Using the fact that the half-life of radium is 1656 years,
determine how much of the original -gram 1 gift was left in 1 98 1

{
468 CHAPTER 7 TRANSCENDENTAL FUNCTIONS

7.10 Mathematical Models for Biological Growth


In this section, we consider some of the mathematical models currently used in the

life sciences to describe biological growth. The construction of realistic mathemati-


calmodels nearly always requires the patient accumulation of experimental data.
Figure I sometimes over a period of many years. In Figure 1, we plotted points (t. N)
,v million showing the population N of the United States in the year 1790 + /. according to the
U.S. Census Bureau. These points seem to lie along a curve that is reminiscent of
the graph of exponential growth (Figure la in Section 7.9). and they therefore
suggest the mathematical model

N= Noe*"

for the growth of the population of the United States.

In 1798. the English economist Thomas Malthus made similar observations about
the world population in his Essay on the Principle of Populaiion. Because Malthus
also proposed a linear model for the expansion of food resources, he forecast that
the exponentially growing population would eventually be unable to feed itself.

This dire prediction had such an impact on economic thought that the exponential
model for population growth came to be known as the Malthusian model.
Consider a population growing according to the Malthusian model
SECTION 7.10 MATHEMATICAL MODELS FOR BIOLOGICAL GROWTH 469

SOLUTION

(a) Expressed as a decimal. 1.1 percent is /(" = 0.01 1. Therefore.

k= In (1 + AT) = In 1.011 =0.011


and we have
Af = Noe*' = 226^°°'"
(b) In the year 2000, t = 20 and

N= 226e'"""-"' = 226^0- = 282 million

(c) In the year 2020, i = 40 and

N= 226^°"""*°' = 226«'"*^ « 351 million

In the example above, the growth constant A- = in (1 + K). rounded off to three
decimal places, is the same as the yearly percentage of population increase ex-
pressed as a decimal A". This is no accident. Indeed, suppose that K is small, and let

y =/(.v) = In .V. By the linear-approximation theorem (Theorem 1, page 249) with


A'l = 1 and A,Y = K, we have

/(I +K)==f{\)+f'{l)K
Here,/(n = In I = 0, and since /'(.v) = l/.v./'d) = 1. Therefore, if A: is small.

A- = In ( 1 -I- K)

In fact, if A^ < 0.06 (6 percent per year), then the error in the approximation k ~K
is less than 3 percent.

For any growth model, it is interesting to ask when the growing quantity doubles.
If T is the doubling time for the Malthusian model A' = N^e'". then

Afge^'+n = 2Noe'" or Z'^*^ = 2e*'

that is.

^k,^kT ^ 2e*' or e'^ = 2

Thus,

kT = In 2

and we have the formula

In 2 In
7" =

E EXAMPLE 2 If the population of the United States grows according to the Malthu-
sian model at 1 . 1 percent per year, in approximately how many years will it double?

Here K= 0.011. so

In 2 _ In 2
~ 63 years
a: 0.011
470 CHAPTER 7 TRANSCENDENTAL FUNCTIONS

Figure 2

c
SECTION 7.10 MATHEMATICAL MODELS FOR BIOLOGICAL GROWTH 471

Therefore.
dN dN
dN = k dt or + = kdt
'
C -N N C-N N
Integrating both sides of the last equation, we obtain

dS f dN
"^ k dt or \n(C - N) + In N= kt + C,
C -N J N
where Ci is the constant of integration. Using the properties of logarithms, we can
rewrite the last equation as

N N
In = kt + C,
C-N C -N
Now we take reciprocals of both sides of the last equation to obtain

C -N
Coe-"
N
where we have put Cq = e "-'.
Thus,

-- 1 = Coe-" = 1 + Coe'
N N
Solving the last equation for A', we obtain the equation for logistic growth:

1 + Q^e-"

If A^o is the value of A' when t = 0. we have

C
Nn° = or No =
1 + Cne°
"
"" 1 + Co
Solving the last equation for Cq. we find that

=
C-Nq
Cn
Nn

In what follows, we consider the case in which < No<C. (The case in which
C ^ A^o is considered in Problem 8. ) Thus, since both C — Nq and A'o are positive, it
follows that Co is positive. Therefore, the denominator 1 + Co^"*' in the equation
for logistic growth is greater than 1 . and we have < N < C for all values of t.

Also, since
C = C
lim A' = lim r- = C
1 + Coe" 1 +
the horizontal line A' = C is an asymptote of the graph of the equation. We leave it

as an exercise for you to show that the graph has a point of inflection P/ = it;. N,)
given by

C
= y In Co and N,=

(Problem 14).
472 CHAP-IKR 7 IRANSCENDENTAL FUNCTIONS

B EXAMPLK 3 Suppose that the population A' million of the United States grows
according to the logistic model
C
1 + Co?-*'

where / is the time in years since 1780 and where k = 0.03. Use the following data:
The population in 1780 was 3 million, and the population in 1880 was 50 million.

(a) Find the carrying capacity C.

(b) Find N in the year 1980.

(c) Find N in the year 2000.

(d) Determine when the inflection occurred.

In each case, round off the population to the nearest 10 million.

SOLUTION

(a) From the data given, A'q = 3 million in 1780 when t = 0. Thus,

C- A^o _ C - 3

yVo 3

and we have
C 3C
N ""
1 + [(C - 3)/3]f-" 3 + (C - 3)e'°°^'

Using the fact that A' = 50 million in 1880 when = t 100, we substitute into the

last equation to obtain

^Q= - 50(3.,C-3).-l = 3C
3^(C-'3).-o"--"»o>

Thus,

150 + 50Ce-^ - 150^--' = 3C or (3 - 50e-^)C = 150(1 - e'^)

and it follows that the carrying capacity is

150(1 - e -')

C = == 280 million
3 - 50e r^ '

(b) Substituting C= 280 in the equation obtained in part (a), we have

_ 3C 840
^" 3 + (C - 3k-° °-" " 3 + 277f-oo"

Hence, in 1980. when / = 200 years,

840
N= 3 -)-
,,
Ty-7„ -0.03(200)
„,,,„„, = 230 million

(The correct value according to the 1980 census was 226 million, so our
logistic model has predicted the correct value with an error of less than 2
percent.)

(c) In the year 2000, / = 220 years and

840
N= ,.,,,,.,„, = 250 million
3 + 277e"'""'"°'
SECTION 7.10 MATHEMATICAL MODELS FOR BIOLOGICAL GROWTH 473

[Compare this with the prediction of 282 million according to the Malthusian
model (page 469). Notice that the "leveling off" built into the logistic model
has apparently taken hold.]

C - No 280 - 277
(d) I,
—1

In
1

In
3 1

= 150 years
k No 0.03 0.03

Thus, according to this logistic model, the inflection would have taken place in

the year 1780 + 150 = 1930.

Problem Set 7.10

E 1 The population of a small countn,' was 10 million in 1980. and it S^ 9 On the same (A'. /) coordinate system, sketch accurate graphs of
is growing according to the Malthusian model at 3 percent per (al the population A' of the deer herd in Problem 7 and (b) the
year, (a) Write an equation for the population .V million of the population A' that would result if the herd grew exponentially
country- r years after 1980. (b) Predict A' in the year 2000. with growth constant /t = 0.1.
(c) Find the doubling time T for the population.
10 In the equation for logistic growth, solve for ; in terms of A' to

2 Suppose that a population that is growing according to the Mal- obtain t = (I A) In [NCo/(C - N)]-
thusian model increases by lOOA" percent per year. Write an
El 11 In 1800, the population of Sweden was 2.35 million. Suppose
exact formula (not an appro.ximation) for the doubling time T in
that the population A' has been growing since then according to a
terms of K.
logistic model with k = 0.023. In 1900 the population of Swe-
El 3 The population of a certain city is growing according to the den was 5.14 million, (a) Find the population carrying capacity
Malthusian model, and it is expected to double in 35 years. C of Sweden according to the logistic model, (b) Use the logistic

Approximately what percent of growth will occur in this popula- model to estimate the population of Sweden in 1970. (Note: In

tion over 1 year? 1970. the population of Sweden was actually 8.08 million.)

4 Explain why, for small values of /. logistic growth is approxi- 12 If a certain population N is thought to be growing according to
mately the same as Malthusian growth. the logistic model, the value of the constant k is often found
experimentally as follows: First, an estimate is made for the
El 5 The fruit fly Drosophila melanogasrer is often used by biologists
carrying capacity C of the habitat. Then values of A' are meas-
for genetic experiments because it breeds rapidly and has a short
ured corresponding to several different values of t. and
life cycle. Suppose a colony of D. melanogaster in a laboratory

is observed to double in T= 2 days. Assuming a Malthusian A'


V = In
model for growth of the colony, determine the approximate daily C -N
percentage increase in the size of the colony.
is calculated for each obser\ed value of iV. The points (r. y) are
6 Suppose that a population is growing according to the Malthu- plotted on a graph (Figure 3). and a straight line L that ""best
sian model with a doubling time T. If A^o is the original size of fits"' these points is drawn. The constant k is taken to be the
the population when f = 0, show that the population r units of slope of the line L. Justify this procedure. (Hint: Use the result
time later is given by N= Nq2''^ .
of Problem 10 to show that kt = y — b, where b is the value of y
when r = 0.)
El 7 Suppose that a herd of 300 deer, just introduced into a game
preserve, grows according to the logistic model with A: = 0.1. Figure 3
Assume that the herd has grown to 387 deer after 5 years,

(a) Find the carrying capacity C of the habitat for deer, (b) Find
the population N of the herd after 7 years, (c) When does the
inflection occur in the population of the herd?

8 Consider the solution of the logistic differential equation for the

case in which C< No- (a) Show that Cn < 0. (b) If C= /Vq.

show N is constant, (c) If C < A^o- show that A' decreases as


that

/increases, (d) If C < A^o. show that the graph of A' is concave
upward for > and has the line N = C as a horizontal asymp-
f

tote.
474 CH AFrER 7 TRANSCENDENTAL FUNCTIONS

13 Consider the inhibited-growth model given by the differential El 19 Suppose that a particular person learning to type is able to type N
equation dN/di = g(N)N. where the variable growth factor has words per minute after / hours of practice. Assume that at the
the fomi ,i;(.\') = ((C/A') - \\k. Because of its extensive use by beginning, when / = 0. this person can type 10 words per min-
psychologists to study the learning process, this is often called ute by the "'two-rmger"" method. If A increases according to the
the learning model, Show that
(a) the solution to the differential learning model in Problem 13 with C = 100 and k = 0.025.
equation above is/V = C(l - Cf where Co = 1 - {NJC).
*'), (a) how many words per minute can this person type after i = 30

(b) Show that A' = C is a horizontal asymptote of the graph of A/ hours of practice and (b) what is the rate of improvement dN/di
as a function of /. in words per minute per hour of practice when t = 30 hours'?

14 Verify the formulas on page 471 for the coordinates (f;. A/) of Ill 20 An advertising campaign is aimed at a projected 100,000 poten-
the point of inflection for the logistic model. tial purchasers of a new high-performance luxury sports car. The
rate at which more of these potential purchasers become aware
115 The learning model in Problem 13 often provides a realistic
of the new sports car because of the advertising is proportional to
model for population growth. Suppose the deer herd in Prob-
the number of people, out of the original 100.000. who have not
lem 7 grows according to this growth model with C = 700,
yet become aware of the car. If no one has heard about the car at
k = 0.05. and A'o = 300. Sketch the graph of A' as a function of
the start of the campaign and after 10 days 30 percent of the
I according to this model, and compare it with the graphs
potential purchasers have heard about the car. how many of the
sketched in Problem 9.
potential purchasers have heard about the car after 30 days?
16 Outline a procedure for determining the constant k for the learn-
21 The Gompertz model is the inhibited-growth model given by
ing model in Problem 13. The procedure should be similar to = g{N)N. where the variable
the differential equation dN/di
that in Problem 12, but with y = In [C/{C -
growth factor has the form ^(A^) = (In C - In N)k. (a) If Co is
A^)].

any constant, show by differentiation that A' = C exp (-Coe~*0


17 Suppose that < A'o < C for the learning model in Problem 13,
is a solution of the differential equation, (b) For the solution in
and show that the graph of ;V as a function of i has no point of
inflection.
part (a), show that Co = In (C/Ao). where N= No when f = 0.

(c) For the solution in part (a), show that A=C is a horizontal
18 A seasonal-growth model is described by a differential equa- asymptote of the graph.
tion of the fonn dh'/di = [g(AO cos {a)t - (f>)]N in which the
22 Does the graph of the Gompertz growth equation in part (a) of
growth factor .^CA^) is multiplied by a periodic function of t to
Problem 2 1 have a point of inflection? If so. w hat are its coordi-
account for seasonal variations in the rate of growth (for in-
nates?
stance, variations caused by seasonal changes in the availability
of food). l{ gi.N) = k. for all values of A. where k is a positive El 23 Suppose that the deer herd in Problem 7 grows according to the
constant, solve this differential equation with the initial condi- Gompertz model in Problem 21 with Ao = 300. C= 700. and
tion that N= No when / = 0. k = 0.07. Sketch the resulting graph of N as a function of /.

7.11 First-Order Linear Differential Equations


If P and Q are continuous functions, then a first-order differential equation of the
form

+ P(.v)y = Qix)
dx

is said to be linear. The growth models considered in Section 7. 10 were all obtained
by solving first-order linear differential equations.
Another important example of a linear differential equation is obtained from
Newton's law of cooling, which states that the rate of change of the temperature of
a cooling object is proportional to the difference between the temperature of the
object and the temperature of the surrounding medium. Thus, if y denotes the
SECTION 7.11 FIRST-ORDER LINEAR DIFFERENTIAL EQUATIONS 475

temperature of the cooling object at time / and a is the temperature of the surround-

ing medium, we have


dv
-^= -k{\ -a)
dt

where k is the positive constant of proportionalit)' and the negative sign in front of

k is to account for the fact that v decreases as t increases.

B EX.\MPLE 1 An object with an initial temperature of 1 50°C is allowed to cool in

a room whose temperature is 25°C. If after 10 minutes the temperature of the object
has dropped to 120°C. what is its temperature after 30 minutes'?

SOLUTION Let y be the temperature of the object after r minutes, so that by


Newton's law of cooling.
d\
= -kiy - 25)
dt

i-k) dt

In (y - 25) = -kt + Ci or y - 25 = e-*'+c,

where Ci is the constant of integration. It follows that

y = 25 + e^'-"" = 25 -t- e'^'e'^' or y = 25 + Co^"*'

where we have put Co = e'-^'. Substituting the initial conditions y = 150 when r =
into the equation y = 25 + Cof^*". we find that

150 = 25 + Coc-" = 25 + Co so that Co = 150 - 25 = 125

Therefore,
y = 25 + 125?-'-''

Now. when / = 10. we have y = 120, so that

120 - 25 19
476 CHAPFKR 7 TRANSCENDENTAL FUNCTIONS

Figure 1

</(lbl
SECTION 7. 1 1 FIRST-ORDER LINEAR DIFFERENTL\L EQIATIONS 477

To develop the idea of an integrating factor, we begin by considering a special


case of the equation above in which Q(.x) is (temporarily) replaced by zero, so that

d\
-- + P{x)\ =
dx

Now, this differential equation is separable, and we can rewrite it as

d\-
^- = -P(.v) dx
V

Integrating both sides of the last equation, we obtain

In |y| + C] = -j Pix) dx or In \y\


= -C^ - I P(x) dx

where Ci is the constant of integration. Therefore,

Hence, y = +^-c,^-fPM,Lx ^^ ,^, ^ ^^-/mxiv


where we have put C = ±e~'-'. It follows that

y^SPMd.^C
If we let

<A(.v) = e^''""^^

we can write our solution as

y(l>(x) = C
The expression y(i>(.v) on the left of the last equation deserves further study. First,
notice that

-^4>i-x) = 4^^'''^"'' = e^''""'^ -4- [^(v) ^v = 4>(x)P(x)


dx dx dx J

Therefore,

—d dx
^ ^ ^
[\(t>{x)] =
d\-

-f-
dx
(blx) +
'
d
\—-ci){x)
dx
=
dv
-f-
dx
(t>ix) + y<l)(x)P(x)

c/>(.v.[-;^-

In other words, if you multiply


(A-

-J- + P(x)y
dx
b\ the factor
d){x) = ef '''"'"
the resulting expression is exactly the derivative of

y<t>(x)

It follows that the linear differential equation

dx
-f- + P{x)x = Qix)
dx

can be solved by multiplying both sides by the integrating factor rf)(.v). The tech-
nique is illustrated in the following example.
478 CHAPIKR 7 TRANSCENDKNTAI. Fl'NCIIONS

2
EXAMPLE 3 Solve the linear differential equation —^v v = .rr .

ax .V

SOLUTION The given differential equation has the linear form

dy
-f + P{x)y = Qix)
ax

with
2
P{,x)
= and Q[x) = x-e'
X

Here, the integrating factor is given by

Multiplying both sides of the given differential equation by the integrating factor,
we obtain

\ ld\ 2 \ 1 ,

x~ ^ ax x ' X

1 d\ 2

A ax .r

Now we know that the left side of the last equation must be the derivative of

V
—A"r = ^r
1

v<^(.v) = \

A-

Hence, we can rewrite the equation as

D \

^Vx^i
Therefore,

= e' dx = + C
1^^\ e'

and it follows that

\ = at' + Cx"

Problem Set 7.11

In Problems I to 4, solve each linear differential equation by sepa- 13 5 An iron ball at a temperature of 90°C is placed in a water bath at

rating variables. a constant temperature of 2°C. After 10 minutes, the ball has
cooled to 25°C. How many more minutes is required for the ball

1
d\
-^ +
dx
.n- = 2 —+
ds
dt
3i = 3
to cool to IO°C?

El 6 A body, initially at the temperature 160°F. is allowed to cool in

dN air. The body cools to IOO°F after 50 minutes. After 100 min-
,
3 — 0. IN = - 100 A
4 d\
J - -,
2y
J J
dx = dx . .
•'
, , „„„,_ .,„ ,

di ' utes. It has cooled to 80 F. What is the air temperature?


SECTION 7.11 FIRST-ORDER LINEAR DIFFERENTIAL EQUATIONS 479

O 7 A baked potato at a temperature of 200°F is placed on Melissa's (d) Assuming that y = Vo when / = 0, show that
dinner plate. She does not
later.

to 175°F.

the air temperature


hot is

is
the potato
70°F?
start eating the potato until 7 minutes
After 4 minutes, the temperature of the potato has dropped
How when Melissa starts to eat it if
y =A + (Vo - A)
m
In Problems 15 to 20, use an integrating factor to solve each linear
[c] 8 If a roast at a room temperature of 70°F is placed into a refrigera- differential equation.
tor with a constant temperature of 35°F. and if the temperature
of the roast after 2 hours is 45°F, what is its temperature after
4 hours?
15^-4v = .^ 16 ^\
dx dx X

El 9 A tank
of
initially

salt is
contains 50 gallons of water in which 10 pounds
dissolved. Pure water runs into the tank at the rate of 3
17 ^ + 2n =
dt
Ite-''

gallons per minute and is uniformly stirred into the solution.


18 (A = .XT dx + e' '- cos .v dx
Meanwhile, the mixture runs out of the tank at the constant rate
of 2 gallons per minute. After how long is only 2 pounds of
dissolved salt left in the tank? + q cos t = e 20 D^v

E] 10 Solve Problem 9 if the water running into the tank is not pure but
21 A circuit consisting of an inductor of L henrys with a
contains m pound of salt per gallon and this water runs into the
resistor of R ohms is called an /?L-circuit (Figure 2). If
in series

/ is the
tank at the rate of 2 gallons per minute.
current in amperes flowing in an /^/.-circuit because of a driving

El 11 Air in a chemical laboratory contains 1 percent hydrogen sul- electromotive force of £ volts, then / as a function of time r

fide. An exhaust fan is turned on which removes air from the satisfies the first-order linear differential equation
room at the rate of 500 cubic feet per minute. Meanwhile, fresh
air is drawn room through cracks under doors and so
into the -/

forth. If the volume of the room is 10.000 cubic feet, what is the
concentration of hydrogen sulfide after 5 minutes? (a) If £ is constant and if / = when t = 0, solve this differen-
tial equation for / and find lim /. (b) Assuming that £ is a
12 Water containing A kilograms of pollutant per cubic meter flows
into a reservoir of constant volume V cubic meters at a rate of R variable, solve the differential equation.

cubic meters per second. Polluted reser\oir water is drawn off at


the same rate R, so as to maintain constant reservoir volume V.
Figure 2
Let V denote the concentration of pollutant in kilograms per
cubic meter at time Show that y satisfies the differential
r. (a)

equation dy/di = {R/V){A - y). (b) Find the general solution of


this differential equation, (c) Find the particular solution satisfy-
ing the condition that y = vq when t = 0.

El 13 A flu-like virus is spreading through a city of population 260,000


at a rate proportional to the product of the number of people
akeady infected and the number of people still uninfected. If 600
people were infected initially and 30,000 people were infected
after 10 days, how many people will be infected after 30 days?
22 Show that the general solution to the linear differential equation

dy/dx + P{x)y = Q{x) is given by


14 In Problem 12, suppose that polluted reservoir water is drawn
off at a rate of r cubic meters per second, where r # /?, so that
Q(x)e^ dx
the volume V of water in the reservoir does not remain constant.
Again, y denote the reservoir concentration of pollutant in
let
E23 Solar Industries has a current inventory of 5000 wind-pww ered
kilograms per cubic meter at time /. (a) If Vq is the volume of
generators. 10 percent of which are defective. Generators are
water in the reservoir when t = 0, show that V = Vq + (/? - r)t.
chosen at random from the warehouse and shipped out to cus-
(b) Show that y continues to satisfy the differential equation
tomers at the rate of 200 per week. Newly manufactured genera-

dx R tors are delivered to the warehouse at the rate of 175 per week.
.V) Because of improved quality control, only 5 percent of the
newly manufactured generators are defective. What percent of
(c) Find the general solution of the differential equation in (b). the generators in the warehouse will be defective 1 year (52
480 HIAIMF.R 7 TRANSCENDENTAL KUNfTIONS

weeks) from now? (Hint: Let N be the number of defective gen- variable, (a) Show that the labor force F at time I consists of
erators in the warehouse / weeks from now. Treat A' as if it were F,) + ki individuals, where k = A + B — c. (b) Show that y sat-

a continuous variable, write a linear differential equation for N. isfies the first-order linear differential equation
and solve this equation.)

24 The labor force industry currently consists of lit t


in a certain Fo
individuals of whom vn are skilled workers and Fi, — vq are un-
Assuming that k # 0. show that the particular solution of the
tc)
skilled. Workers arc quitting randomly at a constant rate of c
differential equation in part (b) that satisfies the initial conditions
workers per unit time. Replacements are being hired at constant
F = and y = when = is
F,i yo /

rates of A skilled workers and B unskilled workers per unit time.


Denote by y the number of skilled workers employed by the
industry at time /. and treat y as though it were a continuous
-7^— ('«-T^)(7^r
Review Problem Set, Chapter 7
In Problems I to 6. show that the functions / and g are inverses of Figure 1
each other.

1 f(.x) = x^ for .V > and g(.x) = \<v

2 /(.v) = 3 -I- .r' and g{x) = V'.v - 3

3 /(.v) and g(.v)

4 f(x) = AT - 3.V -I- 2 for .v s f and gU) = for

x^ -t

5 f(x) = for < .V s -- and


I + sin X
1

S(x) = sin '


\— - Ij for.v>5

6 /(.v) = <'" - f '" and g(x) = ln(.v + V.v" -I- 4) - In 2

In Problems 7 to 10. sketch the graph of each function, and then


use the horizontal-line test to determine whether the function is

invertible.

7 fix) = 3.V -t- 5

8 g(x) = -
X

9 h(x) = 3.r -I- 5

10 F{x) = sin X for — IT 3tt_

11 Sketch the graph of the inverses of/, g, and h in Figure 1.

12 Are there values of the constants a, b, c, and d for which the


ax + b
/U) =
.

function is its own inverse?


ex + a
REVIEW PROBLEM SET 481

17 If/U) = x^ + 3.V-' + 7. show that/ is invenible. In Problems 33 to 38. find the exacl value of each expression.

18 Suppose that A. B, and C are constants with A > 0. Let/be the (—


33 cos (tan"' 3) 34 sin [arctan -^)]
function defined by fix) = Ar + Bx + C for a > -B/(24).
Show that the inverse function/"' is given by 35 sin [sin"' (— rt)] 36 tan [arccos (— §)]

-B + \B- - 4AC + 4Av 4AC - B- 37 arcsin [sin (197r/14)] 38 sin (sin"' 3 + sin"' 5)
r'w 2A
for .V ;

4A
In Problems 39 and 40. show that the given equation is an identity.
19 Suppose that/is an invertible function. If/is increasing, show
that/"' is increasing.
39 tan (arccos x) =
20 Beginning students often confuse /"'(jr) and [/(Jf)]"'. Is there

any continuous function /defined on U such that/ is invertible


mdf~\x) = [/(x)]"' for all values of .v in R? 40 sin —- cos '
.r =
V 2

In Problems 21 to 26. find the value of .v when v = a, and use the


41 Solve the trigonometric equation 6 sin" r
-1-
sin r — 2 = for /
inverse-function rule to find the numerical value of dx/dy when
V = a.
if -7r/2<f S 77/2.

3_5 = 4 A
21 o= 1, v = 7.r 22 a = -8, y SX
3 42 ray of light passing through a plate of material with parallel
faces is displaced, but not deviated (Figure 2); that is, the
5x
23 a -l.y = emerging ray is parallel to the ingoing ray. If rf is the amount of
x + 1 displacement, r the thickness of the plate, o the angle of inci-
dence, and ^ the angle of refraction, show that
24 a = —V3
— -, V = cos X for < .r <w
2
p = arctan I tan a sec a\
25 a = 1 . .\' = X + In x 26 a = 4.\ = .v' + It-^ + 1

Figure 2
In Problems 27 to 29. use the given information and the inverse-
function theorem to find (/"')'(a). (You may assume that the hy-
potheses of the theorem are satisfied.)

27 a = 3./(-5) = 3,/'(-5) = 7

28 a = 0.1. /(O.l) = l./(l) = 0.1, /'(I) = 3,/'(0.1) = 2

29 a = 77, /(TT) = e, fie) = Vl, fiVl) = n, f\e) = e,

f{Vi) = IT

30 Let fix) = -Ix- -t- 8j: - 5 for j: > 2. (a) Use the algebraic
method to find/"', (b) Calculate (/"')'(!) by using part (a),
(c) Calculate (/"')'(!) by using the inverse-function theorem.

] 31 Use a calculator (or Appendix D. Table 1) to evaluate each In Problems 43 to 56. differentiate each function.
expression. Give answers in radians.

0.3750 =
(a) sin'' (b) arccos (-0.3901) Ai fix) 2 sin"'
Y 44 g{x) --

(c) cos"' 0.9273 (d) tan"' 57.29


45 h(x) = sec"' V.t - 1
(e) arctan 1.425 (f) cos"' g

(g) arcsin (-f) (h) tan^' 8 47 f(f) = cos"' VJt


(i) cos"' (-if) (j) cot"' 2.731 49 HUi) = ir tan"' u^

(k) sec"' Vs (1) esc"' (-4)


51 gl.u) = (esc"' u)-
32 Evaluate each expression without using a calculator or tables.

(a) sin"' (-5) (b) arccos (-V3/2)


53 q(x) = X' cot"' 5.r
(c) arctan VI (d) arcsec \fl
482 CHAPTER 7 TRANSCENDENTAL FUNCTIONS

mate the possible error in the calculated value of angle BAC if


55 g(u) = (\7 + .r?-' dx
the measurements of .r and y are subject to at most I percent
'i
error.
pan'. /, -r-y*
56 /i{

In Problems 79 to 126, differentiate each function.

79 f{x) = In (r^ + 7) 80 /(/) = In (cos 3f)


In Problems 57 to 60, find dy/dx by implicit differentiation.
81 g(r) = In (rVm.) 82 G(u) = ln(M - 1)'

57 X cos"' (x + y) - n= y"
83 g{x) =.r tan"' (In x) 84 h(x) = sin (In x^)
58 v tan .r - .rv'* = 15

- = xy 85F(«) = ^^^^ 86 Gix) nnx


59 3jc sin (jr v) + 107 u

60 y tan"' x - x tan"' y = 7r/2


87/(.) = ln^i^ - 88 Hix) = In (.t -I- V?+l)
X
In Problems 61 to 74, evaluate each integral.
89 gix) = e-*"' 90 Hix) = -.re"'
^'
6. ^ 91 Fix) = sin"' (e"^) 92 giu) = e" cot e"

+
dx 93 fit) = In ^-—2
e'

e -
2
94 Hix) = ln(f*'"' + 5)
63
J .r + 36
95 g(x) = cos"' .r - xe^ ~ 96 ^(.v) = f' In (sin .v)
X dx
65 97 fix) = tan e' 98 fix) = eXx^ - 2x 5)
V9 - 2x^
-I-

-«„ ~ f" ^'


99 fix) = (3 - f"')-* - 100 gix) = _,
e + e
49
e' +l
sin X dx 101 fix) = In _ 102 hiz) = ?'- cos"' e^
69 f^ e +2
103 Fix) = x*' 104 gix) =.t"'''"
f cot"' V dv
71 ;— 105 fix) = 5'°" 106 = 3''*=
git)
J \ + V-

107 fix) = r""' 108 gix) = ix^ + 7)2"''


73
Jo 25 + 109 g(x) = 3^' •
2'*^
110 Hix) = e^°''-2*'

112 ^i;(m) = log3


75 If y = sin '
(a sin + sin '
(a sin (2 - t)]. where a is a « + 7
constant, find the critical value(s) ±
=
I +x
76 A thin rod (whose weight can be ignored) is 40 centimeters
113 g(x) VIogio.v U4fix)= V log
-
1 X
long and passes through the center of a small, heavy ball that is
fixed on the rod at a distance of 10 centimeters from one end. 115 glx) = cosh f"*' 116 His) = sinh (sin '
s)

The rod is placed in a smooth, hollow hemisphere of radius 40 '"""^'"">


centimeters, and the slightly rounded ends of the rod slide fric- 117 gU) = csch (e'') 118m)= X
tionlessly on the walls of the hemisphere. Find the angle that
("='*"'
the rod makes with the horizontal when it comes to rest
finally 119 fix) = 120 ^(i<) = sinh ir tanh 3u
at equilibrium. (At equilibrium the center of mass of the rod
121 gii) = In (tanh / + sech r) 122 fix) = tan"' (sinh x')
and ball is as low as possible.)
123 gix) = sinh '
(3.r -I- I) 124 giu) = coth"' e'"
77 An isosceles triangle whose two equal sides are each 6 centi-
meters long has an included angle which is increasing at the 125 fix) = tanh '
e' 126 Fix) = coth"' e"'"
constant rale of 1° per minute. How fast is the area of the
triangle changing? 127 If (In .v)/.v = (In 2)/2. does it necessarily follow that x = 2?
Justify your answer. iHini: Sketch a graph of y = (In x)/x.)
l£l 78 Angle BAC of a right triangle computed from measurements
is

of the opposite side y = \BC\ and the adjacent side .t = |^|. 128 If (In x)/x = (In h/h does it necessarily follow that Jt = i?
(Angle ACB is the right angle.) Use differentials to approxi- Justify your answer.
REVIEW PROBLEM SET 483

In Problems 129 to 138, use logarithmic differentiation to find


dyjdx.

129 y = (3.v)>
484 CHAPTER 7 TRANSCENDENTAL Fl'NCTlONS

where cr and fi are certain constants called the standard devia- El 186 At 2;00 p.m. a dish of cream that has been standing in a warm
tion and the mean, respectively, and a> 0. When students room contains 150.000 bacteria of a certain kind. At 4:00 p.m.

ask to be graded "on the cun'e," the curve in question is the there are 900,000 of these bacteria present. How many were

graph of/. For what value of .v is/(.v) a maximum and what is present at 3:00 p.m.?

the maximum value of/(.v)? El Sketch the graph of/ if o- = 10


El 187 Black coffee is poured into a cup at an initial temperature of
and /x = 75.
200°F. One minute later, the coffee has cooled to I60°F. If the

[H 180 In public health, it is sometimes necessary to screen large num- room temperature is 80°F, find how many additional minutes

bers of people for certain diseases. When this can be done by will be required for the coffee to coo! to 110°F.

means of a blood test, the technique of batch testing is often decomposes


188 Radioactive substance A with half-life (In 2)/a
more efficient than performing individual blood tests. UN peo- radioactive substance B with half-life (In l)/b, where
into
ple are to be tested, they are divided into groups of k people,
a 7^ h: that is, when an atom of substance A decomposes, it is
and the blood samples from each person in such a group are If the number of atoms
transmuted into an atom of substance B.
mixed into a single batch. Each of the N/k resulting batches is
of substance A at time i is given by q = que'"', and if y denotes
tested for the disease. If such a test result is negative, it is
the number of atoms of substance B at this same show
instant,
concluded that none of the people in the corresponding group
that y satisfies the differential equation (t/y /A) + by = aqoe'"'-
has the disease; if the test result is positive, the people in the

group are retested individually. By means of a pilot study, it is 189 In paleontology, radioactive dating is sometimes used to de-

determined in advance that a fraction \ -p of the population termine the age of plant and animal fossils. The age I is calcu-
"

has the disease. The expected number of tests required is then lated in terms of the ratio y/q of radioactive 'daughter" sub-
given by 7" = {N/k) + N(\ - pM. Find the value of A: that mini- stance B to mother" substance A in the fossil. In Problem 188
mizes T for (a) p = 0.95. (b) p = 0.8. and (c) p = 0.7. assume that y = when / = 0. (a) Derive the equation

El 181 You have just won first prize in the state lottery, and you have
your choice of the following: (a) $30,000 will be placed in a
a - b
savings account in your name, compounded continuously at the

rateof 10 percent per year; (b) $0.01 will be placed in a fund in for the number of atoms of substance B present at time /.

your name and the amount will be doubled every 6 months over (b) Show that

the next 12 years. Which plan should you choose? a-


t
=
1

-In
/
1
+
b ^\\
182 Carry out the following steps to show that the function /'defined a - b ^ a q
'

by/(.v) = [1 + (fl/.v)]' is increasing if a is a positive constant


El 190 When a capacitor with capacitance C farads discharges through
and .r > 0. (a) Use the mean-value theorem to show that there
a resistor with resistance R ohms, the current / in amperes
is a number i with
flowing through the resistor at time t satisfies the differential

+ [l/iRO] = = when =
In In = — and f < 1 -I-
equation dl/dt 0. (a) If / /n I 0. find a

(-f) formula giving / in terms of t. (b) If RC = jm and /o = 40


milliamperes, plot a graph of / as a function of I.

(b) From part (a), show that In [1 + (a/x)] 2 a/{a + x).

(c) Use part (b) to prove that D,[l + (a/x)Y > 0. El 191 A new species of plant is introduced on a small island. Assume
that 500 plants were introduced initially and that the number N
183 Interpret the result that/ (jf) = [I + (aA)]' is increasing (Prob- =
of plants increases according to the logistic model with k
lem 182) in terms of compound interest.
0.62. Suppose that there are 1700 plants on the island 2 years

184 Suppose a bank offers savings accounts at a nominal annual after they were introduced, (a) Find the maximum possible

rate r compounded n times per year. By regularly depositing a number of plants the island will suppon. (b) Find N after 10

fixed sum of p dollars in such an account every (lAoth of a years, (c) When does the inflection occur? (d) Sketch a graph

year, money can be accumulated for future needs. Such a plan of /V as a function of time t in years.

is called a sinking fund. If it is required that such a sinking


El 192 An industrial plant dumps 10 pounds of pollutant into a lake
fund yield an amount S dollars after a term of / years, show that
every minute. Fresh water from streams runs into the lake at the
the periodic payment p dollars must be
rate of 100 cubic feet per minute and mixes uniformly with the

Sr polluted water, and meanwhile the polluted water runs out of


the lake into a river at the same rate. The plant commenced
operations 10 years ago. at which time the lake was unpolluted.
The lake now tests at ^ pound of pollutant per cubic foot. How
El 185 A radioactive substance has a half-life of 2 hours. How long many cubic feet of water are there in the lake? (Take 10 years to
does it take it to decay to lO of its original mass? be 5.2596 x 10" minutes.)
TECHNIQUES OF INTEGRATION

Our purpose in this ctiapter is to consolidate the methods of integration presented


previously, introduce some new methods, and give a unified development of the
most important basic techniques of integration. There are really only three general
procedures for evaluating indefinite integrals:

1 Substitution, or change of variable


2 Manipulation of the integrand using algebraic or other identities so as to
convert it into a more manageable form

3 Integration by parts

Although we have made extensive use of the first two methods in the preceding
chapters, we have yet to introduce a few special tricks of the trade such as trigono-
metric substitution and partial fractions. The third method, integration by parts, is

also introduced in this chapter.


The three general procedures are often used in combination; however, none of
these procedures — singly or in combination — is guaranteed to be effective under all

circumstances. Frequently, the choice of the appropriate technique is a matter of


trial and error guided by experience.

8.1 Integrals of Powers of Sines and Cosines


In this section we use the integration formulas on page 257, in conjunction with
appropriate trigonometric identities and changes of variable, to evaluate integrals
involving products of powers of sines and cosines.

Integrals of the Form J / sin'" jc cos" x dx

In order to evaluate / sin'" x cos" .v dx, where m and /; are constant exponents, we
consider separately the case in which at least one of the exponents is a positive odd
integer and the case in which both of the exponents are nonnegative even integers.

485
486 CHAPTER 8 TECHNlQlrtS OF INTEGRATION

Case 1: At Least One of the Exponents m, n Is a Positive Odd Integer

In this case we use the identity sin" v + cos' x = 1 to rewrite the integrand either

in the form F(cos .v) sin .v or in the form F{sin v) cos x. In the former case, the
substitution u = cos x is effective, while in the latter case the substitution u = sin x
works.

hi Examples I to 3, evaliuite the given integral.

EXAMPLE 1 sin-' .V dx
J

SOLUTION sin-* X dx = sin- .v sin .v d^- = I ( 1


~ cos- .v) sin x dx

Making the substitution u = cos x, so that du = —sin x dx. we have

1(1— cos- x) sin X dx = I (1 ~ u~){—du) = — \ (\ — u~) du

u^\
= - (H
1 ,
I ] + C = —cos X ^ cos X +C
\ 3 / 3

EXAMPLE 2 sin- X cos' X dx


J

SOLUTION sin- X cos' X dx = \ sin" .v cos'* x cos x dx

= sin- .V (cos- x)' cos x dx

= sin- -V (1 - sin- x)~ cos x dx

Making the substitution u = sin x, so that du = cos x dx, we have

I sin^ X (1 — sin- x)' cos x dx = u-(\ — «-)- du

= I M-(l - 2m- + «*) du

= (m- - 2m"* + m") du = ^M-' - Im' + h'' + C


J
= ^ sin"' jf — f sin' x + 7 sin^ x + C '

r sin' .V
'

EXAMP LE 3 I <it
J Vcos
'cos X

~
SOLUTION
f
— sin'
.
X
dx =
f
—sin- x
-=^=- sin X dx = f 1

,
cos^ x
sin x dx
J Vcos X J Vcos X J Vcos .X

Put M = cos .Y, so that f/M = -sin x dx and


- cos^
r 1 ;t
sin j: ^= - p— du = (u^^- - u '^-) du
'cos X -'
Vm J

= Im'/- - 2m'''2 + C = I (cos Jt)'/- - 2Vcos A- + C


SECTION 8.1 INTEGRALS OF POWERS OF SINES AND COSINES 487

In the last example, notice that the technique works even when one of the powers of
the sine or cosine is not an integer — just so the other power is an odd positive
integer.

Case 2: Both of the Exponents m, n Are Nonnegative Even Integers

In this case the integral / sin'" x cos" x dx can be evaluated by using the trigono-
metric identities

sin- .V = 2(1 - cos Iv) and cos" .v = |(1 + cos Iv)

which are direct consequences of the double-angle formulas

cos 2v = 1 — 2 sin- .v = 2 cos" .r — 1

In Examples 4 to 6, evaluate the given integral.

EXAMPLE 4 COS" kx dx. where k is a constant, k^O

cos- kx dx= \\(\ + cos Ikx) dx = \ \ dx + I \ co% Ikx dx

= —X + — 1 r
cos Ikx dx
(

2 2 J

Put (/ = 2Lx. so that du = 2k dx and dx = du/{2k). Thus,

— if
= —
sin M sin 2kx
cos 2kx dx = cos u\
I
du cos u du = 1-
C= 1-
C
2k ' 2k J 2k 2k

It follows that

J
,
cos- kx dx = —X + —\ 1 /

V
sin 2Lx \
+ C = —x + sin 2kx
-^ C
2 2 2/t / 2 4/t

EXAMPLE 5 sin"* x dx

SOLUTION sin"* x dx = (sin" xf dx = [i(l - cos It)]- dx


J

= 4 ( 1 ~ 2 cos Zx + cos- Zx) dx

= 4 i dx — 2 \ cos 2x dx + 4 I COS" 2x dx

2^2/ -h—4X2—
X 1 / sin Zx \ 1 / X sin 4x \
=
4
-h
8/ +C
where we used the substitution u = Zx to evaluate the middle integral and we used
the result of Example 4 with A: = 2 to evaluate / cos- Zx dx. Combining terms and
simplifying, we have

f , 3.V sin Zx sin 4.i:


sin .V dx = 1

h C
488 C HAriKR 8 TECHNIQUES OF INTEGRATION

EXAMPLE 6 sin'* Iv cos- Zv dx


/

sin"* Iv cos- Iv dx = I ||(1 - cos 4.v)]-|2(l + cos 4.r)l dx


J

= 8 ( 1
~ 2 cos 4.x + cos- 4,v)( 1 + cos 4.v) dx

= i ( 1
- cos 4.V - cos- 4.V + cos-* 4.v) (iv

= 8 c^.Y — 8 COS 4.V dx - 8 COS" 4.V dx + 8 cos^ 4,v dx

.V sin 4.V 1 /
—+
.V sin 8.v \
+ —1

8 32 8^2 16/8 I cos" 4.V cos 4x dx

.V sin 4.V sin 8.v


(1 - sin- 4.v) cos 4x dx
16 32 128 8

To evaluate the remaining integral, put u = sin 4.v, so that du = 4 cos 4a- dx and

( 1
- sin- 4a) cos 4x dx = — * ^
2.^
~ "''
4

1 / u^
-t- C
32 V 3

sin 4.V sin' 4Ar


+ C
32 96

Therefore,

[ A -^ sin 4a sin 8a sin 4a sin'' 4a


sin 2x cos- 2y dx
J
= + C
J 16 32 128 32 96
sin 8a sin-^ 4a
-I- C
16 128 96

The following chart summarizes the technique for evaluating integrals of the form

sin'" A cos" A dx

where m and n are nonnegative integers:

Cas Procedure

m is odd Use sitr .v = 1


- cos" a, and let u = cos v.

n is odd Use cos^ A = 1 - sin- a, and let u = sin a.

both m and it are even Use sin^ a = i(l - cos 2r) and cos- a = 5( I + cos Iv).

If both m and n are odd, apply the technique to the smaller exponent.
SECTION 8.1 INTEGRALS OK POWERS OF SINES AND COSINES 489

Integrals of Products of Sines and Cosines


Integrals of the form

sin itix cos iLx d\ sin m.x sin nx dx and cos in.x COS nx dx

play an important role in the mathematical analysis of periodic phenomena. These


integrals can be evaluated by using the following trigonometric identities. (See
Problem 38.)

1 sin s cos / = 2 sin (i + ?) + 2 sin (s — t)


2 sin 5 sin f = 2 cos {s— t) — 2 cos (s + t)
3 cos s cos / = 2 cos (s — t) + i cos {s + t)

EXAMPLE 7 Evaluate sin 3.v cos 4.v dx.

SOLUTION By Identity 1,

sin 3x cos 4x dx = [2 sin 7.v + 2 sin (— x)] dx = 2 (sin 7x — sin x) dx

cos 7x

14
+ —^ + C
cos X

Problem Set 8.1

In Problems 1 to 8. use the identity cos" x + sin^ x = 1 and appro-


priate substitutions to evaluate each integraL

1 cos' X dx 2 sin' 4x dx

3 sin' 2/ di 4 cos' 3v dv
J

5 sin^ 2v cos' Zx dx 6 cos' X sin' x dx

7 sin- X cos' x dx 8 sin' 4x COS" 4x dx

In Problems 9 to 14, use the identities cos" x = id + cos 2v) and


sin" X = 2( 1
— cos 2x) and appropriate substitutions to evaluate
each integral.

9
490 CHAPTER 8 TECHNIQUES OF INTEGRATION

39 Find the volume of the solid generated by revolving one arch of


31 sin"* Iv cos- Ix dx 32 sin' x dx
Jo '0 the sine curve about the .v axis.

33
f sin Ittx cos dx 34
f cos —Inx
— cos —Ittx— -
,
dx
40 Find the volume of the solid generated if the region under one
J(,
3ir.it
Jo 5 5 arch of the sine curve is revolved about the line y = —2.

41 Find the area under the curve y = cos" x between .x = and


In Problems 35 to 37, let m and n denote positive integers and verify X = 2tt.
each formula.
42 If n is an odd positive integer, show that /,"cos" x dx = 0.

= (0 if m 7^ n 43 Suppose that a particle is moving on the axis according .v to the


cos mx cos nx dx
Itt if m = n law of motion s =f(t). where v = ds/dt = sitf nt. U s =
when I = 0, find a formula for/(r), and locate the particle when
fO \fm9^n
36 sin mx sin nx dx = t = 8 seconds.
[jT if m = n

44 Suppose that /(at) = 2n=i o„ sin nx. where N is a positive inte-

= ger and a I, 02 a,v are constants. Show that


"I cos w:t sin nx dx

fix) sin mx dx
38 Verify Identities 1, 2, and 3 on page 489 by using the addition
formulas to expand the right sides. holds for m= 1, 2, . N. (Him: Use Problem 36.)

8,2 Integrals of Powers of Other


Trigonometric Functions
In this section we present some techniques for dealing with integrals of the fonn

tan'" (( sec" u du or cot"' u esc" u du

where m and n are nonnegative integers. The simplest integrals of this foiro are

tan u du \ cot u du (m = 1 and n = 0)

and sec u du CSC u du {m = and /; = 1)

To evaluate e tangen
the integral of the tangent, we write tan u as sin u/cos u and make
the change of variable v = cos u. Thus
Thus, since dv = -sin « du. we have

r f
sin M r —dv .

tan u du = du = = — In |v|
I

+ C
J J cos u J V

= —In |cos m| + C = In |cos m|~' + C


= In |sec u\ + C
A similar maneuver handles the integral of the cotangent (Problem 40).
The integration of the secant and cosecant functions requires a clever trick. To
find / sec u du, we write

f r sec M(sec u + tan u) [ see" u + sec u tan u


sec u du = \ (ill
= du
' > sec (/ + tan w •' sec m + tan u
SECTION 8.2 INTEGRALS OF POWERS OF OTHER TRIGONOMETRIC FUNCTIONS 491

and then we notice that the numerator of the integrand is the derivative of the
denominator. Thus, putting v = sec u + tan u. we have
dv = (sec « tan ii + see" u) du

and SO
J
sec u du = I —= In |i'| + C = In |sec u + tan m| + C

A similar trick — writing esc u as esc ;<(csc u — cot «)/(csc u — cot u) — effects the
integration of the cosecant (Problem 40). Thus, we have the following integration
formulas:

1 tan (( du = In |sec m| + C

2 cot /( du = In |sin i(| + C

3 sec u du = In |sec u + tan i(| + C

4 CSC u du = In |csc u — cot ;/| + C

In Examples 1 and 2, evaluate the given integral.

EX.wiPLE 1 tan 4.V dx

SOLUTION Put u = 4a-. so that du = 4 dx. Using Formula 1. we have

an 4.V
, .
dx = {
tan m
du
= —
\ f
tan u du
J 4 4 J

= 4 In |sec «| + C = i In |sec 4x\ + C


dx
EXAMPLE 2
cos 5.1:

SOLUTION Put u = 5.V, so that du = 5 dx. Using Formula 3. we have

J
f

cos
dx
—=
5.x J
[
sec
,
5.V
,
dx =
J
(
sec M —=—
du
5 5
I

J
C
sec u du

= 5 In |sec M + tan m| +C
=i In |sec 5jc + tan 5x| +C

Powers of Tangent, Cotangent, Secant, and Cosecant


In integrating positive integer powers or products of positive integer powers of tan,
cot, sec. and esc, the identities

tan- u = see" // and 1 + cot" u = csc" u

can often be used to advantage. The following examples illustrate the techniques.
492 niAPlKR » TKtHNIQUES OK INTEGRATION

In Examples 3 and 4, evaluate the f^iven integral.

EXAMPLE 3 cot' .V dx

SOLUTION

I cot" X dx = (csc^ .V - 1) dx = I csc" x dx — dx = —cot x -x + C

EXAMPLE 4 tan' 2v dx

SOLUTION We write tan'' Iv as tan 2v tan- Iv and then use the identity
tan- Ix = sec- 2v - 1 to obtain

tan^ 2v dx = tan 2v tan- 2v dx = tan 2v (sec- 2v — 1) dx

= I (tan 2.V sec" 2v — tan 2v) dx

= tan 2.V sec- 2v ^.v — tan 2x dx

To evaluate the integral / tan 2v sec- 2v dx, we make the substitution ii = tan 2r,
so that du = 2 sec" 2x dx and

J
tan 2v sec-
^
2.v dx =
[

J
u —=—
du
2
\

2 J
(
u du =
u'

4
1-
C=
tan- 2v

4
1- C

The integral / tan 2v dx is easily evaluated by the method already illustrated in

Example 1, page 491. Therefore,

J
f
tan^
,
2v dx = tan- 2v

4 2''1
In
,

sec 2v
,

+ C

To evaluate an integral of the form

tan'" .V sec" x dx

where m and /; are nonnegative integers and either n is even or m is odd. proceed as
in the following chart:

Case Procedure

Factor sec" x out of the integrand, use the identity sec" a = 1 + tan" .v

to rewrite what remains in terms of tan .v. and let u = tan x.

m is odd Factor sec x tan .v out of the integrand, use the identity tan" x = sec* x — 1

to rewrite what remains in terms of sec v, and let u = sec v.

Likewise, an integral of the form

cot"' .V esc" X dx

can be evaluated by following the chart above, but with secant replaced by cosecant
and tangent replaced by cotangent.
SECTION 8.2 INTEGRALS OF POWERS OF OTHER TRIGONOMETRIC FUNCTIONS 493

//; Examples 5 and 6. evaluate the given integral.

EXAMPLE 5 tan*" v sec"* .v dx

SOLUTION We follow the procedure for the case in which n is even:

tan*" .V sec"* x dx = tan^ .v see" x see" .v dx

= tan*" .v( 1 + tan- .v) see" .v dx

= (tan* .V +tan'* x) see" v dx


I

Now, we put II = tan x so that du = see- x dx and


r

tan*" A- sec-* .v dx =
r

(«* + k**) du = —+—+C


7 y

= — tan^ X + — tan^ .v + C
7 9

EXAMPLE 6
I
cot'' .V CSC-"" .V dx

SOLUTION Since the integrand involves an odd power of cot x. we follow the
procedure for the case in which m is odd, but with functions replaced by cofunc-
tions. Thus.

cot'' .V CSC"* X dx = cof* .V CSC" a(csc .v cot .v) dx

= (cot- A')- CSC" .v(csc X cot a) dx

= (csc" X — 1)- esc- A (esc a cot a) dx

Putting u = CSC A, so that du = —csc a cot a ^a. we have

cot^ X csc'' A dx = (;r - l)-u-{-du) = - («•* - 2;/" + l);r du

(«*> - 2u* + //-) du


-I
= -7" + 5"' - 3"- -I- C
= — 7CSC^ X + fcsc' .Y — 3CSC'^ X + C

The techniques illustrated above are not particularly helpful for evaluating

tan'" A sec" a dx

when m is even and n is odd. Using the identity tan" a = sec- a: — 1, you can
always rewrite such an integral as a sum of constant multiples of integrals of odd
powers of sec a (Problem 42). Methods for evaluating

sec" A ^A

when n is odd and greater than 1 are discussed later (see pages 504 and 529).
494 CHAPTER 8 TECHNIQUES OF INTEGRATION

Problem Set 8.2

In Problems I to 36, evaluate each integral.

1
SECTION 8.3 INTEGRATION BY TRIGONOMETRIC SUBSTITUTION 495

to obtain

Vcr — ir = Vfl- - a- sirr = Va"(l - sin" d) = Va' cos" 8 = a cos d

Here we are assuming that -tt/I < < tt/I, so that cos d> 0.
The trigonometric substitution

a sin 6 sin 6 =

Figure 1 is illustrated geometrically by the right triangle in Figure la. Likewise, trigonomet-
ric substitutions corresponding to the expressions a~ + ir and ir - a'^ are suggested
by the right triangles in Figure lb and Ic. The three trigonometric substitutions
illustrated in Figure 1 are summarized in the following chart.

For the expression

Figure 2

Figure 3
496 CHAPIER 8 TECHNIQUES OF INTEGRATION

In order to evaluate the last integral, we put v = sin 6, so that civ = cos 6 eld and

f cos e de _J_l"i^___L,^^_
_1_ + C = CSC + C
~ ~ 9v 9 sin 9
9 J sin- e 9 J V-

From Figure 3, csc Q = Va^ + 9/.v; hence,

+
^ ^ V.v' 9
+ C
/^^^ i
J x~\/x^ + 9
x~V}p~+9 9,v

EXAMPLE 3 Evaluate -
,
- ^25
i ,
^
\ J PVr
Figure 4 SOfcUTION Make the substitution t = 5 aec 6 (Figure 4), so that

dt = 5 sec « tan e dd and /- - 25 = 25 sec" 6-15 = 25 tan-

Thus,

5 sec tan 6 dd
J PVF^^5 J (125 sec-' e)V25 tan- 6

\ { de
-,—= If, cos- e ^/(>
125 J sec- e 125 J

125 /4 ( 1 + cos 20) ^e =


250
+ —— sin
I
Id \
I +C
1 2 sin 6 cos
C = (e+ sin 0COS 6) + C
250 250

By Figure 4, 6 = sec ' (r/5), sin = Vr - 25//, and cos = 5/f; hence

r Jf _ 1 / _,j_ 5Vf- - 25
+ c
J f3V,2 -
rVr^ OS
25 950
250 V 5

2V3/3
^„
EXAMPLE 4 Evaluate
i:
2/3 mV9m- + 4
SOLUTION Make the substitution 3/( = 2 tan 6. or » = if tan 6. so that

du = 5 sec- e dd and V'9(/- + 4 = V4 tan- 6 + 4 = 2 sec

Here, 6 = tan"' (3((/2), so that = tt/A when « = § and d = 7r/3 when u

2V3/3. Therefore,
r2V3/3 f -/3 §sec-
:- e de_ "^^ sec Je
^t, ^ _ \_ r

4/3 «V9m- + 4 J„/4


'„/4 3 tan
§ laii (7 • 2 sec 6 /4 tan d

"'^
= —1
f^/-' 1/cos
'— 6 '1 de
at; = — 1 r
rf6»
2 4/4 sin (J/cos 6 2 L
\
I

4/4 sin Q

csc f/6 = — In |csc d - cot 0|


2 •'7r/4 2 1^/4

_1_ 1/V3
['"(vT-^)-'"^^-"] =
2 T'" VI- 1

\fl-\fi
SECTION 8.3 iNTEGRATION BY TRIGONOMETRIC SUBSTITUTION 497

Integrals Involving ax^ + bx + c

If a quadratic expression ax^ + b.x + c occurs in an integrand, it may be useful to


rewrite the expression in the form a(.x — h)- + k by completing the square* and then
to make the substitution u = v — /;.

In Examples 5 and 6. evaluate the given integral,

f dx
EXAMPLE S
(5 - 4.V - .v-)-'^-

SOLUTION Completing the square, we have

5 - 4jc - .r^ = 5 - (.r^ + 4.V) = 5 + 4 - (a- + 4.v + 4)

= 9 - (.V + 2)- = 9 - ir

where we have put u = x + 2. Thus, dii = dx. and we have

jix r dii

- - -)3/- ~ J -
(5 4jc r^)^/- (9 u-f^-

Now. we make the trigonometric substitution m = 3 sin 6. so that dii = 3 cos 6 dd


and 9 - M- = 9 - 9 sin- 6 = 9 cos' 6. Thus,

3 cos B dd 3 cos 6
~
(9 cos- e?'- 27 cos -'
e 9 J cos- e

= h\ sec- ede = hiwe + c


Figure 5 Referring to Figure 5, we find that tan 6 = m/V9 — ir: hence.

+
dx
— tan -I- C= -^ C =
x 2
+ C
J (5- Ax- x^f- 9 9V9^ 9V5 - 4x- x^

X d\
EXAMPLE 6
V4jr + 8.V + 5

SOLUTION Completing the square, we have

4a^ + 8jc + 5 = 4(.v- + 2y) + 5 = 4(a- -t- 2v + 1 ) + 5 - 4

= 4(A- + 1
)- + 1 = 4ir + 1

where we have put u =x + 1 . Here, a = m — 1 and dx = du, so that

X dx ( {11 — \) dii

V4a- -I- 8a + 5 V4h- + 1

Now, we make the trigonometric substitution 2m = tan 6. so that w = 2 tan B.


du = 1 sec- B dB, and Air + 1 = tan- B+ \ = sec^ 6. Thus.

{{ii-\)dit
— tan 0- 0)^0
, ,
=
r (i
—^—
1)(7 sec-
= —1 f
(tan 0-2) sec B dB
VAu- + 1 J . Vsec- B A J

= \\ (sec e tan - 2 sec &) dB


= i sec - i In |sec B + iw B\ + C

*The technique for completing the square is illustrated in connection with Theorem 1.

page 33.
498 CHAPTER 8 TECHNIQUES OF INTEGRATION

Here we have

sec e = Vtan- 6 + 1 = V4tr + 1 = V4.r + 8.r + 5

and tan 6 = 2u = 2(.v + 1 )


= Iv + 2

Hence,

J
,

V4?H+
V4.r
d:x

8.V + 5
= —4 V4jr + 8^ + 5
2
In
I
V4a- + 8.t + 5 + Iv + 2
I
+C

In the examples above, we have not taken care to impose restrictions on the

variables so that quantities under square-root signs are nonnegative. inverse trigono-
metric fun ctions are p roperly defined, and so forth. For instance, in order to write
sec d = Vtan- d + 1 in the last example, we should have restricted B to values for
which sec 6 is positive. Furthermore, Figures 1 through 5 apply only to quadrant I

angles 6. Nevertheless, integral formulas obtained by trigonometric substitution are


quite often correct in spite of such negligence — for an indication of why, see Prob-
lems 52 through 54. Of course, an integral formula — once obtained by any method,
no matter how questionable can and should be checked by differentiation (see
Problem 23).

Problem Set 8.3

In Problems 1 to 22. use an appropriate trigonometric substitution to iv


- r
evaluate each integral.
•^3/8 V'4.r -I- 1
22 fv^25 (if

V9^^ 23 Check your answers to Problems 1. 13. and 19 by differentia-


.rVie - .r^ tion, carefully noting any required restrictions on the values of
the variables involved.
dt
\-. 'V4^^
24 Evaluate the integral / (.r (iv)/Vl - .r in two ways, and recon-
_£_d^_ cile the answers, (a) Use the substitution m = 1 - r^. (b) Use
[ 6 V9 - 2m- du trigonometric substitution. Which method is easier?
i \/4 - Qr J

8 -^ — =
TTH , a > In Problems 25 to 30, complete the square in the quadratic expres-
sion and then use trigonometric substitution to evaluate the integral.

x^ dx_
*" /-v "4 r xdx
25 26
(5 - 4? - r-)-'^-
J V2 - X - r
Jx
xW4 + jc 27 I ,
" . dt 28 f ^
/'
J (r- + 3^-1- 4)- J V2t- - 6/ + :

f dx
A dx X dx
J (.r + 9)- 29 •
30
J Vl -x + 3.r
dt
16
/ Vl6f^ -I- 9
In Problems 3 1 to 48, evaluate each integral. (Some of the integrals

dt may be easier to evaluate by using methods other than trigonometric


18 substitution or completing the square.)
J ,4V^
Vr-9 dx
V9t^ - 4
20 rfy
"I (4x^ - 9f'^
32 /- I dx
SECTION 8.4 INTEGRATION BY PARTS 499

by the curves y = x/(x^ + 16)'^', y = 0, and at = 4 is revolved


33
\vi
r - 4; + 8 J \/9T? about the x axis.

dx 52 Let a > 0. and suppose that it is required to integrate an expres-


35 36 sion involving \'a--ir, where |m| < a. Since \u/a\<l,
8;c-4.r J (?- - 2f + 1)-
we can make the substitution = sin '
(m/o), where
f xdx - 17/2 < e< tt/2. Show that m = a sin e and that
37 38 / .
-^
)'.
.
i V4-.r J (X- + 6,v + 1 (a) Vo^ = a cos 6 (b) du = a cos dO

39 +
dx (c) CSC e = — if H # (d) sec =
JV2 3.V .r ^v
J V4T'9r
f 2v+ 2 f sec^ f rff (e) tan 6 (f ) cot = if M #
41 dx Va^
J \/7~+2xT. J (tan^ f + 9)''

53 Using the results in Problem 52, explain why it is not really


sin V dv
«/. (25 - cos- v)^'2
44 f
^? necessary to assume that u is positive in using the trigonometric
substitution « = a sin d.

Jo (25 - 9/-)^'-
46 r-^ 54 Discuss the trigonometric substitution k
in which u is not necessarily positive.
= a tan for the case

j-6 vr^-9 ^ 55 Find the arc length of the curve y = sin"' e" from x = In 2 to
47 dt

56 A vertical wire of uniform density with total mass M and length /


48 (3y + 2)V9v- + 12v + 3 dy
J stands with its lower end on the v axis at the point (a. 0). If G
denotes the constant of gravitation, calculate the horizontal com-
ponent of the force of gravity exerted by the wire on a particle of
mass m situated at the origin.
49 Find the area bounded by the curves v = 45/Vl6.ir — 175.

y = 0, -v = 4, and v = 5. 57 Solve the separable differential equation

50 Evaluate / (_v" rfy)/(4 + y~)' - by making the hyperbolic substi- -r dy - Vx^ -9 dx =


tution y = 2 sinh I.
58 Find the surface area generated by revolving the arc of the curve

51 Find the volume of the solid generated when the region bounded .V = e^ between (1.0) and (e-. 2) about the y axis.

8.4 Integration by Parts

Nearly ever>' rule or technique for differentiation can be inverted to yield a corre-
sponding rule or technique for integration. For instance, the technique of integration
by change of variable is essentially the inversion of the chain rule for differentia-
tion. In this section we study the integration technique, called integration by parts,
that results from an inversion of the product rule for differentiation.

According to the product rule,

if-8y=f'-g+f-g'
the function/-^ is an antiderivative of/' •
g +f-g''- that is,

[/'(.v)g(.v) +f(x)g\x)] dx =f(x)g(x) + C


J

or

f'(x)g(x) dx + /(.v)g'(.v) dx =/(.r)g(.r) + C


J J
500 CHAPrKR 8 TECHNIQUES OF INTEGRATION

The last equation can be rewritten as

J\x)s'(x) dx =f{.x)g{.x) - g(.x)f'(.x) dx


J J

provided it is understood that the constant C is absorbed into the constant of inte-

gration corresponding to / g{x)f'lx) dx.


The equation obtained above is easier to remember if we let

u=f(x) and V = g(x)

so that du =f(x)dx and dv = g'(x) dx

and we have

II dv = iiv V du

This is the formula for integration by parts.

EXAMPLE I Evaluate .v sin .v dx.

SOLUTION In order to use integration by parts, we put

u = .V and dv = sin .v dx

so that du = dx and •-/ sin .V dx = —cos .v + Co

Thus,

X sin -v dx = u dv = uv - v du = .v( — cos .v + Co) - I (—cos .v + Co) dx


I

" '''
r r
= —.V cos X + .vCo + cos A ^.v - Co dx
= —X cos X + .vCo + sin .v + C - Co-V

= —.V cos .V + sin .V + C

In the preceding example, the constant of integration Co arising from the prelimi-
nary integration \' = /sin .v dx cancels out in the end. In carry ing out integration by

parts, this always happens (Problem 52); hence, the constant arising from the pre-
liminary integration >' = / dv need not be written into the calculation. Thus, we
would write

A sin A dx - u dv = uv - v du = .y(-cos a) - (-cos .v) dx

= —A cos A -I- sin A + C

In order to use the formula

u dv = uv — I
V du

to evaluate an integral, you must begin by choosing u and dv. Often the choice can
be made in several different ways, and a poor choice of u and dv may result in an
integral r

V du
J
SECTION 8.4 INTEGRATION BY PARTS 501

that is more complicated than the one you started with. Try to choose u and dv so
that / I'du is as simple as possible.

In Examples 2 to 4, evaluate the given integral using integration by parts.

EX.4MPLE 2 .V sec- .V d.X


I

SOLUTION Here, there seem to be two reasonable choices:

Choice 1 u = x dv = sec- x dx

so that du = dx v = I sec" x dx = tan x

Choice 2 « = sec- .v dv = x dx

so that du = 2 sec- .v tan x dx v = x dx = —


If we use choice 1

I V du = tan .v dx

whereas if we use choice 2

I V du =
I
( — ) 2 sec- X tan x dx

Evidently, / v du is simpler if we use choice 1. Thus, we let

u =x dv = sec- X dx

du = dx V = tan .\

so that

sec- Y dx = " dv = uv - v du =x tan x - tan x dx


j .^ J
u dv
= X tan X - -In (
|cos .v|) + C = x tan x + In Icos .vl -I- C

EXAMPLE 3 In A dx

SOLUTION Here, we don't seem to have much choice, so we put

// = In X dv = dx

du = — dv1

V =X
X

Thus, hT_v dx^= (( dv = uv - \ v du = (In .v).v - I a|— dx)


J J '^
II dv

= A In X — dx

= X In A -X + C
502 CHAPTER 8 TECHNIQl ES OF INTEGRATION

EXAMPLE 4
J
V^ COS X~ dx

SOLUTION Before using integration by parts, we make the substitution

f = .r so that dt - Iv dx
and

x^ cos .v"^ J.V = 2 .v~ cos .v" (Ix dx) = \ I cos 1 dt

To evaluate the last integral, we use integration by parts with

M = t dv = cos t dt

du = dt '' ^ '^^'^ ' ^' ~ ^'" '

Thus,

x^ cos x~ dx = 2 \ I cos I dt = 2\\ u dv\ = 2! wv - I v du


II dv

= 2y sin / - sin t dt) = hit sin r + cos /) + C


= 2(-v^ sin -ir + cos x~) + C

In the formula for integration by parts, suppose that u and v are functions of (say)
the variable v. Then iiv. as well as the indefinite integrals / u dv and / i' du. de-
pend on Consider the definite integral /;;„ u dv. where we have written the
.r.

limits of integration as x = a to x = b to emphasize that x (rather than u or v) is the


variable in question. We have

t=h rv=/j
= iuv -
u dv = iuv)
r V du

the formula for integration by parts of definite integrals.

EXAMPLE 5 Use integration by parts to find .ir In x dx.


•'1

SOLUTION We put

(/ = in A' dv = X' dx

= — dx1

j^'dx =
du
^
Thus,
re ^ r.=. ,,=, r.=..

x~ \n X dx = \ u dv = (uv) - V du
'1 ';t=l '.t=l '.1=1

"=' /"'=' "=


-r' \ x-^ dx x^ In X
(In .V)
3 '
>=, C 3 .V
" 3 ,=
SECTION 8.4 INTEGRATION BY PARTS 503

Repeated Integration by Parts


The formula / u dv = uv - J v du for integration by parts converts the problem of
evaluating / u dv to the problem of evaluating / v du. By suitable choice of u and
dr. we can often arrange it so that J v du is simpler than / u dv. however, we may
not be able to evaluate / v du directly. A second integration by parts might be
necessary to evaluate J v du. In fact, several successive integrations by parts could
be required.

EXAMPLE 6 Evaluate I .x-e-" dx.

SOLUTION Put M = .r^ and dv = e-^ dx, so du = Ix dx and v = / c"' dx - je'^-

Thus,

.ire^^ dx = uv - v du = i're"'' - .xe'^ dx

Although / .xe-' dx is simpler than the original / .x-f-' dx. a second integration by
parts is required to evaluate it. Thus, put hi = v and rfv, = e'^ dx. so that du^ = dx
and \i
= J e-" dx = 2e~^ Now.

.vp-^ d.x = i<iVi - V| du\ = 2-Te-^ - 2*''^'^ dx = i-vf-' - le""^ + C]

Hence,

x^e-"" dx = kx^e-'' - .xe-'' dx

= Ix'e-' - (ive-^ - \e-' + C,)

= hre~' - he-" + \e-" + C


where C= —C\.

Repeated integration by parts of integrands of the form

e"^"" sin bx e'" cos bx sec" x or esc" x

where a and b are constants and n an odd positive integer, leads back to an
is

integrand similar to the original one. When this happens, the resulting equation can
often be solved for the unknown integral. This is illustrated by the following exam-
ples.

EXAMPLE 7 Evaluate e' cos x dx.

SOLUTION An initial integration by parts with u = e". dv = cos x dx. du =


e^ dx, and v = sin x gives

e' cos X dx = e' sin .v — e^ sin x dx + C\ (1)

A second integration by parts with Ui = e\ dvy = sin x dx, dui = e' dx, and v, =
—cos X applied to J e"" sin x dx gives

e' sin X dx = -e" cos .t + e" cos x dx + Cj (2)


504 CHAI'^lKR « lECHNIQLKS OK INTEGRATION

Substitution of (2) into (1) yields

('" cos .V d.\ = e' sin .v - i-e' cos .v + e' cos .v dx + Czj + C| (3)

or <'" cos A civ = e' sin a + e' cos a - e' cos a d.x + C3 (4)
J

where wc have put C, = -C2 + C|. Solving (4) for / e' cos a dv, we obtain

2 e* cos A dx = f ' sin a + <' cos a + C3

. e'^ sin A + e' cos a


so that e' cos a fiv
= + C

where we have put C = C^/2.

EXAMPLE 8 Evaluate sec' a dx.

SOLUTION We already noticed in Section 8.2 that the methods used there are not
effective in evaluating the integral of sec^ a; however, we now use integration by
parts to find such an integral. Factor sec' a as sec a sec- a and put

u = sec A f/v = sec- x dx

du = sec A tan x dx v = I sec- a dx = tan a

Thus,

sec' X dx = sec a see" a dx = sec a tan a — sec a tan- a dx + Co

Since

sec A tan- x dx = sec A(sec- a


..
— 1) dx = sec'
_ _ a dx — I sec a dx

it follows that

sec' A dx = sec x tan v - ( sec' x dx - sec a dx\ -t- Co

= sec A tan v - sec' a dx + sec a dx + Co

= sec A tan a - sec' a dx + In |sec a + tan a| + Co

From the last equation,

2 sec' A dv = sec a tan a + In |sec a + tan a| -I- Co

sec' V dx = ^r sec a tan a + — In |sec x -t- tan a| + C


2 '
2

where we have put C= jCo-


SECTION 8.4 INTEGRATION B\ PARTS 505

The Tabular Method


Repeated integration by pans can be accomplished by a shortcut called the tabular
method. To see how this method works, suppose that ;/ and v are two functions, and
consider the table

derivative of ii antiderivative of v
derivative of ii. antiderivative of vi

derivative of u„ antiderivative of v„
derivative of «,, antiderivative of i'

Multiply each function — except the last — in the first column by the corresponding
function in the second column, change the sign of every other product, and add the
resulting terms to obtain a sum S:

times ( +

( + )

"2

(+)

Then, as can be proved by using the principle of mathematical induction (Prob-


lem 62),

/( dv "«*i dv„+i
J

where the plus sign is used if /; is odd and the minus sign if n is even.
If M is a polynomial of degree /;, then the in + l)st derivative of u is zero; hence,
M„+i = 0, and the tabular method yields the simple formula

II dv = S + C

where C is the constant of integration.

EXAMPLE 9 Use the tabular method to evaluate (Iv"' - 8.v-^)e"-"' dx.

Let

u = lx* and dv = e-^' dx

so that

(IV* - 8x^)e~^' dx = I iidv


J
Here

i' = j
dv= I e'^' dx = - y-^""
506 CHAPTKR 8 TECHNIQUES OF INTEGRATION

(Again, notice that when you are using the tabular method, constants of integration
can be neglected until the very end.) Thus, our table takes the form

2,4 _ 8^1 Jime^ _y-x. _A±U +ax' - Sx-^X- he-'^^)


o T -,. -> times

~. > .„ times _3 ( +)
24Ar - 4&X > -Jfe
J
^ +(24a^ - 48x)(- ire'^'^
times
48.V - 48 ^ ire-''
^ '
> -(48.t - 48)(8Vf
^3>)
times 1 _,, ( + ) .„, _i_
3^-3.)
-243^ ,
+48(-243
,

48 * *

Adding the terms in the last column to obtain S and simplifying, we have

{2x^ - Sx^)e-^' dx = S + C
1
= (- i;c^ + ¥x^ + ¥x^ + if.r + M)e-'' + c

Problem Set 8.4

In Problems 1 to 12, use integration by parts to evaluate each inte- In Problems 23 to 26, use a suitable substitution to express the
gral. integral in a form so that integration by parts is applicable. Then
evaluate the integral.

1 I .r cos Iv dx 2 I .V sin Lx dx
23 j.rV"^ dx 24 .v' sin Ix^ dx
J
3 jxe^'dx 4 I xe~*' dx
25 V + 1 .V- dx 26 I cos X tan"' (sin x) dx

5 I In 5.r dx 6 \ X \n 2x dx

In Problems 27 to 46, evaluate each integral.


7 cos"' X dx 8 .v' In (.r-) dx
J

27 (2.x:- \)e-' dx 28 X sinh x dx


J
9 I sec"' X dx 10 sin '
3.vaLv
J

29 .ve" sin A dx 30 In (1 + r)dt


J
11 / sec Man / dt 12 tan
I .V dx
J

31 (In .V)- dx 32 sin \/^ dx


J J
In Problems 13 to 22, use repeated integration by parts to evaluate
each integral. 33 X CSC" X dx , 34 cosh '
.V (iv

13
J
.r= sin 3.V dx 14
I
.V" sin' jr dx 35
f^ dx
36
V.r- - 1

15 (Sat^ - 2t + 1) cos jf (ir 16 U^ - 3.v + 2)e~'' dx 37 .v" cosx'* dx 38 .r^'" cos


x^^- Vx ate (

J J J J

f' .Vf'
n j(Y+x)e^dx 18 |.x-
x~ sec'
I jf tan x dx
39 4x' sin 3x dx 40
i 'o + .V)-
(l+.v)-
( 1
vdx

41 dx
19 I e ' cos 2.t d[r 20 I e'^' sin ;c dLx I sec '
,v
«I cos"' X dx

21 esc' X dx 22 e'" sin &v dlx 43 {5x- - 3x + sin x dx 44 sin (In x)dx
r
I
1 )

-'i
SECTION 8.5 INTEGRATION BY PARTIAL FRACTIONS— LINEAR CASE 507

53 Use integration by parts to prove that


45 x sin X dx 46 X tan ' Vx dx

f(x) dx = xfix) - xf(x) dx


J J

In Problems 47 to 50, use the tabular method to evaluate each inte-


54 Show that the volume of the solid generated by revolving the
gral. region under the graph of y = sin x between x = and x = tt

about the v axis is four times the volume of the solid generated

47 X cos Z\ dx
when the same region is revolved about the x axis. (Hint: When
revolving about the y axis, use cylindrical shells.)

55 Derive a formula for / sec' x dx.


48 U' - Iv' + .v)e'^ d:x:

56 If a is a positive constant, derive a formula for / Va^ + x^ dx.

49 f*e-' ^f 57 Suppose that g" = f. Evaluate / x-f(x) dx.


J

58 Evaluate / V2 - 3.v -I- x- dx. (Hint: Complete the square,


50 (.v' -x^ + x)e~' dx change the variable, and then use the result of Problem 56.)

59 Let/U) = xe"' for jr a 0. Suppose thalf(x) takes on its maxi-


51 Use integration by parts to show that mum value when x = a. Find the area of the region under the
graph of / between x = and x = a.
x-f'"M dx = a~f"(a) - 2af'{a) + 2f(a) - 2/(0)
I
-'o
60 Find the volume of the solid generated if the region described in

52 It is desired to evaluate / F\{x)F2ix) dx by parts. To this end, we Problem 59 is rotated about the x axis.

put u = f lU) and dv = Fi(.v) dx. Then du = F[{x) dx and v =


61 If a is a constant, derive a formula for J x" In x dx.
G(x) + Co, where G is an antiderivative of fi and Q is the
constant of integration. Show that Co always cancels in the cal- 62 Using the principle of mathematical induction, justify the tabular

culation of J Fi{x)F2{x) dx by parts. method of integration.

8.5 Integration by Partial Fractions — Linear Case


Recall that a rational function h has the form

P(x)
h(x) =
Q(x)

where P{x) and Q{x) are polynomials and Q(x) is not the zero polynomial. As we
shall see in this section and the next, it is possible to find the integral of a rational
function by expanding it into a sum of simple rational functions called partial
fractions and then finding the integrals of these partial fractions.
A rational function is said to be proper if the degree of its denominator is greater

than the degree of its numerator. If a rational function is not proper, you can
perform a long division

quotient

denominator ) numerator with remainder

numerator remainder
and write quotient
denominator denominator

where the rational function

remainder
denominator
508 CHAPTER 8 TECHNIQUES OF INTEGRATION

is proper. For instance, the rational function

6.V'* + .v-^ + 4.r + 4


Ix- + .V - 1

is not proper; however, by long division we find that

3x- - X +2
Zx~ + Jt - 1 ) 6x'* + x^ + 4jf + 4 with remainder .v + 6

dx'* + x^ + 4x + 4 , X + 6
so that ; = i.v-^ - .V + 2 +
2v^ + JT - 1 Iv- + .r - 1

Thus, by means of long division, a rational function that is not proper can always
be rewritten as the sum of a polynomial function and a proper rational function.
Using the procedure explained above, you can rewrite the integral of a rational
function that isn't proper as the sum of the integral of a polynomial function and the
integral of a proper rational function. Since there's no difficulty in integrating the
polynomial function, the problem is thereby reduced to integrating the proper ra-
tional function. Thus, in what follows, we can concentrate on the problem of inte-

grating proper rational functions. Of course, we assume that all common factors in
numerators and denominators have been canceled, so that the proper rational func-
tions are in reduced form.

The Case in Which the Denominator


Factors into Distinct Linear Factors

2 3
Notice that the two fractions and can be added to obtain
.V - 2 .V + 1

2 3 2(x + 1) + 3(.v - 2) 5.V - 4


,Y+1 {x~2)(x+l) {x-2)(x+l)
It follows that

f
—(5x- 4)dx { 2dx f 3dx ^
-
=
-2
-I- = 2 In
,

Lv -2+3 ,

In Lv
,

+ ,

\\ + C
J (jc 2)(x +1) J x J X + I
'

5.V - 4
Therefore, it is a simple matter to integrate provided that we
(X - 2){x + 1)

know it decomposes into the sum of the partial fractions

2 3
and
.V - 2 X + ]

Decomposition of a rational function into partial fractions is easiest when the


denominator factors into distinct (that is, different) linear factors. In this case, it is

only necessary to provide a partial fraction of the form

for each of the linear factors ax + h in the denominator The constant numerators of .

the partial fractions can be denoted by A, B, C, and so forth.


SECTION 8.5 INTEGRATION BY PARTIAL FRACTIONS— LINEAR CASE 509

5.\- - 4
For instance, the fraction is decomposed as follows:
(.V- 2)(A-+ 1)

5jc - 4 A B
+
ix- 2)(.t + 1) x-2 x+ \

Another example

(3.r -
llx^
l)(x
is

-
+
provided by

4jc

2)[(V2)
+ 13
- 7].v 3x
CD - 1 x + 2 (x/2)
E
- 7
F
x

In the case being considered, there are two methods for finding the values of the
numerators of the partial fractions.

1 Equating Coefficients

We have

P(x) ^
= a sum of , f
partial tractions

Q(.v)

where the numerators of the partial fractions are denoted by A.B, C. and so forth.

Multiply both sides of this equation by the denominator Q(x) to obtain

Fix) = Qix) (sum of partial fractions)

and then collect like terms on the right side. Equate the coefficients of powers of .r

on the left side to the coefficients of corresponding powers of .v on the right side,

and thus obtain a system of equations involving the unknowns^. B. C. and so forth.

Solve this system for the unknowns.

5.r-4
EXAMPLE 1 Find the partial-fraction
'^
decomposition of :;
—by equat-
'--2)(.r+l)
ing coefficients.
510 CHAPTER 8 TECHNIQUES OF INTEGRATION

2 Substitution

Suppose that

ax + b
is any one of the partial fractions in the decomposition of P(.x)/Q{x). Then ax + bis
one of the factors of Q(x). so we can write

eU) = (or + b)QM


where Qi(x) is the remaining part of the denominator. Thus,

Pix)
= A
h Other partial fractions
(ax + b)Qi{x) ax + b

Multiply both sides of this equation by ax + b to obtain

P{x)
=A + (ax + fc)(other partial fractions)

If we now put .v = -b/a, then ax + b = and the equation becomes

P(-b/a) ^^
Qd-b/a)
Thus, we obtain the numerical value of A. Repeat this procedure for each of the
partial fractions.

5x - 4
EXAMPLE 2 Fmd the partial-fraction decomposition of by sub-
stitution.
U--2)U+1)

SOLUTION Again

5.V - 4 A B
+
(x-2)(x+ 1)

where the constants A and B must be determined. Multiply both sides of this equa-
tion by .V — 2, and then substitute .v = 2 to obtain

5.1-4 B
A + (X -2)
x+ \ X + I

10-4 B
A + (0)
2-1-1 2-^1

so that

—6 = -4 or A = 2
3

Similarly, to find B. multiply both sides of the original equation by .v + 1 , and then
substitute x = — 1 to obtain

-5-4
= B or S = 3
1-2
5;c-4 2 3
-., .
Therefore, = 1-

(jc- 2)(x+ 1) x-2 x+ \


SECTION 8.S INTEGRATION B\ PARTIAL FRACTIONS— LINEAR CASE 511

The method of substitution can be shortened simply by temporarily '


'covering
up." or "disregarding. appropriate portions of the equation and then substitut-
"

ing. For instance, to find the constant A in

5.V - 4 A B
{.X- 2){x+ 1) .V - 2 .v+ 1

we cover up, or disregard, everything on the right side of the equation except A, and
we cover up, or disregard, the factor in the denominator on the left side that corre-
sponds to A. This produces the equation

5.V - 4 A B
+
{x - 2)(A + 1 .V - 2 jc + 1

into which we substitute the value a = 2 to obtain

10-4
=A or A = 2
2 + 1

Similarly, to find B. we substitute x = - 1 into

5.V - 4 A B
(A- 2)(x+ 1) x-2 x+ I

-5-4
to obtam = B or B = "?

-1-2
This method is called the short method of substitution.

In Examples 3 and 4. evaluate the given integral.

( Ir - 5
EXAMPLE 3 -^ dx
J X- - X-2
SOLUTION X' - X - 2 = (X - 2)(X + 1)

3.V-5 3.V-5 A B
Hence, -^ = = h
X- - A- - 2 (A - 2)(A + 1 ) A - 2 A + 1

By the short method of substitution

6-5 — 1
so that A =
2+1 3

-3-5
and = B so that
-1-2
Therefore,

J X- - X- 2 J X -2 J A + 1

= iln |a- 2| +1 In |a+ 1| +C


= In [|a - 2|'^-'|a + 1|«/-'] + C

,
15a-* - 17a-' + 41v- - 17a - 8
EXAMPLE 4 5 ; dx
15a-' - I 7a- - 4a
512 CHAPTKR 8 TECHNIQUES OF INTEGRATION

SOLUTION The integrand is an improper fraction. By long division,

-V

15.v' - 17a- - 4x ) \5.x^ - nx^ + 42x- - 17jc - 8


15x^ - 17.r^ - 4x ^
46.r- - 17.V- 8

so that

15.V-* - 17.v' + 41v- - 17.V - 8 46.v- - 17.v - 8


.V +
15.V-' - 17.V- - 4.V '
15.v' - 17.V- - 4.v

Factoring the denominator, we have

15a:^ - 17.v^ - 4x = x(\5x- - \lx - 4) = a:(5.v + l)(3.v - 4)

Hence,

46.v^ - 17.V - 8 A B C
+ +
.v(5.v + 1)(3.Y - 4) A 5a + I 3a - 4

Using (say) the short method of substitution, we find that

A = 2 B = -3 and C= 5

Therefore,

r 15a-*- 17a-' + 42x-- 17a- 8


-dx
\5x- - 17a- - 4a
y
dx
-
,

/ 5a + 1 3a 4

r f dx ( dx r dx
c
A ^A + 2 3 + 5
J J X J 5a + 1 J 3a

= —+
X
2
2 In A
,
,
3

5
In
,

5a + 1
,

+ — 5

3
in
,

3a - 4\ + K

where we have written the constant of integration as K to avoid confusing it with the
numerator of the partial fraction C/(3a - 4). Using the properties of logarithms, we
can rewrite the solution as

r 15a'* - 17a-'

15a-'
+ 42v- - 17a -
- 17a- - 4a
8
^A "
= —+
A"
2
In
a-|3a
-r
|5a +
- 41'^-'

:hr-
l|-'^-'
+ K

The Case in Which the Denominator


Has Repeated Linear Factors
We now consider the case of a rational function P(x)/Q{x) whose denominator Q(x)
factors completely into linear factors, not all of which are distinct. An example
would be

QM = (3a - 2)(A - 1)(3a - 2)(A - 1)(a + 1)(a - 1)

in which the factor (3a - 2) appears twice and the factor (a — 1) appears three
times. The first step in handling such a case is to gather like factors and write them
with the aid of exponents, in the example at hand, we have

e(A) = (A- 1)\3a-2)-(a+ 1)


SECTION 8.5 INTEGRATION BV PARTIAL FRACTIONS— LINEAR CASE 513

If the denominator contains a factor of the form {a.x + Z?)* where k> \. it is
necessary to provide k corresponding partial fractions of the form

CIX + b (ax + b)- (ax + bf' (ax + bf


where Ai ,A2.Aj. are constants that need to be determined. This must be
A,,

done for each factor in the denominator. Nonrepeated factors are handled just as
before
For instance, the partial fraction decomposition for

3.r- + 4.V + 2

x(x + 1
)-

has the form


3.V- + 4v + 2 A B, B.
-v(.v + )- 1 A- .V + 1 (A- + 1 )-

Although the short method of substitution can no longer be used to determine all the
unknown constants, it can still be used to find constants, such as A. that correspond
to nonrepeated factors. Indeed.

(3)(0)2 + (4)(0) + 2
=A
(0 + 1)-

so A = 2.

The short method of substitution is also effective for determining the constant
numerators of the partial fractions involving the highest powers of the repeated
factors. (Why?) Here, for instance.

(3)(-l)- + (4)(-l) + 2
= fi.
(-1)

so B2 = 1
-
Unfortunately, the short method of substitution does not work for the

constant numerators of the partial fractions involving remaining (lower) powers of


the repeated factors, such as B, in the present example. These remaining constants
can be found by the method of equating coefficients.
For instance, we now have

3jc- + 4.V + 2 2 e, 1
+
x(x+ 1)- X x+ 1 lx+ 1)-

In order to find B\ . we multiply both sides of the equation by .v(.v + 1)- to obtain

3.V- + 4.V + 2 = 2(.v + 1)- + 5,.v(.v + I) - .v

that is, 3.V- + 4.v + 2 = (2 + B, )-V" + (3 + B^)x + 2

Equating coefficients of like powers, we have 3 = 2+B, and4 = 3+5i. There-


fore, Bi = 1, and so

3.V- + 4.V + 2 2 1 1
+
.V(.V + 1 )- .V A- + 1 (A + I
)-

Rational functions for which the denominator factors completely into linear fac-
tors can always be expressed in terms of partial fractions of the form

(ax + by
514 CHAPTKR 8 TFXHNIQLIES OF INTEGRATION

r Adx
Since
(ox + b)"

can easily be evaluated using the change of variable ii = ax + b, we see that the
integral of such a rational function can always be evaluated. This is illustrated by
the following examples.

In Examples 5 and 6. evaluate the given integral.

3x^ + 4x +
EXAMPLE 5
r
I

J
^^—x(x + l)"
X(X l)~
'"".
-,
— dx
SOLUTION Using the decomposition above, we have

J X(X + 1 )- J \-X X + \ (A- + 1


)- J

r 2dx r dx f dx
"J JC J X + I J {X + \f

= 2 In LtI + In \x + l| +
-v + 1

= In l.vP + In l.v + 1|
+ + C
A- + 1

1
SECTION 8.5 INTEGRATION BY PARTIAL FRACTIONS— LINEAR CASE 515

Problem Set 8.5

In Problems 1 to 30, evaluate each integral. {Caution: Some ,, f 2 a' + S.v" - 4jc' - a:' + 1 1.V + 12
34 ^7-5 : dx
integrands may require a preliminary long division.) J, xHx^ + 4x + 5)

f x+ :
[ -2
^^V- :^dx f- 2'-' + 1
35 36 ^^= V.
iv
J .v(.v - 2) J x' — X h (.v-2)- J, z(z+l)=

37 Evaluate J [(a.v + />)/(« + d)] dx if c 7^ 0. ( Hmr


J 6v- - y - 2
-^
J 2/- + 7r -
- +

J /^ - /- - 2f J Iv- + 3.V +

7 I

J A-'
3 7
+ .V-
^ -"

- Iv
^v
J --(.--
;(.- - 4)

9f-^
-J .V-'
(iv

X
10 f
i (t+ \\f-
'1^^
- 4f + 3)

'^' <^'^ •^' + 2.v=-3.V+


..
11
J
r

.V- - .V - 6
,,
12
f
J *'
5
—5
+ 3.r2 + 2r
1 ^
(i.V

13 -'-^V""^-^"^ ^. 14 ^^^
Jf -
.V- + -v 12 Jf (.V - l)(.v + 1)U +

+ - -
,^
15
J
f .V-'

x^
^
5.r-

+ 3a^ -
4;f

10
20 ,

±\
.^
16
f
J
^ +- X-Ir'-+Iv
.v''

.V-'
^
1

ax
^

.v-22
dx
J .v' + 3.V- - 4.V J X- + 2
3.V - 10

19
J
[^ x~
.V-
V"

+X—
dx
6

r 5.v' - 6.V- - 68.V - 16 , ,, f v' - 4v - 1

20
J .v^ - 2v^ - 8.V
"^
'M ,(.v-l)-' '

22
J (.v + 2)(.v-- 1) J z-(4z+ 1)

24
V + 3)(.v-
-^+ 4v + 4)
dx 25f—+ ^±^ +
J 7)
dx
(.V 1)-U-

"-^^ ^'-^-^-^'0.
26 . .d. 27 dx
Jf (.V- + 2v + l)(.v - 1)- Jf (.V+ 2)(3a: - 2)"
^-^-3^ + 5.^ 12 ^^ ,
r 4z^.z
28 f
J (v - )-(.v- - 3.V - 4)
1 J (z - l)^(z- - 4z + 3)

(i + 2) dt
30
(r- l)(r + 3)-

In Problems 31 to 36, evaluate each definite integral.

--^ „ f- 5f-
- 3? + 18
31 r 32
- ^
,

(/r
J,
'2 (.V +
(.v+ l)(jc
1 )(JC + 2) Ji '(9 r)

f' 4f-' - 3?" - 6/^ + 4r + 6r

J2 U-l)(r-l)
516 CHAPTER 8 TECHNIQUES OF INTEGRATION

values). Assume — as is, in fact, the case — that such a rational and females; hence, it is proportional to q'. The death rate is

function can always be decomposed into partial fractions. proportional to (/. Hence, dq/dl = Bq' - Aq. where B and A are

constants of proportionality. Solve this differential equation.


49 As we mentioned on page 264, the differential equation
= k(A - + 51 Sociologists sometimes use the following simplified model for
dy/dt y)(B y) applies to the formation of trypsin in
the biochemistry of digestion. Assuming that A, B, and k arc the spread of a rumor in a population: The rate at which the
positive constants, solve this differential equation.
rumor spreads is proportional to the number of contacts between
those who have heard it and those who have not. Thus, if/? is the

50 Ecologists sometimes u.se the following simplified growth proportion of the population who have heard the rumor at time /.
model y^r the population q of a sexually reproducing species: so that 1
—p is the proportion who have not yet heard it, then
The rate of change of </ is the birthrate minus the death rate. The dp/di = kp{\ - p). where k is the constant of proportionality.

birthrate depends on the frequency of contacts between males Solve this differential equation.

8.6 Integration by Partial Fractions


Quadratic Case
We now consider the problem of integrating a rational function P(x)/Q[x) whose
denominator Q(x) cannot be factored completely into linear factors. It can be proved
that any polynomial ^(.v) with real numbers as coefficients can be factored com-
pletely into a finite number of polynomials, each of which is either linear or quad-
ratic. Furthermore, the quadratic factors (if any) can be assumed to be
irreducible — that is, incapable of further factorization into linear polynomials.
For instance,

.V-' + .v" - x^ - 3.V- -I- 2 = U- l)(.v' -I- Iv -I- 2)(.v + l)(.v - 1)

as can be confirmed by direct multiplication of the factors on the right. The quad-
ratic polynomial .v" -I- 2v -I- 2 cannot be factored into linear factors with real coeffi-
cients; hence, it is irreducible.
It is easy to check whether a quadratic polynomial

ax^ + bx + c

is irreducible — // is irrediicihie if and only if its discriminant

b~ - Aac
is negative. For instance, .v" -I- Zv + 2 is irreducible because its discriminant
2^ - (4)(1)(2) = -4 is negative.
As in Section 8.5, we can assume without loss of generality that P(x)/Q{x) is a
proper fraction and that all common factors have been canceled from its numerator
and denominator.

The Case in Which the Denominator Involves


Distinct Irreducible Quadratic Factors

Assume that the denominator Q(x) of P(x)/Q(x) factors into linear and irreducible
quadratic polynomials, but that none of the quadratic factors are repeated. Again,
we seek to decompose P(.x)/Q(x) into suitable partial fractions. This is accom-
plished as follows:
SECTION 8.6 INTEGRATION BY PARTIAL FR.4CTIONS— Ql ADRATIC CASE 517

For each nonrepeated irreducible quadratic factor ax^ + b.v + c in the denomi-
nator of P(x)/Q(x). provide a corresponding partial fraction of the form

Ax + B
ax~ + bx + c

Again, it is necessary to determine the numerical values of A and B. say by equating


coefficients. Partial fractions corresponding to linear factors of the denominator
must be introduced, just as in Section 8.5.

In Examples 1 and 2, decompose the given rational fraction into partial fractions.

8.r- + 3.V+ 20
EX.WIPLE 1
.v^ + X- + 4.V + 4
SOLUTION Factoring the denominator, we obtain

x^ + x^ + 4.V + 4 = x\x + 1 ) + 4(.v + 1 ) = (.V + 1 )(.r- + 4)

The quadratic factor .v" + 4 is irreducible; hence we must provide a corresponding


partial fraction of the form

Ax + B
X- + 4

We must also introduce a partial fraction corresponding to the linear factor .v + 1

Consequently,

8.V- + 3.\ + 20 8.V- + 3.V + 20 Ax + B C


+
.V- + -v- + 4.V + 4 (.V + 1 )(.v- + 4) .V- + 4 .v + 1

We can evaluate the constant C by the short method of substitution, just as in

Section 8.5; thus,

(8)(-l)- + (3)(-l) + 20 ^ ^=
- = C so C ^
5
(-l)- + 4
+ + 20 Ax + B 5
Therefore,
8.V-

+
3.V
:;

+ 4)
= ; — 1

+
(.Y l)(.v- .V- + 4 X \

Multiplying both sides of the last equation by {.v + DCv" + 4) and collecting terms
on the right, we have

8.V- + 3.V + 20 = {Ax + fi)(.v + 1) + 5(.v- + 4)

or 8.V- + 3.V + 20 = (A + 5)a- + (A + B)x + (B + 20)

Equating coefficients of like powers in the latter equation, we have

8=A + 5 3=A + fl and 20 = S + 20

Therefore, B = 0. A = 3, and

8.r- + 3j:+ 20 Sjc^ + 3x + 20 3.v 5


+
x^ + X- + 4.r + 4 (x+ l)(x- + 4) .t" + 4 A" + 1

3.X-' + ll.v- 16
EXAMPLE 2
(x- + l)(.r- + 4.V + 13)
518 CHAPTER 8 TECHNIQUES OK INTEGRATION

3jc^ + 1 l.t - 16 A.X +B Cx + D


+
(,2 + ,)(^.2 + 4, + 13) ,2+1 ,2 + 4^+13
Multiplying by (x- + l)(.v- + Ax + 13), we have

3.v'' + 1 l.v - 16 = (A.V + fi)(.v= + 4.V + 13) + (Cx + D)ix- + 1)

or 3.v^ + 1 l.v - 16 = {A + C)x^ + {4A + B + D)x-


+ (13/1 +4B + C)x + (13B + O)
Equating the coefficients, we have

[
= 3 A + C
= 4A+ B + D
11 = 13A + 4B + C
[-16 = 136 + D

Solving these simultaneous equations for A, B, C. and D (say, by elimination), we


obtain A = 1, B = -1, C = 2, and D= -3. Therefore,

3x^ + 1 l.v - 16 ;v- 1 Iv- 3


+
(a- + l)(.v- + 4.V +13) .V- + 1 X- + 4.V + 13

As soon as a rational function has been decomposed into a sum of partial frac-

tions, it can be integrated by integrating each of these partial fractions. We have


already seen in Section 8.5 that there is no problem in integrating the partial frac-

tions that correspond to the linear factors (repeated or not) in the denominator. The
partial fractions corresponding to nonrepeated irreducible quadratic factors in the

denominator have the form

Ax + B
ax + bx + c

The integral

f Av + B
dx
CLX~ + bx +
can be handled by completing the square in the denominator (if necessary), as in
Section 8.3.

In Examples 3 to 5, evaluate the given integral.

EXAMPLE 3
J
f
——+—+
{x
8.v2

\)(x-
3a: -h

+
20
— dx
4)

SOLUTION In Example 1 (page 517), we saw that the partial-fraction decompo-


sition of the integrand is

8.V- + 3.V + 20 3.V 5


-t-

(.V + l)(.v- -I- 4) .V- +4 .V -I- 1

Hence,
r 8x~ + 3x
^
-1-20
dx = 3
f
\
— dxX
+ 5
f
\
dx
J (X l)(x~ +
+ Uix- 4) J .V- + 4 J .V + 1

= i In (.V- 4- 4) -I- 5 in |.v + \\ + C


SECTION 8.6 INTEGRATION BY PARTIAL FRACTIONS— QLADR.\TIC CASE 519

+ -
EXAMPLE 4
J
r
—(x-
^
3.x^

+
;

l)(.r-
1
^
l.r

+ 4x
:
16

+ 13)

SOLUTION Using the partial-fraction decomposition of the integrand already


found in Example 2 (page 517). we have

r 3.v^ + 1 l.t - 16
dx
(X- + \){x- + 4.V+ 13)

1 r 2y - 3
dx+ —^ dx
jr^ + 1 i X- + 4.\- + 13

J
f4^-
+1 f-^+
+X-
.V- J A- 1
f
J A- + 4a + 13
dx

2v + 4 r dx
dx-1
J A-
X- + \
1 J A- + 1 J A- + 4a + 13 J (a + 2)- + 9

= — 1
In (A-
,
+ 1) - tan^' a
,

+ In (a-
.
+ 4a + 13) - — tan"'
7 , /

\
—— A +- 2 \
j + C

where

^
'--'
dx
X- + 4a + 13

has been found by adding and subtracting 7 in the numerator and splitting the
integral into two integrals such that in the first integral the numerator is the deriva-
tive of the denominator. The second integral is evaluated by completing the square
in the denominator and then substituting m = x -I- 2.

r 3x^ -t-lv - 2
EXAMPLE 5 ;— ; " dx
J \x-
a-(a- + 2)

SOLUTION Here we have

3a' + lx-2 A B Cx + D
+ -^ +
A"(.v" -I- 2) .r X' x~ + 2

By the short method of substitution, B = -2/(0 + 2) = - 1 . Putting 6 = - 1 and


clearing fractions, we have

3a-' + 1x-2 = Av(A- -I- 2) - (A- -I- 2) + {Cx + D)x-


= (A + C)x^ + (D - 1)A- -I- 2Av - 2

Equating coefficients, we obtain 3=A + C,0 = D— 1, and 2 = Z4; hence, A =


1, C= 2, and D= 1. Consequently.

r 3a^ -I- 2a - 2 _ [d^_ [ Hr


C dx_ f
f 9v
Iv -I-
-I- I
1

~ dx
J x-(x- + 2) ^ J A J A- J A- + 2

ixdx
Zx dx dx
= In A -I- — 1
-I-
(
\ r
I

+ 2

1,1
—+ . V'2 V2
--— ^
x + C
,

In A -I- In (x- + 2) + tan '

' '

a: 2 2
520 CHAPTER 8 TECHNIQUES OF INTEGRATION

The Case in Which the Denominator Involves


Repeated Irreducible Quadratic Factors
We now consider the case in which the denominator, after being completely fac-
tored, involves repeated irreducible quadratic factors. After such repeated quadratic
factors as well as all linear factors are gathered with the aid of exponents, the
denominator is a product of factors having the form (ax + bf or (or^ + bx + cf,
where A- > 1 . The factors of the form (ax + bt are handled just as in Section 8.5.
The k partial fractions corresponding to (ax^ + bx + cf are those appearing in
the expression

Aix + Bt A2X + B2 A^ + B^
or + bx + c (ar + bx + cf (ax^ + bx + cf

Again, Ai, 61, At, 62- • . ^*. S* must be determined, say, by the method of
equating coefficients.

In Examples 6 and 7, decompose the given rational fraction into partial fractions.

x^ + .V + 2
EXAMPLE 6
x(x^ + 1)-

+ + Bx + C Dx + E
.r^ .X 2
= —A + —^ +
+ \f
x(x^ + If X + .r 1 (X'

Here, the constant A can be found by the short method of substitution as

0-' + + 2
=A or A = 2
(0- + 1)-

In order to find B, C, D, and £, we put A = 2 and multiply both sides of the


equation by .r(.v~ + 1)" to clear fractions, and we obtain

x^ +X + 2 = 2(.x- + 1
)- + (Bx + C)x(x- + 1 ) + (Dx + E)x

= (Ix* + 4.V- + 2) + (Bx* + Cv' + Bx- + Cx) + (Dx^ + Ex)


= (2 + B)x'* + Cv' + (4 + B + D).r + (C + E)x + 2

Equating coefficients of like powers of .v on both sides of the equation above, we


obtain

= 2 + 5
1 =C
= 4 + B + D
\ = C + E

Solving these equations simultaneously, we have B = —2, C = 1, D = —2, and


E= 0. Hence,

x^ +x + 2 2 2v - 1 Iv
SECTION 8.6 INTEGRATION BY PARTIAL FRACTIONS— QUADRATIC CASE 521

Here we find that B = —2 by the short method of substitution. Thus,

- + + - Cx + D Ex + G
.r^ Ir''
^r-^
Ir*
;
.V 2
= A 2
T + ^i + + T —
x\x- +1)- X x^ x' {X- + 1)- \

Clearing fractions, we have


.r^ - Iv'* + Ir* + .Y - 2 = Ax(x^ + D" - 2{.r^ + 1)-

+ (Cx + D)x^{x^ + 1) + (£v + G)x^


= (Av^ + Z4.r' + Ax) - (Zx* + 4a^ + 2)
+ (Cx^ + D.r-* + Cx^ + D.r=) -I- (f.r' + Gx^)

= (A + C)x^ + {D - 2).V* + + C + E)x^


(2A
+ (D + G - A)r- + Ax-2
Equating coefficients, we have
1 =A + C
-2 =D - 2
2 = 2A + C + £
=D + G - 4
1 = A

Simultaneous solution of these equations yields A = 1 , C = 0, D= 0, £= 0. and


G= 4. Hence,
.r' - Iv-* + Zv + .V - 2 1 2 4
.v-(.v- + 1
)-
u- + 1)-

In order to integrate rational functions containing repeated irreducible quadratic


factors or- + bx + c in the denominator, it is necessary to be able to integrate
partial fractions of the form
Ax + B
(ax- + bx + cf
After we complete the square, if necessary, in the expression ar' + bx + c and
make the usual change of variable, the required integral can be brought into the
form
Cm + D r u du du
dii C ;
T + D
(au- + qf J (au
(air + at (au- + qf
The integral
u du

{au- + qr
can be handled with ease by the substitution r = au- + q (Problem 40). However,
the integral
f du
J (au- + qf
can present more of a challenge. Since the original polynomial air + bx + c was
irreducible, it can be s hown (Problem 41) that q/a > 0. Thus, we can make the
substitution u = \qla ir and obtain

du \ q/a dw Jqla_ dw
(au- + qf (qw- + qf (VV-+ 1)
522 CIIAP-IKR 8 TECHNIQl'ES OF INTEGRATION

The required integral can be found provided that we can evaluate an integral of the
form

r dw
(w^ + \f

The trigonometric substitution w = tan d converts the last integral into the form
/ cos" 6 dd, where n = 2{k - 1 (Problem 42); by ) this or other means (Problems 43
and 44), it can always be evaluated.

r .v^ + .-2
+ 2
.v
— dx.
EXAMPLE 8 Evaluate ^
J x{x^
xix^ + 1)-

SOLUTION Here we must make a decomposition of the integrand into partial


fractions. By Example 6,

x^ + x + 2 _ 2 Ix - 1 Ix

jc(a^+ 1)- " -v .r + 1 (A- + \f


Hence,

~ +
J .V J .v^ 1 J .r= + 1 J U- + I)'

= 2 In l.vl
- In (.r- + 1) + tan"' .v + — + C
XT + 1

where the second and fourth integrals have been evaluated by using the substitution
r = .V- + 1

f r"
- Iv"* + Ix^ + .V - 2
EXAMPLE 9 Evaluate ;-^ ^ dx.
J A-(.V- + 1 )-

SOLUTION By Example 7,

.r' - 2v^ + 2.v' + .V - 2 1 2


)-
+
x\x- + 1 X x^ (X- + 1
)-

Hence,

r A-**
- 2x^ + 2jr-' + .V - 2 r dx { dx { dx
J
r-^
x^{x- +1)-
^ dx=
J
\

X
2
J
—r + 4
X- J
—^
ix- + 1)-

= In Lv
1,2
+ —+ 4
r tan ' X
+ ;
X ] ^
+ C
' '

.V L 2 2(.r + 1) J

where we have used the trigonometric substitution x = tan d to evaluate

/ dx/{x^ + )-. Thus, dx = sec- dO and


1 ti

r dx r sec- Ode r sec^ edd _ r de


J {X- + ])-
~ J (tan- e + \)- ~ i (sec- 0)- J sec- d

20 sin d cos
=
r
cos-
,

dde = —+ sin
= —+ -I- C
J 2 4 2 2
SECTION 8.6 INTEGRATION BY PARTIAL FRACTIONS— QUADRATIC CASE 523

Using an app ropriate right triangle, we have = tan ' .v. sin 6 = .v/V.ir + 1. and
cos 6 = 1/Vjr + 1, so that

+ C

2(-v^+ 1)

Problem Set 8.6

In Problems 1 to 26. evaluate each integral. (Caution: Some In Problems 27 to 32, evaluate each definite integral.
integrands may require a preliminary long division, and others may
(' + dx
be partial fractions to begin with.) f 10 f
^'
i „+l)(r+l)^' 'HoiTTiy
r x^ + Q-r* + 1
dx
'-^^ Adx
It, )ix- + 4) J x^ + 9x 29 f" (2c+
30 r + Ax
Jo l)(4.x:=+ 1) J, r'
r u+ 3)dx
3)

J x(jr + 1) J / - 16 31 ['-^^^dx
x(x- +
h 1 )

3a^ + X - 2
I (X - \)(x^ + 1)
dx
+.V +
ciT P .r-* - .X-' + 2v^ - .V + 2
J .x(.r 1) 32 oLx-
I (X - \)(r- + 2)
r 4xd.x
J .v^ - 1 12v + 19 In Problems 33 to 36, find a change of variable that reduces each
intesrand to a rational function.
{ 2r-t+l r H" — M 21
9
J +
*
,
10
J - + 8m - 4
-
£/«
Kt' 25) 2m-' M-
X dx
r
^7
2x^ + 9
33
J
f_ sin- .X- +
£2ii
sin" .x 9 sin .V + 9
12 ^ ^ dx
J x(;(^ + 4)- J x^ + .r + 12x-

. , r .r' + 2r^ + 7.x + 2


34
J
\-^+ +
j3 J^5>^-4y+i:
- -
1'* ^ ; ; d-x
.xV.v .t 1

3/ /+ 12v 4 - i .X- + 2r + 5
''"-'y-'^-'-

J {X - 2)(1\- - 3.V + D)
16
Jf (v-2)(r +
^ 4y + 5)
35
J
{
f'' - 1
dx 36
J f^^^;±^
(e-"^+ 1)-
ir

I' Iv" - 7.x~


7.x- + 31.V-
31.x- -^
-45.V + 46 37 Find the partial-fraction decomposition of
dx
J 2x-' - 7.x- + 1 1.x-
ax' + bxr + c.v + d
(x'+ 1)-
18 ,-^^-7^ .V
Jf + + + 4
.X-' 4.x- 6.V + fex-^ + c.x + (i

It-
38 Fmd a tormula tor
f o.x-''
^
(-r +
n
1)
ax.
,

1 dx
20
J f-* + 9r 39 Integrate

21 f-,^^^iv
+1)-
22
Jf /, + y+ . dv dx

— +
J ;t2(j:2 v' 12x- '
J +
.x^ 2x^ .1

^4^?— two ways: = + + and


23
Jf
^^r^y"^"-
+ +
^.r^ 4.x^ 4.V
24 f
J (.r - l)-(x^ -.X
in

(b) by using
(a) by usingg the substitution
partial fractions.
tions.
sub u .r^ 2v^ .v

25
4(;+ 1) A f dLv { u du
I" 40 Find a formula for ; -r
J /(r + 2/ + 2)- J .X-' + 3.x-^ + 7.x- + 5 J [au- + qf
524 CHAPTER 8 TF-CHNigiES OF INTECRATION

41 SupfKise that ar^ + bx + c is irreducible, so that b' - 4ac < 0. 45 Use the formula in Problem 43 to evaluate
Complete the square, and make the appropriate change of vari- dw f dw
f
able so that a.r + bx + c = air + q. Then prove that q/a > 0.

42 Prove that the trigonometric substitution iv = tan fl converts the


46 It is an algebraic fact that every rational function can be decom-
integral
f d^, posed into partial fractions. Show, therefore, that the integral of

J (H'^ + n* any rational function can be expressed in terms of rational func-


tions, inverse tangents, and logarithms (of absolute values).
to the form / cos" dO. where n = 2(A: - 1).
47 An integral of the form
43 By differentiation of the right side, verify that, for A & 2,
f dx
f dw _ I w 2A- - 3 f dw
rfi

(a- bxfHc -
+ D* ~ 2k- 2'
"*" J x)
J (H'- (vv- + 1)*"' 2A- - 2 J (vv- + D*
where a. h. and c are positive constants, has to be evaluated
44 Show that to determine the lime required for a homogeneous sphere of

J + D* iron to dissolve in an acid bath. Make the change of variable


(x^
Va - bx = z, and show that the integrand then becomes a
where k is a positive integer, can always be expressed in terms of
rational function of z.
rational functions and the inverse tangent function. {Hint: Use
Problem 43.) 48 Find a formula for the integral in Problem 47.

8.7 Integration by Special Substitutions

In this section we examine some special substitutions that can be effective when the
integrand contains /ith roots of the variable of integration or when it involves ra-
tional functions of sines and cosines.

Integration of Functions Containing nth Roots

If the integrand involves an expression of the form Vx. then the substitution z =
"Vx may be helpful. If the integrand involves both vx and vx. then the substitution
u = ^\/x, where p = nm, may prove effective. In the first case, we have x = z", so
that dx = nz"~^ dz; in the second case, we have x = u^, so that dx = pu''~^ du.

In Examples I to 3. evaluate the given integral.

dx
EXAM PLE 1
(

J 1 + V.v

SOLUTION Put r = Va-. so that .v = r", dx = 2r d:. and

f ^v r 2:d:

Since 2z/(l + z) is an improper fraction, we divide numerator by denominator to


obtain a quotient of 2 and a remainder of -2. Thus,

2z
= 2
1 +z 1 + z
SECTION 8.7 INTEGRATION BY SPECIAL SUBSTITUTIONS 525

It follows that

dz

/t7^=/t^^/('-^)-^/"=-/ 1 + z

= 2z - 2 In |1 + r| + C = iVx - 2 In |l + V^| + C

dx
EXAMPLE 2
/ Vi + v^
SOLUTION We could put ; = V^, but, with some foresight, it seems better to

try z = Vl + \/x. Then

z- = I +\/x z- - 1 = \6c and so x = (z^ - l)^

In particular, we have
dx = 3(z- - l)-(2z dz) = 6z(z- - if dz

r / r^ 2z'
6(y -^- + zj+C
. ,
6 (z-* - 2z2 + 1) dz =
j

dt
EXAMPLE 3
I Vi-\^
SOLUTION Put M = \/7, so that / = M*' and dt = 6u^ du. Thus,

r rfr r 6m-' jk _ r 6m-' du ^ r »-'</;<

J
v7- v? " ' V^- Vi? ~ J «'-«'" Jm-1
Since m7(" ^ 1) is an improper fraction, we divide numerator by denominator to
obtain the quotient if + u + \ and the remainder 1. Hence,

u' ,
1

u- + u + I +
u - I u - I

so that
{
— 7=
dt
—=6 r /
I

^
M-
.
+ j< +
.

1
+
-
1

Jdu
•^
Vr - V7 ^ "

= 6(—
V 3
+ —+
2
M + In Iw
'7 - 111 -f C

= 2VT+ 3V7+ (i\ft+ 6 In |>7r- 1| + C

Naturally, if the integrand involves an expression of the form vw, where ii is a


function of .v, then the substitution z = V« suggests itself. Such a substitution was
effective in Example 2 above. Further examples are given below.

In Examples 4 and 5, evaluate the given integral.

f" JcVx
/
-
EXAMPLE 4 I 3 dx
526 CHAPTER 8 TECHNIQUES OF INTEGRATION

SOLUTION Put z = Vjr - 3, so that z^ = x - 3, x = z^ + 3, dx = 2z dz,z= \

when ;c = 4, and z = 3 when x = \2. Thus,

jcV;t- 3 dt = (z- + 3)z(2z dz) = 2 (z" + 3z^) c/z


I I
•'4 •'1 -^1

13
- ;2 5
+ I T
2Z
3» _
- 756
5 - 12 _
- 744
(5 Z ) 5 5

EXAMPLE 5
f 1 + ^' ^
(ix
TT^
J (3 + xY
;c)"^^

SOLUTION Put z = (3 + .v)'/\ SO that z-^ = 3 + x, x = z^ - 3,dx = 3r dz, and

dx = /I±<^(3z^.z) = 3/(z-6z^ + lOz) rfz


/ (3 + x)^'^

3z'* 18z5
+ 15z' + C

= f (3 + xf^ - ¥ (3 + .v)5/3 + 15(3 + xf^^ +C

Integration of Rational Functions


of Sine and Cosine

It turns out that the substitution z = tan (.v/2) reduces any integrand that is a ra-
tional function o/sin x and cos x to a rational function ofz. The appropriate formu-
las are contained in the following theorem.

THEOREM 1 Tangent-Half-Angle Substitution

Suppose that z = tan (.v/2). Then

1
- z~ 2dz
(1) cos .V
= (il) sin .V (iii) dx
1 + z' 1 + z-

Let z = tan (x/l). Then, using the double-angle formula, we have

2 2
cos X = 2 cos^ 1
= 1
= - 1
sec- (.v/2) 1 + tan' (.v/2)

1 -I- z- 1 -I- z~

so that (i) holds. Also,

sin .V = 2 sin — cos — = 2 sin (.v/2)


— cos —Xy =
-,

s" tan —X 12 cos- —x\


1
I

\ 11
•,

2 2 cos (.v/2) 2

= tan — (cos .v
-(- =
1

T+1 =
2z

2
I ) _
M -^ z- 1 + z-

so that (ii) holds. To prove (iii), note that z = tan (-v/2) implies that

1 + z-
dz = l^sec-
yJY = y(l + tan-
'=f)y I tiv dx

Hence, dx = 2 dz/(\ + z"), as desired.


SECTION 8.7 INTEGRATION BY SPECIAL SUBSTITUTIONS 527

The following examples illustrate the use of the tangent-half-angle substitution.

In Examples 6 and 7, use the tangent-half-angle substitution to evaluate the given


integral.

dx
EXAMPLE 6
1 — cos X

SOLUTION Put r = tan (x/2). so that

1 - z- Idz
dx =
1 + z- 1 + z-

Idz
{ dx r 1+2- { dz^
and ~
J 1 - cos j: J . i\ ^jr\ ~ J r^

-l-C + C

= -cot —+C
d^Y
EXAMPLE 7
sin X — cos X + 1

SOLUTION Put z = tan (.v/2). so that

Idz
dx
1 + 1 + 1 -I- z^

2dz

and
Jf sin X — cos .V + 1
=
Jf
— 2z
1 + z-
h +
+ 1
1 + 1 + z-

= /( dz = In |-| - In Iz + ll -I- C
z + 1

z tan (.v/2)
= In + C = In / + C
z -I- 1 I I tan (X 2) + 1 I

Problem Set 8.7

In Problems 1 to 6. use an appropriate substitution of the form In Problems 7 to 12. use an appropriate substitution of the form
z = Vx and evaluate each integral. z = "s/u. where « is a function of x, and evaluate each integral.

r dx
7 x^^Zx^ - 1 dx 8 .rV5 - 2r dLv
J 1
- v;^ J J

dx d[v

1 - ^ 9
J
.v^'lr'4- 1 dx 10
J
jc'vTTz? dLr

f xdx
+ xY'^ dx
J I + \^
11 I .v^(4.v+ if'-dx
-I x{l
528 CHAPTER 8 TECHNIQIES OF INTEGRATION

Problems 13 to 18. use the tangent-half-angle substitution to eval- eix


In
35 36
uate each integral. 1 + sin X + cos X J sm f + cos t

sec r <fr r du
dx 37 38
14 -
"It, 5 sin X f 1 + cos I

4
I H- sin r J 2 CSC M sin


15 f
^5i^ 16 \
dx
:
39 .vV.v I dx 40
\:-x^lx + -idx
-'
sin X cos .V -t-
J sin jf -t- Vj cos x
' 4-\Zx I
- V^
du 41 dx 42
-
18 I"
— I +.r '4 1 + V^
tan i\n 6 { I cos u)

43
x' dx
44
r" I
- V37n
In Problems 19 and 20, use the substitution x = l/i, dx = -dt/r to
_ , V 1
- X ', I -I- V37T2
simplify each integrand; then evaluate the integral.

^
cos .r
45 dx 46
2 + sin X
19
f
20
J .vVl+7 dx
47 48 r ^^
„/3 CSC .r - cot X -I,
.xV3.r - 2x - 1
In Problems 21 to 48. evaluate each integral.

=
21 [1^.,
+
22
49 If r tanh (.r/2). show that
J I \'i 1+2^ 2z
(a) cosh X = T (b) sinh x
I
-
24
J \ X + \'x J 1 + vTTl (c) dx
2 t/:

I - r
r .r (iv
26
J VI - r J (2 - 3.r)^/^
In Problems 50 to 52. use the substitution suggested by Problem 49
27 _ ^/4
28 ^'Vl - ^'dtt to evaluate each integral.
J ^1/2 /
dx
29 f-'Vl + e- dx 50

J 1
- sinh .r J cosh x sinh x

tanh .r
30 sin .V cos .vV I + sin .v dx 52 ^.v
J I + cosh .r

32
f^^^/^^.
J (I +.t)- V I -I-a:
53 Find the area of the region bounded by the curves y =
5.v/(l -I- V\v). y = 0. and.v = 9.
33
J
f-^^^
Vw + yi 54 Find the volume generated by revolving the region bounded by
the curves given by y = .r + Vx+1 and y = between .v =
and .V = 8 about the x axis.

8.8 Tables of Integrals and Reduction Formulas


When complicated integrals arise in practical scientific work, people often refer to
one inside the covers of this book. The use of com-
tables of integrals such as the
puters with symbol-manipulating capabilities to evaluate indefinite integrals is also
becoming more widespread. Before you use either a table or a computer to evaluate
an indefinite integral, it may be necessary or desirable to simplify the integrand by
means of the techniques of integration presented in Sections 8.1 through 8.7.
SECTION 8.8 TABLES OF INTEGRALS AND REDUCTION FORMULAS 529

The following example illustrates the use of a table of integrals.

EXAMPLE 1 Use the table of integrals inside the covers of this book to evaluate

SOLUTION The expression V7 + 3a-, which appears in the integrand, suggests


that we try the part of the table labeled '"Forms Involving Va- + «-.""
The left side
of Formula 66,

r Va- +
;
ir
du = Va- + ir
+ In
,

III + Va' +
^ M'l
-'
ir u

is very similar to the given integral. (Notice that to save space, constants of integra-
tion are omitted in the table of integrals.) In order to bring the given integral into a
form that matches Formula 66, we let

a = V? and u = \^ x
so that

u^ = 3jr Va- + ir = VTTl? du = V3 dx


and
VtTI? r
f Va- + u^ du 3 r
f Va^
Va- + u-
x^
^
'^^"
"" =
J
rz ^ = —7^
V3 ~ 'Vf
; du
M-/3 J u^

= V3l + In « + Va- + M- + C
^ « /

= V3( rS~^ + In 1^^ + V7 + 3.r=|) + C

= VTTi? ^ ^ + V? +
^ + V3 In
,

I
V3.r
,^

3.r^| + C

Notice that we have restored the constant of integration that was omitted from the
table.

Reduction Formulas
Several formulas in the table of integrals, for instance,

sec u du = sec" - u tan u + sec""- u du


> n - I n - I J

reduce the problem of integrating an expression involving a power to the problem of


integrating a similar expression involving a smaller power. Such a reduction for-
mula may have to be used repeatedly to obtain the final answer. Most reduction
formulas are derived by using integration by parts (see Problems 31, 32, and 34).

EXAMPLE 2 Evaluate j
sec"^ Iv dx.

SOLUTION We begin by making the substitution u = Ix, so that du = 2 dx,


dx = i du, and

sec^ Ix dx = (sec^ u)(h du) = i sec"^ u du


J J J
530 CHAPTER 8 TECHNIQUES OF INTEGRATION

Now we can apply the reduction formula for powers of the secant [Formula (33)]
with « = 5 to obtain

I sec^ 2x dx = — sec^ u dit

=
1/1
—2^5-1 sec
. .
- u tan u H
5 - 2 r
sec-
, ,
- u du]
, \ ^
+ C
5 - 1 J /

= 8 sec^ « tan M + 8 sec"* u du + C


Again, we can apply the same reduction formula, this time with n = 3, to obtain

sec' Zv dx

= — sec"* tt tan H H
8^3-1
sec' - ;/ tan u H
- J
sec"* ' u du]
/
+ C
8 3 1

= 8 sec^ M tan // + fs sec u tan « + to sec u du + C


Finally, using the standard formula for the integral of the secant, we have

sec' 2x dx = s sec^ u tan m + u? sec u tan w + u; In |sec u + tan u\ + C


/
= I scc^ 2.V tan Za: + t^ sec Ix tan Zv + t% In |sec Zv + tan Zv| + C

Nonelementary Integrals
There are continuous functions whose indefinite integrals cannot be expressed in

terms of a finite number of the elementary functions considered in this textbook.*


Examples are

\ —dx e~'' dx -^^^ dx


J
sin .v- d.x and I WT^ dx

Such nonelementary integrals arise in many branches of applied mathematics; for


instance, they are employed in the study of the path of a ray of light through the
atmosphere, the cooling of the earth, the behavior of a pendulum with a large
amplitude, the kinetic theory of gases, and the theory of probability. In Chapter 1 1

we give a brief indication of how nonelementary integrals are handled (see, for

instance. Example 8 on page 698).

*For more details, see D. G. Mead, "Integration," American Mathemutkal Monthly,


Vol. 68, No. 2, pp. 152-156, February 1961.

Problem Set 8.8

In Problems I to 22, use the tabic of integrals to evaluate each


f
C
dv
3 rfv f VTl
V2 + l.-c
dx
ntcgral. - J Vu + 5y^ J .v

dw
1
[ u du
5-
^
2
f
— 5xdx
-
r-^ ,
7
f 1,
/-V13 +
rr^ tt^
8/- dt
, „
8
f
—r—, ,
J (3 + 5uf J (2 3x)' J J wWS - 2w^

V3 +
'/t^ 4|.,.,^,-«. ,j^., ,./^
5z ,

dz
REVIEW PROBLEM SET 531

-
f V2 -
Vl- 3.r 33 Use the reduction formula of Problem 31 to evaluate
11 i-\'r 5 dt d\
(a) J sin" ax dx. (b) / sin' ax dx. (c) / sin* ax dx.

34 Derive the reduction formula for powers of the secant function


13 A-V"!5 - 7a- d:v du
J [Formula (33) in the table inside the front cover].

dt
In Problems 35 to 40. suppose that h = fa'' ^in* x dxfor k = 1,2,
2 + 3f + 4f- J V2T3JC + 4? 3,4, .... and let n denote a positive integer.

f (iv .„ f Vl6f2-5f + 7
17 ,
18 a 35 (a) Show that /, = 1 , A= 7r/4, /, = |, and h= 377-/16. (b) Use
J x\5 - 4.Y + 2r J I
Problem 31 to show that if n > 2, then /„ = - \)/n] /„_2.
[(«

19 sin^' (3v + 2) ^v 20 tan"' (2r + \) di 36 If /t > 1 , show that


J J
•3-5-7 •••(2A: - 1) n
- (a) hk = 2-4-6-8
21 X cos"' 5.V aLr 22 ir sin"' (3iv 1) dw •••(2A-) 2

2 •
4 6 8 • • • (2k)
In Problems 23 to 30. use a reduction formula to evaluate each (b) /.;-+,
3-5-7-9- •• (2A- + 1)
integral
37 Use Problem 36 to show that rr/2 = (Ik + Dlik+i'lik-
f dx
24 38 Use Problem 36 to show that Tr/2 = Ikhk-i-hk-
J (A- + 1
)- J (.r + !)•'

39 Show that if 1 s A- < n, then /„ < h- [Hint: For < .v < 7r/2.
dv f ^y s < s (sin
26 sin A 1. so that (sin .x)" .v)*.]

VS + 2v J 0'- + 2y + 2)''

40 Use Problems 37, 38, and 39 to show that

27 esc' 3a" cir 28 cot' (2v - I) d\ 1 TT 1 77


J
2*: + 1
'
IJ^ ~ -''~1I'U^
29 tan' 7a dx 30 sec^ (r/2) rff

41 Using Problem 40 and part (a) of Problem 36, show that


31 Using integration by parts with u = sin" '
ax and
dv = sin ax dx, derive the reduction formula 2 ^ r l-3-5-7---(2t- 1) 1-
^ J_
7r(2<.-t- 1)
~ l 2-4-6-8---(2/t) J ~TTk
sm ax cos ar
sin" ax dx sm ax dx
prove Wallis's formula
n J 42 Use Problem 41 to for ir/A. namely,

2-4-4-6-6-8-8---
32 Derive the reduction formula for powers of the tangent function ^_" {2k) {2k)

4 ;t-l.'?x 3 3 5 •
5 •
7 •
7 •
9 • • •
(2A- - \){2k + 1)
[Formula (31) in the table inside the front cover].

Review Problem Set, Chapter 8


In Problems 1 to 90. evaluate each integral. 10 (4 + COS .r)(3 - cos .r) dx

1 cos' 2v dx 2 sin' 4a COS" 4a d^v 11 (sin A — cos a)- dx

3 sin' 3a cos' 3a dx 4 Vcos A sin' A dx 12 sin- ( 1


- 2v) cos- ( 1
- 2v) ^
5 sin' Iv) dx 6 sin' ^- cos''- -— dx 6a cos- 6a dx 14 4a cos" 4a dx
( 13 sin- sin''
J 2 2 J

7 sin"-' 5a cos 5a dx 8
r
sm
J

2.V
COS"
,
—ix
dx Isl^^iMld,
J \^n (3f/2)
16f^^^v
a J sin-*

9 sin- (2 - 3.x) dx 17 sin 8a sin 3a dx 18 cos 13a cos Zx dx


J
532 CHAPTER 8 TECHNIQUES OF INTEGRATION

sin X sin Iv sin 3.v dx 20 COS 3a cos 5a cos 9a dx 60 a'" cos x" dx

tan'' (li - 1) iLx 22 cof* (2 - 3a) dx


J

X tan' 5a' dx 24 r cot' (5 - .r') dx

(sec t - tan I)- dt


/
dx
28 Vl +cosA«iv
(1 — sin a)" J

sec'* (1 + 2r) dx 30 I CSC"* (3 - 2v) dx

tan' (2 + 3.V) sec'' (2 + 3jt) ^v

cot' ( 1
- X) CSC"* ( 1
- A) dx

dx
V.r + 64

dx
REVIEW PROBLEM SET 533

fT/g 126 If
99 |tan^ lv| dx 100 cosh'' A- dx
J-t/S
/(.v) for X > 1

101
rI
'
f' dx 102
1 VI - Q/-* show that 2 In .v s/(.v) s In x.

Vr - 25
104 x'\/a~ — jr dx
\11 Show that
103
r
I

-'o

- f-^^^^i^i^
105
rJ
I
1 + cos d.x ,06
J„/4 1 - COS .V
128 If

107
h
r^ U - 2)-
dt
f- S.r - 3.V + 18

^«=f/(-|)(f)
+ + show that g(\/x) = —g{x).
;c^ 3.V 1
dx nof'^^
+ Jo (r 1)^
129 Derive the reduction formula
ri/2
112 (Zx-rf'^dx A^'dn A)" dx
1/2 .vVS.r + 4.r - 1 ()
I

a" ""(In a)"


r m9^ -I
113
\ x+\^
dx
114
Ji
f-*
—V^+
=
+
V.v(.v
1

1
dv m+ \ m++ 1
I
,

J
y"(ln a)"^' dx

130 Use graphical considerations to prove that


P t_di dx
115 116
k U--
(t 1)
J ., Vi + Vi +x
'0
sin"" ! dt = I COS"" t dt
Ai
P" dt
= - =
117 118 ^= 131 Find the area under the curve y Ia"^ 2 In a from a 1 to
-'1/4 V;+ 1 - V7 Aft v-2V^-3 A = 4.

119
dtc 132 Find the area under the curve v = x^e~' between a = and
120
Jo Ve"-' + 2e"-' 1 - sin .r + 2 cos X

133 Find the area bounded by one arch of y = sin a from a = to


121 =
4/4 sin ,v + tan x Jo 4 sin f + 3 cos t A 77.

134 Find the volume of the solid generated by revolving the region
123 Determine constants A and B so tiiat
=a = = 4
bounded by the curves y In a, y 0, and a about the

c sin + rf cos e
=A + " e cos e
„ - / sin —d X axis.

e sin e +/COS e '" ~ e sin +/cos 135 Find the volume of the solid generated by revolving the region

Then obtain a formula for the integral bounded by one arch of y = sin x and the line y = about the
line y = —2.
f c sin 61 + d cos ^
de 136 If the velocity v in meters per second of a particle that is
J e sin e + /cos &
moving along a straight line is expressed by the formula
124 Show that V = (t + 3)/(r' + t), find the distance i in meters that the
particle traveled from t = 1 second to / = 3 seconds.
for < < f 1
'1 - r 137 Evaluate

Then show that


rV'"(f)^^'f^='" (cos a).

<A<
4 vn^ < sin jc for
138 Find the arc length of the curve y
A= V3.
= In a from a = 1 to

125 Evaluate the integral


139 Find the arc length of the curve y = In (esc a) from a = 77/6 to
f A; A = 77/2.
V.r - 2v + 5
140 Find the surface area generated by revolving the arc of the
by using the substitution Vat - 2a + 5 + x = z. curve V = e" from a = to a = 1 about the a axis.
POLAR COORDINATES AND
ANALYTIC GEOMETRY

I
Until now we have specified the position of points in the plane by using Cartesian
coordinates; however, in some situations it is more natural to use a different coordi-
nate system. In this chapter, we study the polar coordinate system, the conversion
from Cartesian to polar coordinates and vice versa, the graphs of polar equations,
and area and arc length in polar coordinates. We also study the conies — the ellipse,

the parabola, and the hyperbola — in both Cartesian and polar coordinates, and we
use the idea of rotation and translation of coordinates to simplify their equations.

9.1 Polar Coordinates

In order to establish a polar coordinate system in the plane, we choose a fixed


point O called the pole and a fixed ray (or half-line) with endpoint O called the

Figure 1 polar axis (Figure 1). An angle in the standard position is understood to have its

vertex at the pole O and to have the polar axis as its initial side.
pole
• Now let P be any point in th£ plane and denote by r the distance from the pole O
O polar axis
; = UP ¥= O. then P on a uniquely
to the point P. so that \0P\ (Figure 2). lies

determined ray with endpoint O, and this ray forms the terminal side of an angle in

the standard position. We denote this angle, measured in degrees or radians (which-
ever is preferred), by d. and we refer to the ordered pair (r. 0) as the polar
coordinates of the point P. (As usual, positive angles are measured counterclock-
Figure wise.) Thus, in polar coordinates

P = (r, 6)

The polar coordinates {r. 0) locate the point P with respect to a "grid" formed by
concentric circles with center O and rays emanating from O (Figure 3). The value of
polar axis
r locates P on a circle of radius r. the value of 6 locates P on a ray which is the
terminal side of the angle in standard position, and P itself lies at the intersection

of the circle and the ray. For instance, the point with polar coordinates (r, 6) =
(4, 240°) is shown in Figure 3.
534
SECTION 9.1 POLAR COORDINATES 535

Figure 3
i:o°
536 POLAR COORDINATES AND ANALYTIC GEOMETRY

EXAMIM.K 2

(a) Plot the point P = (3, Tr/b) in the polar coordinate system.

Give three other polar representations of the point P for which

(b) r < and << 27r. (c) r > and -Itt < 6 < 0,

(d) r<0 and -2tt < < 0.


SOLUTION

(a) The point (3, 7r/6) is 3 units from the pole and lies on the ray that is the

terminal side of the angle 30° = ir/b radian in the standard position
(Figure 6a).

The same point P can also be represented by


777^
(b) ") - (-3. 6
(Figure 6b)

llTT
(c) 277 = 3. (Figure 6c)
6
Itt 5 IT'
(d) -) = (- 3, - (Figure 6d)

Figure 6
/'=(-3,-f)

-"Yi
polar axis

(a)

Conversion of Coordinates
At times it may be advantageous to convert from a Cartesian representation to a
polar representation, or vice versa. When we make such a conversion, it is impor-
tant to realize that the geometric points in the plane do not change — only the

method by which they are assigned numerical "addresses" changes. "

The usual arrangement is to take the pole for the polar coordinate system at the
origin of the Cartesian coordinate system and the polar axis along the positive x
axis, so that the positive v axis is the polar ray = tt/2. If we consider the point P
whose polar coordinates are (r, 0) with r s 0, it is clear that the Cartesian coordi-

Figure 7 nates (.V, y) of P are given by .v = r cos 6 and v = r sin 6 (Figure 7). This is cer-

tainly true if r = 0, while if ;• > 0, it follows from cos 6 = x/r and sin 6 = y/r.

Now, suppose that a point P has polar coordinates (r, d) with r < and we
{(x, j')Cartesian = + and
desire to find the Cartesian coordinates off. Since
(.v, y) (r, d) (-r, 6 tt)
9) polar
—r > 0, it follows from the equations developed above that

X = —r cos {6 + n) -/(cos 6 cos TT - sin sin it) = -r(-cos 6) = r cos

y = -r sin [0 + tt) = -r(sin 6 cos tt -I- cos sin tt) = — r( — sin 6) = r sin 6

Therefore, the equations

r cos H and ;• sin


SECTION 9.1 POLAR COORDINATES 537

work in all possible cases to convert from the polar coordinates (r, 6) of a point P
to the Cartesian coordinates (.v, y) of P.
From the last equations, we have
_x- + y- = r- cos- d + r- sin- 6 = r-(cos- 6 + sin^ 6) = r-

so that

:Va-- + V-

Also, if X 7^ 0, we have
V r sin 6 sin d
= tan
X r cos d cos d

so that

tan = ^ for v 5^

The equations above do not determine r and B uniquely because the point P whose
Cartesian coordinates are (.v, y) has an unlimited number of different representations
in the polar coordinate system. In finding polar coordinates of P, you must pay
attention to the quadrant in which P lies, since this will help to determine 6.

EXAMPLE 3 Convert the given polar coordinates to Cartesian coordinates:

(a) (4, 30°) (b) (-2, 5V6) (c) (-4. -Itt/Z)

SOLUTION

(a) U", y) = (4 cos 30°. 4 sin 30°) = (4 •


V3/2 4 ,

— ) = (2 V3, 2)

(b) (X, y) -2 cos ——, -2 sm


6 6 /

= (V3, -1)
2 /' -2-f j

(c) (.V, y) = -4 cos - 3-;. -4sm^-


-4 sin —277

i\ / Vl\
= (-(-1).- ^ /
(2, 2V3)

EXAMPLE 4 Convert the given Cartesian coordinates to polar coordinates with


r > and - 77 < < tt:

(a) (2, 2) (b) (5, -5/V3) (c) (0, -7) (d) (-3, 3)

SOLUTION

(a) r = V2- + 1- = VS = 2\/2, and tan = f = 1 . Since the point lies in


the first quadrant, it follows that < d < tt/2; hence, 0= 77/4. The polar
coordinates are (2V'2. 77/4).

25 -1
= (-5/V^)
, 10
(b) r = J25+ and 9
'"" tan
'"" " Here the
V3 5 V3
point lies in the fourth quadrant, so that -77/2 < 6* < 0; hence, = -77/6.
The polar coordinates are (lO/Vs, ~ir/6).
538 POLAR COORDINATES AND ANALYTIC GEOMETRY

(c) r 49 = 7. Since =
= VO + and y < 0, the point lies on the negative v
.r

axis; hence, = -v/l. The polar coordinates are (7, ~it/2).


6

(d) r = V9T9 = 3\^. and tan 6 = 3/(-3) = - 1. Since the point lies in
the second quadrant, follows that tt/2 < < tt; hence, 6 = in/A. The polar
it

coordinates are (3\/2, 37r/4).

Graphs of Polar Equations


A polar equation is an equation relating the polar coordinates r and 6. such as
r = 0- or r- = 9 cos 26. Because a single point P in the plane has a multitude of
different polar representations, it is necessary to define the graph of a polar equation
with some care.

DEFINITION Graph of a Polar Equation

The graph of a polar equation consists of all the points P in the plane that have
at least one pair of polar coordinates (r. 0) satisfying the equation.

Thus, if none of the pairs of polar coordinates that represent P satisfy the polar

equation, then P does not belong to the graph of this equation. However, in order
for P to belong to this graph, it is not necessary for all its polar representations to
satisfy the equation — any one will do.

EXAMPLE 5 Sketch the graph of the polar equations

(a) r = 4 (b) r- =16 (c) 6 = ^ (d) e= -^ 5lT

SOLUTION

(a) The graph of r = 4 is a circle of radius 4 with center at the pole (Figure 8).
Figure 8
Notice, for instance, that the point P = (4, -tt) belongs to the graph in spite of
the fact that not all its representations, such as ( -4. 0) or -4. (
277), satisfy the

equation r = 4.

(b) The equation r- = 16 is equivalent to \r\ = 4. and its graph is the same as

the graph of r = 4 (Figure 8).

(c) The graph of 6 = 7r/6 consists of the entire line through O making an
angle of it/6 radian (30°) with the polar axis — not just the ray. as one might
think at first (Figure 9). A point P = (r,77/6 + tt) belongs to the graph of
6 = tt/6 because it can be rewritten as P = — r, 77/6).(

(d) 6 = -577/6 has the same graph as = tt/6 (Figure 9).

Sometimes it is easy to find the shape of a polar graph by converting the polar
equation to a Cartesian equation by means of the equations x = r cos and
y = and then by sketching the Cartesian graph as usual. Conversely, any
r sin
Cartesian equation can be converted to a corresponding polar equation simply by
substituting r cos for .v and r sin for y.

EXAMPLE 6 Find a Cartesian equation corresponding to each polar equation and


sketch the graph.

(a) r = 4 cos (b) r sin = 2 (c) r = 4 tan sec 6


SECTION 9.1 POLAR COORDINATES 539

SOLUTION

(a) We multiply both sides of the equation r = 4 cos by r to obtain r" =


4/- COS 6, or x^ + y~ = 4a:; that is, a" - 4.x: + v" = 0. By adding 4 to both

sides of the last equation, we complete the square for the expression x~ — 4x
and obtain

x- - 4x + 4 + y- =4 or (x - 29 + v^ = 4

Thus, the graph is a circle of radius 2 with center at the point with Cartesian
coordinates (2, 0) (Figure 10a).

(b) The equation r sin = 2 is equivalent to v = 2, and its graph is therefore a


horizontal line (Figure 10b).

Figure 10
540 POLAR COORDINATES AND ANALYTIC GEOMETRY

r = that really does belong. Each case should be checked! (See Problems 49 to
51.)
The methods used in Examples 6a and 7 can be generalized to show that if a is

a positive constant, the graph of a polar equation of either of the two forms

r = ±Ia cos

is a circle of radius a with center at the Cartesian point (±a. 0), respectively; and
the graph of either of the two equations

-.2a sin

is a circle of radius a with center at the Cartesian point (0. ±u). respectively,
(Problem 53).

Problem Set 9.1

In Problems I to 6. plot each point in the polar coordinate system, In Problems 23 to 30. sketch the graph of each polar equation.
and then give three other polar representations of the same point for
which (a) r < and s e < 2it. (b) r > and -lir < e < 0. and 23
(c) r<Oand -27r< S < 0.

1 (3. 77/4) 2 (6, 27r/3)

3 (-2, ir/6) 4 (3. 150°)

5 (4. 180°) 6 (4. 57r/4)

In Problems 7 to 12, convert the given polar coordinates to Cartesian


coordinates.

7 (7, 7r/3) 8 (0, ir/3)

9 (-2, ir/4) 10 (6, 137r/6)

11 (1, -7r/3) 12 (-5. 150°)

In Problems 13 to 18, convert the given Cartesian coordinates to

polar coordinates (r, 6) with r > and — tt < 6 s tt.

13 (7, 7) 14 (I, -V3)


15 (-3, -3V3) 16 (-5, 5)

17 (0, 7) 18 (-2. 0)

In Problems 19 and 20, rewrite the answers to Problems 13 to 18

subject to the conditions given.

19 r>0, 0< e<277 20 r £ 0, s < 27r

In Problems 21 and 22, plot the given point P in the polar coordinate
system. Then give five other polar representations for the same
point.

21 P = (-3, ^rn)
SECTION 9.2 SKETCHING POLAR GRAPHS 541

50 Under what conditions will the graph of a first polar equation be able trigonometric identities.)
the same as the graph of a second polar equation obtained from
53 Suppose that a is a positive constant, (a) Show that the graph of
the first by multiplying both sides by r", where n is a positive
r = ±2a cos is a circle with center at the Cartesian point
integer?
(±a, 0) and radius a. (b) Show that the graph of r = ±2a sin

51 Explain why the graph of r- + 8/- sin S = is the same as the is a circle with center at the Cartesian point (0. ±0) and radius a.
graph of r + 8 sin 6 = 0.
54 (a) Show that the distance between the point (rj, di) and
52 Give a proof — without reference to diagrams — to establish rules the point (ri, 62) in the polar coordinate system is given by
governing the conditions under which (ri ,
6i) = (ri, S2) in polar Vn - 2rir2 cos (0, - 62) + rj. (b) Use the result of part (a)
coordinates. Use the fact that (ri, 61) = (ri, dj) if and only if to show that a polar equation of the circle with center (ro, 60)

ri cos di = rj cos 62 and r, sin 0\ = rj sin 62 . (Hint: Use suit- and radius a is P— Irr^ cos {0 — do) + rl = a'.

9.2 Sketching Polar Graphs

Spiral used as decoration

Figure 1

e
542 POLAR COORDINATES AND ANALYTIC GEOMETRY

is called an Archimedean spiral (Figure 2a), and the graph dt'

is called a logarithmic spiral (Figure 2b).

Figure 2

polar axi;

Archimedean spiral
logarithmic spiral
(a)

Symmetries of Polar Graphs


Symmetries of the graph of a polar equation can often be detected by making
suitable replacements in the equation and checking whether the new equation is
equivalent to the original one. The following table shows some replacements which,
when they produce equations equivalent to the original one, imply the type of
symmetry indicated:

Replace
SECTION 9.2 SKETCHING POLAR GRAPHS 543

In Examples 2 to 5, sketch the graph of the given polar equation and discuss the
symmetry.

] EXAMPLE 2 r = 2{\ - COS d)

SOLUTION Testing all the rules for symmetry, we observe that the graph is

symmetric about the polar axis; indeed, replacing dhy —6 yields

r = 2[l - cos (-6)] = 2(1 - cos 6)

which is the same as the original equation. We construct a table giving the polar
coordinates of some points on the graph corresponding to selected values of 6.
Plotting these points, we sketch the upper half of the graph. The lower half of the
graph is then drawn by using the symmetry about the polar axis (Figure 4). This
curve is called a cardioid because it is ""heart-shaped."

Figure 4

e
544 POLAR COORDINATES AND ANALYTIC (JEOMETRV

coordinates of some points on the graph corresponding to selected values of 8 for


-^ ^ TT. Plotting these points, we sketch the portion of the curve shown in

Figure 6a. The remaining portion of the curve is then drawn by using the symmetry
about the polar axis (Figure 6b). Notice that the graph has two loops — an inner loop
for 27r/3 < < 4Tr/3 and an outer loop for < <
In/ 3 and 4it/3 ^ 6 ^ 2it.

This curve is called a lima(;on. from the Latin word "limax" for snail. (Notice that

the portion of the curve in Figure 6a is similar to the shell of a snail.)

Figure 6 9
SECTION 9.2 SKETCHING POLAR GRAPHS 545

Figure 8 e
546 POLAR COORDINATES AND ANALYTIC CiEOMETRY

Direction of a Polar Graph


Now we derive formulas lor the direction of a polar graph at a point, that is, for the

inclination angle of the tangent line to the polar graph at the point. Thus, consider a
polar curve r = }\B), where /is a differentiable function of 0, and let / denote the
Figure 12 tangent line to the curve at the polar point P = (i\ 0) (Figure 12). Let a be the
angle measured from the polar axis to the tangent line /. Thus, since the polar axis
coincides with the x axis, tan a is the slope of the tangent line / in the Cartesian

xy coordinate system.
If P = (.V, y) in the Cartesian coordinate system, then we have dy/d.\ = tan a as

usual. Using the formulas for conversion from polar to Cartesian coordinates ob-
tained in Section 9.1, we have

.X = rcos O=f(0) cos 6 and /sin e=f{0) sin 6

Hence, and y can be regarded as functions of 8. By the chain


.v rule, dy/dd =
polar axis
(dy/dx){dx/de). Therefore, if dx/dd ¥= 0, we have

d\
SECTION 9.2 SKETCHING POLAR GRAPHS 547

Figure 13
If a polar curve passes through the pole for a particular value
of 6. the tangent line to the curve at the pole has a slope
equal to tan 6.

A polar graph may pass repeatedly through the pole, and it

may have different directions on each passage. (For instance,


consider the three-leaved rose in Figure 10.) To find the di-
rection of the curve on any particular passage, simply use the
appropriate value of 6 corresponding to that passage.

EX.\MPLE 7 Find the slope of the tangent to the four-leaved rose r = cos 20 on
that passage through the origin for which = 5-n-/4 (Figure 13). Also discuss the
motion of the point {r. 0) along the curve as increases from Q to Iv.

SOLLTio.N When = 0, r = 1, and we start at the indicated point on the polar


axis (Figure 13). As increases from to Tr/4. r = cos 20 decreases from 1 to 0.
and the curve makes its first passage through the pole. As continues to increase,
Figure 14 the point (r, 0) moves along the four-leaved rose according to the arrows, passing
through the pole for the second time when = 37r/4. for the third time when
li tangent line
= 57t/4, and for the fourth time when = 1it/4. When reaches 2tt. the point
{r. 0) returns to the starting point. On its third passage through the pole, when
6 = 57r/4. the slope of the tangent line is equal to tan (57r/4) = 1.

In dealing with polar curves, it is often convenient to specify the direction of the
tangent line / at a polar point P = (r. 0) by giving the angle ijj measured from the
radial line through P to the tangent line / (Figure 14a). (The symbol i// is the Greek
letter psi.) Since the three vertex angles of triangle OQP must add up to 180° = n
radians, it follows that + (n - a) + tj/ = n: that is. 4i = a - (Figure 14b).
Using the trigonometric identity for the tangent of the difference between two an-
gles,we have
tan a - tan 6
tan = tan (a - fl)
1 -I- tan a tan

Substituting the value of tan a previously derived into the last equation and simpli-
fying, we find that

tan li/
=
dr/d0

Figure 15
(Problem 32),

EXAMPLE 8 Find tan il/ at the polar point (2, 77/2) on the cardioid
r = 2(l - cos 0) (Figure 15).

SOLUTION Here we have dr/d0 = 2 sin e, so that

-
'an (A =
dr/d0
r
, =
2( I
1—
2sm
cos 0)
= CSC 6» - cot

Therefore, when = 7r/2, we have

tan i> = CSC


TT
cot —=
TT
1
548 POLAR COORDINATES AND ANALVTK (lEOMKTRV

Problem Set 9.2

In Problems I to 8. each of the curves is the graph of exactly one of 17 r = 4 sin 20 (four-leaved rose)

the equations (a) to (h). Match the equation with its corresponding
18 r = 2 sin 40 (eight-leaved rose)
graph.
(a) r = sin « (b) r = 1
- sin (c) r = cos H 19 r = 4(1 + cos 0) (cardioid)

(d) /• = 1 + 2 sin (e) r = 2 + sin (f ) r = 2 sin 20 = -


20 r 2(1 sin «) (cardioid)

(g) r = 0. 0>0 (h) r- = 2 cos 20


= -
21 r 3 2 cos (limagon)

22 r = 3 + 4 sin 61 (lima^on)

23 / = I
— 2 sin S (limagon)

24 r = \/0, 0> (reciprocal spiral)

25 r" = 8 cos 20 (lemniscate)

26 90 = In r (logarithmic spiral)

In Problems 27 to 30, find the slope of the tangent line to the graph
of each polar equation at the given point.

27 '• = 3(1 + cos 0) at (3, it/2)

28 r = 2( 1
- sin S) at ( 1 , tt/6)

29 /• = 8 sin 20 at (0, 0)

30 r = sec- at (4, 7r/3)

31 Suppose that / is a differentiable function, and consider the


polar curve r = f(0). (a) If idr/dO) sin + r cos, = and
Ulr/ild) cos - r sin 6 # 0, show that the tangent line to the
polar curve is horizontal at the polar point {r. 0). (b) Derive a
condition for the tangent line to the polar curve at the polar point
(r, 6) to be vertical.

32 Fill in all details in the derivation of the formula for tan 4>

(page 547).

In Problems 33 to 37. find tan i// at the indicated point {r, 0) on the
graph of each polar curve.

33 r = sm0 at (2, tt/6) 34 r = cos 20 at (0. 77/4)

35 r = e" at (e^ 2) 36 r- = esc 20 at ( 1 . 7r/4)

37 r = 4 sec e at (4. 0)
In Problems 9 to 26. test for symmetry with respect to the polar axis,

the line = tt/2. and the pole. Sketch the graph of the equation. 38 If < a < fo, find the slope of both tangents to the limaqon
r = a + b siT\ at the pole.

9 r = 4 cos e 10 r sin = 5
39 Find all points where the tangent to the lima§on r = 4 -I- 3 sin
11 r cos e = 5 12 /• = 2 is either horizontal or vertical. (Use the results of Problem 31 .)

13 r = 2 sin ff U 0=3 40 Find the polar coordinates of the tips of the petals of the
N-leaved rose r = 2 cos k0 by finding the values of where r
15 r = 2 sin 30 (three-leaved rose)
takes on its maximum values. (Note: Here it is not necessary to
16 r = 2 cos 20 (four-leaved rose) use calculus.)
SECTION 9.3 AREA AND ARC LENGTH IN POLAR COORDINATES 549

41 If < t < a. find the minimum value of r for the limagon 44 In view of Problem 42. the symmetry tests given on page 542
r = a + b sin 0. may fail to detect actual symmetries. Develop a set of tests that

cannot fail.
42 Show that the polar graph of r = sin (0/3) is symmetric about
the line 6 = ± 7r/2 even though the tests for such symmetry 45 Find and sketch the graph of a polar curve such that at each point
given on page 542 fail. Sketch the graph. of the curve, tan (/» = /f , where K is a nonzero constant.

43 Show that the polar graph of r = fid) is (a) symmetric about the 46 (a) If < a/2 < b < a. show that the limagon r = a + b s\n
line = if/ is an even function and (b) symmetric about the has an indentation, (b) If < fc s a/1, show that the limagon
line 6 = ±ir/2 if/ is an odd function. has no indentation.

9.3 Area and Arc Length in Polar Coordinates

Figure I Geometiically. the two fundamental problems of calculus are finding the slope of
the tangent to a curve and determining the area of a region bounded by a curve. In
(/(fJ).0)
Section 9.2 we attended to the first problem for polar curves: in this section we see
how to handle the second. We also develop a formula for arc length of polar curves.

Area of a Region in Polar Coordinates


Consider the curve whose polar equation is r = f{d). where /is a continuous func-
tion (Figure \). As 6 increases from = a to = /3, the point P = (f(d). 0) moves
along the polar curve from ifia), a) to (/(/3). jS) and the radial line segment OP
sweeps out a plane region. We refer to this region as the region enclosed by the
polar cune between 6 = a and 6 = fS. Below, we develop a formula for its area.
Figure 2 The simplest polar region is perhaps the circular sector enclosed by the circle of
radius r between = a and = P (Figure 2). Since the area of the circle is ttt" and
the sector occupies a fraction ()3 — a)/2Tr of the whole circle, the area A of the
sector is given by

A = 7iT~
P- a
— r-ip-a)
More generally, even if the polar curve is not a circle, when the angle increases
from to + d0. the radial segment OP in Figure 1 will sweep out a small region
which is virtually a sector of a circle of radius \r\ = \f{0)\ (Figure 3). If dd is
regarded as an infinitesimal, then the infinitesimal area of this sector is given by

dA = i\r\- d0 = ir- d0 = h[f(e)]- d0

If we "sum,'" that is, integrate, the areas of all such infinitesimal sectors, running
from o to /3, then we obtain the required area A. Thus,

J
^= J
kf(e}]-d0 = hj [f{e)]-de
e + de). e + de)

This formula is also written as

'•-
d0
polar a.xis
550 POLAR COORDINATES AND ANALYTIC CEOMETRY

Figure 4 EXAMPLE I Find the area of the "top half" of the region inside the cardioid
r= 3(1 + cos d) (Figure 4).

SOLUTION As 6 goes from to w. the radial segment OP sweeps out the top half
of the region inside the cardioid. Therefore, the area A of the region is given by

A ^ i r~ dd = k [3(1 + cos 0)]- de


^\ I
-'o

= 1 (1 + 2 cos 0+ cos- e)de


-'n

20
6( + 2 sin 6> + —e + sin 27 TT
square units
2 V " 2 4 / lo 4

Figure 5 EXAMPLE 2 Find the area enclosed by the lemniscate r- = 4 cos 26 (Figure 5).

P = (r.e) SOLUTION Consider the portion of the lemniscate for which r = 2Vcos 28.
When 6 = 0, r = 2, and as 6 increases, the point P = (r, 6) moves to the left along
llO)
the top of the lemniscate until it arrives at the pole O when 6 = 7r/4. Thus, as goes
polar
from to T7/4. the line segment OP sweeps out one-fourth of the desired area.

r^ = 4 cos 29
Therefore, the area A of the entire region is given by

/ r vr/4 \ r 7t/4 n/4


A = aU \ r- dd) = 2 \ 4 cos 26 dd = 4 sin 26
I

\
=4 square units
^ '0 '
^ 'o

Figure 6 EXAMPLE 3 Find the area of the region enclosed by the inner loop of the limagon
r = 2 — 3 sin (Figure 6).

SOLUTION When d = 0. r = 2 and the point (r, d) = (2, 0) is on the polar axis.
Now as 6 begins to increase, r = 2 — 3 sin begins to decrease, and it reaches
when 6 = sin" ' 3. At this point, the radial line segment OP begins to sweep out the
desired region. When 6 reaches the value 7r/2, then r = — 1 and the radial line

segment OP. which points straightdownward, has swept out exactly half of the
desired area. Therefore, the area A of the region is given by

A = 2 U
L
(2 - 3 sin d)- d6
-•
'sin '(2/.M

12 sin e + 9 sin- d) dd

I
n/2
= (40 + 12 cos + 2 » - f sin 26)
Uin '(2/3)

17 2 9
= IVtt

4
r

L 2
sm
„,

3
1-12 cos
/
I

V
sin
_,
—23/ \

4
sin
/
I

V
2

Since cos
/'

I sm
• -1
—2\ = —V5 I

— 4V5
and sin I 2 sin"' — 1 = 2 sin (sin"' -^] cos (sin"'

1777
we have A = 3\/5 s
s quare unit
4 2 3

In using the formula A = 2 ia r' d6 to find the area of the region enclosed by a
SECTION 9.3 AREA AND ARC LENGTH IN POLAR COORDINATES 551

polar curve r = f(d) between d = a and ^ = /3, you must be certain that a < /3 and
that the radial line segment OP. where P = (f(d), d). sweeps just once across each
point in the interior of the region. For instance, if you wish to find the total area
inside the limagon r = 2 - 3 sin (Figure 6), it would be incorrect to integrate
from to 277. The reason is that as 6 goes from to 2it, the radial segment OP
sweeps twice across all the points within the inner loop, so the area of the inner loop
gets counted twice.

Area Between Two Polar Curves


To determine the area of a plane region enclosed by two polar curves, we need to
know where the curves intersect. Although the points where two Cartesian curves
intersect can be found by solving the equations of the curves simultaneously, this
procedure does not necessarily yield all intersection points of two polar curves. The
reason is that an intersection point may have two different polar representations, one
satisfying the equation of the first curve and the other satisfying the equation of the
second curve.
All the intersection points P of the polar graphs of

r=f(e) and r = g(d)

can be found by carrying out the following procedure:

Step 1 Check each polar graph separately to see whether it passes through the
pole O. If both do, then O is an intersection point of the graphs.
Step 2 Solve, if possible, the simultaneous equations

= fie)

= g(d + Inn)
for r, d, and n, where r ^ and n must be an integer. For each such solution,
the point P = (r, 0) is an intersection point of the two graphs.
Step 3 Solve, if possible, the simultaneous equations

--no)

--
-g(e + an+ dtt)

for r, 0, and n, where r t^ and n must be an integer. For each such solution,
the point P = (r, 0) is an intersection point of the two graphs.

EXAMPLE 4 Find all points of intersection of the circle r = -6 cos and the
limagon r = 2 - 4 cos 6.

SOLUTION
Step 1 Since O = (0, 7r/2) is on the circle and O = (0, 77/3) is on the lima^on,
it follows that O is a point of intersection.
Step 2 We solve the simultaneous equations

—6 cos 6

2 - 4 cos (0 + Irnr) for r ?^ 0, /i an integer

Noting that cos {6 + 2mr) = cos d, we rewrite the equations above as

r = -6 cos = 2 - 4 cos
552 POLAR tmiRDINATES AND ANALYTIC GKOMF.TRY

Figure 7

Figure 8

Figure 9

Figure 10
SECTION 9.3 AREA AND ARC LENGTH IN POLAR COORDINATES 553

and
dy = d(r sin d) = dr %\n d \- r cos dO

Thus,

(dxf + (dyf = {drf cos^ d - 2r cos d sin d {dr){de) + r- sin^ [ddf


+ (<ir)- sin- e+lr cos 6» sin (dr){de) + r" cos^ {ddf
= (dr)-(cos- e + sin- 0) + r-(J0)-{sin- + cos" 6)

= (dr)- + r-(de)-

so that

ds = \/(dx)- + (^v)- = V'(rf/-)- + r-(je)-

^r^^<^^^^
Therefore, in polar coordinates, ^([T.r
ds )-= J0
dd
Figure 11
(Figure 11).
The arc length of the portion of a polar curve r = f{6) between 9 = a and 6 = fi

is accordingly given by

'''" +'''"^= V[/'(e)]2 + ifiO)]- do


polar axis "J J yji^) /

provided that the derivative /' exists and is continuous on the interval [a, ji].

EXAMPLE 6 Find the total arc length of the cardioid r = 2( 1 - cos B) (Fig-
Figure 12 ure 12).

SOLUTION Because the cardioid is symmetric about the polar axis and its exten-
sion, we have

dd

= 2 V(2 sin 0)- -I- 4(1 - 2 cos 6 + cos" e)de

= 4 Vsin- e + cos- e + \ - 2 cos 6 dd


'o

= 4 V2 - 2 cos ede = 4 V2 Vl - cos Odd


^ 'o

Since 2 sin- (6/2) = 1 - cos 6, it follows that

= 4V2
J
J2 sin- — de = \V2 j V2 sin — dQ
2

-2 cos ~) I
=16 units
554 P()1,AR COORDINATES AND ANALYTIC GEOMETRY

Problem Set 9.3

In Problems 1 to 6. find the area enclosed by each polar curve Figure 13


between the indicated values of 9. Sketch the curve.

polar axis
1 r = 4e from = tt/4 lo = 5tt/4

2 r = sin- (6/6) from = Q to = tt

3 r = 4/0 from = it/4 to = 5tt/4

4 r = 3 sin 36 from = to 6 = 7r/3 = - 4


30 Find the area of the region inside the limagon r 2 cos

5 r = 3 CSC e from = ir/2 to = Sir/d and outside the circle r = -6 cos (Figure 7).

6 r = 2 sec' from 6 = to 6 = 7r/3


In Problems 31 to 38, find the arc length of each polar curve be-
tween the indicated values of 0.
In Problems 7 to 14, find the area enclosed by each polar curve. In

each case sketch the curve, determine the appropriate limits of inte- 31 r = 6 sin from = to = tt
gration, and make certain that none of the area is being counted
twice. 32 r = -2 from = to = 27r

33 r = 40- from = to = f
7 r =4 sin S 8 r = 2(1 + cos 0)
34 r = 2(1 -I- cos 0) from = to = 27r
9 r = 2 sin 3fl 10 r = 2\/|cos 0\

35 r = e'from = to = 47r
11 r= 2 + cos 12 r = 1 + 2 cos e
36 r = 2 sin-' (0/3) from = to = 37r
13 r = 2 — 2 sin 14 r = 3 cos S + 4 sin
37 / = 3 cos H- 4 sin from = to = 7r/2

In Problems 15 to 22, find all points of intersection of each pair of


38 r = -7 CSC from = 7r/4 to = 377/4
polar curves. Sketch the two curves, showing these points.

39 Find the error in the following argument: Since the graph of the
15 r = -3 sin and r = 2 + sin 6
polar equation r = 4 cos is a circle of radius 2, the circumfer-

16 r = cos and r = sin S 17 r = 1 and r = 2 cos 36 ence of this circle should be given by

18 r = for e > and r = -e for e >


19 r = sin and r = sin 20

20 r = 1 + cos e and r = (1 + V2 ) cos e Vl6 sin^ -I- 16 cos- d0 = \ 4 dO = Stt

21 r = 2 sin 30 and r = -2/sin 22 r = and r = 20


(But the circumference of a circle of radius 2 should be only
47r.)
In Problems 23 to 27, find all points of intersection of the two
curves, sketch the two curves, and find the area of each region 40 Criticize the following naive use of differentials; In Figure 14,

described. ds is virtually the arc length of the portion of the circumference

of a circle of radius \r\ cut off by the angle dO radians; therefore.

23 Inside r = 6 sin and outside r = 3. ds should be given by |/| d0 rather than by V (dr/d0y + r' d0.

24 Inside both r = I and r = 1 + cos 0. Figure 14

25 Inside both r = 3 cos and r = 1 + cos 0.

26 Inside r" = 8 cos 20 and outside r = 2.

27 Inside r = 6 cos and outside r = 6(1 - cos 0).

28 Assuming that h > a > 0. find a formula for the area enclosed
polur axis
by the inner loop of the limagon r = a + b sin 0.

29 Find the area of the shaded region enclosed by the graph of 41 Show that, with suitable restrictions, the area of the surface gen-
r = in Figure 13. erated by revolving the portion of the polar curve r =/(0) be-
SECTION 9.4 THE ELLIPSE 555

tween 6 = a and 6 = p about the polar axis is In Problems 43 to 46, use the formulas given in Problems 41 and 42
to find the area of the surface generated by revolving each polar
A = 277
J
rsineyJ[-J-j + r dd curve about the given axis.

42 Show that, with suitable restrictions, the area of the surface gen- 43 r = 2 about the polar axis
erated by revolving the portion of the polar curve r = f{6) be-
44 r = 4 about the axis 6 = tt/I
tween = a and = p about the axis 9 = tt/I is

45 r = 5 cos 6 about the polar axis


A = 277 r COS e\] + r- dd
J y-l-j 46 r" = 4 cos 16 about the axis 6 = -n/l

9.4 The Ellipse

For the remainder of this chapter we study the conic sections (or conies, for short).
These graceful curves were well known to the ancients; however, their study is

immensely enhanced by the use of analytic geometry and calculus. They are ob-
tained by sectioning, or cutting, a right circular cone of two nappes with a suitable
plane (Figure 1).

Figure I

Ellirse Hyperbola

By shining a flashlight onto a white wall, you can see examples of the conic
sections. If the axis of the flashlight is perpendicular to the wall, then the illumi-
nated region is circular (Figure 2a); if the flashlight is tilted slightly upward, the
illuminated region elongates and its boundary becomes an ellipse (Figure 2b). As
the flashlight is tilted further, the ellipse becomes more and more elongated until it

changes into ^l parabola (Figure 2c). Finally, if the flashlight is tilted still further,

the edges of the parabola become straighter and it changes into a portion of a
hyperbola (Figure 2d).
We have already obtained equations for circles, in both Cartesian and polar form.
In this section, we derive Cartesian equations for ellipses, and we see how such

Figure 2

Circle Ellipse Hyperbola

(d)
556 POLAR COORDINATES AND ANALYTIC GEOMETRY

equations can be simplified by translation of the coordinate axes. Parabolas and


hyperbolas in Cartesian form are studied in the next two sections.*
Ellipses are of practical importance in fields ranging from art to astronomy. For
instance, a circular object viewed in perspective forms an ellipse (Figure 3), and an
orbiting satellite (natural or artificial) moves in an elliptical path. The geometric
definition of an ellipse is as follows.

DEFINITION 1 Ellipse

An ellipse is the set of all points P in the plane such that the sum of the distances
Figure 3
from P to two fixed points F, and Ft 's constant. Here F| and Ft are calle d the
focal points, or the foci, of the ellipse. The midpoint C of the line segment F1F2
is called the center of the ellipse.

Figure 4a shows two fixed pins Fi and Ft and a loop of string of length / stretched

tightly about them to the point P. Here we have

Iff, I
+ IFF2I + 1^2! = / or |FFi| + IFF2I = / - \F\F2\

Figure 4

Hence, as P is moved about, |FFi| + IFF2I always has the constant value / - IF1F2I.
Figure 5
Thus, if a pencil point P is inserted into the loop of string and moved so as to keep
the string tight, it traces out an ellipse (Figure 4b).
Consider the ellipse in Figure 5a with foci F\ and F2 and center C. We denote by
( the distance between the center C and either focus F, or Ft Notice . that the ellipse

is symmetric about the line through F, and F2. Let V\ and V2 be the points where
this line intersects the ellipse. The center C bisects the line se gme nt ViVt- and the
ellipse is symmetric about the line through C perpendicular to ViV2- Let V3 and V4
be the points where this perpendicular intersects the ellipse. The four p oints V, , Vt,
V3, and V4 are called the vertices of t he ellipse. The line segment V^V^ is called the
major axis, and the line segment V3V4 is called the minor axis of the ellipse. Let la

denote the length of the major axis, and let 2h denote the length of the minor axis
(Figure 5b). The numbers a and h are called the semimajor axis and the semiminor
axis of the ellipse.
If a point P moves along the ellipse in Figure 5, then, by definition, the sum

|FF|| + IFF2I

does not change. Therefore, its value when P reaches V, is the same as its value
when P reaches Vy. that is,

|V^F,| + \y\F2\ = |V^F,| + |V^t|

*See Section 9.8 for polar equations of the conies.


SECTION 9.4 THE ELLIPSE 557

By symmetry
IVjFil = IV3F2

so the equation above can be rewritten

But, by symmetry again,


IViF.I = \V2F^\

Hence,

2|V^2| = IVV^il + \y^2\ = \V^i\ + |VSf il = \V\V2\ = 2a

Figure 6 from which it follows that


IV3F2I = a

Therefore, applying the Pythagorean theorem to the right triangle V^CFi (Figure 6),
we find that

If we place the ellipse of Figure 6 in the .r\' plane so that its center C is at the

origin O and the foci F, and F2 lie on the negative and positive portions of the .v

axis, respectively, then we can derive an equation of the ellipse as follows.

THEOREM 1 Ellipse Equation in Cartesian Form

An equation of the
558 POLAR t OORDINATES AND ANALYTIC GEOMKTRY

so that

a"* — a'c' = {a' — c').x~ + a~y~ > a~(a- - C-) = {a- - c-).x^ + aW
Since a- = b^ + c^. we have a- - c~ = b~. and the equation above can be rewrit-
ten as
a'b' = b'x' + a'y~

If both sides of the last equation are divided by a~b-. the result is

\-
1
-V"
= ^r +
a-
^
b-

as desired. By reversing the argument above, it can be shown that, conversely, if

the equation {x~/a-) + (y-/b~) = 1 holds, then the point P = U, v) is on the ellipse

(Problem 48).

Figure 8
If a and h are positive constants and a > b. the Cartesian

equation

b-

Figure 9
SECTION 9.4 THE ELLIPSE 559

and Fi = (0, c) v axis and vertices V, = (0, -a). V2 = (0. a), V, = -b. 0),
on the (

and V4 = The semimajor axis is a. and the semiminor axis is b. The equation
(b, 0).

can be derived as in Theorem 1 the argument being word for word the same except
,

that the variables x and v interchange their roles. Therefore, the equation is

where a > b
Figure II b-

This equation is also called the standard form for the equation of an ellipse.

Translation of Axes

The equation of a curve can often be simplified by changing to a different coordi-


nate system. In practice, this is often accomplished by choosing one or both of the
new coordinate axes to coincide with an axis of symmetry of the curve. For in-
stance, the equation of the ellipse in Figure 1 1 would probably be simplified by
switching from the .n coordinate system to the at coordinate system as shown.
If two Cartesian coordinate systems have corresponding axes that are parallel and

have the same positive directions, then we say that these systems are obtained from
Figure 12
one another by translation.
Figure 12 shows a translation of the "old_^" .x^ coordinate

>') in "old" system


system to a "new" xy system whose origin O has the "old"
r) in "new" system coordinates (/?. k). Consider the point P in Figure 12 having
old coordinates (.v. v). but having new coordinates (.v, y).
Evidently, we have the following equations for translation of
X Cartesian coordinates:

f.v = h +x
560 POLAR coordinatj:s and analytic geometry

We begin by rewriting the equation in the form

4.V- - 31v + 9v- - 36y = -64


or 4(.v- - 8.V ) + 9(.v--4v ) = -64
where we have left spaces for completing the squares. Thus.

4(.v- - 8.V + 16) + 9( V- - 4y + 4) = -64 + 4(16) + 9(4)

or 4(.v - 4)- + 9(.v - 2f = 36

Dividing both sides of the last equation by 36, we obtain

{x-4f ^
(v-2)- _
9 4

which is of the form

— + ^-
9 4

with and V = V - 2

The last two equations represent a translation of the coordinate axes, and the new
rv coordinate system has an origin O whose old .rv coordinates are (4, 2) (Fig-
Figure 14 ure 14). With respect to the xy coordinate system, the graph of

—+—
9 4

is an ellipse with center O, semimajor axis V9 = 3, and semiminor axis Vi = 2,


which we have sketched in Figure 14. But this is the same as the graph of the
equation

(X - 4)- (y^- 2)-


'

9 4

which, in turn, is the same as the graph of the original equation

4.V- + 9y- - 32v - 36y + 64 =

Thus, by means of a translation of coordinates, we have obtained an accurate graph


with a minimum of effort. (If desired, the new xy coordinate system can be drawn
very lightly and then erased after it has done its job.)

EXAMPLE 4 Show that the graph of 25.r + 9v- - 100.v - 54v = 44 is an ellipse.
Find the coordinates of the center, the vertices, and the foci, and sketch the graph.

SOLUTION Here we have

23(.v- - 4v ) + 9(y- - 6v ) = 44

Completing the squares in the last equation, we obtain

25(.r- - 4.V + 4) + 9(.v- - 6v + 9) = 44 + 25(4) + 9(9)

or 25(.v - 2)- + 9(.v - 3)- = 225


Dividing the last equation by 225, we have

(.V - 2)- (V
(y - 3)-
= 1

25
SECTION 9.4 THE ELLIPSE 561

which is of the form

—+—= 1

9 25

with A- and

The last two equations represent^ translation of the coordinate axes, and the new xy
coordinate system has an origin O whose old xy coordinates are (2, 3). The graph of

—+—=
x~ V-
1
9 25

is an ellipse with center O, vertical major axis, semimajor axis a = V25 = 5, and
Figure 15 semiminor axis b = v9 = 3 (Figure 15). The distance c between the center and the
foci is given by

c = Va- - b- = V'25 - 9 = 4 units

Thus, the foci Fi and F2 have -r\' coordinates

F, =(2, 3 -4) = (2. -1) and f. = (2. 3 + 4) = (2, 7)

and the xy coordinates of the four vertices are

(2, -2) (2. 8) (-1, 3) and (5, 3)

Notice that a translation of the coordinate axes does not change the position or the
shape of a curve in the plane — it changes only the equation of this curve. Thus, by
using the translation equations x = x - h and y = y - k. we can see that an equa-
tion of the form

54y = 44

ix^
562 POLAR COORDINATES AND ANAI.VTIC CF.OMETRY

Problem Set 9.4

In Problems I to 8. find the coordinates of the vertices and foci of foci, and sketch the graph showing both the old xy and the new xy
each ellipse, and sketch its graph. coordinate systems.

1
^- + ^= 1
2- + y^=l n r' + 4y' -I- 2v - 8y + 1 =
16 4
18 9x^ + y- - I8x -t- 2y -I- 9 =
3 4a- + V- = 16 4 36.r + 9y- = 144
19 6.v^ -I- 9y' - 24x - 54y -H 5 1 =
5 r+ I6.V- = 16 6 I6.r + 25y- = 400
20 9x^ -I- 4y^ - 18x + 16y - 1 1 =
7 9.V- + 36v- = 4 8 A- + 4v- = 1

21 16.r + 9v^ - 192x -I- 36y -h 468 =

In Problems 9 to 12, find the equation in standard form of the ellipse 22 3.V- + 4y- - 12v -h 8y + 4 =
that satisfies the conditions given.

In Problems 23 to 30, find the coordinates of the center, the vertices,


9 Foci f, = (-4, 0) and F. = (4, 0); vertices V, = (0, 3) and and the foci of each ellipse, and sketch its graph.
V4 = (0, -3)

10 Vertices V| = (-5, 0) and Vi = (5, 0); horizontal major axis;


23ll^l)i^(l±2)l^j
= 9 4
c 3 units

11 FiK-i r, = (0, -12) and F2 = (0. 12); vertices V, = (0, -13) 24<^.<^=l 16 4
and v. = (0, 13)
25 4(a + i)- + y- = 36
12 Foci F] = (0, -8) and F^ = (0, 8); semiminor axis 6 = 6 units
26 25{A -I- 1)- + 16(y - 2)' = 400
13 A new xy coordinate system is obtained by translating the old .vv
27 .r -I- 2y- + dx + 1 =
coordinate system so that the origin O of the new system has old
.n' coordinates (-1,2). Find the new xy coordinates of the points 28 4.r + y- - 8x -^ 4y - 8 =
whose old xy coordinates are:
29 2x^ -I- 5y- + 20a - 30y + 15 = Q
(a) (0, 0) (b) (-2. 1) (c) (3, -3)
-
30 9x2 _,_ 4^,2 j^ ,g^. _ |g^, -11=0
(d) (-3, (e) (5, 5) (f) (6, 0)

14 Describe a translation of coordinates that will reduce the equa- In Problems 31 to 42. find an equation for the ellipse satisfying the

tion conditions given.

x^ + _v^ + 4x - 2\' + 1 = 31 Foci (-4, 1) and (4, 1); vertices (-5. 1) and (5, 1)

of a circle in the old xy coordinate system to the simpler form 32 Foci (I, -2) and (1, 2); vertices (1, -4) and (1, 4)
Tr + y' = r in the new xv system. Find the radius r, and draw a
33 Vertices (0, -8) and (0, 8); containing the point (6, 0)
graph showing the circle and the two coordinate systems.

15 A new xy coordinate system is obtained by translating the old .vv


34 Vertices (0, -3) and (0, 3); containing the point (3, 2 V2 )

coordinate system so that the origin O of the old av system has 35 Center at the origin; containing the points (4, 0) and (3, 2)
coordinates (
- 3, 2) in the new xv system. Find the old xy coor-
dinates of the points whose new xv coordinates are: 36 Vertices (-2V3, 0), (2V3, 0), (0, -4). and (0, 4)

(a) (0, 0) (b) (3, 2) (c) (-3, 4) 37 Vertices (-2, -3). (-2, 5), (-7, 1). and (3, I)

(d) (V2. -2) (e) (0, -7T) (f) (-3. 2)


38 Foci and major axis 10 units long
(1, 3) (5, 3);

16 Let h, c. and d be constants. Show that the translation


39 Center (1, -2); major axis parallel to the y axis, major axis 6
X = .V -I- {h/3), y =y+ (cb/3) - d- (Ib^lll) reduces the
units long; minor axis 4 units long
cubic equation y = x^ ^- fcc^ \- ex -V d\o the formy = x' -I- px.
where p = c — (b-/3). 40 Endpoints of major axis (—3, 2), (5, 2); length of minor axis is

4 units
In Problems 17 to 22, use a suitable translation of axes x = x - h
41 Vertices (2, -3) and (2, 5); foci (2, -2) and (2. 4)
and y = y - k 10 bring each equation into the standard form for an
ellipse. Find the Ay coordinates of the center, the vertices, and the 42 Foci (-4, 5) and (2, 5); semiminor axis /; = 3 units
SECTION 9.4 THE ELLIPSE 563

In Problems 43 to 46. use implicit differentiation to find the slopes 53 An arch in the shape of the upper half of an ellipse with a hori-
and the equations of the tangent and normal Unes to each ellipse at zontal major axis is to support a bridge over a river 100 meters
the point indicated. wide. The center of the arch is to be 25 meters above the surface
of the river. Find the equation in standard form for the ellipse.
43 .X- + 9v- = 225 at (9. 4) 44 4r^ + 9r = 45 at (3, 1)
54 In Figure 4b, sh ow that th e semiminor axis b of the ellipse is

45 .X- + 4y- -lx + 8y = 35 at (3. 2) given by b = ^y/F - 4lc. where c = ilf i^:!-

46 9jr + 25y- - 50y - 200 = at (5. 1) 55 Let A be the area enclosed by an ellipse with semimajor axis a
and semiminor axis b. (a) Show that
47 The segment cut by an ellipse fr.om a line containing a focus and
perpendicular to the major axis is called a focal chord of the —\'a- X- dx
ellipse, (a) Show that Ib'/a is the length of a focal chord of the
Jq a

ellipse whose equation is b-.\~ + a'y- = a'b'. (b) Find the (b) Evaluate the integral in part (a) and thus obtain the formula
length of a focal chord of the ellipse 9.x- + 16y" = 144. A = Trab for the area of an ellipse.

48 Suppose that a and b are constants such that a > b > and 56 Pro\e that the normal line to the ellipse in Figure 18 at the point
let c = V? — b'. Assume that the numbers .v and y satisfy
P = (.r, y) bisects the angle FiPFi. [Hint: The equation of the
the equation (xr/a-) + iy/b') = \. and let P = (x. y), ellipse is {x^/a') + (y'/b') = I. By implicit differentiation, the
Fi = (— c, 0), and A= (c, 0). Prove the following without slope of the normal line at f is m= a-y/ib'x). The slopes oi FiP
reference to geometric diagrams: and F2P. respectively, are
(a) c\x\ < a^ (b) V(.v - c)- + y- < 2a

+ \PF2\ = 2a and
(c) 1^,1 X + c

49 Find maximum area of a rectangle that can be inscribed in


the
where f, = (-c, 0), Fi = (f- 0), and c = \'a- - b'. Verify
the ellipse {x^/a-) + {)r/b') = if the sides of the rectangle are1
- m^m^im = nti + mj. Then show that
that (mi -I- m2)m- -I- 2( 1

parallel to the coordinate axes.


the last equation implies the desired result.]

50 A mathematician has accepted a position at a new university


Figure 18 y
situated 6 kilometers from the straight shoreline of a large lake
(Figure 17). The professor wishes to build a home that is half as
far from the university as it is from the shore of the lake. The
P=U.y)
possible homesites satisfying this condition lie along a curve.
Describe this curve, and find its equation with respect to a coor-
dinate system having the shoreline as the x axis and the univer-
sity at the point (0. 6) on the y axis.

Figure 17 univereity

J
\
^J —'->PO'
I

I
possible
site
home
57 A ray of light will be reflected from a curved mirror (Figure 19)
6km in such a way that the angle a between the incident ray and the
normal is equal to the angle a between the normal and the re-

flected ray. Use the result of Problem 56 to show that a ray of

light emanating from one focus of an elliptical mirror will be


reflected through the other focus. (This is called the reflecting

51 How long a loop of string should be used to lay out an elliptical


property of the ellipse.)

flower bed 20 feet wide and 60 feet long? How far apart should
Figure 19
the two stakes (foci) be?

52 Except for minor perturbations, a satellite orbiting the earth


moves in an ellipse with the center of the earth at one focus.
Suppose that a satellite at perigee (nearest point to center of
earth) is 400 kilometers from the surface of the earth and at
apogee (farthest point to center of earth) is 600 kilometers from
the surface of the earth. Assume that the earth is a sphere of
radius 6371 kilometers. Find the semiminor axis b of the ellipti-
cal orbit.
564 POIAK COORDINATES AND ANALYTIC GEOMETRY

9.5 The Parabola


In Chapter 1 (page 34) we mentioned that the graph of a quadratic function is "a
'

type of curve called a parabola. " In this section we give a proof of this fact based on
the following geometric definition.

DEFINITION 1 Parabola

A parabola is the set of all points P in the plane such that the distance from P
to a fixed point F (the focus) is equal to the distance from P tod fixed line D (the
directrix).

Parabolas appear often in the real world. A ball thrown up at an angle travels
along a parabolic arc (Figure 1), a main cable in a suspension bridge forms an arc of
a parabola (Figure 2), and the familiar "dish antennas"" have parabolic cross sec-
tions (Figure 3).

Figure 1 Mulliflash photo of Figure 2 Bay Bridge. San Figure 3 Parabolic dish antenna
a ball thrown into the air Francisco, showing parabolic
cables

If the focus F of a parabola is placed on the positive y axis at the point (0, p) and
if the directrix D
placed parallel to the x axis and p units below it, the resulting
is

parabola appears as in Figure 4. Its Cartesian equation is derived in the following


theorem.

Parabola Equation in Cartesian Form


An equation of the parabola with focus F (0. /)) and with directrix D: y = —p
is .x~ = 4py. or y = [\/{4p)].\~.

Let f = (.V. y) be any point and let Q = (.v, -p) be the point at the foot of the
perpendicular from P to the directrix D (Figure 4). The requirement for P to be on
the parabola is \FP\ = \PQ\: that is.

V.V- + (y - p)- = V'(y + pf


SECTION 9.5 THE PARABOLA 565

Figure 4 The last equation is equivalent to

A- + (y - p)- = (y + p)-

that is.

- + y- — Ipy + p' = >•" + 2py + p' or .\- = Apy

Obvious modifications of the argument in Theorem 1 provide Cartesian equations


for parabolas opening to the n^?/ir ( Figure 5b), downward (Figure 5c), and to the left
(Figure 5d). Any one of the equations
Q = (x.-p)

~-4py or :4/?.v

is called the standard form for the equation of a parabola.

Figure 5

t
upward f
566 POLAR COORDINATES AND ANALYTIC GEOMETRY

Figure 7 EXAMPLE 2 X- = -16v

SOLUTION The equation has ihe form .v" = -Apy with p = A: hence, it corre-
sponds to Figure 5c. Therefore, the graph is a parabola opening downward with
focus given by

F = (0, -p) = (0, -4)

and directrix D. v = />; that is, D: v = 4 (Figure 7).

A parabola with focus F and directrix D is evidently symmetric about the line
through F perpendicular to D. This line is called the axis of the parabola (Figure 8).
The axis intersects the parabola at the vertex V. which is located midway between
the focus and the directrix. The segment cut by the parabola on the line through the
focus and perpendicular to its axis is called the focal chord, or the latus rectum, of
the parabola (Figure 8). The following example shows how to find the length of the
focal chord.

EX.4MPLE 3 A parabola opens to the right, has its vertex at the origin, and con-
tains the point (3, 6). Find its equation, sketch the parabola, and find the length of
its focal chord.

SOLUTION The equation must have the form v^ = Apx. Since the point (3, 6)
belongs to the graph, we put = 3 and v = 6 in the equation to
.\ obtain 36 = 12/?,

and we conclude that p = 2>. Thus, an equation of the parabola is v" = llv (Fig-
ure 9). The focus is given by F = (p, 0) = (3, 0). The focal chord lies along the line
.V = 3 Putting A = 3 in the equation v" = 2.v and solving for y we obtain t = 36,
. 1 ,

y = ±6. Thus, the points (3, 6) and (3, -6) are the endpoints of the focal chord,
Figure 9 and therefore its length is 12 units.

D. X = -3 V

A parabola with its vertex at the origin which opens upward, to the right, down-
ward, or to the left is said to be in standard position. If a parabola is in the standard
position, then its axis of symmetry is either horizontal or vertical. Conversely, if a
parabola has an axis of symmetry that is either horizontal or vertical, then a transla-
tion of the coordinate axes to the vertex of the parabola will put it into standard

position with respect to the "new" coordinate system.


For instance, if the parabola in Figure 10 has its vertex at the point (/i, k) with
respect to the old .vv coordinate system and opens to the right as shown, then its

equation with respect to the new a v coordinate system is v" = 4/ja . Since x = x - h
and y = y — k. its equation with respect to the old x-y coordinate system is

(j — k)~ = Ap(x — h). Similar arguments can be made for parabolas opening up-
Figure 10
ward, downward, or to the left. The following table summarizes the results:

kY =4/)(.v /I)

Vertex
SECTION 9.5 THE PARABOLA 567

Figure 11

= -12(.r- 2)'
+ l)2

V
568 POLAR COORDINATES AND ANALYTIC GEOMETRY

Figure 14 One of the most important properties of a parabola is its reflecting property: A
ray of light emanating from the focus of a parabolic mirror is always reflected
parallel to the axis (Figure 14). In order to demonstrate the reflecting property
analytically, we recall that a ray of light striking a curved mirror is reflected so that

the angle between the incident ray and the normal to the mirror is the same as the

angle between the normal and the reflected ray.


Thus, in Figure 15 we wish to show that angle a is the same as angle /3. By
implicit differentiation of the equation 4p.x = y- of the parabola,

4p ±)

Figure 15

Figure 16

region of
"good"
approximation
SECTION 9.5 THE PARABOLA 569

9 (y - 2f = 8(.v + 3) 10 (y + !)-= -4(x- 1) 30 Show that the arc length i of the parabola 4py = x' between its

vertex (0, 0) and the point (a, a'/(4p)) is given by


11 (x - 4)- = 12(.v + 7) 12 {x+ 1)- = -8y
aV4p- + a- V4p-
'^ + a' + a
13 v^ - 8y - 6.V - 2 = 14 Zr + 8.V - 3y +4 = s = -. + pln
4p 2p
15 jr - 6.r - 8v + 1 = 16 V- + lOv - .V + 21 =
31 Find the dimensions of the rectangle of largest area whose base
is on the x axis and whose two upper vertices are on the parabola

In Problems 17 to 22, find an equation of the parabola that satisfies whose equation is y = \1 - xr (Figure 17).
the conditions given.

Figure 17
17 Focus at (4, 2) and directrix .r = 6

18 Focus at (3, -1) and directrix y = 5

19 Vertex at (-6. -5) and focus at (2, -5)

20 Vertex at (2, -3) and directrix ^ = —8

21 Axis is parallel to the .v axis, vertex (—2. ^1). and contains the
point (f, 2).

22 Axis coincides with the y axis and parabola contains the points

(2, 3) and (-1, -2).

32 Show that, if p is the distance between the vertex and the focus
of a parabola, then the focal chord of the parabola has length Ap.
In Problems 23 to 25, reduce each equation to "simpler" form by a
suitable translation of axes. Also, sketch the graph in the old as well A roadway 400 meters long is held up by a parabolic main cable.
33
as the new coordinate system. The main cable is 100 meters above the roadway at the ends and
4 meters above the roadway at the center. Vertical supporting
23 y- + 2y - 8.V - 3 = 24 .r + 2v + 4v - 7 = cables run at 50-meter intervals along the roadway. Find the

= lengths of these vertical cables. {Hint: Set up an .rv coordinate


25 5y .r + 4x + 19
system with vertical y axis and having the vertex of the parabola
4 units above the origin.)
26 Let A. B, and C be constants with A > 0. Show that y =
Ax^ + Sj: + C is an equation of a parabola with a vertical axis of 34 The surface of a roadway over a stone bridge follows a parabolic
cur\'e with the vertex in the middle of the bridge. The span of the
symmetry, opening upward. Find the coordinates of the vertex V
bridge is 60 meters, and the road surface is 1 meter higher in the
and the focus F. Find p and the length of the focal chord. Find
conditions for the graph to intersect the x axis. middle than at the ends. How much higher than the ends is a
point on the roadway 15 meters from an end?
27 Find equations of the tangent and normal lines to each parabola
at the point indicated.
35 For a > 0, let (0, /(a)) be the center of the circle tangent to the
parabola whose equation is Apy = .»- at the points whose x coor-
(a) y^ = Sx at (2, -4)
dinates are -a and a (Figure 18). Find (a) a formula for /(a) in
(b) 2y2 = 9x at (2, -3) terms of a and p and (b) lim f(a).
(c) -r = -12y at (-6, -3)

(d) .r + 8y + 4.V - 20 = at (1, ^ Figure 18

(e) y- - 2y + lO.v - 44 = at (|, 1)

28 How can you tell immediately (without computation) that


3y - 2 = 4.<r — 21x + 1 1 is the equation of a parabola opening
upward?

29 Find the coordinates of the vertex of each parabola by differenti-


ating its equation on both sides and then solving for a horizontal
(or vertical) tangent.

(a) y = .r - 2x + 6

(b) Ax^ + 24.r + 39 - 3y =


(c) r - lOy = 4.r - 21 Apy = .r at and near
36 Find the circle that "best fits" the parabola
(d) 3.r = 14y - y- - 43 its vertex by letting a approach in Figure 18.
570 POLAR COORDINATES AND ANALYTIC GEOMETRY

37 In Figure 15. show that tan a = (m - mO/d + mm,) and that Figure 20. If we hold the lower vertex fixed at O and hold the
tan /3 = -111. where m is the slope of the normal line and m, is lower focus fixed at (0, p). but allow the upper vertex to ap-
the slope of the segment FP. [Him: Use the formula proach +^ along the y axis, then the ellipse approaches the
parabola y = (l/4;)).r as a limiting curve. Prove this.
tan /) — tan 0.
tan (»- Oi)
=
1 + tan tan «,
Figure 20 -k)\
(y
from trigonometry.]

38 In Figure 15, verify that a = fi. [Use Problem 37 and the rela-

tions m= —y/2p, nil = y/(x - p), and ^px = y".]

39 Figure 19 shows a cross section of a parabolic dish antenna.


Show that the focus F of the antenna is /) units above the vertex
V, where p = a-/(\()b).

Figure 19 F

41 A calculus student says, "If you've seen one parabola, you've


40 A parabola may be thought of as an enormous ellipse with one seen them all." (a) Explain why this is essentially true.

vertex infinitely far away. To see this, consider the ellipse in (b) Explain why a similar statement cannot be made for ellipses.

9.6 The Hyperbola


Hyperbolas are of practical impoilance in fields ranging from engineering to naviga-
tion. The natural-draft evaporative cooling towers used at large electric power sta-
tions have hyperbolic cross sections (Figure 1 ); comet or other object moving with
a
more than enough kinetic energy to escape the sun's gravitational pull traces out one
branch of a hyperbola (Figure 2); and the long-range radio navigation system known
as LORAN locates a ship or plane at the intersection of two hyperbolas (Figure 3).

/ (!,'(//( / Ihperholic cooling lowers for Figure 2 Comet photographed through a Figure 3 Synchronized pulses transmit-
an cleclric power station telescope ted from three stations locate a ship at the
intersection of two hyperbolas plotted on a
LORAN chart.
SECTION 9.6 THE HYPERBOLA 571

DEFINITION 1

Figure 4

axis of symmetry
I

Figure 5
572 POLAR COORDINATES AND ANALYTIC GEOMETRY

this equation for y in terms of x as follows:

>•-
_ .r

\ a- I ^ a' ' ^ x~

Hence,

v=±I^K/i--.
provided that .v t^ 0. Since

lim , /
1
^= 1

we see that when x is large in absolute value,

y-± —
bx

The two lines whose equations are

Figure 6

Figure 7
SECTION 9.6 THE HM'ERBOLA 573

Figure 8 (0, b). and foci Fi = (0. — c) and Fj = (0. c). Using Theorem 1 but interchanging x
with V and interchanging a with b, we obtain the equation

where a = \ h-
b-

This equation is also called the standard form for the equation of a hyperbola. The
asymptotes are still given by

and V = x

(Why?)
Unlike the ellipse equations (jr/a") + {y'/b~) = 1 and (x^/b-) + (y-^/a^) = 1,
there is no requirement that a > b in the hyperbola equation {x^/a^) — iy'/b') = 1
or in the hyperbola equation (y-/h-) — (x'/a-) = 1.

EX.\MPLE 2 Which of the following hyperbolas have a horizontal transverse axis


and which have a vertical transverse axis?

(a) = 1 (b) (c)


^ ^- = 1
16 4 4

(d) (e) = 1
4 4

SOLUTION Whether the hyperbola has a horizontal or a vertical transverse axis


does not depend on the relative sizes of the denominators but depends on which
term is subtracted from the other. Consequently, (a), (b), and (c) have vertical
transverse axes, whereas (d), (e), and (f) have horizontal transverse axes.

Suppose that a hyperbola has a horizontal or vertical transverse axis and its cemer
is not at the origin. By translating the coordinate axes so that the new origin O is

at the center, we obtain an equation in standard form for the hyperbola with respect
to the XV coordinate system. Therefore, with respect to the original xy coordinate
system, an equation of the hyperbola with center C= (h. k) has the form

(.Y -h)- (y - k)- (y - k)- (X - h)-


= 1

a- b-

depending on whether the transverse axis is horizontal or vertical, respectively. In


either case, the asymptotes have the equations
574 POLAR COORDINATES AND ANALYTIC GEOMETRY

KXAMPI.K 3 Find the coordinates of the center, the foci, and the vertices of the
hyperbola y~ - 4jr - 8.v - 4v -4 = 0. Also find equations of its asymptotes, and
sketch its graph.

SOLUTION Completing the squares, we have


Figure 9
V- - 4v + 4 - 4(.v' + Ix + 1 ) = 4
' - 4»2 ^ 8.V - 4,i' 4=

(y- 2)- - Mx+ 1)- = 4


Dividing by 4, we obtain

(y-2)- (x+\f =
1

the equation of a hyperbola with center (


- 1 , 2) and with a vertical transverse axis.

Since a = 1 and b = 2. the equations of the asymptotes are

2 = — (.V + 1 and 2 = ~(.x+ 1)

that is,

V = Iv + 4 and y = -2v

Also, c = V?Tfo2 = V5, so Fi =(-1, 2 - V5), ^0 = (-1, 2 + VS), V, =


(-1, 0), and ^2 = (-1. 4) (Figure 9).

EXAMPLE 4 Simplify the equation 4.v- - 9v- - 24.x - 90y - 225 = by using a
suitable translation x =1 + h, y = y + k. Sketch the graph, showing both the old
.rv' and the new at coordinate axes.

SOLUTION Completing the squares, we have


Figure 10 - - = 225 + 36 - 225
4(.v' 6x + 9) 9(v- + lOv + 25)

4(.v -3f - 9{y + 5)- = 36


Dividing by 36, we obtain

(.V - 3)- (y + 5f
= 1

Thus, if we make the translation

V = V + 5

4a-2 - 9>'2 - 24,v - 90.1 - ;:5 = the equation simplifies to —= 1 (Figure 10).

EXAMPLE 5 The segment cut off by a hyperbola on a line perpendicular to the


transverse axis and passing through a focus is called a focal chord. Find a formula
for the length of a focal chord of the hyperbola (y^/fe") — (x'/ar) = 1

SOLUTION The hy perbola has a vertical transverse axis, and the upper focus is at

(0, c),c = Va" + b~. Putting y = c in the equation of the hyperbola and solving for
x, we obtain x = ±a'/h. The upper focal chord is therefore the line segment from
(-a^/b, c) to {a-/h, c); its length is 2a- /b.
SECTION 9.6 THE HYPERBOLA 575

Figure 11 EXAMPLE 6Two microphones are located at the points — c, 0) and (c, 0) on the x
(

axis (Figure 11). An explosion occurs at an unknown point P to the right of the v
axis. The sound of the explosion is detected by the microphone at (c, 0) exactly T
seconds before it is detected by the microphone at (-c, 0). Assuming that sound
travels in air at the constant speed of v feet per second, show that the point P must
have been located on the right-hand branch of the hyperbola whose equation is

(..Tr/a~) — (y/b-) = 1. where

(f.O)
vT V4c' v-7-
->
and

SOLUTION Let d\ and ^2 denote the distances from P to (— c. 0) and (c, 0),
respectively. The sound of the explosion reaches (— c. 0) in di/v seconds, and it

reaches (c, 0) in ^2/^' — di/v = T. so d\ — d2 = v7". Putting


seconds: hence, d\/v
a = vT/2. we notice that the condition d\ — d^^ 2a requires that P belong to a
hyperbola with foci F\ = {— c, 0) and Fj = (c, 0). By Theorei 1, an equation of 1

this hyperbola is ix^/a-) — iy'/b') = 1, where

vT V4c- - v-T-

2
and b = Vc- - a-
VMfT

Probiem Set 9.6

In Problems 1 to 8, find the coordinates of the vertices and the foci In Problems 13 to 20, find the coordinates of the center, the vertices,

of each hyperbola. Also find equations of the asymptotes, and and the foci of each hyperbola. Also find equations of the asymp-
sketch the graph. totes, and sketch the graph.

1 ^-= 1 = 1 13
(.V- 1)-
4

3 -^-
16
— =1
4 4 1
= 1

5 4.x- - 16y- = 64 6 49.V- - 16y- = 196 15

7 36v- - lO.v- = 360 8 V- - 4.V- = 1

In Problems 9 to 11, find an equation of the hyperbola that satisfies


the conditions given.

9 Vertices at (-4, 0) and (4, 0), foci at (-6, 0) and (6, 0)

10 Vertices at (0, -5) and (0, 2). foci at (0. -1) and (0. 1)

11 Vertices at (—4, 0) and (4. 0). the equations of the asymptotes


are y = —4X and y = f.v.

12 Determine the values of a" and b- so that the graph of the equa-
tion b'x- — a-y- = a'b- contains the pair of points (a) (2, 5)

and (3, -10) and (b) (4, 3) and (-7, 6).


576 POLAR COORDINATES AND ANALYTIC GEOMETRY

22 Show that the length of a focal chord of the hyperbola 38 It can be shown that the graph of the equation .rv = I is a hyper-
(.v-/a-) - {y-/b-)= I is 2b-/a. bola with center at the origin and with the .v and y axes as asymp-
totes (Figure 12). Find the coordinates of the foci of this hyper-
23 Find the length of a focal chord of the hyperbola x- - 8v" = 16.
bola. {Hint: The transverse axis makes a 45° angle with the

24 Find equations of the tangent and normal lines to each hyperbola x axis.)
at the point indicated.

(a) .V- - y- = 9 at (-5. 4) (b) 4v- - ,v- = 7 at (3. -2)


(c) X- - 4x - y- - 2y = at (0, 0)
Figure 12

(d) 9(x + I)- - I6(y - 2)- = 144 at (3, 2)

25 Find an equation of the hyperbola whose asymptotes are the


given lines and that contains the indicated point.

(a) y = -2.V andy = 2v, (I . I)

(b) y = -2v + 3 and y = Iv + 1.(1,4)


(c) y = + 5 and y = -Jt + 3, (2, 4)
.V

26 Find the volume V of the solid of revolution obtained by revolv-


ing the region inside the right-hand branch of the hyperbola
b'x^ — a'y' = a-b- between the vertex and the focal chord
about the v axis.

In Problems 27 to 3 use a suitable translation of axes to bring each


1 .
39 A hyperbola is said to be equilateral if its two asymptotes are
equation into the standard form for a hyperbola. In each case, sketch
perpendicular. Find an equation of an equilateral hyperbola with
the graph and show both the .vy and the at axes.
horizontal transverse axis and center at the origin. (Denote the
distance from the center to a vertex by a.)
27 3.V- -y- + llv-l- 8y = 7
40 Sketch the graph of the hyperbola
28 4.r- - 25y- + 24.v + 50y + 21 =
{y + bf
29 5y- - 9.V- + lOy + 54.v - 112 = = I

Ibp + p-
30 .V- - 4y- - 4.r - 8y - 4 =
Show that as i-^ +'^, the upper branch of this hyperbola ap-

31 4.r- - 9y- -t- Xbx + 54y = 29 proaches the parabola y = (l/4p).v".

32 A moves so
41 In Figure 13, hold the center C and the asymptotes fixed, but
point that it is equidistant from the point (2, 0) and
allow the foci F\ and Fi to move in toward the center. What
the circle of radius 3 units with center at (-2. 0). Describe the
happens to the hyperbola?
path of the point.

33 A point moves on the hyperbola Ax^ - 9y- = 27 with its ab-


scissa increasing at the constant rate of 6 units per second. How
fast is the ordinate changing at the point (3, I)?
Figure 13

34 A point moves so that the product of the slopes of the line seg-
ments that join it to two given points is 9. Describe the path of
the point.

35 Find the shortest (minimum) distance from the point (3, 0) to the
hyperbola y- - .t' = 18.

36 A region in the plane is bounded by the line .v = 8 and the hyper-


bola .x" - y" = 16. Find the dimensions of the rectangle of max-
imum area that can be inscribed in this region.
42 Give a proof of Theorem I

37 Lxt m be the slope of the tangent line to the hyperbola


- 43 Sound travels with speed s in air. and a bullet travels with speed
(x'/a-) (y-/b-) = I at the point (.v,,, \„). where .Vo > a and
/; from a gun at -h. 0) to a target at {h. 0) in the .rv plane. At
Vii > 0. Find lim m. and relate your answer to the asymptote
(

what points (.v, y) can the boom of the gun and the ping of the
)' = (b/a)x. bullet hitting the target be heard simultaneously?
SECTION 9.7 ROTATION OF AXES 577

9.7 Rotation of Axes


We have studied conies whose axes of symmetry are parallel to the coordinate axes
and found that their equations can be "simplified" and brought into standard form
by a suitable translation of the Cartesian coordinate axes. Notice that the translation
equations

X = X ~ h

y = y - k

are especially simple in the Cartesian coordinate system, whereas the conversion of
these equations to polar coordinates would introduce some complication. The Carte-
sian coordinate system is naturally adapted to translation — the polar coordinate
system is not.
Figure I On the other hand, the polar coordinate system is naturally adapted to rotation

P=
about the pole. Figure 1 shows an "old" polar axis, a "new" polar axis obtained by
(r, e)"new"
polar coordinates rotating the old polar axis about the pole through the angle <^, and a point P in the
P=(r,d) "old" plane. Evidently, if Pj= (r, 6) in the old polar coordinate system, then we have
polar coordinates
P = 6 - (f)) = (r, 6) in the new polar coordinate system. Thus, for polar coor-
^ "new" polar axis
(r,

dinates, we have the very simple rotation equations

Id" polar
axis

which give the new polar coordinates (r, 0} of the point P whose old polar coordi-
nates are (r, 6).

Figure 2 EXAMPLE 1 An equation of the circle of radius a > with center at the point
(a, 0) on the old horizontal polar axis is r = 2a cos 6 (Figure 2). Find an equation
"nc-w"
of the same circle with respect to a new polar axis making the angle </> = tt/2 with
the old polar axis.

SOLUTION Using the polar rotation equations

"old" polar axis

= e + (t>

we rewrite r = 2a cos 6 as 7 = 2a cos (6* -f </>). Putting </> = n/l and noticing that

cos (6 + (t>)
= cos ie + —j = -sin d

we see that an equation in the new polar coordinate system is 7 = —2a sin 6.

Rotation of Cartesian Axes


The rotation equations for Cartesian coordinates are not quite so simple as those for
polar coordinates. Nevertheless, it is often necessary to rotate Cartesian coordi-
nates,and we give the appropriate equations for such a rotation in the following
theorem.
578 POLAR COORDINATES AND ANALYTIC GEOMETRY

Cartesian Rotation Equations

Suppose that the new at coordinate system has the same origin as the old .vv

coordinate system, but that the new a axis is obtained by rotating the old .v axis

Figure 3 about the origin through an angle (p (Figure 3). Let the point P have old Cartesian
coordinates (x, y) and new Cartesian coordinates (.v, y). Then

.V = ^ cos </> —y sin <^

y = ^ sin <^ + y cos (f>

The proof of Theorem 1 is accomplished by converting from Cartesian coordi-


nates to polar coordinates, rotating the polar coordinate system through the angle <^,

and then converting back to Cartesian coordinates (Problem 28a).


07 The Cartesian rotation equations in Theorem 1 can be solved for a and y in terms
of X and y to obtain

+ y sin (h

(b + \ cos (

(see Problem 28b). The last two equations enable us to find the new xy coordinates
in terms of the old .\y coordinates and the rotation angle (J).

EXAMPLE The new xy coordinate system is obtained by rotating the old jr>'
2

coordinate system through —30°. Find the new xy coordinates of the point whose
old xy coordinates are (V3, 2).

SOLUTION Here we have

— 30
V3 V2x
X = X cos (-30 ) + y sin ( ) = x \-
y

and
+ V3
y = -V sin (-30°) + y cos (-30°) .(-!)., =
2/ .(^) • V 2 / 2

Thus,
V3V3 -2 1 V3 + V3(2) 3V3
~ and
2 2

Cf, y) = (l-^)
The General Second-Degree Equation
U A. B. C. D. E. and F are constants and at least one of the constants A, B, or C
is different from zero, the equation

Ax- + B.vy + Cy- + Dx + Ey + F =

is called the general second-degree equation in x and y. Notice that by taking


A = \/a-. B = 0, C= \lb-. D= 0, £= 0. and F= - 1 in the general second-
degree equation, we obtain

—r + ^-v= 1
SECTION 9.7 ROTATION OF AXES 579

the standard form for the equation of an ellipse. Also, if


we set A = 1, B = 0.
C= 0, D= 0. E= -4p. and F= 0. we obtain

X- = 4py
the standard form for the equation of a parabola. Finally,
by
'
putting A = \/a~
B = O.C= -\/b-. D = 0,E = 0,andF= -I, we obtain

the standardform for the equation of a hyperbola.


Every conic section considered until now has had a Cartesian
equation which was
a special case of the general second-degree equation

Av- + S.v\- + Cv- + D.v + Ey + F =


However, the coefficient B has always been zero, so the
"mixed" term fin- did not
show up. As we shall soon see. the mixed term can alwavs
be removed from the
general second-degree equation by a suitable rotation
of the coordinate axes.
Suppose that the old .xt coordinate system is rotated about
the origin through the
angle 6 to obtain a new .n- coordinate system.
If we substitute

X = .X cos <t>
-y sin tt>

and
y = X sin 4) + y cos <i>

from the rotation equations (Theorem I) into the equation

Av- + B.x}- + Cv- + D.v + £y + f =


and collect similar terms, we obtain

Ax' + Bxy + Cy- + Dx + Ey + F =


where the coefficients A. B. C. D, E. and F are given by

A = A cos- (f) + B cos (* sin d> + C sin- (b

^ = 2(C - A) cos 6sin d) + B(cos- 6 - sin- (^)

C = A sin- (^ - fi cos sin cf) + C cos- cb

D = D cos <f> +E sin (f>

E = —D sin <b + E cos (^

? = F

(Problem 39). Since


1 cos sin
and
cos 2rf> = cos- (j)
- sin- dt

it follows that B can be written in the alternative form

B = iC - A) sin 2(b +B cos la

Using the equations obtained above, we can now prove


the following important
theorem.
580 POLAR COORDINATES AND ANALYTIC GEOMETRY

THEOREM 2 Removal of the Mixed Term by Rotation

Suppose that fi 5^ in the general second-degree equation

Ax" + Bxy + Cy- + Dx + Ey + F =


If the coordinate system is rotated about the origin through the angle (t> deter-

mined by the conditions

A- C
< <
cot 2<l) and </)

then the mixed term B.vv will not appear in the new equation

AT- + Cy^ + Dx + ^y + F =

PROOF We have

cot 2<^ =
A-C or
cos 24> A — C
8 sin 2c/) B
Hence,
8 cos 2<A = (/I - C) sin 2({>

and it follows that

fi = (C - A ) sin 2</) -I- fi cos 2</) = -(A - C) sin 2</) 4- B cos 2</) =

EXAMPLE 3 Rotate the coordinate axes to remove the mixed term from the equa-
tion A" - 4xy + v^ — 6 = 0, and sketch the graph, showing both the old xy and the
new xy coordinate system.

SOLUTION The equation has the fomi Av" Bat Cy- + Dx + Ey + F = -I- -I-

with A = 1. = -4, C = 1, D = 0, £ = 0. and F = -6. According to Theo-


fi

rem 2, the mixed term can be removed by rotating the coordinate system through the
angle 4>. where < <f>
< tt/2 and

cot 2(b =
A-C 1-1
8 -4

Hence, we take 2(f)


= 7r/2, so that <f>
= 7r/4, sin </> = V2/2, and cos </> = V2/2.
Then, by the rotation equations.

— =
V2
x = -v cos (b \ s\n (b (.V - y)
2

V2
and V = X sm (b + V cos (.V + y)

Substituting these expressions for a and y into the equation x' - 4xy + y~ — 6 = 0,

we have
V2 ]- v'2 V2 v^ V
[—
r

(^-y)J
r
-4[^(.v-y)J[-^(A
1 r
+
y)J
1
+
r
[^(.v+v)J -6
The last equation simplifies to

h(x - y)- - 2(.v - y)(A + y) + h(^ + y)" = 6

or |(.v- - Ivy -I- y-) - 2(.v- - y-) + |(.v' + Zvv + v") = 6


Figure 4
582 POLAR c()()rdin\tf:s and analytic geomktry

EXAMPLE S \bx~ - lAxy + 9v- - 80.V - 190y + 425 =


SOLUTION Here A = 16, B = -24, C = 9, D= -80, £=-190, and
F= 425; therefore.

cot 2(f)
= cos 2<^
B 24

'
1 +(-A) 3
and sin (j)
=

Figure 6
SECTION 9.7 ROTATION OF AXES 583

We now have the means to sketch the graph of any second-degree equation in .y

and y. If the equation contains a mixed term, then a suitable rotation of the coordi-
nate axes will remove it. Then, by completing the squares if necessary, we usually
obtain the standard form for the equation of a circle, an ellipse, a parabola, or a
hyperbola. However, in some cases this does not happen (Problem 42). In these
exceptional cases, the graph is called a degenerate conic. The only possible degen-
erate conies are (1) the whole .rv plane, (2) the empty two lines,
set, (3) a line. (4)

and (5) a single point.


Assuming that the graph of a given second-degree equation in x and y is a non-
degenerate conic, it can be identified as follows;

1 The
584 POLAR COORDINATES AND ANALYTIC GEOMETRY

27 By rotating the coordinate axes through </> = 45° about the ori- (c) 2x^ + 2y^ -%x+ 12y +1=0
gin, show that the graph of .vv = 1 is a hyperbola with the v and (d) x^ - Ixy- 3y- + 21 =
y axes as asymptotes (Figure 7).
42 The following second-degree equations in x and y have graphs
Figure 7 that are degenerate conies. In each ca.se, verify that the graph is
as described.

(a) 0.V- + Qxy + Oy- + O.v + Oy + = (the whole .vy plane)

(b) x^ + y- + = (the empty set)


1

(c) 2x- - 4.vy + 2y- = (a line)

(d) 4.V- - y- + 16a' + 2y +15 = (two intersecting lines)

(e) x' - Ixy + y' — 18 = (two parallel lines)


(f) x- + y- - dx + Ay + 13 = (a single point)

43 Sketch the graph of Iv" + 3.\t + 2y- = 4.

44 Prove Theorem 3.

45 Inodd Problems 29 to 37, use the algebraic rotation invariants in

28 (a) Derive the rotation equations in Theorem 1 by converting to Theorem 3 to check for a possible error in your calculation.

polar coordinates, rotating the polar axis, and reconverting to 46 Assume that AC > Show
0. that the graph of the equation
Cartesian coordinates, (b) Solve the rotation equations in Theo- Ax' + Cy' + Dj; + Ey + F = is an ellipse, a circle, a single
rem I for X and y in terms of x and y. point, or the empty set.

1 In Problems 29 to 38, determine (a) the angle of rotation that 47 Derive equations of the following conies from the given infor-

removes the mixed term (0 < <^ < tt/1). (b) v and y in terms of a mation. Then rotate the coordinate axes so that the new 'x axis

and and (c) the new equation in terms of x and y. Then sketch the contains the foci of the conic. Find the equation of the conic in
V,
the new at coordinate system, (a) Foci at F| = (3, 1) and Fi =
graph showing both the old vy and the new xy coordinate systems.
(-3, —1); distance from the center to either vertex along the

29 .v' + 4.n- 2y- = 12 major axis is a = 4. (b) Foci at F, = (4, 3) and Fj = (-4, -3);
distance from the center to either vertex along the transverse axis
30 X- + 2.ty + y' -f .v -t- y = 31 "' + 2.n' + y= = 1
=
is a 3.

32 X- + Zxy + y- - 4V2r 4V2y =


-(-
48 Assume that AC < 0. Show that the graph of the equation

ii 9.V- - 24at -I- I6y- = 144 Ax' + Cy' + Da + £y +F= is a hyperbola or a pair of in-
tersecting lines.
34 2v= -I- 4\/3 xy - 2y^ - 4 = 35 6.x- - bxy + I4y' = 45
49 Find a second-degree equation in x and y whose graph consists
36 17.V- - I2.n + 8y- - 68.v + 24y - 12 = of a pair of lines intersecting at the origin and having slopes 3

-16 = and -3, respectively.


37 Zx- + 6xy- 6y^ + 2VlO .v + sVTo y
- 9y' - 5Zv = 50 Show that the graph of the equation Av" + Da + Ey + F = 0,
38 4.V- 12ry -I- -I- 26y -h 27
where A 5^ 0, is a parabola, a pair of parallel lines, a single line,
39 Derive or the empty set.
the equations on page 579 for A, B, C. D, E, and F in

terms of A, B, C, D, E, F, sin (j), and cos <^. [a 51 Sketch the graph of a' + 4.vy + 4y- = 8.

40 Suppose that Ax~ + Bxy + Cy^ -\-


Dx + Ey + F = Q is an 52 Show that lA' + B' + 2C; is a_rotation invariant in the sense
equation of a nondegenerate conic. Show that the conic is: that 2A- + B' + 2C- = 2A' + B' + IC^.

(a) A circle or ellipse if fi" - AAC < - + y' =


53 Sketch the graph of a" 9.\y 12.

(b) A parabola if fi- - AAC =


54 In Theorem 2, suppose that < <^ < 7r/2 and cot 2<^ =
(c) A hyperbola if B' - AAC > (A - C)/B. Prove that

Also show that in (a) the conic is a circle A = C and 6 = 0.


S\/(A -
'\i
~A = h[A + C+ O' + B-]
41 Assuming the facts given in Problem 40, identify the given
and ~C = i|A + C - 5\/(A - C)' + fi-|
conies without bothering to rotate the coordinate system. (As-
sume nondegeneracy.) where S = B/\B\.

(a) bx- - Axy + 3y- + .v + 1 ly +10 = 55 If B # 0, then show that the graph of the equation
(b) 18.v' + llrv + 2v- + .v - 3v - 4 = Ax' + fi.rv' + Cy- + Da + £v + F = cannot be a circle.
SECTION 9.8 ECCENTRICITY AND CONICS IN POLAR FORM 585

9.8 Eccentricity and Conies in Polar Form


In Sections 9.4 to 9.6, we derived equations in Cartesian form for tlie ellipse, the
parabola, and the hyperbola from three special geometric definitions — one for each
type of curve. By using the idea of eccentricity', it is possible to give a unified
geometric definition for these curves. In this definition, f is a fixed point in the
plane, D is a fixed line in the plane, and F does not belong to D.

DEFINITION 1 Conies in Terms of Eccentricity, Focus, and Directrix

Let e be a fixed positive number. A conic with eccentricity e. focus F, and


Figure 1 directrix D is the set of all points P in the plane such that the distance from P
to F. divided by the distance from P to D, is equal to e.
e = eccentricity

Thus, the point P belongs to the conic if and only if

\PF\

where Q is the foot of the perpendicular from P to D (Figure 1 ). Although we use


the same symbol e for both the eccentricity of a conic and the base of the natural
logarithm, no confusion should result —
you can always tell from the context what is
intended.

EXAMPLE 1 Find the equation in Cartesian form of the conic with eccentricity
e = 2 whose focus is at the origin and whose directrix is given by D: .r = — 3.
Figure 2 SOLUTiqN_ From Figure 2, the point P = (.v, y) belongs to the given conic if and
only if \PF\/\PQ\ = 2; that is.

V.x- + V-
Vx- + V- = 6 + Zv

Squaring both sides of the last equation, we have


x^ + y- = 36 + 24a: + 4x-

or
3x^ + 24v - y2 = -36
Completing the square, we obtain

3(.v- + 8.V + 16) - V- = 3(16) - 36

3(.v + 4)- - y- = 12

that is,

(x + 4)- y2

4 12

Therefore, the conic is a hyperbola with center at (-4, 0).

By an argument similar to that in the example above, it can be shown that a conic,
defined as in Definition 1 , is an ellipse, a parabola, or a hyperbola. In fact, we have
the following theorem, whose proof is left as an exercise (Problem 56).
586 POLAR COORDINATES AND ANALYTIC GEOMETRY

THEOREM I Conic Theorem

Suppose that a conic with focus F at the origin and directrix D: x = -d has
eccentricity e. where c and d are positive. Then exactly one of the following

holds:

Case (i) e <A, and the conic is an ellipse with the equation

^^^+4 = a- h-
1

ed
where a b = -, and c = Vfl" — h~ =

Case (ii) e = 1 . and the conic is a parabola with the equation

d
4p{.\ + p) = y- where P -
^
Case (iii) e > 1 . and the conic is a hyperbola with the equation

U + cf V- _ J

a~ b-

ed ed
where a = -, b and (• = Vfl- + b~
Ve- - 1

Figure 3

Figure 4
SECTION 9.8 ECCENTRICITY AND CONICS IN POLAR FORM 587

Hence, we lose no points on the conic by writing its equation as

d + r cos 6

Solving for r, we obtain a polar equation for the graph in Figure 3:

ed
1
- e cos

Because cos {
— d) = cos 6. it follows from the test for symmetry (page 542) that
the graph is symmetric about the polar axis.

The Ellipse in Polar Form


If < e < 1 , the conic
ed
1
— e cos 6

is an ellipse (Figure 5). For such an ellipse. /• takes on its maximum value ed/(l — e)
when 6 = 0. whereas it takes on its minimum value ed/{l + e) when 6 = tt (Prob-
lem 33). Thus, the points with polar coordinates

V, and v. =
\ 1 + ^ / \ 1
- ^ '

are the vertices of the ellipse at the ends of its major axis. It follows that the
semimajor axis a of the ellipse is given by

ed ed
a = jlV^^il = y(|v7l + =
l^2|)
y 1 + e 1 - e

Simplifying the last expression, we find that

ed

(Figure 6a). Similarly (Figure 6b), the distance c between the focus F and the center
C of the ellipse is given by

ed ed ed
= \FC\ = |ViC| - |V,F| =a -
\ + e I - e- 1 + e

Simplifying the last expression, we find that

Figure 6
588 POLAR COORDINATES AND ANALYTIC GEOMETRY

To find the remaining two vertices of the ellipse

ed
-
Q<e<\
1 e cos

consider the angle

Figure 7

rj = (fl,e„i

Figure 8

Figure 9

tirst

directrix
SECTION 9.8 ECCENTRICITY AND CONICS IN POLAR FORM 589

Figure 10

polar ;

Figure 11
590 POLAR tOORDINATES AND ANALYTIC GEOMETRY

Figure 14 EXAMPLE 3 Find a polar equation of the hyperbola of eccentricity e = j. one of


whose foci Ft is at the pole with the corresponding directrix Di perpendicular to the

polar axis and J = 3 units to the left of the pole.

ed 1(3) 15

1 — e cos 1
- T cos fl 4-5 cos

Alternative Polar Forms for Conies


Figure 15
If the focus F of a conic of eccentricity e is placed at the pole O and the directrix D
is placed to the right of the focus and perpendicular to the polar axis (Figure 14),
then a polar equation of the conic is

ed
1 + e cos e
= F polar axis

(a) (Problem 49). Likewise, if the directrix D is parallel to the polar axis, with the
focus F still at the pole O. then a polar equation of the conic is
polar axis

ed
1 ± e sin

where the plus sign is used if the directrix is d units above the polar axis
(Figure 15a) and the minus sign is used if the directrix is d units below the polar axis
(Figure 15b). We leave the derivation as an exercise (Problem 50).

Z\ 3
B EXAMPLE 4 Identify the conic r . and sketch its graph.
Figure 16 1 + V3 sin

SOLUTION The polar equation has the form

ed
\ + e s\ne

with e = V^ and d = 2; hence its graph is a conic with eccentricity V3, focus
F at the origin, and directrix D parallel to the polar axis and d = 2 units
above it. Because e > I. the conic is a hyperbola. Vertex V^ is ed/(\ + e) =
2\/3/(l + V3) == 1.27 units above O, and vertex V, is -ed/(\ - e) =
ed/{e - 1) = 2V3/(V3 - 1) = 4.73 units above O (Figure 16).

EXAMPLE 5 Find a polar equation of the parabola with focus F at the origin and
Figure 17 with directrix D perpendicular to the polar axis and 2 units to the right of the origin
(Figure 17).

SOLUTION The equation will have the form

ed
1 + e cos 9

with (' = 1 and (/ = i- Thus, the desired equation is

1 + cos 2 + 2 cos
SECTION 9.8 ECCENTRICITY AND CONICS IN POLAR FORM 591

Problem Set 9.8

In Problems 1 to 6, find a Cartesian equation of the conic whose


focus F is at the origin and whose eccentricity and directrix are
given. Sketch the graph.

1 Eccentricity e = g, directrix x = — 2.

2 Eccentricity e = |, directrix a:
= — 5

3 Eccentricity e = 1, directrix x = -4

4 Eccentricity e = 1 , directrix x = — 3

5j Eccentricity e = 2, directrix .r
= —3

6 Eccentricity e = V5, directrix .x = -

El In Problems 7 to 10, show that the graph of each polar equation is an


ellipse; find the eccentricity, the directrices, the center, the foci, and
the vertices of the ellipse; and sketch the graph.

20 10
7 r = 8
4 - 3 cos e

20
9 r =
592 POLAR COORDINATES AND ANALYTIC GEOMETRY

41 Fill in the details of the argument on page 589 to show that 51 If the two directrices of an ellipse are held fixed and the eccen-
( = Of. tricity is decreased, what happens to the shape of the ellipse?

42 Show that d = c - (a'/c). 52 If the two directrices of a hyperbola are held fixed and the eccen-
tricity is increased, what happens to the shape of the hyperbola?
43 Show that ec - ed = a.

53 Find all points of intersection of the hyperbola


44 Show that the distance between the two directrices is lir/c
- 4
r = 2/(1 - 2 cos 6) and the lima^on r = 2 cos 0. Sketch
units.
the graphs.

45 Show that e is the ratio of the distance between the two foci to
54 Let ()<(?< 1 , < d. and < a < h. Find the condition or
the length of the transverse axis.
conditions on e and d such that the left vertex of the ellipse

46 Derive a formula for e in terms of c and d. r = ed/(\ - c cos 0) lies on the inner loop of the limagon
r = a - b cos 0.
El 47 Except for minor perturbations, the orbit of the earth is an ellipse
with the sun at one focus. The least and greatest distances (at 55 Find Cartesian equations of the two directrices of the ellipse

perihelion and aphelion) between the earth and the sun have a (.r/a-) + Cv7*") = I. where a > fc > 0.

ratio of approximately ^. Find the approximate eccentricity of


> O
56 Let the focus F of a conic with eccentricity e be the origin
the earth's orbit.
of a Cartesian coordinate system, and suppose that the directrix

48 Suppose that the foci of a particular ellipse lie midway between is D: X = -d, where d > 0. Using Definition 1 directly, prove

the center and the vertices. Find the eccentricity. that (1 - e')x- — {2e'd)x + y" = e'd' is a Cartesian equation of
the conic. Using this result, prove Theorem 1.
49 Derive a polar equation of a conic with focus at the pole and
directrix perpendicular to the polar axis and d units to the right of 57 Find Cartesian equations of the two directrices of the hyperbola
the pole. {x~/a-) - (y'/b-) = 1, where a and b are positive constants.

50 Derive a polar equation of a conic with focus at the pole and 58 In Problem 57. show that the eccentricity of the hyperbola is

directrix parallel to the polar axis. given by e = Vl -I- {b/a)'.

Review Problem Set, Chapter 9


In Problems I to 6, plot each point in the polar coordinate system, 17 (-17, 17) 18 (0. -1)
and then give three other polar representations of the same point for
which (a) r < and s e < 27r, (b) r > and -27r < 6 < 0, and In Problems 19 to 22, convert each polar equation into a Cartesian
(c) r<0 and -2tt ^ 6* < 0. equation.

1 (1, 7r/3) 2 (2, 577/6) 19 r cos 26 = 1

3 (2, 77r/4) 4 (-1. -1177/3)

5 (3, 77) 6 (77. 77)

In Problems 7 to 12, convert the polar coordinates to Cartesian coor-

dinates. =
22 r
-
.0<e<\.d>0
1 e cos 6
7 (0, 0) 8 (17, 0)
In Problems 23 to 26, convert each Cartesian equation to a polar
9 (17, 77) 10 (-3, 77/6)
equation.

11 (II, 377/4) 12 (-3, -77/3)


23 v = Iv - 1 24 (.V- 1)- + (V- 3)- =4
In Problems 13 to 18. convert the Cartesian coordinates to polar

coordinates (r, 6) with r s and — tt < s 77. 25 V- = 4.V 26

13 (17, 0) 14 (2. 3)
In Problems 27 to 32, sketch the graph of each polar equation.
15 (2, -2V3) 16 (-VI, -1) Discuss whatever symmetry the graph may possess.
REVIEW PROBLEM SET 593

27 r = 2 COS 3e 28 r = 2 cos (30 + tt) 49 Inside the circle r = 4 sin and below the line r = esc

50 Inside the circle r = 6 sin and outside the cardioid


29 r = 30 r = 5 + sin e -
r = 2(1 sin 0)
cos + V3 sin 6

= 3
= 51 Ub> a > 0. find the area of the region inside the outer loop
31 r 32 r 1 + ^ sin
+ sin but outside the inner loop of the limaqon r = a + b s.in 6.

52 Find the area inside the lemniscate r= a' cos 20.


In Problems 33 to 36, find the slope of the tangent line to the polar

graph at the point indicated.


In Problems 53 to 56, find the arc length of the portion of each polar
curve between the indicated values of 0.

33 r = at (i 0)
5 cos — 3 sin = between
53 r e""* for and 277
34 r = a s\n kd. a> 0. k a positive integer at (0, n/k)
54 r = 5 sin for between and tt

35 r = 2 - 3 cos at (2. i7/2)


55 r = COS" (0/2) for between and 7r

36 r^ = -9 cos 16 at (3. 7r/2)


= -
56 r 1 COS for between and 277

37 Find all points where the tangent to the limagon


In Problems 57 to 62, identify the type of conic, give its eccen-
r = 3 - 4 cos is either horizontal or vertical.
tricity, and specify the position of the directrix corresponding to

38 In Problems 35 and 36. find tan di at the indicated points. the focus at the pole.

39 Find tan tp in terms of 6 for the curve r = sin' 6.


17

40 Let f 7^ O be a point of intersection of the two polar curves


-' 1.

r =f(0) and r = g(0). If i//, is the value of i/f for r = fid) at P


and i/'T is the value of ip for r = g{0) at P, show that

tan ii(|
- tan l^:
tan <f)

1 + tan i/ji tan 1//2

where 4> is the angle between the tangent to r = /( 0) at P and


the tangent to r = giO) at P.

41 Find the polar coordinates of all points of intersection of the


limagon r = I — 2 cos 6 and the parabola r = 2/(1 - cos 6).

Sketch the graphs showing these intersection points.

42 Find the polar coordinates of all points of intersection of the


circle r = 2 cos 6 and the four-leaved rose r = 2 cos 26.

In Problems 43 to 48. find the area enclosed by each polar curve


between the indicated values of 6.

43 r = 1 + cos e from 6 = to 6 = ir/4

44 r = -0 from = to = 7r/2

45 r = 4 sin from = to = tt

46 r = from = to = 7r/6
sin + cos

47 r = 1 - cos from = to = tt

4S r = e-" from = to = tt

In Problems 49 and 50, find all points of intersection of the two


curves, sketch the two curves, and find the area of each region
described.
594 POLAR COORDINATES AND ANALYTIC GEOMETRY

72 3x^ + 4r - 2&X - 16y + 48 = In Problems 89 to 94. find a Cartesian equation of the hyperbola that
satisfies the conditions given.
73 9*^ + 4v- + 72r - 48v + 144 =
89 Foci (-5, 0) and (5. 0): length of transverse axis is 4.
74 Find equations of the tangent and normal lines to the ellipse
r+ 3y^ = 21 at the point (3. -2). 90 Vertices (3. -6) and (3. 6); foci (3, -10) and (3. 10).

75 At what point(s) on the ellipse \6r + 9v" = 400 does v de- 91 Vertices (-2, 3) and (6. 3); one focus (7. 3).

crease at the same rate as x increases? with equations of asymptotes


92 Containing the point (1. 1);

76 The lower base of an isosceles trapezoid is the major axis of an y = -2r and y = 2r.

ellipse; the ends of the upper base are points on the ellipse.
93 Center (0. 0): transverse axis on the y axis: length of focal
Show that the length of the upper base of the trapezoid of maxi-
chord 36; distance between its foci 24.
mum area is half the length of the lower base.
94 Center (0, 0); one focus (8, 0); one vertex (6. 0).
77 Reduce each of the following equations to an equation of an

ellipse in a "simpler" form by using a suitable translation.


In Problems 95 to 99, find the coordinates of the center, the vertices,
(a) 16r' + >•- - 32x + 4y - 44 =
and the foci of each hyperbola. Also find the eccentricity and the
(b) 9x^ + 4v2 H- 36x - 24>' - 252 =
equations of the asymptotes. Sketch the graph.

78 A point P moves ^that the product of the slopes of the line


95 .r - 9y- = 72 96 y" - 9.r = 54
segments PQ and PR. where fi = (3. -2) and /? = -2, ), is (
1

—6. Find a Cartesian equation of the curve traced out by P. and 97 x^ - 4y- + 4x+ 24y - 48 = 98 16.x- - 9y- - 96.r =
sketch it.

99 4y- - .r - 24y -I- 2x -I- 34 =


In Problems 79 to 82. find a Cartesian equation of the parabola

satisfying the given conditions. Sketch the graph. 100 Find equations of the tangent lines to the hyperbola
x^ - y- + 16 = that are perpendicular to the line

79 Vertex (0, 0). focus on the .v axis, and containing the point 5.V + 3y- 15 = 0.

(-2, 6).
101 Find equations of the tangent and norma! lines to the hyperbola
80 Containing the points (-2. 1), (I, 2). and (-1. 3) and whose .X- - 8y- = 1 at the point (3. 1).

axis is parallel to the axis. - -


.v
102 Find the points on the hyperbola x^ y- 16 = that are

81 Vertex (2, 3), containing the point (4. 5). and whose axis is nearest to the point (0, 6).

parallel to the v axis. -


103 A point moves on the hyperbola x^ 4y- = 20 in such a way
82 Focus (6, -2) and whose directrix is the line .v — 2 = 0. that .V is increasing at the rate of 3 units per second. Find the
rate at which y is changing when the moving point passes
83 Find a Cartesian equation of the circle containing the vertex through (6, -2).

and the ends of the focal chord of the parabola y~ = 8.v.


104 Two points are 800 meters apart. At one of these points the

84 Find equations of the tangent and normal lines to the following report of a cannon is heard 1 second later than at the other.

parabolas at the point given. Show that the cannon is somewhere on a certain hyperbola, and
write an equation for the hyperbola after making a suitable
(a) y- + 5.r = 0at {-5, 5) choice of axes. (Consider the velocity of sound to be 332 me-
(b) 4y = 8-1- 16.v-.r at (1. T*) ters per second.)

85 Find the dimensions of the rectangle of maximum area that can EI105 The Ellip.se in front of the White House in Washington. D.C.,
be inscribed in the segment of the parabola .r = 24y cut off by has a semimajor axis of 229 meters and a semiminor axis of
the line y = 6. 195 meters. What is its eccentricity?

86 Show that the line from the focus of the parabola y" = 24.v to 106 For a main cable in a suspension bridge, the dimensions H and
the point where the tangent to the parabola at the point (24, 24) L shown in Figure 1 are called the sag and the span, respec-
cuts the y axis is perpendicular to the tangent. tively. If the weight of the suspended roadbed (together with
that of the cables) is uniformly distributed horizontally, the
87 At what point on the graph of the parabola y = 4 + Ix - x^ is
main cables form parabolas, (a) Taking the origin of an .vy
the tangent line parallel to the line 2x + y - 6 = 0?
coordinate system at the lowest point of a parabolic main cable
88 A parabola whose axis is parallel to the y axis contains the with sag H and span L. fmd an equation of the cable, (b) Using
origin and is tangent to the line x - 2y = 8 at the point (20, 6). the result in part (a), find the length of the cable. S (c) For the
Find an equation of the parabola and sketch the graph. Ambassador Bridge across the Detroit River, the sag is W= 63
REVIEW PROBLEM SET 595

meters and the span is Z- = 564 meters. Find the length of one round a central region in the shape of an ellipse with a major
of the main cables in the Ambassador Bridge. axis 148 meters long and a minor axis 84 meters long. Find the
area of the central region.

Figure 1 108 We know that the eccentricity e = c/a < 1 for the ellipse
(x^/a^) + (y~/b~) = I, where c^ = a^ - b-. Now, as e comes
closer and closer to zero in value, cja = e approaches zero;
hence, c-jcr approaches zero, so that a approaches b in value.
(Why?) What is the shape of an ellipse in which a and b are
close in value? What if a = W. Does this give an indication of
how to define a conic with eccentricity e = 0? Use sketches
and examples to answer these questions.

109 Find the equation into which the polar equation r- sin Id = 1
is transformed by rotating the polar axis through 45°.
107 The Yale Bowl, completed in 1914, is a football stadium mod-
eled after the Colosseum in Rome. The concrete stands sur- 110 Find the equation into which the Cartesian equation
wx + Vj = 2 is transformed by rotation of the axes through
an angle of 45° and elimination of radicals. Is the graph of this
equation a parabola?

In Problems III to 116, rotate the axes to transform each equation


to an equation that has no mixed term. El Sketch the graph, showing
both the old and the new coordinate axes.

Ill 3.r -h 4V3xy-r = 15 112 4.r -I- 4.rv + 4v- = 24

113 VZir + xy= II 114 r -I- xy - 3.V = 7

115 9.r + 16y- + 54.r - 32y = 48

116 r- - 6.X-V + 9v- + .r - 3v = 4

In Problems 1 17 to 121 , use the invariant B- - AAC to identify each


conic. You may assume that the conic is nondegenerate.

117 13.r -I- (iV3xy + ly- = 16 118 r + bxy + y- -1-8 =


119 Ix^ + 2V3.vy + 5v= -(-
I = 120 r- - 3xy + 5y- = 16

Yale Bowl 121 8lA- - 18xy -H 5;c -I-


r = 41
INDETERMINATE FORMS,
IMPROPER INTEGRALS, AND
TAYLOR'S FORMULA

Since the two basic notions of calculus — the derivative and the definite integral
are defined in terms of limits, it should come as no surprise that derivatives and
integrals can often be used to help evaluate limits. In this chapter we see how certain
limits, whose values are not obvious by inspection, can be found by using deriva-
tives according to special rules called L'Hopital's rules. We also use limits to study
the behavior of improper integrals. The chapter ends with an investigation of Tay-
lor's formula, which can be used to approximate certain functions by polynomials.

10.1 The Indeterminate Form 0/0


From our earlier study of the properties of limits, we know that

lim fix)
fix)
lim
a g(x) lim g(.v)

provided that lim /(.v) and lim g{.x) both exist and that lim gix) is not zero (Prop-

erty 7, page 61). If lim g{.x) = 0, this rule simply does not apply, and special

methods, which we introduce in this section, must be used to find the limit — if it

exists.

To begin, note that if the limit of a quotient exists and the limit of the denomina-
tor is zero, then the limit of the numerator must also be zero. Indeed, if lim gi.x) =
and lim [f(.x)/g{.x)] exists, we have

lim
.^«
/(.v) = lim ^^^(a) =
Lv-a
lim ^^ [lim
x-a
g(.x)] = lim ^^ -0 =
x^,.lg{x) J g(jc)J L,-<, g(.v)J

596
SECTION 10.1 THE INDETERMINATE FORM 0/0 597

Therefore, in studying the problem of finding limits of quotients whose denomina-


tors approach zero, we need to consider only quotients whose numerators also
approach zero. Thus, we are led to the following definition.

DEFINITION 1 Indeterminate Form 0/0


598 INDETERMINATE FORMS. IMPROPER INTEGRALS, AND TAYLORS FORMULA

Notice that fix)/g'{.x) is not the derivative offix)/g{x). In using L'Hopital's rule,
you obtain the traction /'(.v)/j:;'(.v) by separately differentiating the numerator and
denominator of the fraction /(.v)/,i,'(.v). For instance,*

lim
sin .V
= lim —D, sin .v
= lim
cos .v
= 1

In Examples I and 2. use L'Hdpital's rule to find the given limit.

X- - 6.V + 9
EXAMPLE 1 lim — -
-«-.i .V- 7.V + 12

SOLUTION Both numerator and denominator approach zero as .v ^ 3, so the

fraction has the indeterminate form 0/0 at 3. By L'Hopital's rule, we have

lim
v-3
—x":;
- 6x +
- 7.V +
9
12
= lim
.v-3
D,(.v-
^
- 6.V +
- 7.V +
9)
= lim
.v-3
— --
2.v

Iv
6
7
=
-
=
.V- D,,(.v- 12)

EXAMPLE 2 lim
-0 In (x + 1

SOLUTION The fraction has the indeterminate form 0/0 at 0, so

— —
lim
e* e~'
= lim — Oj(e'
'
e~')
= lim
e^ + e"*
^0 In U+ 1) v^o D, In (.v + 1) v^o l/(.v + 1)

= lim (.Y + Die' + e^') = (0 + l)(f" + e°) = 2

Sometimes an application of L'Hopital's rule to an indeterminate form produces a


new indeterminate form. When this happens, a second application of L'Hopital's
rule may be necessary; in fact several successive applications of the rule might be
required to remove the indeterminacy. (However, there are cases in which the
indeterminacy stubbornly persists no matter how many times L'Hopital's rule is

applied — see Problem 36 for an example.)

V"
EXAMPLE 3 Evaluate lim
*ti 1 — cos 2y

SOLUTION The fraction has the indeterminate form 0/0 at 0. Applying


L'Hopital's rule, we have

lim
t^o 1 -
X-

cos Ix
= lim
D^x^
*^o Dx{\ - cos
— r
Ix)
lim ———;— = am —
— = v^o 2v
2 sm 2v

>— sm 2x
,.

o
.t

*A rigorous application of L'Hopital'.s rule requires that we first establish the existence of

lim —
'-« g'(x)

before writing

Iim = am — ;

>-' ,i;(.v) .--" g'ix)

However, in the interest of brevity, it is customary to proceed more infonnaliy.


SECTION 10.1 THE INDETERMINATE FORM 0/0 599

But the fraction .v/sin 2v still has the indeterminate form 0/0 at 0: hence, we use
L'Hopital's rule a second time to obtain

D,.v
lim = lim lim
--(I 1
- cos 2y >— sin Iv >— u D, sin Iv -0 2 cos Iv

In usingL^HopitaFs rule repeatedly as above, you must make certain that the rule
remains applicable at each stage— in particular, you must check that the fraction

under consideration really is indeterminate before each application of L'Hopital's


rule. A common error is illustrated by the following incorrect calculation:

3.V-
hm lim
'1
6.V

2v - 1
lim
.r-.l
—2 = 3?
Here, the first step is correct, but the second is not! (Why?) In fact.

3.V- - 2v - 6.V
lim = lim
H 2v

L'Hopital's rule may be extended in various ways; for instance, it works for
one-sided limits as x ^ a^ or as x -^ a~ As we . shall prove (Theorem 3), it also
works for limits at infinity; that is, for limits as .v -* +x or as .v ^ -oc.

sin (5/.r)
EXAMPLE 4 Evaluate lim
2/.V

SOLUTION Here. Hm^ sin (5/a) = sin [ lim (5/x)] = sin = and

^^^ ^-/-^'^ ^ ^- Hence, the fraction has the indeterminate form 0/0 at +3^. Apply-

ing L'HopitaFs rule, we have


sin (5/a-) ,. D, sin (5/.y) (-5/x-) cos (5/x)
lim :- = lun lim
2/x DA2/X) -2/x-

= lim
.T^+x 2
— cos — ^ cos =
5
lim cos
X

Cauchy's Generalized Mean-Value Theorem and


a Proof of L'Hopital's Rule

To prove L^Hopital's rule, we need the following generalized version of the mean-
value theorem (Theorem 3. page 169), which is attributed to the French mathemati-
cian Augustin Louis Cauchv (1789-1857).

THEOREM 1 Cauchy's Generalized Mean- Value Theorem

Let /and g be functions that are continuous on the closed interval [a, b] and
differentiable on the open interval (a, b). and suppose that g'(x) ^ for all
values of .V in [a. b). Then there is a number c in the interval (a. b) such that

m
gib)
~f(a)
- g(a)
f'(c)

g'ic)
600 INDETERMINATE FORMS, IMPROPER INTEGRALS. AND TAM.ORS FORMULA

PROOF Notice that if ^Ui) = gib), then by Rolle's theorem (Theorem 2, page 168), there is

a number x in (a. b) such that ^'(.v)= 0, contrary to the hypothesis of the theorem.
Therefore, g(a) ¥^ g(b}. so that g(b) - g(a) ^ 0. Define a function K by the equa-
tion

K{x) = \f{b) -f{a)]gM - [gib) - gia)\Jlx) for .v in \u. b]

Evidently, K is continuous on since/and g are continuous on [a.


[a. b\ b\. Like-
wise, since /and g are differentiable on (a, b). so is K: furthermore,

K'ix) = Ifib) -f(a)]g'(x) - {gib) - giu)]f'ix) for .v in {«, b)

Now,

Kib) = [fib) -fia)]gih) - [gib) - gia)]fib) = fib)gia) - fia)gib)

Kia) = [fib) -fia)]gia) - [gib) - gia)]fia) = fib)gia) - fUDgib)

and it follows that Kia) = Kib). Therefore, we can apply Rolle's theorem to the
function K and conclude that there exists a number c in the open interval (a, b) such

that K'(c) = 0; that is,

= [fib) -fia)]g'ic) - [g(b) - g(a)]f'ic)

We have seen that gib) - g(a) ¥= 0; hence, since g'ic) # by hypothesis, the last

equation can be rewritten as

fib)-fia) ^ /'(c)

gib) - gia) g'ic)

and the theorem is proved.

Notice that if gix) = x, then g'(.v) = 1 and the conclusion of Theorem 1 reduces
to the conclusion of the original mean- value theorem.

EXAMPLE 5 If /(-v) = x^ + 12 and gix) = x' - 2, find a number c in the open


interval (0, 2) such that

/(2)-/(0) ^ f'ic)

g(2) - giO) g'ic)

SOLUTION Theorem 1 shows that such a number c exists but does not tell us
how to actually find it. To find c, we calculate as follows: /(2) = 20, /(O) = 12,
g(2) = 2, g(0) = -2, f'ic) = 3c-, and g'ic) = 2c. Therefore, the required con-
dition is

20-12 3c- 8 3c

2 - (-2) 2c 4 2

It follows that c = 3.

We now state and prove L'Hopital's rule for the case in which .v approaches a
from the right. An analogous result holds for the case in which x approaches a from
the left (Problem 33). and the two results, taken together, provide the justification
for our original informal statement of L'Hopital's rule.
SFXTION 10. 1 THE INDETERMINATE FORM 0/0 601

THEOREM 2 L'Hopital's Rule for the Indeterminate Form 0/0 as x

Let the functions/ and g be defined and differentiable on


an open interval (a, b),
and suppose that g(x) ¥^0 for a<x<b. Assume that lim^ f{x) = o]

^lim^ g{x) = 0, and g'{x) t^ for o < .v < b. Then, if lim ^^^ exists, so does

^-«* g{x) v^a* g'ix)

PROOF Define functions F and G as follows:

(/(.v) if a < < a < <


FM = .V fe

G(.v) = .?(v) if A- Z?

if A = a if A = a
for all values of a in [a, b). Notice that F coincides with/on the open interval (a, /;);
hence, F is differentiable on (a, b), and F'{a) =/'(a) for a<x<b.h follows that
F is continuous on (a, b) and, since lim^ F(a) = lim^ /(a) = = F(a), F is actu-

ally continuous on [a, b). Likewise, G is continuous on [a, b) and differentiable on


(a, b) with G'(x) = g'Cx) for a <a< /?.

Now, choose any number a in the open interval (a, b), and notice that F and G
are continuous on the closed interval [«, a] and differentiable on the open interval
(a. A). Furthermore, for any number t in (a, a), G'(t) = g'{t) ¥^ 0. Applying the
generalized mean-value theorem (Theorem 1) to the functions F and G on the
interval \a, a], we conclude that there is a number c in the open interval {a \) such
that

G(a) - G(a) G'{f) ^(A) g'(c)

Notice that o < c <a and that the value of c might depend on the choice of a.
Evidently, as a approaches a from the right, then c—
which is "trapped" between a
and A — must also approach a from the right. Thus,

lim,
V-"
/W =
-—-
g{x)
hm^
x^a*
,.
— —=
/'(f)

g'{c)
,.
lim
c^a*
/'(c)
^^-^-^
g' {c)
lim
fix)
g'ix)

and the proof is complete.

The following theorem shows that L'Hopital's rule is also effective for limits at
mfinity. Weand prove the theorem only for the case a -* -l-oo and leave the
state
statement and proof of the analogous result for the case v -oo as an exercise ^
(Problem 38).

THEOREM 3 L'Hopital's Rule at Infinity

Let the functions /and g be defined and differentiable on an


open interval of the
form it +00), where /: > and ^(a) t^ for a > k. Suppose that lim /(a) = 0,

-
lim
'
g{x) = 0, and ^'(a) 5^ for a > k. Then, if lim
V^ + CC
^^
g'(x)
exists, so does

lim and
C- + OC g(x) Iim = hm
:^ + =» g(x) .V-. +- g\x)
602 INDETERMINATE FORMS, IMPROPER INTEGRALS, AND TAYLORS FORMULA

We put / = l/x{oTx> k. noting that .v = \/t. that < <


/ \/k, and that / ^ 0"^ as

.V ^ +00. We define functions F and G on the interval (0, l/k) by the equations

F(t)=f[—) and CAl) = }i(-y) for 0< < /

Notice that lim F{t)


I— U'
= lim
(—O' '
/'(
\
—I
)
/
= lim
.»—• + «
/(a) = 0. Similarly, lim G{t)
I— o"
= 0.

By the chain rule, F and G are differentiable on (0, 1/A) and we have

F'U) = {-r-)f'[—) and G'U) = (--r -)/( — )


for < < f

Applying Theorem 2 to the functions F and G on the interval (0, 1/A), we have

,.
lim
'-0' G{t)
Fit)
= ,.
lim
'-<)•
—G
F'{i)
;

U)

Hence,

/(
lim
,-.+=0 g^x)
= lim
,^0' 1
N^
= lim
,-,0- G(n
= lim
'-0*
—G'(0 ;

41) '-

r-
= lim = lim = lim —
>^ + « g (JC)

and the proof is complete.

Problem Set 10.1


e' - e ' - 2 sin r
In Problems 1 to 22, use L'Hopital's rule to evaluate each limit. 15 lim 16
4?
jc + sin 2a: sin x — 1
In (sin -v)
1 lim 2 lim m T
— jT — sin 2x -V2 (7r/2) - X nil {-n - 2x)-

2x^ + 3.V - 2 3.r- + 5.V - 3 sin (J/x)


3 lim 4 lim 19
—2 3X-' - X X- + x-2 lim
5/x

Vx cos X — COS 3.V -


1 cos (2/.V)
5 lim 6 lim
^(1' sin V^ sin x~ •+=« tan (3/.V)

sin / — 1 .te^' - X
7 8 lim
; —lim77/2 COS t
'-^"6
I
- COS 2x

sin r
9 lim -^-^

In (.(/7)
M lim 12 lim
-
7 7 -.V ^Q .V sin X

In .V - sin (x - I \n{e' + I) - In 2
13 lim
U- 1)^
SECTION 10.2 OTHER INDETERMINATE FORMS 603

37 In Problem 36. make the change of variable i = 1/a. Then use


4,7'(4,3 + ,2 + 1 , ) ^,
die result ofProblem 69b in the Review Problem Set for Chap-
1

24 lim ter 7 to evaluate the limit.


f^'(-7r' + 6f + S)dr
38 State and prove the analog of Theorem 3 for the case x -^ -x.

39 The equation / = (£//?)( 1


- e"'"'') gives the current / amperes
In Problems 25 to 30, find a number c satisfying Cauciiy"s general-
ized mean-value theorem for each pair of functions on the indicated
at time r seconds in an /?L-circuit with resistance R ohms, induc-

interval.
tance L henry s. and electromotive force £ volts. (See Prob-
lem 21 in Problem Set 7. 1 1 .) If /, £. and L are held fixed, find

25 fix) = Iv' and g(x) = 3x- - I on [0. 2]


the limit of / as R approaches zero.

26 fix) = sin X and g(x) = cos -v on [0, n/4] 40 Let A be an endpoint of a diameter of a fixed circle with center O
and radius r. In Figure 1 , AQ is tangent to the circle at point A.
27 fix) = In A- and g(x) = l/.v on [1, e] and \AQ\ is equal to the length of the arc AP. If B is the point of
= = intersection of the line through Q and P with the line through A
28 fix) sin '
v and g(.v) .v on [0. 1]
and O. find the limiting position of B as f approaches A.
29 fix) = tan x and g(x) = 4x/tt on [-7r/4, 7r/4]

30 /(.v) = .v-(.v- - 2) and g(x) = .v on [- 1. 1] Figure 1

31 Find constants a and b such that

1
_ f' r dt
lim
-0 bx v Jo Va + I

32 Show that 41 A weight hanging from a spring is caused to vibrate by a driving


force so that its displacement at time i seconds is given by
{n + Da" -I- 1 Mil + 1)
lim
(.r-D- 2
V = — - u-
r(sin uT - sm pt)
p-
33 State and prove the analog of Theorem 2 for the case x —>a~.
where A, p, and »• are positive constants withp 9^ if. Determine
34 Explain why the calculation of a derivative directly from the
the limiting value of y as p approaches ir holding A and i fixed.
definition always involves the evaluation of a limit of an indeter-
minate form 0/0. 42 For each positive integer /;, let V„ denote the volume of the solid
obtained by revolving the region under the graph of fix) = x"
sin A
35 Explain why the use of L'Hopital's rule to evaluate lim between x = and a = I about the y axis and let H„ be the

really involves circular reasoning. volume of the solid obtained by revolving the same region about
theA axis. Evaluate
36 What happens when you
applications of L'Hopital's rule'?
try to find lim (e'" '/?) by repeated
(a) lim V„ (b) lim H„ (c) lim —

10.2 Other Indeterminate Forms


In Section 10. 1 we saw that L'Hopitars rule can often be used to handle limits of
fractions having indeterminate form 0/0. In this section we discuss additional inde-
terminate forms, ^/=c, 00-0. 3c - X, 0", ^°, and 1'.

The Indeterminate Form oo/*


By methods similar to those used in Section 10. 1 , it can be proved that L'HopitaFs
rule remains effective when the numerator and denominator become infinite in
absolute value, or, as we say, when the fraction has the indeterminate form ^/^.
604 INDETERMINATE FORMS, IMPROPER INTEGRALS, AND TAYLORS FORMULA

The facts are set forth in the following theorem.

THEOREM I L'Hopital's Rule for the Indeterminate Form oo/x


Suppose that the functions / and g are defined and differentiable on an open
interval /, except possibly at the number a in /. Moreover, assume that
lim |/(.r)| = +0C and lim |^U)| = +^. Then, if ^'U) 5^ for all values of .v

other than a in / and if lim —j^— exists, it follows that lim —— also exists and
' •" 8 (-V) >-" g(x)

lim = lim
<-" g(x) r-" g (X)

This theorem is also correct when x —>^ a* x . ^ a^ . x —* +^, or v —» — ^, pro-


vided that obvious changes are made in the hypotheses. We omit the somewhat
technical proofs of Theorem I and its alternative versions.

/)) Examples I and 2, evahuite the given limit.

In.v
EXAMPLE 1

SOLUTION The fraction has the indeterminate form ^/^ at +=<^. Thus, by
L'Hopital's rule.

D, In.v
lim
V- + -
—Vx In.v
i=r = lim
<- + » D,Vx
p^ = lim
l/.v

•^ + - \/{2Vx)
1=-

2V^
= lim = lim —2-^ =
.,^+« .V A--+=« Vx
7 tan .v
EXAMPLE 2 im
.Tr/2* 5 -I- sec X

SOLUTION The fraction has the indeterminate form ^/^^ at 7r/2. By L'Hopital's
rule, we have

7 tan .v 7 see" .V 7 sec x


lim lim = lim
— 7r/2 *
5 + sec X 1^/2 •
sec .V tan .V v^W2* tan .v

7/cos X .. 7
= lim lim = 7
-"r/2* sin .v/cos .V A^7r/2' sin .V

Just as was the case for the indeterminate form 0/0, it may be necessary to apply
L'Hopital's rule several times in order to evaluate a limit of an indeterminate form
=o/x. Also, it can be shown that L'Hopital's rule still holds if the quotient f'(x)/g'(x)
approaches +^ or — x; that is,

lim = lim = ±00


x-^a g{x) v-« g (x)

This is illustrated by the following example.

EXAMPLE i Evaluate lim — r.


SECTION 10.2 OTHER INDETERMINATE FORMS 605

SOLUTION The fraction has the indeterminate form oo/oo at +0°. Using
L'Hopital's rule 3 times, we have

lim
x^+^
f
-^ =
jc-
>

lim
.v-.+» 3x-
—- =
e'
iim
.v^+»
—=
e'

6jc
Hm
<:^+=«
—=e^

6
+<»

Other Cases of Indeterminate Forms


We now consider indeterminate forms of the type °o-0, oo — oo, 0°, °o°, and 1°°.

These are handled by algebraic manipulation of the indeterminate form so as to


reduce it to the type 0/0 or =o/oo so that L'Hopital's rule can be used. In each case
suitable examples are given to illustrate the technique.

The Case «> •


o

If lim |/(a-)| = +^ and lim g(.x) = 0, we say that the product f{.x)g{.x) has the

indeterminate form oo •
at a. To find lim /(.v)g(.v) in this case, we can write

f(x)g{x) as g(x)/[l/f(x)] or as/(.v)/[l/^(.v)], thus obtaining the form 0/0 or the form
oo/oo, respectively, whichever is more convenient.

EXAMPLE 4 Evaluate lim A" In x.

SOLUTION We write x' In x = (In -v)/.y -, noting that (In x)/x^- has the indeter-
minate form t»/oo at 0. By applying L'Hopital's rule, we obtain

, \r\ X l/.Y -,

hm X \n X = hm _, = lim ^^ = hm (— J
2)x~ =
.v^O* x-^0* X - v^o' -2x ^ v^O*

The Case oo - oo

If lim /(.v) = +00 and lim g(x) = +cc, we say that the difference /(.v) — g{x) has

the indeterminate form oc - oo at a. By performing the indicated subtraction, the


indeterminate form oo — oo can usually be converted to the indeterminate form 0/0.

EXAMPLE 5 Evaluate lim esc x


v^O* V X

SOLUTION The expression has the indetenninate form oo — co at 0; however, if

we write esc x as 1/sin .v and subtract, we have

/ 1 \
=
/I 1 \
=
.Y - sin .V
lim I CSC .V I lim I I lim
v^o* \ X ! v^o* \ sin .V X I v^o* x sin x

The fraction has the indeterminate form 0/0 at 0. Applying L'Hopital's rule twice,
we obtain
606 INDETERMINATE FORMS, IMPROPER INTEGRALS. AND TAYLORS FORMULA

The Case O", «>«, or 1"

If/U) > 0, lim fix) = 0, and lim g(.x) = 0. wc say that the expression /(.v)*''"' has

the indeterminate form 0" at a. Expressions with the indetemiinate forms a:" or 1"
are defined analogously.* In order to evaluate lim fixf^" in such indeterminate
cases we carry out the following procedure:

Step 1 Calculate lini |,i;(.v) ln/(.v)] = L.

Step 2 Conclude that lim /(.v)'^'"' = e^.

This procedure is justified by the observation that

lim /(a:)«"^' = lim e«'''


'"^'>* = e^

Notice that the product g(x) In /(.v) in step 1 has an indeterminate form at a.

In Examples 6 and 7, evaluate the indicated limit.

EXAMPLE 6 lim, .V-'


v—«o'

SOLUTION The indeterminate form is 0'\ so we carry out the procedure above.

Step 1 We must evaluate lim


0*
2v In .v. Here the product has the indetermi-
t—
nate form • <» at 0; hence, we rewrite it as

2 In X
2x In .V
=
l/x

The resulting fraction has the indeterminate form ^/^ at 0. Thus, by


L'HopitaFs rule,

L = lim
I— 0*
2x In .v = lim
v-»0*
2 In

l/x
;
—= jc

v^O*
lim
D,(2 In

D,(\/x)
.v)

= lim
V— 0^
— 2/x
-—;-
-l/.V-
= hm (-2x) =
v^O*

Step 2 lim .v-' = e^ = e" = \

EXAMPLE 7 lim, (cscr)"'"-"


— '
A

SOLUTION The indeterminate form is ^". Since

/ , yinv
lim (esc .v)""'" ' = lim I I

A^o' A-^o* V sin .V


'

*See J. V. Baxley and E. K. Hayashi, "Indeterminate Forms of Exponential Type," Ameri-


can Mathematical Monthly. Vol. 85, No. 6, June-July 1978, pp. 484-486 for an intere.sting

discussion of these cases.


SECTION 10.2 OTHER INDETERMINATE FORMS 607

we begin by making the change of variable u = sin .v. Notice that u -^ 0^ as


X ^ 0^; hence,
lim (esc .t)'""
"^
= Hm I
/
— \"
1
I

Now we use our procedure.

-In u
Step 1 L = Hm
r
« In
/
I
— \1
1

I = lim (-// In u) = lim ;

D„(-lnM) -1/tt
= lim ; = lim —r- = lim m =
u^o* D„(\/ii) "-('• -\/u~ «-o*

Step 2 lim (esc .v)"'" " = lim I — ) = e^ = e° = I

Problem Set 10.2

In Problems 1 to 50. evaluate each

1 + sec X sec Bttj:


1 lim 2 lim
P- tan X i/2 tan 377T

In (17 +.v) 1
- In X
3 lim

e' + 1
5 lim
x' + x'

In
7 lim

y hm
'-+- (r + 2)-

11 lim x{e^' - 1)

13 lim ;ce'/>
x-^O*

15 lim .v(ln .v)-

17 lim tan a tan 2x

19 lim
-1 \ -Y - 1 In .V /

21 lim (esc X — CSC Zx)

23 lim (x- - V.v'' + .V- + 7 :

24 X tan sec x
.1 —lim
7^/2
I

V
,v
2 '
,
608 INDETERMINATE FORMS, IMPROPER INTEGRAI-S, AND TAYLORS FORMULA

051 Use a calculator to obtain numerical confirmation of your result and


in (a) Problem 3, (b) Problem 13. and (c) Problem 27. lim — = 1
'-*' f"(x)
52 Let xf'jx)
f.v- sin (I/.v) if.v^^O Evaluate lim
S**^' = i
I.
n f
if .t = n fix)

Show that ;?'(0) = 0. I X + c Y=


54 Find a value ot the constant c such that lim I I 4.
JT— + » \ X - c '

53 Suppose that the function / has the following properties:

lim fix) = lim f'{x) = lim f"(x) = +^ 55 Evaluate lim


.- + - X
— Jo
e' dt.

10.3 Improper Integrals with Infinite Limits

In Chapter 5, we obtained the areas of admissible regions in the plane by using


definite integrals. Recall, however, that an admissible region must be hounded. If

we wish to find areas of unbounded regions, we have to deal with " improper"
integrals.
Figure I Consider, for instance, the region R under the graph of y = l/.v" to the right of
X = 1 (Figure la). Notice that the region R extends indefinitely to the right and
therefore is unbounded. Offhand, it is perhaps not clear what is meant by the
"area" of such an unbounded region. However, let Ri, denote the bounded region
under the graph of y = l/.v" between x = 1 and x = h (Figure lb). The area of /? ^ is

given by

J, -v- .V I, b

For large values of b. the bounded region R,, might be considered to be a


good approximation to the unbounded region R; in fact, it is tempting to write
R = lim R,,. Hence, we might expect that
SECTION 10.3 IMPROPER INTEGRALS WITH INFINITE LIMITS 609

DEFINITION 1 Improper Integrals with Inflnite Upper Limits

Let /be a function defined at least on the infinite interval [a. +x)
that/ is integrable on the closed interval [a. b] for every value of b larg
Then we define
610 INDETERMINATE FORMS, IMPROPER INTEGRALS, AND TAYLORS FORMULA

"
E 4 e dx
J

SOLUTION I
e '
dx = \\m \ e *
dx = Hm^ \-e M I

= lim {e~" - e") = +'^

Hence, e "
tic is divergent.

The following example illustrates the use of improper integrals in finding areas
of unbounded regions.

EXAMPLE 5 Suppose that p > 1. Find the area A of the region under the curve
y= l/x'' and to the right of the line .v = 1 (Figure 3).

SOLUTION

^
A =
J,
f" dx
—=
x'' />--
lim
+ .'-
J,
"
— = fc--+»Vl-p
dx
.x''
lim
/ x'-P
I,/

/ b^-P V-P \
= lim =0- 1 1

square units
b^ + oc \ \ —p \ —p ) ^ ~P P ~ ^

since the fact that p > 1 implies that lim b^'P = lim (1//?''"') =

Notice that if p < 1, then lim b^ p = +cc, so that /[' (dx/xP) diverges and the

unbounded region in Figure 3 has an infinite area. Wp = 1, the integral also di-
verges (Problem 5); hence.

, . (converges to ifp> 1

P- 1

r^l *'
[diverges if /j < 1

Improper integrals for which both limits of integration are infinite are defined as
follows.

DEFINITION 2 Improper Integrals with Both Limits Innnite

Let the function/be defined for all real numbers, and suppose that the improper
integrals fo fix) dx and /-»/(.v) dx both converge. Then, by definition,

I
f(x)dx= I
/(.v)^.v+
I
f{x)dx

and we say that the improper integral /-„/(.v) dx is convergent.

If either of the improper integrals /o /(v) dx or /- ^/(.v) dx is divergent, we say


that the improper integral fl^Jlx) dx is divergent.
SECTION 10.3 IMPROPER INTEGRALS WTPH INFINITE LIMITS 611

//) Examples 6 and 7, evaluate the given improper integral (if it is convergent),

dx
EXAMPLE 6
X- - 2r + 2

SOLUTION We have
f" dx ^.v
= lim
Jq x^ - 2v + 2 ft-+» Jo A- - Iv + 2

dx
lim
'o (.v-i,^+,=.ii'?a'-"<-^-H1
lim [tan-' (fo-
'—-
d-. + =c
D- tan-' (-1)]
'
= —2 + — = —
4 4
Likewise,

r ^^ , r dx r ,
|o]

.^-2r+2 = .^'"^ = [tan- (.v - 1)


J-. (X - 1)- +
i (,-1)3+1 1
.li-. | J

77 77" 77"
= lim^ [tan-' (-1) - tan"' (a - 1)]
4 2 ~T
Therefore, the improper integral is convergent, and

r dx r" dx ^
r dx _ 77" ^_ "^
i-^ X- -Zx + 2 L^ X--2X + 2 Jo -t- - 2v + 2 4
"^
4 ~
Naturally, we interpret this result geometrically as meaning that the entire un-
bounded region under the curve

.V-
- Iv + 2

has an area of tt square units (Figure 4).

Figure 4 y
ir^a ~ / ~; = IT square units
•^~« X" - 2 X+ 2

EX.AMPLE 7 -V dx
/:

SOLUTION I -V dx =1 X dx + \ X dx

provided that the last two improper integrals converge. But they do not converge; in
particular.

X dx = lim X dx = lim —= -l-c

Therefore, x dx diverges.
612 INDETERMINATE FORMS. IMPROPER INTEGRALS. AND TAYLORS FORMULA

Problem Set 10.3

In Problems I to 24. evaluate each improper integral (if it is conver-


gent).
SECTION 10.4 IMPROPER INTEGRALS WITH UNBOUNDED INTEGRANDS 613

39 The formula in Problem 37 that gives the function P in terms of n\, is defined by
the function / has important applications outside the world of
n! =«(« - l)(n - 2) • •• 3-2-1
business and finance. If the improper integral in this formula is

convergent, then the function P is called the Laplace transform Also, by special definition, 0! = 1. Using the result of Prob-
of the function /. If P is the Laplace transform of /. Q is the lem 41, show that Fin+ 1) = m! holds for all nonnegative inte-
Laplace transform of g, and a and b are constants, show that gers n. Therefore, the gamma function provides a continuous
aP + bQ is the Laplace transform of a/+ bg. generalization of the factorial of a nonnegative integer.

40 Suppose that P is the Laplace transform of/ (see Problem 39) 43 For the gamma function (defined in Problem 41), show that

and that lim e^"f(t) = for all values of r. If/ is differ-

= r{n+ 1) = r"^' e""?" dt


entiable and Q is the Laplace transform of/', show that Q{r) Jo
rP(r)-/(0).
for /I > - 1 and r > 0. (.Hint: Make the change of variable
41 The gamma function, denoted by F (the Greek letter capital .V = rt.)

gamma), is defined by
44 Let /(f) = t". where /; is a constant and n > -I. If P is the

Laplace transform of/ (see Problem 39), show that


r(«) = dx
f
•'o

for all positive real numbers n. This function, which was discov-
ered by Leonhard Euler, is useful for solving differential equa-
tions in applied mathematics. Using integration by parts, show
for r > 0. {Hint: Use the result of Problem 43.)

that r(/; + 1) = nR/!) for n > 0. [Him: Evaluate Tin + I), 45 Find the Laplace transform of /(f) = sin f. (See Problem 39.)
using integration by parts with u — x" and dv = e '
dlv.]
46 If n is a nonnegative integer, find the Laplace transform of
42 Recall that if /i is a positive integer, then n factorial, denoted by /(/) = t". {Hint: Combine Problems 44 and 42.)

10.4 Improper Integrals with Unbounded


Integrands
The improper integrals considered in Section 10.3 enabled us to calculate the areas
of unbounded regions in the xy plane that extend indefinitely to the right or to the
left. In this section we consider unbounded regions that extend indefinitely upward

or downward.
Figure 1 For instance, consider the unbounded region R under the curve y = 1/Vx, to the
right of the V axis, and to the left of the vertical line x = 9 (Figure la). A good
approximation to this unbounded region is afforded by the bounded region R^ under
the curve y = 1/Vx between x = e and .v = 9, provided that e is a small positive
number (Figure lb). As e ^ 0^ , 7?^ becomes a better and better approximation to R,
and it is tempting to write R = lim^ R^. Hence, we might expect that

dx
area of R = lim (area of R^) = lim = lim iVTx
J

lim (6 2Ve) = 6 square units

-6 2 -s/e square units


In view of this calculation, we write

dx
= 6
V~x
614 INDETERMINATE FORMS, IMPROPER INTEGRALS, AND TAYLOR'S FORMULA

However, it is important to realize that, as a definite (Riemann) integral,

/o (dx/^/x) fails to exist because the integrand 1/V.v is undefined on (0, 9] and
unbounded on (0, 9). Thus,
r dx

{ Vx
is called an improper integral. More generally, we make the following definition.

DEFINITION 1 Improper Integral at Lower Limit

Suppose that the function / is defined on the half-open interval (a, h\ and is

integrable on every closed interval of the form [a + e, h] (or < e < b - a.

Then we define the improper integral fi'ifix) dx by

fix) dx = lim. /(A) dx


I

provided that the limit exists as a finite number.

If lim^ ja+e fM dx exists as a finite number, we say that the improper integral

iaf(x) dx is convergent; otherwise, it is called divergent.


Definition 1 takes care of the case in which the definite integral /'j/(.v) dx may
fail to exist because /(a) is undefined. A similar definition.

fix) dx fix) dx

of an improper integral at the upper limit covers the case in which /(/>) is unde-
fined.
To avoid confusing the improper integrals just defined with ordinary definite
integrals, some authors use the notation /,';. f(x) dx or J^ f(x) dx. However, since
you can always determine whether an integral is improper by examining the
integrand, we do not use this notation in what follows.

In E.xainples I to 3. evaluate the given improper integral (if it is convergent).

f* dx
EXAMPLE 1 ^r^rj-
4/2 (A- - i)-/-'

SOLUTION The integral is improper because the integrand is undefined at the

lower limit V- We begin by using the substitution ii =x — ^ to evaluate the corre-


sponding indefinite integral. Thus,

dx du 5 3\'''-'
r
3-^ =
f
-^^ = -T "
,,,
+ C = — [x - —) +C
5 /

Applying Definition 1, we have


dx dx
- 3^2/5
I3/2 iX-t) (.V - if

'^0 L 1(3/2) + J
SECTION 10.4 IMPROPER INTECRALS WTTH LNBOLNDED INTEGRANDS 615

sec X dx
^ \r
SOLLTION The integral is improper because the integrand is undefined at the

upper hmit 77/2- Here we have

sec X dx = lim sec X dx = lim In ( sec .v + tan .v)

= lim In [sec e + tan (


—- - e - In (sec + tan 0)
i
(^ )
) I i

because lim sec [(-tt/2) - e] = +--^ and lim tan [(tt/I) - el = +=c. Therefore,

the improper integral is divergent.

EX.AMPLE 3 /;' dx

SOLUTION Here the integral is improper at the lower limit 0. Making the substi-
tution It = sin v. we have

r cos .V r ,1 /-, ,

.
.
dx = ir^
,
- du = 2h'/-
,

+ C = 2Vsin .v + C
J V sin V J

Therefore.
f tt/- r „/- / W-\
— j,pj ^
, dx = lim —
£.Q5 Y
. dx = lim \ 2\
^__

sin x I

Jq Vsin .V
^^0" \ sin .V
^^o" 1^ /
-'e

lim (2Vsin {tt/I) - 2\ sin e) = 2V sin {tt/I) = 2

Figure 2
616 INDETERMINATE KORMS, IMPROPER INTEGRALS, AND TA\ LORS FORMULA

EXAMPLE 5 Evaluate the integral /"tan x dx (if it is convergent).

SOLUTION Because the integrand is undefined at tt/I, the integral is improper.


It is convergent only if both /i7^- tan .v dx and Sn/2 I'm -^ (l-^ '<^^ convergent. But

rW2 r(W2)-. r |(./2,-e-]


tan x dx = lim tan .x dx = lim In (sec .v)
Jo - £-()• J„ €^0' L lo -

= lim in sec y— - ej - In sec = +^

Hence, Jo^' tan .v d.x diverges. Therefore, /(7tan .v d.x also diverges.

Figure 3
EXAMPLE 6 Let R be the unbounded region under the
graph of

between .v = 1 and .x = 6 (Figure 3). Determine whether the


region R has a finite area, and if it does, find this area.

SOLUTION We cannot simply evaluate

f" dx

as if were an ordinary definite integral because the integrand is not defined at


it

X = However, if we split the region R into two portions, one to the left and the
3.

other to the right of the vertical line .v = 3, then the areas of these two portions are
given by the improper integrals

['"'
f -
^-^3)-/-'
= lim
e^o* -
^^7^=
3)-/' ^-0^
lim f3(.r-3)'/n'
L
1=3(2'/^)
J, (.V J, {X 1
1 -I

and
dx (^ dx r 1^ 1
lim ^7^ = lim 3(.v - 3)'''-' 3(3 '/3)
f
Jj (X - 3)-/' ^-0' Jj^, (X - 3)-

Therefore, the desired area A is actually given by the convergent improper integral

p dx n dx p dx
^ ~ \ - 3)^/^
~ - 3)^/^ ^ - 3)^/-^
(X J, (.V h (.V

= 3(2'/-^) + 3(3'/-'') square units

Problem Set 10.4

In Problems I to 24, evaluate each improper integral (if it is conver- '^^^


cos X dx
gent).
J, \/7- sin X

f ^^dx
dx dx dx
5
-'o V7 4 atV^ Jo N^
I

Jo (1 + x)Vx
SECTION 10.5 TAYLOR POLVAOMIALS 617

r (In.v)-

-'o -V

X dx
10
lo V25 - x'

12
J
618 INDETERMINATE FORMS, IMPROPER INTEGRALS, AND TAYLOR'S FORMULA

DEFINITION Taylor Polynomial

Let/be a I'unction having a derivative/'"' of order n s 1 on an open interval /,

and let a be a fixed number in /. Then the nth-degree Taylor polynomial for the
function / at the number a is the polynomial function P„ defined by

/'(«) f'ia) , f"'{a) ,

+ - ——(x-a)"
Recall that if n is a positive integer, then /;! is defined by

«! = /;(/! - !)(/( - 2) • • • 3-2- 1

Also, by definition.

0! = 1

EXAMPLE 1 Find the fourth-degree Taylor polynomial P4 for/(.v) = sin x at the

number a = n/b.

SOLUTION We arrange the work as follows;

77\ 1

fix) = sin X f[
6/ 2

j (X) = cos .V
/ y—j = —
Brook Tavlor
/"(.v)=-sin.v =
r(f) -Y
V3
f"'(x) = —COS .V /"

/'"'(x) = sin X
/"(f)
Hence,

P.ix)=f(^).l^^^{x-:^)^i::^^yx-
^ V6/ V 1! V 6/ 2! 6

^ /"'(V6) / _ ^y^ ^ /'"'(7r/6)

3! V 6 / 4!

—1 V3/
= + .V .v-^y--^f.v-^r V 48
2 2 ^ 12

EXAMPLE 2 Find the Taylor polynomial Pj. of degree 3 for the function /
defined by

/(-v) for .V > - 1 at the number a =


1 +.V
SECTION lO.S TAYLOR POLYNOMIALS 619

/(.v) = (l +.V)-' /(O) = 1

/'(.v) = -d +.V)-- /'tO)= -1


/"(A-) = 2(l +.v)-^ /"(O) = 2

/'"(A) = -6(1 + A-)--* /'"(O) = -6

Hence,

= 1 - A- + A- - A-'

The Taylor polynomial P„ for the function / at the number a always has the
following property, whose proof is left as an exercise (Problem 24):

The values of the successive derivatives of P„ at the number a—


up to and
including order /;— are equal to the values of the corresponding successive deriv-
atives of/ at the number a.

Furthermore, as is clear from Definition 1 , the value of/ at a is equal to the value of
P„ at a. Thus, we have

f{ci) = P„ia)

f'Ui) = P:,(a)

f"(a) = P:Mi)

f'ia) = P;,"'(fl)

For instance, as you can easily check, in Example 2 above.

/(O) = 1 = P3(0)

/'(0)= -1 =pm
/"(O) = 2 = P'iiO)

and /'"(O) = -6 = P'i'(0)

Perhaps the most important feature of the Taylor polynomial P„ for the function/
at a is that the approximation

/(A) = P„(X)

often is exceedingly accurate, especially if /) is large and a is close to a. For


instance, in Example 1 above (rounding off to six decimal places), we have

sin — = 0.707107 = ^4^— ) = 0.707098


620 INDETERMINATE FORMS, IMPROPER INTEGRALS, AND TA^ LORS FORMULA

Calculators and computers handle transcendental functions by using polynomials


whose values approximate the values of these functions. Often, Taylor polynomials
are chosen for this purpose because of the accuracy of the approximation /(.v) =
P„{x). Thus, in applied mathematics — especially in work that involves calculators
or computers — it is essential to be able to place bounds on the error that results
when the value of a -function is approximated by the value of a Taylor polynomial
for that function. This error, the difference between the true value f(x) and the
estimated value P„(x). is called the Taylor remainder /?„(.v). Thus, we make the
following definition.

DEFINITION 2 Taylor Remainder

If P„ is the /(th-degree Taylor polynomial for the function /at the number a. we
define the corresponding Taylor remainder to be the function /?„ given by

R„(x) =/(.v) - P„(x)

Notice that

fix) = P„(x) + R„(x)

so the approximation fix) = P„{x) will be accurate when |^„(.x)| is small. The ex-
tended mean-value theorem, referred to in the introduction to this section, provides
an effective means for estimating |/?„(.v)|.

Extended Mean-Value Theorem

Let n be a nonnegative integer, and suppose that /is a function having a deriva-
tive/'"^" of order n + 1 on an open interval /. Then if a and b are any two
distinct numbers in /, there exists a number c strictly between a and b such that

/'(a)
J\b) = f{a) + ^(b a)
f"{a) ,
+ —r-(b - af + —rr-(b
f"'(a)
- a)' +

/'"'(«)
{b - a)" + r„

where r,,
= (b — «)"^'.
in + 1)!

PROOF Define the number r„ by

/'(«) f"(a) , f"'Ui)


r„ = fib) - f(a} - ^—^(b - a) (b

r"\a)
ib - a)"

Then

/"(«) /"(«) /'"(«)


f(b) =/(fl) + i-^ib - fl) + ^^(fo - a)~
.
+ -^ib - a)' +

./^,/, -«)" + .,
SECTION 10.5 TAYLOR POLYNOMIALS 621

Define a new function g with domain / by the equation

g(.x) = fix) + ^^{b - .V) + ^—-(b - X)- + ^^-jr"(*


- -V)-' +

/""(-v)
^ (b- .xr
;
^,
(b - ^„
.x)" + r„-
'(b - a)"*'

Then

j^ + —^(b
g i.x) =f U) + [^
- A)

L

2/"(J

2!
(fo - .r) + —^(fc - A)

r 3/'V) . /'"*'(-v) ,1

"/""'-v' /"""(A) 1 (» + 1 )r„(fc- A-)"


;
,,,
{b - xY
,„-, + ,

;
{b - xf ; -—

In the expression above, each set of brackets contains a term of the form

Since

it it 1

k\ k(k- \)(k-l)---\ (A-- 1)

this term can be rewritten as

/"'(A-)
-{b- .r)*-'
{k- 1)

and thus cancels with the term

(fc-.v)*-'
(k- D!
in the preceding set of brackets.
Performing all such cancellations, we find that all but the last two terms drop out,
and we have

/'"""(.v) , „ («+ l)r„(fc-;tr


n\ (b — a)

Going back to our original definition of g and putting a = a. we find that

g{a) =f(a) + ^^(fo - fl) + ^-^(b - a)- + ^—^(b - a? +

+ '
\ b - a)" + r„-l
n\

sothatg(a) =f(b). Also. puttingA = /? in the definition of g. we obtain g(fo) = f(b);


hence. g{a)= g[b). Thus, applying Rolle's theorem to the function g on the closed
622 INDETERMINATE FORMS, IMPROPER INTEGRALS, AND 1 AYI.OR'S FORMULA

interval from a to b, we conclude that there is a number c strictly between a and b


such that g'(c) = 0. Using the formula above for ^'(.v), we have

= /'"^"(D {n+ \)r,.(h-cr .


g (c) ; (b - (•)"
: ZT-,
=
(b-a)"^'
/'"*"{£) (n + \)r„{b - c)"
(b - c)" =
ib-af^'
Since b i^ c, it follows that b — c ¥" 0, and common factor {b - c)" on both sides of
the last equation can be canceled to yield

/'""'(c) («+ IK
ib-a)"^'
(n + 1)«!

Because (« + l)/i! = (« + 1)!, we have

(fc-a)"^'
(n + 1)!

and the proof is complete.

Notice that if we put « = in Theorem 1 . we obtain

/(b)-/(fl) = (fo-«)/'(c)

where c is strictly between a and h. Therefore, for n = 0, the conclusion of Theo-


rem coincides with the conclusion of the mean-value theorem.
1

As promised, we can now use Theorem to obtain an expression for the Taylor 1

remainder/?,,. The appropriate theorem is nothing but a restatement of Theorem 1 as


follows.

THEOREM 2 Taylor's Formula with Lagrange Remainder

Let / be a function having a derivative /*"^" of order n + \ on an open


interval /, and let a be a fixed number in /. Denote the Hth-degree Taylor poly-
nomial for / at a and the corresponding Taylor remainder by P„ and R„.
respectively. Then, for any .v in /. we have
f{x) = P„ix) + R„{x)

If .V # a, there is a number c strictly between a and .v such that

/<"^"(r)
R„{x) -(.V - (/)"'
Ui + 1)!

PROOF Put b = X in Theorem 1 , noting that

f(a) + —jJ-U' - a) + -T^U - a)


f"'(a) , f"\a)
+ -^(x -af + ••• +
^
(X - af = P„(x)

and, therefore, that

R„(x) = r„ = -^{x - a)"'


(/I + 1)!
SECTION lO.S TAYLOR POLYNOMIALS 623

Theorems 1 and 2. which really make the same statement with different notation,
are both referred to as Taylor's theorem, or Taylor's formula. The expression

/'"""(c) ^
(.V - a)
(«+ 1)!

for R„{.x) given in Theorem 2 is called the Lagrange form for the Taylor remainder.

EXAMPLE 3 Find the fourth-degree Taylor polynomial P4 and the corresponding


Taylor remainder R4 in Lagrange form for the function / defined by

/(.v)
= for v > -2 at the number a = 1

x + 2

SOLUTION Here we have

= 2U + 2)-^
f'(x) = -(.V + 2)-2 /"(.v) 2)--^
/'"W = -6(A- +

f^^x) = 24(.x + 2)-5 and f^\x) = - I20(.v + 2)"^

Hence,

/(I) = 5 /'(I) = -5 /"{n = A /'"(!) = -^ r>(i) = ^


and

,,,, .„„ , m,, . „. qi,, - ,= . nil,. - ,. . m,, - „'


= 5 - kx - 1) + ^(.v - 1)- - ftCv - 1)-' + afeC.v - 1 f

In Lagrange form, the corresponding remainder is given by

f'\c) _ ,
120(A- 1)-^
_ (X- \f
'^'""'~ ^'"~
5!
*•'
5\(c + 2f (c + 2f

where c is a number strictly between 1 and x.

a EXAMPLE 4 Use the third-degree Taylor polynomial for the function


= In (1 + a) at
fix) number a =
the to estimate In 1.1. Then use the Lagrange

form of the remainder to place a bound on the error of this estimate.

SOLUTION Here, we have

fix) = (1 + .V)-' f"(x) = -(1 + -v)--

f"'(x) = 2(1 + x)-^ and f'\x) = -6(1 + -v)"'*

Thus. /(0) = /'(0)=1 /"(0)=-l /"'(0) = 2

so the third-degree Taylor polynomial at a = is given by

p,u.=/(0..^(,-o,.^u-o,^.0?>,,_o,.
= + A - ir + iv'

Putting A = 0.1 in the approximation /(a) « P^ix), we obtain the estimate

In 1.1 =/(0.1) = P,(0.1) = 0.1 -*(0.1)- + 5(0.1)^=1^


or In 1.1 =0.0953333 . . .
624 INDETERMINATE FORMS, IMPROPER INTEGRALS, AND TAYLOR'S FORMULA

The error involved in this estimation is given by the Lagrange remainder

/(0. 1 )
- P3(0. 1 )
= /?,(0. 1 )
= ^^-:7-(0. 0)^
4!

-6(1 +c)-
-10"
4! ^•(l +(;)-*(4)

where < c < 0. 1 . Since c > 0, it follows that

/?,(0. 1

lO-'ll + c)-'(4) 10'*(4)

Hence,

/?,(0.1) < = 0.000025


40,000

Notice that R^iO. 1 ) is negative, so that the estimated value In 1 1 ~ 0.0953333


. . . .

is actually a little larger than the true value of In 1.1. However, since the absolute
value of the error cannot exceed 0.000025, the given estimate is correct to at least
four decimal places, and we therefore write In 1.1 ~ 0.0953.

The following theorem can often be used to determine — in advance — the degree

n of the Taylor polynomial required to guarantee that the absolute value of the error
involved in the estimation /{/^) ~ P„(b) does not exceed a specified bound.

THEOREM 3 Error Bound for Taylor Polynomial Approximation

Let/be a function having a derivative/'"^ "of order n + 1 on an open interval/,


and let a and b be two distinct numbers in /. Suppose that M„ is a constant*

(depending only on n) and that |/"'^"(c-)| < M„ holds for all values of c strictly
between a and b. Then if P„ is the Hth-degree Taylor polynomial for/ at a, the
absolute value of the error involved in the estimate /(fc) = P„(b) does not exceed

M,
(n + 1)!

PROOF By Theorem 2, there is a number c strictly between a and h such that

R„(b) = -',
. ,
(fo-fl)"^'
+
,

{n 1)!

Therefore,

\f{b) - p„{b)\ = \R„{h)\ = (b - «r^'


y(n +
I
1)!

/(«+l)(^.) J
''
!
< yv^
J !

'

(« + 1)! (« + 1)!

*In practice, it is often possible to generously overestimate |/"^ '(c)| by M„ and siill get a very

usable error estimate.


SECTION 10.5 TAYLOR POL^'NONHALS 625

''
B EXAMPLE 5 Estimate In 0.99. making certain that the error does not exceed 10
in absolute value.

SOLUTION In Theorem 3, we take/(.r) = In .r. a = 1 , and fe = 0.99. (We have


chosen a = 1 because it is near 0.99 and we know that In 1 = 0.) Here,

f\x) = .r"' f"{x) = -X-- f"'(x) = Ix-^ f%x) = -6.V-*

f"\x) = (-l)"-'(;! - 1)! .V'" and /'"^"W = (-l)"n! .r"'"-""

(If desired, this can be established rigorously by induction on n.) Hence, for
0.99 <c<l.
|/'""'V)| = |(-l)"n! r--"-"|
1
=-^<
^-1 (0.99)"^'
^—

and we can take M„ = «!/(0.99)"^' in Theorem 3. Thus, the absolute value of the
error involved in the estimation In 0.99 = f(b) ~ P„{b) does not exceed

|0.99 - 11"^' /;! (0.01)"^' 1


M„
in + 1)1 (0.99)"^' (« + 1)! in + 1X99)""

The smallest value of n for which \/[{n + 1)(99)"^'] < 10"^ can be found by trial

and error to be /; = 3; hence, ^3(0.99) approximates In 0.99 with the desired accu-
racy. Here.

W =/.!). ^(.v - 1, . /^,.v - 1,^ . :911(.v - 1,-^

= + (.V- i)-kx- i)- + kv- 1)^

so that

In 0.99 = ^3(0. 99) = -0.01 - i(0. 01)- - i(O.Ol)' = -0.0100503333 . . .

Since the error involved in this estimation does not exceed 10~^ = 0.0000001 in
absolute value, we can be certain that the approximation

In 0.99 = -0.0100503

is accurate to six decimal places. (The true value, rounded off to eight decimal
places, is In 0.99 = 0.01005034.)

la EXAMPLE 6 Estimate sin 40^ with an error no more than 10"^ in absolute value.

SOLCTION In Theorem 3 we take

f(x) = sm .V a = — = 45
77 „
and b =
407r
= 2,7
= 40°
4 180 9

(We have chosen a = 45" because it is near 40° and its sine is well known. ) Here.

f\x} = cos X fix) = -sin .v f"'(x) = -cos x /'""(-v) = sin .v

and so forth. Hence. /'"""(r) is one of ±sin c or ±cos c, so that |/'"^"(c-)| ^ 1

and we can take A/„ = 1 in Theorem 3. Since the absolute value of the error of
estimation carmot exceed

\2tT TtI"^' / 77

\b-a\"^^ 19 4 1
'36
M.
(n+ 1)! {n + 1)! (n + 1)!
626 INDETERMINATE FORMS, IMPROPER INTEGRALS, AND TAYLOR'S FORMULA

we must choose n large enough so that (77-/36)"*'/(« + 1)! ^ 10 '. By trial and
error, the smallest such value is /i = 3. Since

, cos — , ,

Piix) = sin —+ ('-Tr-^(-T)


4

is follows that

iTT /27r\
sin 40° = sin
9 ^ 9

V2_ Vi/ IT \ Vi V2
2 ^ 2 V 36/ 4
(--)
V 36/ 12 36/
0.6427859 . . .

with an error no greater than 10


''
= 0.00001 in absolute value. (The true value,
rounded off to seven decimal places, is sin 40° = 0.6427876.)

Problem Set 10.5

In Problems 1 to 16, find the Taylor polynomial of degree n at the absolute value. In each case, write the answer rounded off to four
indicated number a for each function, and write the corresponding decimal places.
Taylor remainder in the Lagrange form.
17 sin 1 18 cos 29°

f(x) = l/.v, o = 2. n = 6
1
19 e (Hint: e = e\) 20 e"'
'

2 g(x) = Vx. a = A. n = 5
21 In 0.98
3 fix) = l/Vt, a = 100, n = A
22 In 17 [//m/.- Write In 17 = In 16(1 + i^) = 4 In 2 + In (1 +^).l
4 f(x) = \/x,a= 1000, M = 4
23 V9.04
5 g(x) = U- 2)--, a = 3, « = 5
24 Use mathematical induction to prove that if P„ is the «th-degree
(, f(x) = (1 - x)'^'-. a = 0, « = 3 /'*'(«) =
Taylor polynomial at a of the function/, then P\^'ia)

7 f(x) = sin .t, a = 0, n = 6 holds for A' = 1, 2, . . . , n.

8 ,i;(.v) = cos ,v, a = —tt/7>. n = 3 25 Give a bound on the absolute value of the error involved in

estimating sin 5° [that is, sin (rr/36)] by using the nth-


9 g{x) = tan .V, a = 7r/4, n = 4 degree Taylor polynomial for sin .v at a = 0. (Hint: Use the fact
that 7r/36< to)
10 /U) = e-\ a = 0. n = 5

= xe\ a = n = 3
|c]26 Give a boun d on t he absolute value of the error involved in
1 fix) 1 ,

estimating Vl + .r by + jx for |.v| < 0.1. 1

12 fix) = e"^, a = 0, n = 3
27 Show that the error of the estimate cos .r = 1 — (.v"/2) does not
13 six) = 2", a = I, « = 3 exceed .v''/24 in absolute value. {Hint: Notice that
- = P,ix).]
I4/U) = Inx, a= 1, n =4 1 (j:V2)

= = = 28 (a) Show that the difference between the arc length i of an arc of
15 gix) sinh Jt, a 0, n 4
a circle of fixed radius r and the length of the corresponding
16 fix) = In (cos AT), a = ir/3. n = 3 chord is given by s - Ir sin [.?/(2r)]. (b) Use a third-degree
Taylor polynomial to approximate i'
— Ir sin \slilr)\. and give
Oln Problems 17 to 23, use an appropriate Taylor polynomial to ap- a bound for the absolute value of the error involved in this ap-
proximate each function value with an error of no more than 10"'' in proximation.
SECTION 10.5 TAYLOR POLYNOMIALS 627

O 29 Use a third-degree Taylor polynomial to approximate the area of 35 (a) Show that, for .t < 0,
the smaller segment cut off by the chord in Problem 28. and give
a bound for the absolute value of the error involved in this ap- e' = \ +x + —+
2 24
+ R^ix)
proximation.

El 30 Use a third-degree Taylor polynomial approximation for sin x at where -— - < /?4(.v) < 0.

a = to find, approximately, a value of .v > for which


5 sin X - 4.V = 0. (b) Replace -v by -r in part (a) to conclude that

31 Suppose that p a nonzero constant and let P„ix) be the nth-


is

degree Taylor polynomial for f(x) = (I + xy at the number


.-'• = , -r ,
+ y-- + --.(n
r^ t" f«

a = 0. Find (a) /',(.v). (b) P2(.x), (c) PjW, and (d) Piix).
,10
where < r(0 s .

32 If a perfectly flexible cable weighs w newtons per meter, has a 120


span of L meters between points of support at the same level.

and has a tension of T newtons at its lowest point, then it has a


(c) \i b> Q. use part (b) to show that

sag of H meters given by


fe ,' ^ b' b' b' b"
'
dt = b -I-

wL Jn 3 10 42 216
cosh
w V IT
where :

1320
Using an appropriate second-degree Taylor polynomial and as-
suming that wL/T is small, obtain an approximate formula £: (d) Evaluate /o' e
'"
dt. rounded off to three decimal places.
for//.
Ej 36 Assume that/is a function with a continuous derivative/'"" of '

'

order n -I- 1 on an open interval /. Let a and b be distinct num-


bers in /, and let P„ be the ;ith-degree Taylor polynomial for/ at
a. Use induction to prove that the value R„^b) of the correspond-
ing Taylor remainder is given by

R„ib) (b - t)"f'"*^'U) dt

This is called the integral form of the Taylor remainder.

37 (a) If

I
fix)

show that the Taylor polynomial P„ for / at a = is given by


P„(x) = I +x+ x- + x^ + •• • + x".

(b) Show that the Taylor remainder corresponding to P„ is

given by

R„{x) =

33 The tension T^ in the cable of Problem 32 at either point of 38 Suppose that/ is a function having a derivative of order n on the
support is given by open interval / and that u is a number in /. Let P be a polynomial
function of degree n such that /(a) = P(a) and /'*'(«) = P'*'{a)
wL
T, = T cosh -—- for <: = 1, 2 n. Prove that P is the nth-degree Taylor
IT
polynomial for /at a.

Using an appropriate second-degree Taylor polynomial and as-


39 Let P„ be the nth-degree Taylor polynomial for/ at a. where the
suming that wL/T is small, obtain an approximate formula for
function/has a derivative of order n on the open interval / and a
belongs to /. Let the function g be defined on / by g(x) =
34 Let /be a polynomial function of degree n. and let P„ be the Jlfit) dt. and let the function Q be defined on / by Qi.x) =
nth-degree Taylor polynomial for/at a. Show that /(a) = P„(x) Si Pn(x) dx. Show that Q is the Taylor polynomial of degree
for all values of x. n -t- 1 for ^ at a. (Hint: Use the result of Problem 38.)
628 INDETERMINATE FORMS, IMPROPER INTEGRALS, AND TAYLOR'S FORMULA

40 Suppose that h is a function that has a derivative of order n on {Him: Use Problem 37.)
the open interval / and that the function i; is defined by g(.v) =
(b) If
a"" '/i(.v) for .V in /. (a) Prove that utO) = and that ^'"(0) =
= f(x)
for <: 1 , 2 ". (b) Prove that the /ith-degree Taylor poly-
nomial for t; at a = is the zero polynomial.
show that the Taylor polynomial Pin of degree 2n for/ at a =
is given by P2„(x) = \ - x- + x" - x'' + • + (.-l)"x^. and
41 Assume that/ is a function having a derivative of order n on the
show that the corresponding Taylor remainder is given by
open interval / and that belongs to /. Suppose that the function

h also has a derivative of order n on / and that P is a polynomial (-ir'.r^* =


of degree n such that/(.v) = P(.\) + x"* '/i(.v) holds for all values l^lnM =
+ x~ 1

of .r in /. Prove that P is the nth-degree Taylor polynomial for/


at 0. \Hini: Use part (a) of Problem 40 and Problem 38.) [Hint: Use Problem 41 and part (a).]

43 Use Problem 42, Problem 39, and the fact that


part (b) of
42 (a) Prove that
tan X = Jo dt/il + t') to show that the Taylor polynomial of
'

degree 2/i + 1 for the inverse tangent function at a = is


.v" + x" - x" +
1 + X-
given by

+ (-l)V"
(-1)
/>.„.,(.)=.-- + --- + ••• + (-1)"^;^^

Review Problem Set, Chapter 10


In Problems 1 to 32, use L'Hopital's rules to evaluate each limit (if
17 lim
it exists). •1 ^ .V- - 1 X- \

8' - 2'
18 lim
1 lim 2 lim -
—I) 4.V 1 .V -I- 2VI + X-

cos 2x — cos X sin .r sin x


-
4 lim 19 lim 20 lim (cosh x sinh .v)

sin~ .r
«— 0* X X — sm X

- 1
6 lim ^7- 21 lim x'/"-^'' 22 lim (-
—u X- -X V— 0' \

2r' 5.V- - 4x-


8 lim
-t- 2>
23 lim

(1-1- —] 24 lim ( I -I- ax^f
-> +« \ X I

25 lim (.V- -I-


4)''''
26 lim. (.V - 4)'"-'*
10 lim

27 lim (cos .t)'''" 28 lim (1 -I- sin.v)"""


.t—
II lim .r\ln.r)' 12 lim
I— 11' t— 0' cot X 29 lim [In (.r -I- 1)]' 30 lim. (tan''.v)'/'"^
x-»0
(ln.v)=
13 lim
<^i' sin (X - 1) 31 lim (in — 32 lim (sin .r>'

,,
14 lim
,. Vl -I- sin .r - W^
>-o tan X
In Problems 33 and 34, find all numbers c satisfying the conclusion

15 lim X sin
«-!
— of Cauchy's generalized mean-value theorem for each of the func-
X tions/and g on the indicated interval [a, b\.

16 lim CSC X sin (tan x)


33 f(x) = VT+T, g(x) = V^, and [a. h\ = (0. 16).
REVIEW PROBLEM SET 629

34 fix) = sin r, g{x) = cos x, and \a. b] = [7r/6, 7r/3].

In Problems 35 and 36. use the fundamental theorem of calculus and


L'Hopital's rule to find each limit.

I + 5)dt
35 lim
'^*"
e'(ir + It + I) di
^

(cos- / + 5 cos r) dt

''
e dt

37 Show that if a is a fixed positive number, then lim x" In x = 0.

38 Find constants a and b so that

/ sin
in 3; a \
iim
;^0 t r /

In Problems 39 to 56, evaluate each improper integi^al (if it is con-


vergent).

dx
39
xVlJr
e-l'' dt
41

43

45

47

49

51

53
INFINITE SERIES

In Section 10.5 we discussed the use of Taylor polynomials to approximate the


value of various functions and observed that the approximation often becomes more
and more accurate as the degree of the Taylor polynomial is increased. For instance,
the approximation 2 3 .«

1 +
1!

becomes bener and better as the number of terms on the right becomes larger and

larger. This suggests that in some appropriate sense, e' should be given exactly by
the "infinite sum" of all terms of the form .x''/k\\ that is.

+ —+
3! (« + 1)

Such infinite sums, which are called infinite series, are studied in this chapter. Most
of the important functions considered in calculus can be represented as a "sum" of
an infinite series in which the terms involve powers of the independent variable;
such a series is called a power series.

In the chapter we How do we assign meaning to


consider the following questions:
the sum of an infinite series? How can we tell whether a given infinite series has
such a sum (that is, whether it is convergent)! When and how can we represent a
function by a power series? When and how can we differentiate and integrate func-
tions represented by power series?
To answer these questions and to develop a theory of infinite series, we begin by
studying the closely related idea of infinite sequences.

11.1 Sequences
The word "sequence" is used in ordinary language to mean a succession of things
arranged in a definite order. Here, we are concerned with sequences of numbers
such as
1. 3. 5, 7, 9

or such as 0, 1. 4. 9. 16. 25. 36. 49. 64.


630
SECTION III SEQLTNCES 631

The individual numbers that appear in a sequence are called the terms of the se-
quence. A sequence having only a finite number of terms (such as the sequence 1

3. 5. 7. 9) is called a finite sequence. Notice that the sequence

0, 1. 4. 9. 16. 25. 36. 49. 64. . . .

(whose terms are the "perfect squares'" arranged in ascending order) involves an
infinite number of terms and is therefore an inflnite sequence. Since we cannot

write down all the terms of an infinite sequence, we use the convention of writing
the first few terms and then appending three dots to mean "and so forth."
Here, our concern is with infinite sequences only, so from now on we refer to an
infinite sequence simply as a sequence. Such a sequence is indicated by

fl], «2. a? fl„. . . .

where Oj is the fu-st term, a, is the second term, a, is the third term, and so forth.
The general term, or the nth term, is here denoted by a„. In order to specify the
sequence, it is sufficient to provide a rule, or a formula, for the nth term a„. For
instance, the sequence whose «th term is given by the formula

= 3/! - 1

has first term

= 3(1)

second term

a. = 3(2) -1=5
and so on. The resulting sequence is

2, 5, 8, 11. 14, 17, 20 3« - 1, . . .

Any letter with a subscript may be used to denote terms of a sequence — there's
nothing special about the letter a. Sometimes a listing of the first few terms of a
sequence indicates beyond any reasonable doubt the rule, or formula, for the gen-
eral term. Examples are

1. 2, 3, 4, 5. 6. . . . {a„ = n)

(b„ = 2n)
632 t HAKI ER 1 1 INHNITE SERIES

In rigorous mathematical treatises, a sequence is defined to be a function /whose


domain is the set of positive integers. Then /( 1 ) is called the first term, /(2) the
second term, and in general /(/)) is called the /ith term of the sequence/. From this

point of view, the sequence

"
'
IL il
4 3 '
^
16
_ J_
/(-''
would be identified with the function / whose domain is the positive integers and
which is defined by f(n) = 6 - {3/n~).
The definition of a sequence as a function not only has the advantage of technical
precision; it also permits many of the ideas previously developed for functions to be
applied directly to sequences. If you are so inclined, we encourage you to regard
sequences as being functions; however, in this book, we deal with sequences more
informally.
We use the notation {a,,} as shorthand for the sequence whose nth term is a„ . For
instance, {/;} denotes the sequence

1, 2, 3. 4, 5, 6,

and

(-1)"
2n ^1
- 1

denotes the sequence

' ' '


1
SECTION U.l SEQIENCES 633

DEFINITION 1 Convergence of a Sequence

We write Hm^ a„ =L and say that the sequence {«„} converges to the limit
^ L
provided that for each positive number e, there exists a positive integer A^ (possi-
bly depending on e) such that

\a„ ~ L\< e whenever n ^N

A sequence that converges to a limit is called a convergent sequence; a sequence


that is not convergent is said to be divergent.
In dealing with an unfamiliar sequence, it's
a good idea to write out the first few
terms to see whether any "trend" is developing.
However, you must be careful
because the tail end— not the first few terms— determines the
convergence or diver-
gence of a sequence.

In Examples 1 and 2. determine whether the given sequence converges or diverges.


If it converges, find its limit.

EXAMPLE 1
10"-

SOLUTION Here the sequence is

\
1 \__ 1
1.
10 100 1000 10.000 10""

and it is clear that the terms are steadily becoming smaller and smaller. By choosing
n large enough, we can make |(1/10"-') - 0| = 1/10""' as small as we please;
hence, by Definition 1. the sequence converges to the limit 0.

EXAMPLE 2 {(-I)"}

SOLUTION This sequence,

-1. 1. -1. 1. -1 (-1)". . . .

jumps back and forth forever between - 1 and 1 ; hence it cannot approach a limit
and is therefore divergent.

Properties of Limits of Sequences

The calculation of limits of convergent sequences is carried out inmuch the same
way as the calculation of limits of functions. In fact, the two procedures are closely
related, as is indicated by the following theorem.

THEOREM 1
Convergence of Sequences and Functions

Let the function/be defined on the interval [1, 3=), and define the sequence {a„}
by a„ =f(n) for each positive integer n. Then, lim f(x) =
if L, it follows that
= '-'^*
lira a„ L.

The proof of Theorem 1 is left as an exercise (Problem 48).


634 CHAPTER II INFINITE SERIES

KXAMPLE Show that the sequence converges, and find


3
1^1 its limit.

SOLUTION The function is indeterminate of the form ^^/^ at +^, so we


A
can apply L'Hopital's rule to obtain

In .V l/.r
lim = lim =

/ In « \
f
In n
By Theorem 1, lim I 1
= 0; that is, the sequence )
\
\ converges to the
„_ + , \ n ' i-
/I J

limit 0.

The following properties of limits of sequences are analogous to the properties of


limits of functions (see Section 1.8); therefore, we simply assume them here and
illustrate their use by examples.

Properties of Limits of Sequences

Suppose that the sequences {a,,} and {/;„} converge and that c is a constant. Then:

1
SECTION II. I SEQUENCES 635

SOLUTION Dividing the numerator and the denominator of the given fraction by
n- and applying Properties 1 through 6, we obtain

3/;- + 7« + 1

Iim ; =

^+- 8n- - 5/! + 3


636 (HAKIKR II INFINITE SF.KIICS

Monotonic and Bounded Sequences


2/1
Consider the sequence . and notice that its terms
5/1 + 3

2 4 6 8 10

8 13 18 23 28 5« + 3

are steadily growing larger and larger. This can be seen algebraically by writing

2/1 _ 2
5/1 + 3
~ 5 + 0/n)
and noticing that as n grows larger. 3//i gets smaller, so that the fraction grows
larger.
More generally, we make the following definition.

DEFINITION 2 Increasing and Decreasing Sequences

A sequence {a,,} is said to be increasing (respectively, decreasing) if a„ < a„+i


(respectively, «„ > a„+i) holds for all positive integers n.

A sequence that is either increasing or decreasing is called monotonic; other-


wise, it is called nonmonotonic.

In Examples 8 to 10. determine whether the given sequence is increasing, decreas-


ing, or nonmonotonic

2/1 + 1
EXAMPLE 8
3/i - 2

Here,

2/1 + 1 2(/i + 1 ) + 1 2/1 + 3


and
3/1-2 3(/i + 1 )
- 2 3/1+1

Hence, for any n.

In + 3 2/1+1 (3/1 - 2)(2/i + 3) - (2/i + l)(3/i + 1)


(i„+\ - a„
3/1 +1 3/1-2 (3/1 + l)(3/i - 2)

(6/1^ + 5/1 - 6) - (6/r + 5/1+1) -7


(3/1 + l)(3/i - 2) (3/1 + 1)(3/! - 2)

If /I is a positive integer, then 3/i + 1 > and 3/i — 2 > 0, so it follows that

a„+i — a„<0 that is. a,, > fln+l

Therefore, the given sequence is decreasing.

EXAMPLE 9 sin

SOLUTION The sequence is

1, 0, -1,0, 1, 0, -1, 0,

and it repeats the pattern 1,0, —1,0 again and again; hence, it is nonmonotonic.
SECTION 11.1 SEQLIENCES 637

n + 1
EXAMPLE 10
n + 2

SOLUTION Consider the function

A+ 1
/(-v)
x+ 2

(.V + 2)- 1
- (A- + !)•
Since /'(a) = > for A > 1
(A- + 2)- (A + 2)-

it follows that /is increasing on the interval [1. x). in particular.

fin) < fin + 1)

holds for all positive integers n: that is, the given sequence is increasing.

Consider the sequence )


1 . and notice that none of its terms
^ n + \ I

J_ ^ ^ 4
2 3 'T" y
is larger than 1 . Thus, we say that 1 is an upper bound for the sequence ]

Similarly, since no term in the sequence is smaller than 0, we say that


n + 1

is a lower bound for the sequence. More generally, we make the following
definition.

DEFINITION 3 Bounded Sequences

A number C (respectively, a number D) is called a lower bound (respectively,


an upper bound) for the sequence {a,,} if C< a„ (respectively. a„^D) holds
for every positive integer /(.

If the sequence {«„} has a lower bound (respectively, an upper bound), it is said to
be bounded from below (respectively, bounded from above). A sequence is said
to be bounded if it is bounded from both below and above. For instance, the
sequence {sin n] is bounded from above by 1 and from below by - 1 : the sequence
{3"} bounded from below by 3. but it is not bounded from above: and
is the sequence
{(-1)"3"} is bounded neither from above nor from below.
It is easy to show that a sequence {a,} is bounded if and only if there exists a
positive constant M such that
jfl„| <M
holds for all positive integers n (Problem 56). If a sequence converges, it must be
bounded (Problem 54); however, a bounded sequence need not converge (Prob-
lem 55).

EX.AMPLE 11 Determine whether the sequence

|(-1)-
3« + 1

is bounded from above or from below.


638 tHAKlKR II INFINITK SERIES

For ever)' positive integer n. we have

I
2/1 I
2/1 2
(-1)"
3/j + 1 I 3/1+1 3 + (1//7) 3

so the given sequence is bounded both from above and from below — from above by
§ and from below by -§.

Imagine a sequence «, at. «? a„. whose terms are growing steadily


. . . .

larger but which has an upper bound D. Thus,

a, < At — flj — — fln — On* I


^ • • • <D
Figure I If we think of the terms of this sequence as corresponding to points on the number
line (Figure 1 ), then we see an inevitable ""pileup" of these points to the left of the
point corresponding to D, and we easily persuade ourselves that the sequence must
be converging to a limit somewhere in the interval [uf. D]. In this instance our
geometric intuition is confirmed by the following theorem, whose proof is beyond
the scope of this book.

THEOREM 2 Convergence of Bounded Monotonic Sequences

Every increasing sequence that is bounded from above is convergent. Likewise,


every decreasing sequence that is bounded from below is convergent.

//I Examples 12 and 13, use Theorem 2 to show that the given sequence is conver-
gent.

EXAMPLE 12

SOLUTION Consider the function /(.v) = .v/e". and notice that

xe '
1

f'M = < for .V > 1

ie')-

Hence, /is decreasing on [1, ^c). it follows that/(/!) >/(/i + 1) for all positive

integers /;; that is, the sequence {n/e"} is decreasing. Since all terms of the sequence
are positive, it follows that it is bounded from below by the number 0. Hence, by
Theorem 2, the sequence converges.

5"
E EXAMPLE 13
3^
SOLUTION Offhand, it isn't clear whether this sequence is increasing, decreas-
ing, or nonmonotonic. In order to gain some insight into its behavior, we calculate a
few terms, rounded off to (say) three decimal places:

1.667, 1.389, 0.772, 0.322, 0.107, . . .

Here we have an indication that the sequence may be decreasing. To prove that it

really is decreasing, denote the general term by a,,, so that

5"
SECTION 11.1 SEQUENCES 639

We want to prove that

On > a„+i

holds for all positive integers n. Because both a„ and «„+, are positive, the last
inequality is equivalent to

> 1

Now,
5" (" + 1)!

5^ 3"

= y 1

3-(/i+ 1)= — («+


3
1)

Hence, what we want to prove is that

hn + 1) > 1

holds for all positive integers /;. But this inequality is obviously true, since it is

equivalent to the condition that

/! + 1 > S for n > 1

Therefore, the given sequence is decreasing, just as we suspected. All terms of the
sequence are positive, so is a lower bound. Since the sequence is decreasing and
bounded from below, it converges by Theorem 2.

We bring this section to a close by showing how Definition 1 is used to make


formal proofs of theorems on limits of sequences. For this purpose, the following
theorem (which is intuitively quite clear) serves as a typical example.

THEOREM 3 Convergent Monotonic Sequences

The limit of a convergent increasing (respectively, decreasing) sequence is an


upper bound (respectively, a lower bound) for the sequence.

We prove only the part of the theorem pertaining to increasing sequences, since the
proof for decreasing sequences is quite similar and only requires reversing some
inequalities (Problem 60). Thus, assume that {a„} is monotone increasing and that
lim a„ = L. We must prove that all the terms of the sequence are less than or

equal to L, If this were not so, there would be at least one term, say a, with L , < a^.
Thus, let e = fl^ — L, so that e > 0. By Definition 1, there exists a positive integer
N such that \a„ — i,| < e holds whenever n > N.
Now, choose the integer n to be larger than both A' and q. Since q < n, it follows
that fl^ < a„, so that L< a^^ a„ and a„ - L > 0. Consequently,

a„ —L= \a„ — L\ < e = Qg —L


from which it follows that a„< a^, contrary to the fact that a^^ a„. Therefore, the
supposition that there is a term a^ with L < a^ leads to a contradiction. It follows
that no term such as a^ can exist. Thus, L is an upper bound for the sequence, and
the theorem is proved.
640 CHAPTER II INHMTF. SKRIES

Problem Set 11.1

In Problems 1 to 4. write out the first six terms of each sequence. 3/1"
31 {(-l)"'l 32
Also write the 100th term.

(-1)" + 34 {V2}
33 I

5/1 - 2 J
n + 1

In (>i + 1) 1
1
+
1
35 36
4 2 /I + 2 1 In n
n' + 5

/I + 5 '

37
In Problems 5 to 8. find an expression for the general term (;ith /r + 6/1 + 4
term) of each sequence.

5 1.1.2.1.3. 6 1. 0, 1. 0. I. 0. . . .
I /I + I J

-, 1 1 i i 1
8 1. 9. 25.49. 81. 121.
41I1--I
In Problems 9 to 26. determine whether each sequence converges or
diverges. If it converges, find its limit. I
sin imr/A) 1 \-3-5-l an - I)
43

100 \
10
n > . 5/r + 1 45 Give an example to show that the sum {a„ + b,,} of two un-
bounded sequences {a„} and {b„} can be a bounded sequence.
n^ - 5/1 In- + I

lir + 2/1 9/r + 5 46 Let a„ = /i!/2".

(a) By writing
14 2 3
3/1 + I J 10" 1
1

,5{i!LlJL„
I +
-|
2/1 J
16 14^'
U" - e~"
noting that all factors after the first three are greater than or
/I I
equal to 2 and that there are /i — 3 such factors, conclude

In (11+ 1) that a„ > 2"~' for n > 3. 4 •

18 {I +(5)" -(f)"}
n + I (b) If AT is any positive number, use the result of part (a) to show
that a„> K holds provided that
ln(l//i)
19 20 {In (e" + 2) - In (e" + I)}
In (;i + 4) (4K/3)
/I > 3 + In
In 2
I

21 22 {In (e" + 2) - n] (c) Explain why the sequence {a„} has no upper bound.
V/i- + 1
- n

'/""'} 47 (a) Write out the first six terms of the sequence
24 {/I

{/I + (/I - !)(/! - 2)(/i - 3)(/i - 4)(/i - 5)(/i - 6)}


5 \"
25i(i+-Ln 26 {(' + I
What
(b) is the seventh term of the sequence in part (a)?

(c) What can you conclude about determining the general term
In Problems 27 to 44, determine whether each sequence is increas- of a sequence from an examination of its first few terms?
ing, decreasing, or nonmonotonic and whether it is bounded from
above or below. Indicate whether the sequence is convergent or 48 Prove Theorem I

divergent.
49 Prove Property 6 on page 634.

2/1 + I
50 Prove Properties 7 and 8 on page 634.
27 28 {sin /itt}
3/1 + 2
51 If r is a constant, determine whether the sequence {c"} converges

29 {3" - n) 30
3"
or diverges for (a) (< -I, (b) r = -1, (c) — 1 <c< I,

I + 3" (d) ( = 1, and (e) c > I.


SECTION 11.2 INFINITE SERIES 641

52 Suppose that {a„} and {h„] are two sequences whose correspond- 57 Suppose that the sequence {a,,} is convergent and satisfies the

ing terms agree from some point on; that is. suppose that there condition that a„+] = A + Ba„ for all positive integers /;, where
is a positive integer k such that a„ = b„ holds for all n a k. If A and B are constants and B ?^ 1 . Find lim a„. (Hint: Take the
lim a„ = L, prove that lim b„ = Z., too.
limit as n -
+00 on both sides of the equation a„+i =
= A + Ba„.)
53 Suppose that the sequence {a,,} has the property that a„+i
2(a„ + a„-i) for n > 2 and that ui = 1 while ch = 3. 58 In advanced calculus. Stirling's formula is proved

(a) Write out the first eight terms of the sequence {o,,}.

(b) Use mathematical induction to prove that


Vliviriy—] < n\ < Vlinr [
— ) ( 1 +
-
12n 1 /

(-1)" Use Stirling's formula to show that the sequence


3.2"-

(c) Find lim a„.


1—1
U"7l!J

has an upper bound.


54 Assume that the sequence {a„} is convergent. Show that {a„} is

bounded. 59 Let |a| < 1 . Show that (a) {na"} converges to the limit and (b)
{«"«"} converges to the limit 0.
55 By means of a suitable example, show that a bounded sequence
need not be convergent. 60 Prove the part of Theorem 3 that pertains to decreasing se-
quences.
56 Show that the sequence {a„} is bounded if and only if there exists
a positive constant M such that \a„\ < M holds for all positive 61 Assume that a and b are constants with a > 1 and h > 0. Show
integers ;;. that the sequence {n''/a"} converges to the limit 0.

11.2 Infinite Series

An indicated sum

Oi + 02 + aji + • • •
+ a„ +
of all the terms of an infinite sequence {a,,} is called an inflnite series, or simply a
series. Using the sigma notation introduced in Section 5.1, we can write this series

more compactly as

a*

The numbers aj , aj. a^. and so forth are called the terms of the series, and a„ is

called the nth term, or the general term, of the series.


Although we cannot literally add an infinite number of terms, it is sometimes
useful to assign a numerical value to an infinite series by means of a special defini-
tion and to refer to this value as being the "sum" of the series. This is accomplished
by using the "partial sums" of the series.

The sum s„ of the first n terms of a series ^^ = i


a^ is called the nth partial sum
of the series; thus.

s„
642 ClIAKrER U INHNITE SERIES

The sequence {5,,} is referred to as the sequence of partial sums of the series.
Notice that for each positive integer n.

v„. :
= .v„ + «„^|

For instance, the first few partial sums of the infinite series

+ —+—+—+ + 2"-
1

2 4 8 16

are as follows:

First partial sum = i^i


= 1

Second partial sum = S2 = 1 + 2 = 1-5

Third partial sum = ^3 = 1 + 2 + 4 = 1-75

Fourth partial sum =54=l+f + i + s=l .875


Fifth partial sum = .?? = 1 + i + 5 + s + A= 1.9375

We can continue in this way as long as we please. In this case, we find that the
sequence s„ of partial sums

1, 1.5. 1.75, 1.875, 1.9375, . . .

seems to be approaching 2 as a limit; for example, the 25th partial sum is

i^, = 1 + —1111
— — + + + + ••• + — 1

2--'
1.99999998
2 4 8 16

Here it is not difficult to verify that the sequence of partial sums really does con-
verge to 2 as a limit; hence, it seems reasonable to define the sum of the infinite
series to be 2 and to write

1 + —+—+
1 1
+ +
2 4 16

More generally, we make the following definition.

DEFINITION 1 Convergence of an Innnite Series

If the sequence {s,,} of partial sums of the infinite series 2jk= "a converges to a 1

limit S = lim .v„, we say that the infinite series "^J^- a^ converges and that its 1

sum is S.

If the infinite series ^t=\ cii, converges and its sum is 5, we write

c - V
5-2. I'k

Of course, an infinite series that does not converge is said to diverge.


SECTION 11.2 INFINITE SERIES 643

In Examples I and 2, write out the first five terms of the given series,
and then write
out the first five terms of the sequence of partial sums
of the series. Find a formula
for the nth partial sum of the series, determine whether the series converges or
diverges, and if it converges, find its sum S.

EXAMPLE 1 2 k{k + 1

SOLUTION Here,

X 1 1,11
+ -— + — + — +
1 1

= ,
k{k+ 1) 2 6 12 20 30

The first five partial sums are

Si = a,
_ 1

52 = 5, + a. = I + I = §

53 = 52 + a, = § + 1^ = f

54 = 53 + ^4 = 4 + 20 = 3

55 = 54 + fls = 5 + 30 = 6

These first five partial sums suggest that the «th partial sum might be aiven bv the
formula
n
s„ = s,,^^ + a„ =

This can be confirmed by mathematical induction on


/;. but we present an alterna-
tive argument. By partial fractions.

1
Ok
k{k+l) k k+l
Hence,

•s« = S flA = S (—
k k+ \
k=l

2 ^ ^
2 3 3 4 „ „ + 1

= 1- '

n + I n + I

Therefore, we have
1- " 1
am s„ = ,-
lim = Um = 1
1 + (lAO

Hence, the series converges and X k{k +


= l

;~i 1
644 CliAKlKK II INFINITE SERIES

EXAMPLE 2 2 k(k - 1

SOLUTION Here,

S kik - 1 ) = + 2 + 6 + 12 + 20 + • •

The first five partial sums are

ii = fli =0
S2 = St + aj = +2 =2
Si = S2 + ai. = 2 +6 =8
54 = i-3 + 04 = 8 + 12 = 20

^5 = S4 + as = 20 + 20 = 40

Using the formulas for the sums of successive integers and successive squares from
Section 5.1, we find that

s„ = S kik - 1) = E (k- - /t) = ^ k' - ^ k


k=\ k=\ k=l k=l

n(n + \)(2n + 1) n(n + I) _ niir - 1)

6 2 " 3

Therefore, we have ,
-,


n(n- 1)
lim s„ = lim = +^
"- + » "- + » 3

Thus, the sequence of partial sums — hence also the given series — diverges.

The series

i k(k
'

+ 1)
k=\

of Example 1 . when rewritten in the form

t^k A+li
is called a telescoping series because of the cancellation that occurs in the calcula-
tion of its partial sums. More generally, if {/;„} is a sequence, then a series of the
form

S (bk-lh,,)
k=\

is called a telescoping series. The nth partial sum is given by

s„ = 1, (hi,- /Ji+i) = [hi - h.) + Oh - h,) + •• + (/'„ - ^,+ i)

= /'I
- 1^2 + li>2
- l/>^ + •• + /<„-/?„+,

= ^1 -^,M
SECTION 11.2 INFINITE SERIES 645

Therefore, if lim fo,,^, exists, say, lim b„+i = L. then we have

z! (^/t
~ ^t*i) = lim s„ = lim (bi - b„+i) = bi -L

EXAMPLE 3 Show that the series

3
y
^^ 9k- + 3k-2
converses and find its sum.

SOLUTION By partial fractions.

3 3

9k- + 3k -2 (3k-l){3k + 2) 3k - \ 3k + 2

1 1

b, - b,.
3k - 1 3(k + 1) - 1

where bi, = \/l3k - 1). Therefore.

*=! 9k-
^" ^3k-2"
-"" ,r,
*=1

3(1)
L

-
3k

1
- 1

lim
"---• 3(«
3(A- + 1 )

+
111 -

1)
1

- 1
= —- = —

In the study of infinite series, it is sometimes useful to be able to manufacture a


series Zak= i ^a with a preassigned sequence {i,,} of partial sums. In this connection,
the equations

fli = S\ and a„ = s„ — s„-\ for n > 1

provide the desired series.

EXAMPLE 4 Find an infinite series whose sequence of partial sums is

^ ' I 2/! + 1

Determine whether this series converges, and if it converges, find its sum.

SOLUTio.N Here we have


3n 3{n - 1) 3/1-3
s„ = and i„-i
= =
2n + 1 2(11 - 1 ) + 1 2/! - 1

Hence, for n > 1

3h 3n - 3 ( 2n - l)(3/i) - (3/; - 3)(2n + 1)

(2/! + 1)(2« - 1)

4«- - 1
646 HI AITKR II INKINITK SERIES

Here,

3(1)
«| = 5,
2(1)+ 1

which happens to be the same as the value of

3
when n = 1

4«- - 1

Thus, the desired series is given by

Since the iilh partial sum of the series is s„. it follows that

3/i
Zj ;
:. = lim .V,, = lim — = lim
4k- - \ "- + - "- + - 2n + 1 "- + -2 + (l//!) 2

Geometric Series
By definition, a geometric series has the form

V.
SECTION 11.2 INFINITE SERIES 647

Subtracting the second equation from the first, we have

^n
~ s„r = a — ar" or s„(\ — r) = a(\ — r")

Therefore.

- r"
-"(^- 1

1 r
if r # 1

By Property 7 in Section 11.1. if |r| < 1. then Um r" = 0; hence.

lim 5„ if r < 1

On the other hand, by Property 8 in Section 11.1. if |7-| > 1 . then the sequence {r"}
diverges, and it follows that the sequence {s„} also diverges. In the remaining case in
which \r\ = 1 . so that r = 1 or r = — 1 . it is easy to see that the sequence of partial
sums di verses (unless o = 0). Thus, we have the following theorem.

THEOREM 1 Geometric Series

The geometric series 2*= i


'"^ '
with initial term a # and ratio r converges if

\r\ < 1 and diverges if |r| s 1 . If \r\ < 1 , then

k=l

In Examples 5 to 7, determine whether the given geometric series converges or


diverges, and if it converges, find its sum.

EXAMPLE 5
648 LHAP1 tK 1 1 INFINITE SERIES

EXAMPLE 7 X (i)*

SOLUTION The series is geometric with initial term a = 2 and ratio r = i. Since
|r| = 2 > 1, the series is divergent.

Applications of Geometric Series

Geometric series arise quite naturally in many branches of mathematics, as the


following examples illustrate.

EXAMPLE 8 The probability of making the point "S" in a game of craps that — is,

the probability of rolling an 8 with two dice before rolling a 7 is given by —


36 + (3(;)(3ii) + (36)(36)" + (36)(36)' + " "
"

Find this probability.

SOLUTION The displayed series is geometric with initial term a = ^ and ratio

' = 36- Its sum is accordingly given by

a _ J6 ^ _5_
1 -r ~ 1 -1 ~
11

Therefore, the probability of making the point 8 is fi-

EXAMPLE 9 A simple air pump is evacuating a container of volume V. The cylin-


der of the pump, with the piston at the top. has volume v. and the total mass of air

in the container at the outset is M. On the Hth stroke of the pump, the mass of air

removed from the container is

Mv / V r"'

Assuming that the pump operates "forever," what is the total mass of the air

removed from the container?

SOLUTION The total mass removed is given by the sum of the infinite series

Mv Mv / V \ Mv / V \- ___ Mv / V
V+ V V + v^V + v' V+ \' V V + \' / V+ I' W+ »'

with initial term a = Mv/(V + v) and ratio r = V/{V + v). Its sum is

Mv
''^'
-
-M
1 r / V''

\V + v'
Thus, all the air is removed if the pump operates forever. (Of course, no pump is

perfect — valves leak, air leaks around the piston, and so forth — so our answer is

only of theoretical interest.)

EXAMPLE 10 Express the infinite repeating decimal 1.267676767 ... as a ratio


of whole numbers.

SOLUTION 1.267676767 . . . = 1.2 + 0.067 + 0.00067 + 0.0000067 + • •


_
— 12
+ , , 67 ,

+
67
+ ,
67
10.000,000 +
4. . . o)
10 (1000 100.000
SECTION 11.2 INFINITE SERIES 649

The geometric series in tlie parentheses has initial term a = tooo and ratio r = too;
hence, it converges and its sum is given by
67 ^-,
a _ 1000 _ 0/
\-r ~ 1 - T^ " 990

Therefore,

Problem Set 11.2

In Problems 1 to 6. write out the first five terms of each series, and
then write out the first five terms of the sequence {s„} of its partial
19 2 (U 20 E (-!)*
k=0
sums. Find a "simple" formula for the ;ith partial sum s„ in terms of
n. determine whether the series converges or diverges, and if it 3*-
converges, find its sum S = lim s„. 21 s (for' 22 2 ,,.,
4*
k=\

23 1-1 + 1-1 + 1-1 + 1-1 +


1 V 5
2 S In ( 1
- -
^t', {k+ \){k + 2) k + 3 24 0.9 + 0.09 + 0.009 + 0.0009 + •

3 2 k{k+ 1) 4Stt 25 E 5" 26 E e'-*


k- + 2k *=i

5l; 6X 2A- + 1 27 E 28 S
(2A-- l)(2/:+ 1) A-(A- + 1 )- k=\

29 I
- 100 ~ 1000
In Problems 7 to 12. find an infinite series with the given sequence
of partial sums, determine whether this series converges or diverges, 30 The series 1-I + 1-1 + 1-1+--- + (-1)""' + is • • •

and if it converges, find its sum. geometric with ratio r= - 1; hence, it diverges. Thus, the cal-
culation 1-I + I-1 + 1-I+--- + (-1)""' + = • •

(l-I) + (l-I) + (I-I) + --- = + + + --- =


7 {5„}
n+ I
8 K} = ;i + 5 must be incorrect. What is wrong with this calculation?

10 {s„} = {n} In Problems 31 to 34. express each repeating decimal as a ratio of

whole numbers by using an appropriate geometric series.

11 M= {i -(-m 12 {s„
1

31 0.33333 . 32 1.11111 . . .

33 4.717171 34 15.712712712
In Problems 13 to 29. find the initial term a and the ratio r of each
geometric series, determine whether the series converges, and if it

converges, find its sum. 35 Is it true lim 2^ O/t = Z/ o^l Explain.

14 E (§/-• 36 Find lim^ (l1


lim ( + -j +
^ 3-"
/t=i

37 In a game of craps, the probability that the shooter wins — that

15 S (-§)*-' 16 i4r is, rolls 7 or 1 1 on the first throw or rolls a number other than 2,
3, or 12 and then, on a successive roll, repeats this number

17 1 +f + A+ before rolling a 7 — is given by the repeating decimal


0.4929292929. . . . Express this probability as a ratio of whole
numbers.
650 CHAPTER II INFINITE SERIES

38 A beaker originally contains 10 grams of sail dissolved in 1000 (c) By part (a), the total distance flown by the fly is given by
cubic centimeters of water. The following procedure is per-

formed repeatedly: 250 cubic centimeters of salt water is poured V?, 2Vd /V- v\" '

1
1 meters
out, replaced by 250 cubic centimeters of pure water, and the ^i V + v\V+ V '

solution is thoroughly stirred, (a) After the procedure is repeated


n times, how many grams of salt have been removed from the Find this distance by summing the series.

beaker? (b) If the procedure is repeated "infinitely often." how (d) Using part (b), set up and sum a series to determine the total

much salt remains in the beaker? time required for Abner to reach the wall.

(e) Determine the total distance flown by the tly without sum-
39 .A rubber ball rebounds to 60 percent of the height from which it

was dropped. If it is dropped from a height of 2 meters, how far ming an infinite series.

does it travel before coming to rest?


41 Let 2t=i Oi be a given infinite series, and let {i„} be its se-
40 Abner starts walking directly toward a brick wall d meters away quence of partial sums. Define the sequence {/?„} by
at a constant speed of v meters per second. At the same instant, a
if /I = 1

tly departs from Abner"s forehead flying directly toward the hn


brick wall at a constant speed of V meters per second, where if n> 1

V> V. Upon arriving at the brick wall, the fly immediately turns
each n> Show
for integer \. that the series 2*=i ^* '^
about and flies back to Abner's forehead at the same speed, V
term by term exactly the same as the telescoping series
meters per second. The fly continues to shuttle between Abner's
2*=i (''<:
""
''*+!)• Thus, conclude that any infinite series can
forehead and the wall in this manner until Abner finally reaches
be rewritten as a telescoping series.
the wall.

(a) Show that on the «th round trip from Abner's forehead to the 42 Show that

wall and back, the fly covers a distance of

2Vd 2 (bt - b,+2) = (^1 + ^2) - (b„+i + b„^2)


k=\
V+ V V V+ V /
{Hint: b, - bt^. = ibt - b,^0 + (fe^+i - bt.2)-]
(b) Show that the fly requires

2d / V^ - V
)
43 Using Problem 42, show that if {b„} is a convergent sequence
seconds
V+ v\V+v' with lim b„ = L, then the series 2]*=i ^^t — bt+i) is conver-

for the /ith round trip. gent and Xr=i (.bt - b^+i) = b^ + bi - 2L.

11.3 Properties of Infinite Series

In Section 11. 2 we were able to find the sum of certain infinite series by finding
"nice" formulas for their partial sums. For instance, the wth partial sum of the
telescoping series

Zj (l>k
~ bii + i) is simply h^ - h„+i
k=l

and the iiih partial sum of the geometric series

is just
"(-Vtt)
k=\

Unfortunately, it is not always so easy to find tidy fomiulas for nth partial sums:
hence, it is important to develop alternative methods for determining whether a
given series converges or diverges and for dealing with its sum if it converges.
SECTION 11.3 PROPERTIES OF INFINITE SERIES 651

Some of these methods are consequences of the general properties of infinite series
which we develop in this section.
The following theorem gives an important property of convergent series.

THEOREM 1 Necessary Condition for Convergence

If the infinite series 2]*=! «/t converges, then lim a„ = 0.

Let {s,,} be the sequence of partial sums of the series 2*=i ^k- If 2?=i ^k con-
verges, then by definition the sequence {s^} converges and lim s„ = S =

2r=i Oa- As n -^ +=c, then also /; — 1 -* +», so that lim i„_i = S. Notice

that a„ = s„ — s„-i; hence,

lim a„ = lim (s„-s„-\)= lim s„ - lim s„_i=S-S =

EXAMPLE I Given that 2^


I.- T

fi-
converges,* find li:

" "•
2"

SOLUTION By Theorem 1, because 2j


V —
2*

I"-
converges, lim
"- + "
—" =2"
0.
*=i

Theorem 1 can be used to show that certain series diverge: An immediate conse-
quence of Theorem 1 is that if lim a„ is not 0, then 2]"= i «a cannot converge. We
record this fact for future use as follows.

THEOREM 2 Sufficient Condition for Divergence

If lim a„ does not exist or if lim a„ exists but is different from zero, then the

series 2^^=! ^a is divergent.

In Examples 2 and 3, use Theorem 2 to show that the given series diverges.

EXAMPLE 2
^ k+
2j
\

SOLUTION Since lim = lim ( 1 + — ) = 1 t^ 0, it follows that

2j —— ;
diverges, by Theorem 2.

*As a matter of fact, the series

2 —
converges to e" — 1. For the proof, see Example 3 on page 696.
652 CHAKIER 11 INFINITE SERIES

EXAMPLE 3 2 (-1)*

SOLUTION Here, lim (-1)" does not exist. By Theorem 2. therefore,

2r=i (-1)* diverges.

Caution: Do not misinterpret TTieorem 1 . It says that the general term of a con-
vergent series must approach zero, but it does not assert the converse.

There are lots of divergent series whose general terms approach zero.

EXAMPLE 4 Consider the series

S In
k+ 1

and notice that the general term approaches zero:

lim In = In I lim In 1 =0
«-+- n + n + 1

From this, can we conclude that the series 2^ In converges?

SOLUTION No! Just because the general term approaches zero is no guar-

antee that the series converges. In fact,


V
2j In
^'
can be rewntten as
"*" '
A=i ^'

Zj [In A:
— In (A- + D], a telescoping series which diverges since

lim In (/I + 1) = +=c.

Many properties of infinite series are analogs of corresponding properties of


sequences. For instance, we have the following theorem.

THEOREM 3 Linear Properties of Series

(i) If sr=
SECTION 11.3 PROPERTIES OF INFINITE SERIES 653

PROOF We indicate the proof of part (i) and leave the remainder as exercises (Problems 35
and 37). Thus, let s„ = 2]l'=i «* and t„ = 21l'=i ^a be the nth partial sums of the
two given series. Then

s„ + '„ = S a, + S /'A
= S («A + Ih)
k=\ k=\ k=]

is the «th partial sum of the series 2^=1 ((h + bk)- Since

lim (s„ + l„) = lim s„ + lim t,, = 2 cu + S ^A


k=\ A=l

Zlk= 1 (O/t + b^) converges and its sum is given by

S (a* + bk) = S a«: + S fe*

A-=I k=\ k=\

By a similar argument (Problem 35),

Z (a*. - ^a) = S flA - 2 bk

EXAMPLE 5 Find the sum of the series 2 ( , ,


+ —-—

SOLUTION Notice that Z, is a geometric series with initial term a = 5


^-_i
A=l ^
and ratio = 2; hence it converges and

\-r 1
T=10
V
Similarly, 2^ ~^jr—c converges and
1

A=l ^

It follows from part (i) of Theorem 3 that 2 (


^._|
H
3*T~^]
converges and
A=l -

2^ 10 +
3 23

The following useful theorem is an immediate consequence of part (i) of Theo-


rem 3.

THEOREM 4 Divergence of a Series of Sums


If the series ^1=^ a^ converges and the series 2a'=i /'a diverges, then the series
2Jk=\ (Qa + ^a) diverges.
654 CHAFIER II INnNlTE SERIES

Assume the contrary; that is, 2*=! (^i + ^t) converges. Since
suppose that

2]r=i Ok converges, Xr=i l(a* + h) -


= 2*=! ^* a'so converges, by part (i)
a*]

of Theorem 3, contradicting the hypothesis that X*=i ^* diverges. Hence, the

assumption that 21=1 («* + ^k) converges must be wrong; that is. Xr=i (a* + ^*)
must diverge.

EXAMPLE 6 Determine whether the series

k
S(ln ^)
k + 1

converges or diverges.

SOLUTION As we saw in Example 4 on page 652, the series


V
Zj In
^

* *=i *^ '

diverges. However, the series Z, —_i :~ is a geometric series with initial term
*=i -

a = - 3 and ratio r = i; hence, it converges. Therefore, by Theorem 4. the series

k \

M'-'Th-i)-M^^ In
k+ 1

must be divergent.

Notice that even if both series Xr=i a* and ^l=\ b^ are divergent, the series

2r=i (a* + bk) may be convergent. For instance, let a„ = n and b„ = -n for all

positive integers n.
Calculations with infinite series are often simplitled by various manipulations
involving the summation index. Such maneuvers are usually obvious and virtually
self-explanatory. For instance, it is not necessary to begin a series with k = I. Thus,
we can write

V
Z—2*r=l+ — + — + — +
1 111 •••
.0 2 4 8

S In = In — + — + —3 +
1

In
2
In
2 3 4

and so forth. Also, there is no particular reason to use the symbol k for the summa-
tion index. Indeed it is possible, and often desirable, to change the summation index
in an infinite series in much the same way that variables are changed in integrals.

For instance, in the series Xr=i (1/2*"'), let us put j = k - \, noting that) =
when A: = 1 . Then we obtain

2J
A— 1 / — yf

As the following theorem shows, the first few terms of an infinite series have no
effectwhatsoever on the convergence or divergence of the series // is only the tail —
end of the series that matters as far as convergence or divergence is concerned.
SECTION 11.3 PROPERTIES OF INFINITE SERIES 655

THEOREM 5 Removing the First M Terms from a Series


MM is a fixed positive integer, then the series Xit=i ^k converges if and only if

the series 2^r=.v/^i «* converges. Moreover, if these series converge, then

A/ r-

For n > M. we have

s z^ ak+ 1
Za Ok
k=\ k=.\t~l

Since 2*=i «*. '^ a constant, it follows that lim X!t=i "k exists if and only if

lim 2jk=M+i Ok exists; that is, 2^=1 «* converges if and only if ^T-.w+i a^

converges. Assuming that these limits do exist and taking the limit on both sides of
the above equation as « -^+^, we obtain

Z a* = 2 Ok
k=M^ 1

Let us illustrate Theorem 5 using the geometric series 2*=! '^r'' '
with |H < 1-

Notice that

2 ar^-' = flr" + ar'' + ar


/t = .V/+ 1

is also a geometric series with initial term ar^'. ratio r, and sum equal to
ar'^Vd - r). Also. X'i^=i
"'"^~' is just the Mlh partial sum of 2r=i ar*'~':
consequently.

2 1 - r

Thus, the equation of Theorem 5 becomes

2 ar'-' = X ar'-'
k= 1 *= 1 k=M+l

an obvious algebraic identity.


By Theorem 2 in Section 11.1. if the sequence {s„} of partial sums of a series

Zjk=i Ck is monotonic and bounded, then the sequence {s,,} — hence also the series

2jk=\ a* — is convergent. In particular, we have the following theorem.


656 CHAPTER II INFINITE SERIES

THEOREM 6 Convergence of a Series of Nonnegative Terms Whose Partial Sums


Are Bounded

Let 2lr= 1 «* be an infinite series whose terms are all nonnegative (that is, a* ^
for all k). If the sequence {s„} of «th partial sums of 2)*= i «* is bounded above
(that is, s„^ M for all /j, where A/ is a constant), then the series 2^k= i
a* is

convergent.

Since s„+\ = s„ + a„+i and a„+| s 0, we have s„+i s: s„ for all integers n ^ \.

Thus, {j,,} is an increasing sequence that is bounded above. It follows (Theorem 2,


page 638) that {s,,} is convergent; hence, the series 2j/:=i "a 's convergent.

V A-
- 1

EXAMPLE 7 Use Theorem 6 to show that the series 2j P converges.

SOLUTION Clearly, each term of the given series is nonnegative. By Theorem 6,

the series converges if the sequence {,v„} of partial sums

k- 1

.„ = s /t-2*

is bounded. Notice that {k - \)/k < 1, so that

k- 1

k-l*" A- V 2 / \ 2

Therefore,

_ V A- V / 1\* _ - (?)"
1_^
1 1 1
J
"~,^, k-2' '^^.^l' 2'
1-i 2" 1-i

A- - 1

so {s,,} is bounded above by M= 1 and 2^ ~; zj~ converges.


k-2'

Problem Set 11.3

In Problems 1 to 8, show that each series diverges by showing that In Problems 9 to 14, use the linear properties of series to find the
the general term does not approach zero. sum of each series.

Yl 5k + 1 vi'"(T^) 9 2 Hht + (h'] 10 1 [(i)*-'-(-i)*^']


,
k=\ k=\

3lc^ + 5k
¥i.
^^ lk^+ \3k + 2
4S 3/ + 7
" ^ llir^-iif^ »2 i l2(^/-3(-i)^-']

5 2> sin 6S „ V / 2* + 3* I \

, ,
4 cos k

7 2. k sin 8S^
*=i ^
SECTION 11.3 PROPERTIES OF INFINITE SERIES 657

15 Does the fact that Verify the last equation directly without using Theorem 5.

lim —= 25 Use the facts that

guarantee the convergence of the series


s" k(k+ 1)
and
k{k+ 1) M+ 1

to find the sum of

16 Given that
7+1 k(k+ 1)

26 (a) Use Theorem 5 to show that if two series agree, term by

converges for each value of the constant c. find


term, except possibly for the first M
terms, then either the\ both
converge or else they both diverge, (b) Show that changing,

Mm — deleting, or adding
or divergence of a series.
a single term cannot affect the convergence

17 Given that 1 -2 + 3-4 + 5-6 + --- = ln2, find the sum 27 Given that

of the series

18 Criticize the following calculation: Let 2!*=i "'* ~ ^t-i) be a


convergent telescoping series. Then

*=1 k=l k=l

= (b^ + bi + bj + • ) - (bi + bi + ) =bi

19 Show that the series

,_ , +
k(k+ 1)
L k{k 1 A- + 1

diverges.

In Problems 20 to 23, rewrite each series by changing the summa-


tion index from k to j as indicated.

20 2 a/-';; = A--l 21 ^ ,,
'
,, ; j = ^-l
658 tHAKIER II INFINITE SERIES

11.4 Series of Nonnegative Terms


In Theorem 6 of Section 11.3 we showed that a series of nonnegative terms con-
verges if its sequence of partial sums is bounded. In this section we present the
integral test and the comparison tests for convergence or divergence of series whose
terms are nonnegative. We begin with the integral test, which uses the convergence
or divergence of an improper integral as a criterion for the convergence or diver-
gence of the series.

The Integral Test

The integral test is based on the comparison of the partial sums of a series of the
form 2r=i f(l<) and certain areas under the graph of the function/. Geometrically,
the basic idea is quite simple and is illustrated in Figure 1. In Figure la. the area

under the graph of a continuous, decreasing, nonnegative function /between x = 1

and j: = « + 1 is overestimated by the sum /( 1 ) + /(2) + /(3) + • • • + f(n) of the


areas of the shaded rectangles; that is,

fix) dx<f{\)+ f{2) + /(3) + • + fin)

Similarly, in Figure lb, the area under the graph of the same function / between
.V = 1 and .V = /; is underestimated by the sum/(2) +/(3) +/(4) + • • • + fin) of
the shaded rectangles; that is,

fil) +/(3) +/(4) + • • •


+fin) < fix) dx
-'i

Adding /(I) to both sides of the last inequality, we obtain

/(I) +/(2) +/(3) + • • •


+fin) sfi\) + fix) dx
•'i

In summary, we have the following result. Iff is a continuous, decreasing, nonneg-


ative function defined at least on the closed inten'al [ 1 , /; + 1 ] , where n is a positive
integer, then

X ^ /( + /( 2 + /( 3 + • • + /( H s: /( + dx
r-fix)
fi ) (/.V 1 ) ) ) ) 1 )

(See Problems 65 and 66 for an analytic derivation of the preceding inequalities.)


These inequalities are used to prove the following theorem.

THEOREM 1 The Integral Test

Suppose that the function /is continuous, decreasing.


SECTION 11.4 SERIES OF NONNEGATIVE TERMS 659

(i)We have seen that the nth partial sum s„=f{l) +/(2) + + f(n) of the • • •

^
seriesXr= M) satisfies Jf fix) dv < i„ < /(I + /^ /(;t) d[r. If /f /(.v) dLr
i
'
)

converges then s„ < /( , + /" /U) dr < /( + /" /(x) dv, so that {s,,} has the
1 ) 1 )

upper bound M = f{\) + f' fix) dx, and consequently, ^^=\f(k) converges
by Theorem 6 of Section 1 1.3.

(ii) If /r/U) dx diverges, then /""^'/W dx grows large without bound as


n—> + oo; hence, since /""^'/W ^— ^n^ it follows that s„ also grows large
without bound as n^ + x. Therefore, in this case, {s„} diverges, and so
2r=i/(^') diverges.

In the integral test, there is no necessity to start the infinite series at k = 1 . For
instance, to test for convergence or divergence of the series ^t=2fik). we would
use the improper integral /? fix) dx.

In Examples 1 and 2, use the integral test to determine whether the given series
converges or diverges.

EXAMPLE 1
^
2j
1

k~ + 1

SOLUTION The function / defined by fi.x) = -^ is continuous, decreasing,


.r- + 1

and nonnegative on the interval [1, y-). Also,

[' 1 (' dx r _, PI
—-, dx = lim -:; = lim (tan .v)
J, .V- + 1 fc- + « J, .V- + 1 fc--- L I [J

= lim (tan ^
b - tan '1) = = —
fc- + « 2 4 4

Thus, the improper integral — dx converges, and so the series


J, .V- + '
'
1

f
Zj -^
1
converees.
,r, ^ +

EXAMPLE 2 2 1

,^2 k(\n kf*

SOLUTION The function / defined by fix)


= rjr is continuous, de-
x{m x)'
creasing, and nonnegative on the interval [2, ^). Here, by the change of variable
M = In jc, we have

^— dx = «-'/* du = —u^'* + C = — (In .X)-''''* + C

so

lim T7ldx= lim — (In fer"-* - — (In 2)' -*


= +=o
660 CHAPTER II INFINITE SERIES

Thus, the improper integral dx diverges, and so the series


.r(ln ;()'/•*

J^^^iiT^''""^^'^-

The integral test makes it especially easy to study the convergence or divergence
of a /» series, which is by definition a series of the form

s kP

where /> is a constant. When p = 1, the p series becomes

X-
2j— =l+
1

k
1111
—+—+—+—+
2 3 4 5
f:^

and is called the harmonic series.

THEOREM 2 Convergence and Divergence of the p Series

The p series Zj — converges if p > 1 and diverges if /? s 1 . In particular, the

harmonic series z^ — diverges.

If p < 0, then lim


1

= +^ (why?), so that 2^
\-i
— 1

j diverges, by Theorem 2 of
n^ + -^
n ^^j k''

Section 11.3. Thus, we can assume that p s 0. The function / defined by/(.v) =
l/.v'' is continuous, decreasing, and nonnegative on [1, oc), and

/;'-^- 1

if p9^ I

dx = \\-p
In b if p = 1

Thus, for p < 1, lim


h^ + ^
["

J.
—1

.v''
dx = +^. so that the improper integral
"
f"
J,
—1

.X''
dx

diverges, and so does the series Z^ — ;-. However, for p > 1,

b^-p - I I
lim dx = lim =
/)-•+» ^ —p P ~ 1

so that
J.
f"
—I

A"
dx — hence also 2j
,^,
V ——1

k'
is convergent.
SECTION 11.4 SERIES OF NONNEGATIVE TERMS 661

EXAMPLE 3 Test for convergence or divergence,

(a) S -^

(b) i JL

SOLUTION

(a) 2i -75- is a p series with p = 3 > 1: hence, it converges.

(b) Z> —y=- - Z 7777 is a p series with p = 5 < 1: hence, it diverges.

The harmonic series 1 +2 + 3 + 4+-"'isa particularly intriguing series, since


it marks the boundary between the convergent and the divergent/? series. Although
its partial sums 5„=l+5 + 5 + --- + (l//z) become large without bound as
n-^ + ^, they do so rather slowly. To see this, consider the continuous, decreasing,
nonnegative function / defined by/(.v) = l/.v for .v > 1. For this function, the in-
equality

fix) dx </{l) +/(2) + • • •


+/(«) </(I) + fix) dx
J

(Figure 1) becomes

In (« + 1) <x„< 1 + In n

If we put n = 1.000.000. we obtain

13.82 s 5,. 000.000^ 14.82

so that the sum of the first million terms of the harmonic series is less than 15.

There is no ••nice" formula for the sum of the n series 2 — l-f


with p> \. The

function C defined on (1 .
x) by Cip) =2 — ^'''
is called the Riemann zeta function
k=]
and plays an important role in analytic number theor>'.

Comparison Tests
The most convergence or divergence of infinite series are based on
practical tests for
the idea of comparing a given series with a series that is known to converge or
diverge. Geometric series and p series are especially useful in such comparison
tests.

We begin with the following definition.


662 CHAP1ER II INHMTE SERIES

DEFINITION I Domination of Series

Let 2y(i=i «* and 2^^=i h^ be two series whose terms are nonnegative. We say
that the series 2!/t- 1 ''«. dominates the series ^t= i «* if "* ^ b^ holds for all

positive integer values of k.

More generally, if there is a positive integer N such that a^ s h^ holds for all
integers k > N. we say that the series 2*=i ^* eventually dominates the series

THEOREM 3 Direct Comparison Test

Let X*=i "* '^"'^ 2*=i ''< ^^ series all of whose terms are nonnegative, and
suppose that ^T-i ^<. dominates 2]*=i "* *'^'' 'h^' 2*=! ^it eventually domi-

nates X*=i «*)•

(') If 2r=i ^A converges, then 2*=! o* converges.

(ii) If 2/^= 1 «* diverges, then 2*= i


fe* diverges.

We prove the theorem under the hypothesis that 2*=i ^k dominates X*=i ^*-

Since the convergence or divergence of an infinite series is controlled by its ""tail

end," conclusions (i) and (,ii) must still hold if 2r=i fc* eventually dominates

Xr=i flA (Problem 70).

(i) Assume that 2Jk=\ ^a converges to the sum B. Then for any positive
integer n. 2l'=i b^ < 2^=1 = B (see Problem 28 of Problem Set 11.3).
^/i

Since a^ ^ b^ holds for all positive integer values of k. we have


2*=! a* — Zj"k=\ bk — X*=i ^«: ~ B- hence, the sequence of partial sums of
Zjk=\ a* 'S bounded above by B. It follows from Theorem 6 of Section 1 1.3

that 2y*=i ^k 's convergent.

(ii) Assume that 2jk=\ "* '^ divergent. Then the partial sums X*=i ^k ^^'
come large without bound as n^* +^ (see Problem 71). Since

Zj"k=\ Ok £ 2^=1 bk- it follows that the partial sums X*=i bk become large
without bound as n -^ +=c: hence, the series 2*=i bk cannot be convergent.

The choice of a suitable series with which to compare a given series is not always
obvious and inay require some trial and error; however, a geometric series or a
constant multiple of a p series whose form is similar to the given series often works.
For example, to test the series

i— 5* + 1

you might try the convergent geometric series

5*

for comparison, because it closely resembles the given series. If the general term Uk
of a given series is a fraction, then an appropriate p series to use for comparison is

often obtained by deleting all but the highest powers of k in the numerator and
denominator of «*. For instance.
SECTION 11.4 SERIES OF NONNEGATIVE TERMS 663

Series to Be Tested Suggested Series for Comparisoti


664 CHAPTER II INHNITE SERIES

EXAMPLE S y,
a- + 2

SOLUTION We are going to show that the given series dominates the series

^11
2 Vk
S 2 Vk
which diverges because the p series X^^i (1/V^) diverges. Thus, we wish to

prove that

J 1_
2 V^ ^k + l

that is,

k + 2< 4k for /t > 1

A- + 2

Since ^ + 2 < 4A: is equivalent to § s k. it follows that

J 1_ 1

2 Vk 'k + 2

holds for A- > 1, and the given series diverges.

EXAMPLE 6
_, \nk

SOLUTION Because Q <\n k < k for A: > 2, we have \/k < I /(In A), so that the

series Zj dominates the series Zj — • Since the harmonic series Zj — di-

verges, so does the series Zj — . by Theorem 5 of Section 11.3. It follows that the

given series diverges.

The following test is essentially another version of the direct comparison test, but
it is .sometimes easier to apply.

THEOREM 4 Limit Comparison Test

Let Zj'k=\ be a series of nonnegative terms and suppose that 2]"=i ^k 's a
('k

ol positive terms such that


series of
a„
lim —^ = c. where c > 0. Then either both
A
,J-. + CC /j^^

series converge or else both series diverge.

PROOF Since lim —^ = c. it follows that given any positive number e. there exists a

positive integer N such that

"n
I

c
I

< e holds whenever n ^N


SECTION 11.4 SERIES OF NONNEGATIVE TERMS 665

The condition y^ - r < e can be rewritten as -e < —^ - c < e, or as


I
"n I
b„

c-e<-— <c
a„

b„
+ e. Putting
.

e = —2 c
, we see that — < —^ < —
r a 3c
holds
2 b„ 2

for all integers n>N. Therefore, if n > A^, it follows that —b„ < a„ < —b„-
2 2

hence, the series 2j a* eventually dominates the series X— ^a. while the series

^ —bk
^
eventually dominates the series 2 a^. Consequently, if the series
*=i k=\

Z bk converges, then the series 2 —b^ converges [Theorem 3(ii) in Sec-

tion 11.3], and so the series Z, "a converges by the direct comparison test. On
k=\

the other hand, if the series 2 b;, diverges, then the series 2 —bk
-
diverges
*=i k=i

[Theorem 3(ii) in Section 11.3 again], and so the series 2 «* diverges, by the
k=\
direct comparison test.

In Examples 7 and 8, use the limit comparison test to determine whether the given
series converges or diverges.

EXAMPLE 7 y. .,

SOLUTION We use the divergent p series Z, ~y^ for the limit comparison

test. Let a„ be the n\h term of the given series and let b„ be the nth. term of the series

2j i/-^ Then

a„
-— = ,. 1/Vn3 +
— = «-+»
1 ^^
'•m lim —17= lim ^, lim -/^
«-+« «-+=» 1/V^ Vn^+
^
«-+» +
fc„ 1
V ,r' 1

= lim '/ ;
—- = 1

"- + - V 1 +(1//!)'

It follows from the limit comparison test that the given series diverges.

^^^^
EXAMPLE 8 2 +
{5k l)-3*
666 CHAKItR II INFINITE SERIEii

SOLUTION We use the convergent geometric series 2^ 'tt tor the limit com-

parison test. Thus, if a„ is the n\\\ term of the given series and b„ is the nth term of

the series Z, ~t. then


_ 3

—hni + -
— = «-
b„
lim
+-
L (5n +
1/3"
1 )
• 3" J
lim
"- + -
7n
5/1
+
+
3

1 "-+-
lim
7

5
+
+
(3//I)

(1//I)

and so the given series converges by Theorem 4.

The following theorem can be proved by slightly modifying the proof of


Theorem 4. Its proof is left as an exercise (Problem 72).

THEOREM 5 Modified Limit Comparison Test

Let 2jk=\ a* be a series of nonnegative terms, and suppose that 2*=i ^i^ '^ ^
series of positive terms.

(i) If lim —^ = and Zj b^ converges, then 2^ «^ converges,

(ii) If lim —^ = +x and 2^ b^ diverges, then Zi Ok diverges.

/// Examples 9 and 10. determine whether the given series converges or diverges by
using the modified limit comparison lest.

EXAMPLE 9
V
Z,
In
-rp
'^

*= I
'''

SOLUTION We use the convergent p series Zj ~T' f^'' '^e modified limit com-

parison test. If a„ is the nth term of the given series and b„ is the nth term of the

series Zj "t- then


^"
k=i

lim
n- + «
— = "-
a„

b„
lim

(In «)/"*
-^
l//i'
—= lim
'.-^-
In

n
n
=
.

lim
'
In

.V
.v

= lim
'- + «
l/.v

1
= lim
>- + -
—=1

.V

where we have used Theorem 1 in Section 11.1 and L'HopitaFs rule to evaluate the
limit. By part (i) of Theorem 5, the given series converges.

EXAMPLE 10
V2/t + 1
SECTION 11.4 SERIES OF NONNEGATIVE TERMS 667

SOLUTION We use the divergent harmonic series 2j — for the modified Hmit

comparison test. If a„ is the ;!th term of the given series and b„ is the /ith term of the

series 2^ — . then

hm
'-+»
— = "-
a„

b„
lim

1/V2n +
\/n
1

= Hm —
«- + - V2/I +
.
n

lim Vh lim Vh
2/! + 1 "-+- \ 2 + {!/«)

By part (ii) of Theorem 5. the given series diverges.

Problem Set 11.4

In Problems 1 to 20, use the integral test to determine whether each In Problems 21 to 32. use the direct comparison test with either a p
series converses or diverges.

,.-, kVk

3A-
3l
,f ,
k" + 16

2k
5l

\nk
9l \ k

112

tan
13 S
m=l

^^
^, ak+ \){3k+ 1)

17 2 coth n
668 tllAKrER II INHNITIC SKRIES

ri
SECTION 11.5 SERIES WHOSE TERMS CHANGE SIGN 669

11.5 Series Whose Terms Change Sign


The tests developed in Section 1 1 .4 allow us to handle series whose terms do not
change sign. (If all terms are nonpositive, we just multiply by - 1 to convert to a
serieswhose terms are nonnegative.) In this section we consider series whose terms
change sign. The simplest such series is an alternating series w hose terms alternate
in sign, for instance, the series

]__ _ J_ J 1
E (-1/
k
~
2 3 T
1_
+ (-1)"^'
k=l

which is called the alternating harmonic series. Note that a geometric series with a
nesative ratio r. such as

--l-i + —
1 1 1

+ (-l)(^)" +
|/-'i-i)
is an alternating series.

The following theorem, which exhibits an important feature of an alternating


series whose terms decrease in absolute value, will be used to prove a test for
convergence of such a series.

THEOREM 1 Alternating Series Whose Terms Decrease in Absolute Value


670 CHAPTER II INFINITE SERIE.S

where, again, each quantity enclosed in parentheses is nonnegative. The next


odd integer after w is m + 2, and we have

Hence, ^i
> 53 s sj > 57 > ^g > • • •

(iii) If n is even, then m + 1 is odd and s„+i = 5„ + a„+i. Therefore,

s„+] — s„ = a„+i

(iv) Again, if n is even, then by (iii), .?„ + , -,v„ = a„,|SO; hence,


s„ ^ ^„+|. By (i), s s„, and by (ii), s„+i s i,, so that

In Theorem 1 , if {a,,} is a strictly decreasing sequence, so that a^ > a2> ai> • • •,

then all inequalities appearing in the proof and in the conclusions can be made strict.

The following theorem, discovered by Leibniz, provides a useful test for conver-
gence of alternating series.

THEOREM 2 Leibniz's Alternating-Series Test


SECTION U.5 SERIES WHOSE TERMS CHANGE SIGN 671

EXAMPLE I Test the alternating harmonic series z^ (— l)*"^' — *•


for convergence.
k=i

SOLUTION The sequence ]


— f
is decreasing, and lim —= 0; hence the alter-

nating harmonic senes

1
111 + + ••• + (-1)"^'
^,1 —+ • • •

2 3 4 n

converges by Leibniz's aUemating-series test.

In Leibniz's theorem (Theorem 2). notice that S — s„ is the error involved in


estimating the sum S of the alternating series by the /!th partial sum s„. If h is even,
then
< (- 1)"(S - s„) = S - s„ so s„ < S

If n is odd, then

0<(-l)"{5 - j„) = -{S - 5„) so 5<5„


Therefore we have the following important result:

Suppose that the hypotheses of Leibniz's theorem (Theorem 2) are true, so that
the alternating series converges to the sum S. If /i is an even positive integer, then
s„ underestimates S: if n is an odd positive integer, then 5„ overestimates S. In
any case.
\S - s„\ < a„+i

that is. the absolute value of the error of estimation does not exceed the absolute
value of the first neglected term * .

For example, if we wish to estimate the sum

S= I
111 + + ••• + (-1)"+'
,1 _ + ...
2 3 4 n
of the alternating harmonic series by a partial sum s,„ then the error cannot exceed
the absolute value of the first neglected term a„+i. For instance,

5==l-* + 5-i + A-i + |-i;)-i = 0.7456 . . .

with an error that does not exceed lo. Moreover, since we have an odd number of
terms in this estimate, we have overestimated S. so that S is less than 0.7456 ....
Actually, it can be shownt that the sum 5 of the alternating harmonic series is equal
to the natural logarithm of 2:

5 = In 2 = 0.6931 . . .

//) Examples 2 and 3, (a) show that the given series is convergent, (b) find the
partial sum
of its first four terms, and (c) find a bound on the absolute value of
S4
the error involved in estimating its sum by s^.

*lf {a,,} is strictly decreasing, so that «] > aj > aj • • , then the conclusion \S - s„\ s a„+t
can be sharpened to < |5 — s„\ < a„+ 1.

tSee Problems 41 to 44 in Problem Set 11.7.


672 CHAPTER II INFINITE SERIES

EXAMPLE 2 X (-!)' '

,v + 3
(a) Let J be the tunction defined by J(x) = . Then
.v(.v + 2)

+ +
f'(x) =
.r
;
6.V
—6 < for .V > 1

Hence, the function /is decreasing on [1, ^). It follows that the sequence
+ 3
{fin)} — that is, the sequence
[
]
I-
"
n{n + 2)
1

>
f
— is decreasing. Since

« + 3 /) + 3
lim = (why?) and > 0. the given alternating series
"^ + ^ n{n + 2) n(n + 2)
converges, by Theorem 2.

49
= 4 6
5 7
(b) 54 + =
3 8 15 24 60

(c) The absolute value of the error of the estimate

S4 = = Z (-1)^ '

60 ;r, k(k + 2)

5 + 3 = 8
does not exceed the fifth term, . Here s, involves an even
5(5 + 2) 35

number of terms, so that


49
underestimates 2j (~1)
V ; + .
' + 3
60 71, k(k + 2)

(-ir
EXAMPLE 3 2
k=\ ^

(a) This series begins with a negative term, — 1/1 !, whereas Leibniz's theorem
as stated above concerns alternating series which begin with a positive term.
However, we can write

i^=-i^ *=i

V <'i»'^' 111.
-)
and apply Leibniz's theorem to the series Zj Here | is a

decreasing sequence of nonnegative terms, and lim —- = 0, so that

" (-1)*^'
Zj converges; hence Zj
V (-1)* ,

also converges.
k\ ,._. k\
SECTION 11.5 SERIES WHOSE TERMS CHANGE SIGN 673

(b) and (c) Here. 1


1115

2
= — I

6 24 8
underestimates
.-,
A
,Z^,
(-1)*
it!

with an error no more than and therefore, overestimates


5! 120 8

(-ir'
k'
= 1 it'

with an error whose absolute value does not exceed T2o.

Absolute and Conditional Convergence


Consider the alternating geometric series

' 2 + 4 8+16 32 +
with ratio r = — 2- Not only does this series converge, but so does the correspond-
ing series
III
I'l + —
4- I

I
1|
2| +
-I- |i|
|4| + _
4- I

I
i| -U l-l-l 4-
8| + |l6| + _ -i-l
32| +
I

I
-t- . . .

of absolute values: that is. the geometric series 1 + 2 + 4 + 8+i^+32 + *'*is


also convergent. Such a series is called absolutely convergent.
On the other hand, consider the altematine harmonic series

which converges by Leibniz's theorem. The corresponding series of absolute values

is the harmonic series

---
i+i + l + i + i + i +
which diverges. Thus, the convergence of the alternating harmonic series actually
depends on the fact that its terms change sign. Such a series is called conditionally
convergent. More generally, we make the following definition.

DEFINITION 1 Absolute and Conditional Convergence

(i) If the series 2lk=\ |a*l converges, we say that the series X*=i ^k is

absolutely convergent.

(ii) If the series 2]*=i ^k is convergent but the series 21^=1 !«*! is diver-
gent, we say that the series X*=i ^k is conditionally convergent.

In Examples 4 to 6, determine whether the given series is divergent, conditionally


convergent, or absolutely convergent.

EXAMPLE 4

Since the series

(-1)*

f . it^ +
674 CHAPTER 11 INFINITE SERIES

converges by comparison with the convergent p series Zj ~^r- •' follows that

V
2j —-,
(-1)*
is absolutely convergent.

k + 1

E 5 S (-1)'
Try
,
,
n + 1

SOLUTION Since lim(— 1)" does not exist (why?), the given series
"-»+« n + 2
is divergent (Section 11.3, Theorem 2).

EXAMPLE 6 Zj
——;—

SOLUTION The given series converges, by Leibniz's theorem. However,

V
2_,
I

\-
(-1)* I

= V
2j
1
diverges since
'1
s— holds for A- s 2 and 2^
V —
1

, I In A' I , , In A: In A k k~2, k
,

k=2
.
k=2

(-1)
diverges. Hence, 2j is conditionally convergent.
In A

THEOREM 3 Absolute Convergence Implies Convergence

If a series 2^it=i ^k '^ absolutely convergent, then it is convergent.

PROOF Assume that Zjk=\ \<^'k\


is convergent. The inequalities — |a^| s a^ < |a^| can be
rewritten as

< a^. + |a,| < 2\ak\

Now, 2]r=i 2|ai| converges since 2^=1 V'k\ converges [part (ii) of Theorem 3
in Section 11.3]; hence, by the comparison test, Zjt=\ («* + |a*|) converges.
Therefore

2 [(a* + k|) - |«*|] = S «*

converges [part (i) of Theorem 3 in Section 11.3].

^ —sin k ^
EXAMPLE 7 Determine whether the series Zu converges or diverges.

SOLUTION Although the given series contains both positive and negative terms,
it is not an alternating series (why?). However, we have

sin k sin k\

A-' + 4 I" k^ + 4 ~ k^

^ sin A
for every positive integer A, so that 2^ —'"
^
is dominated by the convergent
k=] '
'^ ^ '
SECTION 11.5 SERIES WHOSE TERMS CHANGE SIGN 675

p series Z, -j^- Therefore, the given series is absolutely convergent and hence

convergent by Theorem 3.

The following theorem gives one of the most practical tests for absolute conver-
gence.

THEOREM 4 The Ratio Test

Let 2y a* be a given series of nonzero terms

a„-\
(i) If lim

(ii) If lim

(iii) If lim
676 CHAPTKR II INFINITE SVMIKS

converges by the direct comparison test. Therefore, by Theorem 5 of Sec-


tion 1 1.3, ^r=i 1^*1 converges; that is, ^^=1 «* 's absolutely convergent.

a„+i
(ii) Assume that lim

I I

=L> I. Choose and fix a number r


n + » I a„ I

with \ <r<L and put e = L — r. Thus, there exists a positive integer N


such that L - e < ^^^^ < L + e holds for n > N. Hence, 1 < r =
«„ I

"n+li II < ^
L - < = I I

€ -, SO that |a„| |«„+i| holds for n N. Therefore,

|«;v| < |«;v+i| < l«;v+2| < \aN+3.\ <• •


SO that < |a/v| < \a„\ holds for all n ^ N. This shows that the con-
dition lim a„ = cannot hold; hence, 2l*=i ^k '^ divergent (Theorem 2 in

Section 11.3).
"n+l
Similarly, if lim
I I

= 4-oc. then there exists a positive integer A^

"^
such that 1 < holds for all n > N. and we can complete the argument

just as above and see that 2^^=! «* diverges.

(iii) To see that the test reallv is inconclusive if lim = 1, consider


"-^ + « I
a„ I

(a) ^ Tj and (b) 2^ — . Series (a) is convergent, but


^ ^
k=\ k=]

l/(« + 1)- n-
lim -^ = lim
l/n- n^+« n- -I- 2« + 1

Series (b) is divergent, but


l/in +1) n
lim = lim = 1
.- + » l/n "- + - /) + 1

In Examples 8 to 10. use the ratio test to determine whether each series converges
ahsohitely or diverges.

•^ 2*
EXAMPLE 8
,
T(k+ 1)

SOLUTION Here
2"^'
a„ = — 2"
and a„+x
Tin +1) T*\n + 2)

Therefore,

,.
lim
|a„+,|
= ,.
lim
/
I
—— 2"^'
;
7"(/t+l)
2"
)=
^

«- + «
lim
2(/j+l)
l"*\n + 2) 7(w + 2)

2|1+(1A0] 2^j
"-^^- 7|1 +(2//7)] 7

so the series converges absolutely by the ratio test.


SECTION 11.5 SERIES WHOSE TERMS CHANGE SIGN 677

(-l)*+'5*
EXAMPLE 9 2]
k=Q

SOLUTION Here

(-1)"^'5" {-1)"^=5"^'
^
and a„+i
(n + 1)!

Therefore,

lim = lim I lim 0< 1


n^ + « I a„ I
n^+« V(H + 1)! 5" n + 1

SO the series converges absolutely.

1-3-5--- (2A:- 1)
EXAMPLE 10 X
A-=l

SOLUTION Here

1 -S-S • • •
(2n - 1) 1-3-5 •• •
(2» - l)(2>i + 1)
and a„ + i

(n+ 1)!

Therefore,

(In + !)«! In + 1
lim = lim = lim — = 2 > 1

(n+ 1)! n + I

so the given series diverges.

Another useful test for absolute convergence is given by the following theorem.

THEOREM 5 The Root Test

Let
678 CHAPTER II INKINITE SERIF-S

EXAMPLE II Use the root test to decide whether the series

,r, [In (k + 1)]*

converges or diverges.

SOLUTION Here
(-1)"

[In (/!+ 1)]"

Therefore.

lim ,"||a„| = lim "/I = lim = 0<1


,^+„\' "-+' V I
|ln (« + l)]"l "- + -ln(/;+l)
so the given series converges absolutely by the root test and thus converges.

Problem Set 11.5

In Problems 1 to 14. determine whether the given alternating series In Problems 15 to 26. apply the ratio test to determine whether each
converges or diverges. Use Leibniz's alternating-series test to estab- series converges absolutely or diverges,
lish convergence whenever it applies.

*'
15 S Ar^(f)* 16 S
k=l t=i
i2^^!^ 2S
(-D'^'S*
C^l ^
cos kn *=i ''

(-1)**'*

*=i
(S) 2j — Hint: First consider 2/ —
^,=, VFT7
/ t
L
^.^, VFTi
, ,
i

®|,^

^*=i * + 7 J,, 4A-+ 1 25 Z


SECTION 11.5 SERIES WHOSE TERMS CHANGE SIGN 679

33 £ (V^- D* 34 S [2A- + (1A-)1'

In Problems 35 to 54, determine whether each series is divergent,


conditionally convergent, or absolutely convergent. Use whatever
tests or theorems seem most appropriate to justify your answer.

35 2 (-ly
k'.
56;z A(tf
k = \

k=\ In (k + 1) /t^ + 10

(-1)"+' in n

k\
(15)S (-1)*
(2A-+I)!

IK
41 1 J- +
,,^1

4 6
"'
Y 2 • • • • • (2k)

^t, l-4-7---(3;--2)

53 S (-1)^^
680 CHAPTER II INHNITK SERIES

11.6 Power Series

An infinite series of the form

-r.
SECTION 11.6 POWER SERIES 681

/„

S ——
KY"^ 1 1

EXAMPLE 2 = 1 + (.X - 5)- + —(.X - 5)-* + —(.V - 5)" + • •


k=o I"- 2 6

SOLUTION The series converges for .v = 5. For x 9^ 5. we use the ratio test with

=
(.v-5)-" ^ =
(.v-5)-'"+'> U-5)-""-
a„ and a„+i
(n + 1)! in + 1)!

Here.

Hm = lim
I {n+ 1)! "
(.x-5)-"l

(.V - 5)=
«-+-
0< 1

n +
for all values of x: hence, the series converges for all values of .v.

EXAMPLE 3 S (k\)(x + If = 1 + (^ + 2) + 2(a: + 2)" + 6(x + 2f + ••


k=0

SOLUTION Here we have a„ = (n!)(.r + 2)" and a^+i = [(n + l)!](jr + 2)""^'.

For X ¥" -2, the ratio test gives

lim M^ = lim \- — '


= I™ (« + l).v + 2 = +^

Hence, the series diverges. Of course, the series converges for a = -2, and so it

converges only when .v = —2.

The set / of all numbers x for which a power series X*=o Ck(x - af converges is
called its interval of convergence. In Example 1. / = ( — 3, 3), in Example 2,

/ = (-0C, 00), and in Example 3. / is the "interval" containing the single num-
ber — 2.
For any power series Xr=i Q(x - af, the interval of convergence / always has
one of the following forms:

Case 1 I is a bounded inten-al with center a and with endpoints a — R and


a + R, where R is a positive real number.

Case 2 / = (-oc. x).

Case 3 / consists of the single number a.

In case 1 , we call the number R the radius of convergence of the power series. In

case 2. it is convenient to say that the radius of convergence of the power series is

infinite and to write R = +^. Naturally, in case 3. we say that the power series has

radius of convergence zero, and we write R = 0. Examples 1 through 3 above


illustrate these three possibilities.

In case 1 . the endpoints a - R and a + R of the inter\'al of convergence I may


or may not belong to I. In Example neither endpoint belongs to /, so that / is
1 .

an open interval. In general, anything can happen — the series may diverge,

converge conditionally, or converge absolutely at an endpoint of /. Thus, in


682 CHAPTER II INHNITE SERII':S

Figure I case 1 , the interval of convergence can be any one of the


four /=[«-/?, a + R], / = [a - /?.
sets a + R), / =
(a- R,a + /?], or/ = (a - /?, a + /?) (Figure I). The power
series always converges absolutely on the open interval
(a - R. a + R).

series converges absolutely


for [.\ -a\<R The following theorem provides an efficient means for finding the radius of
convergence of a power series.

THEOREM I Radius of Convergence of a Power Series


SECTION 11.5 POWER SERIES 683

kth power of x - a. In such a case, the radius of convergence can often be


found by applying the original ratio test (Theorem 4 in Section 11.5) directly
to
the terms of the series, as in Example 2 on page 681.

5 Theorem 1 says nothing, one way or the other, about whether the power
series converges at the endpoints of its interval of convergence. This has to be
checked by substituting .v = a - /? and .v = a + /? in the
power series and
using the standard tests for convergence of a series.

6 Theorem 1 is still valid for a power series such as 2*=^ qU - «)*. where
A/ is a positive integer and the summation starts at ^ = rather than at A- = M
(Why?)

In Examples 4 to 8. find the center a, the radius of convergence R, and


the inten'al
of convergence I of the given power series. Be sure to check the endpoints
of I for
divergence, absolute convergence, or conditional convergence.

EXAMP LE 4 y. —X*
^r k

SOLUTION Here the center is a = 0. and q = \/k. In Theorem 1 . we put c„ =


\/n and £„+, = l/(/? + 1), so that

—+1)1
,.
f,.+ i l/(« n
hm I I

= hm ,. I

= hm = 1 =1
"-*'- I
C„ I
"--= I
1//J I
n-»*oc „ + 1

Hence. R= \/L = \. Therefore, the series converges absolutely for values of .v in


the open interval (a - R. a + R) = (Q - 1.0+ I) = (-1, 1), and it diverges for
values of .v outside the closed interval [- 1 . 1]. Substituting a: = 1 in the series, we
obtain the harmonic series « ,

k=\

which diverges. For.v = -1, the series becomes the alternating harmonic series

which converges by Leibniz's test. Hence, we have divergence at the endpoint 1

and conditional convergence at the endpoint - 1 The interval of convergence .


is
/=[-!. 1).

EXAMPLE 5
V
\ U" +
;
3)*

SOLUTION The power series is centered at a= -3. We have r„ = 1/3" and


Oi+i = 1/3"*', so that

1/3""'
"^+=
hm
,. \c„^x\
r„
= hm
«-+«
..
I

——-—
1/3"
I

= hm ,.

n^^--.
—=—=L
I 1

I
I
I I
3 3

Hence. /? = 1/L = 3 by Theorem 1. Therefore, the series converges absolutely on


the open interval (a - /?. a +/?) = -3 -
( 3. -3 + 3) = (-6. 0). When .v
= -6,
the series becomes
V ^r-=i-
Z
(-3)'
1 + 1-1 + ---
684 CHAKIEK II INFINITE SERIE.S

which diverges because the general term does not approach zero. When .r = 0, the
series becomes

which also diverges. Therefore, / = (—6, 0).

(-If
EXAMPLE 6 X (x - 17)*

SOLUTION The power series is centered at « = 17. We have

(-1)" (-1)"^'
c„ = ^
and c„+i =
i! (/! + 1)!

so that

0,+ i
(-l)""" "! 1

lim
I I

= lim
I I

= lim = = Z,
«-* + » I f„ I "^ + -l(n+l)! (-1)"! «- + -«+ 1

Hence, R = +^ by part (ii) of Theorem 1. Therefore, / = (-^, °=).

EXAMPLE 7 2 ^*i^*

SOLUTION The power series is centered at a = 0. We have c„ = n" and f„+i =


in + !)"", so that

'
lim
,,^+oc
M^-^ = lim
„-» + «
\-
„"
= I'm
,,^ + oc Lv /
(" + D
I
c^ I I I
/; J

= lim (l+— lim (« + 1) = e Hm («+!)=+«=

Thus, R = by part (iii) of Theorem 1 , and so / consists of the single number 0.

EXAMPLE 8
^ 3*(.v - 4)-*
2j n

SOLUTION The power series is centered at a = 4. Here we cannot use Theo-


rem 1 since 3*/^- is not the coefficient of the kth power of .v — 4. Thus, we resort to
the original ratio test (Theorem 4 in Section 1 1 .5). The nth term (not coefficient!) of

the series is

= 3"{x-4)~"
3"*'(jc-4)2"'-"' 3"*'(A--4)-""'-
a„ ; so a„+ 1
=
(n + 1)^ (n + 1)-

Therefore,

lim = lim
(« + 1
)-
3"(.v - 4)-"

= lim 3 Lv - 4|- = 3|.v - 4|-


«- + " V n + 1
'
' ' ' '
SECTION 11.6 POWER SERIES 685

It follows that the series converges absolutely when 3|.v — 4|- < 1, that is, when
|.v - 4| < I/V3. It diverges when 3|.v - 4|- > 1, that is, when |.v - 4| > l/Vs.
Therefore, R = l/V'3. When

.V = 4 ^
1
the series becomes 2j ~~r
V 1

V3 ,., k-

which converges absolutely. (Why?) Similariy. when

X = 4 -\
1

7^ the series becomes 2j


V 1
~^
V3 ,., k'-

which converges absolutely. It follows that the series converges absolutely on its

convergence
entire interval of

/= 4-^,4 + -^
L V3 \ 3 -I

Problem Set 11.6

In Problems I to 40. find the center a. the radius of convergence R. " (^^ +1)*
and the inter\'al of convergence / of the given power series. Be sure
to check the endpoints of/ for divergence, absolute convergence, or
conditional convergence.

li
k=0
686 CHAPTER II INFIMTE SCRIE-S

\-3-5---{2k- 1) ,^.+ 44 Let 2t=o Ckix - a)* be a given power series.


39 S (-1)*
2-4-6 (2*)
1

(a) If lim N^^l = +=«, prove that the radius of convergence

40 {.r - 8) + (v - 8)- + 2!(.v - 8)' + 3!(.( - S)-* + • • •


of the power series is zero.

(b) If hm VV^I = 0, prove that the power series has an infi-

41 If R is the radius of convergence of X*=o Ck(x - a)* and


integer, find the radius of convergence of nite radius of convergence.
p is a positive

Sr=o c,{x - ar*'. (c) If lim V\c^\ = L 7^ 0, prove that the radius of conver-

42 If R is the radius of convergence of ^^^=0 qU - ")*. gence of the power series is given by R = \/L.
<R< +0C, and p is a positive integer, show that the radius of

convergence of ^k=u f/i(v - a)''* is V^ 45 Suppose that b h a constant greater than 1 . Find the radius of
convergence and the interval of convergence of the power series
43 Let ( be a constant real number.

(a) Show that the series


I -(-fe*

- -
V c(t- \){c 2) • {c n) „
46 Complete the proof of Theorem 1 by considering parts (ii) and
(iii).

converges for |x| < 1.

show < then


47 If a > b a 0, find the radius of convergence of the power series
(b) Use the resuh of part (a) to that if |.v| 1 .

cU- - l)(r - 2) • (c- - n)


lim -x" =
a* + b"

11.7 Differentiation and Integration of Power Series


In this section we study functions of the form

f(x) = E c,U - af

where 2r=o Ck(x — a)* is a given power series. Here it is understood that the

domain of f is the interval of convergence of the power series.


Since a finite sum can be differentiated term by term and since

fM = Co + f,(.v -a) + c.ix - af + cU - af + •


you might guess that the derivative DJ'ix) can be obtained by differentiating term
by term; that is,

DJM = D,Cq + D,c,(.v - a) + D.cijx - af + D,Ci(x - af + • •

= + c, + 2c-2(.v - a) + 3c3(.v - af +

Likewise, you might guess that the integral j fix) dx can be obtained by integrating
term by term; that is,

f(x) dx = (•„ d\ + c\(x - a) dx + C2(.v - af dx + \ c^ix - af dx + •

J J J J

= [c-oU -a) + -y{.v - af + -^(.r - af + -^(.v - af + •


-^
+ C

It turns out that such "calculations'" are quite legitimate, provided that |.v — a| < ^,
where R is the radius of convergence of the power series.
SECTION 11.7 DIFFERENTIATION AND INTEGRATION OF POWER SERIES 687

Notice that term-by-term differentiation or integration of a power series

k=

produces a new power series,

E DAck(x - af] = 2 kc,(x - a)'-'

S Q.(x - a)* dx=^ 1~T^^ ~ k+\


"^'

k=0 k+l
J
k=0

respectively. (In the second equation, we have suppressed the constants of integra-
tion.) Also, the following holds, although we omit the somewhat technical proof.

The three power series

2 a(-v - af 2 Dv[c^(.v - «)*] and 2 1 c

all have the same radius of convereence.

Term-by-term differentiation or integration of a power series is justified by the


following properties, whose proofs we also omit. Suppose that

fix) = S Ck(x - a)"


k=

where the power series has radius of convergence R. Then

*Property I For |.v - a\ < R,

/'(A) = oj X a(.v - a A = X Ar,(.v - «)*"'


k= k= I

Property II For |.v - a\ < R,

jf(.x) dx =
/ [
^(.-
i
=
Q(.v - af] dx = i— ^U- «)*-• -f C

Property III For \b - a\<R,

J
fix) J.V =
J [
2 Q(.v - a)'J ^v = S J
c,(.v - at dx
k=

,
_,, k -H 1

*Because a difterentiable function is continuous, it follows from Property I that/is continu-


ous on the open interval (.a - R, a + R).
688 CHAPIKR II INFINITE SERIES

EXAMPLE 1 Find D,(l + 2r + 3jc- + 4^' + • • •).

SOLUTION By Theorem 1 in Section 11.6, the radius of convergence of the


power series 2r=o (*' + D-v* is R = I; hence, by Property I,

D,( \ + 2x + 3.x- + 4x^ + ) = + 2 + 6.x + lit" +

that is.

oj S (k + l).v*J
= S k(k + l).r*-' for |.x| < 1

fM = S c,(x - at

for .V in the interval /, we say that the power series

X c^ix - af

represents the function /on the interval /. For instance, by Theorem 1 in Section

11.2, the geometric series 1 + .v + .v" + .v^ + • • • converges for |.r| < 1, and its

sum is 1/(1 - .X): that is, the function /(.r) = 1/(1 - .x) is represented by the power
series 1 + .v + .v" + .x^ + on the interval (-1, • • •
1). By starting with a known
power series representation such as

1 -.V
= 1 + .V + .V- + v' + • • • = 2 -V* for - 1 < v < 1

and using Property I or II to differentiate or integrate the power series term by term,
it is often possible to obtain interesting new power series representations. In this
connection, substitution into a known power series representation is also useful.
The following examples illustrate the techniques involved.

In Examples 2 to 7, starting with the power series representation given above for
1/(1— each of the indicated power series representations by using
.v), verify differ-

entiation, integration, or substitution.

EXAMPLE 2
(1-A)-
= 1 + 2v + 3.V- + 4.v' + • • • = S A-V*"' for|.v| < 1

( 1 -
— = D,(—^) =
.V)
Dk( i -v*) = i /t.v*-' for|.r| < 1

EXAMPLE 3 = l-/ + /2-r^ + ---=S(-l )*'* for \t\ < 1

1 + /
SECTION 11.7 DIFFERENTIATION AND INTEGRATION OF POWER SERIES 689

SOLUTION Substituting —t for x in the equation

we obtain

1 + /
= S (-tf = S (-Ift'' = I - t + r - t^ + for \-t\ < 1

k=0 k=0
that is, for |/| < 1.

EXAMPLE 4 In (1 + x) = X ^ + ~^ "j~ "*" *


"

= S (-1)*—-— for|A|< 1

SOLUTION Using the result of Example 3 and Property III, we have

In ( 1 + .V) =
+
= (\ - t + t^ - 1^ + )dt
Jo 1 r Jo

= dt- \
tdt+ r dt- \
r^ dt +
I
Jq Jq -'o
-^0

_xl x^x^
"'^ "^
2 3 4

^k+\
= E (-l)*-
^ '
for \x\ < 1

k=o '^

EXAMPLE 5 r= 1 - A- + X-* - -x" + • • • = S (- l)*jf-* for |jr| < 1

SOLUTION Substituting -A" for x in the equation

we obtain

T= S (-x-f = S (- 1)V* = 1
- A- + A-* - A* + • • • for |a| < 1

-^
1 A=0 A:=0

(Note that we get convergence of the series when |


- a-| < 1; however, |
- a^| =
|a-| = |a|-, and |a|- < 1 holds exactly when |a| < 1.)

EXAMPLE 6
1
= —+
1 A
1
X-
H
X'
!-•••= Z, V ^,..
-T
for
II,
A < 3
3 -A 3 9 27 81 k^o ^
,
690 CUVriKR II INKINITE SERIES

3-x 3 1 - (x/3) 3 m'-'^A4


^r; ^

for —
3
< 1

k = ''
I I

that is, for l.vl < 3.

EXAMPLE 7 -^
2
= —2 + —x + 8 26
.V-
,
+
80
x^
,
+
jc2-4a: + 3 3 9 27 81

3*^' - 1

2 for \x\ < 1

SOLUTION By partial fractions,

2 2 -1 1
+
x~ - 4.x: + 3 (.V - 1 )(.v - 3) .v - 1 .v - 3 1 - .v 3 -x
Therefore, using the result of Example 6, we have

x'--4x + 3 l-.v 3-.V ,t; ,


3*"'

= 2 (^* -
^) = 1 (^^^:^y for kl < 1

(Note that if |.v| < 1 , then the condition |.v| < 3 in Example 6 is automatically satis-
fied.)

Using the result of Example 5 and Property III, we have

tan" ' jc
= = (\ - r + f* - t" + • •) dr
4 1 + r Jo

= dt - \
r dt + I f* dt - \ t^ dt + •••

'^
3 5 7

= l(-lf- for l.vl < 1


2k + 1

The power series in the representation

tan '
X = X —+— 3 5
1 (-1)*
2k + 1
for |.v < I

is called Gregory's series in honor of its discoverer James Gregory (1638-1675), a


Scottish mathematician and astronomer who was one of the first to make a clear
James Gregory distinction between convergent and divergent series.
SECTION 11.7 DIFFERENTIATION AND INTEGRATION OF POWER SERIES 691

It can be shown that for .v = I, Gregory's series still converges to

Hence, 77= 4(1 •)

Although this equation shows that there is an interesting connection between tt and
the reciprocals of the odd positive integers, it is of little practical importance for

finding the (approximate) numerical value of -n because the series converges too
slowly.*

*A more efficient method for estimating the numerical value of 77 is indicated in Problem 40.

Problem Set 11.7

In Problems 1 to 4, differentiate or integrate the power series as

indicated, and find the radius of convergence of the resulting power


series.

1 D4. + -+-+-+ ---)

'V 3! 5! 7! 9!

3 (1 + 2v + 3a:- + 4.V-' + • • •) dx

(I x'
+ x' x' x"
4 \ (x + dx
J V 3! 5! 7! 9! 11!

In Problems 5 to 20, starting with the known power series represen-

tation for 1/(1 - a), |.v| < 1, obtain a power series representation

for each expression by using differentiation, integration, substitu-


tion, or other suitable procedures. In each case, find the radius of
convergence of the resulting power series.

1 - x'

1
692 L HAKl EK 1 1 INHNITE SERIES

l/( 1
- .v) - s„{x). Show that if .v 5^ I , then

Rnix) =
I
-X
33 fix) = 2 2'/'^+ Ip
(Hint: 21^=0 x'' is a geometric series with initial term I and

34/,.v,= ii^ ratio X.

42 Using the result 1/(1 - .v) = s„(x) + R„(x) from Problem 41 and
integrating both sides of this equation from x = —1 to x = 0,
conclude that
In Problems 35 to 38, let the function / be defined by the given
(-1)"
power series. Write a power series for fit) dt. and find its radius ln2=l--^ + -L--L + dx
2 3 4 I, 1 -.V
of convergence.
43 Criticize the following argument: By Example 4.
(-l)V
35 f(t) = S (2*)!
36 /(;) = S -^ \n(\+x)=x-^
2

+ ^-^ +
3 4
--- for |.v| < 1

When X = 1, the series on the right is the alternating harmonic


37 fU) = 2 ak+ 38 /(f) = E k' series, which converges by Leibniz's test. Therefore, In 2 is the
"o 1) 'i

sum of the alternating harmonic series.

39 Given /Cr) = ^".^o [(- l)*.v-V(2A-)!]. find 44 Prove that in 2 really is the sum of the alternating harmonic

(a) /(O) (b) /'(O) (c) /"(O) (d) /'"(O) series (see Problem 43) by using the result of Problem 42 and
showing that
140 Use the identity 77/4 = tan"' 7 + 2 tan"' 3 and Gregory's se-
ro ^„+,
ries to approximate the value of tt to four decimal places.
lim dx = Q
„_ + ,. J 1
_ ^
41 For each fixed value of x, let s„(x) = ^4=0 •** be the nth par-
I

tial sum of the infinite series 2^*=o **> ^nd let R„(x) = [Hint: For - 1 < a: < 1, we have £ 1/(1 - x) ^ 1.]

11.8 Taylor and Maclaurin Series


We have seen in Section 1 1 .7 that a power series ^r=o qU ~ a)* defines a func-
tion/by /(.v) = Xr=o '"Arfv
— a)*. Thus, starting with a power series, we obtained a
function/. In this section, we study the reverse procedure starting with a function

f, we try to find a power series that represents it; that is, we try to expand/ into a
power series. Although a power series expansion cannot always be obtained, most
of the familiar functions in calculus can be represented as the sum of a convergent
power series.
Suppose that / is a function that can be expanded into a power series with a
positive radius of convergence R, so that

fix) = S Cktx - a)* for a - /? < .V < a + /?

Then, by Property I on page 687, / is differentiable on the open interval

(a — R, a + R) and we have

fix) = S kCk(x - a)*" ' for a- R<x<a + R


SECTION 11.8 TAYLOR AND \UCLAURIN SERIES 693

Thus, not only is/differentiable. but also the derivative/' of /can itself be ex-
panded into a power series. Hence, we can apply Property I once again to obtain

f"(x) = E k(k - 1 )Q(.v - af~- for a - R<.\<a + R


k=2

Continuing in this way. we have

f"\x) = ^k(k - l){k - 2)c,(.x - af-^


k=3

f^\x) = E k{k - l)(k - 2)(k - 3)c,(x - «/-•*

/'-"(A-) = E k(k - Dik - 2)(k - 2){k - 4)Q(.v - at~^


k=5

and so on, for a —R< x < a + R. The pattern here is obvious — evidently

/""U) = E k{k - 1)(A- - 2) • • •


{k - II + l)a(.v - af-"
k=n
for a - R<x< a + R
(See Problem 51.)
Putting .V = a in the formula just obtained, we find that all terms uf the series
reduce to zero except for the first term (for which k = n). since all the terms after
the first contain a factor (x — a). Therefore,

/""(a) = n(n - l)(n - 2) • • •


3 •
2 •
1 •
c„ = (n\)c„

It follows that

c =^
694 CHAFl ER 1 1 INFINITE SERIES

The Taylor series for/ at a = is called the Maclaurin series* for/. Note that

there is no implication intended in Definition 2 that the Taylor series for/actually


converges to / Also note the distinction between the /ith Taylor polynomial for
/at a.

I, _ r» ^

and the Taylor series tor /at a.

r\a) - af
(X

The former is a polynomial of degree at most n. while the latter is an infinite series.
In fact, P„{-x) is the nth partial sunt of the Taylor series.

EX,\MPLE I Find the Taylor series for/(.v) = sin x at a = tt/4.


Colin Maclaurin
SOLUTION We arrange our work as follows:

/ 7r \ 77 V2
fix) = sin X
2

= =
V2
fix) cos X
^'(f) 2

V2
fix) = —sin .V
nji--^'" i 2

f"'ix)
= -cos X /'"I = —cos — _ V2_
4/ 4 2

—= V2
f^Kx) = sin X /'*> sm
4

and so forth. Therefore, the coefficients of the Taylor series are

_
/'*'(77-/4) _ ±\/2/2 \/2
^* ~ /(.-! ~ ~ " 2 A-!
A-! •

where the plus and minus signs alternate //( pairs. The Taylor series

i q{., for sin .v at a


il,=0

is accordingly given by

^ 2
^ ^ix
2
- ^)
A ^ '
- ^x 4
2-2! \
-
A '
- ^^(.v -
2-3!^ ^f
A
V2 — I +
2-4!

This special case of a Taylor series is named in honor of the Scottish mathematician Colin
Maclaurin (1698-1746), who developed and extended Newton's work in calculus.
SECTION 11.8 TAYLOR AND MACLAURIN SERIES 695

EXAMPLE 2 Find the Maclaurin series for/(jc) = e'.

SOLUTION The Maclaurin series for ^' is just the Taylor series for e^ at a = 0.
The work is arranged as follows:

yiv) = e' /(O) = e" = 1

/'(.v) = e" /'(O) = e° = 1

fix) = e' /"(O) = e" = 1

f'\x) = i^' /"'(O) = e" = 1

and so forth. Evidently, /'*'(0) = e° = I holds for all ^ = 0, 1, 2, 3, . There-


fore, the coefficients of the Maclaurin series are given by

_ /'^'(O) _ 1

"^^ ~ ;•! ~ k\

Hence, the Maclaurin series 2^=0 ^kix ~ Of = ^k=o a^'^ for e* is

l+.v + -rr +
X
2
X
14
^- — +
X
+••• + — + •••=
X
n
'^
<r-< !
k

Z,
2! 3! 4! k\

As we have seen, if a function can be expanded into a power series, then the
function must be infinitely differentiable and the power series must be its Taylor
series. However, even if a function is infinitely differentiable, there is no automatic
guarantee that it can be expanded into a power series at all! In other words, although
an infinitely dijferentiable function has a Taylor series, this Taylor series need not
converge to the function. (See Problem 50 for an example.) The following theorem,
which is a consequence of Taylor's formula (Theorem 2, page 622), gives a condi-
tion under which the Taylor series for a function actually converges to the function.

THEOREM 1 Expansion of a Function into Its Taylor Series

Let the function/be infinitely differentiable on some open interval containing the
number a. Suppose that there exists a positive number r and a positive constant
M such that

|/""'(.v)| < M
holds for all values of x in the interval (a - r. a + r) and all positive integers n.
Then /can be expanded into a Taylor series; that is,

holds for all values of .v in the interval {a — r, a + r).

Let

Pnix) = Z
i-r,
^T^(-v - af
k-
696 CHAPTER 11 INFINITE SERIES

be the nlh Taylor polynomial for /at a. and let R„{.x) = fix) — P„(.x) be the corre-
sponding Taylor remainder. Notice that P„{x) is just the nth partial sum of the Taylor
series

t=o "•

Hence, this Taylor series converges to/(.v) if and only if/(.v) = lim P„(x). The

latter condition is equivalent to lim [fix) - P„ix)] = 0; that is, it is equivalent to

lim R„ix) = 0-

Now assume that |/""(.v)| < M holds for a — r < x < a + r and for all positive
integers n. By Taylor's formula with Lagrange remainder

/'"+"(<:)
R„ix) = '
—ix - a)"^^
in+ \)\'

holds for some c between a and .v. Hence, for a in (o - r, a + r), we have

\Rnix)\ = \f"'^'Hc)\-
^

< M-
'
'
'-^ '

(n+ 1)! (n+ 1)!

For X m ia - r, a + r), the infinite series

k=
(k+ 1)!

converges by the ratio test, and therefore its general term approaches zero; that is,

lim =

Hence, since A/ is a constant, the inequality

|.v-«|"^'
\R„(x)\ < M-
'
" («+ I)!

shows that lim R„ix) = holds for all values of x in the interval (a — r, a + r).

Therefore, the Taylor series converges to/(.v), and the proof is complete.

The following examples not only illustrate the use of Theorem but also provide 1 ,

power series expansions that are so important that they should be memorized for
future use.

In Examples 3 lo 5, justify the following power series expansions.

EXAMPLE 3 e' = I + X + —+—+—+


.V-

2!
x'

3!
.V*

4!
• • •
tor all values of x.

SOLUTION In Theorem 1, we let/(.v) = e' and « = 0, noting that /is infinitely


differentiable on (-3°, ^) and that/""(.v) = ^' holds for all values of n ^ 1. If r is
any positive number, then for x in the interval (— r, r), we have

|/""(.v)| = \e'\ = e' < e'


SECTION 11.8 TAYLOR AND MACLAURIN SERIES 697

since x<r. Thus, taking M = e' \n Theorem 1 , we conclude that

^ = 2.
*-o
—r^.v' = 2.
^-^
,=0
77.V'
k\
=2.—-=i+A
,^0 k\
+ —+—+
2! 3!
•••

holds for all values of a between -r and r. Since we can choose ; as large as we
please, e' = \ + x + (a-/2!) + (a73!) + • • •
holds for all values of x.

y"
EXAMPLE 4 sm A = A -
v^
—+—
V-'
-—v^ + — --—+...
v"^
for all values of a.

SOLUTION Successive derivatives of sin a give only ±sin x or ±cos a. In any


case, |D; sin a| < 1, so we can take A/ = 1 in Theorem 1, and r can be chosen as
large as we please. Then

e;„
sin A
-
= sin

o _L
H S^ A ^i"
X-
, cos
V^ H
, sin ^
v'* H
cos ,
r^ — . . .

1! 2! 3! 4! 5!

u
that

IS, sin A =A- — + — -—-+••


•^•' x' x'

holds for all values of x.

EXAMPLE 5 cos A = 1
-—- + —--—- + ••. for all values of x.
2! 4! 6!

SOLUTION This expansion can be obtained in essentially the same way as the
expansion for sin a was obtained in Example 4. However, it is more interesting to
obtain the power series for the cosine by differentiating the power series for the sine
as follows:

/ V-' V-' v'


cos A = Dj sin A = D, A h f-

^ 3! 5! 7!

3! 5! 7! 2! 4! 6!

Some elementary consequences of the expansions obtained above are shown in


the following examples.

r-v . ...». ^ , r- J . ^ 1
— cos A
698 CHAPTER II INFINITE SERIES

B EXAiviPl.K 7 Estimate e '


with an error of no more tiian 5/10"*. Round off your
answer to three decimal places.

so that

= s
(-ir
1
- 1 + j_
3!
j_
4!
—+— 1

5!
1

6!

By estimating e '
using only the first seven terms, it follows from Leibniz's theo-
rem that we make an error not exceeding the absolute value of the first omitted term

10

5040 5(10-') 5(10-*) lO'* 10"

Hence, 1 - 1 + + 24 ~ 120 + 720 ~ 0.368

Example 7 illustrates the following conventional rounding-off rule* for numeri-


cal approximations;

If you intend to round off a numerical approximation to /; decimal places, make


sure that the error involved in the approximation does not exceed 5 x 10''""^'*.

For instance, in Example 7, we rounded off to /; = 3 decimal places; our error did
not exceed 5 x 10"'^^" = 5/10''. Use of this rounding-off rule is further illus-
trated in the following example.

B EXAMPLE 8 Estimate Jo sin .v" d^v. Round off your answer to four decimal
places.

SOLUTION Since sin .v" has no elementary antiderivative, we cannot evaluate


the integral by using the fundamental theorem of calculus. However, if we replace .v

by .V" in the Maclaurin series

— — 3!
-I-

5! 7!

we obtain the Maclaurin series

3! 5! 7!
^

- -I-

3! 5!

Integrating term by term, we obtain

J,,
sin jr' dr = I

^
— 3 7(3!
-I-

11(5!) 15(7!)
-I-
•)[

-f -I-

3 7(3!) 11(5!) 15(7!

''For an indication of why the rounding-off rule is reasonable, see Problem 53.
\
SECTION 11.8 TAYLOR AND MACLAURIN SERIES 699

If we use only the first three terms of the series, then by Leibniz"s theorem, our
error will be less than the absolute value of the fourth term.

0.000013 = 1.3 X 10"-'*


< 5 X 10"*
15(7!)

Thus, rounding off to four decimal places, we have

sin x- cLx ~ — 0.3103


3 7(3!) 11(5!)

For convenience, we now assemble some of the more important power series
expansions obtained in this and the previous section.

1 e* = 1 + A- + ^
—+ —+
2!
-

3! 4!
h •

700 ClIAKI KK 1 1 INFINITE SERIES

21 fix) = X- sin 2x 22 f(x) = In ( 1 + x-)

23 f(x) = sin' X. [Hint: sin' -t = i(l - cos Iv).]

24 /U) = x-e~''

it" A ?t

if X =

if .V ?^

if .V =

if .V y^

if v =
SECTION 11.9 BINONUAL SERIES 701

11.9 Binomial Series


We now set for ourselves the problem of finding the Maclaunn series expansion for
/ defined by/(.v) =
the function + i^i ,xf. where p is a constant and 1 + v >
Smcc fix) = p(l +x)P-\ we have

(1 +x)f'(.x)=pf(x)
Suppose that /can be expanded into a Maclaunn series

fW = Co + cix + cz-r^ + C3x^ +


Then

fix) = ci + Ic.x + 3c3jr- + • • •

so that

(1 +x)f'(x) = f'(x) + xf'ix)

= (ci + Ic.x + 3c3.r- + • • •) + (C,.V + 2C2X- + iCyX^ + •)


= ci + (2c2 + ci).x + (3c3 + 2c2)x- + (4c4 + 3c3).r- + • • •

The condition (1 + .v)/'(.r) = pf{x) therefore becomes

ci + (2c2 + c,).r + (3c3 + 2c2)x- + (4c4 + 'icj,)x^ + ••


= PCo + pC^X + pC2X- + pc^X^ + • •

Equating coefficients of like powers of v in this equation (see Problem 52 in Section


11.8), we have

ci = pco Ic. + c, = PC 3C3 + Ic. = pc. 4C4 + 3C3 = pc^


and so forth. Evidently, [n + l)r„_, + nc„ = pc„: that is.

p- n

n +
holds for all « > 0, Hence.

- Dp
c\ = pco '^2
_
- —
P-
— :;
1
c, = (p
; Co

^ _
''-
P~ 2
''
_ (p-2)(p~ Dp
jr. ^0
3

_
— P- 3
C3
_ (p- 3)(p-
— 2)(p - Dp
Co
4 4-3-2
and so forth. Since /(O) = (1 + 0)" = F= 1, it follows that Co = 1.
The calculations above suggest the general formula

n factors
702 CHAPTER II INHNITE SERIES

Thus, putting cq = 1 . we obtain the power series

ic

THEOREM I
SECTION 11.9 BINOMIAL SERIES 703

(iii) Define the function g by g(x) = ^"=0 Q^''- The power series

2r=o CjfcJ:* was set up in the first place so that

(1 + x)D, 2 Qx* =/? E r^.x*


*=o k=0

for |.v| < R; hence, ( 1 + .x)g'(.x) = p^(jr) holds for |.v| < 1 . (For the details, see
Problem 28.) Now we have

_g{x) _ (1 + xfg'ix) - g(x)p{\ + x)P~'


^
(I + x)P (1 + xfP
(1 +.V)''-'
^
(l+xf '^' ^ ''^^'*'* ~ ^^*''^ " °
g(x)
for X < 1. It follows that is a constant, sav
^
(1 +.V)''

= A for \x\ < 1


( 1 + x)P

Putting X = 0, we find that

A — = e(0) = Cn
° = 1
(1 +0^ *

Therefore,

gU) = K{1 +x)P= \-il+ x)P = (1 + x)/" for |x| < 1

that is,

(1 + xF = g(x) = S Q.v* holds for |x| < 1

EXAMPLE I Find a power series expansion for Vl + x, |x| < 1

SOLUTION Take/7 = 5 in Theorem l.Thus,Co = Kc, = 3, c^ = -5.^3 = m^,


and so forth. In general,

M- 1)(5- 2) ••• (g->i + I) ^


~ 1(1 -3)(1 - 6)--- [1 - 3(« - 1)]

n\ 3";i!

(-l)"-''2-5-8- 11 •••(3/! -4)


= ZT-.
^
for/j>2
3"n\

and therefore

VTT7 = I +h-hx^ + -iix^ - •


=
1 V
Z
(-l)*""2-5-8- 11 ••• (3A:- 4) ,

for|x|<I
l+J-^+ ^, -v^'

) EXAMPLE 2 Use the first three terms of the expansion obtained in Example 1 to
approximate V28. Give a bound on the error involved.
704 CHAPTER 11 INFINITE SERIES

SOLUTION Naturally, we wish to use the fact that V27 = 3. Thus, we write

\/28 = ^n + 1 = </21(\ +Y7) = V27 Vl + j7 = 3\/l +^


Putting jc = ^ in Example 1 , we obtain

VI + 27 ~ ' + 3(27) ~ 9(27) — 6561

with an error no larger than ^{-iif. since the series (apart from the first term) is

alternating and Leibniz's theorem applies. Therefore,

^28 = 3\/l + rr - 3(|||t) = 3.036579 . . .

This approximation underestimates V28 with an error that does not exceed
3-^(A)' < 0.0000095. (The true value of \>28 is 3.03658897. . . .)

1 EXAMPLE 3 Estimate I Vl + x^ dx, and place a bound on the error involved.

SOLUTION By Example 1 , for |.v| < 1 , we have


VTT7 = 1 + it - hx- + iix^ - •
Replacing x by x^, we obtain

x/FTP = 1 + ix' -hx^ + hx'-


Therefore,

Vl +.v^ dx
^
ri/2 ri/2 ri/2 ri/2
\ dx+ \ y^ dx- \ iv* dx + rr.v'' dx- ••
^ ^ ^ \)

= I + (M)(b'' - {M){k)' + (A)(ii))(2)"^' - • •


Apart from the first term, the last series is alternating and the terms are decreasing in

absolute value (see Problems 31 and 32). Hence, Leibniz's theorem applies, so that
using (say) the first three terms of the series, we have
fl/2
Vl +.r' dlv « I + (M)(h'' - (5)(7)(5)' = 0.505084 . . .

'0

with an error whose absolute value does not exceed (8T)(to)(2)'" < 0.000007.
Therefore, rounding off to four decimal places, we have /o^~ Vl + x^ dx =
0.5051.

Problem Set 11.9

In Problems 1 to 12, use the binomial series expansion (Theorem D - ' x


to find a Maclaurin series expansion for each expression. Specify '
V'TTlr -^j _ Ji
the range of values of .v for which the expansion is correct.

</\T-x 2vrT7 ' Tm^ »<^ + '^)


3/2

Vl -x^ Vl -;t vrT7


REVIEW PROBLEM SET 705

11 <^lT7 12 (b) Prove that (1 + .v)D, S c^x'' =p S Q-t* for |.x| < 1.
-yvT
29 Compare the binomial series expansion of (1 + .v)~' with the
lln Problems 13 to 18. use the first three terms of an appropriate
geometric series expansion of the same expression.
binomial series to estimate each number. Give an upper bound for
the absolute value of the error. 30 If a is a positive constant and p is any constant, show that

+ xf = + +
^'^~ " qP-'x-
13 VlOl 14 V99 Ui a'' pa''- '
X

15 Vl.03 16 \'^
pip- Dip
— -2)
H a'^ X + • • •

17 vT? 18
\ 100 pip - D ip - k + \)
,
Ti " -^

] In Problems 19 to 26. estimate each quantity, rounded off to three


decimal places. (Take sufficiently many terms so that the absolute for |.v| < a.
value of the error does not exceed 5 x 10"'*.)

dx
31 Let (1 + xy = 2 QV* for |.v| < 1 be the binomial series ex-
19 ["'
VTT7dx 20
Vl +.X

f05 ["' dx
pansion. Show that if s .v < 1 and n > p. then 2 Ckx'' is

21 - A = n+
\ 1 x^ dx 22 1

an altematina series. [Hint: c„^, = c„.


fOA
23 wn^dx > -\ and prove
Jq Vl + X- 32 In Problem 3 1 . assume that p that the terms in
the series ^t=n^\ Q-v* are decreasing in absolute value (so that
''^
25 V'27 + dx 26 Leibniz's theorem is applicable).
J
.v'
f V\6-x' .
.

33 Exactly what happens in the binomial series expansion when the

27 Given a sequence exponent p is a positive integer? Is the expansion still correct?


cq. Ci. ci. c^. c^. . . . such that
For what values of .t is it correct? Why?

= P- >'
>
and for n
—^=^= and
'

Co 0,-i
n + 1 34 From the binomial series expansion for the fact
\ 1 + .V-
where p is a constant, prove by mathematical induction that
that sinh .v =
f'
— ,
di
. find a power series expansion for
— >
Jo Vl +r
c„ = p(p - Dip - 2) (p - n + I) for H 1
sinh '
X.
/I I

28 Let p be a given constant and define cq = 1 and 35 From the binomial series expansion for and the fact

- Vl - X-
c„ = {\/n\)p{p - l)(p 2) • • •
(p - n + 1) for « > 1

that sin =- find a power series expansion for

(a) Prove that c„+


P
c„ for n > 0.
f
'o VI -r
,

n+ 1

Review Problem Set, Chapter 11


In Problems 1 to 12. determine whether each sequence converges or - n\h
7/i-' + 3»-
diverges. If the sequence converges, find its limit.
l-
V3n + 1 J 3n- + n-ih"
n(n + 1) \ sin n \ 1 +(-1)"
M I
3„2 + 7„ J M J

„ J
706 tllAKlKR II INFINITE SERIF:S

25 Zj (sin
I
-
k
— sin
k+
I

26 If {A",,} is a given sequence and/) is a fixed positive integer, find a


formula for the nth partial sum of the series Xt=i ^^k
~ ^*+p)-
9 {n- + (
- 1 )"2n] Assuming that lim b„ = L, find a formula for the sum of the

series 21^=1 (** - b^+p).

I (/!+!) + (-1)"(1 -«) ) 27 Find an infinite series whose nth partial sum is given by

+ '" " +
(2j! 1)! 2n 5

Determine whether the resulting series converges, and if it does,


In Problems 13 to 16, indicate whether each sequence is increasing, find its sum.
decreasing, or nonmonotone.
In Problems 28 to 31, use the facts concerning geometric series to
13 {2"} find the sum of each series.
•^{i
|1^'
15 16 {(-!)"} -,lKir-(i)i »i,
17 Is the sequence {n - (2"/")} monotone? Why?
".I Kir "(in
30 .s
i' I)
18 For each positive integer n.

a„ =
111
—+
It n + \
+
let

n + 2
+ • • • +
In
1

32 Assume that

{a,,} satisfies \a„\


A and B are positive constants and that the sequence
s AB" for all positive integers n. Prove that the
(a) Show that {a„} is a monotone sequence. Is it increasing or series 2*=u «t.v* converges if |,v| < \/B.
decreasing?

(b) Does {«„} converge or diverge? Why? 2n + 1

33 Criticize the following argument: Since lim In = 0,


19 Indicate whether the sequence
n — + '-
2n - 1

1 +
(-1)"- the senes Zj In
V —2* +— 1

must be convergent.
4"-! i-= 1 2A: 1
4 16

is bounded or unbounded; increasing, decreasing, or non-


34 Make an informal argument to show that if the series 2^^= i
b^
monotone; convergent or divergent.
converges and if the sequence {a,,} converges, then the series
20 If {a„} and {h„) are convergent sequences and a„ < b„ holds for 2*= I «i^i must converge.
all positive integers n, prove that lim a„ < lim b„.

In Problems 35 to 38, use the integral test to determine whether each


21 Explain carefully the distinction between a sequence and a
series converges.
.series.

35 2 kf
36 2 10 + k'
1^2 k(ln
In Problems 22 to 25, find the sum of each series by forcing the
terms in the partial sums to telescope.

22 S 37 2 4^
4=1
t,, (*+ lHk + 2)(k + 3)

*+ 1 - Vk Problems 39 to 42, use the direct comparison test to determine the


23 S Vk- + k
In
convergence of each series.

24E (2k - 1)(2* + 3) »i,p^(i)' 40 S 3 + A!


REVIEW PROBLEM SET 707

41 I 42 E 60 E 2*U + 4)*

*=i 5^+1 lOA.-

In Problems 43 to 52. use any appropriate method to decide whether 61


V
Z i
(-1) "TT'-'^"'"
10*
'^'

each series converges or diverges. If it converges, determine


whether the convergence is absolute or conditional.

•-•'*^'^ (-1)' 62 2 -I-5-9-


2-4-6-8
13 ••• (4A:

••(2/t)
- 3)
U+ 6)'

43 S= 1
k + 10
44 S
(-1/2^-'
63 S 2* + 1
U- 3)-

45 2 k+ \ V 9 ^ ,-7-, In (1 + 1A-)
^
642 (1+2 + 3 + 4+ ••• + k)x^-^
1 +(-1)' (-ir
47 i 48 2
", In (e* + e-*)

In Problems 65 and 66. find the Taylor series expansion for each
function about the indicated center a. Give the rMigeof_iaIues of
„ V J o
1-3-5 -
• .v
•••(2A-
^
• •
1 1)
^'^ for which the expansion is correct.

65 /(.v) = In .V. a = 1 66 fix) = Vx.a = 4

jt^i
L 1-3-5 ••• (2A.- 1)
In Problems 67 to 72, find the first four terms of the Taylor series

for each function at the given value of a.

51 S (-l)^*'e- 52 2)
i_,
sin (tt*
\
+
In
-)
= e\ = -\ 68 fix) = tan a = tt/4
A-
67 /(.v) a .v.

69 fix) = Vx.a=\ 70 fix) = In ( l/.v). a = 2


El 53 Estimate the sum of the given series with an error not exceeding
5 X lO""* in absolute value. 71 g{x) = sin 2v. a = tt/A 11 h{x) = sec .v. a = 7r/6

(a) S (-1)*"'
A-
-2*
(b) S (-1)*'
(3Af 73 Show that the Taylor series in Problem 67 actually converges to
the function for all values of .t by direct use of Theorem 1 in

Section 1 1.8.
54 Give an example of a convergent series Xt=i ^* "^ positive

terms for which lim —at- -


^
does not exist.
74 (a) Let / be the function defined by

In Problems 55 to 64. find the center a. the radius R. and the interval for .V #
fix)
/ of convergence of the given power series.
n^4-<. for -r =
(.V - 1)-
55 2 A-5*
56 Z^ (sin '^M.*
Find the Maclaurin series expansion for/(.v) and indicate the
k=0
values of .V for which it represents the function. Show that/
is continuous.

57 2 (cos Trk)(x + 2)* (b) Find the Maclaurin series expansion for/'.
1=0
(c) Use the result of part (b) to find the sum of the series

'
58 S 1-3-5-7 • •(2/1- l).v*
i a+D!
t:,

75 Use a power series to prove that

tan"' /(/f = .v tan"' .r- i In (1 +.V-) for |.t| < 1


708 niAlTER II INFINITK SERIKS

In Problems 76 to 81 , use the binomial series to find a power series 85 Find Maclaurin series expansions for

expansion for each expression. In each case, specify the range of (a) sin X + cos x (b) cos" .v - sin" .t

values of A for which the expansion works.


(c) tan '
.v' (d) 10'

86 Suppose that the power series 2r=(i 'V*^* has a positive radius
r R by
of convergence and that the function / defined

f(x) = 2r=n ''*-«* for |a:| < /? is an even function. Show that
77
V 1 + .r-
Q= for every odd positive integer k.

In Problems 87 to 92. a function / is defined in terms of a power


78 D,\/\ + X'
series. Write a formula for/ in finite temis.

79 -
( 1 2v)-/^
87/(.v)=.v-- +
.v'
---+.v^ .v'
•••

80 (16 + .V
')'/''

81 VI + t- clt

82 Find a function/such that/"(.v) + a/(.v) = 0, where a is a posi- 89 j\x) = X


= Va. 3! 5! 9!
tive constant. /(()) = 0. and/'(0) (Hint: Expand/ into a

Maclaurin series with unknown coefficients. Then determine


these coefficients.) 90 f(x) = I + sm X + -—— +
sin" .V sin- .V

3! 4!

Din Problems 83 and 84. use the first three terms of an appropriate
91 /(,v) = 1 + .V In 2 + —-—
(In 2)-
X-
,
+
(In 2)' ,

.V-'
^
+
(In 2)^ ^

binomial series to estimate each number. Give an upper bound for 3! 4!

the absolute value of the error.


3*+ I

92/(.v)= "Z (-1)


83 \^ 84 Vl + X- dx 2*/l-!
VECTORS IN THE PLANE

Until now. we have dealt exclusively with quantities — such as length, area, vol-
ume, angle, mass, density, speed, time, temperature, and probability that can be —
measured or represented by real numbers. Since real numbers can be represented by
points on a number scale, such quantities are often called scalars.
On the other hand, mathematicians and scientists often must deal with quantities
that cannot be described or represented by a single real number — quantities that
have both magnitude and direction. Such quantities are called vectors and include
force, velocity, acceleration, displacement, momentum, electric field, magnetic
field, gravitational field, angular velocity, and angular momentum.*
Actually, a vector is more than just a quantity with magnitude and direction,
since it can interact with other vectors and with scalars in certain well-defined ways.
Physicists define vectors to be quantities that transform in a particular manner under
groups of symmetries, while mathematicians use an even more abstract definition of
"

a vector as an element of a "linear space." However, for our purposes, a simple


geometric definition is quite adequate.

12.1 Vectors

By definition a vector is a directed line segment: that is. in everyday language, an


arrow. Each vector has a tail end (also called an initial point) and a head end (also
called a terminal point) and is understood to be directed from its tail end to its head
end. Reversing the arrow gives us a vector in the opposite direction.
It is essential to use some special notation for vectors so that they can be distin-
guished from scalars. In this book we use A, B, C, and so forth to denote vectors.

*We have previously treated some of these quantities, for instance, force, velocity, and
acceleration, as if they were scalars — but only in connection with straight-line motion where
the direction wa.s understood.

709
710 VECTORS IN THK PLANE

Figure I y= terminal poinl


If a particle is moved from the point P to the point Q, we say that the particle has
= final position
undergone a displacement from P \o Q (Figure 1). Such a displacement can be
represented by the vector D whose tail end (initial point) is P and whose head end
(terminal point) is Q. When we say that the particle has undergone the displacement
P=
=
initial point
original position
D in Figurewe simply mean that it started at P and finished at Q we do not care
1 , —
how it P to Q. For instance, it may have gone first from P to R. then from
got from
R to Q as in Figure 2. If the particle is first displaced from P to R call this —
Figure 2 displacement A —
and is then displaced from R to Q call this displacement B —
then the resultant displacement is just D. and we write A + B = D (Figure 2). This
illustrates the head-to-tail rule for adding vectors:

If the head end of A coincides with the tail end of B. then the vector D whose tail
end is the tail end of A and whose head end is the head end of B is called the sum
of A and B and written as

D = A + B
Figure 3 PtQt =P2Q2
A vector D whose tail end is at the point P and whose head end is at the point Q is

written as
P,Q,
li = PQ
With this notation, the head-to-tail rule can be written
P,Q
¥Q =PR +RQ
which is quite easy to remember.
If two particles, starting from different points P\ and P;. are moved due north
Figure 4 through the same distance, it is traditional to say that they have undergone the same
displacement (Figure 3). More generally, nvo vectors that are parallel, point in the
same direction, and have the same length are usually regarded as being equal.
Following this convention, the two vectors in Figure 3 are regarded as being equal,
and we write

Actually, the equality convention for vectors is extremely useful, since it implies
that a vector may be moved around freely without being changed — provided only
that it is always kept parallel to its original position and that its direction and length
remain unchanged. Thus, for instance, all the arrows in Figure 4 represent the same
Figure 5
vector A.
The fact that vectors can be moved around as specified above implies that any m-o
vectors A and B can he brought into a head-to-tail position (Figure 5): hence, any
two vectors have a sum.
We can also move any two vectors A and B into a tail-to-tail position (Figure 6a)
so that they form two adjacent sides of a parallelogram (Figure 6b). We refer to this
parallelogram as the parallelogram spanned by A and B.
Figure 6 Notice that the diagonal vector whose tail end coincides with the common tail

ends of A and B in the parallelogram spanned by A and B is the sum of A and B,


as can be seen by looking at the top half of the parallelogram in Figure 7. Glancing
at the bottom half of this parallelogram, we see that the same diagonal vector is the
sum of B and A; that is, we see that

A + B = B + A
SECTION 12.1 VECTORS 711

Figure 7 This equation expresses the commutative law of vector addition. The technique of
finding the sum of two vectors by forming the diagonal of a parallelogram is called
the parallelogram rule for adding vectors.
Vectors also satisfy the associative law of vector addition

A + (B + C) = (A + B) + C
as can be seen by examining Figure 8.

For many of the same reasons that make it useful to have a zero scalar, it is also
convenient to introduce a zero vector, written 0. Intuitively, can be thought of as
an arrow that has shrunk to a single point. Alternatively, it can be regarded as a

Figure 8
vector QQ that starts and ends at the same point Q. Just as nonzero vectors can be
moved around subject to the conditions given above, the zero vector can also be
moved around, and we have = QQ = RR = SS = . Of course, the zero vec-
tor is the only vector whose length is 0, and it is the only vector whose direction is

indeterminate. Notice that PQ + QQ = PQ; that is, we have the law

A + {B + C) = (A + B) + C
A +
which, in algebraic parlance, says that is the additive identity for vectors.
If we turn a vector A around by interchanging its tail and head ends, we obtain a
vector in the opposite direction called the negative of A and written -A (Figure 9).
Figure 9 Evidently, if A = FQ, then -A = QP. Notice that the sum PQ + QP = PP = 0:
that is, we have the law

A + (-A)
which expresses the fact that -A is the additive inverse of A.
We now define the operation of vector subtraction by the equation

B = A + (-B)
Figure 10 Notice that A - B is the solution X of the vector equation X + B = A (Problem
15); that is, A - B is the vector which, when added to B, gives A (Figure 10).
Thus, if A and B have the same tail end, as in Figure 10, then A - B is the vector
running between their head ends and pointing toward the head end of\. We refer to
thismethod of finding the vector difference A - B geometrically as the triangle
rule for vector subtraction.
If A is a vector, it seems reasonable on algebraic grounds to define 2A by the
equation 2A = A + A. Using the head-to-tail rule to add A to itself, we find that 2A
has the same direction as A but is twice as long (Figure 1 1). More generally, we
Figure 11 make the following rules for multiplication of a vector A by a scalar s:

1 If .V > 0, then 5-A is the vector in the same direction as A, but i times as
long.

2 If .9 < 0, then sA is the vector in the direction opposite to A, but l^]


times
as long.

Naturally, we understand that

j-A = if either i = or A= (or both)


712 CHAPTER 12 VKCIORS IN THE PLANE

Figure 12 Figure 12 shows a vector A and some of its scalar multiples. Notice that
(— 1)A = —A. More generally.

\"
V \,A^\- (-,v)A = ,v(-A)

holds for all scalars s and all vectors A. (Why?)


Multiplication of a vector by a scalar s > 1 "stretches"' the vector by a factor of
.y, while multiplication by a scalar 5 between and 1 "'shrinks" it by a factor of s.

If all vectors appearing in a diagram (Figure 13a) are multiplied by the same posi-
tive scalar .v, i 5^ 1 , then the resulting diagram is either a magnification (Figure 13b)
Figure 13 or a compression (Figure 13c) of the original diagram, according to whether s > 1

or s < 1. In Figure 13 notice that A + B =C by the head-to-tail rule; likewise,

s\ + iB = sC Substituting from the first equation into the second, we obtain the
first distributive law .vA 4- iB = s{\ + B), s > 0. Now suppose that s > 0, so
that —s < 0. Using the first distributive law above, we have

(-i)A -I- (-s)B = i(-A) -I- i(-B) = 4(-A) + (-B)] = 5[-(A + B)]

= (-s){A + B)

Hence, the first distributive law works for negative scalars as well. Since it obvi-
ously works when s = 0, we have

(b) .vA -I- .vB = .v(A -H B)

for all scalars .? and for all vectors A and B.


sC There is a second distributive law involving two scalars and one vector:
0<s< 1

(0 5A -I- ;A = (i -I- t)A

This is an obvious consequence of the rules for multiplication by scalars when both
5 and / are nonnegative. If you check the other possible cases, it is clear that the
equation holds for all scalars s and t.

In addition to the two distributive laws, there is an associative type of law for
scalar multiplication:

(i7)A = i(/A)

which can be confirmed geometrically simply by considering the various possible


cases according to whether 5 and / are positive, negative, or zero.
If s is a nonzero scalar and A is a vector, the expression ( 1 A)A is often written as
A/.v. Thus, when we speak of dividing the vector A by the nonzero scalar i, we
simply mean to multiply A by \/s.

In order to show that two vectors A and B are parallel, you must show that one of
the two vectors (either one) is a scalar multiple of the other. Indeed, we take this as
our official definition.

DEFINITION 1 Parallel Vectors

We say that the vectors A and B are parallel if there is a scalar s such that
A = sB or if there is a scalar i such that B = ;A. Two vectors A and B that are
not parallel are said to be linearly independent.
SECTION 12.1 VECTORS 713

If A= sB. where i > 0, then we say not only that A and B are parallel, but also
that they have the same direction; however, if A= sB. where .$ < 0. then we say
that A and B are parallel, but have opposite directions.
We notice three simple consequences of Definition 1 . First, «/iv vector is parallel
(Why?) Second, the zero vector is parallel to every vector because, if A is
to itself.

any vector, then = OA. (This is not unreasonable since the zero vector has an
indeterminate direction anyway.) Third, if two vectors are linearly independent,
then neither of them can be zero since the zero vector is parallel to every other
vector.
The following theorem gives an important property of two linearly independent
(that is, nonparallel) vectors.*

THEORE.M 1 Linear Independence of Two Vectors

If A B are linearly
and independent vectors, then the only scalars s and t for
which 5A + rB = are 5 = and t = 0.

An expression of the form s\ + rB is called a linear combination of the vectors


A and B with coefficients s and t. To say that A and B are linearly independent is

equivalent to saying that the only way to obtain as a linear combination of A and
B is to make both of the coefficients equal to zero.

Figure 14

Figure 15
714 tHAriER 12 VECTORS IN THE PLANE

Figure 16 Notice that these components are just the coordinates of the head end o/R when its

tail end is at the oriiiin.


Now suppose that P\ = {x^, _V|) and P2 = (.Vi, Vi) (Figure 16). Then we have
OP, = x,i + yj and OP2 = -t^i + y^j; hence, by the triangle rule for subtraction,
P,=(x,.y,)
KPz = OP2 - OPf
= U2i + y2J) - (.Vli + .Vij)

= -'^2' + .V2J - v,i - y,j

= ,V2i - .Vli + y.j - vij

and

P,P: = U: -.v,)i + (v. -.v,)j

EXAMPLE 1 If P = (-2, 5) and Q= (7, 3). write the vectors OP and PQ in

component form.

SOLUTION

OP = -2i + 5j and PQ = [1 - (
-2)]i + |3 - 5]] = 9i - 2j

EXAMPLE Use vectors2 to find the coordinates of a point three-fourths of the


way from (-5, -9) to (7, 7).

SOLUTION Put P = (-5^9) and Q = (7, 7). Let R = (.v, y) be the desired
Figure 17 point (Figure 17). so that PR = iPQ. Now, OR = OP + PR = OP + ^PQ.
Since

= 0.l)
OR = vi + yj OP = -5i - 9j
Q
and
PQ = [1 - (-5)]i + [7 - (-9)]j = 12i + 16j

the equation OR = OP + iPQ can be rewritten as

.vi + yj = -5i-9j + f(12i + 16j)

= -5i - 9j + 9i + 12j

= 4i + 3j

Therefore,
P={-S. -9) - - =
(X 4)i + (y 3)j

Since i and j are linearly independent, the last equation implies that .v — 4 = and
y - 3 = 0. Therefore, (.v, y) = (4, 3).

EXAMPLE 3 Show that, ;/ two vectors are equal, then their components are
equal.

SOLUTION If .vi + yj = a\ + b], then (.v - (/)i + (y - h)'] = 0. Since and j are i

linearly independent, the last equation implies that v - a = and y - /? = 0; that


is, X = a and v = b.

Calculation with vectors in component form is really very simple because of the
"
fact that vectors can be added, subtracted, or multiplied by scalars in a "com-
ponentwise" manner. In fact, if

A= fli -f- hj and B = £ i + dj


SECTION 12.1 VECTORS 715

we have

1
716 CHARIER 12 NKCTORS IN THE PLANE

In Problems 17 to 20, write the vectors PQ and QP in component 48 A car travels 30 miles due east, then 40 miles due north, then 30

torni, miles due cast again. Draw a .scale diagram and represent by
vectors the successive displacements of the car. Add these vec-
17 P = (2, 4) and Q= (6. -1) tors, using the head-to-tail rule, and thus find the resultant dis-

placement of the car.


IS P = (5, 0) and Q= (-1. 7)

49 In Figure 19, PQRS a square, and T the midpoint of SR.


W P = (-3. 0) and Q= (-1. 0)
Express PT in terms of
is

PQ and QR
is

20 P = (0. 0) and Q= (.r. v)


50 Let A = PQ.E^= PR. and C = PS where the point S is on the ,

A= + B = -3i + and
line .segment QR and three-eighths of the way from Q to R (Fig-
In Problems 21 to 30. let 4i 2j, 4j.
ure 20). Express C in terms of A and B.
C = -3i + 7j. Evaluate each expression.

21 A + B 22 A - B Figure 19 Figure 20

23 2A + 3B 24 7A - 5C

25 7B + 3C 26 -3A - 8C
27 3(B - 7C) 28 2A - (B + C)

29 38 - C + 2A 3.A±B.|c
51 Let O = (0, 0), P = (3. 6). Q= (-1. 3). R = (-7, -I), and
In Problems 31 to 38, let A= (4, 2), B = (
- 3, 4), and
S = (3, -6) be points in the xy plane. Find the (scalar) com-
C = (
- 5, 7). Evaluate each expression.
ponents of each of the following vectors with respect to the
standard j basis.
32 4B -
i,
31 5B + 2C C
(a) OP (b) OQ (c) PQ (d) QP
33 2A + 33 34 5A - 2C
(e) PR (f) RS (g) PQ +RS (h) 3QR - 5SP
35 -3A + 2B 36 2A - B
52 In Figure 21, let A = PQ . fi = PR, PS = 5X, and FT = 3B.
37 f A - B 38 i\ - sC Express each of the following vectors in terms of A and B;
(a) QR (b) QS (c) sT
In Problems 39 to 44, let A = 2i + 7j, B = i
- 6j, and
C = -5i + lOj.
(d) SR (e) qM (f) MS

39 Solve the vector equation (3.v + 4v - I2)A + (3.v - 8v)B = Figure 21


for x and y.

40 Find a scalar s such that sA + B is parallel to C.

41 Solve the vector equation (.v - v)A = (3.v + 2y)B - A for .v

and y.

42 Find scalars v and ( such that vA + /B = C.

43 Find scalars C| and Ci such that C|A + CiB = -3j.

44 Show that all vectors in the plane can be obtained by forming


53 Let ABCD be_a quadrilateral, and let P, Q. R. and S be the
linear combinations of A and B.
midpoints of /\6, BC. CD. and DA. respectively. Use vectors to

show that PQRS is a parallelogram.


45 Show geometrically that -(A - B) = B - A.
54 If A and B are two fixed linearly independent vectors in the
46 One of the diagonals of the parallelogram spanned by two vec-
plane, give a geometric construction to show how to find .scalars
tors A and B is A + B. Express the other diagonal in terms of A = C. where
,s and / so that .iA + tB C is an arbitrary given
and B.
vector in the plane.

47 Suppose that the four distinct points P. Q. R. and S all lie on the
55 If A = (/B. where it is a scalar, show that there are scalars s and
same line. If the vector PQ has the same direction as the vector
t. not both :ero. such that .vA -I- ;B = 0.
rS. what are the possible orders in which the points lie on the
line? (For instance. S, Q. R, P — meaning first S, then Q, then R. 56 Show that if A and B are linearly independent vectors, then the
and finally P — is one possibility.) only scalars .v and t for which .vA -(-
/B = are i = and / = 0.
SECTION 12.2 THE DOT PRODLCT 717

12.2 The Dot Product


In Section 12.1 we saw that vectors can be added, subtracted, and multiplied by
scalars. Vectors can also be multiplied by each other; in fact, there are several
different products defined for vectors, each with its own special notation. One of the
most useful is the dot product of two vectors A and B, which is so called because of
the notation "A*B"" traditionally used to denote it. In this section we define and
study the dot product (also called the inner product, or the scalar product) and
calculate lengths of vectors, angles between vectors, and projections of vectors
upon other vectors using dot products.
We begin with a geometric definition of the dot product.

DEFINITION I Dot Product of Vectors

Let A and B be two vectors with lengths (magnitudes) a and b, respectively, and
Figure 1 let be the angle between A and B (Figure 1 ). Then the dot product of A and B
is defined by „ =
•^
A'B ,

ab cos d

(If either A or B is the zero vector, then the angle 6 is indeterminate; but. in this case
a = or fc = 0, so that A B =• 0.

Note that the dot product of two vectors is a scalar — not a vector. In order to
measure the angle 6 between two vectors A and B. we can always move A and B
so that they have a common tail end (as in Figure 1 ) and then use a protractor in the
usual way. We express the angle either in degrees or in radians.

EX.\MPLE 1 If the length of A is 5 units and the length of B is 4 units, find


A-B iven that the angle 6 between A and B is (a) 6 = 0, (b) 6 = tt/I, and
E (c) = 40°

(a) A B = (5)(4) cos = (5)(4)( = 20


• 1 )

(b) A-B = (5)(4) cos (V2) = (5)(4)(0) =


(c) A-B = (5)(4) cos 40° « 15.32
Now, suppose that A is a nonzero vector of length a. Since the angle between A
and itself is zero. A' A = aa cos_0_= a-; hence, a = V A A. On the other hand, if -

A= 0. then A A = and \'^A A still gives the length of A. We use the symbol
• -

|A| for the length, or magnitude, of a vector A; hence, we have the formulas

A-A= A- and A=VA-A


Using the notation |A| and |B| for the lengths of the vectors A and B. we can now
rewrite the formula for the dot product as

A-B = lAllBl cos e

where 8 is the anale between A and B.

EX.\MPLE 2 Let i and j be the standard basis vectors in the .vv plane. Find i-
j-j, and i-j.
718 CHAPTER 12 VKCTORS IN THE PLANE

SOLUTION Since |i| = 1, it follows that i'i = |i|- = 1" = 1. Similarly, j'j = 1.

The angle between i and j is 77/2 radians; hence, i*j = |i|


|j| cos (7r/2) =
(1)(I)(0) = 0.

Properties of the Dot Product


Figure Let A and D be nonzero vectors making an acute angle 6 as in Figure 2. Then
perpendiculars dropped from the cndpoints of A to the line through D cut off a
segment ST of length

1^1 = 1^1 = 1^1 cos e = |A| COS e

The number |A| cos 6 component of A //; the direction ofD or


is called the scalar
the scalar projection of A //; the direction ofD. More generally, if d is the angle
between the vectors A and D, we define

compoA = |A| cos 6

even if d is not an acute angle. Note that, if A and D make an obtuse angle, so that
tt/2 < d < tt, then cos 6 <0 and comppA is negative. Of course, compi,A =
when 6 = v/l.
Because
A'D = |A| |D| cos e = (|A| cos d)\D\ = (compoA) |D|

it follows that the dot product of two vectors component of the first
is the scalar
vector in the direction of the second vector times the length of the second vector. If
we solve the last equation for comp^A, we obtain the useful formula

A-D
compijA
IDI

Figure 3 A glance at Figure 3 shows that the sum of the scalar components of A and B in

the direction of D is equal to the scalar component of A + B in the direction of D,


so that
A-D B-D (A + B)-D
|D| D |D|

that is, scalar components are additive. (You should check appropriate diagrams to
see that scalar components are additive in all cases, even when some of the angles
involved are obtuse.) Multiplying the last equation on both sides by |D|. we obtain
the distributive law for the dot product.

(A + B)-D = A-D + B-D

From the rule for the product of a vector by a scalar, we have

,s-A = Lv A

for all scalars .v and all vectors A. If d is the angle between A and B and if s is

a positive scalar, then is still the angle between .vA and B; hence,

(.vA) - B = |.vA| |B| cos e = |.v| |A| |B| cos = \s\(A • B) = .9(A - B)
SECTION 12.2 THE DOT PRODUCT 719

Figure 4 On the other hand, if i is a negative scalar, then sA makes an angle oi tt — with B
(Figure 4), and we have

(iA)-B = \s\\ |B| cos (tt- 6) = \s\ |A| |B|(-cos d)

= (-.y)|A| |B|(-cos 6) = s\k\ |B| cos 6 = s(A-B)


Consequently, (sA) • B = s(\ • B) holds for s ¥^Q. Since this equation also holds
when 5 = 0, we have the homogeneous law for the dot product:

(5A)-B = 5(A'B)

Because of this law, we may simply write sA'B, without parentheses.


We can now present the four basic laws governing the behavior of the dot
product:

1 AB = B-A
720 CHAriKR 12 \ECTORS IN THE PLANE

In words. Theorem says that ihe dot product of two vectors is the sum of the
1

products of their corresponding scalar components. Using the ordered-pair nota-


tion, we have

(u. h)-{c. d) = ac + hd

EXAMPLE 3 Let A = 2i + 3j and B = 4i - 5j. Calculate

(a) A-B (b) A-(2A - 3B)

SOLUTION Using Theorem 1 , we have

(a) A-B = (2)(4) + (3)(-5) = -7


(b) A-(2A- 3B) = (2i + 3j)-(-8i + 21j) = (2)(-8) + (3)(21) = 47

An important consequence of Theorem 1 is the formula

|.vi + yj|
= V.\" + y-

which says that the length of a vector is the square root of the sum of the
squares of its scalar components. Indeed, if A= .vi + yj, then by Theorem 1.
A A=• A" + V", and so

Al = VA • A = V.v- + V-

Notice that if A t' 0, then

A A A A ' '

Hence,

-— - is a vector of unit length in the same direction as A


A

A vector of unit length is called a unit vector, and the procedure of dividing a
nonzero vector by its own length to obtain a unit vector in the same direction is

called normalizing.

EXAMPLE 4 If A = 3i + 4j, find

(a) |A| (b) A unit vector in the same direction as A


SOLUTION

(a) |A| = V3- + A~ = V25 = 5 units

(b) Normalizing A, we obtain

A 31 + 4! 3 . 4 .

EXAMPLE 5 If A= 3i - 5j and D= 4i + 3j. find the scalar component of A in

the direction of D.

A-D (3)(4) + (-5X3) 3


SOLUTION compijA
IDI V4- + 3-
SECTION 12.2 THE DOT PRODICT 721

If A and B are nonzero vectors, then the formula A B =• |A| |B| cos 6 can be
rewritten in the form

=
A-B
cos
A B
and used to find the cosine of the angle between A and B. Note that 9 = tt/I if
and only if A B=• 0; that is.

A and B are perpendicular^^ if and only if A B= •

Since the zero vector has an indeterminate direction, it is convenient to say, by


definition, that is perpendicular to every vector, even to itself.

EXAMPLE 6 Find the angle 6 between A = (2, 3) and B = (3, -1).

SOLUTION

A-B (2)(3) + (3)(-l) 3 3

|A| IBI V'2- + 3-V3- + (-1)- vTsVlo \^T30

Hence. 6 = cos"' (3/vT30) « 74.74°.

EXAMPLE 7 Find a value of the scalar t so that A= -6i + 3j and B = 4i + fj are


perpendicular vectors.

A-B = (-6)(4) + 3f = 3f- 24


Since A and B are perpendicular if and only if A B =- 0, we require that
3f - 24 = 0: that is. t = 8.

All the propositions of Euclidean geometry can be proved algebraically by using


vectors. The following examples indicate how this is done.

EX.AMPLE 8 Use vectors to demonstrate the Pythagorean theorem.

SOLUTION Consider the right triangle made up of the perpendicular vectors A


Figure 5 and B with hypotenuse A + B (Figure 5). We have

|A + Bp = (A + B) - (A + B) = A (A + - B) + B - ( A+ B)

= A-A + A-B + B-A + B-B


= A-A + 2A-B + B-B
= |A|- + 2A-B -I- |B|-

Since A and B are perpendicular. A B=- 0; hence, |A + B\- = |A|- + |B|-. which
is the Pythagorean theorem.

EXAMPLE 9 Show that the points P = (4, 3). 2= (1, 2), andT? = (4, -7) form
the vertices of a riaht triangle.

*Some authors use the word ""orthogonal" rather than the word ""perpendicular."
722 C HAIMKR 12 VECTORS IN THK PLANE

Figure 6 SOLUTION Figure 6 leads us to suspect that the angle PQR is a right angle. We
propose to confirm this by showing that {QP) (QR) =•
0. Here,
c' = (i.:)

P=(4.3) QP = (4- l)i + (3- 2)j = 3i+j


QR = {4- l)i + (-7-2)j = 3i-9j
Hence, -
(QP) (QR) = •
(31 + j) • (3i - 9j)

= (3)(3) + (l)(-9) =

Dot products are useful in mechanics for calculating the work done by a force
acting on a particle in a direction other than the direction of displacement. Indeed,
suppose that a constant force, represented by the vector F, acts on a particle, start-

ing at the point P. and moves it along a straight line to the point Q. Then the work W
=
/? (4, -7) done by F in producing the displacement D = PQ is defined to be the product of the
scalar component of F in the direction of D and the magnitude |D| of the displace-
ment (Figure 7). Thus,

Figure 7
W
FD Dl = F-D

Notice that, when F and D have the same direction, W= F D =


• |F| |D| as usual.

EXAMPLE 10 A person pushes on a lawnmower handle with a force of 35 pounds


and moves the mower through a distance of 100 feet. How much work is done if the

handle makes an angle of 30° with the ground (Figure 8)?

Figure 8 SOLUTION For the force vector F we have |F| = 35 pounds, while for the
displacement vector D we have |D| = 100 feet. Since the angle between F and D
is = 30°,

V3 \
_ I750V3
W= F D =
• |F| |D| cos e = (35)( 100)
I
foot-pounds
2

Problem Set 12.2

In Problems 1 to 8, a and b denote the lengths (magnitudes) of In Problems 9 to 12, use the information to find the scalar compo-
vectors A and B, respectively, while denotes the angle between A nent conipoA (scalar projection) of A in the direction of D. Denote
und B. the angle between A and D by B.

9 |A| = 10, e = 45° 10 A D =


• 5, iDl = 2
1 If o = 15, Z? = 2, and e = 60°, find A • B.
|A| = 6. e = Tr/2 12 A-D = -7, |D| = 2
2 If u = 7, fc = 4, and = 57r/6, find A • B.

= In Problems 13 to 24, let A= - B and C = —


m 3 If a = 1, /^ 3, and e = 47°, find A-B. i 3j, 2i -1-
j, 2i

Evaluate each expression.


[c] 4 If a = 3, b = 4, and e= 10°, find A-B.
14 |3A| and
13 |A| and |B| ,
CI

5 If a t' 0, /j ?^ 0, and A B = • 0, find fl.


|A + 16 |A - 2B|
15 Bi

6 \Ui = l.b = 3. and A-B = -6, find «.


-
17 |A| + |B| 18 I
2B| -H |3C
7 If a = 10, e = 7r/3, and A B = - 30, find b.

19
8 If a = 2 and fe = 3, explain why |A - B| < 6.
SECTION 12.2 THE DOT PRODUCT 723

A + 3B
21 22 |3A - 4B + C|
|A + 3B|

23 A unit vector having the same direction as A —B


24 A unit vector having the same direction as A +B

In Problems 25 to 32, find (a) A • B. (b) cos 6. where 6 is the angle


between A and B. (c) compeA.
25 A = i+j, B = i-j
26 A= 3i + 2j. B = 4i - 5j

27 A = 4i + j. B = i - 2j

28 A =
724 CHAPIER 12 MXTORS IN THK PLANE

12.3 Vector and Parametric Equations


h inure I

Figure 2 p

Figure 3
SECTION 12.3 VECTOR AND PARAMETRIC EQtATIONS 725

More generally, we make the following definition.

DEFINITION I Graph of a Vector Equation

The graph of an equation involving the position vector R is the set of all points
P, and only those points P, whose position vector R satisfies the equation.

Figure 6 EXAMPLE 2 Sketch a graph of the vector equation

|R - Roi = a

where all vectors lie in the same plane, a is a positive constant, and Rq is a constant
position vector.

SOLUTION Let R = OF and Ro = OPq. so that R - Rq = ?b? (Figure 6a).


The condition |R - RqI = « means that the distance between the points Pg and P is
a units. Thus, as R varies subject to the condition |R - R,,] = a, the point P traces
out a circle of radius a with center at the point P^ (Figure 6b).

If N
a fixed nonzero vector and Rn = OPo.
is then the condition
N'(R - = means that the vectors N and R - Rq are perpendicular (Fig-
Rq)
ure 7). As R = OP varies, subject to the condition that R - Ro is perpendicular to
N. is geometrically clear that P traces out a line through Pn perpendicular to N.
it

We say that the vector N is normal to this line. Therefore.

N-(R - Rn) =

is a vector equation of the line containing the point whose position vector is Ro and
having N as a normal vector.
The equation N • (R - Ro) = can be written as N R - N Rq =
• •
or as
Figure 7
N R=N• •
Ro- Since N Rq • is a constant K, the equation can also be written as

NR = ^
We can convert the last equation into Cartesian scalar form simply by writing
R= + vj and N = Ai + Bj, so that N R = Av + fiv. and
jd •
the equation takes the
form Ax + By = K. This equation can also be written as

Av + Bx + C =

where we have put C = -K. In particular, notice that

N= Ai + Bj

gives a normal vector to the line Av + By + C = 0.

EXAMPLE 3 Find a normal vector to the line y = 2v - 3. and write an equation of


this line in vector form.

SOLUTION The given equation can be rewritten as 2v + (-l)y -3 = 0;


hence, N = 2i + (-l)j is a normal vector to the given line. Let the position
vector R be given by R = + yj. Then N R = .ri • (2i - j) • {.vi + vj) = Zv - y, and
the given equation can be rewritten in vector form as N'R = 3.
726 tHAKTER 12 VECTORS IN THE PLANE

KXAMPLK 4 Convert the vector equation N • (R - Rn) = into Cartesian scalar


form if N = -i + 3j and Ro = 7i — 2j.

SOLUTION With R= .vi + vj, we have

R- R(, = (.vi + yj) - (7i - 2j) = (.v - 7)i + (y + 2)j

Hence.

NMR - Ro) = (-i + 3j)-|(-v - 7)i + (y + 2)jl = -(.v - 7) + 3(y + 2)

The condition NMR - Ro) = is therefore equivalent to

-(.V - 7) + 3(y + 2) = or 3y-.v+13 =

Perpendicular Distance from a Point to a Line


We now use the vector equation of a line to derive a formula for the perpendicular
distance from a point to a line.

THEOREM 1 Distance from a Point to a Line

LetP,
SECTION 12.3 VECTOR AND PARAMETRIC EQUATIONS 727

and = Va- + B-
so that the perpendicular distance d from the point (.Vi, Vj) to the line
Av + Sv + C = is

|A.Vi + gvi + C\

VA- + fi-

le] EXAMPLE 5 Find the perpendicular distance from the point P, = (3. 7) to the line

J= Ix: - 5.
SOLUTION We rewrite the equation v = Iv - 5 in the form 2.x - v - 5 = 0,
that is, in the form A.x + By + C = with A = 2. fi = - 1 . and C= -5. Here
Pi = {xi, yi) = (3, 7), so the desired distance d is

|Ayi + By + I
C\ _ |(2)(3) + (-1)(7) + (-5) 6
2.68 units
Va- + B- V2- + (-1)- V5

Parametric Equations
Often the variable position vector that traces out a curve is controlled by a variable
parameter. Equations involving parameters
scalar called a in an explicit way are
called parametric equations.
As an indication of the way in which parameters are used, we begin by deriving a
vector equation for a line L. not in terms of a normal vector N. but rather in terms of
a direction vector M parallel to the line. Figure 9 shows a line L containing the
point Pi) and parallel to the nonzero vector M. Here. R,, is the position vector of Pq-
P whose position vector is R belongs to the line L if and only
Evidently, the point
if PnP is M. that is. if and only if R - R^ is parallel to M. By Defini-
parallel to
tion 1 in Section 12. R - Rq is parallel to M if and only if there is a scalar such
1 . t

that R - Ro = ?M, that is, R = Rq + tM. As the scalar / varies over the real

numbers, the position vector R = Rq + ?M traces out the straight line L. Here t is
the parameter whose value determines the position vector R. The equation

R= R( tM

is therefore called a vector parametric equation for L.


Figure 10

EX-AMPLE 6 Let L be the line in the xy plane that contains the two points
Po = (6, 2) and P, = (3, 5) (Figure 10).
R = (6 - 3/li + + 3/)j
{:
(a) Find a vector M that is parallel to L.

(b) Find a vector parametric equation for L.

SOLUTION Put R = OP = .vi + yj, Rn = OPo = 6i + 2j. and R, =


OPi = 3i + 5j.

(a) M = 7^1 = R,- Ro = -3i + 3j


(b) R = Ro + rM = (6! + 2j) + ?(-3i + 3j)
= (6 - 3r)i + (2 + 3r)j
Hence, R= (6 — 3f)i + (2 + 3r)j is a vector parametric equation for L.
728 CHAPIKR 12 VECTORS IN THE PLANE

Figure II
SECTION 12.3 VECTOR AND PARAMETRIC EQUATIONS 729

SOLUTION Let R = OP denote the position vector of a point on the line, and let

Ro = OPa- The line is parallel to the vector M = fg/'i = (Ai - ,ro)i + O'l
~ >'o)J;
hence, the vector parametric equation is R = Rq + rM, or

A-i + yj = Aoi + y,jj + r[(A-,


- aq)! + (y, - yo)j]
= [ao + /(A, - Ao)]i + [vo + '(vi - yo)]j
The resulting scalar parametric equations are

Ao + r(A, - Ao)

.vi) + '(vi - Vo)

Parametric equations can provide simple descriptions of curves generated by


physical motion in cases where it may be difficult to find a Cartesian equation. One
such curve, called a cycloid, is traced out by a tack P stuck on the rim of a wheel of
radius a as the wheel rolls without slipping along a straight line — say the x axis. The
cycloid consists of a sequence of arches, one arch for each revolution of the wheel
(Figure 13).

Figure 13

Figure 14
730 CHAKIKR 12 VECTORS IN THK PLANE

The scalar parametric equations are

A = a(t - sin

V = a{\ - cos t)

Sometimes it is desirable to convert a paraitietric equation to a mmparametric


equation by eliminating the parameter. The technique is illustrated by the follow-
ing examples.

EXAMPLE 11 Find a scalar nonparametric equation for the curve R = Tit'i + r\.
and sketch the graph.

Figure 16 SOLUTION With R= Ai + vj, we have .vi + vj = 3/i + r'y, hence,

'

{:;^
To eliminate the parameter / from the scalar parametric equations, we solve the first

equation for t to obtain t = .v/3 and substitute into the second equation, so that

y=(f)
or v =
^
(Figure 16).

EXAMPLE 12 Find a scalar nonparametric equation for the curve


Figure 17

/ - 1

_v = 3/ + 1

and sketch the graph.

SOLUTION From the first equation, we have / - 1 = l/.v, or r = 1 + (lA).


Substitution into the second equation yields

3
>. = 3 I + + 1 v = 4 +

(Figure 17).

EXAMPLE 13 Find a scalar nonparametric equation for R = (cosh /)i + (sinh /)j.

SOLUTION In scalar parametric form, we have

.V = cosh t

V = sinh t

Here the parameter / can be eliminated by using the identity cosh" / — sinh" / = 1 to

obtain x' — v" = 1 , .v > 0.

Given the equation of a curve in scalar parametric form, you can find the deriva-
tive dy/dx by eliminating the parameter and then proceeding as usual; however, it
is possible to find dy/dx directly from the scalar parametric equations. Indeed,
assuming the existence of the required derivatives, we have

dy dx dy
d\ dl dl
SECTION 12.3 VECTOR AND PARAMETRIC EQl ATIONS 731

by the chain rule. Hence, if dx/dt ¥ 0, then

dy _ dy/di

dx dx/dt

Similarly, if we put y' = dy/dx. then we can obtain the second derivative d'y/dx^
by using the chain rule again; hence.

d-\ d\' dy'/dt

dx- dx dx/dt

EXAMPLE 14 Given

x = t- - 6

V = r' + 5

find dy/dx and d-y/dx'

dx 3r
SOLLTION y'
d\/dt
= —
3
for f 5^
dx dx/dt It 2

and

d-y _ dy' _ dy'/dt _ i _ 3


for r 5^
dx- dx dx/dt It At

Problem Set 12.3

In Problems 1 to 4. find the position vector R of each point P. 19 IR - il + IR + il = 6 20 |lR-i| -|R + il

Express R in terms of i and j.


In Problems 21 to 25. find the perpendicular distance from the indi-

1 P= (5. 1 P (0. 7) cated point P] = (.vi. yi) to the line whose equation is given.

3> P = O 4 P (a, a-) 21 P, = (1. 2): 3.v-y = 4

22 P, = (-7. 3);y = iv+ 3


In Problems 5 to 8. find the vector R^ — Ri if Ri and Rj are the
position vectors of P\ and f^. resfKctively. Also find |R: - Rij. 23 P, = (1. 2); (i + j)-(R-2i+j) =

5 = (2. -1).P: = (7. 2) 6 P, = P: = (-3. 24 P, = (0. 0): Av + By + C =


/>, (5. 1). 4)

= = = 25 P, = (4. 0): y = Iv - 5
7 P, (2. -4).P, (-2. -11 8 Pi (a.a-).P2 = ib.b-)

26 Find a formula for the perpendicular distance from the origin to


In Problems 9 to 20, convert each vector equation to Cartesian scalar
nonparametric form, and sketch the graph.
the line N R= • K.

9 (i -4j)-R = 1
In Problems 27 to 32, (a) find a vector M parallel to the line L
containing the two given points Pq and Pi: (b) write a vector para-
10 (2i + 3j)'R = (2i + 3j)-j metric equation for L: (c) write scalar parametric equations for L;
and (d) sketch a graph of L.
11 i-R = 2 + j-R 12 R-j =

13 (i+j)-R = 14 (2i + 3j)-(R - i) = 27Po = (1. 2). Pi =(3. 4)

15 |R| = 3 16 R-R = 16 28 Po = (0. 0). Pi =(-1. 3)

17 |R-(2i + 3j)| =4 18 |R + i - jl
= 3 29Po = (-i. t). Pi = (§. -h
732 CHAKIKR 12 VKtTORS IN THK PLANE

30 /»,, = (0, a). P, = (b. 0), a ¥^ 0, b ^ 53 Let N ^ 0. K 9^ 0. and suppose that L is the line whose vector
equation is N R = • Ai:. Put D= (/i'/|N|-)N. Show thatL also has
31 /'„ = (7r. e), Pt=(-V2,V3) the vector equation D • (R - D) = 0.

32 P„ = {.v„. y„). P, = (.vi.y,)


54 Find a vector equation of the parabola with focus at the origin
whose directrix is the line D • (R - D) = 0. where D is a fixed
In Problems 33 to 36. find a vector parametric equation lor each
nonzero vector.
curve.
55 Find a formula for the perpendicular distance from the point with
33 The line through the point whose position vector is R,, = 7i - 2j position vector R, to the line D*(R - D) = 0. where D is a
and which is parallel to the vector M = -i + .3j constant nonzero vector.

34 The line through the point whose position vector is 56 Show that the vector M -Si + /tj is parallel to the line
Ro = — i/7 + j/3 and which has N = 3i - 7j as a normal vector Av + Bv + C = 0.

35 The circle with center at the point whose position vector is 57 A point P is located on a spoke of a wheel of radius a > at a
R() = 3i + 4j and whose radius is 4 units distance h > from the center. Derive vector and scalar para-
metric equations for the curve traced out by P as the wheel rolls
36 The cycloid generated by a point P on the circumlerence of a
without slipping along the .v axis. Assume that when x = 0,
roiling circle of radius 5 units
P = (0. (/ - b).

In Problems 37 to 44. (a) convert the vector parametric equation to 58 Find a vector equation for the ellipse whose foci are at the points
scalar parametric form, (b) eliminate the parameter / and thus obtain f I
and F2 and whose semimajor axis is a > (Figure 18).
a scalar nonparametric equation, and (c) sketch the graph.

37 R = (2 cos /)i + (2 sin 0.i for < < r 277


Figure 18
38 R = (3 cos t)i + (4 sin Oj for < < / 27r

39 R= (5 cos 2/)i - (5 sin 2/)j for < < f 7r/2

40 R= /-i + fj for - 1 £ s r I

41 R= 4/i + (3f + 5)j for -X < < ; oo

42 R = e'i + e 'i for -cc < <x r

43 R = -i + (2i + \ )j for s < r 2


1-2
44 R = (sin /)i + (sin Oj for < <
r 7r/2

In Problems 45 to 52. (a) eliminate the parameter /. (b) write the


59 Derive a vector parametric equation for the curve traced out by

equation in scalar nonparametric form, and (c) express dy/ch and


the point P halfway between A and B in Figure 19 as / varies

d'\/d\' as functions of I.
with < < I TT. Sketch the graph.

[.v = 3/ + I Figure 19
45 46
\y = 2 - /
V = 3f +

47 48 u - 2y-
ly = 5/
-3l + 2

49
(>— 50
[.r = 3 cos" I

ly = 4 sin" I

I V = 2/ +

1-^- Ix = 4 sin' cos


5.
=';-?' 52
t /

ly = / - 3/ ly = 4 sin r cos" /
SECTION 12.4 VECTOR-VALLED FUNCTIONS OF A SCALAR 733

12.4 Vector- Valued Functions of a Scalar

The vector parametric equations developed in Section 12.3 can be better understood
in terms of the idea of a vector-valued function. Thus, a vector parametric equa-
tion has the form

R= F(/)

where the "independent variable'" is the parameter t. F is the vector-valued func-


tion, and the •dependent variable"" is the position vector R. The following detlni-
tion makes this idea more precise.

DEFINITION 1 Vector- Valued Function

A vector-valued function F assigns a unique vector F(/) to each real number /

domain. The range of F


in its is the set of all vectors of the form F(r) as t runs
through the domain of F.

EXAMPLE 1 Let F be the vector-valued function defined by the equation

F(?) = (2 cos f)i -I- (2 sin ;)j < < f 27!"

(a) What is the domain of F?


(b) What is the range of F?
(c) Find F(7r).

(a) The domain of F is the interval [0, Itt].

(b) As t runs from to 27r. F(n = (2 cos /)i -f- (2 sin /)j. regarded as a posi-
tion vector, traces out a circle in the .\t plane with radius 2 units and center at
the origin. Therefore, the range of F is the set of all position vectors of points
on this circle.

(c) F(7r) = (2 cos 7r)i + (2 sin 7r)j = -2i.

A vector-valued function F whose range is contained in the xy plane can always


be defined by an equation of the form

F(/) = gU)i + hini

Figure 1
where g and /; are ordinary real-valued functions called the
scalar component functions of F.
Let F be a vector- valued function, and let A be a fixed
vector. The scalar |F(/) - A| represents the distance between
the head ends (terminal points) of F{f) and A when their tail
ends (initial points) are at the origin (Figure 1). Evidently.
|F(f) - A| is close to zero when Fit) is close to A. in both

magnitude and direction. Thus, we make the following defi-


nition.
734 t HAITKR 12 VECTORS IN THE PLANE

DEFINITION 2 Limit of a Vector- Valued Function

Suppose that the number c belongs to an open interval / and that every number in

/, except possibly c. belongs to the domain of the vector-valued function F. IfA


is a vector such that lim |F(/) - A| = 0, then we say that F{i) approaches A as a
limit when i approaches c. and we write lim ¥{t) = A.

Limits of vector-valued functions can be calculated "componentwise," as is

shown by the following theorem, whose proof is left as an exercise (Problem 31).

THEOREM 1 Limits of Vector- Valued Functions

Suppose that F(/)


SECTION 12.4 VECTOR-\ ALLED FUNCTIONS OF A SCALAR 735

SOLUTION The scalar component functions of F are

gU) and h(i) = 5/ + 1


t-1
where g is continuous for every value of / except for / = 2 and /) is continuous for
every value of The domain of F
t. consists of all real numbers different from 2. and
F is continuous at ever\' number ; in its domain.

Given a vector-valued function F, we can form the difference quotient

F(f + ^t) - F(/) 1

[F(f + \t) - ¥(t)]


It \t

by direct analogy with the difference quotient of a real-valued function. Notice,


however, that ( \/\t)[T(j + A/) - F(n] is the product of a scalar 1/A/ and a vector
F(r -I- A/) - F(/); hence, a vector. Pursuing the analogy with real-valued func-
it is

tions, we make the following definition.

DEFINITION 4 Derivative of a Vector- Valued Function

Let ;

THEOREM 3
736 CHAPIKR 12 VECTORS IN THE PLANE

(a) F'(/) = (-sinOi + (cos /)j


(b) F"(0 = (-cosf)i-(sinOj
(c) F'(/) •
F"(r) = sin cos - sin
; / / cos r = 0.

The Leibniz differential notation is used in connection with vector-valued func-


tions in much the same way as for real-valued functions. For instance, if F is

a differentiable vector-valued function and the variable vector R is defined by


R = F(/), we write

dR
SECTION 12.4 VECTOR-VALUED FUNCTIONS OF A SCALAR 737

Parts (i) through (iv) can be proved by expanding the vectors in terms of their
components and using Theorem 3. We leave parts (i), (ii), and (iii) as exercises
(Problems 39 and 40) and attend here to parts (iv), (v), and (vi). ,

(iv) Let R= .vi + yj, and let S = Mi + vj. Then, R S= • .vm + yv and /
d
dt
^
(R'S) =
^ d
dt
(.VM + VV) =
d.x

dt
II +X
du
dt
+ V
'
— + -^v
dv
dt
d\
dt

I dx dy \ I du dv
\dt dt I ^ dt dt

dx dv du dv
-i \
+ -j).(«i+vJ) + (.i+,J).(-i +
I
-j
=
^R ^S
S + R
dt dt

(v) Assume that R t^ 0. By part (iv), we have

— R = — VR-R =
d
dt
,
,
d
dt
,

2VR-R
;
1 d
dt
(R-R) =
2|R|
\ (

^
dR
dt
R+ R
dR\
dt '

-^—fT ^R \ R dR
~ ' '
2|R| V" dt I |R| dt

(vi) Assume that w t^ 0. By part (iii), we have

— ^w'
^ /R\
dt
— =
dt
d
(ir''R)=
. Id
\dt
— M'-',\
'
R+
,

H'"'
dR
dt
= -VV
-.d\x'
-—-R
dt
,

+ H'
dR
'-—
_,
dt

w(dRldt) - (dw/dt)R

EX.\MPLE 7 Let R= (5 sin 2t)\ + (5 cos 20j, S = e-'i + e -'j, and w= e ^'.

Find

(a) —R d
dt
,
'

(b) — dt
d
(R-S) (c) —dt
d
(H'S)

SOLUTION By Theorem 4, we have

(a) —R f/

dt
,

'
,

'
= —r
R
|R|
dR
dt

- (5 sin 20i + (5 cos 2?)j


=-"[(10cos 2r)i - (10 sin 2f)j]
V(5 sin 20- + (5 cos 2t)

50 sin 2t cos It — 50 cos It sin It

(b) —(R-S)
dt
738 CHAFIER 12 VECTORS IN THE PLANE

d dw dS
(c) —{wS) = —-S + yv—-
dt dt dt

le-"S

The chain rule also works for vector-valued functions, as the following theorem
shows.

THEOREM 5 Chain Rule for Vector- Valued Functions

Let R be a diffen-ntiable vector-valued function of the scalar


differentiable function of the scalar s. Then, regarding R as a
SECTION 12.4 VECTOR-\ ALLED FlTiCTIONS OF A SCALAR 739

/- - 5f + 6 /- + 7f - 30 25 Let F(f) = (sin /)i + (cos Oj and / = s^ . Use Theorems 4 and 5


3 F(f) = .

1 +
.

j; fo = 3
1-3 t - 3 to find

4 F(r) = + (sin = (a) ^F(0 (b)


-f |F(/f
; 1 '
/)j; /q 1 as dt

26 Show that, if dw/dt and A is a


exists constant sector, then
s F(r) = — 1 .
I + (sin 3r)j: fo
•, .
= — T^

d dw ^
I b -— .,
(hA) = --A
dt dt
6 Fir) = In (? + 1 )i + e~-'y, ?„ =
11 Show that, if dR/di exists and c is a constant scalar, then

In Problems 7 to 12. find F'(f) and F"(t). d dR



-—(cR) =
dt
c——
dt
7 F(/) = (3/-- l)i + (9f' + 5)j
28 Suppose that dR/dt exists and that |R| is constant. Show that R
8 F{f) = In (3 + t)i + (sin f)j
and dR/dt are perpendicular.
9 Fit) = e^'i + (In 2f)j
29 Suppose that R is a variable vector that is always parallel to a

10 F(/) = (3 sec Oi + (4 tan f)j


fixed nonzero vector A. If dR/dt exists, show that dR/dt is

always parallel to the fixed vector A.


11 F(r) = (5 cos /)i + (3 sin f)j

30 If R# and dR/dt exists, fmd a formula for


12 F(f) = (tan"' 2t)i + e'"]

In Problems 13 to 16. find dR/dt and d-R/dr. ^ IRI >

31 Pro\ e Theorem 1

13 R= (cos r)i + (sin r)j

32 Pro\e Theorem 2.
14 R= te-'i + le-'j

33 Prove that the sum or difference of continuous vector-valued


15 R= — i + — functions is again continuous.

34 Prove that the dot product of continuous vector-valued functions


t- 2 . f- 3 .
a continuous real-valued function.
16 R is
f + 2' I + i^
35 If F is a continuous vector-valued function, show that the real-

In Problems 17 to 20. find (a) F'(/). (b) F"(/), (c) F'(r)-F"((). and valued function / defined by /(f) = |F(f)| is also continuous.

(d)— |F(f)| 36 If F is a continuous vector-valued function and if / is a con-


di tinuous real-valued function, both of which have the same
domain, show that the vector- valued function G defined by
17 F(r) = (e' + 3)i + (e' + 7)j G(f) =/(f)F(0 is continuous.

18 F(?) = (3 cos 2r)i + (3 sin 20j 37 Give an appropriate definition of the following one-sided limits:

19 F(0 = (4 sin 3/)i - (4 sin 30j (a) lim. F(/) = A (b) lim F(f) = A

20 F(r) = e'^'i + e^'j


38 Prove Theorem 3.

In Problems 21 to 24. find (a) —


d
dt
(R-S) and (b) —
di
d
(iiR).
39 Prove parts (i) and (ii) of Theorem 4.

40 Prove part (iii) of Theorem 4.

21 R= e-'i + «"-"j, S = (cos 2t)\ + (sin 2f)j. u' = e" 41 Prove that a differentiable vector-valued function is continuous.

22 R= Sri + f-j, S = (sec i)\ + (3 sin r)j. «• = cot / 42 Give an appropriate definition of the following; The vector-
valued function F continuous on the closed interval
+ —^j.
is [a. b].
3j
23 R= (In ;)i S = >i' = cos It
43 Assuming the existence of the required derivatives, show that

— i 2 d-R dR dS „ d'S
-^.
,
24 R= + S = (In Oi + /-j, u- = 5r + ,
4^(R-S) •S + -I- R ^
r + 4 1 f-
••
dt- dt dt dt-
740 CHAPTKR 12 VECTORS IN IHK PLANE

12.5 Velocity, Acceleration, and Arc Length


Figure I A vector parametric equation R = F(/) can be regarded as giving the position vector
R of a moving point P at tiie time t (Figure I). As the time ; varies, the point P
traces out a curve C. Even if the parameter t actually represents some quantity other
than time, it may be useful to think of it — at least for the purposes of this section

as corresponding to the elapsed time since some arbitrary (but fixed) initial instant.
This point of view enables us to study the vector-valued function F in terms of the
intuitively appealing idea of physical motion.
For the remainder of this section, we assume that the moving point P traces out a
cui^e C in the plane and that the variable position vector R of P at time t is given by
R = ¥(t). Furthermore, we assume that F has a first derivative F' and a second
derivative F". We imagine that the moving point P is equipped with an odometer to

measure the distance s that it has traveled along the path C, starting at fo when
; = Iq, and that it is equipped with a speedometer to measure its instantaneous
Figure 2
speed

ds_

lit

at any time / (Figure 2). Here s is just the arc length of the curve C between P,, and
P, and V is the instantaneous rate of change of s with respect to the parameter t.

The velocity vector of the moving point P is defined to be the vector V whose
direction is parallel to the tangent line to the curve C at P and whose length is the
speed of P, so that

V = V
Figure 3

(Figure 3). Here we understand that V points in the "instantaneous direction of


motion" of P, so that if the constraints causing P to follow the curve C were
suddenly removed, then P would "fly off along the tangent line" in the direction
of V.
We are now going to give an informal argument to show that the velocity vector of
a moving point is the derivative with respect to time of the position vector of the
point. To begin the argument, let Af = ^ ~"
'i denote a small positive interval of
time, and put

Ri=OP, =F(r,) and R, = OP. = FC:) = F(r, + A/)

(Figure 4). During the time interval Ar from ti to tj. the point P moves along a small

Figure 4
portion of the curve C between P, and Pi. Denote the arc length of this portion of
C by A.s. In moving from P; to Pn, the point P undergoes the displacement

AR = R. - R| = F(f| + At) - F(?|)

Evidently,

lARl

with better and better approximation as A/ — 0^. Dividing by Af, we obtain

A/
= — AR = I—
A/ '
' I Ar I
SECTION 12.5 VELOCITY. ACCELERATION, AND ARC LENGTH 741

Hence, takina the limit as A?—» 0"^, we have

=
dR
|V| = V
dt

Therefore. dR/dr has the same length as the velocity vector V.


To finish the argument, we only need to show that dR/dt has the same direction
as the velocity vector V. If the time interval It is small, then 1/Af is large; hence,
the difference quotient

AR F(fi + At) - F(ri)


AR
\t Ar

not only has the same direction as the displacement vector AR but also is consider-
ably longer than AR (Figure 5).
As A/ ^ 0* . the displacement vector AR becomes shorter and shorter: however,
the difference quotient AR/Af approaches dR/dt. which is not necessarily 0. At the
same time, AR/Af pivots about the point Pi and approaches the direction of the
tangent line to the curve Cat P, (Figure 6). Since JRM = lim AR/Af , it follows

Figur^ 6 that dR/dt is parallel to this tangent line: hence, dR/dt has the same direction as the
velocity vector V. Since dR/dt has the same length and direction as V, we conclude
that

dR
= \
dt

As the point P moves along the curve C, its velocity vector V can change in
length, in direction, or in both. The instantaneous rate of change of V with respect
to time is a vector d\/dt called the acceleration vector of the moving point P and
denoted by A. Thus,

dR
= F'(/) and
d\ dR = F"(f)
dt dt dt-

EXAMPLE 1 A point P is moving on an elliptical path according to the equation


R = (5 cos 2f)i + (3 sin 2f)j. Distances are measured in meters and time in sec-

onds. Find (a) the velocity vector V, (b) the acceleration vector A, (c) the speed v
at the instant t.

dR
(a) V= = (-10 sin 2f)i + (6 cos 2f)j
dt

d\
(b) A= = (-20 cos 2f)i -^ (-12 sin 2f)j
dt

(c) V = |V| = VlOO sin^ 2f + 36 cos- 2f meters per second

EXAMPLE 2 A moving point P has position vector R= (f-/2)i — f^j at time f.

Find (a) the velocity vector V, (b) the acceleration vector A, and (c) the speed v of
the moving point at the instant when r = 10.
742 CHAPIER 12 VEtrORS IN THE I'LANE

(a) V = ri - 3/-j; hence, when / = 10,V = lOi - 300j.


(b) A = i - bty. hence, when / = 10. A = - 60j.
i

(c) V = |V| = Vr + 9/'*: hence, when / = 10, v = V 1 00(1 + 900) =


IOV9OI.

If the velocity vector V is not the zero vector, then we can normalize V by
dividing it by its own length |V|. The T has the same direction as the
resulting vector
Figure 7 velocity vector, but has unit length (Figure 7). Since V is parallel to the tangent line
to C at P, so is T; hence, we call T the unit tangent vector to the curve C at the
point P. Evidently,
SECTION 12.5 VELOCITY, ACCELERATION. AND ARC LENGTH 743

and

f" ^ ^
i = V[i,''(0]- + [h'U)]- dt
J

In Examples 4 to 6, find the arc length s of the given curve between the indicated
values of the parameter.

EXAMPLE 4 R= (3r-)i + (t^ - 3/)j between f = and / = 1

SOLUTION Here. dR/dt = (6f)i + (3r - 3)j and

V(6/)- + (3f- - 3)- = V36f- + 9f^ - 18r + 9


I
dt

= X/Qf-* + 18f- + 9 = V(3r- + 3)- = 3f- + 3

Therefore.

=
Jf, I
—— d? I
(/f =
Jn
(3r + 3) rfr = (r' + 3?)
In
= 4 units

[x = e '
COS r
E EXAMPLE 5 between t = and f = tt
I.
V = e~' sin f

SOLUTION Tiie given scalar parametric equations are equivalent to the vector
parametric equation R = g(t)\ + Ht)']. where g(l) = e~' cos t and hit) = e~' sin /.

Thus,

g'it) = —e '(cos t + sin t) h'U) = -e '(sin t - cos r)

and

V[g'(0]- + [/!'(?)]- ^/f

= Ve~-'(cos- r + 2 cos r sin f + sin- r) + e"-'(sin- r - 2 sin r cos / + cos- t) dt

= e~'V2(cos- t + sin- t) dt

V2e~' dt
'o

= (-V2e-')

= V2(l - e~ '^) « 1 .35 units


"

EXAMPLE 6 >• = f(x) between .v = a and x = b

SOLUTION With X as the parameter, the scalar equation v = /(a) is equivalent to


the vector parametric equation R= vi +/(-v)j. Here.

dR dR
= i+/'(-v)j Vl- + [f'(x)f and Vl +[f'{x)]-dx
dx dx

which coincides with the formula for the arc length in Section 6.4.
744 CHAPTKR 12 VECTORS IN THE PLANE

Problem Set 12.5

In Problems 1 lo 10, the vector equation of motion of a point/' in the In Problems 25 to 38, find the arc length .v of each curve between the
vv plane is given. Find (a) the velocity vector V. (b) the acceleration indicated values of the parameter.
vector A. (c) the speed v of the moving point at the time i.

25 R = ai - 9)i - (5f + 4)j: ; = to r = 2


1 R = (3r - l)i + (2/ + 5)j
26 R = (at + h)i + (ci + d)y. f = /, to f = f.

2 R = (2 cos f)i - (7 sin Oj


27 R = e-'i + (V' - /)j; I = Olo t = i
3 R = ri + (In Oj
28 R = ?-i + r'j; / = to / = 4
4 R = (r + l)i + /'j
29 R = (3 cos 20i + (3 sin 2oj; f = to f = tt
5 R= 2(/ - sin f)i + 2(1 - cos /)j
X = 3 cos / - cos 3/
6 R = (4 cos' + (4 sin- nj 30 ' I = lo t = IT
f)i
3 sin / - sin 3r

7 R = (cos I + sin t)i + (cos I - sin t)j


cos
".""
t
31 r^~ '_, ; / = to/ = 27r
8 R= (2 cot /)i + (2 sin= /)j
>>'
'

sin r

9 R = (e' cos ;)i + (e' sin Oj ix = t - sin t


32 / = to / = 277
ly = 1
- cos /"
10 R = (/>(/)i +f{(t>(i))i

33 R= (cos f + ? sin t)i + (sin / - ; cos /)j; f = to / = 77/4

In Problems 1 1 to 16. the vector equation of motion of a point P in 34 R = n + ;-j: f = to ; = a


the .vv plane is given. Find (a) the velocity vector V, (b) the acceler-
ation vector A, and (c) the speed i' of the moving point at the instant
35 R= (in Vl + r)i + (tan"' t)y. 1 = lo i = 1

'
when I has the indicated value i,. (6f + 9)'"
36 R = - i + j; J = to ; = 4

11 R = (7;- - t)i + (5f- 7)j; (, = 2


1337 R = (cos' t)i + (sin' Oj: f = to / = 77/9
12 R = (/ sin r)i + (cos /)j; (, = n/2
E] 38 R = ri + (cosh Oj; f = to / = 4
13 R= 3( I + cos TTiYi + 4( I + sin 7rf)j; /, = f

14 R = /i + e'y. /, =
= f{d)
39 The curve C whose equation in polar coordinates is r can
15 R = (In sin /)i + (In cos Oj; /j = 7t/6 be expressed in parametric Cartesian form as

16 R = (4 cos r)i + (4 sin /-)j; /, = Vtt/I (x=f(e)cos e


lv=/(e)sin e

In Problems 17 to 24. find (a) the velocity vector V. (b) the speed v by using 6 as parameter. Using the methods of this section, find

at time I. and (c) the unit tangent vector T at the indicated time f,. a formula for the arc length of the portion of C between the point

where 6 = 9i and the point where 0=02-


17 R= (\2i - 3)i - (7r + 9)j; ;, = 3

18 R = (2f - l)i + (3r + 7)j; ?, = 2 40 Find a formula for the unit tangent vector T to the curve C in

- - Problem 39.
19 R = (/ sin /)i + (1 cos /)j; f, = 7r/2

20 R = (cos' t)i + (sin' /)j: ;, = 3it/4


41 The curve C whose Cartesian equation is y =/(.v) can be ex-
21 R = (-7 + cos 2f)i + (5 + sin 2/)j; /, = 77/6 pressed in vector parametric form as R= .ri +/(.r)j, by using a-

22 R = cos + =
as the parameter. Find a formula for the unit tangent vector T to
ii /)i (f sin f)j; /,
the curve C.

,-^
E123
^1 n
R= -—cos » .

i + -—sin I ,

j; ,, = 77/5
1 + cos J I + COS /
42 If R= F(s), where the parameter 5 is the arc length, find a
[c]24 R= In (r + l)i + e-'i: /, = Vs formula for the unit tangent vector T.
SECTION 12.6 NORMAL VECTORS AND CURVATURE 745

12.6 Normal Vectors and Curvature


In this section we continue the study, initiated in Section 12.5, of the curve C traced
out by a moving point P in the plane according to the vector parametric equation

R= OP= F(/)

(Figure 1 ). As in Section 12.5, the parameter t can be thought of as time, so that the
vector derivatives

V= F'(0 and F"(;)

which we assume exist, represent the velocity and acceleration vectors of P, respec-
tively, at the time /. We continue to denote by s the arc length traced out by P along
the curve C since some arbitrary (but fixed) initial time to. so that

is the instantaneous speed of the moving point P.


For the remainder of this section we assume that the instantaneous speed of the
Figure 2
moving point P is never zero. \' ^ 0, so that we can form the unit tangent vector

Figure 3

T = (cos a)i + (sin a)j


746 CHARIKR U VKCTORS IN THE PLANE

Figure 4 other unit normal vector. N^. is always on your right (Figure 4). Evidently,

N, = -N,

Since N, is obtained by rotating T coiinierclockwise through tt/I radians, it

follows that N; makes an angle of a + {tt/2) radians with the positive x axis.

Therefore, since |N/| = 1. we have

N; = cos (a + — )i + sin (a + —-lj

= — sin
( a)i + (cos a)j

Hence,

N, = -N/ = (sin a)i - (cos «)j

If !i and h are the component functions of F. so that

R = F(?) = !^U)i + /i(Oj

then, as we have seen in Section 12.5,

dR/clt g'it) h'U)


T=
.

Ti
|^R/f/f| ^Ig'U)]- + \h'U)]- y'ig'U)]- + [h'U)]

Also, T = (cos a)i + (sin a)y, hence, it follows that

cos a
SECTION 12.6 NORMAL VECTORS AND CIRVATURE 747

length, is called the curvature ofC at the point P and is traditionally denoted by the
Greek letter k (called -kappa""). Thus, by definition.

da
(Is

Figure 5 Notice that k >


if a is increasing as P moves to trace out the curve: that is, k >
if the unit tangent vector turns counterclockwise as .y is increased (Figure
5a).
Similarly, k <
if the unit tangent vector turns clockwise as i is increased
(Fig-
ure 5b).
If we differentiate the equation T = (cos a)i + (sin a)j on both sides with re-
spect to arc length 5. we obtain

dT da da da
I + I cos a -~ji = - — [(-sin a)i + (cos a)j]
ds ds ds ds

Hence,

dT
= kN,
ds

(b)
The equation dT/ds = k'S, is extremely important in the theory of curves in the
plane. Since N/ is normal to the curve C and k is a scalar, it implies that the vector
dT/ds is always a normal vector to the citr\e C at the point P. Furthermore, since
|N/| = 1. it follows that

Figure 6

Figure 7
748 CHAIMKR 12 VECTORS IN THE PLANE

Evidently, the equation

dT
= kN
ds

always holds, provided that k 9^ 0. If k = 0, the principal normal vector is unde-


fined.
The following important theorem relates the acceleration vector A. the unit tan-

gent vector T, the speed v. the curvature vector dT/ds. the principal normal vector
N, and the curvature k.

THEOREM 1 Resolution of the Acceleration Vector into Tangential and


Normal Components
SECTION 12.5 NORMAL VECTORS AND Cl'RVATURE 749

The formula for curvature in Theorem 2 can be rewritten in terms of the compo-
nent functions g and h of R (Problem 22). Thus, if R= g(/)i + /i(f)j. then

g'U)h"U) - g"(t)h'(t)

{[g'u)]- + [h'it)]-y -

EXAMPLE 2 Find the curvature k of R = ?i + f-j.

SOLUTION Here the component functions g and /; are given by git) = t and
h{t) = r. Using the formula above and noting that

g'(t) = 1 g"(t) = h'{t) = It and h"(t) = 2

(I)(2) - (0)(2r) 2
we obtain
[1- + (2/)-]-'''- (1 + 4f-)-'^-

EXAMPLE 3 Find a formula for the curvature k of the graph ofy = /(a), using .v
as the parameter. Then find the cun'ature k of the sine curve v = sin .v at the point
(77/2. 1).

SOLUTION In vector parametric form, we have R = .vi+/(A-)j, so the


component functions g and h are given by g{x) = x and h(x)=f{x). Thus,
replacing t by .v in the formula for k. we have g'(x) = 1 ,
g"(x) = 0, h'(x) = fix), and
h"(x) = f"(x): hence,

g'(x)h"{x) - g"{x)h'{x)

{[g'ix)f + [h'{x)fr^'-

f'M
-
{1 + [/'(.v)]-}^

Using this formula \vith/(.v) = sin x, /'(.v) = cos v. and /"(a) = —sin v, we have
— sin X
K
(1 + cos- A)'^-

Thus, when .v = 77/2, k l/l=-l.

Problem Set 12.6

In Problems 1 to 16, find (a) the unit tangent vector T, (b) the unit f.v = r - 2f

normal vector N;, (c) the curvature k, and (d) the principal unit Mv=I It
normal vector N for each curve.
7 R = ti + e'j

1 R = (7r - 4)i + (9 - 3nj 2 R= {at + b)i + (ci + d]j


8 R = (2 cos en + (5 sin 0)j

\x = 3 cos /

Iv = 3 sin 10 R= t-i + (In sec r)j


/
ly = 2r + 7
4 R= (a cos i)i + (a sin /)j, where a is a positive constant

5 R= 3( 1 + cos 7r/)i + 5(1 + sin 77f)j


11 R= — i + (r + l)j
750 CHARIER 12 VKCTORS IN THE PLANE

13 R= (cos 20)1 + (sin fl)j 14 R = (In ii)i + e-"i ed


27 r = -, d>0
= 1 + e sin
15 y In T. with r as the parameter
28 r =
16 y = 4.v'. with v as the parameter

29 Show that the absolute value of the curvature of a circle is a


In Problems 17 to 21 . find (a) the unit tangent vector T, (b) the unit
constant equal to the reciprocal of its radius.
normal vector N/, (c) the curvature k. and (d) the principal unit
normal vector N for each curve at the point where the parameter has 30 Explain why the curvature vector dT/ds and therefore also the
the indicated value. principal unit normal vector N always point in the direction of
concavity of the curve. (Hint: For s small. dT/ds is approxi-
17 R= 3(1 - cos i)\ + 3(1 - sin f)j when / = tt/6 mated by AT/A.v.)

18 R = (sin ni + (tan /)j when I = 57r/6 31 If we use .v as the parameter, explain the geometric significance
of the algebraic sign of the curvature k of the graph of y = f(x).
fv = In (r + 3)

19 ! ^ when / = I 32 Prove that dfi/ds = -|k|T [Hint: Begin by noticing that N=


V = ^ 1

4"
/*
'
(/</|k|) N/ where N; = (—sin a)i -I- (cos a)j.]

33 Prove that |k| = A • N/v".


20 when « =
34 Assume that R = F(/) is the vector parametric equation of a
curve with constant nonzero curvature k. Prove that the curve is
+ M- 1 1

21 R = -^ — i H r
~ j when ii = 2 a circle (or a portion of a circle) by carrying out the following
ii~ \ II
steps:

22 Using Theorem 2, derive the fomiuia for the curvature k of the (a) Show that R + (1/|k|)N is a constant vector by proving that

derivative zero (use Problem 32).


curve R = g{i)i + hd)].
its is

(b) Put R„ = R + (I/|k|)N.


23 Consider the curve C whose equation in polar coordinates is
(c) Show that R satisfies an equation of the form |R - Rn| =
r =f{0). With as n parameter, the corresponding vector para-
constant.
metric equation is R =
[f(6) cos 0\i + \J\0) sin d]}, and the
component functions g and h are given by g{d) = f(6) cos 6 35 Assume that R = F(r) is the vector parametric equation of a
and hm
=f(0) sin 0. Find formulas for (a) T, (b) N,, (c) k, curve whose curvature is equal to zero at all points. Prove that
and (d) N. the curve is a line (or a portion of a line). [Hini: First use the

24 Show that the principal normal vector for a circle always points relation I^T/rf^l = \k\ to prove that T is a constant. Then select a
toward the center of the circle.
fixed vector N t^ such that N T = 0. Choose and fix a value

of R. say Rq. Show that

In Problems 25 to 28, use the results of Problem 23 to find (a) T, (b)


N/. (c) K. and (d) IN for the curve C whose equation is given in polar |N-(R - Ro)l =
coordinates.
conclude that N (R • is a constant, and then show that

1
- cos e 26 r = I + 2 cos N-(R-Ro) = 0.]

12.7 Applications to Mechanics


Figure I
In this section we use the ideas and techniques developed in Sections 12.5 and 12.6
to study the motion of a particle in the xy plane. Briefly, the idea here is to consider

a curve that is traced out by a physical particle of mass w, rather than by a geometric
point.
Thus, suppose that a particle P of mass in moves in the xy plane and traces out a
curve C (Figure 1 ). The curve C is called the path, or the trajectory, or the orbit of
the particle P. As before, we can locate the particle at any given time t by the
position vector R = OP . and we have

R = F(0
SECTION 12.7 APPLICATIONS TO MECHA^aCS 751

where we assume that F is a twice-differentiable vector-valued function.


The equa-
tion R- F(r) is called the equation of
motion of the particle P
By the results in Section 12.5. the velocity
of the particle at any instant is the
vector

vT
J:

where v is the instantaneous speed of the particle and


T is the unit tangent vector to
the path of the particle.

EX.4MPLE 1 A
moves along the parabola ^^ = Iv from left to riaht away
particle
from the vertex speed of 5 units per second. Find the velocity
at a
vector V as the
particle moves through the point (2. 2).

SOLUTION Using y (not t) as a parameter, we can write a vector parametric


equation of the parabola as

R= ^ i + V

since x = \-/2. Here we have

^R
dv
= .vi + j and —- = Vv- + 1
dv I

Hence,

f/R

dx .vi +
T = __ j

dR V V- -I- 1

dv
is the unit tangent vector to the parabola
at the point (^-/2, v). When v = ? we
have "

V5
and V= vT = (5) -^^ = V5(2i + j)
V5 •'

In the example above, we were not given


the equation of motion of the parti-
cle—we were given only the equation of its path and the
speed of the particle at a
certain instant. Thus, we were not able
to calculate V directly as dR/dt and had
to
rely instead on the formula V = vT.

Newton's second law of motion can now be


expressed in the form

f = mA
where the vector f represents the force acting on the particle and

A- "^^
- ''^-
dr ~ dr
is the acceleration of the particle. (Here we are assuming that the mass m is constant
and we are neglecting relativistic effects.)
752 VF.CTORS IN THK PI.ANK

Tangential and Normal Components of Acceleration

By part (ii) of Theorem I in Section 12.6. we have

A = —T+
dt
IkIv-N
' '
it" k: 5^0

Geometrically, this equation says that the acceleration vector A can be resolved into
a sum of two perpendicular vectors, (dv/dt)T and |/<|\'-N, the first of which is

tangent to the trajectory and the second of which is normal to the trajectory (Fig-

Figure 2 ure 2). (If /< = 0, then the normal component vector of the acceleration is 0, and we
have A = dv/dl T.) The normal component vector |K|t"N of the acceleration is also
called the centripetal acceleration. It can be regarded as that part of the accelera-
tion A caused by the change in direction of the velocity vector. IfP moves along a
straight line, then k = 0, so that the centripetal acceleration is 0. On the other hand,
if the speed \' of the particle is constant, then dv/dl = 0, the tangential component of
the acceleration is 0, and the acceleration is entirely centripetal.

EX.\MPLE 2 A particle P is moving with a constant speed of 10 units per second


in a counterclockwise direction around the ellipse (.v~/4) -I- (y-/9) = 1. Find the
acceleration vector A at the moment when the particle passes through the vertex
(0, 3).

SOLUTION We find A by using the formula

dt ' '

Since v is constant, we have dv/dt = and A= |k|v"N. To find k. we begin by


implicitly differentiating the equation of the ellipse to obtain


2v
4
-H
2v
-^-^
9
(/\-

dx
=

so that

dx
SECTION 12.7 APPLICATIONS TO MECHANICS 753

Centripetal Force

If we combine Newton's law f = wA with the equation A= (dv/dt)T + |k|v-N, we


obtain the equation

f = w — - T + |K|/ni'-N
at

that is. the force f acting on the particle can always be resolved into a sum of two
perpendicular component vectors, a tangential component vector m (dv/dt) T of
magnitude m
and a normal component vector |K|^?n-N of magnitude
\dv/dt\

|k|otv-. The normal component vector |K|/?n-N is called the centripetal force vec-

tor, and its length |K|/?n - is called the magnitude of the centripetal force, or
sometimes simply the centripetal force.

EXAMPLK 3 A particle P is moving on a circle of radius r with center at the point

Po Find
. the tangential and the normal component vectors of the force f acting on
the particle in terms of the instantaneous speed v and its time derivative dv/dt.
Figure 3 Discuss the case in which the speed v of the particle is a constant (Figure 3).

SOLUTION By Problem 29 in Problem Set 12.6, the absolute value of the curva-
ture of the circle is given by |k| = 1/r. Thus,

f = w H- N

where the principal normal unit vector N points directly toward the center of the
circle and the unit tangent vector T points in the instantaneous direction of motion
of the particle. If the particle moves with constant speed v. then dv/dt = 0, and the
force vector f coincides with the centripetal force vector (mv-/r)N. In this case the
centripetal force is

|f|=F^N =

The negative of the centripetal force vector in the example above — that is, the
vector — (wi"/r)N — points directly away from the center of the circle and is called
the centrifugal force vector. If a stone is whirled at the end of a string, the inertia
of the stone causes it to pull away from the center of rotation, and this pull is

represented by the centrifugal force vector. The magnitude mv'/r of the centrifugal
force vector is often called the centrifugal force; it manifests itself as tension in the
string. Notice that the centrifugal force and the centripetal force are numerically
equal, but the corresponding force vectors point in opposite directions.
The formula mv-/r for centripetal (or centrifugal) force gives the force in newtons
when the mass m is in kilograms, v is in meters per second, and r is in meters.*

l EXA.MPLE 4 A 0.1-kilogram mass is whirled around in a circle. 3 times per sec-


ond, at the end of a string 0.7 meter long. Find the tension in the string.

*If m is expressed in pounds, v in feet per second, and r in feet, you must divide mv-/r by
g = 32 feet per second squared if you want the result in pounds of force.
754 CHAKIKR 12 VF.CTORS IN HIK PLANE

SOLUTION The tension in the string is equal to the magnitude of the centrifugal
force, mv^jr. Since the mass travels 3 times around the circle each second, its speed
I' is given by

V = (277-/)(3) = b-nr = (677-)(0.7) = 4.2it meters per second

Thus, the tension is given by

mv- (0. 1 ){4.2tt)-


2.52-77- newtons ~ 24.87 newtons
0.7

Kepler's Laws of Planetary Motion

One of the most interesting applications of vectors to mechanics is the derivation of


Kepler's laws of planetary motion. Johannes Kepler (1571-1630). after learning
the Copemican theory of planetary motion at the University of Tubingen, was in-

vited to join the staff of the Danish astronomer Tycho Brahe (1546-1601) at his
observatory near Prague. When Brahe died, Kepler was appointed director of the
observatory and thus inherited Brahe's highly accurate data on the motions of the
planets. From these data, Kepler was able to formulate his three laws:

1 Each planet moves in a plane, and its orbit is an ellipse with the sun as
one focus.
2 A radius vector from the sun to a planet sweeps out equal area in equal
time intervals.

3 The square of the period of revolution of a planet is proportional to the


cube of the semimajor axis of its elliptical orbit.

The simplicity and elegance of these laws strongly motivated Newton, who,
Tycho Brake in his ohsenalorx
using his newly invented calculus, was able to show that they could be derived from
his second law of mechanics and his law of universal gravitation. This derivation,*
using modem vector notation, is outlined in Problems 15 to 22.

*For a proof that each planet moves in a plane, see Problem 48 on page 806.

Problem Set 12.7

1 A particle moves along the upper branch of the hyperbola 3 A particle moves along an ellipse according to the equations

(yV4) - (x^/9) = 1 from left to right, with a constant speed of


=
l.v cos 2lTl
5 units of distance per second. Find (a) the velocity vector and
ty = 3 sin 277-/
(b) the acceleration vector at the instant when the particle passes
through the vertex (0, 2). Find the speed v and the rate of change of the speed ilv/di of the
particle at time
2 A particle moves along the parabola y = .r" from left to right. As I.

it passes through the point (2. 4), i' has the value 3 units per 4 A particle moves along a curve in the vv plane according to the
second and clv/dt has the value 7 units per second squared. Find equations
(a) the velocity vector and (b) the acceleration vector at this f.v = gC)
point. ly = h(t)
SECTION 12.7 APPLICATIONS TO MECHANICS 755

Derive the following formulas:

dx . dv .

dt dt dt

(c) A=
d-x
-— d"v
H--prJ
. .

(d)

dt- dr dt

dx d~x dy d'y

(e)
^

dv
dt
T=
dt dt-

(dx/dt)-
^

+
dt dt-
T
{dy/dt)-
I dx
756 CHAKIKR 12 VF.CTORS IN THE PLANE

16 In Problem 15, suppose that P is a particle of mass m. We say


that P is moving under the influence of a central force f if there
isa function g such that f = j?(r)u holds for ail values of time /.
Using Newton"s second law of motion, f = mA, show that in
this case;

(a,,(r, = 4^-.(^)"]
^
(b) 2
^
— -— + —T-
dr
dt
de
dl
r
d-0
dl-

(c) — dd\ =
d
dl V^
I
I
^
r"
dl
I

'
[Hint: Multiply both sides o( the equation

in part (b) by r.\

de
(d) r- = K. K a constant
dl

17 In Problem 16. let ,4 denote the area swept out by the radius
vector R during time l. starting from some arbitrary but fixed
value of /. Show that for motion under the influence of a central
force, the radius vector sweeps out eqiicd area in equal time
intervals. This result generalizes Kepler's second law of plane-
tary motion. [Hint: Use the result in part (d) of Problem 16 and
the fact that tlA = ir' dO from Section 9.3.)

18 In Problem 16. let q = \/r, and assume that the particle P is

moving under the influence of a central force f = ^R(r)u. Show


that

(a)
de
—-
dl
^ .
= Kq- u
(b) —
dr
dq
- = -

(c)

(d)
REVIEW PROBLEM SET 757

24 Explain why the speed of a planet in its orbit is maximum when El 26 Using Problems 23 to 25. find the maximum speed of the earth
the planet is closest to the sun. in its orbit.

E125 The eccentricity of the earth's orbit is e = 1.673 x 10"-. Find 1327 The period of revolution of Venus in its elliptical orbit about the
the semiminor axis b of the earth's orbit. (See Problem 23 for the sun is 1.941 x 10^ seconds. Using the information in Prob-
semimajor axis.) lem 23, find the semimajor axis of the orbit of Venus.

Review Problem Set, Chapter 12


1 If A, B, C, and D are points in the xy plane such that 10 Let A= 2i -I-
j and B = i 4- 3j be position vectors in the .r>'

OS -OA =0S - OD, show that ABCD is a parallelogram. plane. Draw the vectors A, B, and C = jA -t- fB on the same
diagram for (a) s = t = i and (b) .s = - and 1 / = 2.
2 Let A and B be position vectors of points P and Q. respectively.
in the plane. Find the position vector of the point four- fifths of 11 Find the midpoint of the line segment containing the terminal
the way from P lo Q. points of the position vectors A= -i -H 3j and B = 2i -(-
7j.

3 If OABC is a parallelogram in the .vy plane with A and C as 12 Find the coordinates of the point that is seven-tenths of the way
opposite vertices, show that from P = (-5, -9) lo Q= (7, 7).

OA + ^(OC - OA) = iOB 13 Let A = 2i - j and B = 2i - 3j. Find each of the following
expressions:
4 Explain the geometric significance of the condition that
(a) 5A (b) -4B
A -H B -I- C = 0, where A. B. and C are vectors in the plane.
(c) A -H B (d) A-B
5 If M and v are scalars such that »A = vA, where A is a nonzero (e) 2A 3B -I- (f) A-B
vector, is it true that ii = vl Why? (g) (2A-3B)-(2A-H 3B) (h) |A|

(i) |A-B| (j) |2Al-h|3B|


6 Sho w that if u = ai + bj, u 7^ 0. then a/Va- + b- = cos 6 and (k) compe A
bjV a- + b- = sin 0, where is the angle between u and the
positive .V axis (Figure 1.) 14 Let A = i -I- 2j. B = 2i - 4j. and C= 3i - 5j. Find scalars s
and / such that C = 5A -1-
rB.

15 Use dot products to find the three vertex angles of the triangle

Figure 1
ABC where -4 = (-2, -1), B = (-1, 6), and C= (2, 2).

16 Find two vectors X and Y in the plane such that both X and Y are
perpendicular to the vector A = 2i - 3j and |X| = |Y| = |A|.

17 Find the scalar components with respect to the standard i. j basis


of the vector obtained by rotating the vector A = .vi -t-
yj coun-
terclockwi.se through 90°.

18 Find the scalar components with respect to the standard i, j basis


of the vector obtained by rotating the vector A = .vi + yj coun-
terclockwise through 8 radians.

19 Given that |A + B|- |A - Bp = 0, show that A must be per-


pendicular to B.
7 If J is a nonzero scalar and if A and B are vectors such that
jA = sB, is it true that A = B? Why? 20 Let A and B be two given vectors in the plane such that
A B = 0. |A| ?^ 0, and
• |B| # 0. Show that if C is any vector in
8 If i, j is the standard basis in the .xt plane, write A as a linear
the plane, then
combination of i and j and find |A|, where A is the vector whose
tail end is (2, -1) and whose head end is (-1. -2).

9 Let A = 2i - 3j and B = 4i -I- j be position vectors in the .xt

plane. Draw the vectors A, B, and C = A -(-


rB on the same 21 Find the scalar component compeA of A in the direction of B if

diagram for (a) / = 5 and (b) t = -2. A= 2i -I- 3j and B = -3i + 4j.
758 CHAPIER 12 VECTORS IN THE PLANE

22 A manufacturer
h dollars per chair. The vector
sells v sofas at a
D=
dollars per sofa and v chairs at
(.v, y) is called the demand
39 —
dl
|F(/) + G(i)\ 40 --IF(0-G(/))
dt

vector, while the vector R= {a. b) is called the revenue vector.


The cost vector is defined to be C = {c, d). where c is the cost of 41 —
dt
|F(f)| 42 —dl
l/i(r)F(OI
manufacturing one sofa and d is the cost of manufacturing one
chair. Give the economic interpretation of

D-R (b) D-C (c) D-(R - C)


43 F\t)'F'U) 44 ^[-^1
LlF(/)|J
dt
(a)

23 Draw two vectors A and B in the plane such that


R=
45 Find the arc length of the curve whose vector equation is

(a)A'B>0 (b)A-B = (c)A-B<0 3(sin / - 1 )i -I- 3(cos / - 1 )j between the point where t = and
the point where r = 2.
24 Sketch the graph of the given vector nonparametric equation,
A = - 3j
where i and B = 2i + j.
46 Find the arc length of the curve

(a)A-(R-B) = (b) (R-B)MR-B) = 9


=
fx 4 COS'' t

(c) A-R = 8 (d) R-(R- 2B) = =


ly 4 sin' /

25 Find the equation of the line that contains the point (3. 4) and has = and the point where =
between the point where t t 2Tr.
slope -f (a) in scalar nonparametric form, (b) in vector nonpar-
ametric form, (c) in vector parametric form, and (d) in scalar 47 The position vector of a particle at time / is given by
parametric form.

26 Rewrite equations (a) through (d) in Problem 24 in vector para-

metric form.
Find the distance traveled by the particle during the interval from
27 Show that the following two vector parametric equations have r = 1 to / = 4.

the same graph:


48 A ball is moving in accordance with the equations
(a) R = (3 sin t)i + (3 cos f)j
= 32f
(b) R= (3 cos 2f)i -I- (3 sin 2t)}
[.V

..v= -16r
28 Let ¥(t) = Vr - li -(-
In (2 - /)j. (a) What is the domain of F?
(b) Find lim F(f). (c) Is F continuous? Why? where / is the time in seconds. How far does the ball move along
its path during the first 3 seconds?

29 Find a vector parametric equation for the hyperbola .v" - y=


1. {Hint: Use hyperbolic functions.) In Problems 49 to 52. find (a) the unit tangent vector T. (b) the

curvature k of the given curve at the given value of i. and (c) the

F'(0 and principal unit normal vector N.


In Problems 30 to 34, find (a) (b) F"(f).

30 F(f) = (/- + 7)i -I- (/ -I- r')j


49 R(n = (3 cos ()i + (sin Oj at ? =

31 F(/) = (f- \)~'i + 3(r -2)''j 50 R(;) = (tan 20i + (cot 2r)j at i = ir/i

32 F(() = (2 cos 5r)i + (5 sin 5f)j ~ ', =


51 I
ly = r
at r 1

33 F(0 = e''i + (e"" + 7)j

34 = 52 r '[ at = 7r/4
F(f) (cos /-)i -I- (cos- Oj /
l.y = In sec /

In Problems 35 to 38. find (a) d\¥{t)\/di and (b) F'(/)-F"(/).


In Problems 53 to 57. find (a) the velocity vector V. (b) the acceler-

ation vector A. (c) the speed v. (d) the rate of change dv/di of the
35 F(r) = U - sin /)i -t- (/ -i- cos Oj
(e) the tangential component vector of the acceleration, and
speed,

36 F(/) = (e~' cos Oi + (e~' sin r)j (f ) the normal component vector of the acceleration for a particle at
time t moving according to the given equation of motion.
37 F(f) = e*'i + e-"j 38 F(f) = ff""! + te^'j

53 Rit) = (1^ + 6)i + at^ - l)j

In Problems 39 to 44, suppose that F(r) = e'i + (r - I)j,


=
54 R(0 e-'i + e"-"j
G(n = (cos- f)i -1-
(sin- /)j, and h{i) = In (r + 1). Evaluate each
expression. 55 R(r) = (3 cos 7/)i -i- (3 sin 7/)j
REVIEW PROBLEM SET 759

56 R(/) = (t cos f)i + (I sin t)j position vector R as a function of the time t. show that

dA_^ \ gU)h'{t) - h{t)g'U)


57 R(o = (i +y)' + yJ
\

di \ 2 I

58 A particle is moving along the branch of the hyperbola .ri' = 1 El 61 The period of Neptune is 164.8 years. Show that it is about 30
that lies in the first quadrant in such a way that its abscissa is times as far from the sun as the earth is.

increasing in time. At the instant when the particle passes


through the point speed given by = 62 Suppose that a particle P. moving under the influence of a cen-
(1, 1). its is \' 2 units per
second, and the time rate of change of the speed is given by
tral force, has a spiral trajectory of the form r = b^", where b is

dv/dl = -3 units per second squared. Find (a) the velocity vec-
a positive constant. Show that there are constants y and tq such

tor V and (b) the acceleration vector A that r = (yt + rg)"-.


at this instant.

59 A pilot is pulling out of a vertical dive by following an arc of a El 63 The period of Mars is 1.881 years. How much farther is Mars
circle of radius 1 mile. The speed of the plane is a constant 500 from the sun than the earth is?

miles per hour. With how many times his usual weight is the
pilot being pressed into the seat at the lowest point on the arc of
64 The angular momentum Z, of a particle P of mass m moving in

the plane according to an equation of motion r = /(fl) is defined


the circle?
by Z, = mr~ dO/dt. Prove that Z, is a constant. (This is the two-
60 Let R= g(t)i + h{t)j be the equation of motion of a particle P dimensional version of the law of consen'cition of angular mo-
in the xy plane. If A(f ) denotes the area swept out by the variable mentum.)
COORDINATE SYSTEMS IN
THREE-DIMENSIONAL SPACE

In this chapter we study three-dimensional geometry with the aid of coordinate


systems and vectors in three-dimensional space. All the operations defined for vec-
tors in the plane — sums, differences, multiplication by scalars. and dot products
carry over directly to vectors in three-dimensional space. In addition, we define the
"cross product" of vectors in space and use vectors to obtain the equations of lines
and planes in space. The chapter also includes vector-valued functions, curves in
space, surfaces of revolution, and quadric surfaces.

13.1 Cartesian Coordinates in Space


In order to assign Cartesian coordinates to a point P in three-dimensional space, we
begin by choosing an origin O as usual. We then set up a Cartesian coordinate
system in the horizontal plane passing through O. with the positive .v axis pointing
toward us and the positive y axis extending to our right (Figure 1). To find the
Cartesian coordinates of a point P in space, we drop a perpendicular from P to the
.r\' plane, and we denote the foot of this perpendicular by Q = (.v, y). We define the
z coordinate of P by : = ±\PQ\. where we use the plus sign if P is above the .vy

plane and the minus sign if P is below the .vy plane. If P lies on the .vy plane, then
P = Q and z = 0. Thus, r is the directed distance from the .vt plane to P. The point
Figure I
P is understood to have three Cartesian coordinates .v, y, and r, and we write
horizontal vi plane
P = U. y, -)

In dealing with Cartesian coordinates in three-dimensional


space, it is customary to introduce a third coordinate axis,

called the z axis, perpendicular to the .vy plane, passing


through the origin O. and directed upward (Figure 2). The z

axis is equipped with its own number scale, just as the v and y
axes are, and the unit of distance on the r axis is usually
chosen to be the same as the unit of distance on the v and y
axes.

760
SECTION 13.1 CARTESIAN COORDINATES IN SPACE 761

Figure 2
762 CHAPIER 13 COORDINATE SYSTEMS IN THKEE-DIMENSIONAI. SPACE

Figure 7

Figure 8

Figure 9
SECTION 13.1 CARTESIAN COORDINATES IN SPACE 763

Figure 10 EXAMPLE 4 Find an equation of the plane containing the point (a. b, c) and
parallel to the vz plane.

SOLUTION On the plane in question, all points have the same x coordinate
X = a. Conversely, any point P of the form P= (a, y, z) belongs to the plane.
Therefore, an equation of the plane is x = a.

We now derive a formula for the distance r bciween the point P = {x, y, z) and
the origin O in xyz space. In Figure 10, let r^ denote the distance in the xy plane
between the point Q= (x, y, 0) and the origin O. By the usual formula for the
distance between two points in the xy plane, rf = x^ +j;^- Applying the Pythago-
rean theorem to the triangle OQP, we have r~ = r] + \PQ\' = .v" + y" + z-; hence.

Q = (x,y, 0)
\ .V- + \- + :-

EXAMPLE 5 Find the distance r between the origin and the point (2, 3, — 1).
Figure 11
SOLUTION From the formula above, we have
r = V2^ + 3- + (-1)- = V\4 units

EXAMPLE 6 Describe and sketch the graph in .vvr space of the equation
X- + y^ + z- = 4.
S OLUTION The point P = (x, y, z) belongs to the graph if and only if

Vp + y^ + z" = 2, that is, if and only if the distance from O to P is 2 units.


Therefore, the graph is the surface of a sphere of radius 2 units with center at the
origin O (Figure 11).

Problem Set 13.1

1 The points (3, 5, 7), (3, 0, 7), (0, 0, 7), (0, 5, 7), (3, 5, 0), (a) P = (1. 2, 4) (b) (3, 5, 0) (c) (2 1. -3)
(3, 0, 0), (0, 0, 0), and (0, 5, 0), form the vertices of arectangu-
6 Two points A and B in space are said to be symmetrically lo-
lar box in xyz space. Sketch this box and label the vertices.
cated with respect to a line that bisects the line segment AB and
2 A perpendicular is dropped from the point P = - ( 1 , 2, 5) to the is perpendicular to it. If A = (.v, y, z). find the coordinates of the
yz plane. Find the coordinates of the point Q at the foot of this point B that is symmetrically located to A with respect to (a) the
perpendicular. -V axis, (b) the y axis, and (c) the - axis.

3 Plot each point in xyz space: 7 A cube each side of which is 6 units long has its center at the

(a) (2, -1, 3) (b) (-2. 5, 3) (c) (2, -1, -3) origin, and its faces are perpendicular to the coordinate axes that
intersect them. Sketch the cube, and find the coordinates of the
(d) (1,0.-1) (e) (4, 3, -2) (f) (-1, -2, -1)
eight vertices.
4 Two points A and B in space are said to be symmetrically lo-
cated with respect to the plane that bisects the line segment AB 8 Two points A and B in space are said to be symmetrically lo-

and is perpendicular to it. If A = (x, y, ;), find the coordinates of cated with respect to the midpoint M of the line segment AB.
the point B that is symmetrically located to A with respect to \{ A = (x, y, :), find the coordinates of the point B that is sym-
(a) the xy plane, (b) the xz plane, and (c) the yz plane. metrically located to A with respect to the origin.

5 In each of the following cases, find the coordinates of the point T 9 A perpendicular is dropped from the point P = (3, 5, 7) to the

that lies on the line through P parallel to the y axis but is 2 units plane y = — I . Find the coordinates of the point Q at the foot of
farther to the right than P. this perpendicular.
764 CIIAriKR 13 t OORDINATK SYSTEMS IN THREE-DIMENSIONAL SPACE

10 If P = (3, 4, 5), find the awrdinales of the indicated points 22 The plane perpendicular to the y axis and intersecting it at the
(Figure 12). point (0, -5, 0)

(a) The point Q symmetric to P with respect to the .rv plane 23 The .V axis
(b) The point R symmetric to P with respect to the v axis
24 The line parallel to the : axis and containing the point
(c) The point S symmetric to P with respect to the origin (-2. -7. -15)
(d) The point T that is 5 units directly below P
25 The line parallel to the v axis and containing the point
(See Problems 4. 6. and 8.)
(5,^, -I)

Figure 12 26 The line containing the two points (5, 0, 7) and (5, -I, 7)

27 The line perpendicular to the v; plane and containing the point


(.5. 4. 7)

28 The plane containing the : axis and making a 45° angle with the
x: plane and the yr plane

In Problems 29 to 36. sketch a graph in xyz space of each set of


simultaneous equations.

.v= -2
29 {^Zo"' 30 31
{;:^ { v=
-5

f.v = -3 f.r + v' + z^ = I f.v


32 33|,_, 34
{y = {^,
In Problems 11 to 18, describe and sketch the set of points
P = (.V, y, z) that satisfy the given conditions.
35 J y = 2 36 + y' + 2' = 1

Y'
11 y = 5 12 .V = y 13 .V =

14 X > 15 ;t = -3 16 z<2
37 Write an equation of the sphere of radius 5 units with center at

17 V = -4 18 .V > 0. V < 0. and : > the origin in .vvz space.

38 Write an equation of the sphere with center at the origin and


In Problems 19 to 28. give an equation or a pair of simultaneous containing the point (1. 2, -1).
equations for the plane or line.

In Problems 39 to 42, find the distance r between the origin and the
19 The horizontal plane 3 units below the origin
point P.
20 The line parallel to the x axis and containing the point (3, 4. 5)

39 P = (-2. 1,2) 40 P = (8, 0, -6)


21 The plane parallel to the xz plane and containing the point
(-1, -2, 3) 41 P = (-4, -3, 0) 42 /> = (1, 4. 5)

13.2 Vectors in Three-Dimensional Space

The concept of a vector and the algebra of vectors in two-dimensional space carry
over almost without change to vectors in three-dimensional space. Thus, a vector in
space is defined to be a directed line segment — that is, an arrow — and the directed
line segment from P to Q is denoted by PQ Two
. vectors in space are regarded as
being equal if they have the same length, are parallel, and point in the same direc-
tion. The definitions of sums and differences of vectors, the negative of a vector,
the product of a scalar and a vector, and the dot product of two vectors ail carry over
verbatim to vectors in space.
SECTION 13.2 VECTORS IN THREE-DIMENSIONAL SPACE 765

The basic algebraic properties of vectors in space are exactly the same as the
corresponding properties of vectors in the plane. We encourage you to sketch dia-
grams illustrating the following.

Figure 1 Basic Algebraic Properties of Vectors in Space


Let A, B. and C be vectors in three-dimensional space, and let i- and / be scalars.
Then

1
766 CHAPTER 13 COORDINATE SYSTEMS IN THREE-DIMENSIONAL SPACE

The sum A+ B, the difference A - B, and the product s\ are calculated "com-
ponent by component"" for vectors A and B in vvr space just as they are for vectors
in the plane. Thus, if

A= uii + «2J + u^k and B = bit + bzj + bjk


then

1
SECTION 13.2 \ECTORS IN THREE-DLMENSIONAL SPACE 767

Basic Properties of the Dot Product in Space

If A, B. and C are vectors in three-dimensional space and i is a scalar, then

1 A B = B A
• •
768 CHAFIKR 13 C(M)RDINATE SYSTEMS IN THREE-DIMENSIONAL SPACE

(c) |A - B| = |i - 7j + 3k| = Vl- + (-7)- + 3- = VSQ


A-B -6 -6
(d) compeA
B VZ- + 5- + (-If V33

Suppose that 6 is the angle between the vectors A and B in ihree-diniensional


space. Then, since A -B = |A| |B| cos 6, we have

A-B
CDS =
lAl IBI

This formula enables us to find the cosine of 6. and hence the angle 0, in terms of
the dot product.

B EXAMPLE 3 Find the angle 6 between A = 2i - 3j + k and B = i + 2j + 5k.

=
A-B (2)(1) + (-3)(2) + (l)(5) 1

SOLUTION COS d
lAl iBl V22 + (-3)2 + l-Vl^ + 2- + 5- 2V105

Hence, COS"' 0.0488 == 87.2°


2V105

Distance Between Points in Space


Figure 6 We can now use vectors to derive a fomiula for the distance between two points
P = (.V, V, z) and Q= (a, h. c) in .vat space (Figure 6). Here.

OP = xi+ yj + zk OQ = ai + bi + ck and PQ = OQ - OP
It follows that

PQ = (a - x)i + {b- .v)j + {c :)k

Thus, the scalar components of the vector PQ in xyz space are the differences of the
coordinates of Q and the corresponding coordinates of P. Therefore, the distance
between P and Q is given by

\PQ\ '=\PQ\ = \ (« - -V)- + (b - V)- + ((' - :)-

EXAMPLE 4 UP= (2. 3. -2) and 2= (-'• 1- ^t- tinti (a) the scalar compo-
nents of PQ and (b) the distance between P and Q.
SOLUTION

(a) PQ = ((-1) - 2]i + [1 - 31j + [5 - (-2)lk = -3i - 2j + 7k


(b) Using the distance formula above, we have

\PQ\ = V[(-l) - 2)- + [1 - 3]- + |5 - (-2)]- = V62 units

Direction Cosines of a Vector

Consider a nonzero vector A in .\y: space. Move A, if necessary, so that its tail end
is the origin O. and let a. /3. and y be the angles between A and the positive .v, y.
SECTION 13.2 VECTORS IN THREE-DIMENSIONAL SPACE 769

Figure 7
and z axes, respectively (Figure 7). The angles a, j8, and y, which are the same as
the angles between A and the standard i, j. and k basis vectors, respectively, are

called the direction angles of the vector A. The cosines of the three direction angles
are called the direction cosines of the vector A and are given by the formulas

A-i
cos )3
A-j
and cos y
Ak
lAl

(Problem 54).
Now, suppose that A = ai + bj + ck, so that

A'i = a X'i = b A-k = c

and lAl = Vcr + ir + P


Then,

cos
Va- + b- + c-
)3
V^T^T?
cos y
Va- + b~ + c-

Notice that

(cos a)i + (cos /8)j + (cos •y)k , ,


1

., ==-(ai +
. ,.
bj + ck) = - —
A
Va' + b- + c- A
Hence, the direction cosines of a nonzero vector A are the scalar components
of the
unit vector A/|A| in the same direction as A. Since (cos a)i + (cos
)8)j + (cos y)k
is a unit vector, it follows that

cos- a + cos- /3 + cos- y

that is, tite sum of the squares of the direction cosines of a nonzero vector is always
equal to I (Problem 59).

EXAMPLE 5 LetP = (-l,2,3)ande = (-3, 3, 5). Find the direction cosines of


the vector A = PQ, and verify that the sum of the squares of these direction cosines
is equal to 1.

SOLUTION A = Pe = (-3 + l)i + (3 - 2)j + (5 - 3)k = -2i -^ j -^ 2k


-2 _2_
Hence,
V(-2)^ +12 + 2-
~
3

1 1
cos jS =
V'(-2)- + \- + 2- 3

cos y =
V(-2)-+ 1^ + 2- 3

and we have

cos- a + cos- /3 + cos- y = {- f)^ + (^)2 + (|)2 = 4^1 + 1^,


770 CHAPTER 13 COORDINATE SYSTEMS IN THREE-DIMENSIONAL SPACE

Problem Set 13.2

In Problems 1 lo 46, suppose A = 3i+j-4k, B = 4i+j-k. 48 If D ?^ 0, show that conipi)(A + B) = compoA + compoB
C = 2i - 5j + 4k, D= (
- 2, 1. 7), and E = (2, 1, 1) arc vectors holds for all vectors A, B, and D.
in space. Evaluate each of the following.
49 Find the distance between the points P^ and P^ if

(a) P, = (7, -1, AYP. = (8, 1, 6)


1 A + B 2 3A + 2B
(b) P, = (I, I, 5);P2 = (I, -2, 3)
3 3C - 2A 4 D- E = -3); P. = -2)
(c) />, (2, 3. (2. I,

5 E- 2D 6 A - B + 2C
50 Under what conditions is it true that |A + B| = |A| + |B|?

7 ^(A + B - 6C) 8 -3A - 4(B + C)

9 2A - (B - C) 10 iD - §E In Problems 51 to 53, find the direction cosines of the vector


A = QP OAlso find the direction angles of A.
AB
.

11 12 D-E
B(2A - B + C) 51 P = (6. - -2) and Q = (4, 9, 9)
13 A-(B + C) 14 1,

15 A(-B) + A-C 16 (AB)C - (AC)B 52 P = (3, 8, I) and Q= (I, 2, 10)

17 (D + E)-(D - E) 18 (2A + B)-(C - A) 53 P = (9, 1, -7) and e= (I, 0, -I)

19 (E-D)E-(E-E)D 20 2|A| + 2|C| 54 Verify the formulas given on page 769 for the direction cosines

- of a vector.
21|D| 22 I
2B| + 2|C|

23 |A + C| 24 |B|A - |A|B
In Problems 55 to 58, determine whether each triple a, ;S, y could
25 |A| + Id 26 |D + E| - |D|
-
lEl possibly be direction angles of a vector A.

55 a = 90°, )3= 135°. 7 = 45°


27 B/|B|
56 a = 27r/3, P = 37r/4, y = 7t/3
29 conipnA 30 cohipaB
57 a = 7r/3, /3 = 7r/6. y = 7r/4
31 compnC 32 compoE
58 a = 120°, p = 45°, y = 60°
33 conipB(A + C) 34 compB(A — C)

35 comp[.;D 36 conip^A 59 Prove that the sum of the squares of the direction cosines of a
nonzero vector is I
37 The angle 0[ between A and B
60 If a = 7r/3 and /3 = 7r/3 are two direction angles of a vector A,
38 The angle fli between A and B - A
find the possible values of the third direction angle y.
39 The angle 63 between A- B and A + B
61 Show that a nonzero vector A in space is completely determined
40 The angle O4 between D and E by its length / and its three direction cosines; cos a. cos /3, and
cos y. |////i/.Showthat A= (/ cos a)i + (/ cos j8)j + {/ cos -)')k.l
41 The direction cosines of A
62 Show that A= 7i + 2j - 10k, B = 4i - 6j + 5k, and C =
42 The direction cosines of E - - 3k
3i 2j are position vectors of the vertices of an isosceles
43 The direction cosines of A + B triangle.

[£l44 The direction angles of E 63 Let |A| = 2, |B| = 6, and suppose that the angle between A and
B is 30°. Find (a) |A + B| and (b) |A - B|.
45 A scalar a such that A is perpendicular to F = ai + j - 5k
64 Give a geometric interpretation of the "average" (A + B)/2 of
46 A .scalar b such that D is perpendicular to G = (I, h. 4>.
two vectors A and B.
47 Use vectors to show that P = (1, 7, 2), (2 = (0, 7, -2), and
65 Let A = Pe where P = (-l,2^3)and^ = (11,11,3). Find
W = (- 6, are vertices of a right triangle. At which vertex is
= such that PS = 2A.
1 , I )
a point S (,v, v, z)
the right angle? {Hini: Show that two of the three vectors PQ
QR . and RP arc perpendicular.) 66 If A is a vector and K is a vector with lEl = I, show that E is
SECTION 13.3 CROSS AND BOX PRODUCTS OF VECTORS IN SPACE 771

perpendicular to A- (compEA)E. Sketch a figure showing A. 70 Show that three vectors are coplanar if and only if one of them
E, (compEA)E, and A - (compEA)E. can be expressed as a linear combination of the other two.

67 If A is a vector in space, show that


71 Are the vectors A= 2i - 3j + 5k. B = 3i + - 2k. and
j

A = (A-i)i + (A-j)j + (A-k)k C= — i 7j + 12k linearly independent? Explain.

(Hini: Begin by writing A as A= .vi + vj + rk. Then take 72 Prove Theorem 1

the dot product on both sides of the equation with i. with j. and
with k.) 73 Suppose that A. B. and C are linearly independent vectors and
that.vA + vB + rC = oA + ^B + cC. Prove that.v = a.y = b.
68 Show that D = (B-B)A - (A-B)B is perpendicular to B.
and r = c.

69 Are the vectors A = (1. 1. 0>. B = (2. -1. 0). and C =


(1, 1, 1) coplanar? Explain. 74 Prove Theorem 2.

13.3 Cross and Box Products of Vectors in Space


In this section we define and study a product that is available for vectors in three-
dimensional space but not for vectors in the plane. This product is called the cross
product (outer product or vector product). By combining the dot and cross prod-
ucts, we obtain the box product (or determitwnt) of three vectors in space.
The cross product of two vectors in space is defined to be a vector with a certain
magnitude and direction. In order to specify the direction of the cross product of two
vectors, we need the following concept: Three linearly independent vectors A. B,
and C are said to form a right-handed triple in the order A, B, C if the thumb,
index finger, and middle finger of the right hand (held in a ""natural"" position) can
be aligned simultaneously with A, B, and C, respectively (Figure 1). For instance,
the standard basis vectors in the order i, j, k form a right-handed triple, but in the
order j. i. k they do not. (Why?)
We can now give the definition of the cross product.

DEFINITION 1 Cross Product of Vectors

Let A and B be two nonparallel vectors in three-dimensional space. Then the


cross product of A and B. denoted A x B. is the vector whose length is numeri-
cally equal to the area of the parallelogram spanned by A and B and whose
Figure I direction is perpendicular to both A and B in such a way that A. B, A x B
is a right-handed triple (Figure 2). If A and B are parallel vectors, we define
A X B = 0. In particular, then, A X A = 0.

Figure 2
772 CHAPTER IJ COORDINATE SYSTEMS IN THREE-DIMENSIONAL SPACE

Figure 3 The condition that A, B, A X B is a right-handed triple can be rephrased in


various equivalent ways. For example, a right-handed screw turned through the
smaller angle from A to B advances in the direction of A x B. Equivalently, if the
parallelogram spanned by A and B is viewed from the tip of A x B. then the angle
6 from A to B is generated by a coiinierclockwise rotation (Figure 3). (Also, see
Problem 38.)
Notice that the direction of the cross product is reversed when the two factors are
interchanged; that is.

ounlcrclockwise a; B X A = -(A X B)
viewed fmm the
tip of A X B
Thus, the cross product is not a commutative operation. Consequently, in calculat-

ing with cross products, you must be very careful about the order in which the
vectors are multiplied.

Figure 4 EXAMPLE 1 Find


M i X j
= k
(a) i X j (b) j X i

SOLUTION

(a) Because the two perpendicular unit vectors i and j span a square of unit
area = area, it follows that i X j is a unit vector perpendicular to both i and j. Since i,
square
I
j, i X j is a right-handed triple, we conclude that i x j = k (Figure 4).
unit

(b) Using the result in part (a), we find that j X i = — (i x j) = — k.

Proceeding as in the example above, we can find the cross products of all pairs of
standard basis vectors:

Figure 5
SECTION 13.3 CROSS AND BOX PRODLCTS OF VECTORS IN SPACE 773

We leave it to you tomake the simple geometric argument showing that the last
equation holds even when s is not positive (Problem 43). Similar considerations
(Problem 44) show that

A X (rB) = ;(A X B)
774 CHAPTER 13 COORDINATE SYSTEMS IN THREE-DIMENSIONAL SPACE

Since the dot product (5 commutative, we have (B x C) • A =A • (B x C), so the


last equation can be rewritten as

AmB X C) = (A X B)-C

Thus, /// the box product, the dot and cross can be interchanged, provided that the
three vectors are kept in the same order.

The Distributive Law and the Cross-Product Formula


Although the preceding discussion describes the cross product A x B geometrically
and relates it to the volume of a parallelepiped, it does not show how A x B can be
found directly from A and B when A and B are given in component form. A

"cross-product formula" that gives the components of A x B in terms of the com-


ponents of A and of B can be derived on the basis of the following theorem.

THEOREM I Distributive Law for the Cross Product

For any three vectors A, B, and C in three-dimensional space,

A X (B -t- C) = (A X B) -I- (A X C)

Let
D= A X (B + C) - (A X B) - (A X C)

and note that it is sufficient to prove D= 0. We do this by showing that

D D=
• |D|- = 0. We have

D-D = D-|A X (B + C) - (A X B) - (A X O]
= D-[A X (B + O] - DMA X B) - DMA X C)

Interchanging the dot and cross in the three box products, we obtain

D-D = (D X A)-(B + C) - (D X A)-B - (D X A)-C


= (DxA)-B + (DxA)-C-(DxA)-B-(DxA)-C =
and the proof is complete.

Naturally, the distributive law for the cross product extends to three or more
summands; for instance.

A X (B + C -h D) = [A X (B -H O] 4- (A X D)

= (A X B) -I- (A X C) -(-
(A X D)

Also, the distributive law works "from the right"; that is,

(A -I- B) X C = -[C X (A -I- B)] = -[(C X A) -I- (C X B)]

= -(C X A) - (C X B) = (A X C) -I- (B X C)

THEOREM 2 The Cross-Product Formula

If A = (7|i + cu'i + «<k and B = fc|i -(-


/jJ + /j.^k, then

A X B = («2/'i - «A/j;)i - (a^h^ - ii^hf )j + (a,/;^ - «;/'i )k


SECTION li.i CROSS AND BOX PRODLCTS OF VECTORS IN SPACE 775

PROOF Using the distributive law for the cross product, we have
A X B =A X (fc,i + b2J + bik)

= [A X (ft,i)] + [A X ib.j)] + [A X (byk)]

= fe,(A X i) + fo-,(A X j) + /7,(A X k)

Here,

A X i = (a,! + a-,j + fl,k) x i

= [(a|i) x i] + [(rtj) X i] + [(fl3k) X i]

= ai(i X i) + a.(j x i) + «3(k x i)

= a,0 + a2(-k) + «3J = fl,j - fl-,k

Similar calculations yield

A X j = a,k - rt^i and A x k = 02! " aj


Therefore, substituting into the original equation, we have
A X B = biia^i - «2k) + /?2(aik - Wji) + /^jla.i - wj)
= feiQ^j - fciOik + Z?2aik - /72«?i + ^3^21 ~ ^3«ij

= (^2^3 - 03^2)' - (01^3 - «3^i )j + (ai^2 - 02^1 )k

The formula in Theorem 2 is not easily recalled as it stands: however, by using


deierminants. it can be rewritten in a much simpler fonn. A determinant of order
2 is defined by

ad - he

and a determinant of order 3 is defined by

In what follows, we often find it convenient to use determinant notation in which


the first row contains vectors rather than scalars, so that, for instance.

J k
1 j+ \u
- k
I" ii'l vi

- (yw - r\)i - (All- - :ii)j + (.vr - yti)k

Using determinant notation, we can now rewrite the cross-product formula in


Theorem 2 in the following easily remembered form. If A= a,i + a,j + a,k and
B = ^,i + b.J + b^k, then

A X B =
776 CHARIER 13 COORDINATE SYSTEMS IN THREE-DIMENSIONAL SPACE

EXAMPLE 2 Evaluate (a) A x B and (b) B x A if A = 2i + 3j - 4k and


B = 5i - 2j - k.
SOLUTION

J k
_. -1 3 -4|,_|22
-41. -4L
-4|,^ 12 31.
(a) A X B = 2 3 -4
-2 -l!'-|5 i|j+L Jk
l5 -2l
Is -2 -1
= l(3)(-l) 4)(-2)]i - [(2)(-l) - (-4)(5)]j
+ [(2)(-2) - (3)(5)]k
= -Hi - 18j - 19k

(b) Since B x A = -(A x B), we have

B X A = -(-Hi - 18j - 19k) = Hi + 18j + 19k

By combining the cross-product formula with the formula for calculating dot
we obtain, in the following theorem, a formula for the box product. The
products,
proof is left as an exercise (Problem 42).

THEOREM 3 The Box-Product Formula


If A =
SECTION 13.3 CROSS AND BOX PRODUCTS OF \T:CT0RS IN SPACE 777

THEOREM 4 Triple Vector Product Identities

For any three vectors A, B. and C in three-dimensional space,


(i) A X = (A-C)B - (A-B)C
(B X C)
(ii) (A X B) X C = (A-C)B - (B-C)A

One immediate — and important — consequence of the triple vector product iden-
tities is that the associative law does not hold for cross products: that is, in general,

(A X B) X Ct^ A X (B X C)

EXAMPLE 5 If A = 7i - j -I- 2k. B = -3i -H 2j - k, and C = 5j - 3k. find


(A X B) X C and A X (B X C).

SOLUTION
(A X B) X C = (A-C)B - (B-C)A = |-11)B - 13A = -58i - 9j - 15k

A X (B X C) = (A-C)B - (A-B)C = (-ll)B + 25C = 33i + 103j - 64k

Using the triple vector product identities and the fact that the cross and dot can be
exchanged in the box product, we can prove the following theorem.

THEOREM 5 Lagrange's Identity

For

PROOF
778 C HAITKR 13 COORDINATE SYSTEMS IN THREE-DIMENSIONAL SPACE

Geometric Applications of the Cross and Box Products


Many geometric applications ot the cross product follow directly from its definition.

EXAMPLE 7 Find the area of the parallelogram spanned by the vectors


A = i - 2j + 3k and B = -i + j + 2k.

SOLUTION The desired area is numerically equal to the length |A x B| of the


cross product A x B. Using the formula obtained above, we have
|A X B| = \/|A|-|B|- - (A'B)- = V'(14)(6) - 3" = 5\/3 square units

Naturally, the box product is useful for computing the volume V of the parallele-
piped spanned by the vectors A. B. and C: in fact.

\- = j(A X B)-C

EX.VMPLE 8 Find the volume V of the parallelepiped spanned by the vectors


A= 3i + j, B = i + 2k. and C = i + 2j + 3k.

SOLUTION Here.
3 1 0[
(AXB)-C = 'l 2=3!° l\- l|! ; +o!l ?|
, 2 3! '- ^' " 3i il 21

= 3(-4) - 1(1) + = -13


Hence, V= |— 13| = 13 cubic units.

The algebraic sign of the box product can be used to determine whether a triple of
vectors is right-handed or left-handed. In fact, A, B, C is a right-handed or left-
handed triple according to whether (A X B) • C is positive or negative, respectively
(Problem 48b). Also, (A X B)'C = if and only if the vectors A. B, and C are

coplanar — that is, linearly dependent (Problem 48a).

EXAMPLE 9 Determine whether A, B, C is a right-handed or left-handed triple if

A = -i + 7j -I- 2k, B = 2i + 3j + k, and C = -i + 5j + 2k.

SOLUTION

(AXB).C = 3 ij=(-l)|J \\-l\_] + l\=-iO<0


|_2 J| 2J_J

Hence, A, B, C is a left-handed triple.

EXAMPLE 10 Show that the three vectors A = (1. —1, 1), B = (2, 1, —1). and
C = (0, —1, 1) are coplanar (linearly dependent).

SOLUTION

2 -1 12 1

<->>!0
ll^'lo -li
SECTION 13.3 CROSS AND BOX PRODUCTS OF VECTORS IN SPACE 779

Many geometric objects can be built up from nonoverlapping triangles: hence,


the following theorem, which gives a formula for the area of a triangle in terms of
the coordinates of its vertices, can be quite useful.

THEOREM 6 Area of a Triangle in Space

Let PQR be a triangle in .vvz space, where P= (xi,)-^, ~i),Q = (.xs . V: >), and
R = (.X3, ,V3, ^3). Then the area a of PQR is given by the formula

a = ^\PQ xPR\

PROOF The area of triangle PQR is half the area of the parallelogram spanned by the vectors
PQ and PR (Figure 7).

Figure 7 EX.WlPLE 11 Find the area a of the triangle whose vertices are P = (2, 1, 5),

Q= (4. 0, 2), and R = (-1. 0, -1).


SOLUTION Here,

PQ = (4 - 2)i + (0 - l)j + (2 - 5)k = 2i - j


- 3k

and PR = (- 1
- 2)i + (0 - l)j + (-1 - 5)k = -3i - j
- 6k
Hence,
i j k
PQ X PR 2 -1 -3 3i + 21j - 5k
-3 -1 -6
Therefore,

a
1
.
= —\PQ X
, — — .
.

P/?|
.,

= — V3- +
1 r-7
(21)-
^ 7
+ (-5)- =
V475
=
^
-5 V19 square units

Problem Set 13.3

1 Determine geometrically whether each triple of vectors is right- In Problems 5 to 34. let A= +j i
-1-
k. B = -2j, C= k - 3j.
handed or left-handed. D= 31 - 2j + k. E = -3k. F = ( - 1. 1. -4). G= (0. -1. 1).

(a) i, -j, k (b) i. -j. -k (c) j, k. 1


and H = (1 - , 1 . -2). Evaluate each expression using the identities
for cross and dot products whenever possible to simplify your calcu-
(d) j, -i. -k (e) -i. -j. -k (f)i+j.j-i. k
lations.

2 Suppose that A. B. C is a left-handed triple. Determine whether


each triple is right- or left-handed. 5 A X B 6 C X B
(a) B, A, C (b) -A, B, -C 7 C X D 8 F X H
(c) C. A. B (d) C. B, A 9 E X B 10 D X A
3 Find each cross product by sketching an appropriate figure and
11 F X G 12DXC+ CXD
using only geometric facts. (Do not use the memory device of
Figure 5 or the cross-product formula.) 13FxG-GxF 14GXH-GXF
(a) i X k (b) (-i) x j
15BxD + BxE
(c) k X i (d) (i + j) X (j - i)
16CXA-EXC + CXF
(e) (-1) X (-j) (f) (i -^ j) X k
17 (-2A) X (3B) 18 (A -H E) X (A - E)
4 Using only the memory device of Figure 5, calculate
19 (A X B)-C 20 C • (D X E)
(a) (i X j) X j and (b) x i ( j x j). Conclude from your result
that the associative law does not work for the cross product. 21 F-(G X H) 22 F-(F X H)
780 CHAPTER 13 COORDINATE SYSTEMS IN THREE-DIMENSIONAL SPACE

23 (B X C)E 24 H(F X H) 48 (a) Show that A. B, and C are coplanar (linearly dependent) if

and only if (A x B)-C = 0.


25 (A X A)- 26 (A X C) MA X C)
(b) If A, B, and C are linearly independent, show that
27 A X (B X D) 28 (F X G) X H (A X B) • C is positive or negative according to whether

29 (A X B) X D 30 (A X B)-(C X I))
A, B, C forms a right- or a left-handed triple, respectively.

49 Let PQR be a triangle in the xy plane with P = (xi, Vi),


31 (A X D)-(A X C) 32 (A X B) X (B X A)
Q = (•«:• .V2). and R = (.vj. vj). Show that ihe area a of PQR is

33 lA X Dl 34 IF X HP - lF|-|Hp Hiven hy
.V, Vi
35 Find the area of the parallelogram spanned by A and B;

(a) A = + j + k, B = - j - k
i i a = absolute value of
(b) A = (2. 5. -1), B = (i ~\.h .vj 1

36 Suppose that (A x B)'C = 4. Find


50 Using Problem 49, explain why the equation
(a) A MB X C) (b) BMC x A)
X y I

(c) BMA X C) (d) CMA X B)


f\ .Vi I

37 Find the volume of the box (parallelepiped) spanned by A. B. X2 .V2 1

and C:
represents the line in the xy plane containing the two points
(a) A = i + 2j, B = 2i - j, C = i + j + k (jr, , Vi) and (.v,, >':).

(b) A= i - 2j + k. B = 3i - 4j - k. C = 4i - j + 3k
Find the area of triangle PQR in the at plane:
51
(c) A = (1, 2, -1), B = (0. I, -I), C = (3, 0, 1) (a) P = (1, -5), Q= (-3, -4). R = (6, 2)

38 If you grab the vector A x B with your right hand so that the (b) P = (-5, 7). Q= (3. 6), R = (2, I)

fingers curl in the direction of the angle fromA to B and your


52 (a) Show that the triangle in the xy plane two of whose adja-
thumb lies along the shaft of the vector A x B (pointing away
from your fist), will your thumb point in the same direction as
cent edges are formed by the vectors A= .V|i -I-
y, j and

the vector A x B or will it point in the opposite direction?


B = .vii + Vij (Figure 8) has an area given by the absolute value
of
39 Determine whether the three vectors A, B, and C are coplanar or
linearly independent. If the vectors are linearly independent, 2 I.V2 .V2I

determine whether the triple A, B, C is right- or left-handed.


(b) Give a geometric interpretation of the determinant
(a) A = + 2ji + 3k, B = 2i - j - k. C = -2i + j - k
(b) A = 2i - j -H 7k, B = j -I- 3k. C = 2i - 3j + k
(c) A = (1, I, -1), B = (I. -I, C =
l>. {
- 1, 1, 1>

Figure 8
40 Suppose that A is perpendicular to B. (a) Explain geomet-
rically why |A X B| = |A| |B|. (b) Prove algebraically that
|A X B| = |A| |B|.

41 Find the area a of triangle PQR in xy: space:

(a) P = (l, 7. 2), Q= {0, 7. -2). R = (-1. 6. 1)


53 Prove Theorem 4. (For simplicity, choose the .v axis in the direc-
(b) P = (-\.0. \).Q = iO. I. 0). /? = (I, 1, I) tion of the vector C, and choose the y axis so that the vector B
is contained in the .vv plane.)
42 Prove Theorem 3.

54 From pari (i) of Theorem 4, it follows that the three vectors B,


43 Show that (.vA) x B = 5(A x B) even if the scalar i is not posi-
tive. (Make direct use of the definition of the cross product.)
C. and A X (B X C) are always coplanar. Explain geometri-
cally why this should be so.
44 Prove that A x (/B) = /(A x B).
55 Let A = OP. B = OQ. and C = OR. Prove that the area of
45 Prove that (A - B) X C = (A X C) - (B X C). \Hinl: triangle PQR is given by 5|(A x B) (B x C) (C x A)|. -I- -t-

A - B = A + (-l)B.|
56 Use the cross product to prove the law of sines; that is. for any
46 Prove the following identity: (A - B) x (C - D) = triangle ABC with a = \BC\, b = \AC\. and c = \AB\
(A X C) - (A X D) - (B X C) + (B X D).
sin B sin C
47 True or false: (A -t- B) x (A - B) = (A x A) - (B x B)? b
SECTION 13.4 EQUATIONS OF LINES AND PLANES IN SPACE 781

57 Prove that 59 If u is a unit vector perpendicular to A, show that

(A - B) X (A + B) = 2(A X B) u X (A X u) = A
58 Prove the Jacob! identity: 60 If u is a unit vector, show that

A X (B X C) + B X (C X A) + C X (A X B) = [(A X u) X (B X u)]'u = (A X B)-u

13.4 Equations of Lines and Planes in Space


In this section we make use of the vector algebra developed in Sections 13.2 and
13.3 to derive vector equations for lines and planes in space. By considering the
scalar components of the vectors in these equations, we also obtain scalar Cartesian
equations for lines and planes in space.
Figure 1 As before, R = OP denotes the variable position vector of the point P, and the
graph of an equation involving the vector R is the set of all points P (now in
three-dimensional space) whose position vector R satisfies the equation.

Planes in Space
One of the simplest ways to specify a particular plane in space is to give a point Pq
contained in the N perpendicular to the plane.
plane and to give a nonzero vector
Figure shows a portion of such a plane. The vector N is called a normal vector to
1

the plane. Two planes with the same normal vector N are parallel to each other;
hence, all the planes that have the same normal vector N form a "stack" of mutu-
Figure 2 ally parallel planes (Figure 2). Specification of the point Fq chooses from this stack
the one and only plane that contains Po-
ll is geometrically clear (Figure 3) that a point P belongs to the plane containing
the point P^, and having the normal vector N if and only if the vector PqP is

perpendicular to N; that is, (PqP ) • N= 0. If R is the position vector of P and R,, is

the position vector of Pq, then PnP = R- Rq; hence, the condition that PqP' is
perpendicular to N can be written

N-(R-Ro) =

This is a vector equation of the plane with normal vector N and containing the
point whose position vector is Ro

Figure 3

Parallel planes formed by Venetian blinds


782 C HAHIKR 13 COORDINATE SYSTEMS IN THREE-DIMENSIONAL SPACE

The equation N (R - Ro) = • is easily converted to scalar Cartesian form by


putting

N = ai + bj + tk R = .vi + vj + rk and Ro = -Voi + Voj + zok

Then. R - R(, = (.v - .v,,)! + (y - yo)J + (^ - 2c))k

N • (R - Ro) = u(x - .v„) + b{y - yo) + c(z - Zo)

and the equation of the plane takes the form

g>4M0^V
a(x-
SECTION 13.4 EQLATIONS OF LINES AND PLANES IN SPACE 783

For instance, the point (.v, 0, 0) where the plane intersects the x axis satisfies
ax + b(0) + c(0) = D. or ax = D. Hence, the .v intercept is given by .v = D/a.
(Note that a t^ 0: otherwise, the plane is parallel to the
and x axis.) Similarly, the v
given by y = D/b and r = D/c. respectively. Since there is exactly
z intercepts are
one plane containing three noncollinear points, the three intercepts determine the
plane uniquely.

Figure 5 EXAMPLE 3 Consider the plane -x + + =


2y 3z 6.

(a) Find a unit normal vector to the plane.

(b) Find the intercepts of the plane.

(c) Sketch a portion of the plane.

SOLUTION

(aMVe can take N= -i + 2j + 3k. so that |N| = V'(-l)- + 2- + 3" =


14. Therefore, a unit normal vector to the plane is given by

N _ -i + 2j + 3k

(b) The X, y, and z intercepts are given by .v = 6/(-l) = -6, v = 6/2 = 3,


and r = 6/3 = 2, respectively.

(c) The shaded triangle in Figure 5 is a portion of the plane.

Figure 6 In general, to find an equation of the plane containing three given noncollinear
points A^ (.To, yo^ zq), B= U,, y,, z,), and C= (jr., y2, zi), notice that the
N = -4B X AC vectors AB and AC are parallel to the plane: hence.

N=ABXAC
gives a normal vector to the plane (Figure 6). Since (.vq, Vq, zq) lies on the plane, we
can find either a vector equation or a scalar Cartesian equation of the plane just as
before.

In Examples 4 and 5, find an equation in scalar Cartesian form of the plane satisfy-
ing the given conditions.

EXA.MPLE 4 Containing the three points A = (1. 1. -1). fi = (3 3 ~>) and


C= (3. -1. -2)

SOLUTION Here.

AB = (3 - l)i + (3 - l)j + (2 + l)k = 2i + 2j + 3k


and AC = (3- l)i + (-l - l)j + (-2 + l)k = 2i - 2j -k
Hence,
N = A5 X AC 4i + 8j - 8k

gives a normal vector to the plane. Since the point ( 1 . 1. - 1) belongs to the plane, a
scalar Cartesian equation is

4(.T- 1) + 8(y- 1) - 8(z+ 1) =


This equation can also be written in the form

4v + 8.V - 8z = 20 or .v + 2v - 2z = 5
784 tllAPTKR 13 COORDINATE SYSIKMS IN THREE-DIMENSIONAL SPACE

KXAMPLE 5 Containing the points (3. 2, I) and (-5, 1, 2), but not intersecting
tiie y axis

SOLUTION Because the plane is parallel to the v axis, its equation has the form
ax + cz = D. (Why?) Therefore, since the point (3, 2, 1) satisfies the equation, so
does the point (3, 0, 1). Letting A = (3, 2, 1). B = (-5, 1, 2), and C = (3, 0,1),
wc have AB = - 8i '^
j + k and AC -2j, so that a normal vector N to the plane
is given by

N = A6 X AC = = 2i + 16k

Therefore, an equation of the plane is

2(.v - 3) + 0( y - 2) + 16(r - 1 ) = = 11

An equation of a plane can also be put into vector or scalar pcinimetric form;
since a plane is two-dimensional, iwo independent parameters are required. We do
not take up this matter here.

Figure 7 Lines in Space

A line in space can be specified by giving a nonzero vector M parallel to the line and
a point Pq = {x^, yo, Zo) on the line (Figure 7). The vector M called a direction
is

vector for the line. Two lines with the same direction vector are parallel to each
other; hence all the lines that have the same direction vector form a "bundle" of M
mutually parallel lines. Specification of the point /'o chooses from this bundle the
one and only line that contains Pq (Figure 8).

It is geometrically clear (Figure 9) that a point P belongs to the line containing the
point P() and having the direction vector M if and only if the vector P^P is parallel to

M, that is,

Figure 8
M X 7VP=0

(Recall that two vectors are parallel if and only if their cross product is the zero
vector.) If R is the position vector of P and Ro is the position vector of Po, then
PiiP=R - Rq; hence, the condition that M be parallel to P^^P can be written

M X (R - R„) =

This is a vector nonparametric equation of the line with direction rector M and
contuininf> the point whose position vector is R(|.

To convert the vector equation M X (R - R,,) = to scalar Cartesian form, let

M = (I'l + hi + ck R = x'l + yj + rk and Ro •V()i + yoj + 'ok

Then M X (R - R,,) = can be rewritten as

« J
a b Oi + Oj + Ok
X - xo y - yo

that is.

\b(z - zo) - c(y - y„)]i - [a(z - Zn) - ((.v - .v„)]j

+ |fl(y - .Vo) - b(x - .vo)lk = Oi + Oj + Ok


SECTION 13.4 EQUATIONS OF LINES AND PLANES IN SPACE 785

Equating the three scalar components, we obtain three simultaneous scalar equa-
tions

b(z - zo) - c(v - vo) = a(z - Zq) - c{x - Xq) =


a(y - Vo) - bix - Xq) =

b(: - -q) = c-(v - Vo) aiz Zo) = c(x - .To) a{y - Vo) = b{x - Xq)

These three simultaneous equations give a scalar Cartesian nonparametric form for
the line.
If the coefficients a, b. and c in the equations above are nonzero, we can rewrite
the equations in the form

.Vo X - Xo .Vo Xo

X
786 CIIAPIER 13 COORDINATE SYSTKMS IN THREE-DIMENSIONAL SPACE

is parallel to the line; hence, an equation of the line in vector form is

AB X {R- R„) =
where Ro = Xoi + V(ij + -ok. Thus, //; scalar symmetric form, equations of the line
through A = (.Vo. Vo. 'o) and B = (.Vi , Vi , Zi) are

(X Va ^
SECTION 13.4 EQUATIONS OF LINES AND PLANES IN SPACE 787

EXAMPLE 9 Find equations in scalar parametric form for the line containing the
point Po = (3, —2, 5) and perpendicular to the lines

SOLUTION
X -
3
I V +
1-2 2

Here, vectors parallel to the indicated lines are given by


and
.V +
12 7 V - 5

M, = 3i + j - 2k and Mj = + i 2j + k
so that i
J k
M = M, X M. l3 1 -2 = 5i - 5j + 5k
il 2 1

is perpendicular to both lines. Therefore, M is parallel to the desired line, and scalar
parametric equations for the line are

[.V = 3 + 5f

iv = -2-5/
[.- = 5 + 5t

Problem Set 13.4

In Problems 1 to 4, find scalar Cartesian equations of the plane In Problems 13 to 16. determine whether the planes are parallel.
containing the point Po and having the given normal vector N.
i3 3.V - V + 2; = 5 and 6.v - 2y + 4; = 10

'> = - 1 2) and N= + i 2j - 3k -X + + = 4 and + + 15r = 21


/'o ( 1 • .
14 v 3; 5.v 5_v

2 Po = (1. 3. -1) and N = 2i + j - k iy,2r +y- r = 3 and 4.v + v - 3r = 3

J Po = (0. 0. 0) and N= 5i - 2j + 10k i& X + 2.V - 2r = 3 and -.r - 2.v + 2z = 3

4 /»(,= (0, 0. 1) andN= j + k


In Problems 17 to 26, find a scalar Cartesian equation of the plane
satisfying the given conditions.
In Problems 5 and 6, give scalar Cartesian equations of the plane
containing the points A. B. and C.
I iTj Containing the point (-1, 3, 5) and parallel to the plane
6x-3y-2z + 9 =
3 A = (2, -1, 0), B = (-3. -4. -5). and C= (0. 8, 0)

18 Containing the point (4. -1. 3) and parallel to the plane


6 A = (2, 2, -2), B =S4. 6. 4), and C= (8. -1.2) - = 4
Ix y + 5r

In Problems 7 to 12. find (a) a unit normal vector to the plane and 19 Containing the point (-1. 3. 2) and parallel to the yr plane
(b) the intercepts of the plane, (c) Sketch a portion of the plane.
20 Containing the point (5. |. -1) and parallel to the xz plane

T 2x + 3.V + 6r = 12 8 .v - 4v + 8r = 8 2 Containing the origin and parallel to the plane 3x - 7y + z = 4

9 N • (R - Ro) = 0. where N= 12j - 5k and Rq = 5j 22 Containing the origin and the points P = {a, b. c) and Q=
- (p. q. r)
10 N • (R Ro) = 0, where N = kandRo = i+j + 3k
23 Containing the points (1,2,3) and -2. 1 1 ( . ). but not intersect-
11 5x = 3y + 4z
ing the X axis. (Hint: Begin by arguing that if the plane
12 3jc = 4z + 12 ax + by + cz =D '\s parallel to the x axis, then a = 0.)
788 (HAITKR 13 t <M)R1)1NATE SYSTEMS IN THREK-DIMF.NSIONAl. SPACE

24 Containing the point (4, -2. I) and perpendicular to the line 45 Symmetric form: L contains the point (7. 1, -4) and is parallel
containing the two points -4 = (2. -1,2) and B = (3, 2, -1) to the X axis.

25 Containing the point {1. 1. 1) and perpendicular to the vector 46 Scalar parametric form: L contains the origin and is perpendicu-
whose direction angles are 7r/3, 7t/4, and 7r/3 lar to the line

y-3
26 Containing the point (2,3,
the origin and the points
1 )

P =
and parallel to the plane containing

(2. 0, -2) and ^= (1. I. 1)


.r-2 = -^ =
y
z

at their point of intersection.


In Problems 27 to 32, give a geometric interpretation ot the condi-

tion imposed on the plane whose equation is ax + by + c: = D by 47 Symmetric form: L contains the point (2, 3, - I ) and is perpen-
the given equation or equations. dicular to the two lines

?7 O= 28 a = X- 3 V - 1

and
29 (/ = and <» = 30 2a + 3h - 4c =
48 Scalar parametric form: L contains the origin and is perpendicu-
31 /> = 32 fr = and r =
lar to the two lines

In Problems 33 to 40. find an equation or equations of the line - 3 V- 2


.V
^ y and 3
satisfying the given conditions in (a) vector nonparametric, (b) sca- 6
~ 5
.t -I-

lar symmetric, (c) vector parametric, and (d) scalar parametric form.

49 Show that the points (2. 0. 0). (-2. -22. -10). and (4. 11, 5)
33 Containing the point (
- 1 . 1.4) and parallel to the vector M =
lie on a line, and find equations of this line in symmetric form.
i + j - 2k
50 Assuming that a. b. and c are nonzero, derive equations in the
34 Containing the point (.'i, 7,-1) and parallel to the vector M =
symmetric form by eliminating the parameter t from the scalar
3i - 2j
parametric equations

35 Containing the point (3, 1, -4) and parallel to the vector M =


X = .V() + at
-2j -^5k
y = Vo + bt
36 Containing the point (1. 3, -2) and perpendicular to the plane
: = r,, -t- CI
.V - V + 2r = 5
Exactly what happens if. say. a = 0, but b and c are not zero?
37 Containing the point (1 ,
- 1 , 2) and perpendicular to the plane

5x - y + 3z = 7

38 Containing the point (0. 0, 1) and parallel to the vector with


In Problems 51 to 56. give a geometric interpretation of the condi-

direction angles In/i. tt/3. and 7r/4


tion imposed on the line

39 Containing the points (I. 3, -2) and (2, 2, 0)


.V = Xo + at

y = Vn + bt
40 Containing the points (-1, 3, 4) and (4, 3, 9)
= Zo + CI

In Problems 41 to 48, find an equation or equations of the line L in


by the given equation or equations.
the indicated form.

51 .v,i = Vo = r„ = 52 a =
41 Symmetric form: L contains the point (0. 4. 5) and is parallel to

the line through the points (1,5, -2) and (7, 7, I). 53 a = and b = 54 5o - 3b + 7c =

42 Scalar parametric form: L contains the origin and is perpendicu- 55 b = 56 fc = and c- =


lar to both of the vectors - 3i -H 2j -I- k and 6i - 5j -t- 2k.
57 Find the coordinates of the point on the line
43 Vector nonparametric form: L contains the point (3, - 1 , 2) and
is parallel to the line .V - 3 ^ y - 2 _ z-5
- 4 ~ 2 ~ 10
X 1 V + 1 z + 2

where this line intersects the plane .v = 5.

44 Vector parametric form: L contains the point (-1, 3, 5) and is 58 Find the coordinates of the point where the line jr
- 2 - 3/.

parallel to the line x = 3t - I, y = 5t + 1, z = -It + 1. y=l+i,z = 4-t intersects the plane Iv - 3y + : = - 16.
SECTION 13.5 THE GEOMETRY OF LINES AND PLANES IN SPACE 789

13.5 The Geometry of Lines and Planes in Space


In Section 13.4 we developed equations in both vector and scalar forms for lines and
planes in we obtain formulas for the dis-
three-dimensional space. In this section
tance from a point to a plane, the distance between two lines, the angle between two
planes, and the angle between two lines.

The Distance from a Point to a Plane

Lines in space formed by supporting


The argument used in Chapter 12 to derive a formula for the distance from a point to

cables of a bridge a line (Theorem 1. page 726) can also be used to prove the following theorem.

THEOREM 1 Distance from a Point to a Plane

Let Ri = OP, be the position vector of a point Pj in space. Then the perpendicu-
lar distance d from P, to the plane with vector equation N (R - Ro) = is

given by

|N-(R, -Ro)|
d=

Let P|| be the point with position vector Ry. so that R,j = OPq, and P,, lies on the
plane N • (R - Rq) = 0. The distance (/ is the absolute value of the component of

P^t = OPt - OPo = Ri - Ro

Figure I in the direction of the normal vector N (Figure 1): thus.

The distance formula in Theorem 1 can be converted to scalar Cartesian form as


follows: Suppose that a scalar Cartesian equation of the plane is a.x + bx + cz = D.
Then

N= ai -I- fcj -t- c-k R= .vi + vj + :k and N •


Rq = D
If P| = (.Y|, y,, r,), so that R, = .v,i -I-
y,j -I- r|k. we have

N • (R, - Ro) = N R, - N Ro =
• • o-v, -f- foy, -I- cr, - D
and

iNl = Vfl- + b- + c-

Now, substituting into the formula in Theorem 1 , we find that the distance dfrom
the point Pj = {x\, yj, Z\) to the plane ax + by + cz =D is given by

d =
790 CHAKIER 13 COORDINATE SYSTEMS IN THREE-DIMENSIONAL SPACE

F.XAMP1.K 1 Find the distance d from tiie point P, = (1, 3. -4) to the plane

Ix + 2y - z = 6.

SOLUTION Here a = 2, /? = 2, c = -1, D = 6, .V| = 1. v, = 3, Z| = -4, and


the distance d is given by

|avi + b\i + czi — D\


d =
Va~ + b~ + c-

|(2)(l) + (2)(3) + (-lK-4)-6|


Vl^ + 2- + (-If
6
- 2 units
V9

EXAMPLE 2 Find the perpendicular distance between the two parallel planes
- + 2z = - II
2jr 3- and Iv - v + 2z + 2 = 0.

SOLUTION The required distance is the distance from any point on the plane
2x — y + 2z=-\l to the plane Iv -y+ 2z + 2 = 0. For instance, the point
(-6, -1,0) belongs to the plane Zv - y + 2z = -11, and its distance from the
plane Zr — y + 2z + 2 = is given by

|(2)(-6) + (-l)(-l) + (2)(0) + 2|


d = = 3 units
V2' + (-1)- + 2-

The Angle Between Two Planes


It is geometrically clear that nvo planes are parallel // and only if their normal
N„-N, = vectors are parallel (see Figure 2 in Section 13.4). By definition, we say that nvo
planes are perpendicular ;/ and only if their normal vectors are perpendicular
(Figure 2).

EXAMPLE 3 Find an equation of the plane containing the points Pd = (1. 0. 3)


and Pi = (0, 1, -2) that is perpendicular to the plane Iv + 3y + z + 4 = 0.

SOLUTION The point Po = (1- 0, 3) belongs to the plane in question, so the


desired equation has the form '>N^
I OV, ^
N-(R-Ro) =
where Ro = OPo = i + 3k and the normal vector N has yet to be determined.
perpendicular planes
Since Pq and P, both lie on the plane. PoP^ is perpendicular to N. The plane
2jr + 3y + z + 4 = has normal vector Ni = 2i + 3j + k; hence. N is perpen-
^l^ dicular to N) , since the two planes are perpendicular. Thus. N will be perpendicular

to both of the vectors PoPi and Ni provided that we take

N Po/'i X N,

The desired equation is therefore

(161 - 9j - 5k) •[«-(! + 3k)] =


SECTION 13.5 THE GEOMETRY OF LINES AND PLANES IN SPACE 791

Figure 3 More generally, we can determine the angle between two planes by finding the
angle between their normal vectors (Figure 3). However, some care must be taken
to ensure that the angle is uniquely defined. Thus, let No'(R - Ro) = and
N| MR ^ Ri) = be equations of two planes having normal vectors No and N). We
detlne the angle a between the two planes to be the smaller of the two angles 6
and TT — 6. where 6 is the angle between No and Ni (Figure 4).* Since

Nq-N,
cos =
|No| IN,

it is easy to check that the smaller of the angles d and tt - 6 \% given by

,No-Nii
|No| |N,|

(Problem 52).

EXAMPLE 4 Find the angle


" a between the two planes 3.v — 2v + z = 5 and
x+ 2y- z= \1.
-

SOLUTION Here, the normal vectors Nq and Ni to the two planes are
No = 3i - 2j + k and N, = i + 2j - k. Therefore,

|No-N,|
== 77.4°
|No| |N,| VuVe V21

The Distance from a Point to a Line


The following theorem provides a handy formula for the distance from a point to a
line in space.

THEOREM 2 The Distance from a Point to a Line

The perpendicular distance d from the point P| whose position vector is


Ri = OPi to the line with vector equation x (R - Rq) = is given by M
|M X (R, Ro)|

'Ml

PROOF In Fiaure 5. we have

(/ = |R, - Rol sin e

_ |M| |Ri - RqI sin e


~
M
-
^ |M X (R, Ro)|

|M|
MX R
( - R„ I
=
by the formula on page 777 for the length of the cross product of two vectors.

*Notice that the two planes are perpendicular if a = 7r/2 and parallel if a = 0.
792 t HAKIKR 13 l.(K)RDINATE SYSTEMS IN THREE-DIMENSIONAL SPACE

(a EXAMPLE 5 Find the distance d from the point Pi = (1, 7, -3) to the line

2-12
- 5 \' + 4 z + 6

SOLUTION A direction vector for the line is M = 2i - j + 2k. and Rj, =


5i - 4j — 6k is the position vector of a point on the line. Thus, M X (R - R,,) =
is a vector equation of the line. Putting R, = 0P\ . we have Ri = i + 7j - 3k,
and so R| — Ro = -4i + llj + 3k. Therefore,

M X (R, - Rf,)
SECTION 13.5 THE GEOMETRY OF LINES AND PLANES LN SPACE 793

SOLUTION Here Mo = 3i + j - 2k. Ro = -2i + - k, M, = -i + + k.


j 4j
and Ri = 3i + j + Ok; hence.

i j kl
Mo X M, = j
3 1 -2|=9i-j+13k R, -Ro = 5i + 0j + k
1-1 4 ll

and (Mo x M,)-(R, - R^,) = (9i - j + l3k)-(5i + 0j + k) = 58


Hence,

|(Mo X Mi)-(Ri -Ro)| |58| _ 58


iMoxM.i - v9^tftfttf~ vir^^-^^""'^^

Theorem 3 has the following useful consequence:

Two nonparallel lines Mo x (R - R^) = and M, x (R - R,) = intersect


each other if and only if

(Mo x M,)-(R, -Ro) =

EXAMPLE 7

(a) Show that the two lines

.V + 9 ^' + 11 V - 5 - _ 1
—r and .v — 2 = =
5 7
meet in space.

(b) Find the coordinates of their point of intersection.

SOLUTION

(a) Direction vectors Mo and M, and position vectors Rq and R, of points on


the two lines, respectively, are given by

Mo = 5i + 7j-k Ro=-9i-llj
Ml = i + 2j + 3k R, = 2i + 5j + k
i
j k
Here we have Mq X M, = 5 7 -1 = 23i - 16j + 3k
1 2 3
SO that
(Mo X M,)-(R, -Ro) = (23i- 16j + 3k)-(lli+ 16j + k)

= 253 - 256 + 3 =
Therefore, the two lines do meet in space.
(b) We have four simultaneous equations

x + 9 y + 1 V - 5 r - 1

in the three unknowns .v. y. r. Choosing any three of these equations and
solving them, we find that .v = 1 . y = 3. and - = -2; hence, (1,3, -2) is the
desired point of intersection.*

*As a check, we could substitute .v = 1. y = 3. 2 = -2 into the fourth equation.


794 tH.\nKR 13 C(H)RDINATE SYSTEMS IN THREE-DIMENSIONAL SPACE

The Angle Between Two Lines


Consider the two lines M,, X (R - R,,) = and Mi X (R - R,) = having direc-
tion vectors M^, and M|, respectively. If 6 is the angle between M,) and M), we
define the angle a between the two lines to be the smaller of the angles 6 and
TT - 0, so that

|Mo-M|
iMnI |M,

Note that, by definition, we speak of the angle a between the two lines even if these
lines do not meet in space. In particular, the two lines are said to be perpendicular
if a = it/2: of course, they are parallel if a = 0.

1 EXAMPLE s Find the angle a between the two lines

v-5 - 7
= V + 3, z = 2 and

SOLUTION Here Mo = -7i + j + Ok and M, = 5i + 3j - 2k; hence.

iMo'M.I 32 16
= 42.77°
|Mo| |M,| -^Vss 5V1'

Intersecting Planes

Figure 7 Two nonparallel planes rtn.v + b^y + Ov = Ai i>nd ci^.x + hiy + c^: = D\ intersect

in a line / (Figure 7). The normal vectors to the two planes are given by

No = woi + /?„j + cok and fl,i + /?,j + r|k

respectively. Since / is contained in both planes, it is perpendicular to both of the


normal vectors No and Ni: hence, / /,v parallel to the vector

M = No X N|

An equation of /, in vector form, is therefore

M X (R - Ro) =

where Ro is the position vector of any particular point Po on /. To find such a point

f„, we just find any solution (.Vo, Vo, -o) of the two simultaneous equations

«o.v + hi^y + Cor Do


«].Y + h\y + C\Z D,

This can be accomplished by setting one of the three variables .v, v, or z equal to
and solving the resulting pair of equations for the remaining two variables.

EXAMPLE 9 Find equations in symmetric form of the line in which the planes

3a' + 2V + ; = 4 and x - 3y + 5z = 7 intersect.


SECTION 13.5 THE GEOMETRY OF LINES AND PLANES IN SPACE 795

SOLUTION The normal vectors to the two planes are given by Nq = 3i + 2j + k


and Ni = i — 3j + 5k; hence, a direction vector M for the line of intersection is

given by
i
J k
M = NoXN, = 3 2 1 =13i-14j-llk
1 -3 5

To find a point (.vq, Vo. -o) common to both planes, we put y = Vo = in the

simultaneous equations

[3.v + 2y+ r = 4
X - 3y + 5r = 7

and then solve the resulting equations

[3.V + z =4
X + 5; = 7
to obtain .v = Aq = if and r = tq = il. The point (aq. Vq- ^o) = (H. 0, 14) belongs
to both planes; hence, it belongs to the line in which they intersect. Equations in
symmetric form for this line are accordingly
13 17
A - T4 \ - rt
r - T4
13 14

Problem Set 13.5

In Problems I to 6, find the distance from the given point to the In Problems 13 to 16, determine whether the planes are perpen-
indicated plane. dicular.

1 (3. 2. -1); 7.V- 6v + 6r = 8 13 2v - v + 7z + 5 = and 9a + 4v - 2z = 13

2 (0, 0. 0): Iv + V - 2r + 7 = 14 6a + 3,v + z - 2 = and 2v - 5y + 3z = 8

3 (-2, 8. -3); 9a -v - 4; = 15 2r + 3y - z + 8 = and 5a - 3v + 2z + 1 =

4 (-1. 2. -1): A + >=6 16 4a - y + 2z = 1 and 6a - 4y - z = 7

5 (0. -3. 0); 2v - 7y + r - 1 = number c such + - y + (2 — =


17 Find a that the plane (c 1)a c)z 5

6 (4, -4. 1): Iv + >•-.- = 3


is perpendicular to the plane 2v + 6y — z + 3 = 0.

18 Find the equation of the plane parallel to the two planes


In Problems 7 and 8. find the perpendicular distance between the 8a — y + 3z = 12 and 8a — + 3z — 17 = bullying midway
y
two given parallel planes.
between them.

7 8a + y - 4.- + 6 = 0; 8a + .v
- 4r = 24 In Problems 19 to 21, find an equation in scalar Cartesian form for

8 Ix + }•-:= 12: 8a + 4y - 4z = 7 the plane satisfying the conditions given.

19 Containing the points (- 1, 3, 2) and (—2, 0, 1 ) and perpendicu-


Elln Problems 9 to 12, find the angle a between the two planes.
lar to the plane 3a — 2y + z = 15

9 A - 2v + 3z = 4; -2v + 2v - z = 3 20 Containing the origin, perpendicular to the plane whose equation


- = = is 14a -I- 2y -I- 1 1 = 0, and having a normal vector making a 45°
10 3a z 7; 2v + 4z 5
angle with the vector k
11 4a - 4.V + 7z = 31; 3a - 2y - 6z = 11
21 Containing the point (2. 0, 1) and perpendicular to the two
12 2v + 3y + 5z = 17; a = 4 planes 2v — 4y — z = 7 and a —y + z = 1
796 CHAPIER 13 COORDINATK SYSTEMS IN THREE-DIMENSIONAL SPACE

22 Show that the distance d between the two parallel planes Oln Problems 37 and 38. find the angle a between the two lines.

+ by + cz = D] and ax + by + cz = Di is
ttv given by
3 \ - 2 z+ \

Di - O2 37 and
£/ = 2 2 5 -3
Va- + b- + c-
X- 1 ; - 3 v+ I

23 Find a formula for the distance from the origin to the plane and
ux + by + cz = D.

24 Show that if the point P^ with position vector R, is on the same 39 Consider the two lines

side of the plane N (R - Rii) = as


• the normal vector N (when
+ + X - 3 -
X \' 3 r I , V r I

the tail end of N is on the plane), then N-lRi - R())/|N| is

positive.

(a) Find the angle a between the two lines, (b) Show that the two
In Problems 25 to 28. find the distance d from the given point to the lines intersect in space, (c) Find the point of intersection of the
indicated line. two lines.

- 3 40 Find a necessary and sufficient condition for the line


(25)l.2.3);^ = ^_j
.V = Xo + at

y = yo + bt
26 (0. 0. 0); .V = ^^— — and r = 4 Z = Zq + Ct

to intersect the .v axis.


-
v-2
27 ( - I , 1 . -\):x
-3 41 Determine whether the line

.V - -Vq _ .V - Vq A- + 3 V z + 1
28 (0, 0. 0);

is perpendicular to the plane 3.v + 2v - z - 11 =0, and find


In Problems 29 and 30. find the distance </ between the two lines.
the coordinates of the point where the line intersects the plane.

29 '
V
—-T~ = 3 z + 4
:
— and — , .V
- = - —+— = —+
V
:;^
1 r
;
2 42 A certain plane contains the
but does not contain any of the points on the line
two points (1,0, I) and (I. 1,0).

R = -4i + - + 4i - 4j + z -
30 (
4j 7k)/ 7k. and JC
^ y ^ I

R = (2i + 2j + k)/ + i
- 2j + 3k I I 1

Find a scalar Cartesian equation of the plane.


In Problems 31 to 34. determine whether the lines are parallel or
perpendicular.
In Problems 43 to 46. determine whether the two planes are parallel.
they are not parallel, find equations in symmetric scalar form of
,.
31
.V -
4
3
= - —-2+ — = —+—
V
-
1 z
-
3
1
and
, x+
3
I V -
3
I z +
-2
1
If

their line of intersection.

x+ I V - 1 z + 2 X" 4 v-2 + 1
43
2v
32 and
3 -2 -4 10

v + 3 .V + 7 V + 3 z + 4
33 —I

and 45
-3 -12

^A
34 —+— =
^
-
3 .v-3
= —+—
z
:
5
and x = 2,
v-4

In Problems 35 and 36. determine whether the two lines meet in

space. If they do meet, find their point of intersection.

,^
35 —x-5— = -
v-3 = z -
-3
— I

and
, .t -
7
1

,.
36 ——=——=
-v-3
z
y-5
:
z and
V - 7 V + 6
SECTION 13.6 CURVES IN SPACE 797

49 Find an equation of the plane that contains the line (d) Find the angle between the line

V - 2 y _ ,- + 3 r - 1 _ y
=
3 " -1 ~ 2 3 " 1
"

but does not intersect the line and the plane x + ly = 1

A = 4f - 1 52 If 6 is the angle between the nonzero vectors No and Ni. show


V = 2t+l that
= 3/
WW |No-N||
- -1
« = '^°^

50 Let l\ be the line of intersection of the two planes .v + v — 3r =


always gives the smaller of the two angles d and -n — 6.
and 2v + 3v — 8r = 1 , and let /^ be the line of intersection of
the two planes 3.v - y - r = 3 and .r +y- 3z = 5. (a) Show 53 Find a point on the line .v = y = - that is equidistant from the
that /] and Ij are parallel lines, (b) Find a scalar Cartesian equa- points (3. 0. 5) and (1, -1, 4).
tion of the plane containing l\ and /;.
54 A tetrahedron is a pyramid with four vertices Pq- P\- Pi- and Pj
51 (a) Give a reasonable definition of the angle a between the line and four triangular faces (Figure 8). Its volume V is one-third

Mo X (R - R„) = and the plane N, -(R - Ri) = 0. of the distance from Pn to the base P\P2P% times the area of
(b) Show that the angle a of part (a) is given by the base. Find a formula for the volume V in terms of the co-
ordinates of the vertices. Pq = Uo. .Vo. zq), P\ = U|, y\, Zi),
-, |Mo-N,|
.

P2 = (-T;. y:. z^). and P., = (.v,. y,, Z3).


iMol |N,|
Figure 8
(c) Find the anale between the line

and the plane Iv -1-


2y = 6.

13.6 Curves in Space


The concept of a vector- valued function and the results obtained in Sections 12.4 to
12.7 can be extended to vectors in three-dimensional space. Indeed, all the defini-
tions in those sections were so formulated as to be applicable not only to vectors in

the xy plane, but also to vectors in space. Furthermore, all the facts about continuity
and differentiability of vector-valued functions carry over immediately to.vvr space,
provided that the vectors are written in terms of three scalar components. For
instance, if F is a vector-valued function of the scalar t. then

F(f) = u(t)i + \V)i + >v(/)k

where ;/. v. and >v are the three scalar component functions of F; furthermore. F is

dijferentiable if and only if u. v. and w are differentiable . If u. v. and w are


differentiable . then

F'(f) = u'U)\ + v'(f)j -I- w'Wk


and so forth.
798 CHAITER 13 COORDINATE SYSTEMS IN THREE-DIMENSIONAL SPACE

The facts concerning the cross product of two vector-valued functions are exactly
as might be expected. Indeed, if F and G are vector-valued functions, then;

1 If liin F(/) and


SECTION 13.6 CURVES IN SPACE 799

Figure 2
gives the instantaneous speed ;• of P. that is.

ds , ,
dR
dt
800 CHAPTER \i C (M>KDINATE SYSTEMS IN THREE-DIMENSIONAL SPACE

dR
V'24f- + 9 + \bi^ = V'(4r + 3)- = 4r + 3
dt

dR/dt 2V6/i + 3j + 4rk


and T = , ._-.., = — :
tor any value of /
|JRM| At- + 3

2V6i + 3j + 4k
(a) For / = 1 , we have T =

dK
(b) v + 3) dt = units
Jn I
dt I J,,

Normal Vector and Curvature


Figure 3 In what follows we denote by i the arc length measured along the curve C between a
fixed starting point f (, and the point P whose position vector is R= F(/) (Figure 3).
Then, by the chain rule.

JR dK dt dR/dt
ds dt ds ds/dt

The vector dl/ds is still called the curvature vector, just as it was in two
dimensions. In three-dimensional space, we do not attempt to give the curvature k
an algebraic sign, and we simply define

K =

Figure 4
SECTION 13.6 CURVES IN SPACE 801

If we differentiate both sides of the equation V= vT with respect to /, we obtain

A = —— = — T + I

dt dt dt

= —
dv
dt
^
T + I
ds

dt
dT
ds

A = --T + v-kN
dt

Hence, even in three-dimensional space, the acceleration vector A decomposes into


the sum of a tangential component vector (dv/dt)T and a normal component
vector i'-kN.
Taking the cross product with V on both sides of the last equation
A= (dv/dt)T + v-kN, and using the fact that V= vT. we obtain

V X A = -^(V X T) + \--K(V X N) = ^v(T X T) + v-kv(T x N)


"' dt
= + v-V(T X N) = v\(T X N)
In what follows, we make extensive use of the equation

V X A= v^kCI X N)
D
Since T and N are perpendicular unit vectors, it follows that |T X N| =
|T| |N| = 1; hence, by the equation above,

|V X A| = li-^Kd X N)| = v^k|T X N| = v^K

Solving for k, we obtain

|V X A|

Because vT = V. the equation V x A= vVd x N) can be rewritten as

V X A= v-K(V X N)

Taking the cross product with V. we obtain

(V X A) X V= v-k(V X N) X V = v-k[(V-V)N - (N-V)V]


where we have used the triple vector-product identity (Theorem 4 in Sectioa 13.3)
toexpand (V x N) x V. Since V-V = |V|- = v- and

N-V = N-(vT) = vN-T= v(0) =


the equation above can be rewritten as

(V X A) X V= v-K(v-N - OV) = v-'kN

Thus.

(V X A) X V
N=
802 C HAFI KK 13 CUURDINATE SYSTEMS IN THREE-DIMENSIONAL SPACE

The equations k = |V x A|/i''' and N= (V X A) X \/(v*k) allow us to find the


curvature and the principal unit normal vector directly in terms of the original
parameter /.

EXAMPLE 3 Find T, N. and k if R= (2l~ + l)i + (/- - 2/)j + (?- - l)k.

SOLUTION Here! V= dR/dt = 4ri + (2/ - 2)j + 2rk. so that

V = IVl = Vl6f- + (2/ - 2f + 4/2 = 2V6r- - 2f + 1

and
A= —^V— = 4i + 2j + 2k
dt

Thus, _ 2fi + (/- l)j + tk


T=
\/6f2 - 2/ + 1

i
J k
V X A 4/ 2/ - 2 2/ -4i + Oj + 8k = -4(i - 2k)
4 2 2

i
J k
(V X A) X V -4 8 (-16f + 16)1 + 40rj + (-8/ + 8)k
4/ 2/ - 2 2/

|V X Al 4|i - 2k| VS
v' 8(6/- - 2/ + 1)'/- 2(6/- - 2/ + 1)3/2

(V X A) X V _ (2 - 2/)i + 5/j + (1 - /)k


N=
V5V6/2 - 2/ + 1

Binormal Vector and Torsion


The vector T x N that appears in the equation

V X A = vV(T X N)

is called the unit binormal vector to the curve and is symbolized by B. Thus, by
definition.

B = T X N

that is.

Figure 5
SECTION 13.6 CURVES IN SPACE 803

(see Problem 67 in Section 13.2). In particular,

ds -1-^-^AM^-NJN+(— -BJB
Differentiation of both sides of the equation
N-T = with respect to s gives
^
(dN/ds) • T + N • (dT/ds) = 0, so that

d^ ^ dT
"^ "^'"^ " -N-(,cN) = -/cN-N = -K(l) = -K
Similarly, differentiation of both sides of the equation N N=

1 gives
^
2WNM)-N = 0. sothat
dN
---•N =
ds

Therefore, substituting into the equation


above for dN/ds, we obtain
'^N
^ f dN \

The scalar (dN/ds) • B in the last equation is called the torsion of the curve
and is
traditionally denoted by the Greek letter r (called "tau"). Thus, by definition.

dN

and the equation dN/ds = -kT + l(dN/ds)-B]B can be written

dN
= -kT + tB
ds

The torsion x is a measure of how rapidly the curve is "twisting" in space The
three equations

dT dN
ds
kN = -kT + tB and —dB-= -tN
ds ds

are called the Frenet formulas. (For a proof


of the third equation, see Problem 44.)
These Frenet formulas— which express the rates
of change of T, N and B with
respect to arc length in terms of T, N, B,
the curvature, and the torsion— are the
fundamental equations for the study of curves in space.
It can be shown that all the
intnnsic geometric features of the curve are
implicit in these three equations
If we differentiate both sides of the identity A = {dv/dt)T+ v^kN with respect to
arc length s and use the dT/ds = kN and
identities dN/ds = -/cT + tB. we obtain
dA^ ^ d(dv/dt) dvdT^ d(v-K) dN
ds ds '^ dt ds ^ ~^^ + "'"-^
_ d(dv/dt) dv d(v-K)

Taking the dot product on both sides of the last


equation with B and using the facts
that T.B = N-B = and B.B=1, we obtain (dK/ds)'B = v^-kt.
Since
804 t IIAI'IKR 13 COORDINATE SYSTEMS IN THREE-DIMENSIONAL SPACE

vidA/ds) = (ds/dt){dA/ds) = d\/dt. we can multiply the last equation by \' to ob-

tain
d\_
I
B = V" KT or B = vVr
^ ds dt

Since B = (V x A)/(iV), we have

dX
d\ i
/ V X /A
\
dt ' ^ vV

{d\/dt)'(\ X A)

This formula gives the torsion r in terms of quantities that can be found directly in

terms of the parameter t.

The following example involves a curve R= (a cos Oi + (« sin /)j + /;/k,

where a and b are constants and a > 0. In scalar parametric form, .v = a cos /,
^J^U,^^ y = a sin t, and z = bt. As the parameter / increases, the point (.v, v, 0) =
Circular helix: the track of a roller (a cos I, a sin t, 0) in the xy plane traces out a circle of radius u. Meanwhile, the :
coaster in an amusement park coordinate z = bt changes uniformly, so that the point (.v, y, r) traces out a circular
helix (Figure 6). When / increases by Itt. x and y return to their previous values
while 2 increases by lirb. We say that the helix has radius a and pitch 27r|fo|.

Figure 6 EXAMPLE 4 If R = (2 cos Oi + (2 sin Oj + 3/k, find V, v. A, T, V x A, k, N,


B. dA/dt, and r.

dR
(-2 sin /)i + (2 cos t)j + 3k
dt

|V| = V4 sin- / + 4 cos- / + 9 = V\3


d\
A= (-2 cos t)i - (2 sin f)j
dt

so that [(-2 sin t)\ + (2 cos t)\ + 3k]


'13

i
J k
V X A = -2 sin r 2 cos / 3 (6 sin /)« ~ (6 cos t)\ + 4k
-2 cos / -2 sin t

Thus,

|V X A| _ V36 sin^ t + 36 cos^ ? -I- 16 _ \/52 _ 2

v^ I3\/T3 13

N =

and
SECTION 13.6 CURVES IN SPACE 805

dA_
Also, (2 sin r)i - (2 cos r)j
dt

SO that

1 dX
'
(V X A)
v"/c- di

— 1

[(2 sin f)i - (2 cos Oj] •


[(6 sin r)i - (6 cos Oj +'^•'1 = ^ 12
=
3

7^

Problem Set 13.6

In Problems 1 to 6. find (a) ¥'{!„) and (b) F"(/o)- 18 R = (cos 30i + (sin 30j + 5fk: a = Q. b = lir

1 F(r) = 5f-i + (3r + l)j + (3 - r)k. f,, = I


19 R= 6/'i + 3r-j + k; a = 0. b = 3

2 F(r) = V7 + i e"-"j + /k. f,, = 4 20 R= 2r^'-i + (r - 3)j + 3rk; a = 6. b = \Q

3 F(f) = ( 1 + /)T + (cos r)j + In ( 1


- ;)k, Iq = 21 R = e'i + e~'i + V2fk: u = 0, b= \

4 F(/) = (2 + sin /)i + (cos Oj + ik. t„ = tt/I


22 R = (r sin Oi + U cos Oj + k: a = 0, fc = 4

5 F(r) = (cos 2?)i + (sin 2r)j + r'k, = 7r/4


23 R= (e~' cos Oi + (e~' sin f)j + e"'k; a = 0. b = tt
to

6 F(f) = e'^'i + e"j + fk. = 24 R= (cos t)i + (In cos Oj + (sin Ok; a = 0. b = tt/A
fo

25 R = (rV3 - lA)i + (rV3 - 7/Oj + 2;k; a= \.b = A


In Problems 7 to 12, find (a) the velocity vector V, (b) the speed v,
26 R = (2/- + 3)i + (3 - 2r)j + 4/k; a = Q.b = 2
and (c) the acceleration vector A at the time / of the particle whose
equation of motion is given.
In Problems 27 to 32. find (a) V, (b) v, (c) A, (d) T, (e) Vx A,
(f ) K. (g) N, (h) B, (i) dk/dt. and t for each curve in space.
7 R= (2 cos 2r)i + (3 sin 2r)j + fk
(j)

8 R= (2 + f)i + 3fj + (5 - 3f)k 27 R = (2 cos Oi + (3 sin Oj + fk

9 R= e'i + le'i + t\ 28 R = (t- - 3)i + (t- + 7)j + (t- + Ok

10 R= (In cos /)i + (sin ?)j + (cos r)k 29 R = {€-' + e~-')\ + (€-' - e"-')j + 5fk

11 R= (/ sin t)\ + (f cos Oj + rk 30 R= e'i + Ite'i + 3e'k

12 R= {e~' cos t)\ + (e~' sin Oj + e~'\i.


31 R n + —^t-\ +
1 .. 1
—t\ ,.

V2 3
In Problems 13 to 16, find (a) V, (b) v. and (c) A for the indicated
value of t. 32 R
cos t .

+
In cos /

J + —sm— t

13 R= (cos 20i + (3f"')j + (sin 2r)k when t =


In Problems 33 to 36. let F(0 = Sri + (r - 2)j + r^k, G(0 =
14 R = (f sin Oi + e'j + (cos /)k when t = n + (1/Oj + '-k, and /i(0 = 3r.

15 R= (4 cos /)i + (4 sin Oj + e'k when r = tt/1


33 Find —-[h(t)¥(t)\.
16 R= (It - r)i + 3;j + (f' + l)k when t = 1 dt

(a) By calculating h(t)¥(t) and differentiating


In Problems 17 to 26, find the arc length of the portion of the given
(b) By using the formula
curve between the point where t = a and the point where t = b.

-|-[MOF(0] = [-^/i(oJf(0 + ''("[-^-foj


17 R= (3 cos f)i + (4 cos Oj + (5 sin r)k: a = Q. b = l
806 CHAPTER 13 COORDINATE SYSTEMS IN THREE-DIMENSIONAL SPACE

42 Suppose that the curve R= ¥(t) in space has curvature (c = at


34 Find ---|F{/) -Gl/)]:
ill all points, (a) Show T
that is a constant vector, (b) Show that

(a) By calculating F(/)-G(0 and ditTcrcnliating


T X R is a constant vector, (c) Show that the curve is a portion
of a line.
(b) By using the formula
43 Prove the lormula


dt
[F(/)-G(/)l = U-F(/) •G{0 + ¥u)-\-^G(i)
I dl i I dt
t/-R
'

d-R
K" \ ds ds' '
ds'

35 Find -—[FCO x GU)]:


1 dX
dt
Hint: Take t = s m the formula t = (V X A).
,f>.,2
(a) By calculating F(/) x G(() and differentiating

(b) By using the formula 44 Prove the third Frcnet formula:

dB
-^|F(/) X G(/)l = [-^J'oJ X G(r) + F(0 x [y«<') = -tN

dn
36 Find (a) -V|F(')| and (b) -Vf(/'). Hint:
dt dt

37 Find T. N, B. k, and t for the circular helix

X = a cos /

y = a sin f

-- = hi

38 Prove that if lim F(f) and lim G(t) both exist, then

lim |F(/) X G(r)| exists and equals [lim F(/)] x [lim G(r)].

39 IfF and G are continuous vector-valued functions, show that

F X G is also continuous.

40 Suppose that F and G are differentiable vector-valued functions


of 1. Prove thatF x G is differentiable and that

^ = d¥ dG
</
^
-— (F xG) ---xG Fx —
^ + ^
dt dt dt

</-(F X G)
41 Evaluate
dt-
SECTION 13.7 SPHERES, CYLINDERS. AND SURFACES OF REVOLUTION 807

Figure 2 Spheres
If f = (a, V, z) and Pq = Uo. >'o, zo). then, as we saw in Section 13.2, the distance
" iX-Xg)- +(:v-.Vo>' +'2-^0 )- =' between P and Pq is given by

l^^ol = V(.v - .vo)- + (V - vo)- + (r - zo)=

Therefore, the equation

(.V - .Vo)- + ( y - Vo)- + (: 3)- = r-

where ;• is a positive constant, means that the distance between P and Pq is r units;
hence, this equation represents the surfcice of a sphere of radius r with center at the
point Po = (ao, }'o, Zo) (Figure 2). In particular, if P,, = (0, 0, 0), then the equation
of the sphere becomes

x~ + y- + z- = r~

EX.\MPLE I Find an equation of the sphere with radius 5 units and center at
(-1, 2,4).

SOLUTION An equation of the sphere is (a + 1)- + (v - 2)- + (z - 4)- = 25.

EXAMPLE 2 Find the radius and the center of the sphere whose equation is

x-+ y- + :- - 6a + 2.v + 4z - = 0. 1 1

SOLUTION Regrouping terms and completing the squares, we obtain

(a- - 6a + 9) + (y- + 2v + 1 ) + (z- + 4z + 4) = 9 + 1 + 4 + 11

Figure 3 (A-3)- + {y + 1)= + (z + 2)- = 25

which represents a sphere of radius 5 with center (3, —1, —2).

Cylinders
A line L in space which moves so as to remain parallel to a fixed Ime Lo while
intersecting a fixed curve C is said to sweep out a cylindrical surface or just a —
cylinder for short (Figure 3). Any particular position of L is called a generator of
Figure 4 the cylinder, and the curve C is called the base curve. If the base curve C is a circle
and all the generators are perpendicular to the plane of the circle, then the surface is

called a right circular cylinder. A cylinder whose base curve is an ellipse, parab-
ola, or hyperbola naturally called an elliptic, parabolic, or hyperbolic cylinder,
PMS^ respectively.
is

In what follows we consider only cylinders whose generators are parallel to one
of the coordinate axes and whose base curve lies in the coordinate plane perpendicu-
lar to that axis. For instance, the cylinder in Figure 4 has generators parallel to the z
axis: hence, if the point Q= (.vn. vq, 0) is on the base curve C, then all points of the
form P = (.Vo, Vn. z) also belong to the cylinder. It follows that the equation of
this cylinder cannot place any restriction on z whatsoever that is. either z is not —
present in the equation, or else it can be removed from the equation by algebraic
manipulation.
808 CHAPTER 13 COORDINATE SYSTEMS IN THREE-DIMENSIONAL SPACE

To sketch the graph of a cylindrical surface whose equation is missing one of the
variables x. v, or z. begin by sketching the graph of the equation in the coordinate
plane involving the variables that are present, thus obtaining the base curve C.
The surface is then swept out by a line perpendicular to this coordinate plane and
intersecting the base curve.

In Examples 3 and 4, the graph of the given equation is a cylinder, (a) Identify the
coordinate axis that is parallel to the generators, (b) Specify the coordinate plane
containing the base curve, (c) Describe the base curve, (d) Sketch the cylinder.

EXAMPLE 3 y^ = X

SOLUTION

Figure 5 (a) The variable z is missing; hence, the generators are parallel to the z axis.

(b) The base curve is contained in the .it plane.

(c) The base curve is a parabola.

(d) The graph appears in Figure 5.

EXAMPLE 4 z = sin Jf

SOLUTION

(a) The variable y is missing; hence, the generators are parallel to the y axis.

(b) The base curve is contained in the .vr plane.

(c) The base curve is a sine curve.

(d) The graph appears in Figure 6.

Figure 6

Figure 7

R = (0, O.z)
Surfaces of Revolution
!2 = (0. .v,.z)
A surface of revolution is defined to be a surface generated by revolving a plane
curve C about a line L lying in the same plane as the curve. The line L is called the
axis of revolution. For instance, a sphere is the surface of revolution generated by
revolving a circle C about an axis L passing through its center. Notice that a surface
of revolution intersects a plane perpendicular to the axis of revolution in a circle or
a single point, provided that it intersects the plane at all.
Figure 7 shows a portion of a surface of revolution obtained by revolving the
curve C in the yr plane about the ; axis. In order to find an equation of this surface,
consider a general point P = (.v, y, :) on the surface, noting that P is obtained by
revolving some point Q on the original curve C about the z axis as in Figure 7.
Perpendiculars dropped from P and Q to the z axis meet the - axis at the same point
SECTION 13.7 SPHERES, CYLINDERS, AND SURFACES OF REVOLUTION
809

R~ (0, 0, z). Notice that P, Q, and R have ihe same z coordinate.


Let / = \PR\ =
\QR\. Since Q yz plane, its x coordinate is zero. Let Q =
lies in the
(0, v, , z), noting
that |vi ^^ r.

Now,
.vf = n = |P^P = {x- Of + (y~ Of +
(z - zf =x'' + y^

Hence, = ±Vx^ + y\ We conclude that if P = {x, v, z) belongs to


v,
the surface
of revolution, then either
Q = (0, V^^TJ-. z) or Q = (0, -V^^^T?-, z) belongs
to the generating curve. Conversely, =
if Q (0, v,, z) is a point on the generating
curve C then any point P = (x.
y, z) such that V^^T^ = |v,| lies on a horizontal
circle of radius r = with center at /? = (0. 0, z); hence, such a
|.v,|
point P belongs
to the surface of revolution. We conclude
that P = (x, y, z) belongs to the surface of
revolution if and only if = (0, ± V:;^^T7, z) belongs to the generating curve In
other words, an equation of the surface
generated by revolving the curve C in the yz
plane about the z axis is obtain ed by replacing
the variable y in an equation ofC
by the expression ±vx~ + y^.

EXAMPLE 5 The parabola z = v" in the yz plane is revolved about the z axis to
form a surface of revolution. A computer-generated
graph of this surface is shown
in Figure 8. Find an equation of this surface.

SOLUTION Replacing y in the equation z = v" by ± Vx" + we


v", obtain

z = (±Vx- + V-)- z = X- + v'

The surface in Figure 8 is called a paraboloid of


revolution. Similarly a surface
of revolution obtained by revolving a hyperbola
or an ellipse about one of its own
axes of symmetry is called a hyperboloid of
revolution or an ellipsoid of revolu-
tion, respectively.

Figure 8

The preceding considerations apply to curves lying in any coordinate plane,


and we have the following general rule for finding an equation of a surface of
revolution:

1 List the three variables in such an order that the


first variable represents
the axis about which the generating curve C
is revolved and the first two
variables represent the plane in which C lies.

2 In an equation for C, replace the second listed variable by plus or minus


the square root of the sum of the squares of the second
and third listed
variables.
810 CHAPTKR 13 COORDINATE SYSTEMS IN THREE-DIMENSIONAL SPACE

This procedure is summarized by the following table.

Axis of revolution
SECTION 13.7 SPHERES. CYLINDERS. AND Sl!RFACES OF REVOLITION 811

Problem Set 13.7

In Problems 1 to 4. find an equation of the sphere satisfying the 24 z = |;c|

given conditions.
25 r = sin 2v

1 Center at (2. -1. 3) and radius 4 units 26 v = e'

2 Its diameter is the Hne segment joining the points (8. - 1. 7) and 27 .V- + 4v =
(-2, -5. 5).
28 .r- + y- = 4.v

3 Center at (4. 1. 3) and containing the point (2. —1. 2)


29 x- = T
4 Containing the points (3, 0. 4). (3. 4. 0). (9. 0. V"? ). and
(-3. 0. 4) 30 Ivl + Izl = 1

In Problems 5 to 9. find the center and radius of the sphere.


In Problems 31 to 40. find the equation of the surface that results

when the graph of the given equation is revolved about the indicated
5 .V- + r + z- - Iv + 4.- - 4 =
axis. Sketch the surface.
6 A- + y- + z^ + 2v + 14y - lOz + 74 =
31 y = A in the .n- plane about the a axis
7 2v- + 2y- + 2z- + 4.v - 4y - 14 =
32 y = 3a + 2 in the vy plane about the y axis
8 a:- + y' + z^ - 8.V + 4v + lOz + 9 =
33 z- = 8a in the az plane about the a axis
9 a:- + y- + z- - 6a + 2y - 4z - 19 =
34 4a- — 9z- = 5 in the az plane about the z axis
10 Find the set of points that are equidistant from the points
35 z = A" in the az plane about the z axis
(1. 3. -1) and (2, -1, 2).

In Problems 1 1 to 30. (a) identify the coordinate axis to which the


36 ^ + —— = 1 in the yz plane about the z axis

generators of the cylinder are parallel, lb) specify the coordinate 37 6y- + 6z^ = 7 in the yz plane about the y axis
plane containing the base curve, (c) describe the base curve, and
38 A" — Azr =A in the xz plane about the x axis
(d) sketch the cylinder.

39 z = 1 — y' in the yz plane about the z axis


11 y = z-
40 y = In A in the xy plane about the a axis
12 AT = 3

13 yz = 1

In Problems 41 to 46, each equation represents a surface of revolu-


14 y- + z- = 4
tion about one of the coordinate axes. Identify this axis, find the

15 .r- + V- = 9 equation of a generating curve in the indicated coordinate plane, and


sketch the graph.

16 = 1

41 A" + y- + z = 3: az plane

17 -- +
9
^=
4
1
42 9z^ = X- + y-. xz plane

43 A- - 9y- - 9z- = 18: xy plane


18 y- = 4 - z
44 4 — y = 3 (a- + z"); yz plane
19 A- - z =
45 A" + y- = sin- z: xz plane
20 y = r
46 9a- -H 9y- -I- 4z- = 36: yz plane
21 y + z = 3

= 47 If A" -(- fl- -t- C- > 4Z), by completing the squares show
22 3a + 2y 6
that the equation a" + y- + z^ + Ax -H By + Cz D = can -I-

23 9y- - 4z2 = 36 be rewritten in the form (x — Ao)^ (y — yo)" • (z — Zq)" = r^.


-I-
812 C HAPTER 13 COORDINATE SYSTEMS IN THREE-DIMENSIONAL SPACE

13.8 Quadric Surfaces


in Chapter 9 we showed that, apart from certain degenerate cases, the graph in the

AT plane of a second-degree equation

Ax- + B.xy + Cy- + D.x + Ey + F =

is a circle, an ellipse, a parabola, or a hyperbola; in any case, it is a conic section. A


second-degree equation in the three variables x, v, and z has the form

Ax- + By- + Cz- + Dx}> + Exz + Fyz + Gx + Hy + Iz + K =

and the graph of such an equation in vvr space is called a quadric surface. By
rotating and translating the coordinate system in three-dimensional space, it is pos-
sible to bring the second-degree equation into certain standard forms and thus to
classify the quadric surfaces. Figures 1 through 6 show computer-generated graphs
of some quadric surfaces.
In this section we do not attempt a detailed study and classification of quadric
surfaces. Rather, our intention is to familiarize you with the general shapes and the

Figure I Figure 2 Figure 3

•'
a2 f2

an elliptic paraboloid a hyperbolic paraboloid


SECTION 13.8 QUADRIC SURFACES 813

Standard equations of the common quadric surfaces. We


beain by mentioning very
briefly, some of
the basic techniques for visualizing
or graphing surfaces in space.
These involve (1) cross sections. (2) traces
and intercepts, and (3) symmetries.

Cross Sections

The simplest (but in many cases the most


effective) technique for visualizing rec-
Figure 7 ognizing, or graphing surfaces in space is
to find the cross sections formed by
intersecting the surface with planes—
especially by planes
perpendicular to the coordinate axes or perpendicular
to the
axes of symmetry of the surface. For
example. Figure 7
shows the cross section cut from a surface by a = a
plane \
perpendicular to the v axis. An equation of the cross section
in the cutting plane y = a can be found simply by putting
y = a in the equation of the surface. The resulting equation,
which involves only v and r, is an equation of
the cross sec-
tion relative to copies of the v
and : axes in the cutting plane
as shown Figure 7. Exactly the same idea applies to
in
cross
sections cut by planes perpendicular to
the other coordinate
axes.

Traces and Intercepts

The cross sections cut from a surface by


the coordinate planes themselves are
called
the traces of the surface. For instance,
the yz trace of the surface (that is
the cross
section cut from the surface by the
yz plane) is obtained by putting
v =
in an
equation of the surface. The xy trace
and the xz trace are defined similarly
Ihe X y. and z mtercepts of the surface
are defined to be the points (if any)
in
which the .V. y, and z axes, respectively, intersect the
surface. For instance to find
the .V intercept, we set y = and = in an equation of the
.-
surface.

Symmetries

Surfaces often display symmetries with


respect to points, lines, or planes Of
course, a surface is said to be symmetric with
respect to a point, line or plane
provided that whenever a point P lies on the
surface, so does the point Q which is
symmetrically located with respect to the point,
line, or plane. For instance if an
equation equivalent to the original equation
of the surface is obtained when
y is
replaced by -y, then the surface is symmetric
with respect to the .v.- plane Symme-
try with respect to the z axis can
be tested by replacing both x and v by - v
and - v
respectively, and checking whether the
resulting equation is equivalent to the origi-
nal one. Similar tests are easily
discovered for symmetries with respect to the
other
coordinate planes or coordinate axes as well
as for symmetry with respect to the
ongm (Problems 25 to 27).

Central Quadric Surfaces


The graph in xyz space of an equation of the form

where a. b. and c are positive constants and not all three algebraic signs are nega-
814 CHAPTER 13 COORDINATE SYSTEMS IN THREE-DIMENSIONAL SPACE

live, is called a central quadric surface. Since only even powers of the variables
appear, a central quadric surface is symmetric with respyect to all three coordinate

planes, all three coordinate axes, and the origin. The central quadric surfaces are
classified as follows:

1 If all three algebraic signs are positive, then the surface is called an ellipsoid.

2 If two algebraic signs are positive and one is negative, then the surface is

called a hyperboloid of one sheet.


3 If one algebraic sign is positive and the other two are negative, then the
surface is called a hyperboloid of two sheets.

We now discuss each of the central quadrics briefly.

Figure The Ellipsoid

The .V, y, and r intercepts of the ellipsoid

x-

Figure 9
SECTION 13.8 QL ADRIC SURFACES 815

The X (±a, 0, 0). but there are no y and no


intercepts are r intercepts. (Why?) The
traces on the coordinate planes are as follows:

1 The .r\' trace is the hyperbola (x^/a~) - (y-/b~) = 1

2 The -vr trace is the hyperbola {x-/a~) - (z^/c-) = 1

Figure 10
3 There is no y: trace.

Although the hyperboloid of two sheets has no trace on the


yr plane, it does have cross sections cut by certain planes
X = k parallel to the yr plane. Indeed, the cross section cut by
the plane x = k has the equation

k~
ii-ii_
b-

provided that \k\ > a. The last equation can be rewritten as

= 1

b-l{k~/a-) - 1] c-Uk'/a-) - 1]

Therefore, the cross sections cut by planes perpendicular to


the X axis and at least a units from the origin are ellipses. A
portion of the surface, which consists of two separated
"parts," or "sheets," is shown in Figure 10.

Elliptic Cones
The graph in xy: space of an equation of the form
816 CIlAinKR li COORDINATE SYSTEMS IN THREE-DIMENSIONAL SPACE

Figure II Cross sections cut by planes perpendicular to the x or y axis that do not pass
through the origin are hyperbolas, and cross sections cut by horizontal planes that
do not pass through the origin are ellipses (or circles) (Problem 31 ). A portion olthe
elliptic cone is shown in Figure 1 1. Notice that \i a = b, the elliptic cone becomes
an ordinary right circular cone.

Elliptic and Hyperbolic Paraboloids


We now consider the graph in vvr space of an equation having one of the forms

^
SECTION 13.8 QUADRIC SURFACES 817

1 The xy trace. (y-/h-) - (x^/a-) = 0. or v = ±{b/a)x. is a pair of intersect-


ing lines.

2 The xz trace, ~x~l(p- = z, is a parabola opening downward.


3 The yz trace, y~lb' = ;. is a parabola opening upward.

Figure 13 Moreover, cross sections cut by horizontal planes above the origin are hyperbolas
with transverse axes parallel to the v axis, and cross sections cut by horizontal
planes below the origin are hyperbolas with transverse axes parallel to the .v axis
(Problem 33). All cross sections cut by planes perpendicular to the x axis or y axis
are parabolas opening upward or downward, respectively (Problem 34). A sketch of
a portion of the hyperbolic paraboloid appears in Figure 13. Near the origin, the
hyperbolic paraboloid has the shape of a saddle.*

Examples of Quadric Surfaces


The surfaces discussed above, together with cylinders whose base curves are conic
sections, exhaust all possibilities for quadric surfaces with the exception of certain
degenerate cases which we do not consider here. In the following examples we
illustrate the technique of identifying and graphing quadrics whose equations are in
standard form.

In Examples 1 to 4. for the given quadric siiiface, (a) find the intercepts, (b)
discuss tlie symmetry, (c) find the cross sections perpendicular to the coordinate
axes, (d) find the traces, (e) identify' the surface, and (f) sketch the graph.

EXAMPLE 1 x~ - 9v- + -- = 81

SOLUTION After dividing by 81, we see that the equation has the form
(xya^) - (yVfe2) + iz-/c^) = 1, where a = 9. b = 3, ?aid c = 9.

(a) The .v intercepts are (±9, 0, 0). There are no v intercepts. The : intercepts
are (0, 0, ±9).

(b) All variables are squared, so the surface is symmetric with respect to all
three coordinate planes, all three coordinate axes, and the origin.

(c) The intersection with the plane — 9y- = 81 — Ir. x = k is the curve z-
which is a hyperbola except = ±9, the intersection
when k = ±9. When k
with the plane x = k is the pair of intersecting lines z = ±3y. The intersection
with the plane y = k is the circle .v" + z- = i\ + 91c oi radius VSl + 9lr.
The intersection with the plane z = k\s the curve x' — 9y~ = 81 — kr, which
Figure 14 is a hyperbola except when k = ±9, in which case it is a pair of intersecting

lines X = ±3.v.

(d) The traces are found by putting A' = in (c). The yz trace is the hyperbola
- — 9y~ = 81. The xz trace is the circle jt" + z^ = 81. The .r\' trace is the
hyperbola .v" — 9v" = 81.

(e) The surface is a hyperboloid of one sheet — in fact, a hyperboloid of revo-


lution about the y axis.

(f) The graph is sketched in Figure 14.

*For this reason, the origin O is called a saddle point of this surface.
818 t'HAPIKR \i COORDINATE SYSTEMS IN THREE-DIMENSIONAL SPACE

EXAMPLE 2 -9x^ - I6y- + z^ = 144

SOLUTION The equation has the form -(.x~/a~) - {y-/b~) + {z~/c^) = 1 with
a = 4. /> = 3. and c = 12.

(a) There are no x intercepts. There are no y intercepts. The z intercepts are

(0, 0, ±12).
(b) All variables are squared, so that the surface is symmetric with respect to
all three coordinate planes, all three coordinate axes, and the origin.

(c) The intersection with the plane x = k is the hyperbola z~ — \ty~ =


144 + 9/r. The intersection with the plane _v = ^ is the hyperbola
^2 _ 9^2 = J44 j^ ^(^fj. Pq^ 1^1 > 12 thg intersection with the plane z = k
is the ellipse 9x^ + 16v" = Ic - 144.

(d) Putting k = Q in (c), we find that the yz and xz traces are the hyperbolas
z- — 16y~ = 144 and -" - 9x- = 144, respectively; there is no .rv trace.

(e) The surface is a hyperboloid of two sheets.

(f) The graph is sketched in Figure 15.

Figure 15

16>'2 +r2 = 144

EXAMPLE 3 =
16

(a) The only intercept along any coordinate axis is the origin (0, 0, 0).

(b) The surface is symmetric with respect to all three coordinate planes, all

Figure 16 three coordinate axes, and the origin.

(c) The hyperbola (_v"/9) - (z~/A) = k~/\b is the intersection with the plane
X = kiox k^'O. For k ¥= 0, the intersection with the plane y = k is the ellipse
(a:-/16) + (z'/4) = Ar/9. For A- t^ 0, the intersection with the plane ; = A: is the
hyperbola (yV9) - U-/16) = /t^/4.

(d) The yz trace consists of two intersecting lines, y = ±.\z. The xz trace
consists just of the origin (0. 0, 0). The xy trace consists of two intersecting
lines, y = ±4.v.

(e) The surface is an elliptic cone.

(f) The graph is sketched in Figure 16.


SECTION 13.8 QUADRIC SURFACES 819

EXAMPLE 4 Qv^ - 25z- = X

SOLUTION The equation has the form {y-/b-) - (z^/c-) = x, where b and

(a) The only intercept along any coordinate axis is the origin (0, 0, 0).

(b) The squared variables are v and z, so the surface is symmetric with respect
to the xy plane, the xz plane, and the a axis.
(c) For k <0, the intersection with the plane = k is the hyperbola .v

Figure 17 25z^ — = —k whose transverse axis is parallel to the z axis. For ^ > 0,
9v'
the intersection with the plane = k is the hyperbola 9v- - 25z- = k whose
.v

transverse axis is parallel to the v axis. The intersection with the plane v = k is

the parabola x = 9^-^ — 25z-, opening in the direction of the negative x axis.
The intersection with the plane z = k is the parabola x = 9}'" - 25A--, opening
in the direction of the positive .v axis.

(d) The yz trace is the pair of intersecting lines y = ±§2. The x: trace is the
parabola x = -25z-. The xy trace is the parabola x = 9y-.

(e) The surface is a hyperbolic paraboloid.

(f) The graph appears in Figure 17.

Problem Set 13.8

In Problems 1 to 8, identify the intersection of each quadric surface 16 X- + y' + z- - 2y + 2z =


with the indicated plane.
17 X- + 5y- - 8z- = 18 -X- - 25y- - 25z- = 25
1 2x^ + 3v' + z- = 6; =
—+
.V 1

19 3z = x^ + y^ 20 4v = y^

= 1; z^
25
21 — ^
16
-
9
= 3z 22 4y= - 9z- - 18a =
3z^-^-_=,;, = 2
23 ^2 ^ V + z- 24
4 3x~ + 4v- = z; a: = 2

5 25;c- + 4/ " lOOz" = 0; z = -1 25 Show that the graph of an equation is symmetric with respect to

6 3jc^ - 4z2 + 5y - z - 2 = 0; z =4 the .vy plane provided that when z is replaced by -z in the equa-
tion, an equivalent equation is obtained.
7 4x" - 16v^ = z; A- = i
26 Find the conditions for symmetry of a graph with respect to
8—--^ + X + V + XV = 4; X (a) the yz plane, (b) the .v axis, and (c) the y axis.
25
27 Find the condition for the symmeu^ of a graph with respect to

Problems 9 the origin.


In to 24, (a) find the intercepts, (b) discuss the symme-
try, (c) find the cross sections perpendicular to the coordinate axes,
28 Discuss the symmetry of the graph of the equation
(d) find the traces, (e) identify the quadric surface, and sketch a - =
(f)
Ixy + 3.VZ 4yz 24.
graph.
29 Find all cross sections cut from the ellipsoid
9 x- + 3y- + 2z- = 6 10 144.v' + 9v- + 16z- = 144 (x~/a^) + {y-/b^) + (z^/c^) = by (a) the 1 planes x = k. (b) the

- 9r + = 36 planes y = k, and (c) the planes z = k.


11 4x- 9z- 12 x~ + 2xy + / = 1

- - = 30 Find all cross sections cut from the hyperboloid of one sheet
13 X- 4y^ 4z- 4 14 z- = 1 - 2y + y^
(x-/a^) + (y'/b-) - (:'/c~) = 1 by planes perpendicular to the
IS y^ - 9x' - 9z- = 9 coordinate axes.
820 C HAPIEK I.' COORDINATE SYSTEMS IN THREE-DIMENSIONAL SPACE

31 Find all cross sections cut from the elliptic cone 36 Prove that if (.«o. .Vo. 'o) is any point on a hyperboloid of one
(x'/a~) + (y'/b') - (z"A'") = by planes perpendicular to the sheet, there are two lines in space, passing through (.Vq. Vq. Zo).

coordinate axes. both of which lie on the hyperboloid.

32 Find all cross sections cut Irom Ihc elliptic paraboloid 37 Find an equation that describes the surface consisting of all

(x'/a') + (.y'/b') = z by the planes perpendicular to the coordi- points P = (x, y, ;) whose distance from the y axis is 3 of its

nate axes. distance from P to the .v: plane.

33 Find all cross sections cut by planes z = k from the hyperbolic 38 Prove that if (.vq, Vo, zo) is any point on a hyperbolic paraboloid,
paraboloid iy'/b') — {x'/a') = :. Discuss the cases k < 0. there are two lines in space, passing through (.Vd. Vo, za). both of
k = 0, and k > separately. which lie on the hyperbolic paraboloid.

34 Find all cross sections cut by planes perpendicular to the a or v 39 A central quadric surface Ax- + By- + Cz" + A: = contains
axes from the hyperbolic paraboloid (y'/b') - (x^/a') = z. the points (3. -2. -I). (0. I. -3), and (3. 0. 2). Find the
equation of the surface, and identify the surface.
35 Write an equation that describes the surface consisting of all

points P = (x. y. z) such that the distance from P to the point 40 A quadric surface Ax- + By + Cz = contains the points

(0, 0, — is the same as the distance from P to the plane c =


I ) I (1,0, I) and (0. 2, 1). Find the equation of the surface, and
Identify this surface. identify the surface.

13.9 Cylindrical and Spherical Coordinates

In Chapter 9 we found that some problems in the plane were easier to formulate and
to solve if polar coordinates, rather than Cartesian coordinates, were used. Simi-
larly, there are situations in which problems in three-dimensional space become
more tractable if non-Cartesian coordinate systems are introduced. Two of the most
important non-Cartesian coordinate systems in space, the cylindrical and the
spherical, are discussed in this section.

Figure I
Cylindrical Coordinates

In Section 13.1 we obtained the Cartesian coordinates of a point P in three-dimen-


sional space by dropping a perpendicular PQ to the horizontal plane through the
origin O and using the Cartesian coordinates of Q in this plane together with the
directed distance z = il/'Cl- The cylindrical coordinate system is quite similar,
except that polar coordinates are used for the point Q in the horizontal plane (Fig-
ure 1). Thus, the cylindrical coordinates of the point P in Figure 1 are (r, 6. z).

Figure 2 shows the point P with respect to the Cartesian coordinate system and
with respect to the cylindrical coordinate system. If the polar coordinates of Q (with
the X axis as the polar axis) are (r, 0). then the Cartesian coordinates of Q are
X = r cos d and v = r sin 6: hence, the Cartesian coordinates oi P are given by the
equations

{;- y, z) Cartesian
0, 2) cylindrical
r cos d

Since the point Q at the foot of the perpendicular from point P to the .r>' plane has
an unlimited number of different representations in the polar coordinate system, it

follows that P has an unlimited number of different representations in the cylindrical


coordinate system. For instance, if P = (r. 6. z). then also P = (-;, 6 + n. z). In
any case, if P = (x, y, ;) in Cartesian coordinates, then the cylindrical coordinates
SECTION 13.9 CYLINDRICAL AND SPHERICAL COORDINATES 821

Figure 3

Figure 4
822 CHAFFER 13 COORDINATE SYSTEMS IN THREE-DIMENSIONAL SPACE

EXAMPLE 3 Write an equation in cylindrical coordinates of the right circular cyl-


inder of radius 17 having the z axis as its central axis.

SOLUTION An equation of the right circular cylinder is r = 17.

F.\ AMPLE 4 Find the equation in cylindrical coordinates for the paraboloid of
revolution whose Cartesian equation is .v" + y = :.

SOLUTION Since r- = x~ + y~, the desired equation is r^ = z.

EXAMPLE 5 Find an equation in Cartesian coordinates of the surface whose equa-


tion in cylindrical coordinates is r = 3r. identify the surface, and sketch its graph.

Figure 7 SOLUTION Squaring both sides of the equation, we obtain "= 9r-, or z~ =
9(.v" -f- y-). The last equation can be rewritten as .v" + \~ — (z~/9) = 0. which rep-
resents an elliptic cone. In fact, since the coefficients of j^r and y- are the same, the
graph is a circular cone (Figure 7).

A curve in space can often be expressed parametrically by giving the cylindrical


coordinates (r, 6, z) of a point P on the curve in terms of a parameter /. Thus, if

=m
= g(')

= hU)

where/, g, and /; are continuous functions, then the point P = (r. 0. z) traces out the
curve as t varies. If/', g' and h' exist and are continuous, then from the equations
,

X = r cos and

we obtain

—=—
dx
dt
dr

dt
cos d — r sm 6
de
dt
and — dr

dl
sin -I- r cos
de
dt

Thus,

\dt' ^ dt' ^dt' \dt'

diy-
'-[ (Problem 39)
'
dt \dt

It follows that the arc length of the curve between the point w here the parameter has
the value t = a and the point where it has the value t = b is given by

-vf\/(?)M?)M¥r^
13 EXAMPLE 6 Find the arc length of the curve

ir = 3/

between the point where / = and the point where r = 1


SECTION 13.9 CYLINDRICAL AND SPHERICAL COORDINATES 823

SOLUTION Here dr/dl = 0, dS/dt = 2ir, dz/dt = 3, and

s = V'O- + 52(2-77)- + 3- dt = VIOOtt^ + 9 dt


'o 'n

= VIOOtt + 9 dt = VIOOtt + 9 = .56 units


31.1
J

Spherical Coordinates
Figure 8 In the spherical coordinate system, the angle 6 has exactly the same meaning as it

does in the cylindrical coordinate system. Thus, 6 locates the point P on a plane
containing the : axis and making the angle with the positive x axis (Figure 8). The
distance between P and the origin O is denoted by the Greek. letterp (called "rho");
thus, p = \0P\. Finally, the angle from the positive z axis to the line segment OP is
denoted by the Greek letter 4) (called "phi"). The spherical coordinates of the point
P are customarily written in the order (p. 6, </>), and they are usually chosen so that

0< e< 277- and < (^ < 77

A point P with spherical coordinates (po. Oq. </>o) is po units from the origin;
hence, it is located on a sphere of radius p,, with center at O (Figure 9). The z axis
intersects this sphere at the "north and south poles," and the .\y plane intersects it

in the"equator." Semicircles cut by half planes passing through the north and
south poles are called meridians, and the meridian that intersects the positive v axis
is called the prime meridian. The angles 0o and (i>o locate Po on the surface of the
sphere, as shown in Figure 9. The spherical coordinate 6q. which is called the
longitude of Pq. measures the angle between the prime meridian and the meridian
passing through Pq. (On the surface of the earth, the meridian passing through
Greenwich, England, is designated as the prime meridian.)
Circles cut on the surface p = po by planes perpendicular to the ; axis (hence,
parallel to the equatorial plane) are called parallels, and the angle measured from
the equator to a parallel is called the latitude of that parallel (or of any point on the
parallel) (Figure 10). Notice that the point Pq with spherical coordinates (po, do, (/>(,)

has latitude (7r/2) - (po'. in other words, the angle 0o is the complement of the
latitude of Pq- For this reason, (t>o is called the colatitude of Pq- For instance, the
latitude of Boston, Massachusetts, is approximately 42.4°, so its colatitude is ap-
proximately 47.6°.

Figure 9 parallel Figure 10


containi

parallels

Equator
Equator

meridian
containing
824 CHAPIKR I? COORDINATE SYSTEMS IN THREE-DIMENSIONAL SPACE

Figure II In Figure 1 1 , suppose that the point P has Cartesian coordinates (.v, y, z) and
spherical coordinates (p, 6. <^). The line segment PQ is parallel to the z axis, so that

angle OPQ is equal to </>. Since OQP is a right triangle.

\OQ\
sin =
\0P\ p

Hence, r = p sin <^

Therefore, we can rewrite the equations

X = r cos and y = r sin 6*

as X = p sin </> cos and y = p sin c/) sin ft

Triangle 0/?P in Figure 1 1 is a right triangle, so that


Q = {.x,y.Q)

cos <p =
\0R\
,
,
= —
z

\OP\ p
Hence, - = P cos (j)

The equations just derived.

A = p sin cos « p sin (b sin p cos <i>

give the Cartesian coordinates of the point P whose spherical coordinates are
The argument just given applies if P lies above the at plane, and (Problem
(p. 6. 4>)-
38) the same equations are effective even when P lies on or below the at plane. In
some applications of spherical coordinates, the conditions p > 0, :S ft < Iv, and
< (^ < 77 are dropped, and (p, ft, </>) is understood to locate P in spherical coordi-

nates if A, y, and z are given by the equations above.


Since p is the distance between P = (a, y, z) and the origin O,

p- = A" + y + z'

Also, if A ^ 0. then

y sin ft

X cos ft

Finally, if p t^ 0, then

Therefore, if we choose the spherical coordinates (p, ft, <!)) so that p s 0,

< ft < 277, and < (i) s 77, we have the formulas
SECTION 13.9 CYLINDRICAL AND SPHERICAL COORDINATES 825

Figure 12 SOLUTION Here we have p = 2, d= tt/3. and <f)


= 277-/3. so that

^i'Mf ,§.-.)

Figure 13
826 CHAPTER 13 COORDINATE SYSTEMS IN THREE-DIMENSIONAL SPACE

Thus, the equation in Cartesian coordinates is x^ + _v" = z~: hence the graph of
the equation is a right circular cone.

(b) In cyhndrical coordinates r- = x' + y'. so the equation obtained in part

(a) can be written as n = z~.

EXAMPLE 10 Rewrite the equation of the paraboloid .v" + y- = z in spherical co-

ordinates.

SOLUTION Here we have

X- + y- = (p sin </> cos d)- + (p sin <i) sin Q)'


= p- sin" <^{cos- + sin" 6) = p" sin" </>

Thus, since z =p cos <^, the equation .v" + v" = z becomes

p~ sin" (f>
= p cos <t>

We lose no points on the graph by canceling p to obtain p sin" (/> = cos 4> since the
latter equation is still satisfied when p = and (t> = tt/I. Therefore, in spherical
coordinates, the paraboloid has the equation p sin" 4> = cos cj).

Problem Set 13.9

In Problems 1 to 4, find the Cartesian coordinates of the point whose 17 ; = 2(.v" -I- y^) 18 x^ + y- - 4.V =
cylindrical coordinates are given, and plot the point.
19 .V = 2 20 y = V3.r

1 (4, 7r/3, 1) 2 (3. tt/2. 4) 21 x~ + v" = 5z- 22 .V- +y- + (z - 1)- = 1

3 (5, 77/6. -2) 4 (2, 2, 2) 23 .v= + y- = 25 24 -v + ly =

25 .v= -t- V- - r- = 26 x- + y- + :- -62 = Q


In Problems 5 to 8, find cylindrical coordinates (r, 0. :) with / s I

and s 8 < 2w for the point whose Cartesian coordinates are


In Problems 27 to 32. each equation is written in cylindrical coordi-
given, and plot the point.
nates. Convert each equation to an equivalent equation in

(a) Cartesian coordinates and (b) spherical coordinates. Sketch a


5 (4, 0, 1) 6 (-2V3, -6, 0)
graph of the surface.
7 (-3V3, 3, 6) 8 (1, 1, -1)

27 28
In Problems 9 to 12, find the Cartesian coordinates of the point

whose spherical coordinates arc given, and plot each point.

29
9 4
30 r = —2 r- sin IS
9 (2, 7r/6, 7r/3) 10 (7, 77/2, 77)

11 (12, 577/6, 277/3) 12 (77, 77, 77) 31 r = 4 cos e

32 r cos e + 3r sin -t- 2z = 6


In Problems 13 to 16. find spherical coordinates (p. 6. </>) with
p 2 0, s < 277, and £ <^ S tt for the points whose Cartesian
each equation written in spherical coordi-
In Problems 33 to 37. is
coordinates are given, and plot each point.
nates. Express each equation as an equivalent equation (a) in Carte-
sian coordinates and (b) in cylindrical coordinates. Sketch a graph of
13 (0. -I, 0) 14 (0. 0, 5)
the surface.

15 {I, 2, -3) 16 (0, 0. 0)


33 p = 2 34 = 77/3

In Problems 17 to 26. convert each equation to an equivalent equa- =


35 p sin </) = 3 36 p 2 cos
tion (a) in cylindrical coordinates and (b) in spherical coordinates.

Sketch a graph of the surface. 37 (^ = 77/3


REVIEW PROBLEM SET 827

38 Show that the equations given on page 824 for converting from 41 Find the arc length of the curve whose equation is given para-
spherical to Canesian coordinates are correct even when the metrically in cylindrical coordinates by
point P lies on or below the at plane.

0=V2t and
39 Complete the proof of

d0\-
dt

on page 822.

40 Consider the curve in space whose equation is given para-


metrically in spherical coordinates by p—fit). 8 = git), and
<t>
= h(i). where/', g', and h' e.xist and are continuous,
(a) Prove that
828 C HAPIKR 1.1 COORDINATE SYSTEMS IN THREE-DIMENSIONAL SPACE

31 Do the vectors A = i
- 2j + 3k. B = 3i + 2j + k, and 49 Find the distance from the point (2, -3, 4) to the plane
C = 2i + 3j form a right-handed triple A, B, C? Why? Ir - 2y + - = 5.

jjij^ind the area of the triangle whose vertices are >4 = (2, 0. - 1 ).
50 Find the distance between the two parallel planes - 2y +
.v

^fi = (5, 3, 3). and C= (-1. 1. 2). 2z = 5 and x - 2y + 2z = 17.

In Problems 33 lo 40, find an equation in scalar form of the plane 51 Show that the lines

satisfying the conditions given.


A - 100 V + 94 z - 51
33 Containing the point f = (1. 2. 3) and perpendicular to the ra- 99 -97 50
dius vector OP at the point P
A - 102 V -I- 96 52
'yMjConlaining the three points (1,7, -1) and
2), (3. 5, 1), and (6, 3, -101
35 Containing the point P = {2, Q. -1) and perpendicular to the intersect, and find an equation of the plane containing the two
line joining the points Q= (3, 4, 4) and /? = (-1, 2. I) lines.

\36jAt a distance of 4 units from the origin and perpendicular to the 52 Given the equations
line joining the points P = (-2, 3, 1) and Q= (-5, 1, -5)
A - A(, y - yn z - Zq
37 Containing the point (4, 3, I) and parallel to the plane
.V -(-
3; = 8
of a line, explain how you would go about finding several points
^jContaining the points (1, 3, 1) and (4, 6. -2) and perpendicular
on this line.
to the plane .v -f- \' — ; = 3
53 Find the distance between the two lines
39 Containing the point (2, -1, 3) and perpendicular to the line
x = 3t + 5, y = 8/ - 4, z = -It + 16
= =
A- 3 v+ 1

A V + 3 and
\40]Containing the point (1,2, 10) and the line

54 Let P be a variable point on a first line in space, and let (2 be a


A - 1 V - 1 z - 6
variable point on a .second line in space. If M, and Mi are direc-
-7
tion vectors for the first and second lines, respectively, show
that the quantity (Mi x Mi) • PQ remains constant as P and Q
In Problems 41 to 48, find the equations of the line (a) in scalar
vary.
parametric form and (b) in symmetric form for which the given
conditions hold. 55 Show that the lines

41 Containing the points -4) and A - .xo y - Vn z - zo


(5, 6, (2, -I, 1)
On ^0
42 Containing the point (-1, 2, 3) and perpendicular to the plane
-2r + 3.V -z = -1
43 Containing the point (2, -3, -2) and parallel to the line

v - 1 V + 7 z

3 -2 7

|44) Containing the origin, perpendicular to the line of intersection of


the two planes x = y — 5 and z = 2y — 3, and meeting the line
of intersection of the two planes y = 2v -(-
1 and z = .v -I- 2

45 Formed by the intersection of the two planes Zv + y - z = 1

and .* - y + 3z = 10

§6) Containing the point (3, 6. 4), parallel to the plane .v - 3y -I-

5z - 6 = 0, and intersecting the z axis

47 Containing the point (2, 1,4) and perpendicular to both the a and
the y axes

48 Containing the point (2, -I, 4) and perpendicular to the lines

.V = 5/ + 1 , y = -3/, 2 = t - 2 and x = 2i - 1 , y = / -t- 1

z = -t
REVIEW PROBLEM SET 829

• STlSuppose A is a known nonzero vector, that X is an unknown


that part (a) as its base curve, (c) Write the equation of the cylinder in

vector, but that the scalar a = A X and vector B = A x X are • part (b).

both known. Show that X is then determined by the equation


69 Find an equation of the surface of revolution generated by rotat-

a A X B ing the curve z = 2(x — 3)' in the xz plane about the x axis.

^~W^ |AP
70 Find an equation of the torus (doughnut-shaped surface) that

(Hint: Start by expanding A X B.) results when the circle y" + (.v — a)- = r- (,a > r > 0) in the
xy plane is revolved about the y axis. Sketch the surface.
58 If A is a nonzero vector and A x X = A x Y, can we
"cancel" and conclude that X = Y? Explain. 71 Find the generating curve in the xz plane and the axis of revolu-
tion of the surface of revolution y" -I- z^ = e'^^. Sketch the sur-

d face.
In Problems 59 and 60. find F'U). ¥"{t). and -- .

F(f) .

(It
72 Find an equation of the surface of revolution generated by
59 F(f) = e-'''i - e"-''j + tk revolving the cardioid r = \
- cos 6 in the xy plane about the
X axis.

60 r(/) = tan yt + — ji + (tan f)j + tan [t - — )k


In Problems 73 to 78. identify and sketch a graph for each quadric
surface.
In Problems 61 and 62. assume that a particle P moves according to
the given equation of motion. Find (a) the velocity vector V, (b) the 73 .V- + 4y- + 4z- = 16 74 .V- + 4y- -I- 4z- = 16x
acceleration vector A. (c) the speed v. (d) the unit tangent vector T.
75 .V- + :- = 4 + y 76 9.V- -I- z^ = y
(e) the unit normal vector N. (0 the unit binormal vector B. and
(g) the distance traveled by the particle along its path from the in- 77 V- - :- + 9.V- = 1 78 V- - z- + 9a-- =
stant / = to the instant t = \.

79 A quadric surface has an equation Ax' + By + Cz = 0.


61 R= (3 cos 27r?)i + (3 sin 27rr)j + 2rk Identify the surface if it contains the points (3. 5. 8) and
(4, -2. -6).
62 R = (?" cos r)i + (e-" sin f)j + e'*'k

80 A quadric surface has an equation Ax' + By- + C:' = I and


In Problems 63 to 66. find (a) V, (b) v, (c) A. (d) T. (e) V x A, contains the points (2. -1, 1). (-3. 0. 0). and (1, -1. -2).
(f) K. (g) N. (h) B, (i) d\/dt, and (j) t for each curve in xyz space. Identify the surface.

"e^R = ii + r-j + t\ 81 Describe and sketch the surface whose equation in cylindrical
coordinates is (a) r = 2; (b) = 7r/6; (c) r = sin 8.
64 R = an + bt-j + ct\, where a, b, and c are constants
82 Describe and sketch the surface whose equation in spherical
65 R = (sin / cos t)'\ + (sin- Oj + (cos t)V. coordinates is p = 5 cos <j).

~BB R = (f sin f)i + (/ cos /)j + tV. 83 Find an equation in cylindrical coordinates of the surface ob-
tained by revolving the curve z = fix) in the xz plane about the
67 A particle moves according to an equation of motion R = F(r). z axis.
Expand and simplify the following expressions:
84 Find the arc length of the curve whose parametric equations in

(a) --(R-V) (b) --(R X V) spherical coordinates are p = f". = 7r/6. 4> = t as t varies over
dt dt
the interval [0, VS].

(c) --(V-A) (d) --(V X A) 85 Convert the equation p" sin 2(/) = 4 in spherical coordinates to
dt dt
Cartesian coordinates, and sketch the graph of the surface.
dJ dH _rfR_ d-R d^R
(f) [c]86 Find the great-circle distance between Honolulu, with latitude
ds ds- ds'
2l.3rN. longitude 157.87°W, and Chicago, with latitude

68 (a) Sketch the curve z = cos (7n) in the yz plane, (b) Sketch the 41 .83°N. longitude 87.62°W. Assume that the radius of the earth
cylinder with generators parallel to the .v axis having the curve in is 3959 miles.
m
PARTIAL DIFFERENTIATION

In the preceding chapters we dealt exclusively with functions of a single real vari-
able; however, there are many practical situations in which functions depend on
several variables. For instance, the frequency of a tuned circuit depends on its

capacitance, its inductance, and its resistance; the pressure of a gas depends on its

temperature and its volume; the demand for a commodity may depend not only on
its price but also on the prices of related commodities, on income level, and on time;
a person's income tax liability depends not only on income but also on several
itemized deductions and the number of dependents; and so forth.
In this chapter we study functions of more than one variable, we see that the
concepts of limits and continuity are applicable to such functions, and we investi-

gate their "partial" derivatives. Chain rules are developed for functions of several
variables. The chapter also includes a study of directional derivatives, tangent
planes and normal lines to surfaces, and maxima and minima of functions of several
variables.

Functions of Several Variables


A right circular cylinder, closed at the top and bottom, with base radius r and height
h, has total surface area S given by

S = 2nrh + Inr-

Here we say that the (dependent) variable S is a function of the two (independent)
variables r and h. and we write

S=f(r,h)
For example, if r = 11 cm and /i = 5 cm. then

5=/(ll, 5) = 27r(ll)(5) -I- 277-(ll)- = 3527r cm-


The graph of a function of two variables
is a surface. Proceeding somewhat more formally, we make the following definition.

830
SECTION 14.1 FUNCTIONS OF SEVERAL \ARIABLES 831

DEFINITION 1 Function of Two Variables

A real-valued function j of two real variables is a rule thai assigns a unique real
number r = fix. y) to each ordered pair (x. y) of real numbers in a certain set D
called the domain of/.

In the equation z = fix. y). we call : the dependent variable, and we refer to x
and y as the independent variables. The set of all possible values of r that can be
obtained by applying the rule/ to ordered pairs (.v, y) in D is called the range of the
function/. If /is defined by an equation, or a formula, then (unless we make a
statement to the contrary) the domain of/ is understood to be the set of all ordered
pairs of independent variables for which the equation, or formula, makes sense.

Figure 1 =
V4
EX-'VMPLE 1 Find and sketch the domain of /(.v. v)
y

SOLUTION The domain of / consists of all ordered pairs (.v. y) for which
x^ + y^ < 4 and y # 0. This is the set of all points not on the x a.xis that are either on
the circle x- + y^ = 4 or in the interior region bounded by the circle (Figure 1).

sin" :
EXAMPLE 2 Find the domain affix, y. r)
= '

SOLUTION Since sin '


z is defined only when |-| s 1, the domain of/ consists
of all ordered triples (x, y, z) such that .v +y ¥= and |r| < 1

We define the graph of a function / of two variables to be the graph of the


equation z =/(.v. y): that is. the set of all points (.v, y, r) in Cartesian three-
dimensional space such that {x. y) belongs to the domain D of/ and z = /(.v, y). The
domain D can be pictured as a set of points in the .xy plane and the graph of/ as
a surface whose perpendicular projection on the .\y plane is D (Figure 2).* Notice
that as the point (x, y) varies in D, the corresponding point

(X. y. z) = (X. y, f(x. y))

varies over the surface. The methods introduced in Sections 13.7 and 13.8 are
useful for sketching graphs of functions of two variables.

Figure 2

*In Figure 2. the point shown as (.r. y) in D is really (.v. y. 0): however, the third coordinate
has been purposely omitted.
832 CHAPTER 14 PARTIAL DIKKERKNTIATION

Figure J In Examples 3 to 5. sketch the t^raph of each function of two


variables.

EXAMPLE 3 fix. y) = 1 - .V - (.v/2)

SOLUTION The domain D of/ is the entire .vv plane. The


point (x, v. ") belongs to the graph of / if and only if

z = 1 - .V - 0/2): that is.

It + y + 2z = 2

Therefore, the graph of/ is a plane with intercepts (1,0, 0),


(0, 2. 0), and (0, 0, 1). A portion of this plane, showing
its traces with the .vv, x:. and y: planes, appears in Fig-

ure 3.

EXAMPLE 4 /(.v, y) = Vl - .v" - v"

Figure 4
SOLUTION The domain D of/ is the circular disk consist-
ing of all points (a, y) with x- + y^ -^ 1 . A point (x, y, i)
belong s to the grap h of/ if and only = /(.r, y), that is,
if z

z = Vl — X- — v". The condition z = Vl — x^ — y" is

equivalent to the two conditions r s and x~ + y~ + z' = 1

Thus, the graph consists of the portion of the sphere


.V- -I- y- -I- z- = 1 lying on or above the xy plane
(Figure 4).

EXAMPLE 5 g(.v. y) = .V- + 3y-

SOLUTION The domain D of g is the entire .v>^ plane, and the graph of g is the
graph of the equation
A- -I- 3v-

By the discussion in Section 13.8 (page 816), this graph is an elliptic paraboloid
(Figure 5).

Functions of three or more variables are defined by an obvious extension of


Definition 1 as follows.

DEFINITION 2 Function of Several Variables

A real-valued function /of n real variables is a rule that assigns a unique real

Figure 5 number \i' =/(V|, as, a, v„) to each ordered /(-tuple (Ai, .xs, a^ a„)
of real numbers in a certain set D called the domain of/.

In the equation vv = /(a i,A2,.V3, v„). we call \i the dependent variable, and
we refer to a, , a^, .vj a„ as the independent variables. The set of all possible
values of ir that can be obtained by applying the rule /to ordered /i-tuples in D is

called the range of the function/. In case n = 2, w =/(ai, .xs) is usually written
z =f(x, y) as in Definition 1. In case /; = 3, ir =/(ai, At. a^) is usually written
w =f(x, y. z).

EXAMPLE 6 Ifg(A. y. r) = for all values of a. y. and z except those


A- + V'

that make the denominator equal to zero, find

(a) ;i,'(2, 3, 7) (b) ^'(sin t. cos /, 0)


SECTION 14.1 FUNCTIONS OF SEVERAL VARIABLES 833

(2)(3)
(a),(2,3.7, = ^,-^^^-^=l

(b) ?(sin t. cos /, 0) = —


sin~
;
sin

t +
r

COS"
cos
;
t
t


= sin / cos t

EXAMPLE 7 If/(.vi, .V2. .V3 .v„) = .rj + .v5 + .x^ + • • • + A^ for all values of

Xi, .vi- -^3. • • • . x,,- find /(I. 2, 3, ... , /!)•

SOLUTION Using the formula for the sum of successive squares (Section 5.1),
we have
/(I, 2, 3, ... , n)= \- + 2- + 3- + •+ n- =
^^
Z, Ir
,
= nin + l)(2/i
Z
+ 1)

Scalar Fields

We have seen that a function / of two independent variables can be thought of in


terms of its graph, which is a surface in .nr space. There is a second way of

picturing such a function, which, for some purposes, is even more suggestive;
namely, the function /is regarded as a scalar field. The idea is as follows: The
domain D of/ is visualized as a set of points (x, y) lying in a certain region in the

xy plane, and each point {x. y) in this region is assigned a corresponding scalar

f(x, y) by the function / (Figure 6). The scalar value /(.v, y) corresponding to the
point {x, y) in D is shown in Figure 6 on a "flag" attached to the point. As the point

{x, y) is moved around in the region D. the flag moves with it, and the number
f(x, y) on the flag changes.
The scalar /(A-, y) assigned to the point (v, y) might represent, for instance, the
temperature at (.v, y), or the atmospheric pressure at {x. y), or the wind speed at
(.V, V), or the intensity of the magnetic field at (.v. y), and so forth.

Figure 6 Figure 7

//tx. v)/ domain


834 CIIAKIKR 14 PARTIAL DIKKKRKM lATION

60 75
(a) /(60. '5' = 80 - ^ ^ - = ^^"F

Figure 8 =
(b) An equation is /(.v, y) 70: that is.

80 = 70 5.x + 4v = 1000
20 25

(c) The curve 5.v + 4v = 1000 is a line (Figure 7).

A curve along which a scalar field has constant value (such as the cui^e along
which the temperature Field in the previous example maintains the constant value
70°F) is called a level curve of the field or of the function/that describes the field.
An equation of the level curve along which the function/assumes the constant value
k is

tiirtt >;<^R Yofiecusi \ \^'


S«\ is !in 1983

Level cunes for various scalar fields: The level curves for various scalar fields are customarily given special names de-
ta) equipolenlial lines for the electric field pending on the nature of the field isotherms for the level curves of a temperature
of a dipole: (b) Computerized Tomography field, equipotential lines for the level curves of an electric potential field, and so
scan of a chest section of a patient
forth. Figure 8 illustrates a number of practical applications of level curves in fields
undergoing radiation therapy showing
ranging from physics to meteorology.
isodose cunes (level curves for radiation
Suppose that the function / gives the height z =/(.v, y) of a certain surface 5
absorbed): (c) weather map showing
above the .vy plane (.v, y). (Then S is the graph of the function/.) The
at the point
isotherms (level cur\es for maximum
temperature).
intersection of the surface 5 with a horizontal plane z = k produces a curve C
consisting of all points on the surface that are k units above the .xy plane (Figure 9a).
The perpendicular projection of the curve C on the .vy plane gives a level curve for
the function y. Such a level curve, whose equation in the .vy plane is

/(.v, y) = k

is called a contour curve for the surface S. By plotting a number of different

contour curves, each labeled with its own value of A', we obtain a contour map of
SECTION 14.1 FUNCTIONS OF SEVERAL VARIABLES 835

Figure 9

)lx.y) = k

(a)

the surfaceS (Figure 9b). Such a contour map enables you to visualize the surface as
if you were looking down upon it from above and seeing its intersections with
horizontal cutting planes at various heights. If these heights are made to differ by
equal amounts, then a crowding of successive contour curves indicates a relatively
steep part of the surface. Figure 10 shows a contour map of Dutch Flat, N.W.,
Arizona.

Figure 10 Figure 11

Not only are contour maps used for representing the surface features of a geo-
graphs of
graphical region, but they are also used to help us visualize complicated
surface
mathematical functions. Figure 1 1 shows a computer-generated graph of a
2 = f(x, v) and its corresponding contour map.

EX.\.MPLE 9 Let 5 be the surface z = .x" - x' + 20 for .v > 0. Plot the contour

curves for this surface* corresponding to z = 0, r = 10, r = 20, r = 30, and z = 40.

SOLUTION For z = we obtain Q = x- - y- + 20, or r - .v" = 20, the equa-

tion of a hyperbola with a vertical transverse axis. Since x > 0, we get only the

and fourth quadrants as the contour


portion of this hyperbola lying in the first

*Note that this surface is a portion of a hyperbolic paraboloid.


836 HIAPTER 14 PARTIAl. DIKKKRKM ATION
I

Figure 12 curve corresponding to : = 0. For r = 10, we obtain \() = x~ — + 20, or


>•"

y- - X- = 10, another hyperbola. For : = 20, the equation is 20 = - y- + 20.


.v"

or X = ±y, two lines meeting at the origin. Continuing in this way, we obtain
the desired contour map (Figure 12).

The idea of a field as a continuous spatial distribution of a physical quantity was


originally conceived by the English experimental physicist Michael Faraday
(1791-1867). Faraday's brilliant conception had a powerful impact on the later

work of James Clerk Maxwell (1831-1879).


the English mathematical physicist
who developed the fundamental equations for electromagnetic fields. The concept
of a field has become more and more important in physics indeed, many modem —
physicists regard physical reality as a network of interacting fields.

Michael Faradax James Clerk Maxwell

Problem Set 14.1

In Problems 1 to 14, evaluate each expression, using the functions/. 11 /(.v. v) + ,i^(.v, y) 12 /(A + luy) -fix.y)
g, and h defined by /(.v, = S.v" +
y) 7,vy, g(x. y) = Vxv, and
-
gix. y + *:) gix. y)
h(x. V, r) = (IxT + z)/(.v- + V- - Z-). 13 /;(.v. v, 0) 14

1 /(3, -4) 2 /(2, 0)s(2, 0)

3 g(k, k) 4 i>(a-. h-) In Problems 15 to 26, find and sketch the domain of each function.

5 h{l, 2, 3) 6 /?(.\, :. y)
15 /(A, y) = —Vv- +
'
y-
-
- 9
16 gl\. y) = .vVl -
/ T
x' -
5
y-
7 fiVa. b) 8 ,i;(sin f). 2 cos «)

/(-v.
9 /)(sin I. cos t, 0) 10
y)
17 «(A, V) =
[g(x, y)Y Vx^ + y^ - 16
SECTION 14.2 LIMITS AND CONTINUITY 837

V36 - .V- - V- 44 fix. y) = V9 - A- - y-; ; = 0. z = 1, z = 2. z =


18 Fix. y) 19 /(.v, v) = In (A- - y)
45 fix. y) = A- - y-; z = -2. z = - 1 , z = 0, z = 1,

20 G(.v. y) = In (4 - .v - v") 21 gtx, y) = V.T + y - 4 = = = = =


46 fix. y) at; z 0, z 1, z 2, z 3

= ^ = - 47 fix. y) = A + y-; z = - z = 0. z = z = 2
22 h{x. X)
,
V36 - 4.r^
.
- =T
9y-
23 h(x. y) In (at 2) 1 . 1 ,

2a
= 48 fix. y) = 0, z = 1. z = 2, z = 3
24 /{.v. v) sin"' (Iv + y)
-,
,

4.v~ - v~ A- + 9y-
25 f (A, 26 Fix. =
V)
2v - V
y)
9v-
49/(.r. y)= ^1 f.^o.=.,,.4
In Problems 27 to 40. find the domain of each function of two A + V
50 fix. y) = ^/
,

—. z = 0. 1. z = 2. z
variables, and sketch the graph of the function.

27 /(A. y) = A + y 51 Suppose that the electric potential V in volts at a poi nt (a. y) in

the xy plane is given by V(a, y) = 4/V9 — a' — v". The level


28 gtx. y) = It - 3y + 6
curves of V are called equipotential curves. Plot the equipoten-

29 /i(A. y) = V4 - A- tial curves corresponding to V= 16, V= 8, V= 4, and V= 2-

30 ;;(A. y) = 3 + V9 - a" 52 Suppose that the temperature T in degrees Celsius at a point


(A. y) in the xy plane is given by T = Zx- + y". The level curves
31 Fix. y) = A- + y- 32 fix. y) = V I + A- + y-
of T are called isotherms. Plot the isotherms corresponding to

33 /(A, y) = 4 34 g(A, y) = A- r = 6. 7 = 4. 7 = 2. and r = 0.

35 /(a. y) = Va- + y- 36 ;i(a. = -Va- + y- 53 What does it mean if the isotherms (level curves) of a tem-
y)
perature field tend to crowd together near a certain point
37 //(A. y) = 4- A- - y- 38 gix. y) = y-
Pr, = (.Vo. yn)?

39 fix. y) = -2y + 1 40 fix. y) = Va- + y- - 1


54 Given amap showing the isotherms of a temperature field, ex-
plain how you would plot a path, stealing at a point Pq. along
41 If /(a,, .tt, .V3 -Vn) = Ai + .ti + .r3 + • • + x„. evaluate which the temperature would increase most rapidly.
and simplify /(I. 2. 3, ... . n).
55 A person is driving at 50 miles per hour from left to right
[342 The present value P of A dollars to be paid 1 years in the along the line y = 75 in Figure 7. How rapidly does this person
future (assuming an 8 percent continuous interest rate) is
feel the temperature dropping, in degrees Fahrenheit per hour,
PiA. t) = Ae""'^'*'. Find and interpret P(100, 13.8). if the temperature at the point (a, y) is given by fix. y) =
80 - (.v/20) - (y/25)?
In Problems 43 to 50. plot a contour map of the graph of j = fix. y)
showing the contour curves corresponding to the given values of :.
56 Look up the following words in a dictionary, and explain how
each refers to the level curves of a certain scalar field; (a) isobar,

43 /(a, y) = 3a + 2y - 1; z = -1, \,z = l (b) isocheim, (c) isoclinal line, and (d) isodynamic line.

14.2 Limits and Continuity


The idea of a limit can be extended to functions of two or more variables. For
instance, if we say that/(.v, y) approaches the limit L as (.v. y) approaches (.vq, Vq).
we mean that the number /(.v, y) can be made to come as close to the number L as
we wish by choosing the point (.v, y) to be sufficiently close to the point (.Xq. >o)-
provided that (.v, >) ¥= (xq, Jo)- The notation is

lim f{x, v) =L
838 CHAPTER 14 I'ARllAl. DIFFERENTIATION

As a specific example, notice that

'""
(x,v)— (0.1)1
/
V4 -
1

^
-
==- = —V4^ = —21 1

X- V-

since the quantity 1/V4 - a" - 3- comes closer and closer to I as the point (x. y)
comes closer and ckiser to (0, 0). Later in the section we give a formal definition of
a limit of a function of two variables: however, for now. we prefer to proceed
informally.
It can be shown* that all the properties of limits of functions of single variables
extend to functions of several variables; for instance, the limit of a sum. difference,
product, or quotient is the sum, difference, product, or quotient of the limits, re-

spectively, provided that these limits exist and that zeros do not appear in denomi-
nators. Also, if /(v. y) depends only on v and not on y, so that

/(A, y) = /7(.Y)

then lim /(v. v) = lim h{x)


|.<..V)-»(-l0.,V0) A— AU

provided that the last limit exists. We abbreviate this by writing

lim /)( a) = lim h(x)


(A.V)— (Am..V,,I A-»Vfl

Similarlv. we have ,.
lim g(\) = ,.
lim g{\)
(t.y)— (.w.yo) v^yo

provided that the last limit exists.

In Examples 1 and 2, evaluate the limit.

EXAMPLE 1 lim 5a-v -f T,-v


Ivv -
(A.v)-.(-1.2l ^ A -I- V

SOLUTION Applying the limit properties for sums and products, and abbreviat-
ing lim as lim.
I. we
W( have
(t,,v)-.(-l,2l

3v-
lim I 5a-v + 2r\'

A + y '

3v-
= lim 5a~v + lim Ivv - lim
A + y
\'~
3 lim
= 5 lim A" lim y + 2 lim a lim y
lim A + lim y

3(2)-
= 5(-l)-(2)-^2{-l)(2
1 -I- 2

EXAMPLE 2 lim (e""'-'*'"'" -t- COS 3.n)


(A..V)^(0.0)

SOLUTION lim
(«.'-i"'5..-+.v)
+ COS 3.XT) = f-nlS""'*"! -f- COS [3(0)(0)]
(A.V) ^(0.0)
= f" -t- COS = 2

*The proofs are similar to the proofs of the properties of limits of functions of a single
variable given in Appendix B.
SECTION 14.2 LIMITS AND CONTINUITY 839

Figure 1 Chapter we observed that lim fix) exists


In 1 if and only if both one-sided limits,

lim fix) and lim fix), exist and are the same. In dealing with a limit of a function

/of two variables, say, lim fix. y), we can allow the point (.v, v) to approach

the point («, b) not only from the right or from the left, but also from above, below,
or indeed, any direction (Figure la). We can even allow (v, y) to approach {a. b)
along a cun'ed path (Figure lb).

To say that lim fix. y) = Z. is to require that as (.v, y) approaches (a. b)

along any path whaisoever. fix. y) approaches the same limit L.

Thus, a con\enient way to show that a particular limit lim fix. y) does not

exist is to show that/(.v, y) can be made to approach two different limits as (.r, y)
approaches (a. b) along two different paths.

2jt"v
EXAMPLE 3 Let /(.Y. V)
3.V- + 3v-

(a) Evaluate the limit of /(.v, y) as (.v, y) approaches (0, 0) along each of the
following paths: (i) the x axis, (ii) the y axis, (iii) the line y = .v, (iv) the
parabola y = .v".

(b) Does lim f(x, v) exist? If so, what is its value?


(.v.vl-^dJ.Ol

SOLUTION
Iv-(O)
(a) (i) On the x axis, y = and/(.v, y) =/(.r, 0) = Ofor.v^O.
3a-- +
Therefore, lim fix. 0) = lim = 0.

(ill On the V axis,

Therefore, lim /(O. v)


x =
=
and/(.v, v)

lim
>-> =
= m. v) = —2(0)-vV =
+ 3;-
3y'
for v ¥^ 0.

0.

(iii) On the line y = .v. /(.v. y) = /(.v, x) =


3.V-
Iv-.v

+ 3.V-
— T =
6.V-

3
for

X ^ 0. Therefore, lim fix. x) lim


A-*u 3
—= 0.

2x~x' 7v-
(iv) On the parabola y = x-.fix. y) =/(.v, x-) =
3.r- + 3.r-* 3 + 3.v-

for v ¥" 0. Therefore, lim fix. v") = lim 0.


I— 3 + 3.Y-

(b) Along all the paths in part (a), the limit is the same, 0. Although
there might be some other path along which the limit is not 0, the evi-
dence leads us to suspect that the limit is 0. Our suspicion is confirmed as
follows: For .r- -H y- # 0, .v" s .r- -I- y-, so .r-|y| s (.r- -I- y-)|y|: hence,
.Y'|y|/(.v- -f- y-) < |v|. It follows that . ,

Ix^y 2
l/(-v, J)|
=
3x-
I
2

3 x^
x~\y\

+
—3 ,

V
''
,

-I- 3v- V-
y.-^'^.
840 CHAPTER 14 PARTIAL DIKKERENTIATION

Therefore, as {x. y) approaches (0, 0),/(.v, y) must approach 0. since |/(.r, y)|

is no larger than 'i\y\, which approaches 0. It follows that

lim
(v..v)-^io.())

3.V
2.rv
;-^
+ 3v"
r=

EXAMPLE 4 Follow the directions in Example 3 for the function

-
yCv, y) =
.V-
.\
x-+
v"^

y
SOLUTION
-
(a) (i) On the a- axis, v = and /(a, v) =/(a, 0) = —A- +;
= I for x ¥= 0.
A"
Therefore, lim /(a, 0) = lim 1 = 1.

=
0- V-
'— = -
(ii) On the v axis, a = and fix. v) =/(0, . v)
y) 1 for v # 0.
+ v
Therefore, lim /(O, y) = lim (-1) = -1

(iii) On the line v = a,/(a, v) =/(a, a) = — ^ = for a 5^ 0. There-


X- + X-
fore, lim fix. a) = lim = 0.

- - X^
(iv) On the parabola v = a-, /(a, v) =/{a, a ) = —A- ^
A-*
j =
1

r for
A" + A 1 + A"
1 - A-
A 7^ 0. Therefore, lim fix, x") = lim r- = 1.
A-.0-' A-o 1 + X-

(b) Since the limits in (i) and (ii) are different, lim /(a, v) does not exist.
(A.v)->(0,0)

The following example shows that /(a, y) can approach the same limit as (a, y)

approaches (a, b) along every line through (a, b), and yet lim fix, y) can fail
to exist.

EXAMPLE 5 Let
, , A + — ) V' if A 5^
fix, y)

'o ifA =

(a) Calculate the limit of fix. y) as (a, y) approaches (0, 0) along the line
y = mx.
(b) Calculate the limit of /(a, y) as (a. y) approaches (0, 0) along the parabola
A = v".

(c) Does lim fix. v) exist?


(A.Vl^(O.O)

(a) On the line y = mx. fix, y) = fix. mx) = \x + i\/x)]imxf for a # 0, so


that lim fix. nix) = lim [a + i\/x)]i/ttxf = lim imV + m'x) = 0.
A-.0 A->0 A—
(b) Along the parabola a = y-,/(A, y) =/(y', y) = [y- + (l/y-)ly- fory ¥^ 0.

so that lim Av", v) = lim jv' + (l7v-)]v- = lim (v"* + 1) = 1.

(c) Since the limits in parts (a) and (b) are different, lim fix, v) does not
(A.V)^(O.O)
exist.
SECTION 14.2 L1\UTS AND CONTINUITY 841

The formal definition of the Hmit of a function of two variables is panemed on the
definition in Section 1.7 of the limit of a function of a single variable.

DEFINITION 1 Limit of a Function of Two Variables

Let/be a function of two variables, and let (.v,,. v„) be a point in the .xt plane.
Assume that there exists a circular disk with center at (ao. Vo) and with positive
radius such that every point within the disk, except possibly for the center
(xo, Vo). belongs to the domain off. Then we say that the limit as (x. v) ap-
proaches {.Xo, Vo) off(x, y) is the number L. and we write

lim f(x, v) = L

provided that for each positive number e, there is a positive number 8 such that
|/(.r, y) - L\< e holds whenever (.v. y) # (.v,,. y,,) and the distance between (.v, y)
and (.Vo, yo) is less than 8.

The condition given in this definition can be written as follows: For every e > 0,
there exists 5 > such that

< (.V - xof + (y - yo)- < 8- implies that \f{x, y) - L\<6

EXAMPLE 6 Show that lim (3x + 2v) = 7 bv direct application of Defini-


...v.-, 1.2,
tion 1.

SOLUTION Let e > be given. We must find 5 > such that |3.v + 2y - 7| < e
holds whenever < (.v - 1)- + (y - 2)- < 5". Now,
|3.v + 2y - 7| = |3.v - 3 + 2y - 4| < |3.r - 3| + |2y - 4|

= |3(.v - 1)1 + |2(y - 2)1 = 3|.r - 1| + 2|y - 2|

Hence, if 3|.v - l| < e/2 and 2|y - 2| < e/2. then

|3.v + 2y - 7| < 3|.v - l| + 2|y -2|< —+—=e


The condition 3|jf - 1| < e/2 is equivalent to 9(.x: - 1)- < e^/4, or to
{x - )- < 6^/36.
1 Similarly, the condition 2|y - 2| < e/2 is equivalent to
4(y - 2)- < e^/4, or to (y - 2f < e^/16. Therefore, if (jr - 1)- < 6^/36 and
(y - 2)- < e^/16, it will follow that \3x + 2y - 7| < e. Thus, choose 5 = e/6,
and note that if

0<(.v- 1)- + (y- 2)2 < S~ =


36

then (.V- 1)- <(.r- 1)- + (V- 2)-<


36

and (V - 2)- <U- 1)- + (V - 2)- < <


36 16

Hence, (.v - 1)- < e2/36 and (y - 2)- < e^/ie. so that liv + 2v - 7| <

The definition and the properties of limits can be extended to functions of three or
more variables. For instance, if the number /(.v. y. z) can be made to come as close
842 ( IIAPTKR 14 HARTIAI. DIKKKRK.NTIATION

to the number L as we wish by taking the point (.v, v, r) sufficiently close to the point
(.V(), .Vo. -()) but different from this point, then we write

lim fix, V. z)= L


{x,y.z)—'{xa.yo.:«)

Continuity
The definition of continuity for a function of two variables is patterned after the
definition in Section 1.9 for continuity of a function of a single variable.

DEFINITION 2 Continuity of a Function of Two Variables

Suppose that / is a function of two variables and that the point (.xv,. Vo) is the
center of a circular disk of positive radius contained in the domain of/. We say
that / is continuous at (.v,), Vo) if both of the following conditions hold:

(i) lim /(.v. v) exists

(ii) lim /{.v. v) = /(.v,,, w,)

In Examples 7 and 8, decide whether the given function is continuous at the indi-
cated point.

EXAMPLt: 7 fix, y) = 3j:- + 2ry at ( - 1 , 3)

SOLUTION By the properties of limits.

lim (3.V- + Zvv) = 3(-l)- + 2(-l)(3) = -3 =/(-!, 3)


(>,>)-.( -1,3)

Hence, /is continuous at (-1. 3).

f 4ai'
, •
. if ix, V) 7^ (0, 0)
EXAMPLE 8 = < V- + >- at (0, 0)
fix, y)

lo if (A-. y) = (0, 0)

SOLUTION Along the x axis, we have

fO if .V #
/(.v. y)=/(.v. 0)
lo if X =
so that lim fix. 0) = 0. Along the line y = x. we have

(2 if .V 7^0

so that lim fix. x) - 2. Thus, lim fix, v) does not exist, and so/ is not
< -0' (<,\l— (0,0)'

continuous at (0, 0).


Figure 2

Let D be a set of points in the at plane. A point (a,), Vo) is called an interior point
of D if all points in some circular disk of positive radius with (Ao, Vo) as its center are
contained in D (Figure 2). On the other hand, a point («, h) is called a boundary
point of D if every circular disk of positive radius with («, /') as its center contains
at least one point that belongs to D and at least one point that does not belong to D
(Figure 2). There is no requirement that a boundary point ia, h) of D belong to the
set D. Note that a point cannot be an interior point of D and also a boundary point of
SECTION 14.2 LIMITS AND CONTINLTTV 843

D. A set D is said to be open if it contains none of its own boundarj' points, while a

set D is said to be closed if it contains all its own boundary points. (This terminol-
ogy is suggested by the ideas of open and closed intervals.)
Notice that Definition 2 applies only to the continuity of a function at an interior
point of its domain. A modified definition of continuity, which we do not set forth
here, is required for continuity of a function at a boundary point of its domain.
Naturally, a function is said to be continuous if it is continuous at every point in its

domain.
In order to show that a set D of points in the plane is open, it is enough to show
that every point in D is an interior point of D (Problem 42).

EXAMPLE 9 Show that the domain of the function /(.v. y) = xy/{.x - y) is an open
set, illustrate this domain with a sketch, and show that /is continuous.

Figure 3 SOLUTION The domain of/ consists of all points in the .r\' plane except those
lying on the line y = .v (Figure 3). Suppose that (.vq. vq) is a point in the domain D
off. so that (.Vq, Vq) is not on the line 3' = x. If d is the distance from (.Vq, Vq) to the

line y = .v. then any circular disk of radius r with < r < d is contained in D:
hence, (.Vo. Vq) lies in the interior of D. If follows that the domain D is open. Since,
for .Vo 5^ Vo,

lim = /(-Vo- .Vq)


) —(.XO.TO) •1^0 - .Vo

it follows that /is continuous at ever\ point (.Yq, Vo) of its domain. Therefore, /is
a continuous function.

Functions of two variables have many of the same properties with regard to
continuitv as do functions of single variables. Amona these are the following.

Continuity Properties for Functions of Two Variables

Suppose that (xq, Vq) an interior point of the domains of the functions / and g of
is

two variables, and assume that /and g are continuous at {xo, yo). Then:

1 hix. y) =/(.v. y) + gix. y) is continuous at (.v,,. y,,)

2 k(x. y) =f(x, y) - g(.v, y) is continuous at (.Vq, Vo)

3 pix. y) =/(.r, y)g(x, y) is continuous at (xq. Vq).

4 If i;(.v„. Vo) 7^ 0. then ^(.v. v) =


/(v. V)
^ IS continuous at (vn. Vo).
gix. y)

5 If IV is a function of a single variable that is defined and continuous at the

number /(.Vo, Vq) and if (.Vo. Vq) is an interior point of the domain of
itv. y) = vr[/(.v. y)]. then r is continuous at (.Vo. Vo).

EX.AMPLE 10 What are the interior points of the domain of the function 11 defined
by u{x. y) = Vl — .x' — v" In (x + y)? Is u continuous at such points? Illustrate
with a sketch of the domain of u.

SOLUTION The expression V 1 — .v" — y- is defined only for — .v" — y~ s 0, 1

that is. for.v" + v" s 1. The condition .v" + v" < holds exactly when (.v. y) is on 1

or inside the circle x- + y- = of radius 1 with center at (0. 0). The expression
1

In (.Y + y) is defined only for v + y > 0. that is. only when the point (v. y) lies
844 CHAPTKR 14 PARTIAL DIKFERENTIATION

Figure 4 above the line .v + y = 0. The domain of m is therefore the shaded region shown in
Figure 4 inside or on the circle x- + y" = 1 but above the line a + y = 0. The

interior points of this domain are just those points not lying on the rim of the circle

(nor on the line + y = 0). Since the square-root function is continuous, the func-
.v

tion /(.v, y) = VI — .V" — y" is continuous inside the circle .x" + y' = 1, by Prop-
erty 5 above. Similarly, the function g(.x. y) = In (jc + y) is continuous above the
line x + y = 0. By Property 3. the function u{x, y) =f(x. y)g(x, y) is continuous at

all interior points of its domain.

The definitions and properties of continuous functions are extended in an obvious


way to functions of three or more variables. Naturally, every polynomial function in
several variables is continuous Also, a ratio of such polynomial functions that is.
.

a rational function of several variables has an open domain and is continuous at —
every point of its domain. For instance, the function

_ Sx'^'-z^ - 7jty^r + -vVr - x + 12y + 1984


/(x, y, z) - '

2^^3^3_2 _ ^^^ ^ 1^^ - , + 33

is continuous at every point (.v. y, z) for which the denominator is not zero. Roughly
speaking, any function built up in a ""reasonable way" from continuous functions is

continuous at every interior point of its domain.

Problem Set 14.2

In Problems to 12. evaluate each limit.


1

14 /(.v, y) = ^ 5^ as (.r. y) approaches (0. 0) (i) along the

lim ^ (5.V- + 3.\y) 2 lim (sin 2r -I- sin 2v) .V axis, (ii) along the y axis, (iii) along the line y = lO.v.
(x.yl-(7r/4.n1
(iv) along the curve y = .v''

(^'"«'^+.vi
3 lim + 5gj. 5^.^,) 4 ijn,
n.vi^(O.O) (T.vi— (0.0) In (.V -I- iy + e) •5 -y as
/(.v. y) (.V, y) approaches (0. 0) (i) along the .v axis.
\- + y-

hm
1— (0,01
cos X
e
TT —+ COS y
+ rr
3e^
6 lim p.v -(-
h'l (ii)

y
along the y axis,
= mx
(iii) along the line y = x. (iv) along the line

8 lim 3;tS'-
).0l Ml'
16 fix. y) Ty as ix. y) approaches (0, 0) (i) along the x
(.v" + y-
9 lim In ( 1 -H 4x\-) 10 lim sV? + 2s axis, along the y axis, along the line y = x. (iv) along the
It. n— 1(1,0)

(i./)-.(4,-2)
(ii) (iii)

curve y = -.v"*

xvz —X + V — z
11 lim
+ Y + z' — 4
(.t..v..-)— (-1.2.0) X'
1(.v -1-
y) sin — if y ^
as ix. y) approaches
12 lim z sm , __ , ,
(.<.>.;)— (0.0.0) \ V.r- + y- + z' if y =
(0, 0) (i) along the x axis (ii) along the y axis, (iii) along the
In Problems 13 to 18, (a) evaluate the limit of fix. y) as (x. y)
= = — Jc'
curve y .t', (iv) along the curve y
approaches (0, 0) along each of the indicated paths (i), (ii), (iii), and
(iv); (b) determine lim /(.t, v). if it exists.
(>,_>)— (0.0) r
—I

sin AT it y #
18 fix. y) = } as ix. \) approaches (0, 0)
13 fix. v) = 1
5xy^
—T as (v, V) approaches (0. 0) (i) alone the A if y =
x~ + y~ [

X axis, (ii) along the y axis, (iii) along the line y = 5.v, (iv) along (!) along the x axis, (ii) along the line y = 2v. (iii) along the
the parabola y = x^ parabola y = x^. (iv) along the curve y = 5a'
SECTION U.3 PARTIAL DERH ATIVES 845

In Problems 19 and 20. prove each statement by making direct use In Problems 29 to 39. (a) sketch a diagram showing the domain of
of the definition of limit; that is. show that for each given e > 0, each function in the .\t plane, (b) specify which points of the domain
there is a 8 > so that the conditions of the definition hold. are interior points, and (c) determine those interior points of the
domain — if — any at which the function is discontinuous.

19 lim (5.v+3v)=13 20 lim (-.r + 2v)


= V^
-

l-v.n— (2.11 (,,._vl^(-4.2) 29 fix. y)

'"'"
Zx-y
In Problems 21 to 28, decide whether each function is continuous at

the point indicated. Justify your answer. X

21 /(.v, y) = V25 - .V- - y- at (-3. 3)

22 fix. y) = e"" In (.v" - 2y + 7) at (0. 0)

—- '-^
if V # Iv
\- Iv
23 cijU. y) at (1, 2)

1 if y = 2v

24 fix. y. r)
+
1

+
— at (0. 0. 1)
.V- V- z-

25 g{x. V) = sm .V
at
/
77.
sm V ^

26 /!(.v. y) at (0. 0)
1 + e'

if .V #
27 fix. y) at (0. 0)

if .V =

7.V-V
if (.V. y) ^ (0, 0)

28 fix. y) = at (0. 0)
if U, y) = (0. 0)
846 CHAPTER 14 PARilAI. DIKFERENTIATION

Hence,

— d
dx
f(x. V) = — d
dx
(A-)
,
+ —
d
dx
(3.VV) + — (I

d.x
(-4v-)
-,

= Zx + 3v + = 2x + 3y

In order to emphasize that only v is being allowed to vary, so that y is held fixed
while the derivative is calculated, it is traditional to modify the symbol d/dx and
write d/d\ instead. (The symbol t) is called the "round </.") Thus, we write the
equation above as

—obf
/(.v. V) = —
Ax
(.V- + 3.n' - 4v^) = lx + 3v

The derivative calculated with respect to .v while temporarily holding y constant is

called the partial derivative with respect to x, and cl/Ax is called the partial
derivative operator with respect to .v. Similarly, if we wish to hold the variable .v

fixed and differentiate with respect to y, we use the symbol d/dy. Thus, for the
function /(.v, y) = x' + 3xy — 4y-, we have


dy
/(A-, 'V) = — dy
(A- + 3.VV - 4v-) = --
dy
(A-) + --
ay
(3.VV) + -— (-4y2)
dy

= + 3a 3a - 8v

More formally, we have the following definition.

DEFINITION I Partial Derivatives of Functions of Two Variables

If/ is a function of two variables and (a, v) is a point in the domain of /, then the

partial derivatives '


" '
"
— and ^ —— of/ at (a. v) with respect to the first
^.v By
and second variables are defined by

dl\x.\)
—= ,. /U + Aa, V)-/(A. V)
lim
dx Aa^o Aa
df(.x. y\ /(A, .v + Ay)-/(.Y, y)
and
Av

provided that the limits exist. The procedure of finding partial derivatives is

called partial differentiation.

KX.\MPLE I Use Definition 1 directly to find dzjdx if r =/(a, y) =


5a;^ - 7Ay + 2y-.

SOLUTION

— dz

dx
= ,.
lim
A>—
/(A+ Aa. V)

Aa
-/(A. V)

[5(A + Aa)- - 7(A + AA)y + ly-\ - (5a- - 7.xy + 2y-)


= lim
A.V

5|a- + Zv Aa + (Aa)-| - 7av - 7v Aa + 1\- - 5a- + lx\' - 2v-


= lim :

Aa -0 Aa
IOa Aa + 5(Aa)- - 7v Aa
= lim = lim (10a-7v + 5Aa)
Aa — (] A.V Aa^U
= 10a - 7v
SECTION 14.3 PARTIAL DERIVATIVES 847

It is convenient to have a notation for partial derivatives that is analogous to the


notation /'(.v) for functions of one variable. Thus, if z = fix, v), we often write
fi(x. y) or/t(.ic, y) rather than clz/dx or cff{x. y)/c)x for the partial derivative of/ with
respect to x. The subscript 1 (or the subscript x) denotes partial differentiation with
respect to the first independent variable (or with respect to .v). Thus

(5.V f7v

Similarly, the subscript 2 (or the subscript \') denotes partial differentiation with
respect to the second independent variable (or with respect to y), so that

dz
848 CHAKIKR 14 PARTIAL DIFFERENTIATION

(b) Holding .v and y constant and differentiating with respect to :. we have

-- (.vv-z-') = AT- -— (z') = .n-(3z-)


'
= lx\~i~
d: dz '
dz

EXAMPLE 3 If /(.v, _v) = .re' + y In v. find

(a)/,(.r, y) (b) /,(.ir, >-)

SOLUTION

(a) /,(.v. V) = —r)x


Ixe' + V In .v) = •—
dx
(xe>) + --
dx
(v In a)

dx d —\
= e^ 1- \ (In x) = e- -^
dx ' dx X

(b) /2(a, v) =— d
(.ve' + \' In a) = —d
(xe-) + — d
{
>•
In .t)

= —
d d\
—^
X (e^) + In A xe- + In A
dx dx

EXAMPLE 4 If/(x, y, z) = e"^"--\ find/,(l, 2. 3).

SOLUTION Holding a and y constant and differentiating with respect to z. we


have
/,(.v, V, z) = -^ (e'^z^) = e'>' -^ (z^) = 3e'^'z^
dz dz
Therefore,
/,(1. 2, 3) = 3e""-'"'3- = lie-*

Note that to fmd/,(I, 2, 3), we first differentiate to find /-(a. y. z) and ihen substi-
tute A = 1 . y = 2. and z = 3.

There are many versions of the chain rule applicable to partial derivatives, the
simplest one being virtually a direct transcription of the ordinar.' chain rule
for functions of a single variable. Thus, let g be a function of more than one
variable — say two variables for definiteness. If w =/(v) and i' = g{x, y), so that
"' ~f[8(X' y)]' then, holding y constant and using the usual chain rule, we have

— chv

dx
=/'lg(-v, y)]«,(.v. y) =/'(v) —=——
dv

dx
dw
dv
dv

dx

provided that the derivatives dw/dv and d\'/dx exist. Similarly, holding a constant
and using the usual chain rule, we have
dw —
dw_

dx
--f'[g(x. y)]g,(A, y) =/'(v) — = --
d\'

dx dv
dv

dx

provided that the derivatives dw/dv and dv/dy exist. In summary, M dw/dv. dv/dx.
and d\'/dy exist, then

dw
SECTION 14.3 PARTIAL DERIVATIVES 849

Put V = 1 — jc" — v", so that w = Vv. By the chain rule.

dw
dx
dw
dv
dv

dx 2Vv
1

Sx
d
X- - V")
—iVv -=. (-2x) = ^j=
vT
dw dw dv \ d
V')
2V^
^(-2v) = —vT7=
dy dv dv 2\/\' d\

In using the version of the chain rule, given above, you proceed in much the same
way as in Section 2.7. except that you muUiply the ordinary derivative of the
"outside function" by the appropriate partial derivative of the "inside function."

EXAMPLE 6 If/(A-, y) = tan (.v" - v"), find/v(.v. y) and /;.(.v, y).

SOLUTION Here the "outside function" is the tangent function, and the "inside
function" is the function g(x. y) = x^ — y. Thus,

/,-(.v, v) = sec- (.V- - y-) —


dx
(.v" y-) = [sec- {X- - y^)](2x)

= 2x sec- (.V- — y^^)

d
and fy{x, y) = sec- (x y') -— U' y-) = [sec- (.V- - y-)](-2y)
dv
= —2y sec- (.v" - y")

EXAMPLE 7 If H' = e"''', find dw/dx and dw/dy

dw
dx dx \y

^
dv
= ,.r/y ± (^)
dv ^v ' V /

Geometric Interpretation of Partial Derivatives


Suppose that /is a function of two variables and that /has partial derivatives /i and
fj- The graph of /is a surface with the equation z =/(.v, y) (Figure 1). Let Zq =
/Uo' .Vo). P = (jto, yo, Zo) is
so that a point on this surface. The plane y = Vo cuts a
cross section APB from the surface, while the plane x - Xo cuts a cross section CPD
Figure 1 from the surface. As a point moves along the curve APB. its .v and z coordinates

vary according to the equation ~ = f(x, Vo), while its y coordinate remains constant
P= (vo. vn. ^o) = APB any point is the rate at which the
with y Vo . The slope of the tangent line to at

z coordinate changes with respect to x; hence, this slope is given by d:/dx =


fi(x, yo). In particular, /i (.Vo , yo) represents the slope of the tangent line to APB at

the point P. Similarly, /2(xo, yo) represents the slope of the tangent line to CPD at

the point P. Thus, in Figure 1, we have


850 CHAPIER 14 PARTIAL DlFreRENTIATlON

KXAMPI.E 8 Find the slope of the tangent line to the cross section cut from the
surface z = 4.v"v — .vv' by the plane y = 2 at the point P = (3, 2, 48).

SOLUTION Here wc hold y constant and find c)z/clx. We have

— = -—
-dx
dz

dx
d
(4x-v)
^
- — d

dx
Cn-')
.
= %xy- y^
,

so that when v = 3 and y = 2,

—= dz

<9jc
8(3)(2) - 2' = 40

Rates of Change
Suppose that a variable quantity, say w, depends on a number of other quantities,
say Ai , .Vt, . . . ,x„. If all but one of the independent variables X\ , .Vt, . . . , a:„ are

held constant, then w depends only on the remaining variable, and the instantaneous
rate of change of w with respect to this variable is given as usual by a derivative — in

this case, the partial derivative of ic with respect to the variable in question.

E EXAMPLE 9 The volume V of a right circular cylinder is given by V= 7rr-/i,

where r is the radius of the base and h is the height.

(a) Find the instantaneous rate of change of the volume with respect to the
height if the radius remains constant with the value r = 6 centimeters while the
height h varies. Evaluate this rate of change at the instant when /; = 10 centi-
meters.

(b) Suppose that the height remains constant with the value /; = 10 centime-
ters. Assuming that the radius is variable, find the rate of change of the volume
with respect to the radius at the instant when r = 6 centimeters.

SOLUTION

(a) —dV- = —d - (irr-h)


,
= irr'
dh r>h

When r = 6 and /i = 10 centimeters, we have dV/dh = 7r(36) = 1 13 cubic

centimeters per centimeter.

(b)
dV
dr
= —dr
d
(irr-h) = 2t7/7;

When r = 6 and /! = 10 centimeters, we have dVjdr = 2M6)t.\0) == 377 cubic


centimeters per centimeter.

Problem Set 14.3

In Problems I to 6, find each partial derivative by direct application 3 /, ( - 1 , 2), where fix. y) = -T.v" -I- S.rv'"

of Definition I

1 /,(.v, V), where /(.v, v) = 8,t - 2v + 13 4 -—fix. y, z). where /(.v, y. -) = Ivy" - 7.vz + 3.xy;^
az
<> -,
1
2 T~/'-^- ^''- where /U, y) = 3.r- + Sxy + 7y u « ' a 2
dx
,,
5 y:(l, -1). where c,
/(.v, y)
^
= S.n-
j.
-I- 6.v
,

-t-
>
II
SECTION 14.3 PARTIAL DERIVATIVES 851

27 /i,(.v. y). where h{x. y) = tan ' (xy)


6 —^ at the point (3, -5). where : = -7.v^v + .rv + 3
28 /,(a. y). where /(.r. y) = e''*'''

In Problems 7 to 22. find each partial derivative by treating all but


29 . where iv = sin"' w and u = Vxy
one of the independent variables as constants and applying the rules '^
of ordinary differentiation.
30 —U e' dt

7 —dx
/(.v, v), where /U. v) = Tx- + 5x-y + 2
dy

= -
31 /,(2. -1). where /(.v. y) V.v" + y- 1

8 — (.\- sin V) 32 /^U, y, z), where /(.v. y. ;) = .r'/Vy" + z'

33 g3(.x, y, z), where gU, y, z) = xz'e" cos yz


9 /!,(.t, y), where h{.x, y) = sin x cos 7y

10 /2(.t. y), where /(x, y) = e"^ tan y 34 — [z sin Uz) cos (.rv)]

11
(?vv

dx
, where »• = ^ +— X-
.r"

V"
v"
'—r
35 . where u- = (.f + v" + z")
^'^

dx

12 —dx
(v sin V — V In a)
36 , where w= e '
sin (s + u)
ds

13 /i(r. 0), where /(r. 6) = r" cos 79 =


37 /,(A. y). where /(A. y) cos [In (a" + y-)]

14 §2(0. — ). where g(Q. <t>) = sin 20 cos <fr


38 , where w = (v" tan .v)"^'^

15 /i(2, -3). where /(.v. y) = A-y' - 3.ty- + 2y


39 gv(x. y), where g(A, y) = cos" (3A-y"')

16 g:(A, y). where g(x. y) = e~' dt


J 40 , where w= sinh V^ cosh" (4i-V')
ds

17-/;(A. y. z), where /(A. y, z) = 6.\tz + 3A-y + 7z


41/3(1, 1. 57r/4). where /(A. y. z) = (xy + 3f

18 —dy
g(x. V. z). where g(x. y, z) = 3.r^-''z'' - 4.ryz^
42 , where j = (sin u)"
du
c>w
=
•,-,-.
+ vz" + .m'
19 . where >v x\-
= -
dx 43 , where p (3m 2vf'°''
dv

20 where vv = a cos z + v sin a + xe' ax |_ „


dz
.

44 ^— , where a = (stu)
,

21 g,(2, 3. -4). where g(A,. aj, .tj) = lv,e" + 2x2?" + 2xye^-


45 Given that if = x'y- - Ixy* + 3x-y^, verify that
22 where p defined by /)(ai. At, A3, .V4) =
P2(-Ti. A2, A3, Xi). is

— d\i' dw
A1A2A3 sin A4 A2Aa'4 sin ai A 1- V = 5n'
dx - dy

In Problems 23 to 44. find each partial derivative by using the chain 46 Given that w= {ax + by + cz)". where a. b. and c are con-
rule. stants, verify that

dw d\v d^v
» = = X + V H z = nw
23 . where \'u and u 3a~ + y-
^ dx dy dz
dx

a 47 Given that »= f- + tan te^'', verify that


1
24 ^— g(A. y). where gix. y) = sin (.n)~
dw dw
, , 1
i + I = 2f-
ds dt
on , ,

25 where » = In m and u = 7a" + 4y"


dx
,

48 Let/be a differentiable function, and let w = /(f 5- - |r-). Ver-

ify that

26 — d
In 7r
ds
+ 5s
dt
=
852 CHAIIKR U PARTIAL DIKKKRENTIATION

49 (iivcn thai » = v" sin — + v" In — + rV'^'. verilv ihal 62 If two sides .v and y of a triangle are held fixed while the angle S
r .V
between them is varied, find the rate of change of the third side z

with respect to 6. (Use the law of cosines.)


Sw dw
X +v +- dw = 2u'
dx •
dy dz ]63 Clairaut' s formula for the weight W in dynes of a 1-gram mass
at latitude L degrees and at height h centimeters above sea level
50 Given tliat w = e"'' + e^'^' + f"''', verify lliat

Sw dw dw ttL
X + y + z = W= 980.6056 - 2.5028 cos
dx ^ dy dz 90 3,000,000

51 Cji\en (hat ii' = In (.v' + 5.v-y + (xxy- + 7y'), verify that (a) If L is held constant at 40°N while h is varied, find the rate of
change of W with respect to /; at the instant when the mass is
dw dw 6 kilometers above sea level.
X + V = 3
dx ' dy (b) If the mass remains at a constant altitude of 6 kilometers
above sea level while its latitude L is varied, find the rate
52 Given that
of change of W per degree of latitude at the instant when
L = 40°N.

64 A circular hill has a central vertical section in the form of a curve


show that
whose equation is z = 10 - (.v"/160), where the units are in

\
dz

dx
X —=
dz

d\
meters. The top is

constant rate of 100 cubic meters per day.


being cut down in horizontal layers at the

How fast is the area of


horizontal cross section increasing when the top has been cut
down a vertical distance of 4 meters?
In Problems 53 to 60. Unci the slope of the tangent line to the cross
section cut from the given surface by the given plane at the indicated 65 The resistance R ohms of an electric circuit is given by the for-
point. mula R = E/I. where / is the current in amperes and E is the
electromotive force in volts. Calculate dR/dl and dR/OE when
53 The surface z = 3.v — 5y + 7 and the plane y = 2 at the point
/ = 15 amperes and £=110 volts, and give an interpretation of

(1.2.0) these two partial derivatives in terms of rates of change.

54 The surface z = V4 — .v" - y- and the plane v = 1 at the point


66 A mortgage company computes the maximum size of a 25-year

mortgage that is willing to give on a one-family house by using


(1.1. V2) it

the formula
55 The surface z = V31 - tc - 3y- and the plane y = 2 at the M = 3.V 4- 0.025y-
point (3. 2. I)
where .v is the size of the down payment in dollars and y is the
56 The surface x' + y" + z" = 14 and the plane v = 1 at the point applicant's monthly salary in dollars. Find dM/dx and dM/dy,
(1.3.2) and interpret their meanings in terms of rate of change.

57 The surface z = e '"


sin 3y and the plane v = I at the point 67 Psychologists define the intellifience quotient Q{M. C) of a per-
(I. 0. 0) son to be 100 times the person's mental age M divided by the
chronological age C. so that Q(M, C) = lOOM/C. Find dQ/dM
58 The surface 6.ic- + 9y- + 4z- = 61 and the plane y = - at the
point (I. -I. 2)
I

and dQ/dC when M = \1 years and C = 10 years, and interpret


the results.
59 The surface z = IrvA-t" + v") and the plane v = 4 at the point
] 68 Medical researchers use the body surface area of a patient to
(3. 4, M) calibrate the results of tests of kidney performance. The formula
60 The surface z = e''''' — e''^' and the plane y = 2 at the point •*-'//" '-'
A = O.OOTVV"
(2, 2, 0)
is often used to calculate the body surface area A in square centi-
meters of a person whose weight is W kilograms and whose
061 The lateral surface area -4 of a right circular cone of height /; and height is H centimeters. Find dA/dW and dA/dH when W= 58
with base radius r is given by A = TTr\/hr + r". kilograms and W= 168 centimeters, and interpret the results.
(a) If r is held fixed, with r = 3 centimeters while is varied,
/;
69 The economists Paul Douglas and Charles Cobb proposed the
find the rate of change of A with respect to h at the instant
theoretical relationship P = A/,"/^'^ among the total production P
when /; = 7 centimeters.
of a fimi or industry, the input L of labor in person-hours, and
(b) If h is held fixed with h = 1 centimeters while r is varied. the investment K in capital equipment. Here a. p. and A are
find the rate of change of A with respect to r at the instant constants depending on the firm or industry in question and the
when r = 3 centimeters. units in which P is measured. Find formulas for the marginal
SECTION 14.4 LINEAR APPROXIMATION AND DIFFERENTIABLE FUNCTIONS 853

productivity of labor dP/dL and for the marf;inal productivity of


capital dP/dK.

70 The formula H= (2.14 + 1.37 VV^- 0.0916V)(91.4 - T)


gives the heat loss (wind chill) H in Btu's per square foot of skin
per hour if the air temperature is T degrees Fahrenheit and the
wind speed is V miles per hour. Find dH/dV and dH/dT when
V = 20 miles per hour and T= 25°F. and interpret the results.

1 71 The ideal gas law PV = itRT is used to describe the relationship


among the pressure P. the temperature T, and the volume V of a
gas, where n is the number of moles of the gas and R is the
universal gas constant.

(a) Find dP/dl OT. and V = 50 cubic


inches.

= -1

El 72 According to Poiseuille's law, the speed V in centimeters


per second of blood flow in a vessel is given by V =
P{R- — r')/(4Lv), where R is the radius of the vessel, r is the
distance from the center of the vessel. L is the length of the
854 C IIAPTER 14 PARTIAL DIFFERKNTIATION

1 EXAMPLE I If/(.r, y) = V.v^ + y~ + 2, use the linear approximation procedure


to estimate/(3 .0 1 , 4.96) and check the accuracy of your result by using a calculator.

SOLU HON In the linear approximation

f(.x + A.V. y + Ay) «/(.r, y) +/,(.y. y) A.v +/2(.x, y) Ay

we put X = 3, y = 5, A.v = 0.01 , and Ay = -0.04, to obtain

/(3.01, 4.96) = V3- + 5- + 2 +/,(3, 5)(0.01) +/2(3, 5){-0.04)

Here, /,(.v. V) = —dx


V.v- + V-

+ 2 = ,
Vx-
,
+
,
V- +
^
2

fiix- y) V.x^ + V- + 2
dv V.v- + y- + 2

5
so that /,(3. 5) and /2(3, 5)
--
36 2 ^/2^ g

Thus, /(3.01, 4.96) == \/36 + 1(0.01) + §(-0.04) = 5.97166 •


The true value of/(3.01, 4.96), correct to five decimal places, is 5.97174, so the
linear approximation procedure has given a result that is correct to the first three
decimal places.

Differentiable Functions of Two Variables

The error E involved in the linear approximation

fix + A.V, y + Ay) =/(.r, y) +/,{.v, y) A.v +/2(.v, y) Ay

is given by

E =f(x + A.V, y + Ay) - [fix. y) +/,(.v, y) Ajr +/2(.v, y) Ay]

= /(.v + A.V, y + Ay) -fix. y) -/i(.v, y) A.v -fzix. y) Ay

Roughly speaking, the condition that /is differentiable is that the error E approach
zero very rapidly as A.v and Ay approach zero, so that the linear approximation is

very accurate when |A.v| and |Ay| are sufficiently small. More formally, suppose that

Uo. .Vo) is an interior point of the domain of /and that the two partial derivatives
/i(f(). .Vo) and/2(.vn, Vn) exist. Then we make the following definition.

DEFINITION 1 Differentiable Function of Two Variables

Wc
SECTION 14.4 LINEAR APPROXIMATION AND DIFFERENTIABLE FliNCTIONS 855

between the condition on the error E in Definition 1 and the condition on the
linear-approximation error for functions of a single variable in Theorem 1 on
page 249.
necessarily continuous
Recall that a differentiable function of a single variable is

result for a
(Theorem 1, page 96). The following theorem establishes an analogous
function of two variables.

THEOREM 1 Continuity of a Differentiable Function

yo) be an interior point of


the
Let/be a function of two variables, and let (.v,,.

domain of/. Then, if/is differentiable at (.v„. yo). it follows that/is continuous

at (.vq. .V()).

PROOF Let (.V. V) be a point in the domain of/, and let

A.V = .v - .Vo Ay = y .Vo

condition y)-^ Uo. Vq) is equivalent to the condition


Note that the (.v.

(A.V, Ay) -^ (0. 0). Because v = .Vq + A.t and y = yo + Ay. we have

fix, V) = /(.Vo + A.X. yo + Ay)


= /(A-o. yo) +/iUo. .Vo) A-x +/2(.ro, yo) Ay + E

where E is the error as in Definition 1 . Thus, we have

lim E = lim (ei A-v + e, Av) =


I
(A.V.AV1—>(0.01

since both e, and 62 approach zero as (A.r, Ay)^ (0. 0). Therefore.

lim /(.r. 'v) = lim [/(.Vo, yo) +/i(.Xo. .Vo) A-x +/2(xo, yo) Ay + E]
{x,y)—'{xo,ya) (-t,v)~*(.TO.vo)

= /(.To. yo) +/1U0. .vo)(0) +/2(-Xo. yo)(0) + =/(.Xo. yo)

and it follows that /is continuous at (.xq. Vq).

partial derivatives/i(.Xo, yo) and/2(xo, yo) of a function


The mere existence of the
/of two variables does not guarantee that /is differentiable at (atq, yo) (see Prob-
of/ provided by the
lem 46). A condition that does ensure the differentiability is

following definition.

DEFINITION 2 Continuous Differentiability

Let/be a function of two variables, and let U be an open set of


points contained

in the domain of/ We say that / is


continuously differentiable on U if the
ever\' poim (.x. y) in U and the
partial derivatives /, (.v. y) and/.t-x, y) exist at

functions /, and fz are continuous on U.

domain
If the of/ is an open set and/ is continuously differentiable on D. then
D
we say that/is continuously differentiable. The following
theorem shows that the

term '•continuously differentiable" is not a misnomer.

A Continuously Differentiable Function Is Differentiable


THEOREM 2

Let U be an open set contained in the domain of the function/of two variables. It

/is continuously differentiable on L'. then fis differeniiable at each point (.xo. yo)

in U.
856 I HAinKK 14 PARTIAL DIKKKRF.NTIATION

Let (.v„, v'o) be a point in U. Since U is open, there is a circular disk of positive
radius with center (.Vo, Vo) such that all points within this disk are contained
in U. In what follows we assume that |A.v| and |Avl are sufficiently small that

(Vo + A.t, Vo + Av) belongs to this disk and hence to the set U. Define

(1) e,=\ ^x
if A,v =

and
f /(.vq + A.Y. Vo + Ay) -/Uo + A.t. .Vo)
/.(.vo.vo) ifAy^^O
(2) 62 = Ay
I
lo if Ay =

Note that

(3) e, A.V =/(.Vo + A.V. Vo) -/(-Vo. yo) -/lUo. yo) Aj:

even if A.v = 0, and

(4) €2 4v =/(vo + A.V. yo + Ay) -/(.Vo + A.v, yo) -/2(.Vo, yo) Ay


even if Ay = 0. Furthermore,

(5) €| A.Y + 62 Ay
= /(.Vo + A.V. yo + Ay) - f(xo. yo) -/lUo. yo) A.v -/2(.Vo, Vo) Ay

Also, because
/(Vo + AA.yo) -/(-Yq.Vq)
hm = /i Uo , yo)
Aa— A.V

it follows that

(6) lim 6, =
(Aa„1.vI^(0.0>

Now choose and temporarily fix A.v and Ay with Ay t^ and define a function </>

of one variable by

(i>(y)=/(.Vo + A.V. y)

Note that

<})'(y) =/2(.Vo + A.V. y)

By the mean-value theorem (Theorem 3, page 169), there exists a number c be-
tween >'o and yo + Ay such that

Vo + Av) - </>( Vo) - /(-v„ + A.V, Vo + A\) - /(.Vo + A.v, Vo)


^ ,^
<p (t)
^
= <i>(

(yo + Ay) - yo A\'

that is,

/(.v„ -I- A.V. \n + Ay) - /(.Vo + A.v, yo)


(7) /.(.Vo + A.v, c) =
Av

*This proof, which is somewhat technical, may be omitted on a first reading of the section. It

is included here becau.se the theorem itself is rather important and is used extensively in what
follows.
SECTION 14.4 LINEAR APPROXIMATION AND DIFFERENTIABLE FUNCTIONS 857

Note here that the value of c may depend on our choice of Ax and Ay; however,
since c is between vo and Vo + Ay, it follows that c—> yo as Ay-^ 0. From Equa-
tions (2) and (7), we have

(8) €2 =f2(Xo + A.V, c) -/.(.v„, yo)

Because /) is continuous and c^ vq as Ay ^> 0, Equation (8) imphes that

(9) lim e, =
(A.v.Ay)^(O.O)

Now let E be the error involved in the linear approximation

/(.v„ + A.Y. y„ + Ay) ==/(.Vo, yo) + f\{xo. yo) A.v +/2(.Vo, yo) Ay

so that

(10) £= e, A.V + €2 Ay

by Equation (5). From Definition 2 and Equations (6), (9), and (10), it follows that/
is differentiable at (xq, vq).

EXAMPLE 2 Is the function /(x, y) = e^'''^*^' differentiable?

SOLUTION The domain of/ is the entire xy plane, so it is an open set. (Why?)
Here,

fi(.\, V)
= — dx
{e^'^'^n = e'"""*'" —dx
(3.V + 4v) = 3e^''^'^y

/,(.v, v) = —
dy
(e-'^^-*^) = e^'"+-*'" —dy
(3.V + 4v) = Ae^'*^''

so that/i and/2 are continuous on the entire xy plane; that is, /is continuously
differentiable. It follows from Theorem 2 that /is differentiable at each point in its

domain; hence, / is differentiable.

EXAMPLE 3 Is the function /(.v, y) = |.xt| differentiable (a) at (1, 1) and


(b) at (0, 1)?

(a) Inside a circular disk of radius, say, 2 with center at ( 1 , 1 ), both .v and y
are positive, so that f(x, y) = |at| = xy. Thus, inside this disk, /(a', y) =
df(x, y)/dx y and/2(.v, y) = of(x, y)/ch' = x; hence/is continuously differen-
=
tiable. It follows that/is differentiable at each point in the disk. In particular, it
is differentiable at the center point (1, 1) of the disk.

(b) Since /(.r, 1) = \x\ and since the absolute-value function (of one variable)
is not differentiable at 0, it follows that the partial derivative /.(O, 1) does not
exist. However, by definition, a differentiable function must have both of its

partial derivatives. Consequently, /cannot be differentiable at (0, 1).

It can be shown that the sum. difference, product, and quotient of continuously
differentiable functions are continuously differentiable: hence, they are differenti-
able at each point of their domains. In particular, every polynomial function in two
variables is continuously differentiable, and so is every rational function in two
variables.
858 CHAPIER 14 PARTIAL DIFFERENTIATION

The Total Differential

Suppose that / is a function of two variables, and let

r=/(.v. y)

Ifjr and y are changed by small amounts A.v and Ay. respectively, then z changes by
an amount Ar given by

Ar = t\x + A.V, y + Ay) - f(x. y)

Assuming that /is differentiable at (.v, y), we know that the error involved in the

linear approximation

f(x + Ax, y + Ay) «/U. y) +/|(.v, y) A.v +/2(.v, y) Ay

will be small, and it follows that we can approximate Ar as

Ar =/,U, y) A.V +/2U, y) Ay

Using the alternative notation dz/dx and dz/cty for the partial derivatives/i(.v, y) and
/2(jc, y), we can write the approximation as

Ar « —
dz

dx
A.V + —flz

dy
A^•

By analogy with the notation used for functions of one variable in Section 4. 1

the changes A.v and Ay


two independent variables x and y are sometimes
in the

called the differentials of these variables and written as dx and dy, respectively.
Thus, if dx and dy are small, then the change Ar in the value of z caused by
changing .v to .v + ^.v and changing y to y + dy is approximated as

A- = —dz

dx
dx + —dz

dy
^\'

Pursuing the analogy with functions of a single variable one step further, we define
the total differential dz of the dependent variable z by
SECTION 14.4 LINEAR APPROXIMATION AND DIFFERENTIABLE FUNCTIONS 859

E EXAMPLE 5 The volume V of a right circular cone of height h and base radius r
is given by V= sirrah. If the height is increased from 5 to 5.01 centimeters while
the base radius is decreased from 4 to 3.98 centimeters, find an approximation to the
change AV in the volume.

SOLUTION We use the total differential JV to approximate AV^. Here,

dV = —-dh + dr = —777^/! ]dh + —TrPh dr


dh dr M ^3 '
dr ^ 3
'

= 377T- dh + sTirli dr

When /; = 5. dh = 0.01, r = 4. and dr = -0.02, we have

AV == t/V = i7r(4)-(0.01) + f7r(4)(5)(-0.02)

that is, AV = -0.6702 cubic centimeter

(The true value of AV^. correct to five decimal places, is -0.66978.)

Functions of Three or More Variables


The notions of differentiability, continuous differentiability, and the total differen-
tial extend in an obvious way to functions of three or more variables. For instance,
if/is a function of three variables, then a point (.Vo, Vo, -o) is called an interior point
of the domain D of/ if there is a positive number r such that all points (.v. y. :)
within a sphere of radius r with center at (.Vq, Vq, :o) belong to the domain D: that is,
if (at - .Vo)" + (y - Vo)" + (r - Tq)" < r", then (.r, y, r) belongs to D. Similarly, a
point (a, b, c) is called a boundary point of D if every sphere with positive radius r
and center (a. b. c) contains a point that belongs to D and a point that does not. We
say that D is open if it contains none of its own boundary points, that is, if it

consists entirely of interior points. Similarly, D is called closed if it contains all its

boundary points.
Now assume that (.vo, Vq, Zq) is an interior point of the domain D of/ and that the
three partial derivatives

'4 =/i(-Vo, yu. ;o) B = fjixo. yo- :o) C = fsixo. yo. :o)

exist. Let E denote the error involved in the linear approximation

f(xo + A.Y, yo + Ay, zq + Az) ==/(.ro, yo, Zq) +A A.x + B Ay + C 1:


Then we say that/is diff'eremiable at (.Vq, yo, Zo) if there exist three functions 6], e^,
and 63 such that

£= 6] A.T + 62 Ay + €3 Az

and as (A.v, Ay, Az) -^ (0, 0, 0),

6, ^ €2-^0 and e, ^
If the domain D
of/is open, we say that/is continuously difTerentiable if the
partial derivatives/], /2. and /3 are defined and continuous on D. By arguments
similar to those given above for functions of two variables, it can be shown that a
continuously differentiable function of three variables is differentiable at each point
of its domain and that a differentiable function of three variables is automatically
continuous. Again, sums, products, differences, and quotients of continuously dif-
860 CllAl'lKR 14 PARTIAL DIKKKKKMIATION

ferentiable functions of three variables are continuously differentiable. Any polyno-


mial function in three variables is continuously differentiable, as is any rational
function in three variables.
If IV = f(x, y, z), where / is differentiable at an interior point (v, y, r) of its

domain, we define the total differential dw, or df, at (v, y, r) by

dw
SECTION 14.4 LINEAR APPROXIMATION AND DIFFERENTIABLE FUNCTIONS 861

Hence,

l^n'l
< g(9) = 1 ohm
The approximate maximum possible error is ohm. Since u' = 100 ohms when 1

x = y = z = 300 ohms, the approximate maximum possible percent error is


Too ^ 100 percent = percent. 1

Problem Set 14.4

El In Problems 1 to 6. use the linear-approximation procedure to esti- 17 d: if : = x sin y" -I- y sin .v"

mate /(.vo + A.V, v'o + Av) for each function / and the indicated val-
18 d: if _- = In (3.v - 4y- + 5)
ues of JTo, Vo. ^x. and Ay. In each case, check the accuracy of your
result by using a calculator. 19 dw if If = tan"' (2y - 3.v)

1 fix. y) = .\' - 5.V- + 6xy: Xq = 1, Vo . Av = 0.07. 20 df if /(//. v) = cos


.,'"'
,

Av = 0.02 ir + V-

= - Ixy + 3y^ = 21 df if fix. y) = e'"-^'


2 fix. y) .V-' yo= 1. A.V 0.03.
Av = -0.01 —
22 dw if u' = tan"'
- X
3 fix. y) = -v In > y In x: xo = 1 . yo = 1 . A.v = 0.0 1

Ay = 0.02 PV
23 dT if T= R . is constant
= - = = = R
4 fix. y) .vV.v y; .Vq 6. yo 2. A.v 0.25. Ay = 0.25

5 fix. y) = y tan"' xy. Xq = 2, Vo = 2. A.v = - 1^. Ay = - ttj 24 df if fill. V) = sin"' —


V

6 /(.v. y) = f" sin (.V + y); .vq = 2. yo = 3. A.v = 0.008.


= -
25 df if fix. y. 2) xy^ Izx' + 3.XTr-
Ay = 0.01
26 dw if u' =X sin yr -I- y cos xz
In Problems 7 to 12, decide whether each function is differentiable
27 dw if w = xe^~ + ye'^-
at the indicated point. Give a reason for your answer.
28 dw if \r = In (Iv- -I- 3y- -I- 42")

7 fix. y) = xe '
at (.v. y) ;/(.v. y) = \xy-\ at (0, 1)
29 dw if If = e" In yz + e^ In xz
3.XT
9/U, y) rat (1, 2) 30 dw if >f = ixyzt
?*'
,v-' + y
Oln Problems 31 to 40. use total differentials to make each required
1

if .V 7^ and y # approximation.
10 fix. y) at (0. 0)

if either v = or y = 31 The power P consumed in an electric resistor is given by P =


E~/R where E is the electromotive force in volts and R is
watts,

= the resistance in ohms. If, at a given moment, £ = 100 volts and


11 fix. y) cos e'"^' at (1. 1)
.V + y R = 5 ohms, approximately how will the power change if £ is
= decreased by 2 volts and R is decreased by 0.3 ohm?
12 fix. y. ;) \xy:\ at (1 . 1. 1)

32 Find the approximate area of a right triangle if the length of the


In Problems 13 to 30. find each total differential. long leg is 14.9 centimeters and the length of the hypotenuse is

17. 1 centimeters.
13 df if /(.v. y) = 5x^ + 4.v-y - 2y3
33 The length / and the period T of a simple pendulum are con-
14 dzif z = V.v- -I- y- nected by the equation T = IttV l/g. If/ is calculated for T = 1

= e" second and g = 32 feet per second squared, approximate the


15 dw if \v
error in / if T is really 1 .02 seconds and ? is really 32.0 1 feet per
16 dw if w = y In (2 + x) second squared. Also find the approximate percentage error.
862 CHAPTKR 14 PARTIAL DIFFERENTIATION

34 The altitude and diameter of a right circular cylinder are 10 and 6 Prove:
centimeters, respectively. If a measurement of the diameter pro- =
(a) ViMfTJKy)^ <^A,x, Ay) e, A.v -I- c; Ay for all points
duces a t'lgurc 4 percent too large, approximately what percent (A.V, Ay) in the domain of <t>.

error in the measurement of the altitude will counteract the re-

sulting error in the computed volume? (b) lim 6| = and lim e-. = 0.
(Ai.Avl— (0.0) (A-v,Av)—(0.0) "

35 The dimensions of a rectangular box are 5. 6, and 8 inches. If

each dimension is increased by 0.01 inch, what is the approxi- 41 Suppose that Uo. .Vo) is an interior point of the domain of/ and
mate resulting change in the volume? there are constants A and B such that

36 The acceleration of gravity as determined by an Atwood ma- /(.to + ^x, vn + Ay) ==/(j:o- .Vo) + A A.v -^ B Ay
chine is given by the formula g = 82.v/r. Suppose that the true
with an error
values of i and arc s = 48 centimeters and
i = 2 seconds. Ap- /

proximate the maximum possible error in the calculation of g if E =/(.ro + A.r. yo + Ay) - f(xo. Vo) - A \x - B Ay
the measurements of and are subject to no more than percent
A =
i- / 1
that satisfies the conditions in Definition 1 . Prove that
error.
/i(.to. Vo). B = fiixo. Vo), and /is differentiable at (.to, .Vq).

37 Two electric resistors r^ and ri are connected in parallel, so that 42 Let (.to, Vn) be an interior point of the domain of/, and suppose
the net resistance /? satisfies the equation \/R = (l/rj) -I- (l/rj).
that there exist constants A and B and a function of two vari-
<i>

Suppose that r, = 30 ohms and r; = 50 ohms originally, but ables such that
that ri increases by 0.03 ohm while ri decreases by 0.05 ohm.
Approximate the resulting change in R. /(.to + Ax, yo + Ay)
.= /(-Vq, yo) + a Hlx + B Ay -i- V(A.t)- -i- (Ay)- (f> (A.t. Ay)
38 A toy manufacturer, which makes two kinds of toys, produces
holds whenever + A.t, + Ay) is in the domain of/. If
(.to yo
V thousand toys of the first kind and y thousand toys of the
lim </)(A.t, Av) = 0, use the results of Problems 40 and
second kind. They sell for 80 - Iv and 60 - 2y dollars per toy. lAi.Ai)— lU.O)

respectively. The total sales revenue, in thousands of dollars, is 41 to prove that / is differentiable at (.to. yo).
given by the function /(.v, y) = 80.v - Ir" -I- 60y - 2y-. Both
43 Let / be defined by
toys are currently selling for $20 per toy. Approximate the
change in sales revenue in dollars if the price of the first kind of
toy increased by $0.50 and the price of the second kind of toy
if (.t, y) # (0, 0)
is
/(.t. y) = { X- + r
is increased by $0.70.
if (.t. y) = (0. 0)

39 Suppose that the concentration C of a drug in the blood-


stream after injection into a vein is given by Cii. d) = (a) Find/|(.t, y) and/:(.t, y) for (.t, y) # (0, 0).

[1 + (t~ + (/*)""]"', where r is the elapsed time in minutes (b) Find /(O, 0) and /^(O, 0) by direct calculation using the
since the injection and d is the distance in centimeters from the definitions of/y and/i.
point of injection. If ; increases from 3 to 3.01 minutes and d
(c) Show that /is continuously differentiable.
increases from 4 to 4. 1 centimeters, find the approximate change
in C. (d) Explain why /is differentiable at (0, 0).

40 Suppose that is a function of two variables, that (0, 0) is an 44 Define ei and €; by


rf)

interior point of the domain of <i>. and that


1^x lAvl
'
if (A-t, Ay) ^ (0, 0)
lim rf)(A.v, Av) = I ,
|A.t|
I
+!

|Ay|
I I

(A.t.Av)— 10.0)
if (A.t, Ay) = (0. 0)
Define
and
\\x\ V(Al?~r(Ay?
(^(A.v, Ay) if A.V ^ |A.t| Ay
|A.r| + lAvl if (A.t, Av) ¥^ (0, 0)
e, = \\^x\ + |Ay|
if A.V =
[o if (A.t, Ay) = (0, 0)

and
(a) Prove that lim Ci = and lim e-. = 0.
(A-t.Av)— (0.0) (A.i.Ayl— lO.O) "
|Ay| ^/(^x)• -t- (Ay)'
(*(A.v. Av) if Ay 9^ (b) Prove that |A.t Ay| = e, Aj: -I- €2 Ay.
Ay '
|Ax| -I- |Ay|
(c) Use parts (a) and (b) to prove that the function /(.t, y) = It^I

P if Av = is differentiable at (0. 0).


SECTION 14.5 THE CHAIN RULES 863

45 Prove that /(.v, v) = V.v" + y' is not dif'ferentiable at (0. 0). Prove that both of the partial derivatives /|(0. 0) and /2(0, 0)
exist but that/ is not differentiable at (0, 0). [Hint: Show that/
46 Let
is not continuous at (0. 0). and use Theorem 1]

f
/';^
o if (A-, V) # (0, 0) 47 Suppose that (ao, Vu) is an interior point of the domain of/. Prove
fix, y) = i -V- + V-
that / is differentiable at (.Vo, Vo) if and only if there exists a
lO if U. y) = (0, 0) function <f> satisfying the conditions of Problem 42.

14.5 The Chain Rules


In Section 14.3 we gave a version of the chain rule for partial derivatives which was
an immediate extension of the chain rule for functions of a single variable. In this

section we discuss some additional versions of the chain rule for partial derivatives
that are not just restatements of the old chain rule.
The simplest such chain rule is suggested by the notation for total differentials

introduced in Section 14.4. Indeed, suppose that r is a function of the two variables
X and y, say z =/U, y), while x and v, in turn, are functions of another variable t.

so that X = g(0 and3' = hit). Then z becomes a function of the single variable t; that

is, z =figit), hit)). Since

dz = —
dz

dx
dx + —
dz

d\
dy•

we might expect that

dz _ dz dx dz dy

dt dx dt d\ dt

This version of the chain rule is, in fact, correct, provided that/, g, and /; are
differentiable functions. Indeed, let Ar denote a small change in f; and let Ajf, Av,
and Az be the resulting changes in the variables x, y, and z, respectively. Since/
is differentiable, we have

Az = — dz

dx
A.V H
dz

dy
Av + Ci A.v + e^ Av

where e, A.v + €2 Ay is the error involved in the linear approximation

Az == —
dz

dx
A.V + — Av
dz

d\

and
lim Ci = lim e-,
"
=
(Aa,Av)^(0,0) (A.v.Av)-.(0.0)

Dividing by Ar, we have

Az _ dz ^x dz Ay A.v Ay
Ar dx A? dy Ar ' A? ^t
864 CHAPTKR 14 PARTIAI, OlfKERENTIATlON

Taking the limit on both sides as At —» 0, and noting that A.v —> and Ay -^ 0, so
that 6; —» and e^ -^ as Af — » 0, we obtain

dz ()z dx r'lz dy dx dy Hz dx Hz dy

dt (l\ dt fly dt dt dt dx dt cty dt

as claimed.
More t'ormally. we have the following theorem.

THEOREM I First Chain Rule

Let /be a function of two variables, and let /^ and /; be functions of a single
variable. Assume that (xn, yo) is an interior point of the domain of/ and that/
is differentiable at (.v,), Vd). Suppose that .v,, = iido). that y,, = hdu). and that both
g and /( are differentiable at /,). Define the function F(i) = figH). iHt)}. Then F is
differentiable at /(,, and

F'(/„) =/|(.v„. .VoKi;'(/,,) +/2(.v„. yo)h'(tu)

E EXAMPLE 1 If z = \/x~ + y-, x = 2t + 1 , and y = ?', use the first chain rule to
find dz/dt.

dz dx dz dv
SOLUTION + ?(2) + ^Ot~)
dx dt dy dt Vjc- + y V.v- + y-

Ix + 3rv 2(2/ + 1) + 'ht\i^) 3r-' + 4r + 2

Va-- + y- \/(2f + 1)- + (r^)- Vr* + 4/- + 4/ + 1

EXAMPLE 2 If F(r) =/(^{r), /((n), where /(.v, y) = e", ^(/) = cos /, and h(t) =
sin /, find F'(t).

SOLUTION By Theorem 1

F'(t) =MgU). h(t))g'(t) +f2(g(t), hU))h'U)

Here,/|(.v, y) = ye",f2(x, y) = .ve ", g'{t) = -sin t, and /;'(?) = cos I. Therefore,

"•'"
F'(t) = sin t f""' """'( — sin r) + cos t c ^^ '(cos /)

= (cos~ t — sin" t)e'^"^


"^'"' = cos 2/ f""" '
'^'"

EXAMPLE 3 The resistance R in ohms of a circuit is given by R = E/l, where /

is the current in amperes and E is the electromotive force in volts. At a certain


instant when £ = 120 volts and / = 15 amperes, E is increasing at a rate of to volt
per second and / is decreasing at the rate of 20 ampere per second. Find the instanta-
neous rate of change of R.

SOLUTION Denoting time in seconds by the variable f, we have

dR _
SECTION 14.S THE CHAIN RULES 865

„, , dR = \ dE E dl
Therefore, r
dt I dl I- dt

When E = 120, / = 15, dE/dt = A, and dl/dt = - ^, we have

The first chain rule (Theorem 1 ) extends in an obvious way to functions of more
than two variables. In fact, if vv is a function of /; variables -Vj , .v^ x„ and each
of these n variables is, in turn, a function of a single variable t. then

dw
866 t HAPTKR U PARTIAL DIFFERENTIATION

provided that / is differentiabie and that the partial derivatives ctxjdu and dyjdu
exist. Similarly, it/is ditlercntiable and the partial derivatives (ixjdv and ctyldv both
exist, then

dz

THEOREM 2
SECTION 14.5 THE CHAIN RULES 867

EXAMPLE 6 Let /be differentiable at (3. 1) and suppose that/, (3, 1) = 2 and
./2{3. 1) - -5. If V =/(Zr + 3y, e"). find dV/dxmd ^V/,hwhen.v = Oandy = 1.
SOLUTION Let 5 = 2v + 3.V and = e\ so that V = f{s.
/ i) and
dV_
^ dy_d^ dV dt _ dV dV
dx ~ ds^
ds dx dt dx ^^~A
= 2/, (5. t) + eY2(s. t)

When .V = and v = 1 . we have :r = 3. r = 1 . and

—dV
= 2/,(3. l) + e°/2(3. 1)

= (2)(2) + (l)(-5) = -1
Similarly,

dV dV ds dV dt dV dV
d\-

Thus, when x = and v = 1 , we have .? = 3, r = 1 , and

dV
-- = 3/,(3. 1) = (3)(2) = 6
dy-

The second chain rule admits a natural extension to functions


of more than two
vanables. In fact, w
is a differentiable function of
if
variables >,, v,, .... v„, m
and if each of these variables, in turn,
is a function of n variables
x[,xi 'x' ' "'
.
.

• "'
then

dii
^ J^ d\i J>w dy2_ dw d};„
dxj dyj dxj dy2 dxj d\„ dx,

holds for each ;=1, 2 n. provided that the partial derivatives


dyxidxj.dyjdxj dyjdxj exist. The equations above can be written more
compactly as

dw
dw dw
_ Y d}\
fory= 1. 2.

For example, if w =f(x, v, z), .v = g{s. t. u), y = h(s. t. u). and z = p(s. t. u). and
if/ is differentiable, then

()W

ds
868 CHAPTER U PARTIAL DIKKKRF.NTIATION

EXAMPLE 7 Let w= xy' + yz' + zx', x = r cos 6 sin rf>, v = r sin 6 sin <i>, and
z = r cos <]). Find (iwjdr, dw/dd, and dw/c)<i>.

SOLUTION Using the chain rule, we have

dw dw c)x (hv fly flw flz

dr dx flr fly flr flz flr

= (y- + Ivr) cos s\n (i>


+ (Ixy + z') sin sin <f>
+ (2yr + a") cos <^

flw fl^v flx flw fly flw flz

fld
" flx fld fly flO flz fld

= ()•- + 2xz)(-r sin 6 sin (/>) + {2xy + z-){r cos sin (t>) + (2yz + x-){0)

= —(y- + Zxz)r sin 6 sin <f>


+ (2xy + z^)r cos 6 sin (f>

and

(?n' ^vv flx flw fly flw flz

fl^ flx fl<j) fly fl<i> flz fl<i)

= (y~ + Z\z)r cos 6 cos </> + {Ixy + z-)r sin 6 cos <l) + (2yz + a")(— r sin d))

EXAMPLE 8 Suppose that / is a differentiable function at (0. 0. 0) and that

/i(0, 0. 0) = 3,/2(0. 0. 0) = 7, and/,(0. 0. 0) = -2. If the function g is defined by


the equation g{x. y) = fix- - y-. 4a - 4y. 5a- — 5). find gi{\. I) and gzi^. D-

SOLUTION Let u X' — y-.


= = 4a - 4y. i- and w= 5x - 5. and put z =
fUi. V. w). Then z = fix- - v", 4a - 4y. 5a - 5) = ^(a. y). so

gi(x. v) = —=
flz

flx
flz

flu
flu

flx
+
flz

fl\'
fl\-

flx
+
flz

fl^v
flw

flx

= /,(«. v. v.)(lv) +/:((/. V. u')(4) +/3(//. V. w)(5)

When A = 1 and y = \. we have u = 0. \' = 0. w = 0. and

g,(L 1) = 2/,(0. 0. 0) + 4/3(0. 0. 0) + 5/,(0. 0. 0)

= 2(3) + 4(7) + 5(-2) = 24


Likewise.
flz flz flu flz fl\' flz flw
g,(A, V)= = + +
fly flu fly fl\' fly flw fly

= /,(«, V. >r)(-2y) +/:(//. v. u)(-4) +/,((/. v. i.)(0)

= -2yJ](u. V. w) — 4f2(u. V. w)

and so ^2(1. D = -2/i(0. 0. 0) - 4/2(0. 0. 0) = -2(3) - 4(7) = -34

Implicit Differentiation

The procedure of imphcit differentiation, originally discussed in Section 2.8. can be


formulated with more precision and generalized by using partial derivatives.
For instance, given an equation involving the variables x and y. we can transpose
all terms to the left of the equality sign, so that the equation takes the form

fix. y) =
SECTION 14.5 THE CHAIN RULES 869

where /is a function of two variables. This equation is said to define v implicitly as
a function g of a: if

f(x. g(x)) =
holds for all values of x in the domain of g. Assuming that both/and g are differen-
tiable, then by Theorem 1 we can differentiate both sides of the equation
/U. g(x)) = with respect to x and obtain

dx d
/,(.v, gix))—- +/.(.Y, g(x))--g(x) =
dx dx

dv
Mx,y)+f2(x, v)-^ =
dx

where y = g(x). If /2(.r, y) # 0. we can solve the last equation for dy/dx and thus
obtain

dy_ _ /|(.v. y)

dx fyx. y)

EXAMPLE 9 Suppose that y is a function of x given implicitly by the equation


JcV + 3xv^ + S-v^* = 2y + 7. Find the value of dy/dx when jc = 1 and y = 1.

SOLUTION By transposing all terms to the left, we put the equation in the form
f{x, y) = 0, where /(.v, y) = .v-^- + 3x}-- + S.r"* - 2y - 7. Here,

/i(.v, y) = 3.v-y- + 3y- + 2av-' and f.Jx, y) = 2.v'S' + 6.XT - 2

Hence,

dy_ /i(.v, y) 3.v-y- + 3y- + 20.Y-'


dx /2(.v. y) Zv-v + 6xy - 2
Therefore, when x = 1 and v = 1,

dy_ 3 + 3 + 20
dx 2 + 6-2

More generally, given an equation of the form

/U, .V, z) =
involving three variables, may
be possible to solve for one of the variables, say y,
it

in terms of the other two variables x and z. If this solution has the form

then

/U, g{x, z), z) =


holds for all points (x, z) in the domain of the function g. Again, we say that the

equation /(x, y, -) = defines y implicitly as a function g of x and z. Assuming that


the functions / and g are differentiable, we can take the partial derivatives with
870 CIIAIMKK 14 PARTIAL DIKKKRKNTIATION

respect to v and also with respect to r on both sides of the equation /(.v. y, r) = to
obtain

/,U, y, z) —
dx

ax
+/,(.v, y. -)
^ +/,U.
d\

ax
y. z) —=
dz

ax

and

/,(.v. y, z) —
dx

f/Z
+/2(.v, y. z)
d\
-^ + fyx.
r/Z
y. z) —=
dz

rfz

Since v and z are independent variables, we have dz/Ax = 0, dx/dz = 0, Av/Av = 1

and dz/dz = 1. Therefore, we can rewrite the equations above as

/2(.v, y. z) -^ = -/,(.v. y. z) and /.(.v. y. z) -:;- = -/jU, y, z)


Av (9z

Hence, if /2(.v, y, z) ?^ 0. we can solve for dy/dx and dy/dz to obtain

(?\'
SECTION 14.5 THE CHAIN RULES 871

Problem Set 14.5

In Problems 1 to 16, use the first chain rule to find each derivative.
18 and , where w= Ax' + 5xv - Iy' . v = 3r + 5s. and
dr ds

where - = x^\' — 3rv + v'. x = 2l. and v = 6r y = Ir's


dt

2
dw
. where w = e' ', x = sin I, and v = cos t
19
di

du
and —dz

d\'
, where 4.v' — 3.x"v^, X = u cos V. and

dt =
V v sin u

3 , where w = u sin v + cos (u - v). u = .v". and i' = x^ 20 and , where w= w — uv + Sv, u = x cos 2v, and
dx dx dy

dw V = .V sin 2y
4 , where w = V«" — v", u = sin 6, and v = cos
dO (?H' dw , , -> 1 J
21 and , where n' = In (u' + v"). u = x- + \'~. and

5 —dz
dt
, where z = 3m~
,
+ v,
,
u = le . and v = te V
dx
= 2x' +
dy
3xv

— 22 —^ and ^^, where z = e'''. s = n'-, and r = 5x + 2v'


6 , where w = 3a- y', x = s~e \ and v = s^e^^ dx dy
ds

23 and . where u = cosh (3.v + 7v), .v = r-e'\ and


7 , where w = In (.w), x = r'^ . and v = vr + 2 dr ds

dz
where z = e"'^' , u = vx, and v = (x + 3)"*
24 F,(r, .s) and F2(r, s), where Fir. s) = figir. s), hir. s)),

fix, y) = tan"' (v/.v), gir. s) = 2r + s. and hir, s) = r-s

9
dw
, where w = In
—— -
, .v = It. \ - sec t, and : = cot r 25 Fi(u. v) and Fiiii, v), where Fin, v)=figiu, v), hiu, v)),
dt 5:
fix, y) = e" , gill, v) = irv.and hiu. v) = uv

10 . where iv = a^v'z'', x = cos ;, v = sin ;, and z = e


'
dw dw -,

dt 26 and , where w = 6xyz', x = rs, v = 2r + s, and


dr ds

"" T -)
1
. where H = x-y + xz' — yz- + xyz. x = e . y = e . and
du dw dw ^ 7 -, T 7
=
z = cosh u 27 and , where w= 2x + 3v- + z-, a- = m cos v, v
du dv
=
12 —dt
, where z = ue"'' + ve""', u = cos f, v = sinh t. and m'
u sin V, and z uv

28 Fiir, s), and Fjir, s), where Fir, s) = qifir. s). gir. s), hir. s)).
q(x, y, z) = xy- + yz',fir, s) = r cosh i, gir, s) = r sinh i. and
13 F'(0, where F is defined by F(t) = figil), h(l)). hir, s) = re'

fix. y) = sin (.v + v) + sin (.v - _v), g(l) = 3t. and h(t) = /'
dw dw dw
29 , and where h' =x + y — z",
GU) =
,

14 G'{t). where gifU), hit)). g(u, v) = In {u^ + v^). dp de d(j)

fit) = e-'. and hit) = e^'" p sin 4> cos 6, y = p s'm 4> sin 6, and z = p cos 4>

15 F'U). where F(t) = f(gU). hit)), fiu. v) = i(e"^'' - e'"h. dw dw


git) = sinh t, and h(t) = t 30 and , where w = e'' dt. rs*. and v = r'^s
dr ds

16 F'it). where Fit) = figU). hit), pit)), f(x, y, z) = tan"' (xyz),


git) = r, hit) = t\ and pit) = r"*
31 Given a differentiable function / of three variables such that
/,(0, 0, 0) = 4, /2(0, 0, 0) = 3, and /3(0, 0, 0) = 5, let

In Problems 17 to 30, use the second chain rule to find each partial gir, s) =/(; - s, r- - I, 3s - 3). Find g,(l, 1) and gjd, 1).

derivative.
32 Let g and h be differentiable functions of two variables,

17
dz

du
and —dz

dv
. where z = 3x-
,
— 4v~,
,
v = cos u + sm v, and
and define fir, s) = [gir, .?)]'''''". Assume that gi\. 2) = 2,
hil. 2) = -2, ,?,(!, 2) = -1. ,!;.(1. 2) = 3, /i,(l, 2) = 5. and
h.il. 2) = 0, Find (a) /(I, 2), (b)/i(l. 2), and iO f.H, 2).
872 C HAPTER 14 PARTIAL DIKKERENTIATION

In Problems 33 to 36, assume that v is given implicitly as a differen- 44 Suppose that / is a differentiable function of n variables and
ilable function g of x by the given equation fix. v) = 0. that it is possible to solve /(.Vi. .ti -Vj v„) =
hh variable as a differentiable function
(a)
,,
Use ^
the result
dy
—= -
dx
/lU, y)
/lU, v)
to
^
nnd
_, —dy—
dx
for the
of the remaining variables. Prove that
.vj

(b) Find the slope of the tangent line to the graph of y = gix) at the ^Xk ^ /(-vi. ^2 -n x„)

given point (.r. y). dx, /tUi, .vj, . . . , .vi x„)

holds for 1 s / :£ n, i 9^ k, iifkiX], .vi, . . . . x^ v„) 5^ 0.


33 6.V- - llry + 4y= + 2 = 0: ( 1 . 1

45 Let / be a differentiable function of three variables, and let


34 (v- - y-)- - x-r - 55 = 0; (3. 1

W = fix - y, y — 2, z - x). Show that w satisfies the partial dif-

35 sin (.V - y) + cos (x + y) = 0; (7r/4, 77/4) ferential equation

- = dn' dw dn'
36 tan^' (y/.v) + 3f-'"'' 3 7r/4; (1,1) + + =
dx dy dz

In Problems 37 and 38. assume that y is given implicitly as a differ-


46 Let w = g{r, 0). where ^ is a differentiable function. If
cntiable function of the remaining variables in each equation. Find = Vx- + y- and 8 = tan
r '
(y/.v). show that
the value of the indicated partial derivatives when the variables have
the values given. flw
_ X fl^^^ y flw

flx Vx- + y- dr x- + y- dO
37 If.ry-r - 3.\-yr + 0, find ^ dx
and -^ when
dz
v = 1
and

.o
z

,,-
= -
sin(v-.r)
1 , and V =

=
:

cos (.v + v)
— —
fly fl\'
and
ch
when
—=
dw
dy V.v-
, ;
V

+ y-

dw
dr
+ —x
.v"
—y —+
7
dw
dO
38 ; ^ find
It
:' w—3 .

flx
.

dz
,

dw
D47 The height of a right circular cylinder is increasing at the rate of

X = — . V = —4 . ; = 1, and w = —2. 2 centimeters per minute, and the radius is decreasing at the rate
4 -

of 0.5 centimeter per minute. Is the volume increasing or de-


creasing at the instant the height is 10 centimeters and the radius
39 If / is a differentiable function of two variables and iv =
is 6 centimeters'?
f(ay — X, X — ay), where a is a constant, prove that
D 48 The volume V of a right circular cone is given by V =
I
— = (7rr-/3)V'5- - r", where r is the radius of the base and s is the
flx fly slant height. At a certain instant when /• = 4 and 5=10 centi-

meters, r is decreasing at the rate of 2 centimeters per minute


40 If/is a differentiable function of two variables and if a and b are
and s is increasing at the rate of 3 centimeters per minute. Find
constants, show that iv =/(.v + az. y + bz) provides a solution
the rate of change of V at this instant.
of the partial differential equation

49 At a certain instant, the legs of a right triangle have lengths 2 and


dw flw (?M'
= a + b 4 feet, and they are increasing at the rates of 1 foot per minute
dz dx dy
and 2 feet per minute, respectively, (a) How fast is the area of
41 Let M'=/(.v. y). where / is a differentiable function. If the triangle changing? (b) How fast is the perimeter of the tri-

X = r cos and y = r sin 6, show that angle changing?

A A and toward point B 25


/

^
flw \-

dx' ^
/(?>v\-
^ dy
'
_ ldw_Y
~ ^
dr
(
^ r\fle) '
I
^Y D 50 motorcyclist starts at point
miles per hour on a straight-line trail
travels
AB which is 56 miles long.
at

At the same time, a second motorcyclist leaves fi in a direction


42 Let z =f(u, v). where / is a differentiable function. If
which makes an angle of 60° with AB and travels at 30 miles per
= X cos - y sin / and v = sin + y cos where a
II ; .v t I, ; is
hour. How fast is the straight-line distance between the two rid-
constant, show that ers changing at the end of I hour?

51 Suppose the temperature T at the point (.r, y, z) is given by


^dx' \dy' ^ du ' ^d\' '

rt.v, y. z) = .V- -I- 3.XT -I- 3y- + 3;"


43 Let u- = lf(r- - r). where /is a differentiable function. Show
that
A P moves so that its coordinates at time r are =
point .v 3/ -I- 2,
^v dw ni' y = — 2t. and - = 3 - 4/. What is the rate of change
5 of tem-
dr dl t perature at the point P when = 2? /
SECTION 14.6 DIRECTIONAL DERIVATIVES, GRADIENTS, NORMAL LINES, AND TANGENT PLANES 873

] 52 The surface of a mountainous landscape over the .rv plane is Suppose that
given by
z = cos X + sin V + 2

A person is walking on this surface so that at time I the person's


X and y coordinates are x = 3l + (n/i) and y — 4t + (7r/3). At
the instant r = 0, is the person's altitude increasing or decreas-
ing?

EI53 The equation of a perfect gas is PV = kT, where T is the temper-


ature. P is the pressure, V is the volume, and k is a constant.
(The temperature is given in degrees Kelvin, the abbreviation of
which is K.) At a certain instant, a sample of gas is under a
pressure of 2x 10^^ newtons per square meter, its volume is
5 X 10"' cubic meter, and its temperature is 300 K. If the pres-
sure is increasing at the rate of 1.5 x 10'* newtons per square
meter per minute and the volume is decreasing at the rate of
7.5 X 10~* cubic meter per minute, find the rate at which the
temperature is changing.

El 54 The magnitude F of the force of gravity exerted on a rocket by


the earth is given by
GmM
F=

where G is the universal gravitational constant, m is the mass of


the rocket, M (constant) is the mass of the earth, and r is the
distance from the center of the earth to the rocket. When the
rocket is 6500 kilometers from the center of the earth, its mass is

3700 kilograms, it is rising at the rate of 100 kilometers per


second, and it is using fuel at the rate of 45 kilograms per sec-
ond. How fast is the force F decreasing?
874 CHAPTER 14 PARIIAl. OIKFERKNTIATION

Figure 2 In order to find dz/ils. we introduce a unit vector u =


cii + /;j parallel to L and in the direction of motion of P along
L (Figure 2). Iff = (.v, y) is i units from a fixed point P„ =
(.Vo, _v,)) on L. then P„P = .?u; that is,

(a- - .v„)i + (y - .\(,)j


= asi + hsj

Equating components, we have a - Ao = as and y — Vn = bs:


that is. x = Ao + as and y = yo + bs. Therefore,

Pn = < Vn. r„ )
dx
SECTION 14.6 DIRECTIONAL DERIVATIVES, GRADIENTS, NORMAL LINES. AND TANGENT PLANES 875

The vector i makes the angle 6 = with the positive .v axis; hence,

D\Z = — d:

dx
cos H
dz

dy
sin = —
dz

dx

Similarly, the vector j makes the angle d = Tr/2; hence,

D-.z = —dz
cos
TT
1
dz
sin —=—
dz
TT

'
dx 2 dy 2 dy

Therefore, the directional derivatives of z in the directions of the positive x and y


axes are just the partial derivatives of z with respect to x and y, respectively.
The directional derivative D^z can be expressed as a dot product

dz dz dz dz
D^z = a + b = a + b
dx dy dx dy

I dz dz \
= (a\ + bi)-[—x + —i)
^ dx dy '

= u-
I

^
— +—
dz

dx
i

dy
dz
••

The vector (dz/dx)\ + (dz/dy)] whose scalar components are the partial derivatives
of z with respect to .v and y is called the gradient of the scalar field z (or of the
function/) and is written as Vz (or as V/). The symbol V, an inverted Greek delta,
is called "del," or "nabla." Thus, we have
876 CHAPTKR 14 PARTIAL DIKKKRKNTIATION

B EXAMPLE 3 A temperature field in the .vv plane is given by

\ 20 ' Ws '

where z is the temperature in degrees Fahrenheit at the point (.v, y) and distances are
measured in miles. How rapidly is the temperature changing in degrees Fahrenheit
per mile as we move upward to the right through the point (60. 75) along a line L
that is parallel to the vector u = (cos 30°)i + (sin 30°)j'.'

SOLUTION Here u = (cos 30°)i + (sin 30°)j = (V3/2)i + 2J, and

d:
/••
dx
f?.v /h-
(?>• V
\ 20 / V
\ 20 /
' \ 25 / V 25

Putting A = 60 and y = 75, we have

Vr = tiji + ^j
Therefore, the rate of change of z as we move through the point (60, 75) in the

direction of u is given by

Figure 3
V3 \ 6 _ I5V3 + 12
D„z = Vr u = /J_\

10 2 / 25 W 100
0.38 °F/mi

^fixo,yo^
Note that if we fix a point (.v,), Vo) in the .vv plane, then the
directional derivative

DJ\xo.yo) = u-V/(.Yo,.v„)

depends only on our choice of the unit vector u. since the


gradient vector yftxo. Vo) is fixed. If a denotes the angle
tvo.yo' between u and y/'(.V(), y,,) (Figure 3), then by the definition of
the dot product.

u-y/'(.v,i, Vu) = |u| |y/(.Vo. yo)| cos a


Since |u = 1 . it follows that

Ou/(.v„. y„) = |V/(.v„, y„)| cos «

As we vary the angle a in the last formula, we obtain the value of the directional
derivative in various directions at the point (Vo. y^). Taking a = tt/2. we have
cos a = 0. so that D„J\xo. Vq) = 0. Therefore:

1 The directional derivative is zero when it is taken in a direction perpen-


dicular to the gradient.

Since cos a assumes its maximum value, namely 1 , when a = 0, we also obtain the
following:

2 The directional derivative assumes itsmaximum value when is taken it in

the direction of the gradient, and this maximum value is |y/'(.V(i. yo)|.
SECTION 14.6 DIRECTIONAL DERIVATI\ ES. GRADIENTS, NORMAL LINES. AND TANGENT PLANES 877

In other words:

The gradient of a scalar field, evaluated at a point P. is a vector that points in the
direction in which the scalar field increases most rapidly, and the length of the
gradient vector is numerically equal to the instantaneous rate of increase of the
field per unit distance in this direction as we move through the point P.

For instance, if you are at a given point in a temperature field and wish to ""warm
up" most rapidly, then you should move in the direction of the gradient vector at
this point. On the other hand, if you move perpendicular to the gradient, the instan-
taneous rate of change of temperature is zero, and you find yourself moving in the
direction of the isotherm through the given point. Moving in the direction opposite
to the gradient (that is. in the direction of the negative of the gradient), you •"cool
off" most rapidly.

EXAMPLE 4 Find (a) the maximum value of the directional derivative and (b) a
unit vector u in the direction for which this maximum is attained for /(.v, y) =
2x^' + xe^' at the point (1, 0).

SOLUTION Here,

V/(.v, y) =/,(.t, y)i +/2(.r, y)j = (4.n' + e>')i + (Ir- + Ixye'lj

so that V/(l, 0) = i + 2j

Therefore:

(a) The maximum directional derivative at (1. 0) is

|V/(1, 0)| = |i + 2j| = Vl- + 2= = \5

(b) The maximum directional derivative at (1 . 0) is attained in the direction of


V/(l. 0) = i + 2j. A unit vector u in the same direction is

V/(1,0) - i + 2j
u =
|V/(1,0)| V5

IB EXAMPLE 5 The temperature T in degrees Celsius at the point (.v. v) on a heated


metal plate is given by

300
.V- + y- + 3

where .v and y are measured in centimeters.

(a) In what direction should one move from the point (-4. 3) in order that T
increase most rapidly?

(b) How rapidly does T increase as one moves through the point (
-4. 3) in the
direction found in part (a)?

,
(a)V r-T-
vr =
^
^— .

I
+, ^ .
= — ;
-600.V
; ^
.

i + — -600v
-^
J j
dx ay (X- + V- + 3)- U- + V- + 3)-
''
878 { HAPTF.R 14 PARTIAL DIKKKRKNTIATION

Hence, when x = — 4 and v = 3, we have

V/ —
rrr 2400 •
~ IKO
— 784 > 78-

and
lT7-rl
|V/|
_ /, 2400
- >V( .?
+ (~
, / 1800 ^> _

3000 _ 375
— 9W
784 ) 784 ) 784

Therefore, in order to maximize D„T at (—4, 3), we choose u to be a unit


vector in the direction of V7"; that is, we take

= 7'-7J
(b) As we move through the point (-4. 3) in the direction of u, the instantane-
ous rate of change of T with respect to distance is given by

375
£>u7" = |V7"| = 3.83 degrees Celsius per centimeter
98

Figure 4

y
SECTION 14.6 DIRECTIONAL DERIVATIVES. GRADIENTS, NORMAL LINES. AND TANGENT PLANES 879

SOLUTION The curve can be regarded as the level curve /(.v, y) = 1 of the scalar
field z = fix, v), where /(.r, y) = Ijr - Axy^ + y^ . Here

/,(.r. y) = 4.V - 4y-' /.(.v, y) = - I Ivy" + Sy*

and the gradient of/ at (2, 1) is given by

V/(2. l)=/,(2, l)i+/2(2. l)j=4i- 19j

Thus, 4i - 19j is normal to the curve at (2, 1 ). Also, an equation of the tangent line
to the curve at (2. 1) is

4(.v- 2) - 19(y- 1) = 4.V - 19v+ 11=0

Directional Derivative and Gradient in Space


Figure 5 Just as a function of two variables can be regarded as specifying a scalar field in the
plane, a function/of three variables can be thought of as describing a scalar field in
xyz space; that is, we can think of/ as assigning a scalar iv, given by

vv=/(.Y. y, _-)

to each point (,v, y, z) in its domain (Figure 5). Examples are temperature fields,
pressure fields, density fields, electric potential fields, and so forth.
All the ideas and techniques introduced above for scalar fields in the .vy plane
extend naturally to scalar fields in xyz space. For example, if iv =/(.v, y, z), where/
is a differentiable function, we define the gradient of w (or of/) by
880 ClIAniK 14 PARTIAL DIKKKRKNTIATION

E] EXAMPLE 8 The electric potential V in volts at the point P = (x. y. :) in .n'z

space is given by V = 100(.v- + v" + z")""''-. where x. v, and - are measured in

centimeters. How rapidly is the potential V changing at the instant when we move
through the point fo = (2, 1, -2) in the direction of the point P^ = (4, 3, 0)?

SOLUTION The rate of change of V. in volts per centimeter, is given by DuV,


evaluated at Po = (2, 1, -2) in the direction of the unit vector

= NP\ = —V3^ 1

u ,
(I + J
' + k)
\P<>P>\

Here.

= - 100.x:(.v- + V-
'
+ z-)"-'/- = - 100v(.v-
'
+ v"
'
+ z')'^^-
dx cty

and = - 100z(.v- + y- + z~)-^'-


dz

Hence.

VV= i
+ j
+ k= - 100(.v- + V- + z-)~^'-(n + vj + rk)
dx (')y dz

Putting A = 2. V = 1. and z = —2. we find that the gradient at Po = (2. 1. -2) is

VV= -W(2i+j-2k)
Hence, the directional derivative at Pq in the direction of u is

D„V=u-VV= [--^)[-^^] (i+j + k)-(2i+j -2k)

-100 -100
(2 + 1
— 2) = T^ ~ —2.14 volts per centimeter
" ~
27\/3 27\'3

Just as for scalar fields in the .vy plane, the gradient of a scalar field in xyz space
points in the direction for which the directional derivative attains its maximum, and
its length is numerically equal to this maximum directional derivative (Problem 66).

EXAMPLE 9 Let /(.v. y, r) = x}- + xz + yr + xjz. Find (a) the maximum value of
the directional derivative of/ at (8, — I. 4) and (b) a unit vector in the direction in
which this maximum directional derivative is attained.

SOLUTION Here.

V/(.v. y. r) =/,(.v. y. z)i +/,{.t, y. r)j +/3(.r. y. --)k

= (y + - + yr)i + (.v + r + .Yr)j + (.v + y + .vy)k

(a) The maximum value of the directional derivative at (8. —1. 4) is

|V/(8, -1. 4)1 = |-i + 44j - k| = \ (-1)- + 44- + (-1)- = \ 1938

(b) The required vector, a unit vector in the direction of V/(8. -1.4). is given
by
V/(8. -1.4) -i + 44j - k
~
|V/(8, -I. 4)1 Vl938
SECTION 14.6 DIRECTIONAL DERIVATI\ES, GRADIENTS. NORMAL LINES, AND TANGENT PLANES 881

Normal Lines and Tangent Planes


Let /be a differentiable function of three variables. If k is a constant belonging to
the range of/, then the graph in xy: space of the equation

fix. y. z) = k

is called a level surface for/ [or for the scalar field w = fix. y. :) determined by/].
Earlier in this section we showed that, in the .it plane, the gradient vector at a
point P in a scalar field is normal to the level cur\'e of the field at P. This idea can
be extended to level surfaces in xy: space as follows: Let (.to, Vq. :o) be a point on
the surface /(.v, y, z) = k. and assume that V/Uq. Vq. ^o) ^ 0. Then the line contain-
ing the point iX(,. Vq, Zq) and parallel to the gradient vector V/(.Xo, Vo. ^o) is called
the normal line to the surface at (xq. Vq. ^o) (Figure 6). The plane containing the
point (.Vq, 3o, Zq) and perpendicular to this normal line is called the tangent plane to
the surface at (,ro, Vq, Zq) (Figure 7). Because

V/(.Vo. Vn. To) =/i(.Vo, vo, Zo)i + fiixo- .Vo- ;o)j +/3(-Vo. .vo. -o)k

it follows that scalar Cartesian equations of the normal line and the tangent plane
are:
882 I llAnKR 14 PARTIAI, DIKKKRF.NTIATION

that is. V/- T= 0, where

T= — dx
ds
i +
d\
^j
ds
+ —
dz

ds
k

It follows that the unit tangent vector T to a curve C on the surface /U, _v, z) — kis
perpendicular to the gradient vector V/ (Figure 8).

Figure S

Therefore:

The tangent plane to the surface /(.v. y. r) = A at a point F contains the tangent
vectors at P to all (smooth) curves on the surface that pass through P.

In Examples 10 to 12. find equations of {a) the tangent plane and (b) the normal line
to the given surface at the indicated point.

EXAMPLE 10 The sphere .v" + y- + r- = 14 at the point (-1. 3. 2)

SOLUTION Here we put /(.v, y, z) = .x~ + y- + r"^, so that the equation of the
sphere is/(.t, y. z) = 14. We have

f,(x, y, z) = 2.x f.i.x. y. z) = 2y and f^tx, y, :) = li

Hence,

/,(-!. 3. 2) = -2 /.(-I. 3, 2) = 6 and /,(-!. 3, 2) = 4

(a) An equation of the tangent plane at (-1. 3. 2) is

,/,(-l. 3, 2)(.v+ l)+/2(-l. 3. 2)(y-3)+/3(-l, 3, 2)(r - 2) =


that is,

- 2(.v + 1) + 6(y - 3) + 4(r - 2) = or -Iv + 6y + 4r = 28

(b) Equations of the normal line at (—1, 3. 2) are

A + 1 y - 3 _ -- - 2

/,(-l. "/.(-I,
~
3, 2) 3. 2) /,(-l. 3, 2)

that is,

+ V - 3 z-1 + V - 3 r - 2
.V

-2
1
=
6
=
4
or
jr

-13 1

= =
2
SECTION 14.6 DIRECTIONAL DERIVATIVES, GRADIENTS, NORMAL LINES, AND TANGENT PLANES 883

EXAMPLE 11 The graph of z = gCv, v) at the point Uo, Vo, gUo- Vo)). where g is

a differentiable function of two variables

SOLUTION Here we put/(jf, y, z) = g(x, v) - z, so that the graph of z = g(x, y)


is the level surface /(.v, y, z) = of/. We have

fi(x, y, z) = —-
d

dx
lg(x, y) - z] = — d

dx
g(x, y) = g,(.v, y)

f2(x, y, z) = —
d
- lg(x. y) - z] = —-
d
g{x, y) = gjix, y)

and

Mx, y,z) = ^
^z
[g(x, y) - z] = ^
dz
(-z) = -1

Therefore,

Mxo, Vo, gUo, Vo)) = giUo, Vq)

/2U0, .Vo- gUo. yo)) = ^2^0, .Vo)

and
fiixo^ .Vo, gUo, .Vo)) = -1

(a) An equation of the tangent plane at (.vq, yo, g(xo, yo)) is

giUo, yo)(x - xo) + giixo, .vo)(y - yo) + (-l)U - gixo, yo)] =

g(xo. yo) + gi(-V(i. VoX.v - ,V(i) + g2(.Y,i, yo)(y - yo)

(b) Equations of the normal line at {xq. Vo, g{xo, yo)) are

.V - .Vo

giUo, Vo) giixo, yo) -1

EXAMPLE 12 The graph of g(jt, y) = 3A-"*y - lx\ - x- + y + I at the point

(1,2,-6)
SOLUTION We use the results in Example 11. Here we have

gi(x, y) = IZx-'y - 2lA:2y - 2x and gzU, y) = 3jc* -1x^+1


Hence, gi(l, 2) = - 20 and g.Cl, 2) = -3
(a) An equation of the tangent plane is

z = g(l, 2) + g,(l, 2)(x - 1) + g2(l, 2)(y - 2)

= -6 - 20U - 1) - 3(y - 2)

or z = 20 - 20.1- - 3y

(b) Equations of the normal line are

x-\ y-1 z-g(l,2) x-\ y-1 z + 6

g,(l,2) g2(l,2) -20 -3 -1


884 ( IIAIMKR 14 I'ARIIAI, DIKKKRKM lATION

Problem Set 14.6

In Problems I to 22, find (a) the gradient V/of each scalar field, (7r/3, 0, 1) in the direction of the unit vector

(b) the value of V/at the given point, and (c) the directional deriv- u = (cos 27r/3)i + (cos 7r/4)j -I- (cos 7r/3)k

ative Du/a' 'he given point in the indicated direction.


In Problems 23 to 30, find (a) the maximum value of the directional
1 /(.v, v) = 4.v-v\ Uo, Vo) = (2, - 1) in the direction 8 = n/i
derivative and (b) a unit vector u in the direction of the maximum
2 yiv, y) = v- In x. {x„. Vn) = (I, 4) in the direction 6 = 77/4 directional derivative for each function at the indicated point.

3 /(.v, v) = xe- + ye\ (.v,,, y,,) = (0. 0) in the direction 8 = 2tt/3


23 /(A, y) = A- - Ixy -t- 4y- at ( I ,
- 1

4 = y- tan x. = -3) in the direction =


fix. y) (.v,,. y,,) (tt/4. 24 g(A, y) = (A -I- y - 2)- -(-
(3a - y - 6)- at ( 1 , I

nil
55^ h{x, y) = x- - y^ - sin y at ( 1 , 7t/2)
5 /(.v, y) = X In y, Uo, Vo) = (5. I) in the direction 8 = -7r/6
26 F(A, y) = e '' sin
5y at (0, 7r/20)
6 /(.v, y) = (I + at)-''-, (aq, yu) = (K 3) in the direction « =
-7r/4 27 /(A, y, z) = (A- + y- + z")"' at (I, 2, -3)

(t^TU, y) = Ix- - 3v + 4, (All, Vo) = (1. 1) in the direction of 28 h(x. y, z) = (A -^ y)- -(-
(y -(-
z)= -I- (a + zf at (2, - 1 , 2)
u = (\/3/2)i + *j
29 ;i;(A, y, z) = f ' cos (yr) at ( 1 . 0. tt)

8 f(x, v) = x\', (Ao, Vii) = (2, -I) in the direction of u =


'(V2/2)i+'(V2/2)j 30 /(A, y, z) -T at (3, 1. 1)

9 /(A, y) = 2a- + 3y- - 1, (a,,, y,,) = ( 1, 2) in the direction of


u = li - (V3/2)j 31 Find a unit vector u that is perpendicular to the level curve of
= 3A-y — 2Ay" at the point (-2,
u =
/(A, y) 3).
10 /(at, y) = f^', (Aq, y(i) = (l. I) in the direction of
(V2/2)i - (V2/2)j 32 Find a unit vector u that is perpendicular to the level surface of

= A-y + = (-2. /(a, y, z) = Ix- -I- y- -I- z - 8 at the point (2.1, 2).
11 /(a, y) Irv'-, (Ao, Vo) I) in the direction of

u = (V3/2)i - Ij
In Problems 33 and 34, find the direction at the point P in which
12 /(A, V) = (A- + V-)"', (A„, V(i) = (2, 3) in the direction of u = each function increases most rapidly.
13' + 13j
33 z =A-y' + .vy-, P = (-2, I)
$3) /(a, y) = tan '
(v/a). (a„, y„) = (1, -2) in the direction of

V = 4i - 3j 34 W= 3.vy- + 3yz- -I- 3za-, P = (I, I, -2)

14 /(A, y) = In Va- + y-, (Ao, Vq) = (5, - 12) in the direction of


35 The temperature T at the point (a, y) of a heated circular metal
v = '-5i- I2j
plate with center at the origin is given by
15 /(A, y) = y In (v/y). (a,,, Vq) = (2, I ) in the direction from (2, 1

400
to (-6, -2)
2 + A- -I- V-
16 /(a. y) = (' '' cos 4a, (A(,, Vn) = (0, 0) in the direction from
where T is measured in degrees Celsius and a and y are measured
(0, 0) to (-3, 4)
in centimeters, (a) In what direction should one move through
(17 /(A, y, z) = A^y + 3yz-, (Ao, yo, Zo) = (1 .
- • - ' ) in the direc- the point (I, I) in order that T increase most rapidly? (b) How
tion of u = 51 - 3J + §k rapidly does T increase as one moves through the point ( I , I ) in

the direction found in part (a)?


18 /(A. y, z) = In (A- -*- y- -I- r"). (a„, Vo. ro) = ( I • I • D- in the

direction of u = - §1 -•-
3J -t- f [3>36 Marine biologists have determined that a shark detecting the

presence of blood will respond by moving continually in tJie


19 /(a, y, z)= Af'-. (Ao. yo. zo) = (1. ('. 1) in the direction of
direction in which the concentration of blood increases most
V = 2i - j + 2k
rapidly. Suppose that the concentration C of blood (in parts per
20 /(a, y, z) = z tan '
(v/a). (a,,, yn. zo) = ( 1 . 1 . 3) in the direction million of water) is given by
of V = i -I-
j
- k
C(A, y) = f'-«^-=>^'/'»'

21 /(a, y, z) = - c' sin y. (Ao, yn,


z Zo) = (In 3, 37r/2. -3) in the
where a and y are horizontal coordinates measured in meters
direction of u = yi ^- ?j + yk
from the blood source. In what direction will the shark move
22 /(a. y, z) = <' '
sin a -H ^<' '' sin 3a + z' (Ao, yo, Zo) = through the point (3, D? Find a unit vector in that direction.
SECTION 14.6 DIRECTIONAL DERI% ATIVES. GR.\DIENTS, NORMAL LINES, AND TANGENT PLANES 885

37 Let / be a differentiable function of two variables such that 60 z = In cos Vx- + y- at (0. 0. 0)
/i(-l, 2) = 2 and /.(-I. 2) = -3. Find the directional deriva-
61 z = In V.r- + y- at (0, - 0)
u = (\/2/2)i -
1 .
tive DJ"(-1. 2) if (V2/2)j.
62 z = xe'- at (1. 0. 1)
38 Suppose that / is a differentiable function of two variables and
Ou/(3, 4) = -4 if u = ifei - Hj and DJO. 4) = 5 if v =
63 Assuming that / and g are differentiable functions of three vari-
Hi + ftj- Find (a)/, (3, 4) and (b)/:(3. 4).
=
ables and that a and b are constants, show that V(a/ + bg)
/SSOSuppose that the temperature 7" (measured in degrees Celsius) at aVf+bVg.
a point (x, v, z) in space is given by
64 Assuming that / is a differentiable function of three variables,
7"(.Y, y, z) = 100 - x- - v- - 2z- prove that the rate of change of the scalar field ii- = fix. y, z)
with respect to distance measured in the direction of the unit
where x, y, and r are measured in centimeters, (a) Find the rate
vector u is given by u • V/.
of change of T at the point P= (2, 1 . 1 ) in the direction of the
vector u = gi + Ij - |k. (b) In what direction should one 65 Suppose th at/is a differe ntiable function of a single variable and
move from the point P in order to cool off most rapidly? (c) that r = V.v" + y" + z". Verify the formula
What is the maximum rate of change of T at PI
xi + yj + zk
V/lr )=/'{/-)
(5)40 Suppose that the electric potential V (in volts) at a point (x, y, 2)

in space is given by
66 Prove that the gradient of a scalar field in xyz space points in the
V{x. y. z) = In Vx- + y" + z' direction for which the directional derivative attains its maxi-

(a) Find the rate of change of Vat the point/' = (2. - 1. 2) in the
mum and that its length is numerically equal to this maximum
directional derivative.
direction from P toward (0, 0, 0). (b) In what direction is the rate
of change of V at P a maximum? (c) What is the maximum rate 67 Assume that / is a function of two variables which is differenti-
of change of V at P? able at (-Vn, yo). Give a geometric interpretation of the differen-
tial (// in terms of dx. dy. and the tangent plane to the graph of
In Problems 41 to 62. find equations of (a) the tangent plane and /at Uo, yo./(.vo, .Vo))-
(b) the normal line to each surface at the point indicated.
68 Supfwse that a point P is moving along a smooth curve C in a

^ X- + 2y- + 3r- = 6 at ( 1 . 1 . 1
scalar field
field 11- is
if. Assuming
differentiable. that
that the function that gives the scalar

T is the unit tangent vector to C,


42 X- - 2y- + r-= 11 at (2. 1, 3) and that P is moving with the speed ds/dt, prove that the instan-
taneous rate of change of u with respect to time as measured at
(^xyz = 6 at (1, 2, 3)
the point P is given by
44 8.\ - y' = at (2, 4. 7)
ds ds
4£)a:' + y- - 6.n- + r = at (2. 2. 8)
T-Vw Dtw

46 sin.vr = e" at (1. 0. 77/2)


In Problems 69 to 72. use the result of Problem 68.
47 Ix- - 3y- - - = at (2, 1, 5)

- 69 The temperature T at any point (.v. y of a rectangular plate lying


-4r- =
)

48 .V- 3y- 2 at (3. 1. 1)


in the .vy plane is T=x sin 2y. The point P = (x. y) is moving
49 .V- + 2y- - 5r- = 4 at (1. 2. 1) clockwise around a circle of radius 1 unit with center at the

origin at a constant speed of 2 units of arc length per second.


50 vGc + Vy - z = at (4, 9. 5)
How fast is the temperature at the point P changing at the instant

51 cos (xy) + sin (yz) = at (1, 77/6, -2) when (.V, y) = (i, V3/2)?

52 In xy + sin yz = 2 at (e. 1, tt/2) 70 Suppose that the surface

53 V^ + Vy + V^ = 6 at (9, 4. 1)

5280
54 tan"' (y/.v) - In xyz = 7r/4 at (1. 1. 1)

represents an uneven terrain and that a group of tourists is con-


55 z = ^.r\"- at (2. 1. 4)
gregated at the origin. Here, .v, y, and z are measured in miles. A
56 z = 5.vV - 3y at (1, -1. 8) Moslem tourist sets out for Mecca, going directly east along the
positive direction of the a axis. If he travels at a constant speed
57 z = V9-.r--y- at (-1. 2. 2)
of 3 miles per hour, how steep is his climb, in feet per minute, at

58 z = .V sin (7n/2) at (0. 0. 0) 59 z = x> at (1. I. I) the end of I hour?


K86 C HAPTKR 14 PARllAI. DIFFERENTIATION

71 The pressure P. in newtons per square meter, of a certain gas at


Figure 9
'''*'>' *-"',
ilie point (A. y. :) in space is given by P = lOOOf
where .v, y, and r are measured in centimeters. A variable point
Q = (.V. y. :) is moving along a certain curve C in space. At the
when = (1, -2. ). its speed is given by cis/dt = 50 V/x \g
instant ^ I

centimeters per second. If the unit tangent vector T to C at

(I, —2, 1) is given by

T = cos
I

f)-('
find the instantaneous time rate of change of pressure at C? just as

Q passes through (I. -2. 1).

72 The electric potential V in volts at the point (.v. y, :) in space


caused by pla cing a charge q at the origin is given by K = q/r.
where r = V.v" + y
+ z'. (Assume that distances are meas-
In Problems 74 to 77. use the result of Problem 73 to find a tangent
ured in centimeters.) Let C be a curve in space having a unit
vector at the given point to the curve C in which the two surfaces
tangent vector T at the point P. MP moves along C in the direc-
intersect.
tion of T at a con.stant speed of \' centimeters per second, write
an expression for the instantaneous time rate of change of elec-
74 X- - y- - z- + 12 = and S.v" -I- y- -I- r = 4 at ( 1 . 2. -3)
tric potential at P.

75 "xz + Zx + Az = 5 and 4.vy + 3y -I- 6z = 56 at (2. 5. g)


73 Suppose that two surfaces in xyz space, say /(.v. y, z) = kx and
= 76 .V- + (yV4) - (Z-/9) = 1 and x' + y- + z' = 14 at (- 1. 2. 3)
g{x. y. z) A.1. intersect in a smooth curve C (Figure 9). As-
suming that /and i; are differentiable and that (.Vq. y'o. Zo) 's a 77 .V- - Ixz + y-z = I and \xy + 2yz = -6 at ( 1 . -2.0)
point on C. show that the vector

V/(.v„. y„. X 78 Let /(.v. y) = xy-/{x- + y^) for (.v. y) t^ (0. 0) and /(O. 0) = 0.
.-,,) V^C.vo. yo, :o)
Show that £>u/(0. 0) exists for all unit vectors u. but that/is not
is tangent to the curve C at the point (xo, yo, Zo)- continuous at (0. 0).

14.7 Higher-Order Partial Derivatives


In studying functions of a single variable, we found it useful to consider not only the
first derivative but also higher-order derivatives. Similarly, in studying functions of
several variables, it is useful to consider higher-order partial derivatives.
Thus, consider a function/of two variables having partial derivatives/] and/2, so
that

/,(.v, y)=/,(.v, y) fix. y) and /2(.v, y)=/,(.Y, y) = —


ax
/(.v, y)

The functions/] and/2 are functions of two variables, and they may themselves have
partial derivatives. For instance, if /(.v, y) = iv'y' + 6xy~. then

/i(.v, V) = /;(.v, y) = — dx
+ (3.v-v' 6.vv=) = 6.n-' + 6v-

Ux, y) = /;.(.v, y) = — OxY + 6.vy-) = 9.v->- + \Zxy


dy
SECTION 14.7 HIGHER-ORDER PARTIAL DERIVATIVES 887

Therefore,

_d_
[^^'-•^^] 6y'
dx Ax

d
Mx.y) = ^crv >-
^-/f-^- y)
Ax J
= --
(9v
(6xv-'
'
+ 6v') = \S.xy^ + 12v

fiix, y) — {9xY +
, /{-v. y) =
[^^^--^^]
Av -i
I

dx
IZw) = 18xv- + I2y

—fiix. y) = -/u. =— y) (9.vV 12xy) = 18x-v + \2x


Ay i[i^<^'-^']
The four partial derivatives of partial derivatives found above are called the
second-order partial derivatives of the original function /. Naturally, we can
denote the partial derivatives of the function /i with respect to the first and second
variables as (/, ), and (/,)2, respectively. However, for simplicity, we omit the
parentheses and write these second-order partial derivatives as/n and/12, respec-
tively. Likewise, we write /^;r and/^^, rather than (/,)^ and (/;),.. respectively. For
instance,

/l!-/„ =
A)> \dx/
^dx

The symbolism —d
dy
(
\
\
3f\
^-
dx I
is also abbreviated
dv dx
d~f
much in the same way that

d-y
Y is used as an abbreviation for the ordinary second derivative. Similarly, we
dx

write
dx-
'— for the second-order partial
"^ derivative — — dx ^dx'
, and so forth. In sum-

mary, the four second-order partial derivatives of/ can be written as follows:

/.I
888 CIIAPIKK 14 PAKTIAI. DIKKERENTIATION

from left to rif-h! shows the order of partial differentiation, uhilc in the notation

dx dy

we must read the


"

'denominator" " from right lo left.

EXAMPLE I If /(.v. y) = 7.V- - 13.VV + ISv', find /, , /;, /,, , /,.. /:, , and /jj.

SOLUTION Here,

/,(.v. V) = — Ax
''

(7.V- - 13.VV + 18v-) = 14.x: - 13v

f.Jx. v) = — (7.V- - 13.VV + 18v-) = -13.V + 36v


rh'

Hence. /|,(.v. v) = —
cl.x
[/'lU, v)| =— Ax
(14v - 13v) = 14

/,,(.v. V) = — l/|(.v. v)l = — (14.V - 13v) = -13

/2i(.v. y) = — |/2(.v, v)| = — (- 13.V + 36v) = - 13


dx dx

/22(.v. V) = -^ [f.J-x, v)l = — (-13.V + 36v) = 36


e/y dy

EXAMPLE 2 If /(.v, V) = Ic' COS v. find


df
-^. -^.
dx
df
ch;
—d-f4r.
dx-
——.
d-f
dy dx
d-f
dx d\
—. and

SOLUTION Here.

—=— dx dx
(2e cos v) = Ae^ cos v

—=— df
d\
d
d\
,

(2e-' cos v) = -2c-' sin v

Hence, —- = — 1^1 = —
^'f
dx~
d

dx
I

^ dx
df\
' dx
d
(4c'-'
,
cos v) = 8e-' cos y

d-f
dy dx
— — =— =
d
dy
d\-
I
I
V
^ dx
df\
I

I' dv
d\
d
(4e-' cos v) = -4e-' sin v

—— = — -^ = —
d-f
dx dy
d
dx
I

^
df\
dy '
d

dx
(-2c-' sin V)) = -4c-' sin y

d-f
dy-
— =— d
dy
I
I

^
df
dy '
I

dy
(-2('-' sin v) = —2c-" cos v

EXAMPLE 3 Let /I, in. and k be constants, and suppose that /(.v, v) = Lx"y"'. Find

—^f
dx
.
—^
dv
,
—d-f
T-
dx- d\-
d'-f

dx
.

dx dv
d-f
. and —d'-f

d\'-
^.
SECTION 14.7 HIGHER-ORDER PARTIAL DERIVATHES 889

SOLUTION Here,

—- = -—
dx dx
(kx"\'") = knx" 'v' and —=—
of d
(kx"\^) = kmx"\'

d
Hence. (hxx" 'y'") = Av!(« - l).r""V
dx- dx ^ dx

( knx" ' \"' ) = knmx" " ' \-™ " '

d\- dx d\ ^ dx d\

I d I df\ d
dx ay-
•A' dx ^ dy '
dx

f d I df\ d
T = —[-Zrr^^ {kmx'W" ') = km{m - l).r"v'

The second-order partial derivatives

and
d\- dx dx d\-

are called the mixed second-order partial derivatives of/. In all three examples
above, notice that the mixed partial derivatives are the same. By Example 3. the
mixed partial derivatives of any term in a polynomial function of two variables are
the same; hence.

d-f _ d-f
dv dx dx dv

holds for any polynomial function f of nvo variables. As a matter of fact, the two
mixed partial derivatives are equal for a wide class of functions.*

THEOREM 1 Equality of Mixed Second-Order Partial Derivatives

Let/ be a function of two variables, and suppose that U is an open set of points
contained in the domain of/. Then if both of the mixed partial derivatives/,! and
/21 exist and are continuous on U. it follows that/i2(.v. v) =/2i(.v, v) for all points
(X. v) in [/.

The proof of this theorem, which depends on the mean- value theorem for func-
tions of one variable, is usually presented in advanced calculus courses and is not

given here.
The notation used for partial derivatives of order higher than 2 is almost self-
explanatory. Thus.

^y
f'-^-'-f>y^
d,- dx\dy'-l

d'f
- /9\-- V
=
/!2122=A,
d\-- dx d\- dx d\'-

*In Problem 54 we give an example of a function for which the mixed partial derivatives are
not equal.
890 ( HAIIER 14 PARTIAI. DIFFERENTIATION

and so forth. Theorem 1 on the equality of mixed partial derivatives extends to such
hiijher-order cases. For instance.

d\ dx
d'f
ct\
y
d\- dx

holds on a set such as U in the theorem, provided that all three mixed partial

derivatives exist and are continuous on U. In summarv:

The order in which successive partial derivatives are taken when you form
higher-order partial derivatives is irrelevant, provided that all the partial deriva-
ti\cs in question exist and are continuous.

EX.WlPl.F. 4 Let/(.v, y) = e" + sin (.v + v). Find

(a) Arv (b) /v,, (c) /,„..„

SOLUTION

(a) /t(.v, v) = ye"^' + cos (.v + y), and

/„(.v, v) = — [
\£'" + cos (.V + v)] = V"?" — sin (.v + v)

so that

fx.xy{x. v) = —dy
[ y-f" — sin (x + v)]

= 2ye" + .rv'-e" — cos (x + y)

(b) fy(x. y) = xe" + cos (.v + y), and

/vv(.v, v) = — l-Vf" + cos (.V + y)] = .v-f" — sin (x + v)

so that

fwvx(x, v) = —Av
- [.»:-(?" - sin (.v + v)]

= 2j:e"' + yjf"e" — cos (.v + y)

(c) Since /„v(-'f' }') — 2ye" + xy'e'^ — cos (.v + y), we have

fx.nx(x, v) = —
dx
[2ve>** + .n-e" - cos (.v + v)]

= 2y"c'" -t- y"e" + .v\"V" + sin (.v + y)

= 3y~e" + xy^e^^ + sin (.v + y)

so that

/cvvcv(v. ^•) = —
dx
(3vv'^ + .n-\'" + sin (.v + v)]

= 3y-^f " + y'c" + .xy'*«'" + cos (.v + y)

= 4yV" + jry'^e" + cos [x + y)


SECTION 14.7 HIGHER-ORDER PARTIAL DERIVATINTS 891

Naturally, the concept of higher-order partial derivatives


extends immediately to
functions of more than two variables. For instance, if w = f{x. v, r), then

Sx d\ dz dx

^^''
dx dz- d\
d^-
~ dx L <?.-- W/J ~^'-'^'

and so forth. Furthermore, the conditions for the equality


of mixed partial deriva-
tives remain effective.

EXAMPLE 5 Let >v = jcVz -I- sin .r\'. Verify by direct calculation that

d^w fiw
dx dy dz dz dy dx

SOLUTION Here, —— '^*'


f"*'
4
^- = .vV
X \-
. ,
7
and
_,
—d-w
—- =
d^w
Iv^^-
dz ' d\- dz

^° *^' = S.v'v. On the other hand,


-1 . '

dx .:>.
d\- dz

— - 4x-y-z + y cos .xi' and -7— = 8.v\- + cos .x:^'


- x}- sin .n-

d^w
so that = 8.v^^ Therefore.
dz d\' dx

d^w „ , d^w
dx dy dz -
dz dy dx

Higher-order partial derivatives are usually calculated by


successive differentia-
tion, as in the last example. At each stage of such
a calculation, the standard
differentiation rules, including the chain rules, are
effective. The use of the chain
rules in this connection is illustrated by the following example.

EX.A.MPLE 6 Let w = iri\ where u = x- + 3y- and v = Zv" - y-. Find

d^w
dx dy

SOLUTION By the chain rule.

d^f _ dw du dw dv
~d^~'d^^^l7^^ (2m-^)(6y) + (3«V)(-2y) = (12«v-' - 6u\'-)y
Therefore,

d^w
dx d\-
= —d [(12//v-^
:,

- ,
6u-v-)]y
1
=
r
[12 —o"

iuv') - 6 — /)

(«-v-)jy
1

= (12(2\-i- -I- 12«v-.v) - 6{4u.xT- + 8M-i-.v)]y

= 24(v-^ -I- 5uv- - 2u-v)xy 1


892 ( HAPTER 14 PARTIAL DIFKERtNTIATION

Problem Set 14.7

Problems 29 show
In Problems 1 to 18. (a) find -^.
r)x-
(b) find ~.
(?>"
(c) find —7^
dy dx
In to 32. that

partial differential equation in two dimensions, namely.


each function/ satisfies Laplace's

d^f a^f Srf


^— <?/
(d) find , and (e) verify that
dy dx dx dy
Ar^
2+2=0„ <?-/

dy^

1 /(.v. y) = 6.V- + 7.n' + S.v"


29 /(.v, y) = f' sin y + f' sin x
2 y) = 4.v-y-' - .vS' + .v
-
/'(.v.
30 /(.r, y) = e-'(cos- y sin" y)

3 f(x. y) =X cos y - y' 4 /(.v, v) = tan '


- 31 /(.v. y) = In (x" + y^)

.t + V 32 /(.r, y) = tan"' —
5 f(x. y) = (X- + y^f'^ 6 /(.v, y) =

= = —
7 /(.v. y)
^ 8 /(,r. V) In
y In Problems 33 and 34. show that

partial differential equation in three dimensions, namely.


each function/satisfies Laplace's

9 = y cos - xe^" 10 /(,v. = e^


^"^
/(.v. y) .v y)
d-f d-f d-f
11 /(.(. y) = sin (.V + 2y) 12 /(.v. y) = <-'•"'
- 2y^ =
dx- dy- dz-

13 /(.v. y) = S.vcosh 2y
33 fix. y. z) = 3.V- - 2y- + 5.rv + 8.vz - z'
14 fix, y) = In sin VPTy
34 fix. y. z) = ix- + y- + z")"'/-
15 fix. y) = 3f^V 16 fix. y) = In V.v' + .

= e" = 35 Let/U, y) = cos ix + y) + cos (.r - y). Show that /„(.v. y) =


17 fix. y) sin y 18 fix. y) sin' .vy
y).'
/vvU,

In Problems 19 to 26. find each partial derivative. 36 Let fix. y) = sin (v - y) + In (v + y). Show that /ii(,r. y) =
/22U, y).
19 fix. y. z) = xy-z + 3.v=e>: (a)/„(.v. y. ;). (b)/,.-(.v. y. z)
37 Show that if = e"'' cos a satisfies the partial differential

20 ^(.v. y. z) = cos i.xyz-y. (a) g,,-(r. y. z), (b) g,.-(.v. y. z) equation

21 /i(.v. y. z) = xe> + ze' + c"'"'; (a) /im:(.v. y. z). (b) /i^isU. .v. z)
—<?"«'

dy-
; 2
d'w
dx-
^= 4v-vf
-,

22 fix. y. z) = .v-e~-* sin z:

(a)/,i,(I. -I, 77/4). (b)/:2,i(l. "l- V4) 38 Assuming that the functions / and g are twice differentiable,

show that, if w = fix + y) + gix - y). then


23 /(.v. y. z) = ^->--; (a)/„,,(I. 2. 3). (b) A„,(l. 2. 3)
d'w d-w
24 fir. s. t) = In (5r + 8i - 6f-); (a) ./iitr. 5. /), (b) /,,2('-. s. I)
~
dx- dy^
d^h d*h
25 /;(.v. y. z) = sin .v"y (a) (b) 39 If w = Ax^ + Bxy + Cy- + Dx + Ey + F. determine the condi-
dx d\- dz dx dy dz-
tions the constants A, B. C. D, E. and F so that w satisfies
on
d-f s-f Laplace's partial differential equation in two dimensions,
26 fix. V. z) = V In (.r - esc z). (a) —-7-, (b) , , namely
d\ dz dx dz

27 If w = (Av" + By")\ where /I and B are constants, verify that


d'w
dx-
T+ —^ =
d'w
dy-

40 A function / of two variables is said to be homogeneous of


<?.t^ ^ d)' dx^ degree n n) = t"f{x. y) holds for all values of .v. y. and t.
if/(/.v.

(a) Give an example of a function that is homogeneous of degree


by direct calculation.
I. (b) Give an example of a function that is homogeneous of

- degree 2. (c) Suppose that/ is differentiable and that it is homo-


28 w = + + show
If In (.V V) tan ix v). that
^ geneous of degree n. Prove that xf\ix. y) + yfzix. y) = nfix, y).
SECTION 14.7 HIGHER-ORDER PARTIAL DERI\ ATI\ES 893

41 Let w = f(u. v). where u = x + y + z and ;• = 2x + v — z. As- 49 Let w =fiu), where u = g(x. y). Assuming that the required
suming the existence and continuity of the required partial deriv- derivatives exist and are continuous, express

atives oi /. rind a lormula tor a^w d-w


dx- <?\'-

dx- dv-

42 If/ is a twice-differentiable function and c is a constant, show


in terms of/'./", and partial derivatives of g.

that w = fix - ct) is a solution of the wave equation, —r^r =


50 If u' = fiu. v). u = g(x. y). and v = h(x. y). show that

, d-w
c —. Also, show that if =/(.v + cr) is a solution of this d-w
dx-
dy dx
equation

43 Show that n- = (a cos ex + b sin cx)e . where a. h. c. and k


dw d-iv
are constants, is a solution of the heat equation ^— = k '

dx-

44 Suppose that »• =/(.v, v. z) and let a = p sin <^ cos 0, y =


p sin (t> sin 6. and ; = p cos <l>. Thus, if (p, 0, 4>)
=
(4, 7r/3, 77/6). then U, j, z) = (1. V3, 2V3). Suppose that/
has continuous partial derivatives and that

/,(1, V3, 2\/3) = V3 /(I, V3, 2^3) = 2

/„(1. Vs. 2\/3) = 4 /„.(1, V3, 2\/3) = -1

/„.(I, VI, 2V3) = 4 /,-(!, V3, 2 V3) = -V3


/,(!, VI. 2V3)= 1

/,.-(!, VI, 2\^) = 2

/;,(!, V3. 2V3)= -1

Find

IT ItA
(a) (b)
d(b <9p (i^rf)

45 If H- =/U, y), ,v = /• cos 0. and y = r sin 6. show that

(9"U' d'w d'w 1 d-w 1 (?lf

dx' d\' dn

46 If z =/U, y), .r = g(f). and y = /i(r), show that

dj__^^(dx_\- ^ d-z dx dy d-z I dyV


dr dx- ^ dt ' ~ dx dy dt dt df \ dt '

dz d'x dz d-y
dx dr d\- dr

47 If: fix. y). X = e'^ cos 6. and y = e' sin 6. show that

d-z d-z _ _-,( d-w d^w \

dx- dy-
~ '^
\ dr dO^ I

48 If u- = fill. \). u = g(.r, y). and v = h(x. y), show that

d-w ^ d^w I (?i<\- / ^-H' d-w \ du d\'

dx^ du^ \ dx' \ dv du du dv ) dx dx

d^w / dv\- dw d'u dw d-v


+ -{ + r +
dv- \dx/ du dx- dv dx-
894 CHAKIKR 14 PARTIAL DIFKKRKNTIATION

14.8 Extrema for Functions of


More Than One Variable

In Chapter 3 we saw that the derivative is an indispensable tool for solving problems
involving extrema of functions of single variables. In this section we study the
problem of finding maximum and minimum values of functions of more than one
variable, and we see that partial derivatives are especially useful in this connection.
The basic concepts, such as relative extrema, absolute extrema, critical points,
and so forth for functions of several variables are the natural analogs of the corre-
sponding concepts for functions of one variable. For simplicity, we formulate these
concepts for functions of two variables; it should be obvious how they can be
extended to functions of more than two variables.

DEFINITION 1 Relative Extrema

(i) A function /of two variables has a relative maximum value /(a, b) at

the point («, b) if there is a circular disk with positive radius and with center
at (a. b) such that if (x, y) is a point in this disk, then (.v, y) is in the domain
of/ and /(.v, y)^f(a, b).

(ii) Similarly, a function/ of two variables has a relative minimum value


f{a. b) at the point («, b) if there is a circular disk with positive radius and
with center at (a. b) such that if (.x, y) is a point in this disk, then (.v, y) is

in the domain of/ and /(.v, y) ^f(a, b).

(iii) A relative maximum or minimum value of a function is called a rela-

tive extreme value, or a relative extremum, of the function.

Figure I

U^.h,

Consider, for instance, the surtace in Figure 1 as the graph of a function /of two
variables. The point P at the apex of the "hiir" represents a relative maximum of/,
since/(«, , b^ ) — the height of P above the xy plane — is larger than all nearby values
of/(.v, y). Similarly, the point Q at the bottom of the "hollow" corresponds to a

relative minimum, /(a^, bj,).

The point 5 in the "pass" is neither a relative maximum nor a relative minimum:
indeed, we increase our height above the xy plane as we move from S up the hill
SECTION 14.8 EXTREMA FOR FtNCTIONS OF MORE THAN ONE VARIABLE 895

toward P. whereas we decrease our height above the xy- plane as we move from 5
down toward the point R on the "rim"" of the surface. Incidentally. R does not
represent a relative extremum of /because there is no circular disk with positive
radius and with center at (04. b^) that is contained in the domain D of/. Notice that,
by definition, a relative extremum of a function /can occur only at an interior point

of the domain D of/ never at a boundary point of D.
Sometimes it is possible to locate the relative extrema of a function/of more than
one variable by algebraic considerations or by sketching the graph of/. This is

illustrated by the following example.

EXAMPLE 1 If /(.v, V) = 1 - .V- - y-, find all relative extrema of/.

SOLLTIO.N Here, f{x. y) = 1 - (.v" + v") and x- + y' > 0, so that/(.v, y) < 1

for all values of .v and y. Since /(O, 0) = 1, /attains a relative maximum value of 1

at (0, 0). The graph of/, a paraboloid of revolution about the : axis, clearly shows
that /has no other relative extrema (Figure 2).

Figure 2 Figure 3

relative ma.\imuin

Suppose that / is a function of two variables which has a relative maximum


c =f{a. b) at the point (a. b) (Figure 3). Let C be the cross section cut from the
graph of/by the plane y = b. and notice that the curve C attains a relative maximum
at the point (a, b). Therefore, if C is a smooth curve, it must have a horizontal
tangent line at (a, b. c), by Theorem 1 on page 1 12. Since the slope of the tangent

line to C at (a. b, c) is given by the partial derivative /i (a, b). then/i(a, b) = 0.

By cutting the surface in Figure 3 with the plane x = a and reasoning similarly,
we can also conclude that if/2(a. b) exists, then/2(a, b) = 0. Likewise, if/ has a
relative minimum at (a, b) and if/i(a. b) and/2(a, b) exist, then/i(a, fc) = and

fjia, b) = 0. Thus, we have the following theorem.

THEOREM 1 Necessary Condition for Relative Extrema

Let {a, b) be an interior point of the domain of a function/, and suppose that the
two partial derivatives /, (a, b) and /.(n, b) exist. Then if / has a relative
extremum at {a. b). it is necessary that both /(a, b) = and /2(a, b) = 0.
896 CHAPTER 14 PARTIAL DIKKKRKNTIATION

Note that the condition /)(«, b) = h(a, £>) = in Theorem 1 can also be ex-
pressed as Vf(u. b) =
=/i(a, b)\ +/2(a. b)y Therefore, at a point
0, since V/(a, b)

where a function attains a relative extremum. its gradient must either fail to exist or
be the zero vector. Thus, we make the following definition.

DEFINITION Critical Point

Let/be a function of two variables. A point {a, b) in the domain of /such that

either ^fUi. b) fails to exist or V/(a, b) = 0. is called a critical point of/. A


critical point of /which is in the interior of the domain of/ is called an interior
critical point of the domain of/

Now, Theorem 1 can be restated thus: If a function f has a relative extremum at


a point (a, b), then (a, b) tnust be an interior critical point of the domain off.
Therefore, to locate all the relative extrema of/, begin by finding all the interior

critical points of its domain. Some of these critical points may correspond to relative

extrema; however, some may occur at "saddle points." that is. critical points where
the function has neither a relative maximum nor a relative minimum. The point 5 on
the surface in Figure 1 corresponds to a saddle point: thus, the interior critical point

(a2. bi) in Figure 1 would not give a relative extremum for the function. (Notice that

the graph of the function has the shape of a saddle near the point 5 in Figure 1.)

In order to sort out the interior critical points of a function and determine which
correspond to relative maxima, relative minima, or saddle points, the following
theorem (whose proof is best left to a course in advanced calculus) may be helpful.

THEOREM 2 Second-Derivative Test

Let [a. b) be an interior point of the domain of a function/, and suppose that the
first and second partial derivatives of/exist and are continuous on some circular
disk with (a. b) as its center and contained in the domain of/. Assume that {a. b)

is a critical point of/, so that/|(a, b) =f2(a, b) = 0. Let

fu(a. b) /,2(a. b)\


A = /,i(fl. b)f22(a. b) - [/,:(£/. b)]-
./i2(a. b) f22(a, b)\
Then

(i)* If A > and/i|(a. b) < 0. then /has a relative maximum at {a. h).

(ii)* If A > and/ii(a. h) > 0. then /has a relative minimum at (a. b).
(iii) If A < 0, then /has a saddle point at (a. h).

(iv) If A = 0. then the test is inconclusive, and other methods must be used.

In E.xamples 2 to 4. find all the interior critical points of the domain of the given
function, and then apply the second-derivative test to decide (if possible) at which of
these points relative maxima, relative minima, or saddle points occur.

EXAMPi-i: 2 /(.v. >') = x- + y-^ - Zr -I- 4v -I- 2

*If A =/,i(«. /')/::(«. h) - !/,:((;. /))]- > 0. then/,|(u. b)f22Ui. h) must be positive, so that
same algebraic sign. Therefore, in parts (i) and (ii) of Theo-
/ii(a. h) and /::(«. h) have the
rem 2, we can replace /ntu. h) by/22(a. b).
SECTION 14.8 EXTREMA FOR FUNCTIONS OF MORE THAN ONE VARIABLE 897

SOLUTION Here.

f\ix. V) = 2v - 2 /.(.r, V) = 2v + 4
/ll(A-, y) = 2 /l2(-V. y) =/2,(A-, V) = /22(A. v) = 2

To find all critical points, we must solve the simultaneous equations

J/,(A. y) = , .
rZv - 2 =
1 r that IS.
l/2(A. y) = {
by + 4 =
In this case, the only solution is x = 1, y = -2; hence. (I. -2) is the only critical
point of/. Calculating the quantity A of Theorem 2 at the critical point (1, -2), we
obtain

/u(I. -2) /,2(1. -2)


A = |2 0[
=4-0=4>0
/,2(1. -2) /22(l. -2) |o 2

Also, /,,(!. -2) - 2 > 0. Therefore, by Theorem 2. /has a relative minimum at


(1, -2).

EXAMPLE 3 /(.v. y) = llxT - 4.v-y - 3at-

SOLUTION Here.

/i(A-. y) = 12y - Sx}' - 3y- = (12 - 8a - 3y)y

/2(A. y) = IZr - 4a- - 6.rv = (12 - 4a - 6y)A

/„(.v, y)= -8y


/i2(-v, y) =/2i(A. y) = 12 - 8a - 6y

and /,2(A. y) = -6a


To find all critical points (a, y), we must find all solutions of the simultaneous
equations

(12 - 8a- 3y)y =


(12 -4a- 6y)A =
Obviously, a = 0. y = gives a solution, so that (0. 0) is a critical point. If a ^
and y = 0. the equations become

=
12 - 4a =
so that A = 3. y = gives a solution, and (3. 0) is a critical point. If a = and
y 5^ 0, the equations become

12 - 3y =
=
giving the critical point (0. 4). Finally, if a ^ and y t^ 0. the simultaneous equa-
tions become

12 - 8a - 3y =
12 - 4a - 6y =
Solving the latter equations as usual (say. by elimination of variables),

S-ar putea să vă placă și